You are on page 1of 434

@BEST300MCQ

CONTENT

1. NUMBER SYSTEM AND SIMPLIFICATION ............................................................................ 3


2. LCM AND HCF ............................................................................................................................. 27
3. SURDS AND INDICES ................................................................................................................. 46
4. RATIO, PROPORTION AND PARTNERSHIP ........................................................................ 71
5. MIXTURE AND ALLIGATION ............................................................................................... 102
6. PERCENTAGE ................................................................................................................................ 124
7. PROFIT AND LOSS ..................................................................................................................... 146
8. SIMPLE INTEREST AND COMPOUND INTEREST................................................................. 173
9. AVERAGE ................................................................................................................................... 200
10. TIME AND WORK ..................................................................................................................... 222
11. PIPE AND CISTERN .................................................................................................................. 259
12. SPEED, TIME AND DISTANCE .............................................................................................. 280
13. BOAT AND STREAM ............................................................................................................... 310
14. DATA INTERPRETATION ...................................................................................................... 326

15 Practice Sets (Based on Latest Pattern)


Practice Set – 01 ...................................................................................................................................... 370
Practice Set – 02 ....................................................................................................................................... 375
Practice Set – 03 ........................................................................................................................................ 380
Practice Set – 04 ........................................................................................................................................ 384
Practice Set – 05 ..................................................................................................................................... 389
Practice Set – 06 ....................................................................................................................................... 392
Practice Set – 07 ...................................................................................................................................... 396
Practice Set – 08 ...................................................................................................................................... 401
Practice Set – 09 ..................................................................................................................................... 406
Practice Set – 10 ..................................................................................................................................... 409
Practice Set – 11 ....................................................................................................................................... 413
Practice Set – 12 ....................................................................................................................................... 418
Practice Set – 13 ....................................................................................................................................... 422
Practice Set – 14 ....................................................................................................................................... 425
Practice Set – 15 ....................................................................................................................................... 429
Chapter
Number System and Simplifications
1
Real Number

Rational Number Irrational Number

Fraction Integers

Positive Negative

Whole Number Natural Number

Natural Numbers ® 1, 2, 3, ....... ¥ Whole Numbers ® 0, 1, 2, 3 ....... ¥


Integers ® - ¥ ....... –3, –2, –1, 0, 1, 2 ....... ¥ Rational Numbers ® Integers and Fractions.
Fraction: Any number that can be represented in the form of p/q, where p & q are integers & q is not equal to zero
is called a rational number.
Irrational Number ® Any real number that cannot be expressed as a ratio of integers, i.e as a fraction.
Example: 5, 8
Prime Number: A number which has exactly two factors 1 & itself is called a prime number.
Prime numbers from 1 – 100 are 2, 3, 5, 7, 11, 13, 17, 19, 23, 29, 31, 37, 41, 43, 47, 53, 59, 61, 67, 71, 73, 79, 83, 89, 97
i.e. there are total 25 prime numbers up to 100.
Some results on Prime Numbers:
(i) Up to 100 total prime numbers = 25
(ii) Up to 50 total prime numbers = 15
(iii) Sum of two prime numbers is always even except 2.
(iv) Sum of three prime numbers is even if and only if one number is 2.
(v) All prime numbers are odd except 2.
(vi) 2 is only even prime number.
(vii) Each prime number has two factors 1 & itself so 1 is not prime number.
(viii) Smallest prime number of three digit is 101
(ix) Largest prime number of three digit is 997
(x) If square of any prime number (except 2 and 3) is divided by 24 then remainder is always 1.
1
Example: ´ ( 112 , 13 2 , 17 2 , 19 2 , 232 ) = (remainder 1 in each case).
24
Composite No.: A number which has more than two factor is a composite number.
Example: 4, 6, 8, 9 .......
Note: 1 is neither prime nor composite number, 2 is only even prime number.
Co-Prime No.: The pair of numbers which have no common factor other than one are called co-prime numbers.
Example: (4, 5), (15, 8)

3 @BEST300MCQ For More Study Material


Visit: studyiq.com
Tests of Divisibility:
(i) Divisibility by 2 : A number is divisible by 2, if its unit place is any of 0, 2, 4, 6, 8
(ii) Divisibility by 3 : A number is divisible by 3 only when the sum of its digits is divisible by 3.
(iii) Divisibility by 4 : A number is divisible by 4 if the number formed by its last two digits is divisible by 4.
(iv) Divisibility by 5 : A number is divisible by 5 if its unit digit is 5 or 0.
(v) Divisibility by 6 : A number is divisible by 6 if it is divisible by both 2 & 3.
(vi) Divisibility by 8 : A number is divisible by 8 when the number formed by its last 3 digits is divisible by 8.
(vii) Divisibility by 9 : A number is divisible by 9 if the sum of its digits is divisible by 9.
(viii) Divisibility by 10 : A number is divisible by 10 only when its unit digit is zero.
(ix) Divisibility by 11 : A number is divisible by 11, if the difference of the sum of its digits at odd places & the
sum of its digits at even places is either 0 or a number divisible by 11.
Some results on division:
(i) (xn – an) is divisible by (x – a) for all value of n.
(ii) (xn – an) is divisible by (x + a) for even value of n.
(iii) (xn + an) is divisible by (x + a) for odd value of n.
Dividend = ( Divisor ´ Quotient ) + Remainder
Some Results on Numbers:
(i) The product of four numbers which are consecutive natural numbers is always divisible by 24.
101 ´ 102 ´ 103 ´ 104 7 ´ 8 ´ 9 ´ 10
Example: or
24 24
(ii) The difference of square of two consecutive natural numbers is always equal to sum of those numbers.
Example: 92 – 82 = 9 + 8, 1192 – 1182 = 119 + 118
(iii) The difference of square of two consecutive odd numbers is always divisible by 8.
Example: 112 – 92 = 121 – 81 = 40
40
= 5.
8
(iv) The difference of square of two consecutive even numbers is always divisibly by 4.
Example: 102 – 82 = 100 – 64 = 36
36
= 9.
4
(v) Any digit repeated 6 times is divisible by 7, 11, 13 & 37.
Example: 5 5 5 5 5 5 or 2 2 2 2 2 2
are divisible by 7, 11, 13 & 37.
(vi) Any two digit number repeated 2 times is always divisible by 101.
Example: 3 4 3 4 or 5 6 5 6 is divisible by 101.
(vii) If P is prime number & a is an integer then (aP – a) is always divisible by P.
Example: (511 – 5) is divisible by 11.
(viii) If n is an odd number then (22n + 1) is always divisible by 5.
(ix) If n is an even number, then (22n – 1) is always divisible by 5.
(x) The product of three consecutive natural numbers is always divisible by 6.
1 1
Example: ´ ( 8 ´ 9 ´ 10 ) or ´ ( 11 ´ 12 ´ 13 )
6 6
(xi) The product of three consecutive natural numbers starting with even number is always divisible by 24.

1 1
Example: ´ ( 8 ´ 9 ´ 10 ) or ( 18 ´ 19 ´ 20 )
24 24
¯ ¯
even even
(xii) Any number written in the form 9 (10n – 1) is always divisible by 3 & 9 both.
(xiii) Any natural number of the form (n3 – n) is always divisible by 6.

4 @BEST300MCQ For More Study Material


Visit: studyiq.com
Unit digit : 34 = 81 = 1 i.e. 1 is unit digit
3215 × 5163 × 7298
product of unit digits = 5 × 3 × 8 = 120, i.e. unit digit is zero.
The unit digit of the numbers in following forms is:

5n = 5 4 odd = 4 9 odd = 9
6n = 6 4even = 6 9 even = 1
0n =0
1n =1

Example :
(i) 234567 + 566133 (ii) 24933 + 25034 + 25135
Unit digit = 4 + 6 = 10 = 0 unit digit = 9 + 0 + 1 = 10 = 0
Remaining digit : (2, 3, 7, 8)
• 212 79 Þ 2 79 4 = 2 3 = 8 • 378 41925 Þ 825 4 = 81 = 8

• 4732188 Þ 388 4 = 34 = 81 = 1
• In case remainder is zero, then power would be 4
Example : 214 2164 Þ 4 64 4 = 4 4 = 256 = 6
Testing of prime numbers • Test wether 221 is prime or not
• Test whether 191 is prime or not
Clearly 15 is > 221
Clearly 14 > 191
Prime numbers up to 15 are 2, 3, 5, 7, 11, 13
Prime numbers up to 14 are 2, 3, 5, 7, 11, 13 Clearly, 13 divides 221 exactly
No one of these divides 191 exactly So, 221 is not prime.
\ 191 is a prime number.
(i) Sum of n natural numbers (ii) Sum of squares of n natural numbers

n (n + 1) n ( n + 1)( 2n + 1)
1 + 2 + 3 + .............. + n = 12 + 22 + 32 + ................ + n2 =
2 6
(iii) Sum of cube of n natural numbers

é n ( n + 1) ù
2

1 + 2 + 3 + ................. + n = ê
3 3 3 3
ú
ë 2 û
Odd number : Those numbers which are not divisible by 2, are known as odd numbers
Example: 1, 3, 5, 7, ..........
tn + 1
n= , where n = total number of term, tn = last term.
2
Sum of Ist n odd numbers = n2
Example : 1 + 3 + 5 + ...................... + 49
49 + 1
n= = 25 , sum = (25)2 (since, n = 25)
2
= 625
Example: Find the sum of the series
51 + 53 + ................. + 99

=
( Last term + I st
)
term ´ ( Last term - Pr evious term of I st term )
=
( 99 + 51 )( 99 - 49 ) 150 ´ 50
= = 1875
4 4 4
5 @BEST300MCQ For More Study Material
Visit: studyiq.com
Even Numbers: Those numbers which are divisible by 2 are known as even numbers.
Example : 2, 4, 6, 8, .............
tn
n= , Where n = total numbers of term, tn = last term
2
sum of Ist n even numbers = n(n + 1)
Example : 2 + 4 + 6 + ................... + 58
58
n= = 29 , sum = n(n + 1) = 29(29 + 1) = 870
2

Remainder Theorem:
1. When a1, a2, a3 ..... an are divided by 'd' individually the respective remainders are R1, R2, R3.... Rn and when (a1+ a2+
a3.......an) is divided by 'd' the remainder can be obtained by dividing (R1 + R2 + R3 ....Rn) by 'd'
Example : Find remainder when 38 + 71 + 85 is divided by 16

38 + 71 + 85 6+7+5
= =
16 16
(Remainder obtained when numbers are individually divided by 16)

18
= Þ Remainder = 2
16
2. When a1, a2, a3... an are divided by a divisor d the respective remainders obtained are R1, R2, R3 .....Rn, and the
remainder when (a1× a2×a3 ....× an) is divided by 'd' can be obtained by dividing (R1×R2×R3....Rn) by d.
Example : Find Remainder when 77 is divided by 4.

77 7 ´ 7 ´ 7 ´ 7 ´ 7 ´ 7 ´ 7 3 ´ 3 ´ 3 ´ 3 ´ 3 ´ 3 ´ 3
= = (Remainder obtained individually)
4 4 4

9 ´ 9 ´ 9 ´ 3 1´ 1´ 1 ´ 3
= = Þ Remainder = 3
4 4
So we can say that remainders can be added as well as multiplied.
Some results on remainder

( nx + 1 )
n
nx
l For , Remainder = 0 l For , Remainder = 1
n n

( nx – 1 ) ( nx – 1)
even odd

l For , Remainder = 1 l For , Remainder = –1 or (n–1)


n n
Where x and n are any positive integers.
Recurring Decimal : A decimal number in which a digit or a set of digits repeats regularly, over a constant period, is
called a recurring decimal.

Example : 2.3333..... , 7.5555.... , 1.3333.... they are represented as 2.3, 7.5, 1.3
(i) Pure Recurring decimal : A decimal fraction in which all the figures occur repeatedly is called a pure
recurring decimal e.g 7.4444.... , 2.1111.... , 3.4545...
(ii) Mixed Recurring decimal : A decimal number in which some of the digits do not recur is called a mixed
recurring decimal e.g. 0.1777, .087373...
(iii) Non recurring decimal : A decimal number in which there is no regular pattern of repitition of digits after
decimal point is called non-recurring decimal e.g. 3.24662676...

6 @BEST300MCQ For More Study Material


Visit: studyiq.com
p
Fraction : The word fraction means a part of anything. It can be expressed in the from of where p and q are integers
q
and 'q' is not equal to '0'.
Proper fraction : When the numerator is less than the denominator, then the fraction is called a proper fraction.

7 5 12
Example : , , etc.
12 17 43
Improper fraction : When the numerator is greater than the denominator, then the fraction is called an improper
fraction.

17 18 45
Example : , , etc.
13 14 19
Like fraction : Fractions having same denominator are called like fractions.

1 5 7
Example : , , etc.
9 9 9
Unlike fraction : Fractions having different denominators are called unlike fractions.

14 17 53
Example : , , etc.
23 43 19
Compound fraction : It is a fraction of a fraction.

1 5 7 61 9 7
Example : of , of , of
3 9 9 53 13 19
Complex fraction : In such a fraction, both the numerator and the denominator are fractions.

Example : 12 5 13
+
13 , 17 72
17 74 7
+
21 43 9

Mixed fraction : Those fractions which consist of a whole number and a proper fraction, are known as mixed
fractions.

7 4 13
Example : 5 , 7 , 12 etc.
8 9 17
Continued fraction : It contains an additional fraction in the numerator or in the denominator.

1
Example : 12 +
14
12 +
2
65 +
3

Decimal faction : In such a fraction, the denominator has power of 10.

45 7 71
Example: 0.45 = , 0.7 = , 0.000071 = etc.
100 10 1000000

7 @BEST300MCQ For More Study Material


Visit: studyiq.com
1. A number when divided by 91 gives a remainder 17.
517 517 1517
When the same no is divided by 13, the remainder (iii) Þ =1
will be 2 2

3165 ( 3 ) ´ 3
41
4 1
17 2243165
Sol. = 4 remainder (iv) Þ = =3
13 5 5 5
2. (461 + 462 + 463) is divisible by:
2 129 ( 2 ) ´ 2
32
4
Sol. 461(1 + 4 + 42) = 461 × 21 7 129
(v) Þ = =2
i.e. 21 is divisible by 3 5 5 5
3. Find the number of zeros in the product of 1 × 2 × 3 ×
( -1 )
123
................ × 99 × 100. 8 123
(vi) Þ = 9-1 = 8
100 20 9 9
Sol. = 20 and =4
5 5
( 2 3 ) ´ 2 = ( -1)25 ´ 2 = -2 = 7
25
2 76
i.e. total numbers of zeros = 20 + 4 = 24 (vii) Þ
4. Find the total number of zeros in the product of 1 × 2 9 9 9 9
× 3 × ................ × 250.
( -1 ) + ( -1 )
20 40
19 20 + 19 40 2
(viii) Þ = =2
250 50 10 20 20 20
Sol. = 50 , = 10 and =2
5 5 5
(4 )
2 37
´ 4 + ( 42 )
38
i.e. total numbers of zeros = 50 + 10 + 2 = 62 475 + 476
(ix) Þ
5. Find the total number of zeros in the product of 51 × 17 17
52 × 53 × ................ × 100.
( -1 ) ´ 4 + ( -1 )
37 38
-1 ´ 4 + 1 -3
100 20 = = = = 14
Sol. = 20 , =4 17 17 17
5 5
2 517 ( 2 )
129
50 10 517 517
4
´ 21 1129 ´ 2
and, = 10 , =2 (x) Þ = = =2
5 5 5 5 5 5
So, total number of zeros = (20 + 4) – (10 + 2) = 12
7. Find the unit digit in the following questions.
6. Find the remainder in the following questions
(i) (124)372 + (124)373
537 275 (ii) (4387)245 + (621)72
(i) (ii)
8 5 (iii) 256521 + 36528 + 7354
517 517 2243165 (iv) 771 × 663 × 365
(iii) (iv)
2 5 (v) (251)98 + (21)29 – (106)100 + (705)35 – 164 + 259
7 129 8123 Sol. (i) (124)372 + (124)373 = 6 + 4
(v) (vi)
5 9 Þ unit digit = 0
276 1920 + 1940 (ii) (4387)245 + (621)72 = (7)1 + (1)72 = 7 + 1
(vii) (viii) = 8 (unit digit).
9 20
(iii) 256521 + 36528 + 7354 = 5 + 6 + (3)2 = 5 + 6 + 9 = 20
475 + 4 76 517 517
(ix) (x) \ unit digit = 0
17 5
(iv) 771 × 663 × 365
( 52 ) ´ 51 = 2518 ´ 5 = 118 ´ 5 = 5
18
5 37 = 73 × 63 × 31 = 3 × 6 × 3
Sol. (i) Þ = 4 (unit digit)
8 8 8 8
(v) (251)98 + (21)29 – (106)100 + (705)35 – 164 + 259
( 2 ) ´ 2 = 1618 ´ 8 = ( 1) ´ 8 = 3
18
2 75
4 3 18
= 1 + 1 – 6 + 5 – 6 + 9 = 16 – 12
(ii) Þ = 4 (unit digit)
5 5 5 5

8 @BEST300MCQ For More Study Material


Visit: studyiq.com
Ques tions (a) 10 (b) 12
(c) 14 (d) 16
12. In a question of division, the divisor is 7 times the
1. The sum of all those prime numbers which are less quotient and 3 times the remainder. If remainder is
than 31 is 28, then the dividend is
(a) 119 (b) 129 (a) 588 (b) 784
(c) 132 (d) 137 (c) 823 (d) 1036
2. The sum of all even numbers between 21 and 51 is 13. If 17200 is divided by 18, the remainder is
(a) 518 (b) 540 (a) 17 (b) 16
(c) 1 (d) 2
(c) 560 (d) 596
14. Which of the following numbers is not divisible by 18
3. Which of the following is one of the factors of the sum (a) 54036 (b) 50436
of first 25 natural numbers (c) 34056 (d) 65043
(a) 26 (b) 24 15. It is given that (232 + 1) is exactly divisible by a certain
(c) 13 (d) 12 number. Which one of the following is also definitely
4. The digit in the unit place of the product divisible by the same number.
(2464)1793 × (615)317 × (131)491 is (a) 296 + 1 (b) 7 × 233
(c) 216 – 1 (d) 216 + 1
(a) 0 (b) 2
(c) 3 (d) 5 4 9
16. The least number among , , 0.45 and (0.8)2 is
5. The digit in the unit place of 9 49
[(251)98 + (21)29 – (106)100 + (705)35] is
4 9
(a) 1 (b) 4 (a) (b)
9 49
(c) 5 (d) 6
(c) 0.45 (d) (0.8)2
6. Find the remainder value in the following expression
p
( 23 2
+ 29 2 + 312 + 37 2 ) 17. The number 0.121212 ... in the form
q
is equal to

24
4 2
(a) 13 (b) 17 (a) (b)
11 11
(c) 4 (d) 3
7. Find the value of given series 4 2
(c) (d)
1 – 2 + 3 – 4 + 5 – 6 + ..........................+ 95 – 96 + 97 – 98 33 33
(a) 49 (b) 53 15 19 24 34
18. The least among the fraction , , , is
(c) –49 (d) –53 16 20 25 35
8. Find the total number of zeros in the following series
34 15
2 × 4 × 6 × ............................................ × 248 × 250 (a) (b)
35 16
(a) 31 (b) 37
(c) 39 (d) 43 19 24
(c) (d)
9. 101 × 102 × 103 × 104 is a number which is always 20 25
divisible by the greatest number in the given option. 19. If 1 + 2 + ... + 9 = 2025, then the value of
3 3 3

(a) 6 (b) 24 (0.11)3 + (0.22)3 + ... + (0.99)3 is close to


(c) 48 (d) 16 (a) 0.2695 (b) 2.695
10. Find the number of total prime numbers up to 100 (c) 3.695 (d) 0.3695
20. Which of the following number is the greatest among
(a) 27 (b) 23
all?
(c) 25 (d) 26
0.9, 0.9, 0.09, 0.09
11. When two numbers are separately divided by 33, the
remainders are 21 and 28 respectively. If the sum of (a) 0.9 (b) 0.9
the two numbers is divided by 33, the remainder will
be (c) 0.09 (d) 0.09

9 @BEST300MCQ For More Study Material


Visit: studyiq.com
21. How many natural numbers divisible by 7 are there 32. 1008 should be divided by which single digit number
between 3 and 200? to get a perfect square?
(a) 27 (b) 28 (a) 9 (b) 4
(c) 29 (d) 36 (c) 8 (d) 7
22. The sum of three consecutive odd natural numbers is 33. (12 + 22 + 32 +....+102) is equal to
87. The smallest of these numbers is (a) 380 (b) 385
(a) 29 (b) 31 (c) 390 (d) 392
(c) 23 (d) 27
n
23. What will be the unit digit in 7105?
2
34. Given that 12 + 22 + 32 + ......+ n = ( n + 1 )( 2n + 1 ) ,
6
(a) 5 (b) 7
then, 102 + 112 + 122 + ...... + 202 is equal to
(c) 9 (d) 1
(a) 2616 (b) 2585
24. Which one of the following will completely divide
(c) 3747 (d) 2555
571 + 572 + 573 ?
35. The largest natural number, which exactly divides
(a) 150 (b) 160
the product of any four consecutive natural numbers,
(c) 155 (d) 30 is
25. When 233 is divided by 10, the remainder will be (a) 6 (b) 12
(a) 2 (b) 3 (c) 24 (d) 120
(c) 4 (d) 8 36. If 13 +23 + 33 + 43 +53 + 63 = 441. Then find the value of
26. When a number is divided by 24, the remainder is 16. 23 +43 +63 +83 + 103+ 123
The remainder when the same number is divided by (a) 882 (b) 1323
12 is
(c) 1764 (d) 3528
(a) 3 (b) 4
(c) 6 (d) 8 2 5 11 7
37. The greatest fraction among , , and is
27. The remainder when 3 is divided by 5 is
21 3 6 15 8
(a) 1 (b) 2 7 11
(c) 3 (d) 4 (a) (b)
8 15
28. A 4-digit number is formed by repeating a 2-digit
number such as 1515, 3737, etc. Any number of this 5 2
(c) (d)
form is exactly divisible by 6 3
(a) 7 (b) 11 38. 0.423 is equivalent to the fraction
(c) 13 (d) 101
491 419
29. How many numbers less than 1000 are multiples of (a) (b)
both 10 and 13? 990 990
(a) 9 (b) 8 49 94
(c) (d)
(c) 6 (d) 7 99 99
30. What number should be divided by 0.25 to give the 39. 0.393939 ..... is equal to
result as 25? 39 13
(a) (b)
(a) 25 (b) 50 100 33
(c) 12.5 (d) 125 93 39
(c) (d)
31. The smallest number that must be added to 803642 100 990
in order to obtain a multiple of 11 is 40. If one-third of one-fourth of a number is 15, then three-
(a) 1 (b) 4 tenth of the number is
(c) 7 (d) 9 (a) 35 (b) 36
(c) 45 (d) 54

10 @BEST300MCQ For More Study Material


Visit: studyiq.com
1. Which is the largest of the following fractions? (a) 3.3 (b) 2.2
2 3 8 7 11 (c) 1.1 (d) 4.4
, , , ,
3 5 11 9 17 1
8. If x + = 12, then find the value of x.
2 1
11 2+
(a) (b) 1
3 17 3+
1
4+
7 3 5
(c) (d)
9 5
1816 2012
2. Which one of the group is in descending order? (a) (b)
157 153
7 9 13 13 9 7
(a) , , (b) , , 1818 1818
12 17 24 24 17 12 (c) (d)
151 157
9 13 7 7 13 9
(c) , , (d) , , 9. Find the value of 5.12 + 3.21 + 4.31 = ?
17 24 12 12 24 17
1 1 1 1 1 64 74
3. 1 + 11 + 111 + 1111 + 11111 = ? (a) 12 (b) 12
2 2 2 2 2 99 99

1 1 77 84
(a) 12347 (b) 12346 (c) 12 (d) 12
2 2 99 99

1 1 10. The difference of 5.76 and 2.3 is.


(c) 12345 (d) 12344
2 2 (a) 3.54 (b) 3.73
1 1 1 1 1 (c) 3.46 (d) 3.43
4. 3 + 33 + 333 + 3333 + 33333 = ?
3 3 3 3 3 2x - 7
11. If x is a prime number and -1 £ £ 1, then the
2 1 5
(a) 37031 (b) 37037 number of values of x is:
3 3
(a) 4 (b) 3
2 1 (c) 2 (d) 5
(c) 37036 (d) 37032
3 3 12. The sum of a natural number and its square equals
1 1 1 1 1 1 1 the product of the first three prime numbers. The
5. + + + + + + =? number is:
6 12 20 30 42 56 72
(a) 2 (b) 3
5 7 (c) 5 (d) 6
(a) (b)
18 18 1 3
13. The rational number between and is:
11 13 2 5
(c) (d)
18 18 2 4
(a) (b)
1 1 1 1 5 7
6. + + + ....... =?
5 ´ 9 9 ´ 13 13 ´ 17 61 ´ 65 2 1
(c) (d)
4 3 3 3
(a) (b) 14. What is the sum of two consecutive even numbers,
45 65
the difference of whose square is 84?
2 3 (a) 38 (b) 34
(c) (d)
35 35 (c) 42 (d) 46
15. The sum of all the natural numbers from 50 to 100 is:
3 (a) 5050 (b) 4275
7. If = x , then find the value of x.
2 (c) 4025 (d) 4005
2+
2 16. The last digit of (1001)2008 + 1002 is:
2+
2 (a) 0 (b) 3
2+
3 (c) 4 (d) 6

11 @BEST300MCQ For More Study Material


Visit: studyiq.com
17. The unit digit in the product 771 × 663 × 365 is: (a) 3 (b) 1
(a) 1 (b) 2 (c) 5 (d) 0
(c) 3 (d) 4 2 5 11 7
18. Unit's digit of the number (22)23 is: 25. The greatest fraction among , , and is:
3 6 15 8
(a) 4 (b) 6
(c) 8 (d) 2 7 11
(a) (b)
19. The digit in unit's place of the product (2153)167 is: 8 15
(a) 1 (b) 3 5 2
(c) 7 (d) 9 (c) (d)
6 3
20. If the sum of the digits of any integer lying between 26. If (6767 + 67) is divided by 68. Then, the remainder is
100 and 1000 is subtracted from the number, the result (a) 1 (b) 67
always is:
(c) 63 (d) 66
(a) divisible by 2 (b) divisible by 9 27. [22 + 32 + 42 + 52 + 62 + 72 + 82 + 92 + 102] is equal to
(c) divisible by 5 (d) divisible by 6 (a) 385 (b) 2916
21. In a division, the divisor is 10 times the quotient and (c) 540 (d) 384
5 times the remainder. If the remainder is 46, then the
28. If 13 + 23 + ... + 103 = 3025. Then, 4 + 32 + 108 + ... +
dividend is:
4000 is equal to
(a) 4236 (b) 4306
(a) 12000 (b) 12100
(c) 4336 (d) 5336
(c) 12200 (d) 12400
22. If a and b are two odd positive integers, by which of
29. Which of the following fractions is the smallest?
the following integers is (a4 – b4) always divisible.
(a) 3 (b) 6 7 7
(a) (b)
(c) 8 (d) 12 6 9
23. A number, when divided by 136, leaves remainder 4 5
36. If the same number is divided by 17, the remainder (c) (d)
5 7
will be:
(a) 9 (b) 7 30. 0.001 is equal to
(c) 3 (d) 2 1 1
24. A number, when divided by 899, leaves remainder (a) (b)
1000 999
63. What will be the remainder if the same number is
divided by 29 ? 1 1
(c) (d)
99 9

1. The sum of the squares of three consecutive natural 5. The last 5 digits of the following expression will be
numbers is 2030. Then, what is the middle number? (1 !)5 + (2 !)4 + (3 !)3 + (4 !)2 + (5 !)1 + (10 !)5
(a) 25 (b) 26 + (100 !)4 + (1000 !)3 + (10000 !)2 + (100000 !)
(c) 27 (d) 28 (a) 45939 (b) 00929
2. In a division, the divisor is 10 times the quotient and (c) 20929 (d) can't be determined
5 times the remainder. If the remainder is 40, then the 4
dividend is 6. What fraction of must be added to itself to make
7
(a) 240 (b) 440
1
(c) 4040 (d) 4000 the sum 1 ?
14
3. If m and n are positive integers and (m – n) is an even
number, then (m2 – n2) will be always divisible by 7 1
(a) (b)
(a) 4 (b) 6 8 2
(c) 8 (d) 12 4 15
4. Both the ends of a 99 digits number N are 2. N is (c) (d)
7 14
divisible by 11, then all the middle digits are 7. Find the sum of the first five terms of the following
(a) 1 (b) 2
1 1 1 1
(c) 3 (d) 4 series + + + ... +
1 ´ 4 4 ´ 7 7 ´ 10 13 ´ 16

12 @BEST300MCQ For More Study Material


Visit: studyiq.com
(a) 0.16 (b) 1.6
9 7
(a) (b) (c) 16 (d) 0.016
32 16
16. A divisor is 25 times the quotient and 5 times the
5 1 remainder. The quotient is 16, the dividend is
(c) (d)
16 210 (a) 6400 (b) 6480
8. The sum (53 + 63 + ... + 103) is equal to (c) 400 (d) 480
(a) 2295 (b) 2425 1 1
(c) 2495 (d) 2925 17. Given that 3.718 = . Then, is equal
0.2689 0.0003718
9. If (10 + 25) – (10 – 25) = 10n, then the value of n is
12 2 12 2
to
(a) 20 (b) 14 (a) 2689 (b) 2.689
(c) 10 (d) 5 (c) 26890 (d) 0.2689
10. The value of 18. Largest four digit number which when divided by 15
3 5 7 9 11 13 leaves a remainder of 12 and if the same number is
+ 2 2+ 2 2+ 2 2+ 2 2+ 2 2 divided by 8 it leaves the remainder 5. Such greatest
1 ×222
2 ×3 3 ×4 4 ×5 5 ×6 6 ×7
possible number is:
15 17 19 (a) 9963 (b) 9957
+ + + is
7 2 × 8 2 8 2 × 9 2 9 2 × 10 2 (c) 9945 (d) 9999
19. Number of zeros at the end of the following
1 99 expression (5 !)5! + (10 !)10! + (50 !)50! + (100 !)100! is:
(a) (b)
100 100 (a) 165 (b) 120
(c) 125 (d) None of these
101
(c) (d) 1 20. A fraction in its lowest form is such that when it is
100
1
1 1 1 1 squared and then its numerator is reduced by rd
11. + + + ... + 3
1+ 2 2+ 3 3+ 4 99 + 100
1
is equal to and denominator is reduced to th , it results as twice
5
(a) 1 (b) 5
of the original fraction. Then the sum of numerator
(c) 9 (d) 10 and denominator can be:
12. When simplified, the sum (a) 7 (b) 8
1 1 1 1 1 1 (c) 9 (d) 17
+ + + + + ... +
2 6 12 20 30 n ( n + 1 ) is equal to 21. The value of the expression
7777 + 7777 ´ 7777 ´ ( 5 ¸ 77 ) ´ ( 11 ¸ 35 ) :
1 1
(a) (b)
n n+1 (a) 1234321 (b) 12344321
(c) 77777 (d) None of these
2 ( n - 1) n
(c) (d) 32
n n+1 22. Find the last digit of 32 32 .
x ( x + 1 )( 2x + 1 ) (a) 6 (b) 8
13. If 12 + 22 + 32 + ... + x2 = , then (c) 10 (d) 4
6
23. Find the last digit of 222888 + 888222.
12 + 32 + 52 + ... + 192 is equal to
(a) 8 (b) 4
(a) 1330 (b) 2100
(c) 0 (d) 6
(c) 2485 (d) 2500
24. Find the unit digit of 111! (fractorial 111).
14. (1 – 2 + 3 – 4 + 5 – 6 + ... + 92 – 102) is equal to
2 2 2 2 2 2
(a) 0 (b) 2
(a) –55 (b) 55
(c) 3 (d) 4
(c) –56 (d) 56
25. Which is not the factor of 46n – 64n for any positive
15. The sum of the first 20 terms of the series integer n?
1 1 1 (a) 5 (b) 25
+ + + ... is
5´6 6´7 7 ´8 (c) 7 (d) None of these

13 @BEST300MCQ For More Study Material


Visit: studyiq.com
26. 19n – 1 is: (a) 175 (b) 75
(a) always divisible by 9 (c) 680 (d) does not exist
(b) always divisible by 20 29. Total number of factors of a number is 24 and the sum
(c) is never divisible by 19 of its 3 prime factors out of four, is 25. The product of
(d) only (a) and (c) are true all 4 prime factors of this number is 1365. Then such
a greatest possible number can be :
27. Find the remainder when 101 + 102 + 103 + 104 + 105 +
... + 1099 is divided by 6. (a) 17745 (b) 28561
(a) 0 (b) 4 (c) 4095 (d) can't be determined
(c) 2 (d) 6 30. How many numbers are there in the set S = {200, 201,
202, ..., 800} which are divisible by neither 5 nor 7?
28. A number when divided by 5 gives a number which
is 8 more than the remainder obtained on dividing (a) 411 (b) 412
the same number by 34. Such a least possible number (c) 410 (d) None of these
is:

1. I multiplied a natural number by 18 and another by 10. 216 – 1 is divisible by


21 and added the products. Which one of the (a) 11 (b) 13
following could be the sum? (c) 17 (d) 19
(a) 2007 (b) 2008 11. A certain number when divided by 175 leaves a
(c) 2006 (d) 2002 remainder 132. When the same number is divided by
2. Out of six consecutive natural numbers, if the sum of 25, the remainder is
first three is 27, what is the sum of the other three? (a) 6 (b) 7
(a) 36 (b) 35 (c) 8 (d) 9
(c) 25 (d) 24 12. (461 + 462 + 463) is divisible by
3. Which one of the following is a factor of the sum of (a) 3 (b) 11
first 25 natural numbers? (c) 13 (d) 17
(a) 26 (b) 24 13. The digit in the unit place of the product
(c) 13 (d) 12 (2464)1793 × (615)317 × (131)491 is
4. The sum of all the natural numbers from 51 to 100 is (a) 0 (b) 2
(a) 5050 (b) 4275 (c) 3 (d) 5
(c) 4025 (d) 3775 14. (2 + 2 + 2 + 2 ) is divisible by
71 72 73 74

5. The unit digit in the sum of (124)376 + (124)375 is (a) 9 (b) 10


(a) 5 (b) 4 (c) 11 (d) 13
(c) 2 (d) 0 15. In a division problem, the divisor is 4 times the quotient
and 3 times the remainder. If remainder is 4, the
6. The unit digit of the expression 256527 + 36526 + 7354 is
dividend is
(a) 6 (b) 5
(a) 36 (b) 40
(c) 4 (d) 0
(c) 12 (d) 30
7. The digit in the unit place of [(251)98 + (21)29 – (106)100 16. If a number is divisible by both 11 and 13, then it must
+ (705)35 – 164 + 259] is be necessarily
(a) 1 (b) 4 (a) divisible by (11 + 13)
(c) 5 (d) 6 (b) divisible by (13 – 11)
8. If n is even, (6 – 1) is divisible by
n
(c) divisible by (11 × 13)
(a) 37 (b) 35 (d) 429
(c) 30 (d) 6 17. A common factor of (137 + 117) and (135 + 115) is
9. 'a' divides 228 leaving a remainder 18. The biggest (a) 24 (b) 135 + 115
two digit value of 'a' is (c) 13 + 11
2 2
(d) None of these
(a) 21 (b) 35 18. Sum of three consecutive even integers is 54. Find the
(c) 30 (d) 70 least among them.

14 @BEST300MCQ For More Study Material


Visit: studyiq.com
(a) 18 (b) 15 24. The decimal fraction of 2.349 is equal to
(c) 14 (d) 16
19. The unit digit in the product (122)173 is 2326 2326
(a) (b)
(a) 2 (b) 4 999 990
(c) 6 (d) 8 2347 2347
20. What least number of 5 digits is divisible by 41? (c) (d)
999 990
(a) 10045 (b) 10004
25. (52 + 62 + 72 + ... + 102) is equal to
(c) 10041 (d) 41000
(a) 330 (b) 345
21. A number divided by 13 leaves a remainder 1 and if
(c) 355 (d) 360
the quotient, is divided by 5, we get a remainder of 3.
What will be the remainder if the number is divided 26. Two numbers are in the ratio 1 : 2 when 4 is added to
by 65? each, the ratio becomes 2 : 3. Then, the numbers are
(a) 28 (b) 16 (a) 9 and 12 (b) 6 and 8
(c) 18 (d) 40 (c) 4 and 8 (d) 6 and 9
22. If p,q,r are in Geometric Progression, then which is 27. [13 + 23 + 33 + ... + 93 + 103] is equal to
true among the following? (a) 3575 (b) 2525
(c) 5075 (d) 3025
p+r
(a) q = (b) p2 = qr 28. A number, when divided by 899, leaves remainder
2
63. What will be the remainder if the same number is
p r divided by 29?
(c) q = pr (d) = (a) 3 (b) 1
r q
(c) 5 (d) 0
10 n - 1 29. When 25 is divided by 26, the remainder is
25
23. If 1 + 10 + 102 + ... upto n terms = , then the
9 (a) 1 (b) 2
sum of the series 4 + 44 + 444 + ... upto n terms is (c) 24 (d) 25
4 4n 4 4n 30. A number when divided by the sum of 555 and 445
(a)
9
( 10 n - 1 ) -
9
(b)
81
( 10 n - 1 ) -
9
gives two times their difference as quotient and 30 as
the remainder. The number is
40
(c)
81
( 10 n - 1) - 4n
9
(d)
40
9
( 10n - 1) - 4n
9
(a) 220030
(c) 1220
(b) 22030
(d) 1250

S olut i ons 25 ´ ( 25 + 1 )
= = 25 ´ 13
2
1. (b); The prime numbers Less than 31 are 2, 3, 5, 7, 11, i.e. 13 is one of the factor
13, 17, 19, 23, 29
(4)
1793 4
\ required sum = 2 + 3 + 5 + 7 + 11 + 13 + 17 + 19 4. (a); ´ 5´1
+ 23 + 29 = 129 4 × 5 × 1 = 20 So, unit digit is 0.
2. (b); Total even numbers from 1 to 50 = 25 5. (a); 1 + 1 – 6 + 5 = 1
Total even numbers from 1 to 20 = 10 6. (c); If square of any prime number is divided by 24
Sum of even numbers = n(n + 1) then remainder is always 1.
Required sum = sum of even numbers from 1 to
50 – sum of even numbers from 1 to 20 ( 1 + 1 + 1 + 1) 4
so, = i.e 4 is unit digit.
= 25(25 + 1) – 10(10 + 1) 24 24
= 25 × 26 – 10 × 11 = 540 7. (c); (1 + 3 + 5 + ........... + 97) – (2 + 4 + 6 + ............ + 98)

n ( n + 1) 97 + 1 98
n1 = = 49 , n 2 = = 49
3. (c); sum of first n natural numbers = 2 2
2
\ sum of 1st 25 natural numbers sum = n12 – n2 (n2 + 1)= 492 – 49 × 50 = – 49

15 @BEST300MCQ For More Study Material


Visit: studyiq.com
250 125 25 5 æ 11 ö 3
3

÷ ( 1 + 2 + ... + 9 )
8. (a); = 125 , = 25 , =5, =1 =ç 3 3
2 5 5 5 è 100 ø
i.e. required numbers of zero = 25 + 5 + 1 = 31
9. (b); 24 1331
= ´ 2025
10. (c); 25 1000000
[Q 13 + 23 + ... + 93 = 2025]
( 21 + 28 )
11. (d); Required remainder = = 16 2695275
33 = = 2.695275 » 2.695
1000000
12. (d); Let quotient = x
divisor = 7x also divisor = 3 × (remainder) 20. (b); Decimal equivalent of fractions
= 3 × 28 = 84 9 9 9 1
7x = 84, x = 12 0.9 = , 0.9 = = 1, 0.09 = =
10 9 90 10
Dividend = Divisor × Quotient + Remainder
= 84 × 12 + 28 = 1036 9 1
and 0.09 = =
99 11
an
13. (c); Since it is form of \ 0.9 is greatest.
a+1
21. (b); Natural numbers between 3 and 200
17 200 = 200 – 3 = 197
i.e.
17 + 1 Now divide 197 by 7
\ Remainder = 1, Since n is even positive integer 28
14. (d); A number is exactly divisible by 18 if it is divisible 7 197
by 2 and 9 both. 14
since, 65043 is not divisible by 2, so it is not 57
divisible by 18. 56
15. (a); by checking option 1
So 28 natural numbers are there
296 + 1 = (232)3 + 13 = (232 + 1) (264 – 232 + 1) 22. (d); Let the consecutive odd no. are x, x + 2, x + 4
16. (b); Decimal equivalent of fractions x + x + 2 + x + 4 = 87
3x + 6 = 87
4 9 3
= 0.44; = = 0.43 81
9 49 7 x= = 27
3
(0.8)2 = 0.64
so, smallest number is 27.
9 23. (b); 7105
\ Least number = 0.43 =
49 Cyclicity of 7 is 4.
17. (c); Expression = 0.121212 ...
105
So = Remainder is 1.
12 4 4
= 0.12 = =
99 33 71 = Unit digit
[Since, 12 is repeating after decimal] 24. (c); 571 + 572 + 573
18. (b); Decimal equivalent of fractions 571(1 + 5+ 52)
15 19 24 34 571 × 31
= 0.94, = 0.95, = 0.96, = 0.97 570 × 155
16 20 25 35
so 155 divides the expression completly
15 25. (a); We know that 21 = 2, 22 = 4, 23 = 8, 24 = 16
\ Least fraction =
16
33
19. (b); Given, 13 + 23 + ... 93 = 2025 Remainder = = 1.
4
Then, (0.11)3 + (0.22)3 + ... + (0.99)3
Unit's digit in 233 = unit digit in 21
3 3 3
æ 11 ö æ 22 ö æ 99 ö Hence units digit = 2
=ç ÷ +ç ÷ + ... + ç ÷
è 100 ø è 100 ø è 100 ø Remainder on division by 10 = 2.

16 @BEST300MCQ For More Study Material


Visit: studyiq.com
26. (b); Remainder = 16 35. (c); Let four consecutive natural numbers are
Divisor = 24 1, 2, 3, 4
Let number = x 1 × 2 × 3 × 4 = 24
x = 24y + 16 where y is quotient. So 24 is a natural number which divides four
Since 24 is a multiple of 12 consecutive natural number completely
16 36. (d); Given, 13 + 23 + 33 + 43 + 53 + 63 = 441
Remainder = =4 23 + 43 + 63 + 83 + 103 + 123
12
= 23(13 + 23 + 33 + 43 + 53 + 63)
321 = 23 × 441 = 3528
27. (c);
5
2 5 11 7
37. (a); , , and
( 34 )5 ´ 3 ( 81) 5 ´ 3 3 6 15 8
=
5 5 Using cross multiplication method.

15 ´ 3 2 5
= ´ = 12 < 15
5 3 6
so, remainder = 3
5 2
28. (d); Let the two digit number be xy So, >
6 3
xy xy = xy × 100 + xy
= xy (100 + 1) = 101 xy 5 11
´ = 75 > 66
29. (d); Numbers which are multiple of both 10, 13 will 6 15
be multiple of 130 also
Numbers less then 1000 which are multiple of 5 11
So , is greater than
both 10 and 13 6 15

1000 5 7
= =7 ´ = 40 < 42
130 6 8

x 7
30. (c); = 25 So is the greatest fraction.
0.25 8
x = 25 × (0.5) = 12.5
423 - 4 419
31. (c); Required number = (Sum of digits at odd places) 38. (b); 0.423 = =
990 990
– Sum of digits at even place)
= (2 + 6 + 0) – (8 + 3 + 4) = – 7 39. (b); 0.393939 ..............
smallest number to be added = 7 39 13
= 0.39 = =
32. (d); Factor of 1008 99 33
= 2 × 2 × 2× 2 × 3 × 3 × 7
40. (d); Let number = y.
so number is divided by 7 to make it perfect
According to question
square.
33. (b); 12 + 23 + 32 ...... + 102 1 1
´ y = 15 , y = 180
3 4
n ( n + 1 ) ( 2n + 1 ) 10 ´ 11 ´ 21
= = = 385
6 6 3 3
so, y= ´ 180 = 54
34. (b); Sum of squares from 1 to 20 – Sum of squares 10 10
from 1 to 9
20 ´ 21 ´ 41 9 ´ 10 ´ 19
= - = 2870 – 285 = 2585
6 6

17 @BEST300MCQ For More Study Material


Visit: studyiq.com
1 1 1 1
1. (c); 2 3 2 8 8 7 7 11 6. (b); + + + ......... =?
5.9 9.13 13.17 61.65
3 5 3 11 11 9 9 17
10 > 9 22 < 24 72 < 77 119 > 99
Using formula:
Taking Taking Taking 7
+1 é 1 1 ù
greater of greater of greater of 9 is the ê – ú
these two these two these two largest Difference of ë First value Last value û
fractions fractions fractions denominator value
and the and the and the
next one next one next one 1 é 1 1 ù 1 é 13 - 1 ù 1 é 12 ù 3
= - = = =
4 êë 5 65 úû 4 êë 65 úû 4 êë 65 úû 65
2. (d); 7 13 13 9
12 24 24 17 7. (c); 3
x=
168 > 156 221 > 216 2
2+
2
7 13 13 9 2+
> and > 2ü 8
12 24 24 17 2+ ý =
3þ 3
7 13 9
Hence descending order = > >
12 24 17 3
=
2
1 1 1 1 1 2+
3. (a); 1 + 11 + 111 + 1111 + 11111 2ü
2 3 3 11
2 2 2 2 2 ï
2+ 8ý =2+ ´ =2+ =
1 8 4 4
= [1 + 11 + 111 + 1111 + 11111] + 3 ïþ
é1 1 1 1 1ù
ê2 + 2 + 2 + 2 + 2ú 3 3
ë û = =
2 2 4
2+ 2+ ´
1 1 11 1 11
= 12345 + 2 = 12347
2 2 4

1 1 1 1 1
4. (c); 3 + 33 + 333 + 3333 + 33333 3
3 3 3 3 3 =
2 4ü 8 30
2+ ´ ý = 2+ =
= [3 + 33 + 333 + 3333 + 33333] + 1 11 þ 11 11
é1 1 1 1 1ù
ê3 + 3 + 3 + 3 + 3ú =
3 3 11 11
= ´ = = 1.1
ë û
30 1 30 10
2 2 11
= 37035 + 1 = 37036
3 3
8. (a); 1
x+ = 12
1 1 1 1 1 1 1 1
5. (b); + + + + + + 2+
6 12 20 30 42 56 72 1
3+
1
1 1 1 4+
= + + ..... + 5
2 ´ 3 3´ 4 8´9

1 1 1 1 1 1 1 1
= - + - + ..... + - 12 = x + =x+
2 3 3 4 8 9 1 1
2+ 2+
1 5
3+ 3+
1 é 1 1ù 7 21 21
= ê - ú=
1 ë 2 9 û 18 5

18 @BEST300MCQ For More Study Material


Visit: studyiq.com
Þ x2 + 4x + 4 – x2 = 84
1 1 68
12 = x + = x+ = x+ Þ 4x = 84 – 4 = 80
1 157 157
2+
68 68 80
Þ x= = 20
21 4
Two numbers are 20 and 22.
68 \ The required sum = 20 + 22 = 42
x = 12 -
157 15. (d); Required sum = (sum of natural numbers from 1
157x = 1884 – 68 = 1816 to 100) – (sum of natural numbers from 1 to 49.)
Sum of [1 + 2 + 3 + 4 + ..... + 100]
1816
x=
157 n ( n + 1) 100 ( 101 )
= = = 5050
2 2
12 21 31
9. (a); 5.12 + 3.21 + 4.31 = 5 +3 +4 and sum of [1 + 2 + 3 + 4 ..... + 49]
99 99 99
n(n + 1) 50(49)
64 64 = = = 1045
= (5 + 3 + 4) + = 12 2 2
99 99
Hence, sum of [50 + 51 + 52 + 53 + ..... + 100]
76 3 43 = 5050 – 1045 = 4005
10. (d); 5.76 - 2.3 = 5 -2 = 3 = 3.43
99 9 99 16. (b); Given, (1001)2008 + 1002
Unit digit of (1001)2008 = 1
2x - 7
11. (b); Given, -1 £ £1 Last digit of 1002 = 2
5
\ The last digit = 1 + 2 = 3
Þ – 5 £ 2x – 7 £ 5 ... (i) 17. (d); Given, 771 × 663 × 365
Þ – 5 + 7 £ 2x – 7 + 7 £ 5 + 7 Then, 71 = 7, 72 = 49, 73
[by adding 7 in eq. (i)] = 343, 74 = 2401
Þ 2 £ 2x £ 12 \ Unit digit of (7)71 = 3
Þ 1£x£6 31 = 3, 32 = 9, 33 = 27, 34 = 81
So, number of values of x = 3 (2, 3 and 5) Unit digit of (3)65 = 3
12. (c); Let the required number be x. Unit digit of (6)63 = 6
According to the question, \ Product = 3 × 6 × 3 = 54
x2 + x = 2 × 3 × 5 \ Unit digit = 4
Þ x2 + x – 30 = 0 18. (c); 21 = 2, 22 = 4, 23 = 8, 24 = 16, 25 = 32
Þ x2 + 6x – 5x – 30 = 0 Unit digit repeats itself after 4 powers.
Þ x(x + 6) – 5 (x + 6) = 0
23
Þ (x – 5)(x + 6) = 0 Remainder of =3
4
\ x=5
\ (22)23 = (22)3 = 23 = 8
1 3 Unit digit = 8.
13. (b); Required number between and
2 5 19. (c); Given, (2153)167
1 3 167
+ Then, remainder of =3
Þ 2 5 4
2 \ Unit digit of 33 (i.e., 27) = 7
5 + 6 11 4 20. (b); Such number is always divisible by 9. To make it
Þ = » clear, you can take some example.
20 20 7
Example:
14. (c); Let two consecutive even numbers are x and
496 – (4 + 9 + 6) = 477,
(x+2).
which is divisible by 9.
\ According to the question,
971 – (9 + 7 + 1) = 954,
(x + 2)
2
- x 2 = 84 which is divisible by 9.

19 @BEST300MCQ For More Study Material


Visit: studyiq.com
21. (d); According to the question,
11 7
Divisor = 5 × Remainder = 0.73 ; = 0.875
15 8
= 5 × 46 = 230
\ Greatest fractions is 7/8.
230 26. (d); 6767 = (68 – 1)67 when divided by 68, leaves
Quotient = = 23
10 remainder (– 1)67 = – 1
Dividend = Divisor × Quotient + Remainder \ Required remainder = – 1 + 67 = 66
Dividend = 230 × 23 + 46 = 5290 + 49 = 5336 27. (d); We know that,
Sum of squares of 1st n natural numbers
\ Dividend = 5336
22. (c); Given, a and b are odd positive integers. n ( n + 1)( 2n + 1 )
=
a4 – b4 = (a2 + b2) (a + b) (a – b) 6
Let two positive odd integers be 1 and 3. Required sum = (Sum of squares of natural
\ Required number numbers from 1 to 10) – 12
= (12 + 32) (3 + 1) (3 – 1) = 80
10 ( 10 + 1 )( 2 ´ 10 + 1) 10 ´ 11 ´ 21
Which is divisible by 8. = - 12 = -1
6 6
Last remainder = 385 – 1 = 384
23. (d); Required remainder =
New divisor 28. (b); Here, 13 + 23 + ... + 103 = 3025
Now, 4 + 32 + 108 + ... + 40000
36 2 = 4(1 + 8 + 27 + ... + 1000)
Required remainder = =2
17 17 = 4(13 + 23 + 33 + ... + 103)
\ Remainder = 2 = 4 × 3025 = 12100
Last remainder 29. (d); To find the smallest fraction first we have to find
24. (c); Remainder = the decimal equivalent of fractions
New divisor
7 7 4 5
= 1.166, = 0.777 , = 0.8 and = 0.714
63 5 6 9 5 7
Remainder = =2 =5
29 29
5
25. (a); Decimal equivalent of fractions Therefore, the smallest number is .
7
2 5 1
= 0.67 ; = 0.83 30. (b); 0.001 =
3 6 999

1. (b); Let the three consecutive natural numbers be x,


Divisor
x + 1 and x + 2. and remainder = = 2x
5
According to the question,
According to the question,
x2 + (x + 1)2 + (x + 2)2 = 2030
Remainder = 40
Þ x2 + x2 + 2x + 1 + x2 + 4x + 4 = 2030
Þ 2x = 40
Þ 3x2 + 6x + 5 = 2030
Þ 3x2 + 6x – 2025 = 0 \ x = 20
Þ x2 + 2x – 675 = 0 Now, Dividend = Divisor × Quotient
Þ x2 + 27x – 25x – 675 = 0 + Remainder
Þ x(x + 27) – 25(x + 27) = 0 = x × 10x + 40 = 10x +40 = 4000 + 40 = 4040
2

Þ (x – 25) (x + 27) = 0 3. (a); Given, m and n are positive integers and m-n is
an even number.
Þ x = 25 or – 27
\ Three consecutive natural numbers are 25, 26 Let, m – n = 2p ... (i)
and 27 where, 2p is the even difference
Now, required number = 26 So, it is clear that both m and n may be either odd
2. (c); Let quotient = x or even
Then, divisor = 10x So, m + n = 2q ... (ii)

20 @BEST300MCQ For More Study Material


Visit: studyiq.com
where, 2q is the even sum of the numbers. Then,
1 1 1 1 1
on multiplying Eqs. (i) and (ii), we get 7. (c); + + + +
1 ´ 4 4 ´ 7 7 ´ 10 10 ´ 13 13 ´ 16
(m – n) (m + n) = 2p × 2q
Þ m2 – n2 = 4pq æ 1 1 1 1 1 1 1 1 1 ö 1
= ç1- + - + - + - + - ÷´
\ m2 – n2 will be divisible by 4. è 4 4 7 7 10 10 13 13 16 ø 3
4. (d); Since, the middle digits are given to be same.
æ 1 ö 1 15 1 5
\ Let the 99 digits numbers be = ç1- ÷´ = ´ =
è 16 ø 3 16 3 16
2 x x ... x x 2 8. (d); Required sum = [Sum of cubes of 1 to 10 natural
14243
97 digits numbers] – [Sum of cubes of natural numbers
from 1 to 4]
Sum of digits at odd places
é 10 ´ ( 10 + 1) ù é 4 ( 4 + 1) ù
2 2

2 + x+ x+... x+ x+ 2 =ê ú -ê ú
= 1442443 = 4 + 48x ë 2 û ë 2 û
48 digits
2 2
(there are 99 digits in all, 50 at odd places and 49 é 10 ´ 11 ù é 4 ´ 5 ù
=ê ú -ê ú
at even places) ë 2 û ë 2 û
Sum of digits at even places = 3025 – 100 = 2925
= x + x + ... + 49 terms = 49x 9. (b); Given,
Difference between the sum of digits at odd and (1012 + 25)2 – (1012 – 25)2 = 10n ... (i)
even places (a + b)2 – (a – b)2 = 4ab
\ (1012 + 25)2 – (1012 – 25)2
= 4 + 48x – 49x = 4 – x
= 4 × 1012 × 25 ... (ii)
Now, 4 – x = 0 or a multiple of 11 On comparing Eqs. (i) and (ii), we get
4–x=0 Þ x=4 10n = 4 × 1012 × 25 = 1014
5. (b); (1 !)5 = 1 i.e., 10n = 1014
(2 !)4 = 16 \ n = 14
(3 !)3 = 216 10. (b); Expression
(4 !)2 = 576 3 5 7 17 19
= + + + ... + 2 2 + 2
(5 !)1 = 120 12 × 2 2 2 2 × 3 2 3 2 × 4 2 8 ×9 9 × 10 2
The last 5 digit of (10 !)5 = 00000 On arranging
The last 5 digit of (100 !)4= 00000 æ 1 1 ö æ 1 1 ö æ 1 1 ö
=ç 2 - 2 ÷+ç 2 - 2 ÷+ç 2 - 2 ÷ + ...
(1000 !)3 = 00000 è1 2 ø è2 3 ø è3 4 ø
(10000 !)2 = 00000
æ 1 1 ö æ 1 1 ö
(100000 !)1 = 00000 +ç 2 - 2 ÷+ç 2 - 2 ÷
è8 9 ø è9 10 ø
Thus the last 5 digits of the given expression
= 00929 1 1 1 1 1 1 1 1 1 1
= - + - + - + ... + 2 - 2 + 2 - 2
[Q 1 + 16 + 216 + 576 + 120 + 00000 + 00000 12 2 2 2 2 32 32 4 2 8 9 9 10

+ 00000 + 00000 + 00000 = 00929] 1 1 100 - 1 99


= 1- 2
= 1- = =
6. (a); Let the fraction be x. 10 100 100 100
According to the question, 11. (c); Let

4x 4 15 4x 15 4 1 1 1 1
+ = Þ = - + + + ... +
7 7 14 7 14 7 1+ 2 2+ 3 3+ 4 99 + 100

15 - 8 7 1 1 7 7 2-
= = = Þ x= ´ = = ´ = 2 -1
14 14 2 2 4 8 1+ 2 2 +1 2 -1

7 1 3- 2
\ must be added. = = 3- 2
8 2+ 3 ( 3- 2 )( 3- 2 )
21 @BEST300MCQ For More Study Material
Visit: studyiq.com
\ Given expression = 16. (b); According to the question,
2 -1+ 3 - 2
Divisor = 25 × Quotient
+ 4 - 3 + ... + 100 - 99 Divisor = 25 × 16 = 400
= 100 - 1 = 10 - 1 = 9 Also, divisor = 5 × Remainder
12. (d); Expression
400
\ Remainder = = 80
1 1 1 1 1 1 5
= + + + + + ... +
2 6 12 20 30 n ( n + 1) \ Dividend = divisor × Quotient + Remainder
= 16 × 400 + 80 = 6400 + 80 = 6480
1 1 1 1 1
= + + + + ... +
1´ 2 2 ´ 3 3´ 4 4 ´ 5 n ( n + 1) 1
17. (a); Given, 3.718 =
0.2689
æ 1ö æ 1 1ö æ1 1ö æ1 1ö
= ç 1 - ÷ + ç - ÷ + ç - ÷ + ç - ÷ + ...
è 2 ø è2 3ø è3 4ø è4 5ø 1
Then, = 0.2689 ´ 10000 = 2689
0.0003718
1 1
+ - 18. (b); Let the smallest possible number be x, then
n n+1
x = 15k + 12 and x = 8l + 5
1 n Þ 15k + 12 = 8l + 5
= 1- =
n+1 n+1
Þ 15k + 7 = 8l
n ( n + 1)( 2n + 1 ) 15k + 7
13. (a); Sum of squares of n terms = Þ l= ,
6 8
Required sum = (sum of squares of natural
l must be an integer putting k = 1, 2, 3, ... etc.
numbers from 1 to 20) – 22 × (sum of squares of
natural numbers from 1 to 10) But at k = 7, we get a number which on being
divided by 8, gives 'l' as an integer.
20(20 + 1)(40 + 1) 2 2 ´ (10)(10 + 1) ´ (20 + 1) So, x = 15 × 7 + 12, x = 117
= -
6 6 The next higher numbers
20 ´ 21 ´ 41 4 ´ 10 ´ 11 ´ 21 = (L.C.M. of divisors) m + 117
= -
6 6 = (L.C.M. of 15 and 8) m + 117 = 120m + 117
= 2870 – 1540 = 1330 So consider the highest possible value of m such
14. (a); Taking in pairs. that 120m + 117 £ 9999 (largest possible number
of four digit)
[Q (a2 – b2) = (a – b) (a + b)]
(1 – 2 ) + (3 – 4 ) + ... + (92 – 102)
2 2 2 2 Thus at m = 82, the value of 120m + 117 = 9957,
which is the required number.
= (1 + 2) (1 – 2) + (3 + 4) (3 – 4) + ...
+ (9 + 10)(9 – 10) 19. (b); The number of zeros at the end of (5 !)5! = 120
= – 3 – 7 – 11 – 15 – 19 = – 55 [Q 5! = 120 and thus (120)120 will give 120 zeros]
and the number of zeros at the end of the (10 !)10!,
1 1 1
15. (a); First term = = - (50 !)50! and (100 !)100! will be greater than 120.
5´6 5 6
Now since the number of zeros at the end of the
1 1 1 whole expression will depend on the number
Second term = = - which has least number of zeros at the end of the
6´7 6 7
number among other given numbers.
1 1 1 So, the number of zeros at the end of the given
20th term of series = = -
24 ´ 25 24 25 expression is 120.
\ Required sum
x
20. (b); Let the fraction be , then
æ1 1ö æ1 1ö æ 1 1 ö y
= ç - ÷ + ç - ÷ + ... + ç - ÷
è5 6ø è6 7ø è 24 25 ø 2
æxö x2
1 1 5-1 4 ç ÷ =
= - = = = 0.16 èyø y2
5 25 25 25

22 @BEST300MCQ For More Study Material


Visit: studyiq.com
So, the required remainder
2 2
x
3 10x 2 4 + 4 + 4 + ...99 times 396
= = =
then 1 2 3y 2 6 6
y
5 Thus the remainder is zero.
28. (b); Let the number be N then
10x 2 x x 3
Thus 2
=2 Þ = N = 34Q + R, ... (i)
3y y y 5
where Q is any quotient
Hence x+y=3 +5=8 Again N = 5D and D is also a quotient
21. (d); 7777 + 7777 × 7777 × (5 ÷ 77) × (11 ÷ 35) but D= R+8
5 11 so N = 5 (R + 8) ... (ii)
= 7777 + 7777 × 7777 × ´ \ 5 (R + 8) = 34Q + R
77 35
5R + 40 = 34Q + R
= 7777 + 1111 × 1111 = 7777 + 1234321 = 1242098
Þ 34Q – 40 = 4R
22. (a); The last digit of 32 3232 is same as 2 3232 Þ 17Q – 2R = 20
But 32 So the minimum possible value of Q = 2 and the
2 32 = 2 32´32´ 32´...´ 32 times corresponding value of R = 7
Þ 32
2 32 = 2 4´8´(32´32´...´ 31 times ) So N = 34 × 2 + 7
N = 75
Þ
32
2 32 = 2 4n , Hence (b) is correct Choice.
where n = 8 × (32 × 32 × ... × 31 times) 29. (a); Since the product of 4 prime factors = 1365
Again 24n = (16)n Þ unit digit is 6, for every n Î N = 3 × 5 × 7 × 13
Hence, the required unit digit = 6. and the sum of the 3 prime factors
23. (c); The last digit of the expression will be same as = 25 = (5 + 7 + 13)
the last digit of 2888 + 8222. Now, total number of factors of the required
Now the last digit of 2888 is 6 and the last digit of number N = 24
the 8222 is 4. = 23 × 3 Þ (1 + 1)(1 + 1)(1 + 1)(2 + 1)
Thus the last digit of 2888 + 8222 is 0 (zero), since 6 Let N can be expressed as N = 3p × 5q × 7r × 13s
+ 4 = 10. Thus, for N to be greatest possible number in the
24. (a); 111! = 1 × 2 × 3 × 4 × 5 × ... × 100 × 111 above expressed form, the power of the greatest
Since there is a product of 5 and 2 hence it will prime factors will be greater.
give zero as the unit digit. So, N = 3 × 5 × 7 × 132
Hence the unit digit of 111! is 0 (zero). = 105 × 169 = 17745
25. (d); 46n – 64n = (64)2n – (36)2n = (64n + 36n) (64n – 36n) 30. (a); Total numbers in the set = (800 – 200) + 1 = 601
For n = 1, 3, 5, ... etc. (64n + 36n) is divisible by 100 Number of numbers which are divisible by 5
and all its factors. Also (64n – 36n) is divisible by ( 800 - 200 )
28 and all its factors. = + 1 = 121
5
Again for n = 2, 4, 6, ... etc. (64n – 36n) is always
Number of numbers which are divisible by 7
divisible by 100 and all its factors. Also it is
divisible by 28 and all its factors. ( 798 - 203 )
= + 1 = 86
26. (d); 19n – 1 is divisible by 18 = (19 – 1) when n is even 7
or odd. So (a) is correct. Number of numbers which are divisible by both
19n – 1 is divisible by 20 only when n is even so 5 and 7
(b) is wrong. æ 770 - 210 ö
19n – 1 is never divisible by 19 which is correct. =ç ÷ + 1 = 17
è 35 ø
Thus (d) is the most appropriate answer.
\ Number of numbers which are either divisible
27. (a); The remainder when 101 is divided by 6 is 4 by 5 or 7 or both
The remainder when 102 is divided by 6 is 4 = (121 + 86) – 17 = 190
The remainder when 103 is divided by 6 is 4 Thus the number of numbers in the given set
The remainder when 104 is divided by 6 is 4 which are neither divisible by 5 nor by 7
The remainder when 105 is divided by 6 is 4 = 601 – 190 = 411.
Thus the remainder is always 4. Hence (a) is correct option.

23 @BEST300MCQ For More Study Material


Visit: studyiq.com
1. (a); Let the natural numbers be x and y.
54
\ Required sum = 18x + 21y = 3 (6x + 7y) Now, remainder of =2
4
Hence, the sum is divisible by 3.
\ Unit digit = (3)2 = 9
Out of the given options, only 2007 is completely Required digit = Sum of (9 + 5 + 6) = 20
divisible by 3.
Hence, required unit digit = 0
2. (a); Let first three consecutive natural numbers be x,
7. (b); Unit digit of given numbers
x + 1 and x + 2.
(251)98 = ... 1
According to the question,
(21)29 = ... 1
x + x + 1 + x + 2 =27
(106)100 = ... 6
Þ 3x + 3 = 27 Þ x = 8 (705)35 = ... 5
First three consecutive numbers 8, 9 and 10. (16)4 = ... 6
\ Sum of next 3 consecutive numbers 259 = ... 9
= 11 + 12 + 13 = 36 \ Required answer = 1 + 1 – 6 + 5 – 6 + 9 = 4
n (n + 1) Hence, unit digit = 4
3. (c); Sum of first n natural number = 8. (b); We have, (6n – 1)
2
If n is even, then taking n = 2,
Sum of first 25 natural numbers.
6n – 1 = 62 – 1 = 36 – 1 = 35
\ 1 + 2 + 3 + ... + 25
Here, number 35 is divisible by 35.
25 ( 25 + 1 ) Hence, for any even value of n, (6n – 1) is divisible
= = 25 ´ 13 by 35.
2
\ The factor 13 is one of numbers 9. (d); Given, divisor = a
and remainder = 18
4. (d); Required sum = (sum of natural numbers from 1
to 100) – (sum of natural numbers from 1 to 50.) We know that,
sum of [1 + 2 + 3 + 4 + ... + 100] Dividend = (Divisor × Quotient) + Remainder
Through opotins
n(n + 1) 100(101) 228 = (70 × 3) + 18
= = = 5050
2 2 Hence, biggest two digit value = 70
and sum of [1 + 2 + 3 + 4 + ... + 50] 10. (c); Expression = 216 – 1 = (28)2 – 1
= (28 + 1) (28 – 1) = (256 + 1) (256 – 1)
n(n + 1) 50(51)
= = = 1275 = 257 × 255
2 2
which is exactly divisible by 17.
Hence, sum of [51 + 52 + 53 + ... + 100]
(since, 17 × 15 = 255)
= 5050 – 1275 = 3775
11. (b); Dividend = (Divisor × Quotient) + Remainder
5. (d); Given, (124)376 + (124)375 = 175 × q + 132 = 25 × 7 × q + 25 × 5 + 7
41 = 4 ; 42 = 16 ; 43 = 64 = 25(7q + 5) + 7
Remainder on dividing 376 by 4 = 0 It is clear that when the number is divided by 25,
\ Unit digit of (124)376 = 6 remainder will be 7.
Remainder on dividing 375 by 4 = 3 12. (a); Given, 461 + 462 + 463 = 461 (1 + 4 + 42) = 461 × 21
\ Unit digit of (124)375 = 4 Here, 21 is divisible by 3.
\ Sum = 6 + 4 = 10 13. (a); Given, (2464)1793 × (615)317 × (131)491
Hence, unit digit = 0. Then, unit digit of (2464)1793
6. (d); Unit digit in the expansion of 56527 = 5 1793
Remainder of =1
(5 repeats for every power increased) 4
36526 = Unit digit in 6526 = 6 Hence, unit digit of (2464)1793 = 4
(6 repeats for every power increased) Unit digit of (615)317 = 5
Now, 31 = 3, 32 = 9, 33 = 27 ; 34 Unit digit of (131)491 = 1
= 81, 35 = 243 ... so, unit digit = 4 × 5 × 1 = 20 = 0

24 @BEST300MCQ For More Study Material


Visit: studyiq.com
14. (b); Expression = (271 + 272 + 273 + 274) 21. (d); Let the number be x and quotient be y.
= 271(1 + 2 + 4 + 8) = 271 × 15 = 271 × 3 × 5
x 1
Which is exactly divisible by 10. \ Case I =y
13 13
15. (b); Given, remainder = 4
According to the question, Case II Now, quotient is divided by 5 and
remainder is 3.
Divisor = 3 × Remainder
Þ Divisor = 3 × 4 = 12 y 3
\ =1
Again, divisor = 4 × Quotient 5 5
Þ 4 × Quotient = 12
\ y = (5 × 1) + 3 = 8
12
Þ Quotient = =3 x 1
4 and =8
13 13
Dividend = Quotient × Divisor + Remainder
x = (8 × 13) + 1 = 105
= 3 × 12 + 4 = 40
16. (c); If a number is divisible by two numbers 105 40
separately, then it should be divisible by their Now, =1
65 65
product.
Remainder = 40
17. (a); (xn + yn) is exactly divisible by (x + y)
when, n is odd. 22. (c); Since, p, q, r are in geometric progression.
Here, x = 13, y = 11 \ q2 = pr
and n = 5, 7. Then, q = pr
\ The common factor
= x + y = 13 + 11 = 24 23. (c); Expression = 4 + 44 + 444 + ... to n terms
18. (d); Let three consecutive even integers be x, x + 2 = 4 (1 + 11 + 111 + ... to n terms)
and x + 4, respectively. multiplying and dividing by 9.
According to the question.
4
x + x + 2 + x + 4 = 54 = (9 + 99 + 999 + ... to n terms)
9
Þ 3x + 6 = 54 Þ 3x = 48
\ x = 16 On rearranging
\ The least even number = 16 4
19. (a); We know that = [(10 - 1) + (100 - 1) + (1000 - 1) + ... n terms]
9
21 = 2, 22 = 4,
23 = 8 4
= [(10 + 10 2 + 10 3 + ... n terms) - n]
24 = 16 9
25 = 32
4
2 repeats at unit's place after power of 4. = [(10(1 + 10 + 10 2 + ... n terms) - n]
9
Now, (122)173 = (1224)43.122

40 ( 10 - 1 ) 4
Unit digit in (122)173 = Unit digit in n

(1224)43 × Unit digit in 122 = . - n


9 9 9
= Unit digit in (6 × 2) = 2
20. (b); The least number of 5-digits = 10000
æ 10 n - 1 öù
41) 10000 (243 [Sum of n terms of the GP given as ç 9 ÷ú
è øû
82
180 40 4
164 =
81
( 10 n - 1 ) - n
9
160
123 24. (b); Expression = 2.349 = 2 + 0.349
37
( 349 - 3 ) 346 2326
\ Required number =2+ =2 =
990 990 990
= 10000 + (41 – 37) = 10004

25 @BEST300MCQ For More Study Material


Visit: studyiq.com
25. (c); Given, (52 + 62 + 72 + ... + 102) 27. (d); Sum of cubes of 1st n natural numbers
Sum of squares of first n natural numbers
ìï n ( n + 1 ) üï
2

n ( n + 1)( 2n + 1 ) =í ý
= ïî 2 ïþ
6
\ 13 + 23 + 33 + ... + 103
Required sum = (sum of squares of natural
numbers from 1 to 10) – (sum of squares of natural é 10 ´ ( 10 + 1) ù
2

numbers from 1 to 4) =ê ú [Q n = 10]


ë 2 û
10 ´ ( 10 + 1)( 20 + 1) 4 ´ ( 4 + 1)( 8 + 1 )
= - 2
6 6 æ 10 ´ 11 ö
÷ = ( 55 )
2

è 2 ø
10 ´ 11 ´ 21 4 ´ 5 ´ 9
= - = 55 × 55 = 3025
6 6
= 385 – 30 = 355 Last remainder
26. (c); Let the two numbers be x and y, respectively 28. (c); Remainder =
New divisor
x 1 63 5
\ = (given) =2 =5
y 2 Remainder =
29 29
or y = 2x ... (i)
( 25 )25 ( 252 ) ´ 25
12
Now, when 4 is added to each number the ratio 29. (d); =
becomes 2 : 3. 26 26

x+4 2 ( 625 )12 ´ 25 ( 1)12 ´ 25


Then, = = =
y+4 3 26 26
On solving, \ Remainder = 25
3x + 12 = 2y + 8 30. (a); According to the question,
3x +12 = 2(2x) + 8 [from Eq. (i)] Divisor = 555 + 445 = 1000
3x + 12 = 4x + 8 Dividend = ?
Þ x=4 Quotient = (555 – 445) × 2 = 110 × 2 = 220
Now, from Eq. (i) Remainder = 30
y = 2x Dividend = (Divisor × Quotient) + Remainder
i.e., y=2×4=8 = (1000 × 220) + 30 = 220000 + 30 = 220030
\ The numbers are 4 and 8.

26 @BEST300MCQ For More Study Material


Visit: studyiq.com
Chapter
LCM and HCF
2
Factors and Multiples: If a divides b exactly, we say that a is a factor of b and also we say that b is multiple of a.
i.e. 7 is a factor of 14, 8 is factor of 24 e.t.c. or 14 is multiple of 7, 24 is multiple of 8.
HCF/G.C.D/G.C.M: The HCF of two or more than two numbers is the greatest number that divides each of them
exactly.
Method of finding HCF
(i) Factorization method: Express each one of the given numbers as the product of prime factors. The product of
common prime factors with least power gives HCF.
Example: Find the HCF of 42, 63 and 140, 42 = 7 × 2 × 3, 63 = 7 × 3 × 3, 140 = 7 × 5 × 2 × 2 , So HCF = 7
(i) Division Method: Suppose we have find the HCF of two given numbers. Divide the larger number by the
smaller one. Now divide the divisor by the remainder . Repeat this process till remainder is zero. The last
divisor is required HCF.
Example: Find the HCF of 148 and 185
148 185 1
148
37 148 4
148
XX
i.e. HCF = 37.
LCM : The least number which is exactly divisible by each one of the given number is called their LCM.
Methods
(1) Factorization method: Resolve each of the given number into a product of prime factors. LCM is the product
of terms of highest power of all factors.
Product of two numbers = their LCM × their HCF
Co-prime: Two numbers are said to be co-prime if their HCF is 1.
i.e. 4 and 3 arc co-prime numbers.
HCF and LCM of fractions

HCF of numerators LCM of numerators


(i) HCF = (ii) LCM =
LCM of denominators HCF of denominators
Important Results
(i) Product of two numbers = HCF of the numbers × LCM of the numbers
(ii) The greatest number which divides the number x, y and z leaving remainders a, b and c respectively.
= HCF of (x – a) (y – b) (z – c)
(iii)The least number which when divided by x, y and z leaves the remainder a, b and c respectively, is given by
[LCM of (x, y, z) + K], where K = (x – a) = (y – b) = (z – c)
(iv) The least number which divided by x, y and z leaves the same remainder k in each case, is given by
[LCM of (x, y, z) + K]

27 @BEST300MCQ For More Study Material


Visit: studyiq.com
(v) The greatest number that will divide x, y and z leaving the same remainder in each case, is given by
[HCF of (x –y), (y – z), (z – x)]
(vi) When the HCF of each pair of n given numbers is a and their LCM is b, then product of these numbers is given
by (a)n – 1 × b or (HCF)n–1 × LCM

2 4 6 LCM = 84
1. HCF of , and
3 5 7 12x = 84
x=7
HCF of numerators
Sol. HCF of fractions = Greatest Number = 4 × 7 = 28
LCM of denominators
5. A rectangular piece of cloth has dimentions 16 m and
HCF (2, 4, 6) 2 12m. What is the least number of equal square that
= LCM (3, 5, 7) = 105 .
can be cut out of this cloth?
Sol. HCF of 16 and 12 = 4
2. The HCF (GCD) of a, b is 12 and a and b are positive
16 ´ 12
integers and a > b > 12. The smallest value of (a, b) are No of pieces = 4 ´ 4 = 3 ´ 4 = 12
respectively
Sol. Given HCF of a, b = 12 6. Find the largest number, which divides 34, 90, 104,
leaving the same remainder in each case.
Let the numbers be 12x and 12 y, where x and y are
co-prime. Sol. Difference between numbers = 90 – 34, 104 – 90

But given a > b > 12 and 104 – 134

i.e. a = 36 = 56, 14 and 70.

b = 24 HCF of 56, 14 and 70 = 14

3. The LCM of three different numbers is 120. Which of i.e. 14 is largest number.
the following cannot be HCF 7. Which is the smallest multiple of 7, which when
(i) 4 (ii) 12 (iii) 35 (iv) 8 divided by 6, 9, 15 and 18 respectively leaves 4 as
remainder in each case.
Sol. LCM of three no is = 120
Sol. LCM of 6, 9, 15 and 18 = 90
Now, factors of 120 = 2 × 2 × 2 × 3 × 5,
Remainder = 4 , but number is also divisible by 7 so
Hence HCF can be 4, 8, 12
required number
But 35 can't be HCF.
= 90k + 4 ... (i)
4. Two numbers are in the ratio 3 : 4. If their LCM is 84,
By putting
then the greater number is
k = 1, 2, 3, 4 ...... and checking if it is divisible by 7
Sol. Let the number be 3x and 4x
Required number = 90k + 4 = 90 × 4 + 4 = 364
LCM of 3x and 4x = 12x

28 @BEST300MCQ For More Study Material


Visit: studyiq.com
Ques tions 11. The product of two numbers is 4107. If the HCF of the
numbers is 37, the greater number is
(a) 185 (b) 111
2 4 5 (c) 107 (d) 101
1. LCM of , , is
3 9 6 12. The ratio of two numbers is 3 : 4 and their HCF is 5.
20 10 Their LCM is
(a) (b)
3 3 (a) 80 (b) 48
20 8 (c) 120 (d) 60
(c) (d) 13. The LCM of two numbers is 1820 and their HCF is
27 27
26. If one number is 130. Then, the other number is
2. The HCF of two numbers is 8. Which one of the
following can never be their LCM? (a) 70 (b) 1690
(a) 24 (b) 48 (c) 364 (d) 1264
(c) 56 (d) 60 14. The HCF of two numbers 12906 and 14818 is 478.
3. The LCM of two numbers is 520 and their HCF is 4. If Their LCM is
one of the numbers is 52, then the other number is (a) 400086 (b) 200043
(a) 40 (b) 42 (c) 60012 (d) 800172
(c) 50 (d) 52 15. What is the greatest number which will divide 110
4. The HCF of two numbers is 96 and their LCM is 1296. and 128 leaving a remainder 2 in each case?
If one of the numbers is 864, the other is (a) 8 (b) 18
(a) 132 (b) 135 (c) 28 (d) 38
(c) 140 (d) 144 16. The smallest perfect square divisible by each of 6, 12
5. The product of two numbers is 216. If the HCF is 6, and 18 is
then their LCM is (a) 196 (b) 144
(a) 72 (b) 60 (c) 108 (d) 36
(c) 48 (d) 36 17. When a number is divided by 15, 20 and 35, each
6. Two numbers are in the ratio 3 : 4. If their HCF is 4, time the remainder is 8. Then, that smallest number
then their LCM is is
(a) 48 (b) 42 (a) 428 (b) 427
(c) 36 (d) 24 (c) 328 (d) 388
7. The LCM of two numbers, which are multiples of 12 18. Find the HCF of 35 and 30.
is 1056. If one of the numbers is 132, the other number (a) 5 (b) 6
is
(c) 7 (d) 8
(a) 12 (b) 72
19. The HCF and the product of two numbers are 15 and
(c) 96 (d) 132 6300 respectively. The number of possible pairs of
8. Two numbers are in the ratio 3 : 4. The product of the numbers are.
their HCF and LCM is 2028. The sum of the numbers
(a) 4 (b) 3
is
(c) 2 (d) 1
(a) 68 (b) 72
20. The LCM of two numbers is 12 times their HCF. The
(c) 86 (d) 91
sum of the HCF and the LCM is 403. If one of the
9. Two numbers are in the ratio 3 : 4. If their LCM is 240, numbers is 93, then other number is
the smaller of the two number is
(a) 124 (b) 128
(a) 100 (b) 80
(c) 134 (d) 138
(c) 60 (d) 50
21. Find the least number exactly divisible by 12, 15, 20,
10. The HCF of two numbers is 16 and their LCM is 160.
27.
If one of the number is 32, then the other number is
(a) 450 (b) 540
(a) 48 (b) 80
(c) 230 (d) 640
(c) 96 (d) 112

29 @BEST300MCQ For More Study Material


Visit: studyiq.com
22. The largest number which divides 25, 73 and 97 to 27. Four runners started running simultaneously from a
leave the same remainder in each case, is: point on a circular track. They took 200 s, 300 s, 360 s
(a) 24 (b) 23 and 450 s to complete one round. After how much
(c) 21 (d) 6 time do they meet at the strarting point for the first
time?
23. Find the largest 5 digits number which is exactly
(a) 1800 s (b) 3600 s
divisible by 12, 15, 18, 27.
(c) 2400 s (d) 4800 s
(a) 90000 (b) 99999
28. The LCM of two positive integers is twice the larger
(c) 99010 (d) 99900 number. The difference of the smaller number and
24. Find the least number which when divided by 20, 25, the HCF of the two numbers is 4. The smaller number
35, 40 leaves remainders 14, 19, 29 and 34 is:
respectively. (a) 12 (b) 6
(a) 1220 (b) 1394 (c) 8 (d) 10
(c) 1365 (d) 1470 29. Six bells commence tolling together at intervals of 2,
25. 3 different containers contain 496l, 403l and 713l 4, 6, 8, 10 and 12 s, respectively. In 30 min., how
mixture of milk and water. What biggest measure of many times do they toll together?
a container can measure all the 3 quantities exactly?. (a) 16 (b) 15
(a) 31 l (b) 41 l (c) 10 (d) 4
30. Rajesh is incharge of buying bread rolls and buns for
(c) 51 l (d) 52 l
a party. There are 10 buns in each box of buns and 8
26. Three bells ring simultaneously at 11am. They ring bread rolls in each box of bread rolls. Rajesh wants
at regular intervals of 20 min, 30 min and 40 min, to buy exactly the same number of buns and bread
respectively. The time when all the three ring together rolls. What is the smallest number of boxes he should
next is: buy for buns alone?
(a) 2 pm (b) 1 pm (a) 10 (b) 8
(c) 1:15 pm (d) 1:30 pm (c) 4 (d) 5

1. Three tankers contain 403 l, 434 l, 465 l of diesel, 1


respectively. Then, the maximum capacity of 5. The sum of two numbers is 45. Their difference is
container that can measure the diesel of the three 9
containers in exact number of times is of their sum. Their LCM is
(a) 31 l (b) 62 l (a) 200 (b) 250
(c) 41 l (d) 84 l (c) 100 (d) 150
2. A, B and C start together from the same point to travel 6. The ratio of the sum to the LCM of two natural
around a circular path of 30 km in circumference. A numbers is 7 : 12. If their HCF is 4, then the smaller
and B are travelling in the same direction and C in number is
the opposite direction. If A travels 5 km, B travels 7 (a) 20 (b) 16
km and C travels 8 km in an hour, then they all come (c) 12 (d) 8
together again after: 7. If the students of a class can be grouped exactly into
(a) 25 h (b) 30 h 6 or 8 or 10, then the minimum number of students in
(c) 15 h (d) 20 h the class must be
3. The traffic lights at three different road crossings (a) 60 (b) 120
change after 24 s, 36 s and 54 s, respectively. If they (c) 180 (d) 240
all change simultaneously at 10 : 15 : 00 am, then at 8. Three numbers are in the ratio 2 : 3 : 4 and their HCF
what time will they again change simultaneously? is 12. The LCM of the numbers is
(a) 10 : 16 : 54 am (b) 10 : 18 : 36 am (a) 144 (b) 192
(c) 10 : 17 : 02 am (d) 10 : 22 : 12 am (c) 96 (d) 72
4. Sum of two numbers is 384, HCF of the numbers is 9. The greatest common divisor of 33
333 334
+ 1and 33 + 1 is
48. The difference of the number is
(a) 100 (b) 192 (a) 33
333
+1 (b) 20
(c) 288 (d) 336 (c) 2 (d) 1

30 @BEST300MCQ For More Study Material


Visit: studyiq.com
10. A milk vendor has 21 l of cow milk, 42 l of toned milk 19. The sum of two numbers is 36 and their HCF is 4.
and 63 l of double toned milk. If he wants to pack How many pairs of such numbers are possible?
them in cans, so that each can contains same number (a) 1 (b) 2
of litres of milk and does not want to mix any two (c) 3 (d) 4
kinds of milk in a can, then the least number of cans
required as 20. The HCF of two numbers 12908 and 14808 is 672.
Their LCM is
(a) 3 (b) 6
(a) 284437 (b) 200043
(c) 9 (d) 12
(c) 60012 (d) 800172
11. There are 24 peaches, 36 apricots and 60 bananas
and they have to be arranged in several rows in such 21. Two jars of capacity 50 l and 80 l are filled with oil.
a way that every row contains the same number of What must be the capacity of a mug that can
fruits of only one type. What is the minimum number completely measure the oil of the two jars?
of rows required for this to happen? (a) 5 l (b) 15 l
(a) 12 (b) 9 (c) 10 l (d) 20 l
(c) 10 (d) 6 22. The traffic lights at three different road crossings
12. Three numbers which are coprimes to one another change after every 48 sec, 72 sec, and 108 sec
are such that the product of the first two is 551 and respectively. If they all change simultaneously at 8 :
that of the last two is 1073. The sum of the three 20 hours, then at what time will they again change
numbers is simultaneously?
(a) 75 (b) 81 (a) 8 : 27 : 12 hours (b) 8 : 25 : 14 hours
(c) 85 (d) 89 (c) 8 : 24 : 12 hours (d) 8 : 29 : 12 hours
13. Three sets of English, Mathematics and Science books 23. The LCM of two numbers is 2310 and their HCF is
containing 336, 240 and 96 books, respectively have 30. If one of the number is 7 × 30, Find the other
to be stacked in such a way that all the books are number.
stored subject–wise and the height of each stack is (a) 320 (b) 330
the same. Total number of stacks will be (c) 340 (d) 350
(a) 14 (b) 21 24. What is the least multiple of 7, which when divided
(c) 22 (d) 48 by 2, 3, 4, 5 and 6 leaves the remainders 1, 2, 3, 4 and
14. The sum of two numbers is 36 and their HCF and 5 respectively?
LCM are 3 and 105, respectively. The sum of the (a) 117 (b) 119
reciprocals of two numbers is (c) 121 (d) 123
2 3 25. The least number, which when divided by 12, 15, 20
(a) (b) or 54 leaves a remainder 4 in each case is:
35 25
(a) 450 (b) 454
12 2 (c) 540 (d) 544
(c) (d)
35 25 26. The maximum number of students among whom
15. What is the least number of square tiles required to 1001 pens and 910 pencils can be distributed in such
pave the floor of a room 15 m 17 cm long and 9 m 2 cm a way that each student gets same number of pens
broad? and same number of pencils, is
(a) 840 (b) 841 (a) 91 (b) 910
(c) 820 (d) 814 (c) 1001 (d) 1911
16. The HCF (GCD) of a, b is 12. a, b are positive integers 27. A, B and C start running at the same time and from
and a > b > 12. The smallest values of (a, b) are the same point in the same direction in a circular
respectively. stadium. A completes a round in 252 s, B in 308 s and
(a) 12, 24 (b) 24, 12 C in 198s. After what time will they meet again at the
(c) 24, 36 (d) 36, 24 starting point?
17. If P = 23 ×310 × 5 and Q = 25 ×3 × 7, then HCF of P and (a) 26 min 18 s (b) 42 min 36 s
Q is (c) 45 min (d) 46 min 12 s
(a) 2 . 3. 5.7 (b) 3.23 28. Three men step-off together from the same spot. Their
(c) 2 .3
2 7
(d) 25 . 310 . 5.7 steps measures 63 cm, 70 cm and 77 cm, respectively.
18. The sum of two numbers is 84 and their HCF is 12. The minimum distance each should cover, so that all
Total number of such pairs of numbers is can cover the distance in complete steps, is
(a) 2 (b) 3 (a) 9630 cm (b) 9360 cm
(c) 4 (d) 5 (c) 6930 cm (d) 6950 cm

31 @BEST300MCQ For More Study Material


Visit: studyiq.com
29. Find the largest number of four digits such that on 30. The total number of integers between 100 and 200,
dividing by 15, 18, 21 and 24 the remainders are 11, which are divisible by both 9 and 6 is
14, 17 and 20, respectively. (a) 5 (b) 6
(a) 6557 (b) 7556 (c) 7 (d) 8
(c) 5675 (d) 7664

1. The largest possible length of a tape which can (a) 231 (b) 301
measure 525 cm, 1050 cm and 1155 cm length of (c) 371 (d) 441
cloths in a minimum number of attempts without
9. Three bells, toll at interval of 36 sec, 40 sec and 48 sec
measuring the length of a fraction of the tape's length
respectively. They start ringing together at particular
is
time. They next toll together after :
(a) 25 (b) 105
(a) 6 minutes (b) 12 minutes
(c) 75 (d) None of these
2. There are three drums with 1653 litre 2261 litre and (c) 18 minutes (d) 24 minutes
2527 litre of petrol. The greatest possible size of the 10. Mr. Black has three kinds of wine, of the first kind
measuring vessel with which we can measure up the 403 litres, of the second 434 litres and of the third 465
petrol of any drum while every time the vessel must litres. What is the least number of full corks of equal
be completely filled is: size in which these can be stored without mixing?
(a) 31 (b) 27 (a) 31 (b) 39
(c) 19 (d) 41 (c) 42 (d) 51
3. Mr. Baghwan wants to plant 36 mango trees, 144 11. Abhishek, Bobby and Charlie start from the same
orange trees and 234 apple trees in his garden. If he point and travel in the same direction round an
wants to plant the equal no. of trees in every row, but Island 6 km in circumference. Abhishek travels at the
the rows of mango, orange and apple trees will be
separate, then the minimum number of rows in his 1
rate of 3km/hr , Bobby at the rate of 2 km/hr and
garden is : 2
(a) 18 (b) 23 1
(c) 36 (d) Can't be determined Charlie at the rate of 1 km/hour. In how many
4
4. Find the least possible perfect square number which hours will they come together again?
is exactly divisible by 6, 40, 49 and 75
(a) 6 hrs (b) 12 hrs
(a) 176400 (b) 15000
(c) 24 hrs (d) 15 hrs
(c) 175600 (d) 16500
12. The LCM of two numbers is 4 times their HCF. The
5. Three bells in the Bhootnath temple toll at the interval
sum of LCM and HCF is 125. If one of the numbers is
of 48, 72 and 108 seconds individually. If they have
100, then the other number is
tolled all together at 6 : 00 AM then at what time will
they toll together after 6 : 00 AM? (a) 5 (b) 25
(a) 6 : 07 : 15 AM (b) 6 : 07 : 12 AM (c) 100 (d) 125
(c) 4 : 04 : 12 AM (d) 6 : 06 : 12 AM 13. LCM of two numbers is 120 and their HCF is 10.
6. What is the least possible number which when Which of the following can be the sum of those two
divided by 18, 35 and 42 leaves 2, 19 and 26 as the numbers?
remainders respectively? (a) 140 (b) 80
(a) 400 (b) 740 (c) 60 (d) 70
(c) 614 (d) 621 1
7. Find the HCF of 0.0005, 0.005, 0.15, 0.175, 0.5 and 14. A fraction becomes when 4 is subtracted from its
6
3.5.
numerator and 1 is added to its denominator. If 2
(a) .0005 (b) .005 and 1 are respectively added to its numerator and
(c) .05 (d) .5
8. The least number which when divided by 2, 3, 4, 5 1
the denominator, it becomes . Then, the LCM of the
and 6 leaves the remainder 1 in each case. If the same 3
number is divided by 7 it leaves no remainder. The numerator and denominator of the said fraction, must
number is: be

32 @BEST300MCQ For More Study Material


Visit: studyiq.com
(a) 14 (b) 350 (a) 70 (b) 77
(c) 5 (d) 70 (c) 63 (d) 56
15. From a point on a circular track 5 km long, A, B and C 18. The number of integers in between 100 and 600, which
started running in the same direction at the same are divisible by 4 and 6 both is
1 (a) 40 (b) 42
time with speeds of 2 km/h, 3 km/h and 2 km/h,
2 (c) 41 (d) 50
respectively. Then, on the starting point all three will 19. The least number, which when divided by 18, 27 and
meet again after: 36 separately leaves remainders 5, 14 and 23
(a) 30 h (b) 6 h respectively, is
(c) 10 h (d) 15 h (a) 95 (b) 113
16. The HCF of two numbers is 15 and their LCM is 300. (c) 149 (d) 77
If one of the numbers is 60, the other is
20. The largest number of five digits which, when divided
(a) 50 (b) 75 by 16, 24, 30, and 36 leaves the same remainder 10 in
(c) 65 (d) 100 each case, is
17. HCF and LCM of two numbers are 7 and 140, (a) 99279 (b) 99370
respectively. If the numbers are between 20 and 45, (c) 99269 (d) 99530
the sum of the numbers is

1. The HCF and LCM of two numbers are 44 and 264 (a) 48 (b) 36
respectively. If the first number is divided by 2, then (c) 24 (d) 16
quotient is 44. The other number is
8. The product of the LCM and the HCF of two numbers
(a) 147 (b) 528
is 24. If the difference of the numbers is 2, then the
(c) 132 (d) 264 greater of the number is
2. The ratio of two numbers is 5 : 6 and their LCM is
(a) 3 (b) 4
480, then their HCF is
(a) 20 (b) 16 (c) 6 (d) 8
(c) 6 (d) 5 9. The HCF and product of two numbers are 15 and
3. Product of two coprime numebers is 117, then their 6300, respectively. The number of possible pairs of
LCM is the numbers is
(a) 9 (b) 13 (a) 4 (b) 3
(c) 39 (d) 117 (c) 2 (d) 1
4. The LCM of two numbers is 48. The numbers are in 10. The LCM of two numbers is 20 times their HCF. The
the ratio 2 : 3. The sum of the numbers is sum of HCF and LCM is 2520. If one of the numbers
(a) 28 (b) 32 is 480, the other number is
(c) 40 (d) 64 (a) 400 (b) 480
5. The ratio of two numbers is 4 : 5 and their HCF is 8. (c) 520 (d) 600
Then, their LCM is
2 4 6
(a) 130 (b) 140 11. HCF of , and is
3 5 7
(c) 150 (d) 160
6. If the HCF and LCM of two consecutive (positive) 48 2
(a) (b)
even numbers is 2 and 84 respectively, then the sum 105 105
of the numbers is
1 24
(a) 30 (b) 26 (c) (d)
105 105
(c) 14 (d) 34
12. The LCM and the HCF of the numbers 28 and 42 are
7. The HCF and LCM of two numbers are 8 and 48 in the ratio
respectively. If one of the numbers is 24, then the other (a) 6 : 1 (b) 2 : 3
number is
(c) 3 : 2 (d) 7 : 2

33 @BEST300MCQ For More Study Material


Visit: studyiq.com
13. If the LCM and HCF of two expressions are (x2 + 6x+ 22. What least number must be subtracted from 1936, so
8) (x+ 1) and (x + 1) respectively and one of the that the resulting number when divided by 9, 10 and
expression is x2 + 3x + 2, then find the other. 15 will leave in each case the same remainder 7?
(a) x2 + 5x + 4 (b) x2 –5x+4 (a) 37 (b) 36
(c) x2 + 4x + 5 (d) x2– 4x+5 (c) 39 (d) 30
14. Two numbers are in the ratio 3 : 4. Their LCM is 84. 23. The largest number, which divides 25, 73 and 97 to
Then, the greater number is leave the same remainder in each case, is
(a) 21 (b) 24 (a) 24 (b) 23
(c) 28 (d) 84 (c) 21 (d) 6
15. Two numbers, both greater than 29, have HCF 29 24. The least multiple of 13, which on dividing by 4, 5, 6,
and LCM 4147. The sum of the numbers is 7 and 8 leaves remainder 2 in each case, is
(a) 966 (b) 696 (a) 2520 (b) 842
(c) 669 (d) 666 (c) 2522 (d) 840
16. The HCF and LCM of two numbers are 11 and 385, 25. What is the greatest number that will divide 307 and
respectively. The numbers are 330 leaving remainders 3 and 7, respectively?
(a) 55 and 57 (b) 55 and 77 (a) 19 (b) 16
(c) 17 (d) 23
(c) 44 and 77 (d) 22 and 770
26. Let the least number of six digits which when divided
17. The HCF and LCM of two 2 digit numbers are 16 and
by 4, 6, 10 and 15 leaves in each case same remainder
480, respectively. The numbers are
2 be N. The sum of digits of N is
(a) 40, 48 (b) 60, 72
(a) 3 (b) 5
(c) 64, 80 (d) 80, 96
(c) 4 (d) 6
18. Find the greatest number which will exactly divide 27. The smallest square number divisible by 10, 16 and
200 and 320. 24 is:
(a) 10 (b) 20 (a) 900 (b) 1600
(c) 16 (d) 40 (c) 2500 (d) 3600
19. The greatest number which can divide 1356, 1868, 28. The greatest number, which divides 989 and 1327
2764 leaving the same remainder in each case is leaving remainders 5 and 7 respectively, is
(a) 260 (b) 64 (a) 8 (b) 16
(c) 124 (d) 128 (c) 24 (d) 32
20. What is the least number which, when divided by 5, 29. Find the largest number of four digits such that on
6, 7 and 8 gives the remainder 3 but is divisible by 9? dividing by 15, 18, 21 and 24 the remainders are 11,
(a) 1463 (b) 1573 14, 17 and 20 respectively.
(c) 1683 (d) 1793 (a) 6557 (b) 7556
21. The greatest number of four digits which when (c) 5675 (d) 7664
divided by 12, 16 and 24 leave remainder 2, 6 and 14 30. The greatest number, that divides 122 and 243 leaving
respectively is respectively, 2 and 3 as remainders is
(a) 9974 (b) 9970 (a) 12 (b) 24
(c) 9807 (d) 9998 (c) 30 (d) 120

S olut i ons 2. (d); HCF of two numbers is 8. This means 8 is a factor


common to both the numbers.
\ LCM must be multiple of 8. By going through
2 4 5 the options, 60 cannot be the LCM since it is not
1. (a); Given, fractions , and
3 9 6 a multiple of 8.
LCM of numerators 3. (a); HCF × LCM = Product of two numbers.
LCM of fractions =
HCF of denominators 4× 520 = 52 × Second number
LCM(2, 4, 5) 20 4 ´ 520
= = \ Second number = = 40
HCF(3,9,6) 3 52

34 @BEST300MCQ For More Study Material


Visit: studyiq.com
4. (d); HCF × LCM 12. (d); The ratio of two numbers is 3 : 4 and their HCF is
= First number × Second number 5. Their LCM is
Given that the numbers are in ratio of 3 : 4,
96 ´ 1296
\ Second number = = 144 First number = 3 × 5 =15
864 Second number = 4 × 5 = 20
5. (d); HCF × LCM = Product of two numbers. LCM × HCF = Product of two numbers
Then,
15 ´ 20
\ LCM = = 60
216 5
LCM = = 36
6 13. (c); We know that,
6. (a); Let the numbers be 3x and 4x LCM × HCF = Product of two numbers
HCF = x = 4 26 ´ 1820
\ Second number = = 364
LCM of 3x and 4x = 12x = 12 × 4 = 48 130
7. (c); Given, first number = 132 14. (a); We know that,
Since, each of the two numbers is a multiple of 12 HCF × LCM = Product of two numbers
(given),
12906 ´ 14818
\ HCF = 12 and LCM = 1056 (given) \ LCM = =
478
LCM × HCF = First number × Second number
LCM = 400086
1056 ´ 12 15. (b); Required number
\ Second number = = 96
132 = HCF of {(110 – 2) and (128 – 2)}
8. (d); Let the numbers be 3x and 4x respectively. = HCF of 108 and 126
HCF ×LCM By Division Method
= First number × Second number 108 126 1
\ 2028 = 3x × 4x 108
2028 18 108 6
x2 = = 169 , x = 13 108
12 ×
Numbers are = 39, 52
\ Greatest number = 18
Sum of numbers = 39 + 52 = 91 16. (d); The LCM of 6, 12 and 18 = 36
9. (c); Let the numbers be 3x and 4x. 36 is a perfect square of 6.
LCM of 3x and 4x = 12x 17. (a); The smallest number
But given, LCM = 240 = [LCM of (15, 20, 35) +k]
\ 12x = 240
3 15, 20, 35
240 5 5, 20, 35
Þ x= = 20
12 1, 4,7
\ Smaller number = 3x = 3 × 20 = 60
10. (b); HCF × LCM = Product of two number LCM = 4 ×7 × 5 × 3= 420
Smallest number = 420 + 8 = 428
16 ´ 160
\ Second number = = 80 18. (a); 30 5 × 3×2
32
35 5 ×7
11. (b); HCF × LCM = Product of two numbers
So, HCF = 5
4107
\ LCM = = 111 , ab × HCF = LCM 19. (c); Let two numbers be 15x and 15y.Where x and y
37 are co-prime to each other.
111 \ 15x × 15y = 6300
where a, b are prime factors ab = =3
37 6300
x×y=
Prime number pairs (3, 1) 15 ´ 15
Numbers = 3 × HCF, 1 × HCF x × y = 28
\ Numbers are 111 and 37. Factors of 28 are 1, 2, 4, 7, 28
\ 28 = 1 × 28 or 4 × 7
Hence, 111 is the greater number
\ There are only two possible pairs

35 @BEST300MCQ For More Study Material


Visit: studyiq.com
20. (a); HCF + LCM = 403 LCM = 10 × 2 × 3 × 2 = 120
HCF + 12 HCF = 403 Hence, the bells will simultaneously ring after
(It is given that LCM = 12 × HCF) 2h i.e., at 1 pm
13 × HCF = 403 27. (a); Required time = LCM of (200, 300, 360, 450) s.
10 200, 300, 360, 450
HCF = 31
Q Product of two number is equal to product of 5 20, 30, 36, 45
their LCM and their HCF. 3 4, 6, 36, 9
\ 93 × other number = 31 × 12 × 31 2 4, 2, 12, 3
31 ´ 12 ´ 31 2 2, 1, 6, 3
Other number = = 124 3 1, 1, 3, 3
93
21. (b); We need to find the LCM of 12, 15, 20, 27. 1, 1, 1, 1
12 = 22 × 3 \ LCM = 10 × 5 × 3 × 2 × 2 × 3 = 1800 s
15 = 3 × 5 28. (c); Let the numbers be ax and bx, where x is the HCF
and bx > ax.
20 = 22 × 5
\ LCM = abx
27 = 33
abx = 2bx
LCM = Product of highest powers of factors
Þ a=2
= 22 × 33 × 5 = 540
Again, ax – x = 4
22. (a); Let x be the remainder then (25 – x), (73 – x) and
(97 – x) will be exactly divisible by the required Putting the value of a, we get
number. 2x – x = 4
Required number = HCF of (73 – x)– (25 – x), (97 Þ x =4
– x) – (73 – x) and (97 – x) – (25 – x) \ Smaller number = ax = 2 × 4 = 8
= HCF of (73 – 25), (97 – 73) and (97 – 25) 29. (a); All the 6 bells ring together will be LCM of
= HCF of 48, 24 and 72 = 24 (2, 4, 6, 8, 10 and 12)
23. (d); LCM of 12, 15, 18, 27 = 540
Largest number of 5 digits = 99999 2 2, 4, 6, 8, 10, 12
On dividing 99999 by 540, remainder = 99 2 1, 2, 3, 4, 5, 6
\ Required number = 99999 – 99 = 99900 3 1, 1, 3, 2, 5, 3
24. (b); 20 – 14 = 25 – 19 = 35 – 29 = 40 – 34 = 6 1, 1, 1, 2, 5, 1
Required number = (LCM of 20, 25, 35, 40) – 6 \ They will ring together after
= 1400 – 6= 1394 = 2 × 2 × 2 × 3 × 5 = 120 s
25. (a); To find the biggest measure, we have to find the i.e., they will ring together after 2 min
HCF of 496, 403 and 713.
\ Number of time they will ring together in 30
HCF of 496, 403 and 713 = 31
26. (b); Required time = LCM of 20, 30 and 40 30
min = 1 + = 1 + 15 = 16 times
2
10 20, 30, 40 30. (c); Smallest number of boxes for buns alone
2 2, 3, 4
LCM of 10 and 8 40
1, 3, 2 = = =4
Number of buns in a box 10

36 @BEST300MCQ For More Study Material


Visit: studyiq.com
1. (a); Capacity of three containers containing diesel is 4. (c); Let the numbers be 48a and 48b, where a and b
403 l, 434 l and 465 l, respectively. are coprimes,
Now, maximum capacity of the container that \ 48a + 48b = 384
can measure the diesel of the three containers Þ 48 (a +b) = 384
exactly
384
= HCF of quantity of three containers Þ a+b= =8 ... (i)
48
= HCF (403, 434, 465) Possible valid pairs of a and b satisfying this
condition are (1, 7) and (3, 5).
403 434 1
403 \ Numbers are 48 × 1 = 48
31 403 13 and 48 ×7 = 336 or
403 or, 48 × 3 = 144
× and 48 × 5 = 240
\ Required difference = 336 – 48 = 288
Again, 31 465 15
465 or 240 – 144 = 96
× 5. (c); Let the numbers be a and b.
So, Capacity of container = 31 L According to the question,
a + b = 45 ... (i)
30
2. (b); Time taken by A = h 1
5 Again, a – b = (a + b)
9
30
Time taken by B = h 1
7 Þ a – b = ´ 45 Þ a – b = 5 ... (ii)
9
30 Adding equ. (i) and (ii), we get
Time taken by C = h
8 2a = 50
Time taken by all to come together 50
a= = 25
30 30 30 2
= LCM of , and
5 7 8 On putting the value of a Eq. (i) we get
25 + b= 45
LCM of 30, 30and 30 30 Þ b = 45 – 25 = 20,
= = = 30h
HCF of 5,7 and 8 1 a = 25 and b = 20
3. (b); LCM of 24, 36 and 54 \ LCM of 25 and 20.

2 24, 36, 54 5 25,20


2 12, 18, 27 5 5, 4
2 6, 9, 27 1, 4
3 3, 9, 27
LCM = 5 ×5 × 4 = 100
3 1, 3, 9 6. (c); Let the numbers be 4a and 4b where a and b are
3 1, 1, 3 coprimes.
1, 1, 1 LCM = 4ab

Required time = LCM of 24, 36 and 54 s = 216 s ( 4a + 4b ) = 7


\
4ab 12
216 36
= = 3 min 1 1 7 1 1
60 60 Þ + = = +
= 3 min 36 s a b 12 3 4
\ Time when they will change simultaneously Þ a = 3, b = 4
\ First number = (4 × 3) = 12
= 10 : 15 : 00 + 3 min 36 s
Second number = (4 × 4) = 16
= 10 : 18 : 36 am
\ Smaller number = 12

37 @BEST300MCQ For More Study Material


Visit: studyiq.com
7. (b); Minimum number of students = LCM of 6, 8, 10. 12. (c); Let the numbers be p, q and r which are coprime
to one another.
2 6, 8, 10
Now, pq = 551 and qr = 1073
2 3, 4, 5
q = HCF of 551 and 1073
2 3, 2, 5
3, 1, 5 551 1073 1
551
LCM = 2 × 2 × 2 × 3 × 5 = 120 522 551 1
Hence, the minimum number of students = 120 522
8. (a); Let the number be 2x, 3x and 4x, respectively.
29 522 18
\ HCF = x = 12
522
\ Numbers 2 × 12 = 24, 3 × 12 = 36, 4 × 12 = 48
×
LCM of 24, 36, 48
551
2 24, 36, 48 \ q = 29 \ p= = 19
29
2 12, 18, 24
2 6, 9, 12 1073
and r= = 37
2 3, 9, 6 29
3 3, 9, 3 \ Sum of three numbers.
3 1, 3, 1 = 19 + 29 + 37 = 85
1, 1, 1 13. (a); Number of books in each stack = HCF of (336,
240, 96)
\ LCM = 2 × 2 × 2 × 2 × 3 × 3 = 144
9. (d); Given, 240 336 1
240
333 334 96 240 2
33 + 1and 33 + 1 or 27 333 + 1333 and 27 334 + 1334 192
Now, xm + am is divisible by (x + a) when m is 48 96 2
odd. 96
27333+ 1333 is divisible by (27 + 1) =28 ×
Similarly, 27334 + 1334 is never divisible by (x+ a)
\ Number of books in each stack = 48
So, the greatest common divisor between

(3 ) ( )
336 240 96
3333 + +
334
+ 1 and 33 + 1 is 1. \ Total number of stacks =
48 48 48
10. (b); Maximum quantity in each can = 7 + 5 + 2 = 14
= HCF of (21, 42 and 63) L = 21 L 14. (c); Givn, HCF = 3, LCM = 105
By Division Method
Now, let the numbers be 3a and 3b,
21 42 2 \ 3a + 3b = 36
42
Þ a + b = 12 ... (i)
×
21 63 3 and LCM 3ab = 105 ... (ii)
63 Dividing Eq. (i) by Eq. (ii), we have
×
a b 12
HCF = 21 L + =
\ Least number of cans 3ab 3ab 105

21 42 63 1 1 4 1 1 12
= + + = 1 + 2 + 3 = 6 cans. Þ + = Þ + =
21 21 21 3a 3b 35 a b 35
11. (c); To find the minimum number of rows, we 15. (d); Length of the floor = 15 m 17 cm = 1517 cm
determine the HCF of 24, 36 and 60. Breadth of the floor = 9 m 2 cm = 902 cm
\ HCF of 24, 36 and 60 = 12
Area of the floor = 1517 × 902 cm2
Thus, 12 fruits are there in a row.
The number of square tiles will be least, when
24 36 60 the size of each tile is maximum.
\ Number of rows = + +
12 12 12 \ Size of each tile = HCF of 1517 and 902
= 2 + 3 + 5 = 10

38 @BEST300MCQ For More Study Material


Visit: studyiq.com
902 1517 1 22. (a); LCM of 48, 72 and 108 = 432
902 The traffic lights will change simultaneously
615 902 1 after 432 seconds or 7 m 12 secs.
615 \ They will change simultaneously at
287 615 2 = 8 : 20 hours + 7 m + 12 sec. = 8 : 27 : 12 hrs.
574 23. (b); Product of two numbers = H.C.F. × L.C.M.
41 287 7
7 × 30 × Second number = 30 × 2310
287
30 ´ 2310
Second number = = 330
HCF = 41 7 ´ 30
\ Required number of tiles 24. (b); LCM of 2, 3, 4, 5 and 6 = 60
Area of the floor 1517 ´ 902 Other numbers divisible by 2, 3, 4, 5, 6 are 60k,
= = = 814
Area of each square tile 41 ´ 41 where k is a positive integer. Since 2 – 1 = 1, 3 – 2
= 1, 4 – 3 = 1, 5 – 4 = 1 and 6 – 5 = 1, the remainder
16. (d); Given, HCF of a and b = 12 in each case is less than the divisor by 1. Now,
Let the numbers be 12x and 12y, where x and y the required number is to be divisible by 7. Hence,
are coprime. we must choose the least value of k which will
But given a > b > 12 make (60k – 1) divisible by 7. Putting k equal to 1,
2, 3 etc. in succession, we find that k should be 2
Smallest coprime pair for the above condition
= (3, 2) \ The required number = 60k – 1
\ a = 36 and b = 24 = 60 × 2 – 1 = 119
17. (b); Given, 25. (d); Required number
P = 23 × 310 ×5, Q = 25 × 3 × 7 = (LCM of 12, 15, 20 & 54) + 4
[Q23 and 3 is common factor to both P and Q] = 540 + 4 = 544
HCF = 23 × 3 26. (a); Required number of students = HCF of 1001 and
18. (b); HCF = 12 910
Numbers = 12a and 12b where, a and b are 910 1001 1
coprimes 910
\ 12a + 12b = 84 Þ 12 (a +b) = 84 91 910 10
910
84
Þ a+b = =7 0
12
Hence, HCF = 91
\ Possible pairs of numbers satisfying this
27. (d); Required time = LCM of 252, 308 and 198 s.
condition = (1, 6), (2, 5) and (3, 4)
Hence, pairs of required numbers = 3. 2 252, 308, 198
19. (c); HCF of two numbers = 4 2 126, 154, 99
Hence, the numbers can be expressed as 4a and 7 63, 77, 99
4b, where a and b are coprime, 9 9, 11, 99
4a + 4b = 36, a +b = 9 11 1, 11, 11
Now, possible pairs satisfying above condition
1, 1, 1
are (1, 8), (4, 5), (2, 7).
\ 3 pairs are possible \ LCM = 2 × 2 × 7 × 9 × 11
20. (a); We know that,
2772
HCF × LCM = Product of two numbers = 2772 s =
60
12908 ´ 14808 1
\ LCM = = 284437 = 46 min = 46 min 12 s
672 5
21. (c); Factors of 50 = 52 × 2 28. (c); Minimum distance each should cover, so that all
Factors of 80 = 51 × 24 can cover the distance in complete steps
H.C.F. of 50 & 80 = 51 × 21 = 10 l = LCM of (63, 70, 77) = 6930 cm
The capacity of the mug must be 10 l

39 @BEST300MCQ For More Study Material


Visit: studyiq.com
29. (b); LCM of 15, 18, 21, 24 2520 9999 3
2 15, 18, 21, 24 7560
2 15, 9, 21, 12 2439
Required number = 9999 – 2439 – 4 = 7556
2 15, 9, 21, 6
ì 15 - 11 = 4 or ü
3 15, 9, 21, 3 ï 18 - 14 = 4 or ï
ï ï
3 5, 3, 7, 1 Where, 4 = í ý
ï 21 - 17 = 4 or ï
5 5, 1, 7, 1
ïî 24 - 20 = 4 ïþ
7 1, 1, 7, 1
30. (b); LCM of 9 and 6 = 18
1, 1, 1, 1 Total numbers from 1 to 200 divisible by 18 = 11
LCM = 2 × 2 × 2 × 3 × 3 × 5 × 7 = 2520 Total numbers from 1 to 100 divisible by 18 = 5
Largest number of four digit = 9999 \ Required numbers from 100 to 200
divisible by 18 = 11 – 5 = 6

1. (b); The largest possible length of the tape = HCF of = (LCM of 18, 35 and 42) – 16
525, 1050, 1155 = 105 = 630 – 16 [(18 – 2) = (35 – 19)= (42 – 26) = 16]
Hence (b) is the correct answer. = 614
2. (c); The maximum capacity of the vessel = HCF of
1653, 2261 and 2527 = 19 7. (a); 0.0005 Þ 5
Hence, (c) is the correct option. 0.0050 Þ 50
3. (b); Minimum number of rows means max. number
0.1500 Þ 1500
of trees per row, also equal number of trees per
row is required so we need to find the HCF of 36, 0.1750 Þ 1750
144 and 234 to find the maximum number of trees 0.5000 Þ 5000
in a row. 3.5000 Þ 35000
Thus HCF of 36, 144 and 234 = 18
Thus the number of rows = Then the HCF of 5, 50, 1500, 1750, 5000 and 35000
Total no.of trees is 5.
36 + 144 + 234
= = = 23 So the HCF of the given number is 0.0005 (since
No.of trees in a row 18
there are four digits in all the adjusted (or
Hence (b) is correct answer. equated) decimal places.)
4. (a); The required number must be divisible by the 8. (b); The required number = (LCM of 2, 3, 4, 5, 6) K +
given numbers so it can be the LCM or its multiple 1 = 7 l = 60K +1 =7l
number.
60 k + 1
Now the LCM of 6, 40, 49 and 75 Þ =l
=2×2×2×3×5×5×7×7 7
But the required number is a perfect square Now put the least possible value of k such that l
must be a positive integer. Hence at k = 5, l is an
Thus the LCM must be multiplied by 2 × 3 = 6.
integer. Thus, the required value is 60 × 5 +1 =
Thus the required number 301.
= (2 × 2 × 2 × 3 × 5 × 5 × 7 × 7) × (2 × 3)
9. (b); The required time = LCM of 36, 40 and 48
=2×2×2×2×3×3×5×5 ×7×7
= 720 seconds = 12 minutes
= 176400
Hence, (b) is the right choice.
5. (b); The three bells toll together only at the LCM of
the times they toll individually. 10. (c); Minimum number of corks =
Thus the LCM of 48, 72 and 108 is 432 seconds. 403 + 434 + 465 1302
= = 42
Therefore all the bells will toll together at 6 : 07 : HCF of (403, 434, 465) 31
12 AM
( 432 seconds = 7 minutes 12 seconds) 11. (c); Time taken for each of three persons is
6. (c); Since the difference between the divisors and the 6 6 6
respectively , and hrs
respective remainders is same. 3 2 1 1
1
Hence the least possible number 2 4

40 @BEST300MCQ For More Study Material


Visit: studyiq.com
2 12 24 On subtracting Eq. (ii) from Eq. (i), we get
i.e., , and hrs.
1 5 5 6x - y = 25
So, it is required to find the LCM of 3x - y = -5
2 12 24 24 3x = 30
, , = = 24hr
1 5 5 1 30
x= = 10
Hence, (c) is the right choice. 3
12. (b); Let LCM be x and HCF be y. on putting the value of x in Eq. (i), we get
According to the question, 6 × 10 – y =25
LCM = 4 × HCF Þ – y = 25 – 60 Þ – y = – 35
Þ x = 4y Þ y =35
According to the question, \ x = 10 and y = 35
LCM + HCF = 125 LCM of 10 and 35
x + y = 125
5 10,35
Putting the value of x, we get
2,7
5y = 125
Þ y = 25 LCM = 2 × 7 × 5 = 70
\ HCF = 25 amd LCM = 4 × 25 = 100 15. (c); A makes one complete round in 5/(5/2)
We know that, HCF × LCM
æ Dis tan ce ö
= First number × Second number =2h çQ Speed = Time ÷
è ø
HCF ´ LCM
Second number = First number 5 5
h and C completes in h .
B completes in
3 2
100 ´ 25 Hence, the required time
= = 25
100
æ 5 5ö
13. (d); Let the numbers be 10a and 10b, where a and b = LCM of ç 2, and ÷ h
è 3 2ø
are coprime
\ LCM of 10a and 10b = 10ab LCM of 2, 5, 5 10
= = = 10h
Þ 10ab = 120 Þ ab = 12 HCF of 3, 2 1
Possible pairs = (3, 4) or (1, 12) Hence, A, B and C will meet after 10 h.
Sum of the numbers :
(a, b) = (3, 4) : 16. (b); By the Technique
(3×10)+ (4×10) = 30 + 40 = 70 First number × Second number = HCF × LCM
(a, b) = (1, 12) :
15 ´ 300
(1 × 10) + (12 × 10) \ Second number = = 75
60
= 10 + 120 = 130
17. (c); Let the numbers be 7a and 7b, where a and b are
x coprime. Now, LCM of 7a and 7b = 7ab
14. (d); Let the original fraction be y . \ 7ab = 140
According to the question, 140
ab = = 20
x-4 1 7
= Þ 6x - 24 = y + 1 Now, required values of a and b whose product
y+1 6
is 20 are 4 and 5.
Þ 6x– y= 25 ... (i) Numbers are 28 and 35 and they lie between 20
Again, according to the question, and 45.
x+2 1 Sum of the numbers = 28 + 35 = 63
= 18. (c); We know that
y+1 3
Numbers divisible by 4 and 6 will be multiples
Þ 3x + 6 = y +1 of the LCM of 4 and 6 i.e., 12
Þ 3x – y = – 5 ... (ii) Now, numbers from 1 to 600 divisible by

41 @BEST300MCQ For More Study Material


Visit: studyiq.com
600 20. (b); LCM of 16, 24, 30 and 36.
12 = - 1 = 49
12 2 16, 24, 30, 36
(minus 1 because 600 is excluded) 2 8, 12, 15, 18
Now, numbers divisible by 12 from 1 to 100 2 4, 6, 15, 9
100 2 2, 3, 15, 9
= =8
12 3 1, 3, 15, 9
\ Numbers divisible by between 100 and 600 3 1, 1, 5, 3
= 49 – 8 = 41
5 1, 1, 5, 1
19. (a); 2 18, 27,36 1, 1, 1, 1
2 9,27,18 LCM = 2 × 2 × 2 × 3 × 2 × 5 × 3 = 720
3 9,27,9 We know that largest five digit number is 99999
3 3,9, 3
720 99999 138
3 1, 3,1 720
1,1,1 2799
2160
LCM = 2 × 2 ×3 × 3 × 3 = 108 6399
Required number 5760
= [LCM of (18, 27, 36)]-k = 108 –13=95 639
{where, k = 18 – 5 or 27 – 14 or 36 – 23 = 13} Required number = (99999 – 639) + 10 = 99370

1. (c); Here, first number = 2 ×44 = 88, 5. (d); Let the numbers be 4x and 5x.
HCF = 44 and LCM = 264. \ HCF = 8 = x
By the formula First number = 8 × 4 = 32
1st number × 2nd number = HCF × LCM Second number = 8 × 5 = 40
Þ 88 × 2nd number = 44 × 264 \ LCM of 32, 40 = 160
264 ´ 44 6. (b); Let the number be 2a and 2b, where a and b are
Þ 2nd number = = 132 coprime
88
Þ 2nd number = 132 \ LCM = 2ab
2. (b); Let the common ratio = x 2ab = 84
Then, numbers are 5x and 6x ab = 42 = 6 × 7
Now, HCF of these two numbers is x \ Numbers are 12 and 14.
By the technique \ Sum 12 + 14 = 26
LCM × HCF = Product of two numbers 7. (d); By the technique
Þ 480 ×x = 5x ×6x Þ 480x = 30x2 HCF × LCM = First number × Second number
\ x = 16 \ HCF is 16
3. (d); We know that, 8 ´ 48
\ Second number = = = 16
LCM of two coprimes is equal to their product, 24
Hence, LCM = 117 8. (c); Let the larger number be a.
4. (c); If the numbers be 2x and 3x, then LCM of 2x and Smaller number = a – 2
3x = 6x HCF × LCM = Product of two numbers
LCM = 48 24 = a(a – 2)
48 Þ a2 – 2a – 24 = 0
\ 6x = 48 Þ x = =8
6 Þ a2 – 6a +4a – 24 = 0
\ The numbers are (8 ×2 = 16) Þ a(a– 6)+4(a – 6)= 0
and (8 × 3= 24), respectively. Þ a = 6, - 4
\ Sum = 16 + 24 = 40 But a ¹ –4 \ a=6

42 @BEST300MCQ For More Study Material


Visit: studyiq.com
9. (c); Let the numbers are ax and bx 13. (a); LCM = (x2 +6x+8)(x+1)
where x = HCF = 15 or (x+4) (x+2) (x+1)
ATQ, HCF = (x+1)
ax × bx = 6300 1st expression = x2+ 3x+2
or (x+1) (x+2)
6300
ab = = 28 As we know that,
225
product of two expressions = LCM × HCF
\ possible pairs Þ 28 × 1, 7 × 4 (x+1) (x+2) × 2nd expression
Hence there are two pairs. = (x+4)(x+2)(x+1)(x+1)
10. (d); Let LCM be x and HCF be y. 2nd expressions
According to the question.
( x + 4 )( x + 2 )( x + 1)( x + 1 )
LCM = 20 × HCF =
( x + 1 )( x + 2 )
i.e., x = 20 y
and x + y = 2520 = (x + 4)(x + 1)= x2 + 4x + x + 4 = x2 + 5x + 4
Putting the value of x, we get 14. (c); Let the numbers be 3x and 4x.
20y +y = 2520 LCM of 3x and 4x = 12x
Now, LCM = 84
Þ 21y = 2520
Then, 12x = 84
2520
Þ y= = 120 84
21 Þ x= =7
12
\ LCM =x = 120 × 20 = 2400
\ Greatest number = 4x = 4 ×7 = 28
LCM × HCF = Product of two numbers
15. (b); Let the number be 29a and 29b, respectively
2400 × 120 = 48 × x, x = 600 where a and b are coprimes
2 4 6 LCM of 29a and 29b = 29ab
11. (b); Given fractions = , and Now, 29ab = 4147
4 5 7
4147
HCF of numerators
\ ab = = 143
HCF of fractions = LCM of deno min ators 29
Thus, ab = 11 × 13
HCF(2, 4,6) 2 First number = (29 ×11) = 319
= =
LCM (3,5,7) 105 Second number = (29 ×13) = 377
12. (a); LCM of 28 and 42 \ Sum of numbers = 319 + 377 = 696
2 28, 42 16. (b); Factors of 11 and 385 are
2 14, 21 11 = 11 × 1, 385 = 11 ×5 × 7
\ LCM = 11 × 5 × 7 = 385
7 7, 21
HCF = 11
1, 3
First number = 11 × 5 = 55
\ LCM = 2 ×2 ×7 ×3= 84 Second number = 11 × 7 = 77
HCF of 28 and 42 Þ (11, 385) or (55, 77)
By Division method 17. (d); HCF of the two digit numbers = 16
28 42 1 Hence, let the numbers be 16a and 16b.
28 where, a and b are coprimes.
14 28 2 Now, HCF × LCM = Product of two numbers.
28 Þ 16a × 16b = 16 × 480
0
\ HCF = 14 16 ´ 480
Þ ab = = 30
16 ´ 16
LCM 84 6
\ Ratio = = = =6:1 Possible pairs of a and b satisfying the condition
HCF 14 1 ab = 30 are (3, 10), (5, 6), (1, 30), (2, 15). Since the

43 @BEST300MCQ For More Study Material


Visit: studyiq.com
numbers are of 2 digit each. 22. (c); LCM of 9, 10 and 15
Hence, required pair is (5, 6). 2 9,10,15
First number = 16 × 5 = 80 3 9, 5,15
Second number = 16 × 6 = 96
18. (d); Greatest number that can exactly divide 200 and 3 3, 5, 5
320 = HCF of 200 and 320 = 40 5 1, 5, 5
200 320 1 1, 1,1
200 \ LCM = 2 × 3 × 3 × 5 = 90
120 200 1
90 1936 21
120 180
80 120 1 136
80 90
40 80 2 46
80 \ Required number = 46 – 7 = 39
× 23. (a); Required number
Hence the greatest number is 40. = HCF of [|25 – 73|,|73–97|,|97–25|]
19. (d); Required number = HCF of = HCF of {48, 24, 72}
{(1868–1356), (2764–1868), (2764–1356)} HCF = 2 × 2 × 2 × 3= 24
= HCF of (512, 896, 1408)
\ HCF = 24
512 896 1 \ Largest number = 24
512 24. (c); LCM of 4, 5, 6, 7 and 8 = 840
384 512 1 Required number = 840x +2
384 By Hit and Trial
128 384 3 Putting x = 3
384 we get = 840x + 2 = 840 × 3+ 2 = 2522
×
2522 is a multiple of 13.
Hence, required number is 128.
20. (c); LCM of 5, 6, 7, 8 = 35 × 24 =840 25. (a); Greatest number
Required number = 840x + 3, such that it is exactly = HCF of [(307 – 3), (330 – 7)]
divisible by 9. = HCF of (304, 323)
By hit and Trial 304 323 1
for x = 2, it is divisible by 9. 304
Required number = 840x + 3 = 840 ×2 + 3 = 1683 19 304 16
(these type of questions can be solved with the 304
help of given options) ×
(Out of all the given options, only 1683 is divisible \ Required number = 19
by 9.) 26. (b); Least six digit number is 100000
21. (a); LCM of 12, 16 and 24 LCM of 4, 6, 10, 15
2 12, 16, 24 2 4, 6, 10,15
2 6, 8, 12 2 2, 3, 5,15
3 3, 4, 6 3 1, 3, 5,15
2 1, 4, 2 5 1,1, 5, 5
2 1, 2, 1 1, 1,1,1
1, 1, 1 \ LCM = 2 × 2 × 3 × 5= 60
= 2 × 2 × 2 × 2 × 3 = 48 60 100000 1666
60
48 9999 208
96 400
360
399 400
384
15 360
\ Greatest four digit numbers divisible by 48 400
9999–15= 9984 360
\ Required number = 9984–10 = 9974 40
(10 is the difference of each remainder) \ Required number

44 @BEST300MCQ For More Study Material


Visit: studyiq.com
= 100000 + (60 – 40) +2 = 100022 29. (b); LCM of 15, 18, 21, 24
\ Sum of the digits of 2 15,18, 21, 24
N = 1 +0 + 0 + 0 + 2 + 2 = 5 2 15,9, 21,12
27. (d); LCM of 10, 16, 24 2 15,9,21,6
2 10,16, 24 3 15,9,21,3
2 5, 8,12 3 5, 3,7,1
2 5, 4,6 5 5,1,7,1
5, 2, 3 7 1,1,7,1
\ LCM of 22 × 22 × 5 × 3 1,1,1,1
[Qpowers must be equal for number to be perfect LCM = 2 × 2 × 2 × 3 × 3 × 5 × 7 = 2520
square]
Largest number of four digit = 9999
\ Required number
2520 9999 3
= 22 × 22 × 52 ×32 = 4 × 4 ×25 ×9= 3600 7560
28. (c); By the technique 2439
Required number Required number = 9999 – 2439 – 4 = 7556
= HCF of [(989 – 5), (1327 – 7)] ì 15 - 11 = 4 or ü
= HCF of (984, 1320) ï 18 - 14 = 4 or ï
ï ï
where, 4 = í ý
984 1320 1 21 - 17 = 4 or
ï ï
984 ïî 21 - 20 = 4 ïþ
336 984 2
30. (d); Required number = HCF of [(122–2),(243–3)]
672
i.e., HCF of (120, 240)
312 336 1
312 By Division Method
24 312 13 120 240 2
312 240
× ×
\ HCF = 24 \ Required number = 120
\ Required number = 24

45 @BEST300MCQ For More Study Material


Visit: studyiq.com
Chapter
Surds and Indices
3
Index: a × a × a × a × a ............................. m times
or (a × a × a × ..............................m times) × (a × a × a × .............................n times)
i.e. a × a × a × ........................... (m + n) times
Important formulae : If a > 0, a ¹ 1, m and n are integers then

am
(i) am × an = am+n (ii) am × an × aQ = am+n+Q (iii) (am)n = amn (iv) = am - n
an
1
(v) a0 = 1 (vi) a–m = (vii) a m n = a(m )n (viii) (ab)n = a n bn
am

Surd : If 'a' is a rational number and n is a positive integer such the nth root of 'a', i.e., a1/n n a is an irrational number,
then a1/n is called a surd. In other words, an irrational root of a rational number is called a surd.

Example: 2, 3
4, 4
18 , 7
4, 3
9 etc. are surds
i.e. we can say that every number expressed in a surd is an irrational number.
Types of surds:

(i) Pure Surd: 7 , 3 11, 4


125 are pure surds.

(ii) Mixed Surd: 3 2 , 7 2 11, 32 are mixed surds.

(iii) Similar surds: 3 3 , 3 and 6 5 , 7 125 = 35 5

order of surds: 7, 3
4, 4
8, 5 125 are respectively surds of order 2, 3, 4 and 5
Conjugate of surds : Two binomial surds which differ only in sign (+ or –) between the terms connecting them, are
known as conjugate surds
Example : Conjugate of 5 + 7 is 5 - 7
Condition for two Surds to be equal : If a, b, c, d are all rational numbers and b and d are not perfect square then
a+ b = c+ d , i.e. a = c and b = d

Square root of surd of a + b form

a + a2 - b a - a2 - b 2
-b a - a2 - b
+ b= + , - b= -
2 2 2 2
Important formulae:

1 1 y
(i) a- P = (ii) If ay = n then a = ( n ) y (iii) If ax = by then a = ( b ) x
aP
m

( a)
1 m
(iv) xn = a, x = n a (v) n
a =a n (vi) n
= an

(vii) n
a × n b = n ab

46 @BEST300MCQ For More Study Material


Visit: studyiq.com
1. The greatest among 7 – 5 , 5 – 3, æ 1 1 1ö
4. Is 2x = 3y = 6–z then ç x + y + z ÷ is equal to
9 – 7 , 11 – 9 è ø
Sol. Let 2x = 3y = 6–z = k
Sol. On rationalising
i.e. 2 = k1/x, 3 = k1/y, 6 = k–1/z,
( 7– 5 ´ ) ( 7+ 5 ),( 5– 3 ´ ) ( 5+ 3 ) 2×3=6
, k1/x × k1/y = k–1/z, k1/x + 1/y = k–1/z
7+ 5 5+ 3
1 1 1 1 1 1
+ =– , + + =0
( 9– 7 ´ ) ( 9+ 7 ),( 11 – 9 ´) ( 11 + 9 ) x y z x y z
9+ 7 ( 11 + 9 ) 5. Find the value of 3 +
1
+
1
+
1
3 3+3 3–3
2 2 2 2
= , , ,
7+ 5 5+ 3 9+ 7 11 + 9 1´ 3 1 3– 3 1 3+ 3
+ ´ + ´
2
Sol. 3 + 3´ 3 ( 3 + 3) 3– 3 ( 3–3 ) 3+ 3
smallest one = = 11 – 9
11 + 9 3 3 – 3 3 + 3 18
= 3+ + – = =3
2 3 6 6 6
Greatest one = = 5– 3
5+ 3
2. Greatest among the following numbers 1
6. The value of 5+2 6 –
3
9 , 3 , 4 16 , 6 80 5+2 6

Sol. LCM 3, 2, 4, and 6 = 12 1


Sol. Expression 5+2 6–
12
912/3, 312/2, 1612/4, and 80 6 5+2 6
94, 36, 163, 802
( )
2

6561, 729, 4096, 6400 5+2 6 –1 5+2 6–1


= =
i.e. largest one = 3 9 . 5+2 6 5+2 6

3. By how much does 12 + 18 exceed 3+ 2? 4+2 6 æ 3- 2ö


= ×ç ÷
Sol. Required value will be the differnce between 3+ 2 è 3- 2ø

12 + 18 and ( 3 + 2) = 4 3+6 2 –4 2–4 3=2 2


7. If a = 64 and b = 289, then the value of
= ( 12 + 18 ) – ( 3 + 2 )
( )
1/2
a+ b – a
= (2 3 + 3 2 ) – ( 3 + 2)
( )
1/2
Sol. + 289 – + 289
= 3+2 2
( )
1/2
8 + 17 – – 8 + 17 = (5 - 3)1/2 = 2

47 @BEST300MCQ For More Study Material


Visit: studyiq.com
Ques tions æ pö
n -1
æ qö
n -3

11. If ç ÷ =ç ÷ , then the value of n is:


è qø è pø
1. If 3x – 3x – 1 = 18, then xx is equal to
(a) 3 (b) 8 1 7
(a) (b)
(c) 27 (d) 216 2 2
2. If a 2x + 2
= 1, where a is a positive real number other (c) 1 (d) 2
than 1, then x = ? 12. Value of ? in 3
512 ¸ 4 16 + 576 = ? is:
(a) – 2 (b) – 1 (a) 24 (b) 31
(c) 0 (d) 1 (c) 28 (d) 18
2
1 1 1
ëé( 12 ) ûù
-2

=? 13. Value of 3 + + + – 3 is:


3. -2 3 3+ 3 3 -3
éë( 12 ) ùû
2

(a) 3 + 3 (b) 3
(a) 12 (b) 4.8
(c) 1 (d) 0
12 14. 165/4 = ?
(c) (d) 1
144 (a) 64 (b) 31
4. Value of ? in expression (c) 32 (d) 33
78.9 ÷ (343)1.7 × (49)4.8 = 7? is -3/5
æ 32 ö
(a) 13.4 (b) 12.8 15. çè ÷ =?
243 ø
(c) 11.4 (d) 9.6
5. If {(2 ) } = 256, find the value of '?'.
4 1/2 ? 27 27
(a) (b)
(a) 1 (b) 2 8 7
(c) 4 (d) 8 27 27
6. (16) ÷ (16) × 16 = (16)
9 4 3 ? (c) (d)
6 2
(a) 6.75 (b) 8 16. Find the value of (243) × (243)0.04
0.16

(c) 10 (d) 12
1
7. (42 × 229) ÷ (9261)1/3 = ? (a) 0.16 (b)
3
(a) 448 (b) 452 (c) 3 (d) 0.04
(c) 456 (d) 458 17. 173.5 × 177.3 ÷ 174.2 = 17?
8. Evaluate (0.00032)2/5. (a) 8.4 (b) 8
1 1 (c) 6.6 (d) 6.4
(a) (b)
625 225 x
18. If 289 = 17 5 , then x = ?
1 1 (a) 16 (b) 8
(c) (d)
125 25
2
(c) 10 (d)
7- 5 5
9. If = a + b 35 , then the value of (a – b) is:
7+ 5 -1
é ì 1 2 ü -2 ù
æ ö =?
(a) 5 (b) 6 19. ê íç - ÷ ý ú
ëê î 2 ø þ ûú
è
(c) 8 (d) None of these
1
10. If P = 124, then 3 P ( P 2 + 3P + 3) + 1 = ? (a) (b) 16
16
(a) 5 (b) 7
1
(c) 123 (d) 125 (c) - (d) – 16
16
48 @BEST300MCQ For More Study Material
Visit: studyiq.com
20. Find the value of (10)200 ÷ (10)196. 5

(x )
(a) 10000 (b) 1000 é -
5
ù
-3
ú =?
3
26. ê 5
5
(c) 100 (d) 100000 ë û
3a (a) x5 (b) x– 5
21. If æç ö÷
1
= 0.008 , find the value of (0.25) . a
è 5ø 1
(c) x (d)
(a) 20.5 (b) 22.5 x
(c) 0.25 (d) 6.25

1
27. ( )
+ ... ¸ 2 = ?
2

22. Value of +2 6 - is: (a) 0 (b) 1


5-2 6
(c) 2 (d) 8
(a) 2 2 (b) 0 28. If a = b, b = c and xyz = 1, then what is the value of cz?
x y

(a) a (b) b
(c) 2 3 (d) 5 -1
a
1 1 1 (c) ab (d)
23. + + =? b
1 + xb- a + xc - a 1 + xa - b + xc - b 1 + x b- c + xa - c
29. If 16 × 8n + 2 = 2m, then m is equal to:
(a) xa – b – c (b) 1
(a) n + 8 (b) 2n + 10
(c) 0 (d) 3
(c) 3n + 2 (d) 3n + 10
24. [P(b – c)](b + c) × [P(c – a)](c + a) × [P(a – b)](a + b) = ?
(a) 0 (b) Pabc 3+1 3 -1
30. If x = and y = , then x2 + y2 is equal to:
(c) 1 (d) Pa + b + c 3-1 3+1

25. If 5 + 3 x = 3 , then the value of x is: (a) 14 (b) 13


(c) 15 (d) 10
(a) 64 (b) 125
(c) 9 (d) 27

16 ´ 2 n + 1 - 4 ´ 2 n 6 5
1. =? (a) (b)
16 ´ 2 n + 2 - 2 ´ 2 + 13 13
(a) 1 (b) 2
8 7
1 1 (c) (d)
(c) (d) 13 13
2 4 5. If 2a + 3b = 17 and 2a+2 – 3b+1 = 5, the values of a and b
2 ¸ (2
23
) 2 3
´2 -2 are :
2. =? (a) a = 2, b = 3 (b) a = 4, b = 6
4 23
¸ (4 ) 2 3
´ 4-2 (c) a = 3, b = 2 (d) a = – 3, b = – 2
1 é 3 2 4 3 6 ù
(a) 2 (b) - +
2 6. ê ú is simplified to
(c) 1 (d) – 2 ë 6+ 3 6+ 2 3+ 2û
1 (a) 0 (b) 1
( 0.6 ) - ( 0.1 )
0 -1 -
æ 8 ö 3
3. -1 3 -1
+ç ÷ =? (c) 3 (d) 6
æ 3 ö æ3ö æ 1ö è 27 ø
ç 3 ÷ ç ÷ +ç- ÷
è2 ø è2ø è 3ø 2( 2 + 6)
7. Find the value of .
(a) 1 (b) 0 3 2+3
1 1
(c) (d) 1 2
2 4 (a) (b)
3 3
( 81) ´ ( 27 ) ´ 97
4x x

4. If = 39 , then find the value of x. 2 4


( 729 )
x +2
(c) - (d)
3 3

49 @BEST300MCQ For More Study Material


Visit: studyiq.com
5 -2 2n+4 - 2 ´ 2n
8. If x = , then x 4 + x -4 is 17. n+3
+ 2 -3 = ?
5+2 2´2
(a) a surd (a) 1 (b) 2
(b) a rational number but not an integer 1 1
(c) an integer (c) (d)
2 3
(d) an irrational number but not a surd 18. If 100.48 = x, 100.7 = y and xz = y2, then z = ?

19 + 8 3 11 11
9. The simplified value of is : (a) 1 (b) 2
12 12
7-4 3
11 11
(a) 11 - 6 3 (b) 11 + 6 3 (c) 3 (d) -2
12 12
(c) 10 + 5 3 (d) 10 - 5 3 -
9
2
é 5
ù
( )
10
19. The simplified value of ( 27 )
- -
1 1 3 + ê -2 3
úû is :
( ) - (7 - 4 3 ) ë 2 3
-
10. The simplified value of 28 + 10 3 2 2 is:
11 11
(a) 1 (b) 2 (a) (b) -
(c) 3 (d) 4 18 18
17 17
11. The simplified value of 6 - 4 3 + 16 - 8 3 is : (c) (d) -
18 18
(a) 3 (b) 3+1 20. Find the value of 8 + 3 7 - 7 + 3 5.
(c) 3-1 (d) 3+ 2
14 + 10 14 - 10
(a) (b)
+2 6 2 2
12. If x = , then the value of x2(x – 10)2 is
5-2 6 14 + 10 14 - 10
(c) (d)
(a) 0 (b) 1 4 4
(c) – 1 (d) 2
21. Find the value of 38 + 5 3 + - 5.
- 7
13. The simplified value of is: 5 6 - 10 5 6 + 10
+3 7 -2 2 (a) (b)
2 2
(a) 1 (b) 2
(c) – 1 (d) – 2 5 6 - 10 5 6 + 10
(c) (d)
4 4
( 625 ) ´ ( 25 )
6.25 2.6

14. =? 22. Find the value of 4- 7 + 8+


( 625 ) ´ ( 5 )
6.75 1.2

(a) 5 (b) 10 (a) 2 ( 7 + 1) (b) 2 +1


(c) 15 (d) 25 (c) 1 (d) 2 +2
2+ 3 23. If 5 = 2.236 and 10 = 3.162 , then the value of
15. The simplified value of is :
2- 3
15
(a) 7 + 4 3 (b) 7 - 4 3 is:
10 + 20 + 40 - 5 - 80
(c) 7 + 2 3 (d) 7 - 2 3 (a) 4.398 (b) 5.398
(c) 4.938 (d) 5.938
1
( 2 2 n - 3.2 2 n - 2 )( 3n - 2.3n - 2 )
( 0.3 ) æç ÷ö ( 9 ) 6 ( 0.81 ) 3
1
1 4 1 2
=?
3 24. 3n - 4 ( 4 n + 3 - 2 2 n )
è 27 ø
16. The simplified value of 2 1 1 is :
( 0.9 ) 3 ( 3 ) ( 243 )
- -
2 4 1 1
(a) (b)
2 4
(a) 2.2 (b) 2.7
(c) 2.4 (d) 2.6 1 1
(c) - (d)
2 6

50 @BEST300MCQ For More Study Material


Visit: studyiq.com
2+ 3 (a) 8 5 (b) -8 5
25. If = a - b 6 , then the value of (a + b)2 is :
3 2 -2 3 (c) 4 5 (d) -4 5
28. The simplified value of
13 49
(a) (b) 1 1 1 1 1
36 36 - + - + is :
3- 8 8- 7 7- 6 6- 5 5 -2
7 16
(c) (d) (a) 3 (b) 4
6 39 (c) 5 (d) 6
26. The simplified value 1 4
-
( 16 ) 4 æ 1 ö
-3 3
7 3 2 5 3 2 29. ( 9 ) ´ ´ç ÷ =?
- - is : ( 6 ) -2 è 27 ø
10 + 3 6+ 5 15 + 3 2
(a) 2 (b) 4
(a) 1 (b) 2 (c) 6 (d) 8
(c) 4 (d) 6
30. 7 7 7..............¥ = ?
5 -2 5+2
27. The simplified value - is : (a) 7 (b) – 7
5+2 5-2
(c) 4 (d) – 4

3 1+ x 1-x ( 2.4 )6 + 9 ( 5.76 ) + 6 ( 2.4 ) 4


1. If x = , then + is equal to 6.
2 1+ 1+ x 1- 1-x ( 2.4 ) 4 + 6 ( 5.76 ) + 9

2 1 1 1
é( 3-2 ) -5 ùû 5 + éë( 4-3 ) -6 ùû 6 - ( 3-4 )
-
(a) 1 (b) 2
3 +ë 1 =?
(c) 2 - 3 (d) 2 éë( 2 -3 )-4 ùû 4

(a) 10.2 (b) 9.4


2 24 x+ 8 x + 12
2. If x = , then the value of + is: (c) 10.4 (d) 10.8
3+ 2 x- 8 x - 12
7. If x is a positive real number and the index be a rational
(a) 1 (b) 2 number, then
(c) – 1 (d) – 2 a 2 + b2 - ab b2 + c 2 - bc c 2 + a 2 - ca
æ xa ö æ xb ö æ xc ö
ç -b ÷ ´ ç -c ÷ ´ ç -a ÷ =?
2 6 x+ 2 x+ 3 èx ø èx ø èx ø
3. If x = , then the value of + is:
3+ 2 x- 2 x- 3
(a) xa3 + b3 + c3 (b) xa2 + b2 + c2
(a) 2 (b) 3 (c) x 2 ( a 3 + b 3 ) (d) x2 (a3 + b3 + c 3 )
(c) 6 (d) 2
1 1 1
8. If 2x = 4y = 8z and + + = 4, then x = ?
3 2x 4y 4z
4. If x = , then the value of 1 + x + 1 - x will be
2
7 7
(a) (b)
1 15 16
(a) (b) 2 3
3 8 1
(c) (d)
(c) 3 (d) 2 15 2

4 +2 + -2
5. - 2 30 + - 2 10 - =? 9. Find the value of - -2 2
6+ 2 5 +1
(a) 0 (b) 3
(a) 2 (b) 2 +1
(c) – 1 (d) 2
(c) 1 (d) 2 +2

51 @BEST300MCQ For More Study Material


Visit: studyiq.com
8 1 1
+4 7 - -4 7 + (a) 3 (b) 2
10. 2 2
33 + 10 8 - - 10 8
=? 1 1
(c) 4 (d) 5
2 2
1 1
(a) 2 (b) 3 1
2 2
2 é( 625 ) 5 ´ ( 1024 )- 5 ¸ ( 25 ) 5 ù 2 ( 10 3 ) 2 ¸ ( 10 3 2 )
6 3 6 3
ë û +
1 1 15.
( 10 2 )3 ¸ 102
5 1 3

( 3 128 ) 2 ´ ( 125 ) 5
-
(c) 1 (d) 4
2 2
11. The simplified value of (a) 1.2 (b) 1.4
(c) 1.6 (d) 1.1
26 - 15 3 10 + 18
+ 1 1
3
( 18 - 8 2 )2 + (6 - 4 2 )
-
(3 - 16. + =
5)
é5 2 -
2
8+ is : 2
ë + 5 3 ûù +8 3 - -8 3

1 1 11 - 2 11 + 2
(a) 1 (b) 2 (a) (b)
3 3 2 2
1 1 11 - 2 2 11 + 2 2
(c) 3 (d) 4 (c) (d)
3 3 2 2
12. The simplified value of
11
17. 7 + 4 3 - 28 + 10 3 + =
1 1
7 20 + 6 11 + - 6 11
( 28 - 10 3 ) - (7 + 4 3 )
-
2 2 +
+6 7 - -6 7 1 1
is: (a) 2 (b) 1
2 2
1 1 1 1
(a) 1 (b) 2 (c) -1 (d) -2
2 2 2 2
1 1 1 3 4
(c) 3 (d) 4 18. - - =?
2 2 11 - 2 30 7 - 2 10 8+4 3
(a) 0 (b) 1
4 3 30 18
13. - - =? (c) – 1 (d) -2 3
2- 2 4 3 - 18 3-2 3

(a) 2 6 (b) 6 3 1+ x + 1-x


19. If x = , then =?
2 1+ x - 1-x
(c) 3 6 (d) 4 6
(a) - 3 (b) 3
14. The simplified value of
(c) - 2 (d) 2
8
3
é 3 52 ù
( ) a + 2b + a - 2b
15

ê 8 - 4 ú ´ 16
4
( 12.12 ) - ( 8.12 )
2 2
ë û 20. If x = , then the value of
+ a + 2b - a - 2b
( 0.25 )2 + ( 0.25 ) ´ ( 19.99 ) é 3 -5 ù
1 is :
3
ë {
ê ( 128 -5 ) 7 ú
û } bx2 – ax + b is :
(a) 0 (b) – 1
(c) 1 (d) – 2

52 @BEST300MCQ For More Study Material


Visit: studyiq.com
3 -6 2 x-1
æ3ö æ3ö æ3ö é 3+ 2 3- 2ù
1. If ç ÷ ç ÷ =ç ÷ , then x is equal to 8. ê - ú simplifies to
è5ø è5ø è5ø ë 3- 2 3+ 2û
(a) – 2 (b) 2
(a) 2 6 (b) 4 6
(c) – 1 (d) 1
(c) 2 3 (d) 3 2
7 -2
= a 7 + b , then the value of a is
2. If
7 +2 9. By how much does ( 12 + 18 ) exceed ( 2 3 + 2 2 )

11 4 (a) 2 (b) 3
(a) (b) -
3 3 (c) 2 (d) 3

4 4 7 10. The greatest number among 5, 3 4 , 5 2 , 7 3 is:


(c) (d) -
3 3
(a) 3
4 (b) 7
3
5+ 3 5- 3 (c) 5 (d) 5
2
3. If x = and y = then (x + y) equals:
5- 3 5+ 3
(a) 8 (b) 16 11. If 7 7 7 7..... = ( 343 ) y - 1 , then y is equal to

(c) 2 15 (d) 2 ( 5 + 3 ) 2
(a) (b) 1
3
2 -1
4. If 2 = 1.414 , the square root of is nearest to
2 +1 4 3
(c) (d)
(a) 0.172 (b) 0.414 3 4
(c) 0.586 (d) 1.414 1
12. is equal to
1 3
4 + 3 2 +1
5. For what value(s) of a is x + x + a2 is a perfect
4 (a) (b)
3
2 +1 3
2 -1
square?
(c) - 3 2 - 1 (d) 1 - 3 2
1 1
(a) ± (b)
18 8 1 1 1 1
13. + + + ..... +
1+ 2 2+ 3 3+ 4 99 + 100
1 1
(c) - (d) is equal to
5 4
6. Which of the following numbers is the least ? (a) 10 - 99 (b) 2 - 10
(c) 7 (d) 9
( 0.5 ) 2 , 0.49 , 3 0.008, 0.23
14. If 6 ´ 15 = x 10 , then the value of x is
(a) (0.5)2 (b) 0.49
(a) 3 (b) ± 3
(c) 3
0.008 (d) 0.23
(c) 3 (d) 6
7. Arrange the following in descending order :
15. The largest among the numbers
3
4 , 2 , 6 3, 4 5
7 - 5, 5 - 3, 9 - 7 , 11 - 9 is :
(a) 3
4> 5> 2> 3
4 6
(a) 7- 5 (b) 5- 3
(b) 4
5>34>63> 2
(c) 9- 7 (d) 11 - 9
(c) 2>63>34>45 16. The largest among the numbers (0.1)2, 0.0121 , 0.12
(d) 2> 4> 3> 5
3 6 4
and 0.0004 is :

53 @BEST300MCQ For More Study Material


Visit: studyiq.com
(a) (0.1)2 (b) 0.0121 2+ 3 2- 3
24. If a = and b = , then the value of (a2 +
(c) 0.12 (d) 0.0004 2- 3 2+ 3
b + ab)
2

17. 6 + 6 + 6 + ..... is equal to (a) 169 (b) 195


(a) 2 (b) 5 (c) 168 (d) 196
(c) 4 (d) 3 7
25. is equal to
( 81 ) ´ ( 9 )
3.6 2.7
16 + 6 7 - 16 - 6 7
18. The value of is :
( 81 )4.2 ´ ( 3 )
1 1
(a) 3 (b) 6 (a) (b)
2 3
(c) 9 (d) 8.2
1 1
3- 2 3+ 2 (c) (d)
4 5
19. If a = ,b= , then the value of
3+ 2 3- 2
26. By how much does 12 + 18 exceed 3+ 2?
2 2
a b
+ is : (a) 2( 3 – 2 ) (b) 2( 3 + 2 )
b a
(a) 900 (b) 970 (c) 3+2 2 (d) 2 –4 3
(c) 1030 (d) 930 -3/2
27. When simplified (256)–(4 )
is
5+ 3 5- 3
20. If 2 x = - , then the value of x is : 1
5- 3 5+ 3 (a) 8 (b)
8
(a) 6 (b) 30
1
(c) 15 (d) 15 (c) 2 (d)
2
21. If 3 = 1.732 , the value of 28. The largest among the numbers 0.9, 0.9 0.9 is
3+ 6 (a) 0.9 (b) (0.9) 2

is :
5 3 - 2 12 - 32 + 50 (c) 0.9 (d) 0.9
(a) 4.899 (b) 2.551
(c) 1.414 (d) 1.732 29. 3
0.004096 is equal to
(a) 4 (b) 0.4
22. 10 + 2 6 + 2 10 + 2 15 is equal to (c) 0.04 (d) 0.004
(a) ( 2 + 3 - 5) (b) ( 3+ 5 - 2) 30. The ascending order of (2.89)0.5, 2 – (0.5)2, 3 and
(c) ( 2 + 5 - 3) (d) ( 2 + 3 + 5)
3
0.008 is
(a) 2 – (0.5)2 , 3, 3 0.008 ,(2.89)05
( 12 - 8 )( 3 + 2 )
23. The value of is :
5 + 24 (b) 3
0.008,(2.89)0.5 , 3, 2 – (0.5)2
(a) 6- 2 (b) 6+ 2 (c) 3
0.008 , 3, (2.89)05 , 2 – (0.5)2
(c) 6 -2 (d) 2 - 6
(d) 3, 3 0.008, 2 – (0.5)2 , (2.89)05

S olut i ons 2. (b); a2x + 2 = a°


2x = – 2 Þ x = – 1
1. (c); 3x – 1 (3 – 1) = 18 12 -4
3x – 1 = 9 = 32 3. (d); =1
12 -4
x – 1 = 2 Þ x = 3 so, xx = 27

54 @BEST300MCQ For More Study Material


Visit: studyiq.com
7 8.9 ´ ( 7 2 )
4.8 -1
7 18.5 ì 1 ü
{}
-1
4. (a); 7 ? = = = 7 13.4 é -2
ù ï ï 1
( 7 3 )1.7 7 5.1 19. (a); ê 1 ú =í 1 ý = 16 -1 =
ë 4 û 16
îï 16 þï
? = 13.4
20. (a); 10200 – 196 = 10000
1
5. (c); 2 4´ ´?
= 28
2 3a 3
æ 1ö 8 æ 1ö
?=4 21. (c); çè ÷ø = 0.008 = =ç ÷
5 1000 è 5 ø
( )9 ´ ( 16 )3 3a = 3 Þ a = 1
6. (b); ( 16 )? = 16 = 16 9 + 3 - 4 = 16 8 (0.25)a = 0.25
( 16 ) 4
1
22. (b); ) 2) +2 3 2 -
2 2
?=8
( 3) +( 2) - 2 3 2
2 2

( 42 ´ 229 ) 42 ´ 229
7. (d); 1
= = 458
( 9261 ) 3 21 1
= 3+ 2- = 3+ 2- 3- 2 =0
3- 2
2 2
2
æ 32 ö 5 ìæ 2 ö ü 5 æ 1 ö
5
1 1 1 1
8. (d); ç ÷ = í ç ÷ ý = ç ÷ = + +
è 100000 ø îè 10 ø þ è 5ø 25 x b
x c
x a
x c
x b
xa
23. (b); 1 + + 1 + + 1 + +
xa xa xb x b xc x
7- 5
=
( 7 - 5 )( 7 - 5 )
= 6 - 35
9. (d); xa xb xc
7+ 5 2 = + + =1
xa + xb + x c xa + xb + x c xa + xb + x c
on comparision, a = 6 and b = – 1
a – b = 6 – (– 1) = 7 24. (c); P b2 - c2 × P c 2 - a2 × P a2 - b2 = P b2 - c2 + c2 - a2 + a2 - b2 = 1

25. (a); 5+ 3 x = 3
10. (d); P ( P 2 + 3P + 3) + 1 = 3 P 3 + 3P 2 + 3P + 1
3
x = 9 - 5 Þ x = 4 3 = 64
= ( P + 1) = P + 1
3 3

5
é - ù
{ }
5
-3 1 -5
P = 124, = P + 1 = 125 26. (c); ê - 35 ´ 15 3 ú = x 5 ´ 5 ´ 3 ´ 5 = x
ë x û
n -1 3-n
æ pö æ pö 27. (c); 56 = 8 × 7
11. (d); ç ÷ =ç ÷
è qø è qø
so, 56 + 56 + 56 + ..... = 8
n – 1 = 3 – n, 2n = 4 Þ n = 2
= 8 ÷ 22 = 2
12. (c); ? = 8 ÷ 2 + 24 = 28 28. (a); b = ax
by = axy
13. (d); 3 +
1
+
(3 - 3) +
( 3 + 3)
–3 byz = axyz
3 ( 3 + 3 )( 3 - 3) ( 3 - 3 )( 3 + 3 ) cz = a
29. (d); 24 × 23(n + 2) = 2m
1 3- 3 - 3 -3 1 1 2m = 2(3n + 10)
= 3+ + – 3 = 3+ - -3 = 0
3 6 3 3 3n + 10 = m
2 2
5
14. (c); ( 16 ) 4 = ( 2 4 ) 4 = 32
5
æ 3 + 1ö æ 3 - 1 ö
30. (a); x + y = ç
2 2 +
è 3 - 1 ÷ø çè 3 + 1 ÷ø
3 3
æ 243 ö 5 æ 35 ö 5 27
15. (a); ç =ç 5÷ = 4+2 3 4-2 3
è 32 ÷ø è2 ø 8 = +
4-2 3 4+2 3
16. (c); (243)0.2 = (35)0.2 = 3
( 4 + 2 3 ) + (4 - 2 3 )
2 2
17. (c); 173.5 + 7.3 – 4.2 = 17x
=
x = 6.6 ( 4 - 2 3 )( 4 + 2 3 )
x
x 16 + 12 + 16 3 + 16 + 12 - 16 3 56
18. (c); 17 2 = 17 5 Þ = 2 Þ x = 10 = = 14
5 =
16 - 12 4

55 @BEST300MCQ For More Study Material


Visit: studyiq.com
16 ´ 2 n + 1 - 4 ´ 2 n 313x+ 2 = 39 Þ 13x + 2 = 9
1. (c); Expression =
16 ´ 2 n + 2 - 2 ´ 2 + 7
13x = 9 – 2 = 7 Þ x=
13
2 4 ´ 2 n+ 1 - 2 2 ´ 2 n 2 n + 5 - 2 n +2
= = 5. (c); The equations are
2 4 ´ 2 n+ 2 - 2 ´ 2n +2 2n+ 6 - 2n + 3
2a + 3b = 17 and 2a × 22 – 3b × 3 = 5
2 n+ 5 - 2n + 2 2n +5 - 2n +2 1 Let x = 2a and y = 3b
= = =
n+ 5
2´2 - 2´2 n+ 2
2 ( 2n + 5 - 2n +2 ) 2 then, x + y = 17 ... (i)
and 4x – 3y = 5 ... (ii)
2 2 ¸ ( 2 2 ) ´ 2 -2
3 3
By equation (i) × 3 + (ii),
2. (c); Expression =
42 ¸ ( 42 ) ´ 4 -2
3 3
3x + 3y = 51
4x - 3y = 5
2 2 ´ 2 ´ 2 ¸ 2 2 ´ 3 ´ 2 -2
= 7x = 56
4 2 ´ 2 ´ 2 ¸ 4 2 ´ 3 ´ 4 -2

1 1 From equation (i),


-2 28 ´ ´ y = 17 – x = 17 – 8 = 9
2 ¸2 ´2
8 6
26 22 = 1
= = \ x = 8 and y = 9
48 ¸ 4 6 ´ 4 -2 1 1
48 ´ 6 ´ 2 \ x = 2a = 8 Þ 2a = 23 Þ a = 3
4 4
and y = 3b = 9 Þ 3b = 32 Þ b = 2
3. (b); Expression
\ a = 3 and b = 2
1
-
( 0.6 )0 - ( 0.1 )-1
æ 8 ö 3 3 2 4 3 6
= -1 3 -1
+ç ÷ 6. (a); Expression = - +
æ ö æ ö +æ
3 3 1 ö è 27 ø 6+ 3 6+ 2 3+ 2
ç 3 ÷ ç ÷ ç- ÷
è2 ø è2ø è 3ø
3 2 6- 3 4 3 6- 2
= ´ - ´
-1 6+ 3 6- 3 6+ 2 6- 2
1
1 - æç ö -
1
è 10 ø æ 8 ö 3 6 3- 2
= -1 3 -1
+ç ÷ + ´
æ 3ö æ 3 ö +æ 1ö è 27 ø
ç ÷ ç ÷ ç- ÷ 3+ 2 3- 2
è8ø è2ø è 3ø [Rationalising the respective denominators]
1 3 2 ( 6 - 3) 4 3 ( 6 - 2) 6( 3- 2)
1 - 10 æ 27 ö 3 = - +
= +ç ÷ 6-3 6-2 3-2
8 27
´ + ( -3 ) è 8 ø
3 8 = 2 ( 6 - 3) - 3 ( 6 - 2 )+ 6 ( 3 - 2 )
1
-9 é æ 3 ö 3 ù 3 9 3 3 3 = 12 - 6 - 18 + 6 + 18 - 12 = 0
= + êç ÷ ú = - + = - + = 0
9 - 3 ëè 2 ø û 6 2 2 2 2( 2 + 6)
7. (d); Let x =
( 81 ) 4x ´ ( 27 ) x ´ 9 7 3 2+ 3
4. (d); = 39
( 729 )x + 2
é2( 2 + 6 )ù
2

Squaring both sides, x 2 = ê ú


( 34 ) 4x ´ ( 33 ) x ´ ( 32 )7 ëê 3 2 + 3 úû
Þ = 39
(3 )6 x+ 2

4( 2 + 6 ) 4( 2 + 6 )
2 2

x2 = Þ x2 =
316 x ´ 33x ´ 314 9 (2 + 3 )
(3 )
2
Þ 6x + 12
= 39 2+ 3
3

316 x +3x + 14 4(2 + 6 + 2 ´ 2 ´ 6 )


Þ = 39 Þ x2 =
36x + 12 319x + 14 - 6x- 12 = 39 9 (2 + 3 )

56 @BEST300MCQ For More Study Material


Visit: studyiq.com
4 ( 8 + 2 12 ) 4 ( 8 + 2 22 ´ 3 )
1

= éë( 5 )2 + ( 3 ) + 2 ´ 5 ´ 3 ùû 2
2
x2 = Þ x2 =
9(2 + 3 ) 9(2 + 3 )
1

4 (8 + 4 3 ) 16 ( 2 + 3 ) 16 = éë( 5 + 3 ) ùû 2 = 5 + 3
2

x2 = Þ x2 = =
9 (2 + 3 ) 9 (2 + 3 ) 9
1 1
4 Second term = ( 7 - 4 3 )- 2 = ( 7 - 2 ´ 2 ´ 3 )- 2
x=
3 1
= (4 + 3 - 2´ 2 ´ 3)
-
2
5 -2
8. (c); x = 1
5+2 -
= éë( 2 ) 2 + ( 3 ) - 2 ´ 2 ´ 3 ùû
2 2

( 5 - 2)
2
5+4-4 5
= = =9-4 5 -
1 -1
= (2 - 3 )

( 5 + 2 )( 5 - 2 ) = éë( 2 - 3 ) ùû
2
5-4 2 2

1 1 1
= (2 - 3 ) =
-1
\ = 9 + 4 5 , x 4 + x -4 = x 4 + 4
x x (2 - 3)

1´ (2 + 3 )
2
æ 1 ö
2
é 2
ù
= ç x 2 + 2 ÷ - 2 = êçæ x + ÷ö - 2 ú - 2
1 = =
(2 +3)
=2+ 3
è x ø ëè xø û (2 - 3 )( 2 + 3 ) 4-3
2 \ Expression
= ëé( 9 + 4 5 + 9 - 4 5 ) - 2 ûù - 2
2

= (5 + 3 ) - (2 + 3 ) = 5 + 3 - 2 - 3 = 3
= [( 18 ) - 2] - 2 = ( 322 ) - 2 = 103682 (integer)
2 2 2

11. (c); Expression = 6 - 4 3 + 16 -


19 + 8 3
9. (b); Expression = = 6 - 4 3 + 16 - 2 ´ 2 3 ´ 2
7-4 3

19 + 2 ´ 4 ´ 3 16 + 3 + 2 ´ 4 ´ 3 = 6 - 4 3 + 12 + 4 - 2 ´ 2 3 ´ 2
= =
7 - 2 ´2´ 3 4 +3 -2´2´ 3
(2 3 )
2
= 6-4 3 + + ( 2 )2 - 2 ´ 2 3 ´ 2
( 4 )2 + ( 3 ) + 2 ´ 4 ´ 3 (4 + 3)
2 2

= =
(2 3 - 2)
2
( 2 )2 + ( 3 ) - 2 ´ 2 ´ 3 (2 - = 6-4 3 +
3)
2 2

éëQ ( a ± b ) 2 = a2 + b 2 ± 2ab ùû = -2

4+ 3 = - = + - ´ 3´
=
2- 3
( 3 ) + (1) - ´ 3 ´
2
= 2

Rationalising the denominator.

=
(4 + 3) (2 +
´
3)
=
8+2 3 +4 3+3 = ( 3 - 1) = 3 - 1
2

(2 - 3 (2 +
) 3) 4-3
5+2 6
= 11 + 6 3 12. (b); x = Rationalising,
5-2 6
1
10. (c); First term = ( 28 + 10 3 ) 2 5+ 2 6 5+ 2 6
x= ´
1 5-2 6 5+2 6
= ( 28 + 2 ´ 5 ´ 3 ) 2
(5 + 2 6)
2
1
= ( 25 + 3 + 2 ´ 5 ´ 3 ) 2 = = 5+2 6
25 - 24

57 @BEST300MCQ For More Study Material


Visit: studyiq.com
\ x2(x – 10)2 1
1 4
( 0.3 ) æç ö÷ ( 9 ) 6 ( 0.81 ) 3
1 1
2
= ( 5 + 2 6 ) ( 5 + 2 6 - 10 )
2 2
3

16. (b); Expression = è 27 ø


2 1 1
= ( 5 + 2 6 ) ( 2 6 - 5)
2 2
( 0.9 ) 3 ( 3 ) - 2 ( 243 )- 4

= ( 25 + 24 + 20 6 )( 24 + 25 - 20 6 ) 1 1 2 1 1
( 0.3 ) 3 ( 9 ) 6 ( 0.81) 3 ´ ( 3 ) 2 ( 243 ) 4
=
= ( 49 + 20 6 )( 49 - 20 6 ) 2 1
( 0.9 ) 3 ´ ( 27 ) 4
= ( 49 ) 2 - ( 20 6 ) = 2401 - 2400 = 1
2

1 1 2 1 1
( 0.3 ) 3 ( 32 ) 6 ( 0.9 )2´ 3 ´ ( 3 ) 2 ( 35 ) 4
=
- 7 -2 7 2 1

13. (a); = ( 0.9 ) 3 ´ ( 33 ) 4


8+3 7 -2 2 +6 7 -4
1 1 5
(Multiplying numerator and denominator by 2 ) 1 4
( 0.3 ) 3 3 3 ´ ( 0.9 ) 3 ´ 3 2 ´ 3 4
= 3
2
(1) + ( 7 ) - 2 ´ 1 ´ 7
2
2
( 0.9 ) 3 ´ 3 4
=
( 3 )2 + ( 7 ) + 2.3. 7 - 4
2
1 1 5 3
1 4 + + -
= ( 0.3 ) 3 ´ ( 0.9 ) 3 - 3 ´ 3 3 2 4 4

{\ 8 = 1 + 7 = 1 2
+ ( 7 ) and 16 = 9 + 7 = 32 + ( 7 )
2 2
} 1 2 4 + 6 + 15 - 9
= ( 0.3 ) 3 ´ ( 0.9 ) 3 ´ 3 12

- 1)
2

= {Q a2 + b2 – 2ab = (a – b)2} 1 2 16

( = ( 0.3 ) 3 ´ ( 0.3 ´ 3 ) 3 ´ 3 12
7 + 3)
2
-4
1 2 2 4

7 -1 7 -1 = ( 0.3 ) 3 ´ ( 0.3 ) 3 ´ ( 3 ) 3 ´ ( 3 ) 3
= = =1
7 +3-4 7 -1 1 2
2 4
+ 1+2
2+ 4
= ( 0.3 ) 3 + 3 ´ 3 3 3
= ( 0.3 ) 3 ´3 3
= 0.3 ´ 32
( 625 ) ´ ( 25 )
6.25 2.6

14. (d); Expression = = 0.3 × 9 = 2.7


( 625 )6.75 ´ ( 5 )1.2
2n ´ 24 - 2 ´ 2n 1
( ) 4 6.25 ´ ( 52 )
( 5)
2.6
´(5) 4 ´ 6.25 2 ´ 2.6 17. (a); Expression = +
= 5 6.75 = 2 ´ 2n ´ 23 2
( 54 ) ´ ( 5 )1.2 ( 5 ) 4´ 6.75 ´ ( 5 ) 1.2
2 n ( 24 - 2 ) 1 16 - 2 1 7 1
= + 3 = + = + =1
2 (2´ 2 ) 2
25 + 5.2
( 5) ´ ( 5)
25
( 5)
5.2
( 5) 30.2 n 3
16 8 8 8
= = =
( 5 ) 27 ´ ( 5 )1.2 ( 5 ) 27 + 1.2 ( 5 ) 28.2 18. (b); 100.48 = x, 100.7 = y and xz = y2
= (5)30.2 – 28.2 = (5)2 = 25 \ (100.48)z = (100.7)2
2+ 3 Þ 100.48z = 101.4 Þ 0.48z = 1.4
15. (a); Expression =
2- 3 1.4 140 35 11
Þ z= = = =2
0.48 48 12 12
=
(2 + 3) (2 +
´
3)
(2 - 3 ) (2 + 3) -
9
é 5
ù
( )
2 - 10
-
19. (a); Expression = ( 27 ) 3 + ê -2 3
éë Multiplying N r and D r by ( 2 + 3 )ùû ë 23 û
9

(2 )
2 -2 -5 -9
(2 + 3)
2 -2 - 3´- ´ ´
2 -5
= = ( 33 ) 3 + - ´
3 3
10
=3 3
+2 3 3 10
[Q (a + b)(a – b) = a2 – b2]
( 2 )2 - ( 3 )
2

1 1 1 1 2 + 9 11
= 3 2 + 2 -1 = + = + = =
4 + 3 + 2´2 3 32 2 9 2 18 18
= =7+4 3
4-3

58 @BEST300MCQ For More Study Material


Visit: studyiq.com
(8 + 3 7 )´ 2 (4 - 7 )´2
20. (b); First term = 8+3 7 = 22. (a); First term = 4- 7 =
2 2

32 + ( 7 ) + 2 ´ 3 ´ 7 ( 7) +1-2 7
2 2
16 + 6 7 8-2 7
= = = =
2 2 2 2

( 7 - 1)
2
(3 + 7)
2
3+ 7 7 -1
= = = =
2 2 2 2

(7 + 3 5 ) ´ 2 (8 + 3 7 )´ 2
Second term = 7+3 5 = Second term = 8+3 7 =
2 2

32 + ( 7 ) + 2 ´ 3 ´ 7
2
32 + ( 5 ) + 2 ´ 3 ´ 5 +6 7
2
14 + 6 5 = =
= =
2 2 2 2

(3 + 7)
2
(3 + 5) 3+ 7
2
3+ 5 = =
= = 2
2 2 2

3+ 7 3+ 5 7 -1 3+ 7 2 7 +2
- \ Expression = + =
\ Expression = 2 2 2
2 2
2 14 + 2 2
=
3+ 7 -3- 5
=
7- 5
=
14 - 10 = = 14 + 2 = 2 ( 7 + 1)
2 2
2 2
23. (b); Denominator of expression
( 38 + 5 3 ) ´ 2 = 10 + 20 + 40 - 5 - 80
21. (b); First term = 38 + 5 3 =
2
= + 2
´ + 2
´ - 5- 4
´
76 + 10 3 75 + 1 + 2 ´ 5 3 ´ 1 = + + - 5-
= =
2 2 = ( 1 + 2 ) 10 + ( 2 - 1 - 4 ) 5

(5 3 + 1)
2
5 3 +1 = 3 10 - 3 5 = 3 ( 10 - 5 )
= =
2 2 15
\ Expression =
2 10 + 20 + 40 - 5 - 80
Second term = - 5= (3 - 5 )´
2 15 5 10 + 5
= = ´
3 ( 10 - 5 ) 10 - 5 10 + 5
( 5) + 1- 2 5
2
6-2 5
= =
2 2 5 ( 10 + 5 )
= = 10 + 5
10 - 5
( 5 - 1)
2
5 -1 = 3.162 + 2.236 = 5.398
= =
2 2 æ 2n 2 2 n ö æ n 2.3n ö
ç 2 - 3. 2 ÷ ç 3 - 2 ÷
è 2 øè 3 ø
+ 5-
\ Expression = + 24. (b); Expression = 3n n 3
2 2 ( 4 .4 - 2 2n )
34
5 3 +1+ 5 -1 5 3+ 5
3n .2 2n æç 1 - 2 ö÷çæ 1 - 2 ö÷
3 2
= =
2 2 = è 2 øè 3 ø
æ2 -1ö
6

5 3+ 5 2 5 6 + 10 3n .2 2n ç 4 ÷
= ´ = è 3 ø
2 2 2

59 @BEST300MCQ For More Study Material


Visit: studyiq.com
æ 3 öæ 2ö 1 7 = 3 ( 10 - 3 ) - 2 5 ( 6 - 5 ) + 2 ( 15 - 3 2 )
ç 1 - ÷ç 1 - ÷ ´
7 81 1
è 4 øè 9ø 4 9 = 30 - 3 - 2 30 + 2 ´ 5 + 30 - 3 ´ 2
= = = ´ = .
64 - 1 63 36 63 4
81 81 = -3 + 10 - 6 + 30 - 2 30 + 30
25. (b); Rationalising the denominator, = + ( - + ) 30 =

( 5 - 2) - ( 5 + 2)
2 2
2+ 3 2+ 3 3 2 +2 3 5 -2 5+2
= ´ - =
27. (b);
3 2 -2 3 3 2 -2 3 3 2 +2 3 5+2 5 -2 ( 5 + 2 )( 5 - 2 )
2 ´3 2 + 2 ´2 3 + 3´3 2 + 3´2 3
= é( 5 ) 2 + 2 2 - 2 ´ 5 ´ 2 ù - é( 5 ) 2 + 2 2 + 2 ´ 5 ´ 2 ù
(3 2 ) - (2 3 ) =ë û ë û
2 2

( 5 ) - (2)
2 2

´2 + ´3 + ´3 + ´
=
9´ 2 - 4´3 =
(5 + 4 - 4 5 ) - (5 + 4 + 4 5 )
5-4
3´ 2 + 2 6 + 3 6 + 2 ´3
= = 9 - 4 5 - 9 - 4 5 = -8 5
18 - 12
28. (c); Rationalising the denominator,
12 + 5 6 12 5 6 5 6
= = + = 2+ + 8+ 7
6 6 6 6 ´ - ´ +
3- 8 3+ 8 8- 7 8+ 7 7- 6
+ 3
\ =a-b 6 + 6+ 5 5+
3 2-2 3 ´ - ´ + ´
7+ 6 6- 5 6+ 5 5-2 5+2
5 6 -5
Þ 2+ = a - b 6 Þ a = 2 and b = 3+ 8 8+ 7 7+ 6
6 6 = - +
3 -( 8) ( 8) -( 7 ) ( 7 ) -( 6)
2 2 2 2 2 2

2 2 2
æ 5 ö æ 12 - 5 ö æ 7 ö 49
\ (a + b) = ç 2 - ÷ = ç ÷ =ç ÷ =
2

è 6ø è 6 ø è6ø 36 + 5 5+
- +
( 6) -( 5) ( 5 ) - (2)
2 2 2 2
26. (a); Rationalising the denominator,

7 3 2 5 3 2 + + + 6 6+ 5 5+
- - = - + - +
10 + 3 6+ 5 15 + 3 2 9-8 8-7 7 -6 6-5 5- 4

7 3 10 - 3 2 5 6- 5 = (3 + 8 ) - ( 8 + 7 ) + ( 7 + 6 ) - ( 6 + 5 )
= ´ - ´ -
10 + 3 10 - 3 6+ 5 6- 5
+( 5 + 2)
15 -
´ = + 8- 8- 7+ 7+ 6- 6- 5+ 5+
15 + 3 2 15 - 3 2 =3+2=5
7 3 ( 10 - 3 ) 2 5 ( 6 - 5) 1
-
4
= - - ( 16 ) 4 æ 1 ö -3 3
29. (d); Expression = ( 9 ) ´ ´ç ÷
( 10 ) - ( 3 ) ( 6) -( 5)
2 2 2 2
( 6 )-2 è 27 ø

3 2 ( 15 - 3 2 ) 1 4
( 24 ) 4 -

´ æç 3 ÷
1 3
= (3 ) ´ 2 -3
( 15 ) - ( 3 2 )
2 2
-2
( 2 ´ 3) è3 ø

7 3 ( 10 - 3 ) 2 5 ( 6 - 5 ) 2 4 é æ a ö- n æ b ön ù
= - - = 3 -6 ´ ´ ( 33 ) 3 êQ ç ÷ = ç ÷ ú
10 - 3 6-5 2 ´3
2 -2
ë èbø èaø û

3 2 ( 15 - 3 2 ) é n 1 ù
15 - 9 ´ 2 = 3–6 × 2 × 34 × 22 × 32 êëQ a = a- n úû

60 @BEST300MCQ For More Study Material


Visit: studyiq.com
= 3–6 × 34 × 32 × 22 × 2
x 2 = 7 7 7 7.................¥
= 3–6+6 × 22+1 éëQ a m .a n = a m + n ùû
Þ x2 = 7x Þ x2 – 7x = 0
=3 ×2 =1×8=8
0 3
Þ x(x – 7) = 0 Þ x = 0 or, x = 7
30. (a); Let x = 7 7 7................¥ But x ¹ 0
\ Required answer = 7
Squaring both sides,

3 x+ 8 2 3+ 3+ 2
1. (b); Given x = Þ =
2 x- 8 2 3- 3- 2

1+ x 1- 1+ x 1-x 1+ 1- x (By componendo and dividendo)


´ + ´
1+ 1+x 1- 1+x 1- 1-x 1+ 1-x 3 3+ 2
=
+x -1-x 1-x +1-x 3- 2
= +
1-1-x 1-1+ x Again,

-x +1-x 1+ x -1-x x 2 2 x + 12
= - = Þ
x x 12 3+ 2 x - 12

-x +1-x- 1+ x +1+x 2 2+ 3+ 2 3+3 2


= = =
x 2 2- 3- 2 2- 3

3 3 x+ 8 x + 12 + 2 3+
2 + 1- - 1+ \ + = +
-x 2 2 x- 8 x - 12 3- 2 2- 3
= =
x 3
2 + 2 3+ + 2- 3-
= - =
3- 2 3- 2 3- 2
2- 3 2+ 3
2+ - 2 3 -2 2 2( 3 - 2 )
= 2 2 = = =2
3 3- 2 3- 2
2
2 3´ 2
3. (d); x =
3+ 2
4-2 3 4+2 3
2+ -
= 2 2
x 2 3
3 Þ =
2 3+ 2
2
x+ 2 2 3+ 3+ 2 3 3+ 2
+ 3- - 3- Þ = =
= = x- 2 2 3- 3- 2 3- 2
3 3
(By componendo and dividendo)
2 24 Similarly,
2. (b); x =
3+ 2
x 2 2
=
2 3´8 2 3´ 8 3 3+ 2
Þ x= =
3+ 2 3+ 2
x+ 3 2 2+ 3+ 2 3+3 2
x 2 3 Þ = =
Þ = x- 3 2 2- 3- 2 2- 3
8 3+ 2

61 @BEST300MCQ For More Study Material


Visit: studyiq.com
\ Expression
4( 6 - 2 ) 4( 6 - 2 )
= = = 6- 2
x+ 2 x+ 3 + 2 3+ ( 6) -( 2) 6-2
2 2
= + = +
x- 2 x- 3 3- 2 2- 3
\ Expression

=
+ 2
-
3+
=
+ 2- 3- =( 6 - 5) +( 5 - 2 ) -( 6 - 2 )
3- 2 3- 2 3- 2
= 6- 5+ 5- 2- 6+ 2 =0
2( 3 - 2 )
= =2 ( 2.4 )6 + 9 ( 5.76 ) + 6 ( 2.4 ) 4
3- 2 6. (c); First part =
( 2.4 ) 4 + 6 ( 5.76 ) + 9
3
4. (c); x = ( 2.4 )6 + 9 ( 2.4 ) 2 + 6 ( 2.4 )4
2 =
( 2.4 ) 4 + 6 ( 2.4 ) 2 + 9
3
\ 1+ x = 1+
2 ( 2.4 )2 ëé( 2.4 )4 + 9 + 6 ( 2.4 )2 ûù
= = ( 2.4 )2 = 2.4
( 2.4 )4 + 6 ( 2.4 )2 + 9
+ 1)
2
2+ 3 4+2 3 3 +1
= = = = 1 1
2 4 4 2 1
éë( 3-2 ) -5 ùû 5 + éë( 4 -3 )-6 ùû 6 - ( 3-4 )
-
2

3 -1 Second part = 1

\ -x = éë( 2 -3 ) -4 ùû 4
2
\ 1+ x + 1- æ 1ö
( 3 )( -2 )´( -5)´çè 5 ÷ø + ( 4 )( -3 )´( -6 )´ 6 - ( 3 )( -4)´çè - 2 ÷ø
1 æ 1ö

= æ 1ö
3 +1 3 -1 3 +1+ 3 -1 ( 2 )( -3)´( -4)´èç 4 ø÷
= + = = 3
2 2 2
éQ é( x )y ù z = a xyz ù
ë ëa û û
5. (a); First part = 11 - 2 30 = 11 - 2 6 ´ 5
32 + 43 - 32 9 + 64 - 9 64
= - ´ 6´ = + - ´ ´ = = = =8
23 8 8
\ Expression = 2.4 + 8 = 10.4
( 6) + ( 5) - 2´ 6 ´ 5
2 2
=
a 2 + b2 - ab b2 + c 2 - bc c 2 + a 2 - ca
æ xa ö æ xb ö æ xc ö
7. (d); ç - b ÷ ´ ç -c ÷ ´ ç -a ÷
( 6 - 5) = 6 - 5
2
= èx ø èx ø èx ø
a 2 + b2 - ab b 2 + c2 - bc c2 + a2 - ca
Second part = 7- = ( xa+b ) ´ ( xb+c ) ´ ( xc+a )

= x( a + b ) ( a + b2 -ab )
´ x( b + c ) ( b + c 2 - bc )
´ x( c + a ) ( c + a 2 - ca)
2 2 2

= - ´2 = ´ 2
3
+ b3 3
+ c3 3
+ a3
= xa ´ xb ´ xc
= ´ 2
3
+ b 3 + b 3 + c 3 + c3 + a 3 2 ( a3 + b 3 + c 3 )
= xa =x
( 5) +( 2) - 2´ 5 ´ 2
2 2
= 8. (b); It is given 2x = 4y = 8z = k (Let)
1 1 1
( 5 - 2) = 5 - 2
2
= Þ 2 = k x , 4 = k , 8 = kz y

4 Now, 4 × 8 = 32 = 25
Third part =
6+ 2 1

(k )
1 5 1 1
1 5
Þ ky ´kz = x Þ k y+z = k x
Rationalising the denominator,

=
4 ( ´
6 - 2) 1 1 5
+ =
Þ
( 6+ 2) ( 6 - 2) y z x

62 @BEST300MCQ For More Study Material


Visit: studyiq.com
1 1 1 ( 7 - 2) = 7 - 2
2

Now, + + =4 =
2x 4y 4z
8
1 1æ 1 1ö 1 1æ 5ö Third term =
Þ + ç + ÷=4 Þ + ç ÷=4 + 10 8 - 33 - 10 8
2x 4 è y z ø 2x 4 è x ø

1 5 2+5 8
Þ + =4 Þ =4 =
( 5 ) 2 + ( 8 ) + 2 ´ 5 ´ 8 - ( 5 )2 + ( 8 ) - 2 ´ 5 ´ 8
2 2
2x 4x 4x

7 7
Þ = 4 Þ 16x = 7 Þ x = 8
4x 16 =
(5 + 8) - (5 - 8)
2 2

5 +2 + 5 -2
9. (c); Let x =
5 +1 8 8 1
= = =
On squaring both sides. 5+ 8 - 5+ 8 2 8 2

é
2
1 1
+2 + 5 -2 ù \ Expression = 2 + 7 - 7 + 2 + =4
Þ x =ê ú
2
2 2
êë 5 +1 úû
11. (b); Expression
2
é 26 - 15 3 10 + 18
+2 + 5 - 2 ùû = +

(3 - 5)
2
2 é5 2 - + 5 3 ùû 8+
é + 1 ùû ë
ë

5 +2+ 5-2+2 5+2× 5 -2 Now, 38 + 5 3 = ( 38 + 5 3 ) ´ 2


= 2
5 +1
76 + 2 ( 5 3 ) 75 + 1 + 2 ( 5 3 )
( 5) -( ) = =
2 2
+ 2 2
=
5 +1
(5 3 ) + ( 1) + 2 ( 5 3 ) ´ 1
2 2

2 5+2 2( 5 + 1) =
= = =2 \ x= 2 2
5 +1 5 +1

(5 3 + 1)
2
Again, 3 - 2 2 = 2 + 1- 2´ 2 ´1 5 3 +1
= =
2 2
( 2 ) + ( 1) - 2 ´ 2 ´ 1
2 2
= 2

(5 ) æ 5 3 +1ö
2

\ 2- + =ç - ÷
( 2 - 1) = 2 - 1
2
= è 2 ø

(9 - 5 3 )
2 2
+2 + -2 æ 10 - 5 3 - 1 ö
\ - -2 2 =ç ÷ =
5 +1 è 2 ø 2

= 2 - ( 2 - 1) = 1 ( 2
and - 5) = (3 - 5)´
2
10. (d); First term = 11 + 4 7
6-2 5 5 + 1 - 2 ( 5 )´ 1
= 22 + ( 7 ) + 2 ´ 2 ´ 7 = (2 + 7) =2+ 7 = =
2 2

2 2
Second term
( 5 ) + ( 1) - 2 5 ( 5 - 1)
2 2 2
5 -1
= 11 - 4 7 = ( 2 ) + ( 7 ) - 2 ´ 2 ´ 7
2
2
= = =
2 2 2

63 @BEST300MCQ For More Study Material


Visit: studyiq.com
2( 5+ )
1 1
- \ Expression = ( 28 - 10 3 ) 2 - ( 7 + 4 3 ) - 2
2
+
\ æ5 2 - 5 3 + 1 ö 5 -1
ç ÷ 8+
2 7
è 2 ø +
16 + 6 7 - 16 - 6 7
( - ) 2 2( + )
= + 1
(9 - 5 3 ) = 5 - 3 - (2 - 3 ) +
2
4+ 5 -1
2

52 - 30 3 2 (3 + 5 ) 1 1 1
= + = - 3- + 3+ = + =
156 - 90 3 (3 + 5) 2 2 2

52 - 30 3 1 1 4 3 30 18
= +2 = +2 =2 13. (d); - -
3 ( 52 - 30 3 ) 3 3 2- 2 4 3 - 18 3-2 3
Rationalising the denominators by correspond-
1
12. (c); First term = ( 28 - 10 3 ) 2 ing conjugates

1 4 3 (2 + 2 )
1
=
= ( 25 + 3 - 10 3 ) = éë( 5 ) + ( 3 ) - 2 ´ 5 ´ 3 ùû
2
(2 - 2 )( 2 + 2 )
2 2 2

30 ( 4 3 + 18 ) 18 ( 3 + 2 3 )
1

= éë( 5 - 3 ) ùû = ( 5 - 3 ) 2 = ( 5 - 3 )
2 2 2´
- -
1 1
(4 3 - 18 ) ( 4 3 + 18 ) (3 - 2 3 )( 3 + 2 3 )
Second term = ( 7 + 4 3 ) -
2
= (4 + 3 + 4 3 )
-
2

( + ) ( + 18 ) ( + )
1 = - -
( 2) - ( 2 ) (4 3 ) - ( 18 ) ( 3) - ( 2 3 )
- 2 2 2 2
= éë( 2 ) + ( 3 ) + 2 ´ 2 ´ 3 ùû
2 2 2 2 2

1 1
[(a – b) (a + b) = a2 – b2]
-
= éë( + 3 ) ùû =( + )
2 2 2 ´-
2

1 ( + ) ( + 18 ) +( )
= (2 + 3 )
-1
= = - -
(2 + 4-2 48 - 18 9 - 12
3)

1 2- 3 2- 3
= ´ = = 2- 3 ( + ) ( + 18 ) ( + )
2+ 3 2- 3 4-3 = - -
2 30 -3
7
Third term = = ( + 2)-( + 18 ) + ( + )
16 + 6 7 - 16 - 6 7
= 4 3 +2 6 -4 3 -3 2 +3 2 +2 6
7
= = 2 6 +2 6 = 4 6
9 + 7 + 2´ 3´ 7 - 9 + 7 - 2´ 3´ 7
( 12.12 ) 2 - ( 8.12 )2
7 14. (c); First term =
= ( 0.25 )2 + ( 0.25 )( 19.99 )
(3 + 7) - (3 - 7)
2 2

( 12.12 + 8.12 )( 12.12 - 8.12 )


7 7 =
= = 0.25 ( 0.25 + 19.99 )
(3 + 7 ) - (3 - 7 ) 3+ 7 -3+ 7
éëQ ( a 2 - b 2 ) = ( a + b ) ( a - b ) ùû
(Taking square root)

7 1 20.24 ´ 4 4 4 ´ 100
= = = = =
2 7 2 0.25 ´ 20.24 0.25 25

64 @BEST300MCQ For More Study Material


Visit: studyiq.com
= 20 × 50 = 1 × 1 = 1
400
= = 16 = 4
( 10 3 ) ¸ ( 103 ) 102´ 3 ¸ 109
2
25 2

Second term = = 2 ´3
( 102 ) ¸ 10 2 10 ¸ 108
3 3
8
é -3 5
ù 15
Second term =
ê 84
ë ( ) 2 3
ú ´ 16 4
û 106 ¸ 10 9 10(6 - 9 )
1 = =
é 3
ù
-
5 106 ¸ 108 10(6 - 8 )
êë{( 128 ) } 7 úû
3
-5

10-3 1
8
= -2
= 10( -3 + 2) = 10 -1 =
3 10 10
é ù 5

{ } ( 4 )4
15

ú ´ 2
3 2
ê ( 2 3 )- 4 \ Expression
= ë û éëQ 8 = 2 3 ,16 = 2 4 , 128 = 27 ùû
1 1
-
é 7 -5 73 ù 5 2 é( 625 ) 5 ´ ( 1024 ) - 5 ¸ ( 25 ) 5 ù 2 ( 10 3 ) 2 ¸ ( 10 32 )
êë{( 2 ) } úû
3 6 3 6 3

= ë û +
( 102 ) 3 ¸ 10 2
5 1 3

( 3 128 ) 2 ´ ( 125 ) 5
-
-3 5 8 3
3´ ´ ´ 4´
2 4 2 15
´2 4
= 3 æ 1ö 1
7 ´( -5 )´ ´ ç - ÷´ 1 10 + 1
2 7 è 5ø 3 = 1+ = = 1.1
10 10
éQ é m n p ù q = a mnpq and 3 x = ( ) 13 ù
ëê ë{( a ) } û
1
x ú 16. (a); First term = ( 18 - 8 2 ) 2
û

2 -3 ´ 2 3 2 -3 + 3 2 0 1 1
= = = =
{ }
1
= ( 16 + 2 - 8 2 ) 2 = ( 4 )2 + ( 2 ) - 2 ´ 4 ´ 2
2 2
2 2 2 2
\ Expression = First term + second term
1
= (4 - 2 )

1 1 2 = 4- 2
= 4+ = 4
2 2 1 1
Second term = ( 6 - 4 2 )- 2 = ( 4 + 2 - 4 2 )- 2
1

2 é( 625 ) 5 ´ ( 1024 )- 5 ¸ ( 25 ) 5 ù 2
6 3 6 3
ë û 1

{ }
-
= 22 + ( 2 ) - 2 ´ 2 ´ 2
2
15. (d); First term = 5 1
2

( 3 128 )
-
2 ´ ( 125 ) 5

-1
1
= (2 - 2 ) = (2 - 2 ) =
2´ -1
1 2
1
2 é( 5 4 ) 5 ´ ( 2 10 )- 5 ¸ ( 52 ) 5 ù
6
3 6 3 2 2- 2
= ë û
5
2+ 2 2+ 2
{( 128 ) } 1
- 1
´ ( 53 ) 5
1
3
2
= ´ =
2- 2 2+ 2 2

1 1
3
2 6 é 4´ 5 2´ ù 2
3 -6 3
ë
10´
5 ´2 5 ¸5 5 û Third term =
= 19 + 8 3 - 19 - 8 3
5

{( 2 ) }
1 -
1 3´
7 3
2
´5 5
3
=
16 + 3 + 8 3 - 16 + 3 - 8 3
1 1 1 1

2 é ù 2 é ù
6 12 6 2 6 6 2
ë 5 ´ 2 -12 ¸ 5 û =
5
ë5 ´ 2 -12 û 5 5 3
= æ 1 -5 ö -35 3
=
( 4 ) + ( 3 ) + 2 ´ 4 ´ 3 - ( 4 )2 + ( 3 ) - 2 ´ 4 ´ 3
3 2 2 2
ç7´ ´ ÷
2è 3 2 ø ´ 5 5 2 6 ´ 55

1 3 3
2 6 ´ 5 5 ´ 2 -6
æ1 35 ö
ç -6 + ÷
3 3
- =
= =2 ´5 (4 + 3) - (4 - 3)
è6 6 ø 5 5 2 2
- 35 3
2 6
´5 5

3 3 1
æ 1 - 36 + 35 ö
ç ÷
36 - 36 = = =
=2 è 6 ø
´5 = 2
0 6
´5 0
4+ 3 -4+ 3 2 3 2

65 @BEST300MCQ For More Study Material


Visit: studyiq.com
+ 2 ) 6) ( 5 + 6)
2 2 2
´6
\ Expression = 4 - 2 + +
2 2 [a2 + b2 + 2ab = (a + b)2]
8 - 2 2 + 2 + 2 + 1 11 - 2 = ( 5 + 6)
= =
2 2
3
17. (d); First term = 7 + 4 3 = 2 2 + ( 3 ) + 2 ´ 2 ´ 3
2 Second term =
- 2 10

(2 + 3) = 2+ 3 9 ´ ( 7 + 2 10 )
2
= 9
= =
Second term (7 - 2 10 ) (7 - 2 10 )( 7 + 2 10 )

3)
2
= + 10 3 = ( 5 )2 ´ 3 9 ´ ( 7 + 2 10 ) 9 ´ ( 7 + 2 10 )
= =
49 - 40 9
(5 + 3) = 5+ 3
2
=
= 7 + 2 10 = 5 + 2 + 2 10
11
( 5) + ( 2 ) + 2 5 ´ 2
2 2
Third term = =
20 + 6 11 + - 6 11

( 5 + 2) = ( 5 + 2)
2
11 =
=
32 + ( 11 ) + 2 ´ 3 ´ 11 + 32 + ( 11 ) - 2 ´ 3 ´ 11
2 2
4
Similarly, third term =
+4 3
11
=
( 16 ´ ( 8 - 4 3 )
11 + 3 ) + ( 11 - 3 )
2 2
16
= =
(8 + 4 3 ) ( 8 + 4 3 )( 8 - 4 3 )
11 11 1
= = =
11 + 3 + 11 - 3 2 11 2 16 ´ ( 8 - 4 3 ) 16 ( 8 - 4 3 )
= =
64 - 48 16
1
Expression = 2 + 3 - 5 - 3 +
(8 - 4 3) = ( 6) +( 2 ) - 2´ 6 ´ 2
2 2
2 =
1 -6 + 1 -5 1
= -3 + = = = -2 = ( 6 - 2) = ( 6 - 2)
2
2 2 2 2
\ Expression
1
18. (a); First term = 1 3 4
11 - 2 30 = - -
- 2 30 - 2 10 +4 3
1 1 ´ ( 11 + 2 30 )
= = = ( 5 + 6)-( 5 + 2)-( 6 - 2)
11 - 2 30 ( 11 - 2 30 )( 11 + 2 30 )
= 5+ 6- 5- 2- 6+ 2 =0
(Rationalising the denominator)
+x+ -x
( 11 + 2 30 ) 19. (b); Expression =
1+x - 1-x
=
( 11 ) 2 - ( 2 30 )
2
On rationalising the denominator,
[Q (a + b) (a – b) = a2 – b2] +x+ -x +x+ -x
= ´
11 + 2 30 ( 11 + 2 30 ) 1+ x - 1-x 1+ x + 1-x
= =
121 - 120 1
( -x)
2
+x +
=
( + x) -( -x)
2 2
= + 2 30 ) = 5 + 6 + 2 ´ 5 ´ 6

66 @BEST300MCQ For More Study Material


Visit: studyiq.com
+x -x 2
+ 2b + - 2b + 2b + - 2b
= = = ´
(1 + x) - ( 1 - x) x a + 2b - a - 2b a + 2b + a - 2b
3
( - 2b )
2
Putting x = + 2b +
2 =
( + 2b ) - ( - 2b )
2 2
2
æ ö
1+ 1-ç 3 ÷
è 2 ø
\ Expression = a + 2b + a - 2b + 2 a + 2b ´ - 2b
3 =
( a + 2b ) - ( a - 2b )
2
3 4-3 \ 2bx = a + a 2 - 4b2
1+ 1- 1+
= 4 = 4
Þ 2bx - a = a 2 - 4b 2
3 3
2 2
Þ ( 2bx - a )2 = ( - 4b 2 )
2
2
1 3
1+ (Squaring both sides)
= 2 = 2 = 3´ 2 = 3
3 3 2 3 Þ 4b2x2 + a2 – 4abx = a2 – 4b2
2 2 Þ 4b2x2 – 4abx + 4b2 = 0
Þ 4b(bx2 – ax + b) = 0
a + 2b + a - 2b
20. (a); x = Þ bx2 – ax + b = 0
a + 2b - a - 2b
(Dividing both sides by 4b)

3 -6 -
æ3ö æ3ö æ3ö 8 + 2 15
1. (c); ç ÷ ç ÷ =ç ÷ = = 4 + 15
è5ø è5ø è5ø 2
- - 2x - 1
æ3ö æ 3ö æ3ö æ3ö 5- 3
Þç ÷ ç ÷ ç ÷ =ç ÷ \ y= = 4 - 15
è5ø è 5ø è5ø è5ø 5+ 3

æ3ö æ3ö æ3ö


- 2 x-1
\ x + y = 4 + 15 + 4 - 15 = 8
Þ ç ÷ ç ÷ =ç ÷
è5ø è5ø è5ø
4. (b); 2 = 1.414 ( Given )
Þ 2x – 1 = – 3 Þ 2x = – 3 + 1 Þ x = – 1

7 -2 7 -2 7 -2
2 -1
=
( - )( 2- )
Now,
2. (b);
7 +2
=
7 +2
´
7 -2
2 +1 ( 2 + 1)( 2 - 1)
(Rationalising the denominator)
( 2 - 1)
2

= ( 2 - 1)
2
=
( 7 - 2)
2
7 + 4 - 4 7 11 4 7 2-1
= = = -
7-4 3 3 3
-1
- 1) = 2 - 1
2
\ =
7 -2 11 4 2 +1
\ =a 7 +b Þ - 7 =a 7 +b
7 +2 3 3
= 1.414 – 1 = 0.414
4 11 1 1
Clearly, a=- and b = 5. (b); x + x + a 2 = ( x ) + 2. x. + ( a )2
2
3 3 4 8

5+ 3 5+ 3 5+ 3 1
3. (a); x = = ´ Clearly a = .
5- 3 5- 3 5+ 3 8
2
æ 1ö
( 5 + 3)
2
5 + 3 + 2 15 Then, expression = ç x + ÷
= = è 8ø
5-3 2
67 @BEST300MCQ For More Study Material
Visit: studyiq.com
6. (c); ( 0.5 ) 2 = 0.49 = 11. (c); Let x = 7 7 7 7.....
3
0.008 = 0.2, 0.23 On squaring both sides,
x2 = 7x Þ x2 – 7x = 0
\ 0.49 > ( 0.5 ) 2 > 0.23 > 3 0.008
Þ x(x – 7) = 0 Þ x = 7
7. (a); 3
4 , 2 , 3, 5 6 4
\ 7 = (73)y – 1 = 73y – 3
LCM of 3, 2, 6, 4 = 12 Þ 3y – 3 = 1 Þ 3y = 4

1 4 1 1 4
3
4 = ( 4 ) 3 = ( 4 ) 12 = ( 4 4 ) 12 = ( 256 ) 12 Þ y=
3
1 6 1 1
2 = ( 2 ) 2 = ( 2 ) 12 = ( 2 6 ) 12 = ( 64 ) 12 1
12. (b); Expression = 2 1
1 2 1 1 2 + 23 + 1
3
6
3 = ( 3) = ( 3)6 12 = ( 32 ) 12 = (9) 12

1 1

4
5 = ( 5) = ( 5)
1 3
= (5 )
1
= ( 125 )
1
23 - 1 23 - 1
4 12 3 12 12 = =
1 1 1 1
(2 1
3 )( 2
- 1 23 + 23 + 1
1
) (2 )
1
3
3
-1
\ ( 256 ) 12 > ( 125 ) 12 > ( 64 ) 12 > ( 9 ) 12
1
or, 3
4>45> 2>63 = 23 - 1 = 3 2 - 1
[Q (a – b) (a2 + ab + b2) = a3 – b3]
3+ 2 3- 2
8. (b); Expression = -
3- 2 3+ 2 1 1 2 -1
13. (d); = ´
2+ 1 2+ 1 2 -1
( 3 + 2) -( 3 - 2)
2 2

= = 2 -1
( 3 + 2 )( 3 - 2 )
\ Expression
3+ 2+2 6 -3-2+ 2 6 4 6 = - + - + - 3+
= = =4 6
( 3) -( 2) 3-2
2 2
+ 99 - 98 + 100 - 99
9. (c); 12 + 18 = 3 ´ 2 ´ 2 + 2 ´ 3 ´ 3 = 100 - 1 = 10 - 1 = 9

= 2 3 +3 2 14. (a); 6 ´ 15 = x 10
\ Required difference Þ 3´ 5
=2 3 +3 2 -2 3 -2 2 = 2 Þ 2 ´ 5 ´ 3 = x 10
10. (c); LCM of the orders of the surds = LCM of 2,3,5
and 7 = 210 Þ 3 10 = x 10 Þ x=3

1 105 1 1 70 1 ( 7 - 5 )( 7 + 5 ) 2
5 2 = 5 210 = ( 5105 ) 210 , 4 3 = 4 210 = ( 470 ) 210 15. (b); 7- 5= =
7+ 5 7+ 5
1 42 1 30
1 1
Similarly,
2 5 = 2 210 = ( 2 42 ) 210 , 3 7 = 3 210 = ( 330 ) 210
2 2
1 5- 3= , 9- 7 =
\ The largest number = 5 2 = 5 5+ 3 9+ 7

Quicker Approach 2
11 - 9 =
5 is the largest radicand and its order is smallest. 11 + 9

\ Largest number = 5 \ Largest number = 5 - 3 because its


denominator is the smallest.

68 @BEST300MCQ For More Study Material


Visit: studyiq.com
16. (c); (0.1)2 = 0.01 21. (d); Expression
0.0121 0.11 ´ 0.11 0.11 3+ 6
=
0.0004 = 0.02 , So largest number = 0.12 5 3 - 2 12 - 32 + 50

17. (d); Let x = 6 + 6 + 6 + ..... 3+ 6


=
5 3 -4 3 -4 2 +5 2
On squaring both sides,
3+ 6 3( 3 + 2)
x2 = 6 + 6 + + ..... = = = 3 = 1.732
3+ 2 3+ 2
Þ x2 = 6 + x
Þ x2 – x – 6 = 0 22. (d); Expression = 10 +
Þ x2 – 3x + 2x – 6 = 0
Þ x(x – 3) + 2(x – 3) = 0 = ´ 5
Þ (x + 2) (x – 3) = 0
= 2 + 3 + 5 + 2´ 2 ´ 3 + 2´ 2 ´ 5 + 2´ 3 ´ 5
Þ x = 3 and x ¹ – 2 because numbers are positive.
( 2) +( 3) +( 5) + 2´ 2 ´ 3 + 2´ 2 ´ 5 +2´ 3´ 5
2 2 2
=
( 81 )3.6 ´ ( 9 ) 2.7
18. (c); Expression =
( 81 )4.2 ´ 3
( 2 + 3 + 5) = 2 + 3 + 5
2
=

( 4 ) ´ ( 32 )
3.6 2.7
314.4 ´ 35.4 [(a + b + c)2 = a2 + b2 + c2 + 2ab + 2ac + 2bc]
= 3 =
( 34 )4.2 ´ 3 316.8 ´ 3
36 - 24 + 24 - 16
[Q (am)n = amn; am × an = am+n; am ÷ an = am–n] 23. (c);
5 + 24
14.4 + 5.4 19.8
3 3
= 16.8 + 1
= 17.8 = 319.8 - 17.8 = 32 = 9 6-4 2 2
3 3 = = =
5 + 24 5 + 24 5 + 6´ 4
3- 2 3- 2 3- 2
19. (b); a = = ´
2 2 5-2 6
3+ 2 3+ 2 3- 2 = = ´
5+ 2 6 5+ 2 6 5-2 6
( 3 - 2)
2

= = 3+ 2 -2 6 = 5-2 6
3-2 2 (5 - 2 6 )
= = 2(5 - 2 6 )
25 - 24
3+ 2
\ b= = 5+2 6
= 2 éë( 3 ) + ( 2 ) - 2 3 2 ùû
2 2
3- 2
Þ a + b = 10;
= 2( 3 - 2 ) = 2 ( 3 - 2 ) = 6 - 2
2

ab = ( 5 - 2 6 )( 5 + 2 6 ) = 25 - 24 = 1

a 2 b 2 a 3 + b 3 ( a + b ) 3 - 3ab ( a + b ) 2 + 3 = ( 2 + 3 )( 2 + 3 )
\ + = = 24. (b); a =
b a ab ab 2- 3 ( 2 - 3 )( 2 + 3 )
= 103 – 3 × 10 = 1000 – 30 = 970
4+4 3 +3
= =7+4 3
5+ 3 5- 3 4-3
20. (d); 2 x = -
5- 3 5+ 3
2- 3
\ b= =7-4 3
( 5 + 3) -( 5 - 3) 2+ 3
2 2

= 4. 5. 3
= = 2 15
( 5 - 3 )( 5 + 3 ) 5-3 \ a + b = 7 + 4 3 + 7 - 4 3 = 14

\ 2 x = 2 15 Þ x = 15 ab = ( 7 + 4 3 )( 7 - 4 3 )

69 @BEST300MCQ For More Study Material


Visit: studyiq.com
= 49 – 48 = 1 28. (d); (0.9)2 = 0.81
\ a2 + b2 + ab = (a + b)2 – ab
9
= (14)2 – 1 = 196 – 1 = 195 0.9 = 0.948, 0.9 = 9 = 1

7 \ 0.9 is greatest.
25. (a); Expression =
16 + 6 7 - 16 - 6 7
29. (b); Expression = 3
0.004096
7 In this type of question the decimal digit is
=
converted into decimal fraction.
9 + 7 + 2´ 3´ 7 - 9 + 7 - 2´ 3´ 7
4096
7 7 1 = 3
= = = 1000000
(3 + 7 ) - (3 - 7 ) 3 + 7 - 3 + 7 2
We know that, 3
4096 = 16
26. (c); Required value will be the difference between
( 12 + 18) and ( 3 + 2 ) and 3
1000000 = 100

= ( 12 + 18) – ( 3 + 2 ) 16
\
100
= (2 3 + 3 2 ) – ( 3 + 2 )
We know that, 16 = 4 and 100 = 10
= 3+2 2
16 4
–( 4 -3/2
æ 1 ö
– ç 3/2 ÷ \ = = 0.4
27. (d); Given, (256) = (256) è4 ø 100 10
1
æ 1 ö
– ç 2 3/2 ÷ 30. (b); (2.89)0.5 = (2.89) 2 = (1.72)1/2 = 1.7
= (256) è(2 ) ø

Þ 2 – (0.5)2 = 2 – 0.25 = 1.75,


1 1
1 1 1 Þ

p–n = p n = (256) 8 = = = 3 = 1.732
1 1
8 8 2
(256) 8
(2 ) and 3
0.008 = 3 0.2 ´ 0.2 ´ 0.2 = 0.2
Arrange in ascending order
3
0.008 < (2.89)0.5 < 3 < 2 – (0.5)2

70 @BEST300MCQ For More Study Material


Visit: studyiq.com
Chapter
Ratio, Proportion and
4 Partnership
The comparision between two quantities in terms of magnitude is called ratio.
For example, Mohit has 5 pens and Amit has 3 pens. It means the ratio of number of pens between Mohit and Amit is
5 is to 3. It can be expressed is 5 : 3.
a
So the ratio of any two quantities is expressed as or a : b. The numerator ‘a’ is called the antecedent and denominator
b
‘b’ is called as consequent.
Rule of Ratio: The comparison of two quantities is meaningless if they are not of the same kind or in the same units (of
length, volume or currency etc.) We do not compare 5 girls and 7 toys or 15 kilometers and 3 cows. Therefore, to find the
ratio of two quantities (of the same kind), it is necessary to explain them in same units.

Properties of Ratio:
1. The nature of ratio does not change when the numerator and denominator both are multiplied by same
quantities.

a ka la 3 6 9
i.e, = = etc Þ e.g. = = ... etc have the same ratio.
b kb lb 4 8 12
2. The value of a ratio does not change when the numerator and denominator both are divided by same quantities.
a a /k a /l 3 3 /2 3 / 3
i.e, = = etc, Þ e.g. = = . etc are in same ratio
b b /k b /l 4 4 /2 4 /3
3. The ratio of two fractions can be expressed in ratio of integers.

3/4 3 4 3 a /b a d
e.g. = ´ = Þ = ´
5/4 4 5 5 c /d b c
4. When two or more than two ratio are multiplied with each other, then it is called compounded ratio e.g.,
2 4 6 16 2 4 6
´ ´ = is compounded ratio of , ,
3 5 7 35 3 5 7
a c e k
´ ´ .... = ( Compound ratio )
So,
b d f m
5. When the ratio is compounded with itself, it is called as duplicate, triplicate ratio etc.
2
a a a2 æ a ö a
´ = 2 = ç ÷ is called duplicate ratio of
b b b è bø b
3
a a a æ aö a
and ´ ´ = ç ÷ is called triplicate ratio of
b b b è bø b
1
æ aö æ aö 2
similarly çè ÷ø = çè ÷ø is called as sub-duplicate ratio and
b b
1
æ aö æ aö 3 a
3
çè ÷ø = çè ÷ø is called as sub-triplicate ratio of b .
b b
If four quantities a, b, c and d form a proportion, many other proportions may be deduced by the properties of
fraction. The results of these operation are very useful. These operations are

71 @BEST300MCQ For More Study Material


Visit: studyiq.com
a c b d a c a b
1. Inverterdo: if = the = 2. Alternado: if = then =
b d a c b d c d
a c æ a + bö æ c + dö a c æ a - bö æ c - dö
3. Componendo: if = , then ç = 4. Dividendo: if = , then ç =
b d è b ÷ø çè d ÷ø b d è b ÷ø çè d ÷ø

a c a + b c+d
5. Componendo and Dividendo: if = , then =
b d a-b c-d
Concept 1: If two numbers are in the ratio of a : b and the sum of these number is x, then these numbers will
ax bx
beand respectively.
a+b a+b
Example : Two numbers are in the ratio of 4 : 5. If sum of these two number is 810, find the numbers?
Sol. Ratio of two number = 4 : 5
Sum = 810
ax 4 ´ 810 bx 5 ´ 810
First number = = = 360 , Second number = = = 450
a+b 4+5 a+b 4+5
Example: a : b = 3 : 4 and b : c = 2 : 5 Find a : b : c ?

Sol. a :b =3 : 4
b : c=2 : 5
a : b : c = 3×2 : 4×2 : 4×5
= 6 : 8 : 20
= 3 : 4 : 10
Example: a : b = 1 : 2, b : c = 3 : 2, and c : d = 1 : 3. Find a : b : c : d?

Sol. a b c d
1 2 2 2
3 3 2 2
1 1 1 3
=
a : b : c : d 1×3×1:2×3×1:2×2×1:2×2×3
= 3 : 6 : 4 : 12
Partnership
Concept 1: If a group of n persons invested different amount for different period then their profit is in the ratio s
At1 : Bt2 : Ct3 : Dt4 : .....: Xtn
Here first person invested amount A for t1 period, second persons invested amount B for t2 period and so
on.
Example: A starts a business with Rs 2,000, B joins him after 3 months with Rs 4,000. C puts a sum of Rs 10,000 in the
business for 2 months only. At the end of the year the business gave a profit of Rs 5600. How should the
profit be divided among them ?
Sol: Ratio of their profits (A’s : B’s : C’s) = 2 × 12 : 4 × 9 : 10 × 2 = 6 : 9 : 5
Now, 6 + 9 + 5 = 20
5600 5600 5600
Then A’s share = ´ 6 = Rs 1680, B’s share = ´ 9 = Rs 2520, C’s share = ´ 5 = Rs 1400
20 20 20
Concept 2: If investments are in the ratio of a : b : c and the timing of their investments in the ratio of x : y : z then the
ratio of their profits are in the ratio of ax : by : cz.
Example : A, B and C invested capital in the ratio 2 : 3 : 5, the timing of their investments being in the ratio 4 : 5 : 6. In
what ratio would their profit be distributed?
Sol. We should know that if the duration for their investments be in the ratio x : y : z, and investment is in the
ratio a : b : c then the profit would be distributed in the ratio ax : by : cz.
Thus, following the same rule, the required ratio = 2 × 4 : 3 × 5 : 5 × 6 = 8 : 15 : 30

72 @BEST300MCQ For More Study Material


Visit: studyiq.com
p q r
: : .
Concept 3: If investments are in the ratio a : b : c and profits in the ratio p : q : r, then the ratio of time is
a b c
Example : A, B and C invested capital in the ratio 5 : 6 : 8. At the end of the business term, they received the profits in
the ratio 5 : 3 : 12. Find the ratio of time for which they contributed their capital?
5 3 12 1 3
Sol: Using the above formula, we have the required ratio = : : = 1: : = 2:1:3.
5 6 8 2 2

1. A sum of Rs 9000 is to be distributed among A, B and


x 3x 50x - 3x 47x
C in the ratio 4 : 5 : 6. What will be the difference Remaining money = - = =
between A’s and C’s shares ? 5 250 250 250
Sol. Total amount = Rs 9000 47x 47 ´ 250
A’s share = 4x Þ B’s share = 5x \ = 47 Þ x= = Rs 250
250 47
and C’s share = 6x
4. Rs 180 contained in a box consists of Rs 1,50 paise
Then, 4x + 5x + 6x = 9000 Þ 15x = 9000; \ x = 600 and 25 paise coins in the ratio 2 : 3 : 4. What is the
Now, A's share = 4 × 600 = Rs 2400 and number of 50 paise coins ?
C’s share = 6 × 600 = Rs 3600 Sol. Ratio of the values of the coins
Difference between A’s and C’s share
3 4 3
= Rs (3600 – 2400) = Rs 1200 = 2: : =2: :1=4:3:2
2. Rs 6400 are divided among three workers in the ratio 2 4 2
Sum of the ratios = 4 + 3 + 2 = 9
3 5
: 2 : . The share (in Rs) of the second worker is
5 3 3
\ Values of 50 paise coins = ´ 180 = Rs 60
Sol. Total amount = 6400 9
3 Numbers of 50 paise coins = 120
Let first worker’s share = x
5 5. A, B and C enter into a partnership with shares in the
Second worker’s share = 2x 7 4 6
ratio : : . After 4 months, A increase his share by
5 2 3 5
Third worker’s share = x
3 50%. If the total profit at the end of the year was Rs
43200. Then, the B’s share in the profit is:
3 5
Then, x + 2x + x = 6400 Sol. Ratio of initial shares of A, B and C in the partnership
5 3
7 4 6 7 ´ 15 4 ´ 10 6 ´ 6
9x + 30x + 25x A:B:C= : : = : :
Þ = 6400 2 3 5 2 ´ 15 3 ´ 10 5 ´ 6
15
Þ 64x = 6400 × 15 = 1500 105 40 36
= : :
\ Second worker’s share = 1500×2 = 3000 30 30 30
3. A boy, after giving away 80% of his pocket money to (LCM of 2, 3, 5 = 30) = 105 : 40 : 36
one companion and 6% of the reminder to another, Let the respective shares of A, B and C be Rs 105x, Rs
has Rs 47 left with him. How much pocket money did 40x and Rs 36x
the boy have in the beginning ? New shares of A, B and C in the partnership
Sol. Let the boy had Rs x.
150
4x A = Rs 105x for 4 months + 105x ´ for 8 months
Money given to first companion = 80% of x = 100
5
æ 3 ö
4x x = (105x × 4) + ç 105x ´ ´ 8 ÷ = 420x + 1260x = 1680x
Remaining money = x - = è 2 ø
5 5
Money given to the another companion B = 40x for 12 months = 40x × 12 = 480x
C = 36x for 12 months = 36x × 12 = 432 x
6 x 3x A : B : C = 1680 : 480 : 432 = 35 : 10 : 9
= ´ =
100 5 250

73 @BEST300MCQ For More Study Material


Visit: studyiq.com
It is a type of simple partnership, so the profit or loss Sol. Ratio of equivalent capitals of A and B
of the business is distributed among the investors in = 100000 × 36 : 200000 × 24 = 36 : 48 = 3 : 4
the ratio of their invested money. 3
\ B’s share in profit Profit gained by A = ×84000 = Rs. 36000
7
B's investment 4
= ´ Total profit Profit gained by B = ×84000 = Rs.48000
( A + B + C) 's investment 7
Required difference = 48000 – 36000 = Rs. 12000
10 10 9. The ratio of income of A and B is 3 : 4 If the ratio of
= ´ 43200 = ´ 43200 = 8000 expenditure of both is 2 : 3 and each save Rs 200, find
35 + 10 + 9 54
the income of both A and B?
Hence, B’s share in the profit = Rs 8000. Sol. Income – Saving = Expenditure
7. In a business partnership among A, B, C and D, the According to question
profit is shared as follows Let the income of A and B be 3x and 4x respectively
A 's share B's share C's share 1 3x - 200 2
= = = = , x = 200
B's share C's share D 's share 3 4x - 200 3
Income of A = 3x = 3 × 200 = 600
If the total profit is Rs 400000, the share of C is: Income of B = 4x = 4 × 200 = 800
Sol. Given, A : B = 1 : 3 10. Rs 7800 distributed among A, B, and C. The share of
B:C=1:3=3:9 3 2
A is th of the share of B, and share of B is th of the
C : D = 1 : 3 = 9 : 27 4 3
Now, by joining the above three ratios, we get share of C. Then, find the difference between share of
A : B : C : D = 1 : 3 : 9 : 27 B and C?
Sum of the ratios = 1 + 3 + 9 + 27 = 40 3 2
Sol. A = BÞA:B=3: 4 B= CÞB:C=2:3
4 3
9
\ C’s share in profit = ´ 400000 = Rs 90000 A : B : C = 6 : 8 : 12 = 3 : 4 : 6
40
4
8. A started a business with a capital of Rs 100000. 1 yr Share of B = ´ 7800 = 2400
13
later, B joined him with a capital of Rs 200000. At the
end of 3 yr from the start of the business, the profit 6
Share of C = ´ 7800 = 3600
earned was Rs 84000. The share of B in the profit 13
exceeded the share of A by Difference of share of B and C = Rs. 1200

Ques tions (a) 140 (b) 130


(c) 60 (d) 150
5. 30% of a number is 190.8. What will be 175% of that
1. One year ago the ratio of Ramu and Somu age was 6 :
number?
7 respectively. Four years hence their ratio would
become 7 : 8. How old is Somu? (a) 1113 (b) 1115
(a) 24 years (b) 30 years (c) 1502 (d) 1002
(c) 32 years (d) 36 years 6. What will be 32% of 3/8 th of 1000?
2. If 33% of A is equal to 55% of B then find the ratio of A (a) 115 (b) 125
and B? (c) 120 (d) 129
(a) 3 : 5 (b) 5 : 3 7. Two numbers are respectively 20% and 50% more
than a third number. The ratio of the two numbers is:
(c) 3 : 4 (d) 5 : 4
(a) 2 : 5 (b) 3 : 5
3. What is the value of 68% of two fifth of 550?
(c) 4 : 5 (d) 6 : 7
(a) 149.6 (b) 150
8. A sum of money is to be distributed among A, B, C, D
(c) 154 (d) 158
in the proportion of 5 : 2 : 4 : 3. If C gets Rs. 1000 more
4. If 24 is deducted from 45% of a number result becomes than D, what is B’s share?
3 (a) Rs. 500 (b) Rs. 1500
48. What will be part of that number?
8 (c) Rs. 2000 (d) Rs. 2400

74 @BEST300MCQ For More Study Material


Visit: studyiq.com
9. If 0.75 : x : : 5 : 8, then x is equal to: 22. Find the triplicate ratio of 7 : 5?
(a) 1.12 (b) 1.2 (a) 125 : 343 (b) 343 : 125
(c) 1.25 (d) 1.30 (c) 344 : 125 (d) 343 : 126
10. The sum of three numbers is 98. If the ratio of the first 23. What will be the inverse ratio of 17 : 19?
to second is 2 : 3 and that of the second to the third is (a) 19 : 17 (b) 18 : 17
5 : 8, Then the second number is: (c) 17 : 18 (d) 19 : 5
(a) 20 (b) 30 24. Find the compound ratio of 2 : 7, 5 : 3 and 4 : 7?
(c) 48 (d) 58 (a) 147 : 40 (b) 40 : 147
11. If Rs. 872 is divided into three parts, proportional to (c) 147 : 30 (d) 30 : 147
1 2 3 25. If A : B = 3 : 4 and B : C = 8 : 9, then find the value of A
: : , then the first part is: : B : C?
2 3 4
(a) 3 : 4 : 5 (b) 1 : 2 : 3
(a) Rs. 182 (b) Rs. 190
(c) 7 : 12 : 17 (d) 6 : 8 : 9
(c) Rs. 227.46 (d) Rs. 204
26. If a : b = 3 : 5 and b : c = 4 : 7, then a : c is equal to:
12. The fourth proportional to 5, 8, 15 is:
(a) 11 : 35 (b) 35 : 11
(a) 18 (b) 24
(c) 35 : 12 (d) 12 : 35
(c) 19 (d) 20
13. Two numbers are in the ratio 3 : 5. If 9 is subtracted P Q R
27. If P : Q : R = 2 : 3 : 4 , then find : : ?
from each, then new numbers are in the ratio 12 : 23. Q R P
The smaller number is : (a) 8 : 9 : 24 (b) 9 : 8 : 24
(a) 27 (b) 33 (c) 24 : 8 : 9 (d) 8 : 24 : 9
(c) 49 (d) 55
14. If a number is reduced by 40% it become two third of a b
28. If = , then (a + 3) : (b + 8) is equal to:
another number. What is ratio of the second number 3 8
to first number? (a) 3 : 8 (b) 8 : 3
(a) 6 : 10 (b) 9 : 8 (c) 5 : 8 (d) 3 : 5
(c) 8 : 9 (d) 9 : 10 29. The ratio of 43 : 25 is same is:
5a + 3b 23 (a) 2 : 1 (b) 4 : 1
15. = then the value of a : b is: (c) 7 : 5 (d) 7 : 10
2a - 3b 5
30. If 2 : x : : 5 : 7, then x is equal to:
(a) 1 : 2 (b) 1 : 3
(a) 1.12 (b) 2.80
(c) 4 : 1 (d) 2 : 3
(c) 1.25 (d) 1.30
16. If P : Q = 8 : 15 and Q : R = 3 : 2, then find P : Q : R?
31. The salaries of A, B, C are in the ratio 2 : 3 : 5 . If the
(a) 8 : 15 : 7 (b) 7 : 15 : 8
increments of 15%, 10% and 20% are allowed
(c) 8 : 15 : 10 (d) 10 : 15 : 8 respectively in their salaries, then what will be the
17. If P : Q = 8 : 15, Q : R = 5 : 8 and R : S = 4 : 5, then P : S new ratio of their salaries?
is equal to: (a) 3 : 3 : 10 (b) 10 : 11 : 20
(a) 4 : 15 (b) 2 : 15 (c) 23 : 33 : 60 (d) 25 : 27 : 29
(c) 3 : 19 (d) 7 : 15 32. If Rs. 782 be divided into three parts, proportional to
18. Find the 4th proportional to 4, 16 and 7? 1 2 3
(a) 28 (b) 29 : : , then the first part is:
2 3 4
(c) 22 (d) 25 (a) Rs. 182 (b) Rs. 190
19. Find the mean proportional between 9 and 64? (c) Rs. 196 (d) Rs. 204
(a) 25 (b) 24 33. Two numbers are in the ratio 1 : 2. If 7 is added to
(c) 27 (d) 35 both, they changes to 3 : 5. The greatest number is:
20. What will be the duplicate ratio of 2 : 7 ? (a) 24 (b) 26
(a) 4 : 49 (b) 49 : 4 (c) 28 (d) 32
(c) 4 : 14 (d) 8 : 343 34. The ratio of three numbers is 3 : 4 : 5 and the sum of
21. Find the sub-duplicate ratio of 81 : 64? their squares is 1250. The sum of the numbers is :
(a) 8 : 9 (b) 4 : 9 (a) 30 (b) 50
(c) 9 : 8 (d) 7 : 8 (c) 60 (d) 90

75 @BEST300MCQ For More Study Material


Visit: studyiq.com
35. Sachin is younger than Rahul by 4 years. If their ages 43. Anil and Nikhil started a business with investing Rs.
are in the respective ratio of 7 : 9, how old is Sachin? 3300 and Rs. 5000 respectively. If they earned profit
(a) 16 years (b) 18 years of 1660 then what will be Anil’s share?
(c) 14 years (d) 15 years (a) 640 (b) 650
36. At present, the ratio between the ages of Arun and (c) 655 (d) 660
Deepak is 4 : 3. After 6 years. Arun’s age will be 26 44. Rahul started a business by investing Rs. 45,000. 6
years. What is the age of Deepak at present? months later Sanjay joined him with Rs. 30,000. In
(a) 12 years (b) 15 years what ratio should the earned profit be distributed at
the end of the year?
1 (a) 1 : 3 (b) 3 : 1
(c) 19 years (d) 21 years
2 (c) 2 : 3 (d) 3 : 2
37. Present ages of X and Y are in the ratio 5 : 6 respectively. 45. Anil, Nikhil and Saurabh started a business in
Seven year hence this ratio will become 6 : 7 partnership by investing in the ratio of 6 : 5 : 8
respectively. What is X’s present age in years? respectively. At the end of the year they earned a profit
(a) 35 (b) 42 of Rs. 83,600. What will be Saurabh’s share?
(c) 49 (d) Cannot be determined (a) Rs. 30,200 (b) Rs. 35,200
38. Present ages of Sameer and Anand are in the ratio of (c) Rs. 32,500 (d) Rs. 34,000
5 : 4 respectively. Three years hence, the ratio of their 46. Ram, Karan and Rohan invested capital in the ratio
ages will become 11 : 9 respectively. What is Anand’s of 2 : 3 : 4 for time period of 6 : 4 : 3. Find the ratio of
present age in years? profit distributed?
(a) 24 (b) 27 (a) 12 : 13 : 14 (b) 13 : 12 : 14
(c) 40 (d) Cannot be determined (c) 1 : 1 : 1 (d) 14 : 12 : 13
39. The total of the ages of Jayant, Prem and Saransh is 93 47. Ravi and Kavi start a business by investing Rs. 8000
years. Ten years ago, the ratio of their ages was 2 : 3 : and Rs. 72000, respectively. Find the ratio of their
4. What is the present age of Saransh? profits at the end of year.
(a) 24 years (b) 32 years (a) 2 : 9 (b) 5 : 9
(c) 34 years (d) 38 years (c) 7 : 9 (d) 1 : 9
40. Hitesh is 40 years old and Ronnie is 60 years old. 48. Srikant and Vividh started a business investing
How many years ago was the ratio of their ages 3 : 5? amounts of Rs. 185000 and Rs. 225000, respectively.
(a) 5 years (b) 10 years If Vividh’s share in the profit earned by them is Rs.
(c) 20 years (d) 37 years 9000, what is the total profit earned by them together?
41. Aditya and Sanjay started a business investing 45000 (a) Rs. 17400 (b) Rs. 16400
and 30000 respectively. What will be the ratio (c) Rs. 16800 (d) Rs. 17800
between their profit? 49. Rajan and Sajan started a business initially with Rs.
(a) 2 : 3 (b) 3 : 2 14200 and Rs. 15600, respectively. If total profits at
(c) 4 : 9 (d) Cant be determined the end of year is Rs. 74500, what is the Rajan’s share
42. Mahesh and Kishore started a business with investing in the profit?
Rs. 60,000 and Rs. 50,000 respectively. If after 1 year (a) Rs. 39000 (b) Rs. 39600
they earned Rs. 33,000 profit, what will be kishore (c) Rs. 35000 (d) Rs. 35500
share? 50. A starts a business with Rs. 4000 and B joins him
(a) 15,000 (b) 30,000 after 3 months with Rs. 16000. Find the ratio of their
(c) 35,000 (d) 20,000 profits at the end of year.
(a) 1 : 3 (b) 2 : 33
(c) 1 : 9 (d) 1 : 7

76 @BEST300MCQ For More Study Material


Visit: studyiq.com
1. The difference between a two-digit number and the 625 x
number obtained by interchanging the digits is 36. 10. = then what will be the value of x
What is the difference between the sum and the x 1156
difference of the digits of the number if the ratio (a) 800 (b) 850
between the digits of the number is 1 : 2 ? (c) 900 (d) 950
(a) 4 (b) 8 11. A mixture contains alcohol and water in the ratio 4 :
(c) 16 (d) 20 3. If 5 liters of water is added to the mixture, the ratio
2. Seats for Mathematics, Physics and Biology in a school becomes 4 : 5. Find the quantity of alcohol in the given
are in the ratio 5 : 7 : 8. There is a proposal to increase mixture.
these seats by 40%, 50% and 75% respectively. What (a) 5 Ltr. (b) 7.5 Ltr.
will be the ratio of increased seats? (c) 10 Ltr. (d) 12 Ltr.
(a) 2 : 3 : 4 (b) 6 : 7 : 8 12. In a bag, there are coins of 25 p, 10 p and 5 p in the
(c) 6 : 8 : 9 (d) 4 : 8 : 9 ratio of 1 : 2 : 3. If there are Rs. 30 in all, how many 5 p
3. In a mixture 60 litres, the ratio of milk and water is coins are there?
2 : 1. What quantity of water should be added so that (a) 50 (b) 100
the ratio becomes 1 : 3? (c) 150 (d) 200
(a) 80 L (b) 100 L
13. If Rs. 510 be divided among A, B, C in such a way that
(c) 120 L (d) 60 L
4. The ratio of the number of boys and girls in a college 2 1
A gets of what B gets and B gets of what C gets,
is 7 : 8. If the percentage increase in the number of 3 4
boys and girls be 20% and 10% respectively, what then their shares are respectively:
will be the new ratio? (a) Rs. 120, Rs. 240, Rs. 150
(a) 8 : 9 (b) 17 : 18 (b) Rs. 60, Rs. 90, Rs. 360
(c) 21 : 22 (d) Can’t be determined (c) Rs. 150, Rs. 300, Rs. 60
5. Salaries of Ravi and Sumit are in the ratio 2 : 3. If the (d) Rs. 70, Rs. 90, Rs. 350
salary of each is increased by Rs. 4000, the new ratio
becomes 40 : 57. What is Sumit’s salary? 14. Rs. 366 are divided among A, B and C so that A may
(a) Rs. 17,000 (b) Rs. 20,000 1 2
get as much as B and C together, B may get as
(c) Rs. 25,500 (d) Rs. 34,000 2 3
6. The salaries of A, B and C are in the ratio 2 : 3 : 5. If the much as A and C together, then the share of A is :
increments of 15%, 10% and 20% are allowed (a) Rs. 122 (b) Rs. 129.60
respectively in their salaries, then what will be new (c) Rs. 146. 60 (d) Rs. 183
ratio of their salaries?
15. Ratio of the earnings of A and B is 4 : 7. If the earnings
(a) 3 : 3 : 10 (b) 10 : 11 : 20
A increase by 50% and those of B decrease by 25%,
(c) 23 : 33 : 60 (d) 3 : 4 : 5 the new ratio of their earnings becomes 8 : 7. What are
7. In a bag, there are coins of 25 p, 10 p and 5 p in the A’s earnings?
ratio of 2 : 3 : 4. If there is Rs. 50 in all, how many 5 p
(a) Rs. 21,000 (b) Rs. 26,000
coins are there?
(c) Rs. 28,000 (d) Data inadequate
(a) 50 (b) 100
(c) 50 (d) 200 16. An amount of Rs. 735 was divided between A, B and
C. If each of them had received Rs. 25 less, their shares
8. A sum of money is divided among C, A and B in ratio
would have been in the ratio of 1 : 3 : 2. The money
of 4 : 5 : 6 and another sum of money is divided between
received by C was :
M and N equally if B got 2000 more than M then how
much C get? (a) Rs. 195 (b) Rs. 200
(a) Rs. 1000 (b) Rs. 500 (c) Rs. 225 (d) Rs. 245
(c) Rs. 4000 (d) Can't be determined 17. An amount of Rs. 2430 is divided among A, B and C
9. Three number A, B and C are in ratio of 12 : 15 : 25. If such that if their shares be reduced by Rs. 5, Rs. 10
sum of these numbers be 364 find the ratio between and Rs. 15 respectively, the remainders shall be in the
difference of B and A and the difference of C and B? ratio of 3 : 4 : 5. Then, B’s share was:
(a) 3 : 2 (b) 3 : 10 (a) Rs. 605 (b) Rs. 790
(c) 3 : 5 (d) 4 : 2 (c) Rs. 800 (d) Rs. 810

77 @BEST300MCQ For More Study Material


Visit: studyiq.com
18. Gold is 19 times as heavy as water and copper is 9 26. The ratio of the incomes of A and B is 5 : 4 and the
times as heavy as water. In what ratio should these be ratio of their expenditures is 3 : 2. If at the end of the
mixed to get an alloy 15 times as heavy as water ? year, each saves Rs. 1600, then the income of A is :
(a) 1 : 1 (b) 2 : 3 (a) Rs. 3400 (b) Rs. 3600
(c) 1 : 2 (d) 3 : 2 (c) Rs. 4000 (d) Rs. 4400
19. 15 litres of mixture contains 20% alcohol and the rest 27. A and B are two alloys of gold and copper prepared
water. If 3 litres of water be mixed with it, the by mixing metals in the ratio 7 : 2 and 7 : 11
percentage of alcohol in the new mixture would be : respectively. If equal quantities of the alloys are melted
to form a third alloy C, the ratio of gold and copper in
2
(a) 15% (b) 16 % C will be :
3
(a) 5 : 7 (b) 5 : 9
1 (c) 7 : 5 (d) 9 : 5
(c) 17% (d) 18 %
2 28. The ratio between the present ages of P and Q is 5 : 7
20. 85 L of a mixture contains milk and water in the ratio respectively. If the difference between Q’s present age
27 : 7. How much more water is to be added to get a and P’s age after 6 years is 2, what is the total of P’s
new mixture containing milk and water in the ratio and Q’s present ages?
3:1 ? (a) 48 years (b) 52 years
(a) 5 L (b) 6.5 L (c) 56 years (d) Cannot be determined
(c) 7.25 L (d) 8 L 29. The ratio of the father’s age to his son’s age is 7 : 3.
1 1 1 The product of their ages is 756. The ratio of their ages
21. The sides of a triangle are in the ratio : : and its after 6 years will be :
2 3 4
perimeter is 104 cm. The length of the longest side is : (a) 5 : 2 (b) 2 : 1
(a) 52 cm (b) 48 cm (c) 11 : 7 (d) 13 : 9
(c) 32 cm (d) 26 cm 30. The present ages of three persons are in proportions
22. The ratio of the number of boys and girls in a school is 4 : 7 : 9. Eight years ago, the sum of their ages was 56.
3 : 2. If 20% of the boys and 25% of the girls are Find their present ages (in years).
scholarship holders, what percentage of the students (a) 8, 20, 28 (b) 16, 28, 36
does not get the scholarship? (c) 20, 35, 45 (d) 13, 16, 25
(a) 56 (b) 70 31. The ratio of the ages of a man and his wife is 4 : 3.
(c) 78 (d) 80 After 4 years, this ratio will be 9 : 7. If at the time of
23. Three containers have volumes in the ratio 3 : 4 : 5. marriage, the ratio was 5 : 3, then how many years
They are full of mixtures of milk and water. The ago were they married?
mixtures contain milk and water in the ratio of 4 : 1, (a) 8 years (b) 10 years
3:1 and 5 : 2 respectively. The contents of all these (c) 12 years (d) 15 years
three containers are poured into a fourth container. 32. The ratio between the present ages of A and B is 5 : 3
The ratio of milk and water in the fourth container is: respectively. The ratio between A’s age 4 years ago
(a) 4 : 1 (b) 151 : 48 and B’s age 4 years hence is 1 : 1. What is the ratio
(c) 157 : 53 (d) 5 : 2 between A’s age 4 years hence and B’s age 4 years
24. x varies inversely as square of y. Given that y = 2 for ago ?
x = 1. The value of x for y = 6 will be equal to : (a) 1 : 3 (b) 2 : 1
(a) 3 (b) 9 (c) 3 : 1 (d) 4 : 1
1 1 33. A person’s present age is two-fifth of the age of his
(c) (d) mother. After 8 years, he will be one-half of the age of
3 9
his mother. How old is the mother at present ?
25. The electricity bill of a certain establishment is partly
(a) 32 years (b) 36 years
fixed and partly varies as the number of units of
electricity consumed. When in a certain month 500 (c) 40 years (d) 48 years
units are consumed, the bill is Rs. 1680. In another 34. Four years ago, the father’s age was three times the
month 600 units are consumed and the bill is Rs. 1980. age of his son. The total of the ages of the father and
In yet another month 700 units are consumed. The the son after four years, will be 64 years. What is the
bill for that month would be: father’s age at present?
(a) Rs. 2280 (b) Rs. 1680 (a) 32 years (b) 36 years
(c) Rs. 1840 (d) Rs. 1950 (c) 44 years (d) 40 years

78 @BEST300MCQ For More Study Material


Visit: studyiq.com
35. The present ages of Reena and Usha are 24 and 36 yr, 44. Ram and Anil enter into partnership for a year. Ram
respectively. What was the ratio between the ages of invests Rs. 1200 and Anil invests Rs. 1500. After 3
Usha and Reena respectively 8 yr ago? month, Aditya enters with Contribution of Rs. 900. If
(a) 7 : 4 (b) 4 : 7 Anil withdraws his Money after 6 Months. What will
(c) 11 : 8 (d) 8 : 11 be the share of Aditya, if the profit is Rs. 2450?
36. At present Anil is 1.5 times of Purvi’s age. 8 yr hence, (a) Rs. 700 (b) Rs. 1820
the respective ratio between Anil and Purvi’s ages (c) Rs. 630 (d) Rs. 1120
will be 25 : 18. What is Purvi’s present age? 45. A bag contains 10-paise and 25-paise coins in the
(a) 50 yr (b) 28 yr ratio of 17 : 6. If the total money in the bag is Rs. 112,
(c) 42 yr (d) 36 yr the number of 10-paise coins is
37. The difference between the ages of Meena and Seema (a) 35 (b) 210
is 3 yr and the ratio between their ages is 7 : 8. What is (c) 490 (d) 595
the sum of their ages? 46. P & Q enter into a partnership. P puts in Rs. 50 and Q
(a) 43 yr (b) 41 yr puts Rs. 45. At the end of the 4 months P withdraws
(c) 45 yr (d) 48 yr half his capital and at the end of 5th month Q
38. The ratio of the present age of Manoj to that of Wasim withdraws half of his capital. S then enter into with
is 3 : 11. Wasim is 12 yr younger than Rehana. the capital of Rs. 70. At the end of 12 months they got
Rehana’s age after 7 yr will be 85 yr. What is the profit of Rs. 1272.5 What was S share in the profit?
present age of Manoj’s father, who is 25 yr older than (a) 460 (b) 480
Manoj? (c) 490 (d) Can’t be determined
(a) 43 yr (b) 67 yr 47. A and B enter into a partnership by investing Rs.
(c) 45 yr (d) 69 yr 12000 and Rs. 16000 respectively. After 4 months B
39. The ratio between the present ages of Indira and Lizzy withdraws Rs. 6000 and A invest Rs. 5000 more. After
is 3 : 8, respectively. After 8 yr, Indira’s age will be 20 2 more months C joined with Rs. 20,000 capital. What
yr. What was Lizzy’s age 5 yr ago? will be A’s share in profit if total profit of Rs. 30,100 is
(a) 37 yr (b) 27 yr earned at the end of one year?
(c) 28 yr (d) 38 yr (a) 12360.5 (b) 12365
40. At present, Kavita is twice Sarita’s age. 8 yr hence, the (c) 12362.5 (d) Can’t be determined
respective ratio between Kavita’s and Sarita’s ages 48. P, Q and R invested their capital in the ratio of 8 : 6 : 5
will be 22 : 13. What is Kavita’s present age? at the end of the business they received the profit in
(a) 26 yr (b) 18 yr the ratio of 1 : 3 : 5. Find the ratio of time for which
(c) 42 yr (d) 36 yr they contributed their capital?
41. 10 yr before, the ratio of ages of A and B was (a) 1 : 4 : 8 (b) 4 : 1 : 8
13 : 17. After 17 yr from now, the ratio of their (c) 8 : 4 : 1 (d) Can’t be determined
ages will be 10 : 11. The present age B is 49. Aditya and Manish continued in a joint business for
(a) 23 yr (b) 40 yr 36 months. Aditya contribute Rs. 300 for certain time
(c) 27 yr (d) 44 yr and Manish invested Rs. 500 for remaining time. If
42. Ram, Rohan and Karn invested in business in the out of total profit of Rs. 1020, Aditya gets Rs. 495, then
ratio of 3 : 2 : 5 respectively. If Ram earn 100% more for how long Aditya kept his money?
profit than Rohan and Karn earns 40% more profit (a) 16 months (b) 14 months
than Rohan. What will be the share of Rohan in profit? (c) 8 months (d) 22 months
(a) 2350 (b) 3640 50. Aditya, Manish and Gaurav enter into partnership
(c) 1235 (d) Can’t be Determined with shares in the ratio of 7/2 : 4/3 : 6/5. After 4
43. Ram started a business by investing Rs. 45000. After months, Aditya increase his share by 50%. If at the
3 Months, Sanjay joined him with a capital of Rs. end of one year profit be Rs. 43200. Then what will be
60,000. After 6 Months, Aditya joined them with a Manish share in profit?
capital of Rs. 90,000. At the end of the year they earned (a) 8000 (b) 7200
a profit of Rs. 16500. What is the Ram share in (c) 28000 (d) 24400
business?
(a) 6600 (b) 5900
(c) 5500 (d) 5000

79 @BEST300MCQ For More Study Material


Visit: studyiq.com
1. An alloy of copper and zinc is taken in the ratio 1 : 2, (a) Rs. 696 (b) Rs. 626
and another alloy of the same metal is taken in the (c) Rs. 656 (d) Rs. 956
ratio of 2 : 3. How many parts of the two alloys must 9. Sita and Gita invested the same capital in a business.
be taken to obtain a new alloy consisting of copper At the year-end they share the profit in the ratio of 3 :
and zinc that are in the ratio 3 : 5? 2. If Sita has invested her capital for the whole year,
(a) 7 and 9 (b) 5 and 7 for how many months Gita has invested her capital?
(c) 3 and 5 (d) 5 and 3 (a) 8 months (b) 7 months
2. In two alloys, gold and silver are in the ratios of 4 : 1 (c) 5 months (d) 9 months
and 1 : 3. After alloying together 10 kg. of the first 10. A year ago, the cost of Maruti and Figo are in the ratio
alloy, 16 kg. of the second and several kilograms of of 3 : 4. The ratio of present and past year costs of
pure gold, an alloy was obtained in which the ratio of Maruti and Figo are 5 : 4 and 3 : 2. If the sum of their
gold to silver was 3 : 2. Find the weight of the new present cost is 7.8 lacs. Find the cost of Figo a year
alloy. ago.
(a) 12 kg. (b) 19 kg.
(a) 3.2 lac (b) 1.6 lac
(c) 24 kg. (d) 35 kg.
(c) 3.5 lac (d) 4.0 lac
3. Rs. 5625 are divided among A, B and C so that A
11. One year ago, the ratio between Ram’s and Shyam’s
received half as much as B and C together and B
salaries was 3 : 5. The ratio of their individual salaries
received one fourth as much as A and C together.
of last year and present year are 2 : 3 and 4 : 5
Find the sum of A’s and B’s share together.
respectively. If their total salaries for the present year
(a) Rs. 2500 (b) Rs. 3000
is Rs. 8600, find the present salary of Ram ?
(c) Rs. 1500 (d) Rs. 4000
(a) 3200 (b) 3600
4. A man spends Rs. 2310 on an average during the first
(c) 4000 (d) 4400
8 month from the income. During next 4 months he
spends Rs. 1800 on an average from his income but 12. Two candles of the same height are lighted at the same
he had taken a loan of Rs. 1680 apart from his income time. The first is consumed in 8 hours and the second
to meet his both ends. If he spent all what he had then in 6 hours. Assuming that each candle burns at a
the ratio of average income and average expenditure? constant rate, in how many hours after being lighted,
the ratio between the height of first and second candles
(a) 50 : 53 (b) 100 : 103
becomes 2 : 1?
(c) 100 : 109 (d) 100 : 107
(a) 4 h (b) 5 h
5. In a family the ratio of expenses to the savings is 5 : 3.
But his expenses is increased by 60% and income is (c) 4 h 30 m (d) 4 h 48m
increased by only 25% thus there is decrease of Rs. 13. The total of the daily wages of Ram, Shyam and
3500 in the saving. Find the increased income of the Mohan is Rs. 450. If they, respectively spend 25%,
family. 20% and 50% of their incomes then the ratio of their
(a) Rs. 35000 (b) Rs. 28000 savings becomes 9 : 8 : 4. What is the income of Ram?
(c) Rs. 25000 (d) Rs. 18,500 (a) 180 (b) 190
6. Arti has coins of the denomination of Rs. 1, 50 paise (c) 200 (d) 175
and 25 paise in the ratio of 12 : 10 : 7. The total worth 14. There is 1000 litres milk in a pot. 100 litres of milk is
of the coins she has is Rs. 75. Find the number of 25 taken out and same amount of water is poured into it.
paise coins. Again 200 litres of mixture is taken out and same
(a) 28 (b) 36 amount of water is poured into it and finally 400 litres
(c) 42 (d) 32 of mixture is taken out and same amount of water is
7. Two horses cost as much as 5 dogs, 6 dogs as much poured into it then what is the amount of milk in the
as 8 oxen, 10 oxen as much as 50 sheep, 14 sheep as resulting mixture?
much as 9 goats. If the price of one goat is Rs. 700, find (a) 432 (b) 436
the cost of one horse. (c) 425 (d) 438
(a) Rs. 7500 (b) Rs. 6400 15. On entire Earth the ratio between land and water is
(c) Rs. 5200 (d) Rs. 4800 1 : 2 and at northern hemisphere this ratio is 2 : 3.
8. Rs. 2186 is distributed among A, B and C. If money What will be the ratio of land with water at southern
given to them is decreased by Rs. 26, Rs. 28 and hemisphere?
Rs. 32 respectively, then the ratio of their share become (a) 11 : 4 (b) 4 : 11
9 : 13 : 8. What is the amount given to A ? (c) 15 : 4 (d) 4 : 15

80 @BEST300MCQ For More Study Material


Visit: studyiq.com
16. The ratio of the incomes of A, B and C is 3 : 7 : 4 and actual mixture would have been 13.33% less than that
the ratio of their expenditures is 4 : 3 : 5. If in the of the new mixture. What is the concentration of petrol
income of Rs. 2400, A saves Rs. 300 then the savings in actual mixture?
of B and C is respectively are (a) 20% (b) 16.66%
(a) Rs. 4025 and Rs. 575 (c) 26.66% (d) 8.33%
(b) Rs. 1575 and 2625 19. The ratio of age between A and B is 6 : 5 and the age of
(c) Rs. 2750 and Rs. 1025 9
(d) Rs. 3725 and Rs. 1525 each C and D is times that of B. Age of F is less
10
17. The ratio of wine and water in a drum is 3 : 1. What than A but greater than B. The ratio of ages between B
part of the mixture is taken out and same amount of and E is 2 : 3. Also age of A is 3 years less than E. What
water is added to it so that the ratio of wine and water is the ratio of ages of A and F if all the ages are in
in the mixture become 1 : 1? integers?
1 1 (a) 12 : 11 (b) 9 : 7
(a) (b)
4 3 (c) 24 : 19 (d) 12 : 13
20. Three vessels having volumes in the ratio of 2 : 3 : 5
3 2
(c) (d) are full of a mixture of water and milk. In the first
4 3 vessel ratio of water and milk is 1 : 3, in second 2 : 3
18. In a mixture of petrol and kerosene, petrol is only 99 and in third vessel, 2 : 5. If all the three vessels were
litres. If this same quantity of petrol would be poured out in a large container, what is the resulting
presented in another mixture of petrol and kerosene ratio of milk and water?
where total volume would be 198 litres less than the (a) 43 : 96 (b) 438 : 962
actual mixture then the concentration of petrol in the
(c) 348 : 962 (d) 481 : 219

1. If A : B = 5 : 7 and B : C = 6 : 11, then A : B : C is : 8. Salaries of Ravi and Sumit are in the ratio 2 : 3. If the
(a) 55 : 77 : 66 (b) 30 : 42 : 77 salary of each is increased by Rs. 4000, the new ratio
(c) 35 : 49 : 42 (d) None of these becomes 40 : 57. What is Sumit’s present salary?
(a) Rs. 17,000 (b) Rs. 20,000
A B C
2. If A : B : C = 2 : 3 : 4, then : : is equal to : (c) Rs. 25,500 (d) None of these
B C A
9. If Rs. 510 be divided among A, B, C in such a way
(a) 4 : 9 : 16 (b) 8 : 9 : 12
(c) 8 : 9 : 16 (d) 8 : 9 : 24 2 1
that A gets of what B gets and B gets of what C
3. The ratio of 43.5 : 25 is same as : 3 4
(a) 2 : 1 (b) 4 : 1 gets, then their shares are respectively:
(c) 7 : 5 (d) 7 : 10 (a) Rs. 120, Rs 240, Rs. 150
4. If (x : y) = 2 : 1, then (x – y2) : (x2 + y2 ) is :
2 (b) Rs. 60, Rs. 90, Rs. 360
(a) 3 : 5 (b) 5 : 3 (c) Rs. 150, Rs. 300, Rs 60
(c) 1 : 3 (d) 3 : 1 (d) None of these
5. If (4x – 3y ) : (2x + 5y ) = 12 : 19, then (x : y) is :
2 2 2 2
10. The ratio of the number of boys and girls in a school
(a) 2 : 3 (b) 1 : 2 is 3 : 2. If 20% of the boys and 25% of the girls are
(c) 3 : 2 (d) 2 : 1 scholarship holders, what percentage of the students
6. If (a + b) : (b + c) : (c + a) = 6 : 7 : 8 and (a + b + c) = 14, does not get the scholarship?
then the value is c is : (a) 56 (b) 70
(a) 6 (b) 7 (c) 78 (d) 80
(c) 8 (d) 14 11. In a school, 10% of the boys are same in number as
7. If Rs. 782 be divided into three parts, proportional to 1
th of the girls. What is the ratio of the boys to girls
1 2 3 4
: : then the first part is :
2 3 4 in that school?
(a) Rs 182 (b) Rs. 190 (a) 3 : 2 (b) 5 : 2
(c) Rs. 196 (d) Rs. 204 (c) 2 : 1 (d) 4 : 3

81 @BEST300MCQ For More Study Material


Visit: studyiq.com
12. The electricity bill of a certain establishment is partly (a) 3,00,000 (b) 5,00,000
fixed and partly varies as the number of units of (c) 2,00,000 (d) 6,00,000
electricity consumed. When in a certain month 540
units are consumed, the bill is Rs. 1800. In another a+b b+c c+a
20. If = = then find the value of a : b : c
month 620 units are consumed and the bill is Rs. 5 6 7
2040. In yet another month 500 units are consumed. and if a + b + c = 18 then find the value of b.
The bill for that month would be : (a) 4 (b) 2
(a) Rs. 1560 (b) Rs. 1680 (c) 6 (d) 5
(c) Rs. 1840 (d) Rs. 1950 21. If 2a = 3b = 4c then find the value of a : b : c
13. A sum of Rs. 53 is divided among A, B, C in such a (a) 6 : 4 : 3 (b) 4 : 3 : 1
way that A gets Rs. 7 more than what B gets and B
(c) 2 : 4 : 3 (d) 2 : 6 : 4
gets Rs. 8 more than what C gets. The ratio of their
shares is : 22. There are 1 rupee, 50 paise and 25 paise coins in a
bag. The ratio of number of 1 rupee and 50 paise
(a) 16 : 9 : 18 (b) 25 : 18 : 10
coins is 5 : 6, while the ratio of number of 50 paise
(c) 18 : 25 : 10 (d) 15 : 8 : 30 and 25 paise coins is 3 : 5. If there is total Rs. 262.50
14. In a bag, there are coins of 25 p, 10 p and 5 p in the in the bag then find the number of coins in 25 paise?
ratio of 1 : 2 : 3. If there are Rs. 30 in all, how many 5
(a) 250 coins (b) 540 coins
p coins are there?
(c) 220 coins (d) 650 coins
(a) 50 (b) 100
23. An amount of Rs. 11, 700 was distributed among 10
(c) 150 (d) 200
males, 15 females and 5 children in such a way that
15. There is 34000 rupees in a bag in the denomination share of 3 males is equal to the share of 4 females and
of Rs. 50, Rs. 100, Rs. 500 and Rs. 1000 currency notes. the share of 6 children is equal to the share of 5
If the ratio of the number of these notes is 2 : 3 : 4 : 1 females. How much amount will each child get?
then find the number of notes of 500 denomination.
(a) 300 (b) 600
(a) 20 (b) 40
(c) 500 (d) 700
(c) 30 (d) 50
24. The ratio of the incomes of Ram and Shyam is 5 : 4
16. In a cricket tournament three batsman A, B and C
while the ratio of their expenditure is 10 : 7. If each of
scored a total of 1750 runs. C scored three-fourth runs
them saves Rs. 2500 then find the income of Shyam
of total runs scored by A and B together. While B
income.
12 (a) Rs. 6000 (b) Rs. 4000
scored part runs of total runs scored by A and C
23 (c) Rs. 5000 (d) Rs. 3000
together. How many runs A batsman scored? 25. A milkman A, B and C hire a pasture in Rs. 2670. A
(a) 400 (b) 500 grazes 4 cows and 3 calves for 5 days, B grazes 5
(c) 200 (d) 600 cows and 4 calves for 3 days while C grazes 6 cows
17. There are 68 employees in a office of class -1, class -2 and 5 calves for only 2 days. If one cow grazes three
and class -3 who are in the ratio of 5 : 10 : 2 times of a calf then. What will be the difference
respectively. Their total salary is Rs. 400000 which between the rent of A and C?
is in the ratio of 2 : 3 : 5 respectively. What is the (a) Rs. 435 (b) Rs. 250
salary of employee of class-3? (c) Rs. 500 (d) Rs. 300
(a) 40000 (b) 4000 26. Om, Jai and Jagdish start a business together. Om
(c) 10000 (d) 6000 invests Rs. 5,000 for whole year. Jai invests Rs. 3,000
18. A person travelled a distance of 68 km by an auto- initially but increases it to 4,000 after 4 months.
rickshaw, a car and a bullock-cart in the ratio of 5 : 10 Jagdish invests Rs. 7,000 initially, but takes Rs. 4,000
: 2. The fares of these vehicles per kilometer are in the back after 8 month. If their total profit at the end of
ratio of 2 : 3 : 5 then how much money he paid to car the year is Rs. 34,400 then what are Jagdish’s
vehicle if he totally spend Rs. 400? dividend?
(a) Rs. 240 (b) Rs. 150 (a) Rs. 13,600 (b) Rs. 10,000
(c) Rs. 340 (d) Rs. 300 (c) Rs. 15,000 (d) Rs. 12,000
19. The price of diamond is directly proportional to the 27. Ram, Krishna and Ganesh started a business by
square of its weight. A diamond broke into 4 pieces investing capital in the ratio 5 : 4 : 3. Three months
in the ratio of 1 : 2 : 3 : 4. The broken pieces got sold in 2
Rs. 2,10,000 less. What was the actual price of the later Ram took back of his capital; four months
5
diamond?

82 @BEST300MCQ For More Study Material


Visit: studyiq.com
34. A and B enter into partnership with capitals in the
1
later Krishna took back part of his capital while ratio 5 : 6. At the end of 8 months A withdraws his
4 capital.They received profits in the ratio 5 : 9. Then
1 for how much time, B remained in business?
Ganesh invested part of his initial capital again. (a) 6 months (b) 8 months
3
After 9 months there was a total profit of Rs. 19,200 (c) 10 months (d) 12 months
in the business. How much the maximum profit? 35. A, B and C started a business investing Rs 42000, Rs
(a) Rs. 6000 (b) Rs. 6600 30000 and Rs 28000, respectively. After 4 months, A
(c) Rs. 5000 (d) Rs. 1200 withdrew Rs 12000, B withdrew Rs 6000 and C
28. A and B started a business after investing Rs. 12,000 withdrew Rs 8000. If, after 10 months, a total profit of
and Rs. 14,400 respectively. At the end of the year B Rs 46420 is earned, what is the share of C?
gets 12% of the total profit as management fees and (a) Rs 12580 (b) Rs 13160
the rest amount is distributed between both the (c) Rs 13020 (d) Rs 12760
partners. If total annual profit is Rs. 7,500, then find 36. An amount of money is to be divided among P, Q and
the difference between their profits. R in the ratio of 3 : 5 : 7, respectively. If the amount
(a) Rs. 1400 (b) Rs. 1033 received by R is Rs 4,000 more than the amount
(c) Rs. 1500 (d) Rs. 1405 received by Q, what will be the total amount received
29. In a joint family the capitals invested by Laxmi and by P and Q together?
Saraswati are in the ratio 7 : 5 and the ratio of their (a) Rs 8,000 (b) Rs 12,000
profits is 4 : 3. If Laxmi’s capital was invested for 20 (c) Rs 16,000 (d) Cannot be determined
months then for how much time Saraswati’s capital
37. A cat takes 5 leaps for every 4 leaps of a dog but 3
would have been invested ?
leaps of the dog are equal to 4 leaps of the cat. What
(a) 21 months (b) 40 months is the ratio of the speeds of the cat to that of the dog?
(c) 30 months (d) 15 months (a) 11 : 15 (b) 15 : 11
30. If p : q = r : s = t : u = 2 : 3 then find the value of (mp +
(c) 16 : 15 (d) 15 : 16
nr + ot) : (mq + ns + ou)
38. Nandita scores 80% marks in five subjects together,
(a) 1 : 3 (b) 1 : 2
viz, Hindi, Science, Mathmatics, English and Sanskrit,
(c) 2 : 3 (d) 3 : 2 where in the maximum marks of each subject were
31. If 2 : (1 + 3) :: 6 : x , then what is the value of x? 105. How many marks did Nandita score in Science,
if she scored 89 marks in Hindi, 92 marks in Sanskrit,
(a) 3 + 3 (b) 3 – 3 98 marks in Mathmatics and 81 marks in English?
(a) 60 (b) 75
(c) 3 3 (d) 2 + 3 (c) 65 (d) 70
32. Vinay started a business investing Rs 50000. After 1 39. Out of 120 applications for a post, 70 are males and
year, he invested another Rs 30000 and Aditya also 80 have a driver’s license. What is the ratio between
joined him with a capital of Rs 70000. If the profit the minimum to maximum number of males having
earned in 3 years from the starting of business was driver’s license?
Rs 87500, find out the share of Aditya in the profit? (a) 1 : 2 (b) 2 : 3
(a) Rs 37500 (b) Rs 32500 (c) 3 : 7 (d) 5 : 7
(c) Rs 38281 (d) None of these 40. In a factory, the ratio of the numbers of employees of
33. Average score of Rahul, Manish and Suresh is 63. three types A, B and C is 9 : 13 : 18 and their wages
Rahul’s score is 15 less than Ajay and 10 more than are in the ratio of 10 : 7 : 4. If number of employees of
Manish. If Ajay scored 30 marks more than the type C is 54 and wages of every employee of type B is
average scores of Rahul, Manish and Suresh, what is Rs 1400, then find the total wages of all the employees
the sum of Manish’s and Suresh’s scores? of type A?
(a) 120 (b) 111 (a) Rs 51000 (b) Rs 54000
(c) 117 (d) Cannotbe determined (c) Rs 56000 (d) Rs 57000

83 @BEST300MCQ For More Study Material


Visit: studyiq.com
S olut i ons 3
´ 1000 = 375 Þ 32% of æç th of 1000÷ö
3
6. (c);
8 è8 ø
1. (d); Let present ages of Ramu and Somu be R and S
respectively. 32 ´ 375
= 32% of 375 = = 120
(R – 1) : (S – 1) Û 6 : 7 100
One year ago ® ­ : ­ 7. (c); Let the third number be n.
R : S n 6n
¯ : ¯ Four years hence 20% more than n = n + =
5 5
(R + 4) : (S + 4) Û 7 : 8
n 3n
50% more than n = n + =
R-1 6 R+4 7 2 2
= =
S-1 7 S+4 8 6n 6n 2
3n
7R - 7 = 6S - 6 8R + 32 = 7S + 28 Ratio of and Þ ´ =4:5
5 2 5 3n
7R - 6S = 1 8R - 7S = -4 Shortcut
7R – 6S = 1 ... (i) Let the third number be 100
8R – 7S = – 4 ... (ii) Then the two numbers are,
Multiplying equation (i) by 8 and equation (ii) by 20% more and 50% more
7, we get ® ß ß
56R – 48S = 8 ... (iii) 120 150
56R – 49S = – 28 ... (iv) Ratio = 120 : 150 = 4 : 5
Subtract (iv) from (iii) : S = 36 years 8. (c); A : B : C : D Þ 5 : 2 : 4 : 3
Shortcut: Þ C gets 1 unit more than D.
One year ago ® R : S Û 6 : 7 Þ S = 7 Given, C gets Rs. 1000 more than D.
Only option (d) has a number from which, if we Þ 1 unit = Rs, 1000 Þ B’s share = 2 units
subtract ‘1’, we get a number divisible by 7. = 2 × 1000 = Rs. 2000
9. (b); 0.75 : x :: 5 : 8
33 ´ A 55 ´ B
2. (b); 33% of A = = = 55% of B 0.75 5
100 100 = Þ 5x = 6.00 Þ x = 1.2
x 8
33 ´ A 55 ´ B A 55 Shortcut
= = Þ =
100 100 B 33 0.75 : x :: 5 : 8 Þ 0.75 ® 5 Þ 1® 0.15 Þ 8 ® 1.20
A:BÛ5:3 10. (b); Three numbers = I, II, III
3. (a); 68% of two fifth of 550 I + II + III = 98, I : II ® 2 : 3, II : III ® 5 : 8
10 : 15 Ü (2 : 3) × 5 ¬ (I : II) × 5
æ2 ö
68 ´ ç ´ 550÷ 15 : 24 Ü (5 : 8) × 3 ¬ (II : III) × 3
è5 ø 68 ´ 220 748
= = = = 149.6 I : II : III, 10 : 15 : 24, 10 + 15 + 24 = 49
100 100 5
Given I + II + III = 98, 98 ÷ 49= 2
4. (c); Let, number is ‘n’ Þ 1 unit = 2
æ 45n ö 9n Þ Second number = 15 × 2 = 30
çè ÷ø - 24 = 48 Þ 20 - 24 = 48
100 1 2 3
11. (c); : :
9n – 480 = 48 × 20 Þ 9n = 960 + 480 2 3 4
1440 3 3 L.C.M. of 2, 3 and 4 = 12
n= = 160 Þ n = ´ 160 = 60 On multiplying by a number throughout, the ratio
9 8 8 remains same. Here, we multiply by 12.
5. (a); 30% of the number = 190.8
1 2 3
190.8 ´ 12 : ´ 12 : ´ 12
1% of that number = 2 3 4
30 6 : 8 : 9 Þ 6 + 8 + 9 = 23
1908 23 units = 872 Þ 1 unit = 37.91
175% of the number = ´ 175 = 159 × 7 = 1113
300 Þ 1st part = 6 units = 6 × 37.91= 227.46

84 @BEST300MCQ For More Study Material


Visit: studyiq.com
12. (b); Fourth proportional to 5, 8, 15
5a + 3b 23
5 : 8 :: 15 : ? 15. (c); =
2a - 3b 5
5 15 Dividing numerator and denominator by ‘b’
= Þ x = 24
8 x
Shortcut æ aö
5ç ÷ + 3
è bø 23
´3 =
5 ¾¾® 15 æ aö 5
2ç ÷ - 3
8 ¾¾® 24 è bø
´3
13. (b); Let the two numbers be a and b a 5x + 3 23
Let =x Þ =
b 2x - 3 5
a 3
= Þ 5a = 3b ... (i) 25x + 15 = 46x – 69 Þ 21x = 84
b 5
a
a-9 12 x=4 Þ =4 Þ a:b Û 4:1
= Þ 23a – 207 = 12b – 108 b
b-9 23 16. (c); P : Q = 8 : 15
Þ 23a – 12b = 99 ... (ii) P:Q:R=?
5a Q : R = 3 : 2) × 5 Þ Q : R = 15 : 10
b= From (i) ... (iii) P : Q = 8 : 15 Þ 15 : 10 = Q : R
3
P : Q : R = 8 : 15 : 10
Using (ii) and (iii) 17. (a); P : Q = 8 : 15 ... (i)
5a Q : R = 5 : 8) × 3 Þ Q : R = 15 : 24 ... (ii)
23a - 12 ´ = 99 Þ 3a = 99 Þ a = 33
3 From (1) and (2)
P : Q : R = 8 : 15 : 24 ... (iii)
Shortcut
R : S = 4 : 5) × 6 Þ R : S Û 24 : 30 ... (iv)
Given = Two numbers are in the ratio
P : Q : R = 8 : 15 : 24 Þ 24 : 30 = R : S
=3:5
P : Q : R : S = 8 : 15 : 24 : 30 Þ P : S = 8 : 30 = 4 : 15
¯
18. (a); Let the fourth proportional to 4, 16, 7 be x.
Smaller number must be a multiple of 3
4 : 16 :: 7 : x
Only options (a) and (b) have ‘3’ as a factor.
Given : On subtracting 9 from each, the two 4 7
= Þ 4x = 16 × 7 Þ x = 4 × 7 = 28
numbers are in the ratio 12 : 23 16 x
On subtracting 9 from 27 ® option (a), 19. (b); Let the mean proportion be = r.
We get 18, but the ratio becomes 12 : 23, 9 r
9 : r :: r : 64 Þ =
So, option (a) ® discarded. r 64
On subtracting 9 from 33 ® option (b), r2 = 9 × 64 Þ r = 3 × 8 = 24
We get 24 20. (a); Duplicate ratio = ratio of squares
¯ Duplicate ratio of
multiple of 12. 2 : 7 = 22 : 72 = 4 : 49
21. (c); Sub-duplicate ratio = ratio of square roots
14. (d); Let the two numbers be a and b.
Sub-duplicate ratio of
2
a – (40% of a) = b 81 : 64 = 81 : 64 = 9 : 8
3
22. (b); Triplicate ratio = Ratio of cubes.
40a 2 60a 2 Triplicate ratio of
a- = bÞ = b 7 : 5 = 73 : 53 = 343 : 125
100 3 100 3
23. (a); Inverse ratio of 17 : 19 = 19 : 17
b 60 ´ 3 9 24. (b); Compound ratio of = 2 : 7, 5 : 3 and 4 : 7 will be
= =
a 2 ´ 100 10 2 × 5 × 4 : 7 × 3 × 7 = 40 : 147
Þ Ratio of the second number to the first number 25. (d); B:C=8:9
= 9 : 10 A : B = 3 : 4) × 2 Þ A : B = 6 : 8
8:9=B:C Þ A:B:C=6:8:9

85 @BEST300MCQ For More Study Material


Visit: studyiq.com
26. (d); a : b = 3 : 5) × 4 Þ a : b = 12 : 20 32. (d); 782
b : c = 4 : 7) × 5 Þ b : c = 20 : 35
Þ a : b : c = 12 : 20 : 35 Þ a : c = 12 : 35
27. (a); P : Q : R = 2 : 3 : 4 1 2 3
: :
P ® 2, Q ® 3, R ® 4 2 3 4
P 2 Q 3 R 4 L.C. M. of 2, 3 and 4 = 12
= , = , =
Q 3 R 4 P 2
æ 1 2 3ö
P Q R 2 3 4 æ2 3 4ö çè : : ÷ø ´ 12 Þ 6 : 8 : 9 Þ 6 + 8 + 9 = 23
2 3 4
: : = : : = ç : : ÷ ´ 12 = 8 : 9 : 24
Q R P 3 4 2 è3 4 2ø
782
a b a 3 23 units = 782 Þ 1 unit = = 34
28. (a); = Þ = 23
3 8 b 8
First part = 6 units = 6 × 34 = 204
a ® 3, b ® 8, a + 3 ® 6 33. (c); Let the two numbers be a and b
a+3 6 3 a:b=1:2
b + 8 ® 16 Þ = =
b+8 16 8 a 1
29. (a); 43 : 25 = ? = Þ 2a = b ... (i)
b 2
2 3 5
= (2 ) : 2 a+7 3
= Þ 5a + 35 = 3b + 21 ... (ii)
b+7 5
26 From (i) and (ii)
= 2 6 : 25 = Þ 2:1
25 5a + 35 = 3 × 2a + 21 Þ 5a + 35 = 6a + 21
30. (b); 2 : x :: 5 : 7 a = 14
Greatest number = b = 2a = 28
2 5 2 ´7
= ,x= Þ x = 2.80 Shortcut:
x 7 5
On adding 7 to both, ratio changes to 3 : 5
31. (c); Ratio of salaries ® A : B : C = 2 : 3 : 5
Þ On adding 7 to the greatest number, the
A ® 2x, B ® 3x, C ® 5x
resulting number must be a multiple of 5
15 ´ 2x 46x only option (c) has the number 28, in which, if we
A + 15% of A = 2x + =
100 20 add 7, we get 35, which is a multiple of 5.
3 : 5 :: ? : 35
3x ´ 10 33x
B + 10% of B = 3x + = ¯
100 10
21
5x ´ 20 Þ Smaller number = 21 – 7 = 14
C + 20% of C = 5x + = 6x
100 14 : 28 = 1 : 2
New ratio of their salaries 34. (c); Ratio of 3 numbers is 3 : 4 : 5
46x 33x Let the numbers be 3x, 4x, 5x
A:B:C= : : 6x Sum of their squares = 1250
20 10
9x2 + 16x2 + 25x2 = 1250 Þ 50x2 = 1250
¯ Multiply by 20
x2 = 25 Þ x = 5 Þ 3x + 4x + 5x = 12x = 60
46x : 66x : 120x Þ 23x : 33x : 60x Þ 23 : 33 : 60
35. (c); Let age of Sachin = S
Shortcut
Age of Rahul= R
Ratio of salaries of A, B and C ® 2 : 3 : 5
S=R–4 ... (i)
Let the salaries be
S 7
200 300 500 = Þ 9S = 7R ... (ii)
R 9
¯ 15% ­ ¯ 10% ­ ¯ 20% ­ From (1)
230 330 600 9S = 9R – 36 Þ 9S = 7R Þ 7R = 9R – 36
2R = 36 Þ R = 18
New ratio = 230 : 330 : 600 = 23 : 33 : 60
Age of Sachin = S = R – 4 = 18 – 4 = 14 years

86 @BEST300MCQ For More Study Material


Visit: studyiq.com
Shortcut: Anand’s present age must be a multiple of 4.
Sachin : Rahul Check option (a) = 24
S : R Sameer : Anand
7 : 9 ¯ ¯
5 : 4
Difference = 2 units = 4 years
¯ ×6 ¯×6
Þ 1 unit = 2 years
30 24
Sachin’s age = 7 units = 7 × 2 = 14 years ¯+3 ¯+3
36. (b); Arun : Deepak 33 : 27
A:D=4:3 c
Let A = 4x Þ D = 3x 11 : 9 Satisfied
After 6 years, Arun’s age = 4x + 6 39. (d); Ten years ago, Ratio of ages of Jayant, Prem
Deepak’s age = 3x + 6 and Saransh ® J : P : S
4x + 6 = 26 (given) 2:3:4
4x = 20 Þ x = 5 Let the ages be 2x 3x 4x
Deepak’s present age = 3x = 3 × 5 = 15 years.
Shortcut
¯ ¯ ¯
After 6 years, Arun’s age will be 26 years. Present ages ® (2x +10) (3x + 10) (4x + 10)
Total of the present ages = 93 years.
Þ Present age of Arun = 20 years
Þ 2x + 10 + 3x + 10 + 4x + 10 = 93
Ratio of present ages = Arun : Deepak
9x + 30 = 93 Þ x = 7
4 : 3
Þ Present age of Saransh = 4x + 10
¯×5 ¯×5 = 4 × 7 + 10 = 38 years.
20 years 15 years Shortcut:
37. (a); Present Ages Þ X : Y On subtracting 10 from the present age of
5:6 Saransh, we must get a multiple of 4, because
Let the ages be 5x and 6x ten years ago, the ratio of ages of Jayant,
¯ ¯ 7 yrs. hence Prem and Saransh was 2 : 3 : 4.
5x + 7 6x + 7 From the option ® (a) and (d) can be
discarded.
5x + 7 6
= -10
6x + 7 7 (a) 24 yrs ¾¾¾
® 14 yrs.
Þ 35x + 49 = 36x + 42 Þ x = 7 (not a multiple of 4.)
X’s present age = 5x = 5 × 7 = 35 years. -10
(b) 32 yrs. ¾¾¾
® 22 yrs.
Shortcut
(not a multiple of 4)
Present ages 5 : 6
Check option (c) 34 yrs.
¯+ 1 ¯+ 1 = 1unit = 7
10 years ago 34 – 10 = 24 yrs.
After 7 years 6 : 7
J : P : S
Þ X’s present age = 7 × 5 = 35 years 2 : 3 : 4
38. (a); Ratio of present ages ® Sameer : Anand ¯×6 ¯×6 ¯×6
5 : 4 12 18 24
Let present ages be 5x and 4x ¯+10 ¯+10 ¯+10
Three years hence ® (5x + 3) and (4x + 3) 22 + 28 + 34 ¹ 93
Check (d) 38 yrs.
5x + 3 11 38 – 10 = 28 yrs.
= Þ 45x + 27 = 44x + 33 Þ x = 6
4x + 3 9 2 : 3 : 4
Anand’s present age = 4x = 4 × 6 = 24 ¯×7 ¯×7 ¯×7
Shortcut 14 21 28
Ratio of present age S : A ¯+10 ¯+10 ¯+10
5 : 4 24 + 31 + 38 =93 yrs.

87 @BEST300MCQ For More Study Material


Visit: studyiq.com
40. (b); Hitesh Ronnie 46. (c); Ram Karan Rohan
¯ ¯ Ratio of capitals 2 : 3 : 4
40 yrs. 60 yrs. Ratio of time 6 : 4 : 3
It is clear from option (b) that 10 yrs. ago Ratio of profits Ram Karan Rohan
their ages were 30 yrs. and 50 yrs., i.e., in ¯ ¯ ¯
the ratio 3 : 5 2×6 : 3×4 : 4 ×3
41. (b); Ratio of profits = ratio of investments 12 : 12 : 12
= 45000 : 30000 = 45 : 30 = 3 : 2 1 : 1 : 1
47. (d); Ravi Kavi
42. (a); Since the investments were made for the
same period of time, ratio of shares of profit Investments 8000 72000 (Rs.)
= ratio of investments = 60000 : 50000 = 6 : 5 æ Ratio of ö
çè Investments÷ø 8 : 72
5 5
Kishor’s share = ´ total profit = ´ 33000
11 11 1 : 9
= 5 × 3000 = 15000 Ratio of investments = Ratio of profits = 1 : 9
43. (d); Investments 48. (b); Srikant Vividh
¯ Investments 185000 225000
Anil Rs. ® 3300 Ratio 185 : 225
Nikhil Rs. ® 5000 37 : 45
Ratio of profit shares = ratio of investments Let the total profit be P
= 3300 : 5000 45
Ratio of profit shares = 33 : 50 Vividh's share = ´ P = 9000
45 + 37
¯ ¯
Anil Nikhil 45
´ P = 9000
Profit earned = 1660 Rs. 82
33 82 ´ 9000
Anil’s share = ´ 1660 = Rs. 660 P= = 82 × 200 = 16400
83 45
44. (b); Rahul invested for one year i.e, 12 months. 49. (d); Rajan Sajan
6 months later, Sanjay joined him. Investments ® Rs. 14200 Rs. 15600
So, Sanjay invested for 6 months only. Ratio ® 142 : 156
Rahul Sanjay c
71 : 78
(I) Investment ® 45000 30000
Total profit = Rs. 74500
(T) Time period® 12 months 6 months
¯ ¯ 71
´ 74500 = 35500
Rajan’s share =
Product ® 45000 × 12 30000 × 6 149
Ratio of profits = 45000×12 : 30000×6 = 3 : 1 50. (a); A B
45. (b); Ratio of investments = Ratio of profit shares Investments ® Rs. 4000 Rs. 16000
=6 : 5 : 8 c
¯ ¯ ¯ Ratio of investments 1 : 4
Anil : Nikhil : Sourabh Ratio of times periods 12 months : 9 months
Profit earned = Rs. 83,600 c
4 : 3
8 Ratio of profits : (1 × 4) : (4 × 3)
Saurabh’s share = ´ 83600 = 35,200 Rs.
19 =4 : 12
c
1 : 3

88 @BEST300MCQ For More Study Material


Visit: studyiq.com
1. (b); Let the ten’s digit and the unit’s digit of the 5. (d); Ravi : Sumit
number be x and y respectively. 2 : 3
The number is ® 10x + y Let ® 2x , 3x
on interchanging digits ® 10y + x
+4000 ¯ ¯ +4000
x : y = 1 : 2 ® y = 2x
(2x+4000) (3x+4000)
Þ 10x + y = 12x

10y + x = 21x
]® éëdifference = 36 (given )ùû 2x + 4000
3x + 4000
=
40
57
114x + 228000 = 120x + 160000
21x – 12x = 36
9x = 36 Þ x = 4 68000
6x = 68000 Þ x =
Þ The number is 12x 6
Þ 12x = 12 × 4 = 48 68000
Sumits’s salary = 3x = 3 × = Rs. 34000
on interchanging ® 21x = 21 × 4 = 84 6
Sum of digits = x + y = 12 6. (c); A B C
difference of digits = y – x = 4 Ratio of salaries 2 : 3 : 5
difference of sum and difference of digits Let the salaries be 200 300 500
= 12 – 4 = 8
2. (a); Maths Phy. Bio.
¯ 15% ¯ 10% ¯ 20%
¯ ¯ ¯ 230 330 600
500 700 800 New ratio ® 23 : 33 : 60
7. (d); 25p, 10p and 5p coins are in the ratio of 2 : 3 : 4.
40%inc ¯ ¯ 50%inc ¯ 75%inc So, number of 25p coins = 2 units
700 1050 1400 Its value = 25 × 2 = 50 units
New ratio = 70 : 105 : 140 = 2 : 3 : 4 number of 10 p coins = 3 units
Its value = 3 × 10 = 30 units
3. (b); 60 litres number of 5p coins = 4 units
Its value = 4 × 5 = 20 units
3 units = 60 L Total value = 50 + 30 + 20 = 100 units = 50 Rs.
milk water
Þ 1 unit = 20 L
1
40 L 20 L 1 unit = Rs. Þ 1 unit = 50p
2
1 : 3] × 2 (Multiply by 2, to keep milk constant) value of 5p coins = 20 units = 20 × 50 p.
¯ 20 ´ 50
Þ Number of 5p coins = = 200
2:6 5
¯ 8. (d); sum 1 sum 2
6 units = 6 × 20 = 120 L
So, 120 – 20 = 100 L
¯ C : A : B M N
4 : 5 : 6 1 : 1
quantity of water to be added. 4x : 5x : 6x y y
4. (c); Boys Girls 6x - y = 2000 4x = ?
¯ ¯ ¯
7 : 8 Can’t determine x from this equation
Let ® 700 800 9. (b); A : B : C Þ 12 : 15 : 25
Let ® 12x, 15x, 25x
20%inc ¯ ¯ 10% inc Sum = 52x = 364
840 880 364
x= Þ x=7
New ratio = 840 : 880 = 21 : 22 52

89 @BEST300MCQ For More Study Material


Visit: studyiq.com
A B C 14. (a); Let ® A¢s share be A
B¢s share be B and C¢s share be C
12 × 7 15 × 7 25 × 7 366

84 105 175
A B C
A + B + C = 336 ... (i)
diff diff 1
A= (B + C)
2
2A = B + C ... (ii)
21 : 70 = 3 : 10
From (1) and (2)
625 x A + 2A = 366 Þ A = 122
10. (b); =
x 1156 A¢ share = Rs. 122
x2 = 625 × 1156 Þ x2 = (25)2 × (34)2 15. (d); Data inadequate
x = 25 × 34 = 850 16. (d); Ratio ® 1 : 3 : 2
11. (c);
Let x 3x 2x
Alcohol : Water
Inital ratio
4 : 3 2 unit increase = 5 L +25 +25 +25
New ratio 4 : 5 1 unit = 2.5 L
(x + 25) (3x + 25) (2x + 25)
Alcohol in initial mixture = 4 units
= 4 × 2.5 = 10 L x + 25 + 3x + 25 + 2x + 25 = 735
6x + 75 = 735 Þ 6x = 660 Þ x = 110
12. (c); 25p 10p 5p Money received by C = 2x + 25
1 : 2 : 3 = 2 × 110 + 25 = Rs. 245
17. (d); Ratio of remainders A B C
3 : 4 : 5
1×25 2×10 3×5
Let ® 3x 4x 5x
25p 20p 15p Sum of original shares ® (3x + 5) + (4x + 10) + (5x
+15) = 2430 Þ 12x + 30 = 2430
Total = 60 units 12x = 2400 Þ x = 200
= 30 Rs = 3000 p B’s share = 4x + 10 = 810 Rs.
18. (d); Gold Copper
1 unit = 50p
value of 5p coins = 15 units = 750p 19 9

750
Number of 5p coins = = 150 15
5
13. (b); Let C’s share be ® C
6 4
1 =3:2
Then, B’s share = C
4
19. (b); 15 litres
2 1 1
A’s share = ´ C = C
3 4 6
C C Alcohol Water
Total = Rs. 510 Þ + + C = 510
6 4
20% 80%
2C + 3C + 12C
= 510 Þ 17C = 12 × 510
12 3 litre 12 litre
C = 12 × 30 = 360
+3 litre
So, C’s share = Rs. 360
B’s share = Rs. 90 3 litre + 15 litre = 18 litre
A’s share = Rs. 60
Shortcut: only option (b) satisfies the given 3 50 2
alcohol = ´ 100 = % = 16 %
conditions. 18 3 3

90 @BEST300MCQ For More Study Material


Visit: studyiq.com
20. (a); 85 L
K
24. (d); Let, x =
y2
[ K = cons tan t ]
milk water
27 : 7 Þ 34 units = 85 L K
1= Þ K=4 Þ x= 4
85 5 4 y2
Þ 1 unit = L= L
34 2
New ratio ® [3 : 1] × 9 [because. water is added] 4 1
y=6Þx= =
New ratio ® 27 : 9 36 9

Milk Water 25. (a); Let the fixed amount be Rs. x and the cost of each
27 : 7 unit be Rs. y
2 units increase Then, 500y + x = 1680 ... (i)
27 : 9 600y + x = 1980 ... (ii)
5 Subtract (i) from (ii)
= 2´ = 5 L water
2 100y = 300 Þ y = 3
Putting y = 3 in (i)
æ 1 1 1ö
21. (b); Ratio of sides ® ç : : ÷ ´ 12 = 6 : 4 : 3 1500 + x = 1680 Þ x = 180
è 2 3 4ø
So, fixed charges = Rs. 180
perimeter = 6 + 4 + 3 = 13 units = 104 cm
Charge per unit = Rs. 3.
1 unit = 8 cm Total chargers for Consuming 700 units
Longest side ® 6 units = 6 × 8 = 48 cm = 700 × 3 + 180 = 2100 + 180 = 2280
22. (c); 26. (c); A B
boys girls
Let 300 200 Income ® 5x 4x
Expenditure ® 3y 2y
scholarship holders 20% 25% Saving ® (5x – 3y) (4x – 2y)
5x – 3y = 1600) × 2 4x – 2y = 1600) × 3
60 50 10x – 6y = 3200 (1) 12x – 6y = 4800 (2)
Students who got scholarship ® 60 + 50 = 110 From (1) and (2) 2x = 1600 Þ x = 800
students who did not get scholarship A’s income = 5x = 5 × 800 = 4000 Rs.
® 500 – 110 = 390 27. (c); Gold : copper
390 A®7:2]×2
Percentage of student = ´ 100 = 78% A ® 14 : 4 ® total = 14 + 4 = 18
500
23. (c); Change the ratios into fractions : B ® 7 : 11 ® total = 7 + 11 = 18
Containers Milk Water Volume of 18 units of A and 18 units of B are melted to
containers form C
4 1 3 1 Gold Copper
I = ¯ ¯
5 5 12 4
C ® (14 + 7) : (4 + 11)
3 1 4 1
II = 21 : 15
4 4 12 3
Ratio ® 7 : 5
5 2 5 28. (a); Let present ages be ® P Q
III
7 7 12 ® 5x 7x
Ratio of milk and water in the fourth container is: P’s age after 6 yrs. = 5x + 6
4 1 3 1 5 5 1 1 1 1 2 5 7x– (5x + 6) = 2 Þ 2x – 6 = 2
´ + ´ + ´ : ´ + ´ + ´
5 4 4 3 7 12 5 4 4 3 7 12 2x = 8 Þ x = 4
1 1 25 1 1 5 314 106 So, present ages of P and Q are 20 years & 28
= + + : + + = :
5 4 84 20 12 42 420 420 years respectively.
= 314 : 106 = 157 : 53 Total = 48 years

91 @BEST300MCQ For More Study Material


Visit: studyiq.com
29. (b); Let the present ages be ® Father Son 34. (d); Let present ages of father and son be f and s
¯ ¯ respectively.
7x 3x Father Son
7x × 3x = 756 Þ 21x2 = 756 Present ages f s
x2 = 36 Þ x = 6 4 years ago (f – 4) (s – 4)
Þ Present ages ® father ® 7 × 6 = 42 yrs. After 4 years (f + 4) (s + 4)
son ® 3 × 6 = 18 yrs. f – 4 = 3(s – 4) (f + 4) + (s + 4) = 64
After 6 years ® father ® 48 years. f – 4 = 3s – 12 f + s = 64 – 8
son ® 24 yrs. f – 3s = –8 ...(i) f + s = 56 ...(ii)
Ratio ® 48 : 24 = 2 : 1 3f + 3s = 168
30. (b); 8 years ago, the sum of their ages was 56 years. f – 3s = –8
So, sum of their present ages = 56 + 24 = 80 years On solving (i) and (ii)
Proportion ® 4 : 7 : 9 4f = 160 Þ f = 40
So, father’s age at present = 40 years.
4 35. (a); Usha Reena
So, the present ages are ® ´ 80 = 16 Years
20 Present ages ® 36 yrs. 24 yrs.
7 ¯ –8 ¯ –8 (8 yrs. ago)
´ 80 = 28 Years
20 28 yrs. 16 yrs.
9 Ratio ® 28 : 16 (Usha : Reena)
´ 80 = 36 Years
20 =7:4
36. (b); Ratio of present ages
31. (c); Ratio of present ages = 4 : 3 ¯
Let the ages be ® man ® 4x
Anil : Poorvi
wife ® 3x
3 : 2
After 4 yrs ® man ® 4x + 4
Let ® 3x 2x
wife ® 3x + 4
¯ ¯
4x + 4 9 After 8 yrs. ® (3x + 8) (2x + 8)
=
3x + 4 7
3x + 8 25
28x + 28 = 27x + 36 Þ x = 8 = Þ 54x + 144 = 50x + 200
Present ages ® man ® 32 yrs. 2x + 8 18
wife ® 24 yrs. 4x = 56 Þ x = 14
From the option if we check option (c) ® 12 yrs. Þ Poorvi’s present age = 2x = 28 yrs.
then 12 yrs. ago ® man ® 20 yrs. Shortcut :
wife ® 12 yrs. Check ® option (b)
Ratio ® 20 : 12 = 5 : 3 Poorvi’s present age = 28 yrs.
So, they were married 12 years. ago. 3
32. (c); A B Þ Anil’s present age = ´ 28 = 42 yrs.
2
Present ages ® 5x 3x After 8 yrs. ®
A’s age 4 yrs. ago = 5x – 4 Anil ® 50 yrs.
B’s age 4 yrs. hence = 3x + 4 Poorvi ® 36 yrs.
5x – 4 = 3x + 4 Þ 2x = 8 Þ x = 4 ratio ® Anil : Poorvi
A’s age 4 yrs. hence = 5 × 4 + 4 = 24 yrs. 50 : 36
B’s age 4 yrs. ago = 3 × 4 – 4 = 8 yrs. 25 : 18
Ratio ® 24 : 8 = 3 : 1 Hence verified
33. (c); Let mother’s present age be ‘M’ years
37. (c); Ratio of ages
2
present age of the person = M
5 Meena : Seema
After 8 yrs. ® mother’s age = M + 8 7 : 8
2
person’s age = M+8
5
2 M M difference = 1 unit = 3 yrs.
M + 8 = + 4, = 4 Þ M = 40
5 2 10 Sum = 7 + 8 = 15 units = 15 × 3 = 45 years.

92 @BEST300MCQ For More Study Material


Visit: studyiq.com
38. (a); Rehana’s age after 7 yrs. = 85 yrs. 41. (c); Age of A and B 10 years ago A B
Þ Rehana’s present age = 85 – 7 = 78 yrs. 13x 17x
Wasim is ® 12 younger than Rehana
Ratio of ages of A and B after 17 years
Þ Wasim’s present age = 66 yrs
Ratio of present ages 13x + 27 10
=
Manoj : Wasim 17x + 27 11
3 : 11 143x + 297 = 170x + 270
×6 ×6 27x = 27, x = 1
18 yrs. 66 yrs. Present age of B = 17x + 10
= 17 × 1 + 10 = 27 years
Present age of Manoj's 42. (d); Can’t be determined.
father = 18 + 25 = 43 yrs. 43. (c); Ram Sanjay Aditya
39. (b); Ratio of present ages ® ¯ ¯ ¯
Indira : Lizzy 45000 60000 90,000
3 : 8 12 months 9 months 6 months
Let ® 3x 8x
Ram Sanjay Aditya
After 8 yrs. ® ¯ ¯ ¯

3x + 8 8x + 8 Ratio of Profits ®
3x + 8 = 20 Þ 3x = 12 Þ x = 4 45000×12 : 60000×9 : 90000×6
Þ Lizzy’s present age Þ 8x = 8 × 4 = 32 yrs. 5400 : 5400 : 5400
Þ Lizzy’s age 5 yrs. ago Þ 32 – 5 = 27 yrs. 1 : 1 : 1
Shortcut :
Indira : Lizzy Net profit = Rs. 16500
3 : 8 1
Indira’s age after 8 yrs. = 20 yrs. Ram’s share = ´ 16500 = Rs. 5500
3
Þ Indira’s present age = 12 yrs.
3 units ® 12 years 44. (c); Ram Anil Aditya
Þ 1 unit = 4 years ¯ ¯ ¯
Lizzy’s present age = 8 units = 8 × 4 = 32 yrs. 1200 1500 900
Lizzy’s age 5 yrs. ago = 32 – 5 = 27 yrs.
12 months 6 months 9 months
40. (d); Let Sarita’s present age = s
Kavita’s present age = 2s Ram Anil Aditya
8 years. hence ¯ ¯ ¯
Sarita’s age = s + 8 Ratio of profits 14400 : 9000 : 8100
Kavita’s age = 2s + 8
144 : 90 : 81
2s + 8 22
= 16 : 10 : 9
s+8 13
Total profit = Rs. 2450
26s + 104 = 22s + 176
4s = 72 9
s = 18 Þ Kavita’s present age = 2s = 36 years Aditya’s share = ´ 2450 = Rs. 630
35
Shortcut
Check option (d) 45. (d); Let number of 10-paise coins be 17 k
Kavita’s present age = 36 yr. and number of 25-paise coins be 6 k.
Sarita’s present age = 18 yrs. 17 k × 0.10 + 6 k × 0.25 = 112
Sarita Kavita
Þ 1.7 k + 1.5 k = 112
18 36
¯ +8 ¯+8 Þ 3.2 k = 112
26 44 Þ k = 35
Ratio = 13 : 22 \ Number of 10-paise coins = 17 × 35 = 595
Hence verified

93 @BEST300MCQ For More Study Material


Visit: studyiq.com
46. (c); P Q S 49. (d); Total time = 36 months
¯ ¯ ¯ Let Aditya’s time of contribution = tA
(50, 4 months) (45, 5 months) (70, 7 months) Manish’s time = tM
300 × tA : 500 × tM = Ratio of their profit shares.
(25, 8 months) (22.5, 7 months)
Total profit = Rs 1020
P Q S Aditya’s share = 495
¯ ¯ ¯ Manish’s share = 1020 – 495 = 525
Ratio (50 × 4+ 25 × 8) : (45 × 5 + 22.5 × 7) : (70 × 7) Ratio of profits = 495 : 525 = 33 : 35 ... (ii)
of profits From (1) and (2)
400 : 382.5 : 490 (× 0.4) 300 ´ tA 33 tA 33 ´ 500 11
= Þ = =
160 : 153 : 196 500 ´ t M 35 t M 35 ´ 300 7
Profit at the end of 12 months = Rs 1272.5 Time period for which Aditya invested
196
S’s share = ´ 1272.5 11
160 + 153 + 196 = ´ 36 = 22 months
18
196 196 ´ 12725 50. (a); Aditya Manish Gaurav
= ´ 1272.5 = = Rs. 490
509 5090 7 4 6
47. (c); A invests Rs. 12000 for 4 months and Rs. 17000 Ratio of shares : :
2 3 5
for 8 months B invests Rs. 16000 for 4 months
and Rs. 10000 for 8 months. C invests Rs. 20000 7 4 6
´ 30 : ´ 30 : ´ 30
for 6 months. 2 3 5
A ® 12000 × 4 + 17000 × 8 = 1, 84000 (105 : 40 : 36) × 2
210 : 80 : 72
B ® 16000 × 4 + 10000 × 8 = 1, 44000
Let their shares be ® 210, 80, 72
C ® 20000 × 6 = 1,20,000
\ Ratio of profits = 23 : 18 : 15 Aditya Manish Gaurav
Total profit = Rs. 30,100
Shares 210 (210+105) 80 72
23
As share = ´ 30,100 = Rs. 12362.5
(23 + 18 + 15) For 4 For 8 For 12 For 12
48. (a); P Q R months months months months
Ratio of investments 8 : 6 : 5
Ratio of profits 1 : 3 : 5 Ratio of Aditya Manish Gaurav
Check option (a) ® Ratio of time periods profit shares
® 1:4:8 (210×4+315×8) (80×12) (72×12)
Ratio of profits = Ratio of products of
corresponding values of investments and time Ratio of profit (840+2520) 960 864
periods
960 864
8×1 : 6×4 : 5×8 3360
= 8 : 24 : 40
Profit at the end of one year = Rs. 43200
= 1 : 3 : 5 = Ratio of profits.
960
Manish’s share = ´ 43200 = Rs. 8000
5184

94 @BEST300MCQ For More Study Material


Visit: studyiq.com
1. (c); Alloy 1 Alloy 2 Expense is increased by 60%

1 2 8
Increased expense = 5 ´ = 8 unit
3 5 5
3 New saving = 10 – 8 = 2 units
8 Difference between saving = 3 – 2 = 1 unit
1 unit = Rs. 3500
2 3 1 3 1 1 10 units = Rs. 35000
- = - =
5 8 40 8 3 24 6. (a); Let the numbers of Rs. 1, 50 paise & 25 paise coins
are 12x, 10x and 7x.
1 1 According to question
Required Ratio = : = 24 : 40 = 3 : 5
40 24
3 and 5 parts of the two alloys must be taken.
10x æ xö 7x
12x + + 7 ç ÷ = 75 Þ 12x + 5x + = 75
2. (d); Quantity of gold and silver in first alloy, 2 è 4ø 4
Gold = 8 kg, silver = 2kg
7x 75x
In second alloy 17x + = 75 Þ = 75, x = 4
4 4
Gold = 4 kg, silver = 12 kg
Total amount of gold in new alloy = 8 + 4 + pure Number of 25 paise coins = 7 × 4 = 28 coins
gold added 7. (a); Given, 2 Horses = 5 Dogs
Total amount of silver in new alloy = 12 + 2 = 14 kg 6 Dogs = 8 Oxens
Ratio given = 3 : 2 10 Oxens = 50 Sheep
2x = 14 kg Þ x = 7 kg 14 Sheep = 9 Goats
Total weight of new alloy = 5 × x = 5 × 7 = 35 kg
9 ´ 50 ´ 8 ´ 5
B+C B+C 2 A+C A+C 4 Cost of one horse = ´ 700 = Rs. 7500
3. (b); A = , = Þ = B, = 14 ´ 10 ´ 6 ´ 2
2 A 1 4 B 1
8. (c); Let their share after decrease in money = 9x, 13x
B + C 2 ´ 5 10 A + C 12 and 8x
= = Þ =
A 1´ 5 5 B 3 (9x + 26) + (13x + 28) + (8x + 32) = 2186
B = 3, A = 5, C = 7 Þ A : B : C = 5 : 3 : 7 30x = 2186 – 26 – 28 – 32 Þ x = 70
8 Amount given to A = 9 × 70 + 26
(A + B)’s share = ´ 5625 = Rs. 3000
15 = 630 + 26 = Rs. 656
8 ´ 2310 + 4 ´ 1800 - 1680 Sita's Profit 3
4. (d); Average income = 9. (a); =
12 Gita's Profit 2
24000
= = 2000 Let Gita invested capital for x months and Sita
12 invested her capital for 12 months.
8 ´ 2310 + 4 ´ 1800
Avg. Expenditure = 3 12 12
12 = Þ x= ´ 2 = 8 Þ x = 8 months
2 x 3
25680
= = 2140
12 LM 3
10. (a); Last year, ratio of cost of Maruti and Figo, =
Average Income 2000 100 LF 4
= = = 100 : 107
Average Expenditure 2140 107 PM 5
Present year & last year ratio of maruti, =
Expense 5 LM 4
5. (a); Given, =
Saving 3 5 5 15 PF 3
PM = LM, PM = ´ 3 = Þ =
Let Income = 8 units 4 4 4 LF 2
5 3 3
Increased income = 8 ´ = 10 units PF = LF Þ PF = ´ 4 = 6
4 2 2
95 @BEST300MCQ For More Study Material
Visit: studyiq.com
15 1
PF : PM = 6 : = 24 : 15 = 8 : 5 Mohan Saving = 50% of z = z
4 2
8 Let their saving be 9k, 8k and 4k then
PF = ´ 7.8 = 4.8 lacks
13 3 4
x = 9k, x = 12k, Þ y = 8k, y = 10k
4 5
LF 2 2
= , LF = ´ 4.8 = 3.2 lacs
PF 3 3 1
z = 4k, z = 8k
11. (b); Ratio of Ram’s salary for last and present year = 2
2:3 According to question
Ratio of Shyam’s salary for last and present year x + y + z = 450 Þ 12k + 10k + 8k = 450
=4:5
Ratio of their salary during last year = 3 : 5 450
k= = 15
Now, we change the first two ratio 30
2´ 3 3´ 3 9 Ram’s income = 12 × 15 = Rs. 180
2:3= : =3: 14. (a); After taking out 100 litres of milk and adding
2 2 2
same amount of water.
æ 5ö æ 5ö 25
and 4 : 5 = 4 ç ÷ : 5 ç ÷ = 5 : 900
è 4ø è 4ø 4 Ratio of milk and water = =9: 1
100
Now the ratio of salary of present year
After taking out 200 litres of mixtures, then the
9 25
= : = 18 : 25 æ 9 ö
2 4 Quantity of milk = 800 ç ÷ = 720 litres
è9+1ø
18
The Present salary of Ram = ´ 8600
18 + 25 720 18
The new ratio = = = 18 : 7
= Rs. 3600 280 7
12. (d); Let the candle height be x After taking out 400 litres of mixture the rest
Let time required = T hour amount of mixture will be 600.
x
In one hour first candle lighted = æ 18 ö
8 Quantity of pure milk = 600 ç ÷ = 24 × 18 = 432
è 25 ø
T litre
In T hour, first candle lighted = x
8 The amount of pure milk will be 432 litres.
æ Tö 15. (b); Let the land and water part on earth be x and 2x.
After T hour, remaining candle = x ç 1 - ÷ Total area = 3x
è 8ø
3
æ Tö Total area of northern hemisphere = x
Similarly for second candle = x ç 1 - ÷ 2
è 6ø
Land part of northern hemisphere
æ Tö æ Tö 2 3x 3x
xç1 - ÷ ç1- ÷ 2 = ´ =
è 8 ø 2 8ø
= , è = 2+3 2 5
æ Tö 1 æ Tö 1
xç1 - ÷ ç1- ÷ 3x 2x
è 6ø è 6ø Land part of southern hemisphere = x - =
5 5
T T T T Water part of northern hemisphere
1- =2- , - =1
8 3 3 8 3 3x 9x
= ´ =
24 2+3 2 10
T = = 4 h 48 min
5 Water part of southern hemisphere
13. (a); Income of Ram, Shyam and Mohan be x, y and z 9x 11x
respectively. = 2x - =
10 10
3
Ram’s Saving = 75% of x = x 2x
4
11
4 Required ratio = 5 = 2 : = 4 :11
Shyam’s Saving = 80% of y = y 11x 2
5 10

96 @BEST300MCQ For More Study Material


Visit: studyiq.com
16. (a); Income of A = Rs. 2400 a2 – 992 = 992 × 15 Þ a2 = 992 × 16 Þ a = 396 litres
7 99
Income of B = ´ 2400 = Rs. 5600 Actual concentration = ´ 100 = 20%
3 ( 396 + 99)
4 5 9
Income of C = ´ 2400 = Rs. 3200 19. (a); Here, B = A, C = D = B
3 6 10
Expenditure of A = 2400 – 300 = Rs. 2100
2 5 2
3 B= E and E – A = 3 Þ B = A = E
Expenditure of B = ´ 2100 = Rs. 1575 3 6 3
4
A 4 5 5
5 = or E = A Þ E – A = A – A
Expenditure of C = ´ 2100 = Rs. 2625 E 5 4 4
4
Saving of B = Rs. (5600 – 1575) = Rs. 4025 1
3= A, A = 12
Saving of C = Rs. (3200 – 2625) = Rs. 575 4
17. (b); Let the quantity of wine and water in drum are 3x E = 15 and B = 10, C = D = 9 and B = 10
and x respectively. C = D = 9 and F = 11, A : F = 12 : 11
and let y litre of mixture is taken out and y litres
W1 1
water is mixed. 20. (d); In first Vessel =
M1 3
In y litre of mixture,
3 1 W2 2
wine = y litre, water = y litre In second Vessel =
4 4 M2 3

3y W3 2
amount of wine in new mixture = 3x - litre In third Vessel =
4 M3 5
y 3y
and amount of water = x - +y = x+ 1 2 2
4 4 Proportion of water = : :
4 5 7
According to question
1 5 ´7 2 4 ´7 2 4 ´ 5
3y 3y 3y 4x ´ : ´ : ´
3x – = x+ Þ 2x = , y= 4 5 ´7 5 4 ´7 7 4 ´ 5
4 4 2 3
35 56 40
1 : :
Which is of the total mixture. 140 140 140
3
Now, since all these mixtures are mixed in ratio
18. (a); Petrol Kerosene Total mixture of 2 : 3 : 5.
99 a 99 + a
99 a – 198 a – 99 70 168 200
Therefore new ratio = : :
280 420 700
99 99
´ 100 - ´ 100 = 13.33 Amount of water = 70 + 168 + 200 = 438
a - 99 a + 99
Amount of milk = (280 + 420 + 700) – 438 = 962
æ 198 ö 962 481
9900 ç 2 = 13.33
è a - 992 ÷ø Ratio of milk to water =
438 219
= 481 : 219

97 @BEST300MCQ For More Study Material


Visit: studyiq.com
1. (b); A : B = 5 : 7 8. (d); Let the original salaries of Ravi and Sumit be 2x
B : C = 6 : 11 and 3x.
So, we have to quantity B which to be equal in 2x + 4000 40
both case. Then, =
3x + 4000 57
So, A : B = [5 : 7] × 6
3x = 34000
B : C = [6 : 11] × 7
Sumit's present salary = (3x + 4000) = 38000
So, A : B : C = 30 : 42 : 77
Shortcut: 2 1
9. (b); A = B, B = C
3 4
A B C
So, A : B : C = 2 : 3 : 12
5 7
6 11 2
A's share = ´ 510 = 60
17
So, A : B : C = 5 × 6 : 7 × 6 : 7 × 11
= 30 : 42 : 77 3
B's share = ´ 510 = 90
2. (d); Let A = 2x, B = 3x and C = 4x 17
A 2x B 3x C 4x 12
So, = , = and = C's share = ´ 510 = 360
B 3x C 4x A 2x 17
A B C 2 3 4 10. (c); Let Boys = 3x and girls = 2x
Þ : : = : : = 8 : 9 : 24 No. of students who do not get scholarship
B C A 3 4 2
39x
43.5 2( 2 ´ 3.5) 2 7 4 = (80% of 3x) + (75% of 2x) =
3. (b); = = 5 = 10
25 25 2 1
æ 39x 1 ö
x 2 x2 4 x2 + y2 4 + 1 Req. percentage = ç ´ ´ 100 ÷ % = 78%
4. (a); = , = Þ = è 10 5x ø
y 1 y 2
1 x2 - y 2 4 - 1
1
x2 - y2 3 11. (b); 10% of B = G
= 4
x2 + y2 5
B 5
5. (c); 19 (4x2 – 3y2) = 12 (2x2 + 5y2) =
G 2
52x2 = 117y2 Þ 4x2 = 9y2
12. (b); Let fixed amount = x
x 3 Cast of each unit = y
=
y 2 540y + x = 1800 ...(i)
6. (a); Let (a + b) = 6 k, (b + c) = 7k and (c + a) = 8k 620y + x = 2040 ...(ii)
then, 2 (a + b + c) = 21k from (i) and (ii)
4 x = 180, y = 3
2 × 14 = 21k Þ k = So, total charges for consuming 500 units
3
= (180 + 500 × 3) = 1680
4 13. (b); Let C gets Rs. x
So, (a + b) = 6 × =8
3
and B gets Rs. (x + 8)
c = (a + b + c) – (a + b) = 14 – 8 = 6 A gets Rs. (x + 15)
1 2 3 So, Sum of A + B + C = 53
7. (d); Given ratio = : : = 6:8:9
2 3 4 x + x + 8 + x + 15 = 53
x = 10
6
So, Ist part is = ´ 782 = 204 So, A : B: C = 25 : 18 : 10
23

98 @BEST300MCQ For More Study Material


Visit: studyiq.com
14. (c); Let the no. of 25p, 10p and 5p coins be x, 2x and 19. (a); Price µ (weight)2
3x respectively. P µ w2, P = k×w2
So, Sum of their values P = K×(10x)2
...(i)
æ 25x 10 ´ 2x 5 ´ 3x ö 60x Now, total price of broken pieces of diamond
=ç + + ÷ = Pn = K[x2 + (2x)2 + (3x)2 + (4x)2] = 30 K×x2 ...(ii)
è 100 100 100 ø 100
A.T.Q.
60x P — Pn = 2,10,000
= 30
100 K×100x2 – K×30×x2 = 2,10,000
So, x = 50 70×K×x2 = 2,10,000
So, the no. of 5p coins = 3 × 50 = 150 Price of unbroken diamond = 100K×x2
Shortcut: = 3,00,000
25p 10P 5P 300 a+b b+ c c+a a+ b+ b+c+ c+a
= 20. (a); = = =
No. of coins 1 : 2 : 3 6 5 6 7 5+ 6 +7
So, No. of 5p coins = 50 × 3 = 150 coins
2(a + b + c)
15. (b); From the question =
18
50 × 2K + 100 × 3K + 500 × 4K + 1000 × K = 34000
K = 10 a + b = 5x, a + b + c = 9x
So, No. of 500 notes = 4 × 10 = 40 b + c = 6x
and c + a = 7x So, a : b : c = 3 : 2 : 4
C 3 B 12
16. (a); From the Question = , = Now, a + b + c = 18
A + B 4 A + C 23
18
C 3 So, the value of x = =2
= 9
A+B 4 So, b = 2 × 2 = 4
So, the sum of both term of the ratio 21. (a); 2a = 3b
C+A+B=3+4=7
a 3 b 4
3 = , =
So, runs scored by C = ´ 1750 = 750 runs b 2 c 3
7
So, a:b:c=6:4:3
Similarly,
22. (a); The ratio of
12
and runs scored by B = ´ 1750 = 600 runs 5 3 3
12 + 23 1 rupees : 50 paise : 25 paise = ´ : :1
6 5 5
and runs scored by A = 1750 – 750 – 600 = 400 run
= 5 : 6 : 10
5 From the Question
17. (c); No. of employees in class - 1 = ´ 68 = 20
17
50 25
10 5K + ´ 6K + ´ 10K = 262.50
No. of employees in class - 2 = ´ 68 = 40 100 100
17 K = 25
2 So, the no. of 25 paise coin is = 250
No. of employees in class - 3 = ´ 68 = 8
17 male amount 4
So, Salary of each class employee be 2x, 3x and 23. (a); female amount = 3
5x.
So, 20 × 2x + 40 × 3x + 8 × 5x = 400000 female amount 6
x = 2000 and =
children amount 5
So, salary of employee of class - 3 is = 10000
18. (a); According to the Question So, ratio of male amount : female amount :
20 × 2x + 40 × 3x + 8 × 5x = 400 childeren = 8 : 6 : 5
x=2 from the question,
So, he spent 20 × 2 × 2, 40 × 3 × 2, 8 × 5 × 2 on 10 × 8K + 15 × 6K + 5 × 5K = 11700
those three vehicles respectively. K = 60
So, the spent on car = 240 So, each child get = 300

99 @BEST300MCQ For More Study Material


Visit: studyiq.com
24. (a); From the question
p r t 2 mp nr ot 2
30. (c); = = = Þ = = =
5x - 2500 10 q s u 3 mq ns ou 3
=
4x - 2500 7
x = 1500 mp + nr + ot 2 + 2 + 2 2
Þ = =
Shyam's income = 4 × 1500 = 6000 mq + ns + ou 3 + 3 + 3 3
25. (a); 1 cows = 3 calves
The ratio of the cattles of A, B and C. 2 6
=
=4×3+3:5×3+4:6×3+5
31. (a);
(1 + 3 ) x
= 15 : 19 : 23
Ratio of rents = 15 × 5 : 19 × 3 : 23 × 2 x = 3(1 + 3)
= 75 : 57 : 46
x= 3+ 3
2670 ´ 57
Rent of B = = 855 32. (d); Ratio of Vinay and Aditya for one month
75 + 57 + 46
= (50000 × 12) + (80000 × 24) : (70000 × 24)
The difference between rent of A and C
= 600000 + 1920000 : 1680000 = 3 : 2
75 - 46
= ´ 2670 = 435 Hence, share of Aditya in the profit earned from
75 + 57 + 46
87500
26. (a); The ratio of capitals of OM, Jai, Jagdish the business = ´ 2 = 35000
3+ 2
= 5000 × 12 : 3000 × 4 + 4000 × 8 : 7000 × 8 + 3000
×4 33. (b); Let the score of Ajay = x
= 15 : 11 : 17 Rahul = x – 15
17 Manish = x – 25
So, Jagdish's share = ´ 34400 = 13600 As per the question
43
27. (b); Let Ram, Krishna and Ganesh invested 5x, 4x x = 63 + 30 = 93
and 3x. score of Ajay = 93
Ram's Capital = 5x × 3 + 3x × 6 = 33x Rahul = 93 – 15 = 78
Krishna's Capital = 4x × 4 + 3x × 5 = 31x Manish = 93 – 25 = 68
Ganesh's Capital = 3x × 4 + 4x × 5 = 32x Total makrs of Rahul, Manish and Suresh
So, clearly shows Max. profit gained by Ram. = 3 × 63 = 189
33 Suresh = 189 – (78 + 68) = 43
Ram's profit = ´ 19200 = 6600
96 Manish + Suresh = 68 + 43 = 111
28. (c); The amount given to B as management fees = 34. (d); Let A's amount be 5x
12% fo 7500 = 900
B's amount be 6x
the rest amount of profit = 7500 – 900 = 6600
Again let B invested the capital for y months.
12000 5
Ratio of capitals of A and B = = 5x ´ 8 5 40 5
14400 6 = Þ =
6x ´ y 9 6y 9
5
A's share = ´ 6600 = 3000
5+6 9 ´ 40
y= = 12 months
B's share = 7500 – 3000 = 4500 6´5
So, Difference between A and B share is = 1500 35. (d); Ratio of profit among A, B and C
29. (a); Let the capital of Laxmi and Saraswati be 7x and = (42000 × 4 + 30000 × 6) : (30000 × 4 + 24000 × 6)
5x. : (28000 × 4 + 20000 × 6)
Let, Saraswati's capital was invested for y months = 348 : 264 : 232
So, the ratio of their capital = 7x × 20 : 5x × y
46420
140x 4 Hence C's share = ´ 232 = 12760
= , y = 21 months 844
5xy 3

100 @BEST300MCQ For More Study Material


Visit: studyiq.com
36. (c); Difference of amount received by R and Q is (7 – 39. (c); There are 70 males out of 120 applicants, there
5) = 2 must be 50 females.
Total amount received P and Q = (3 + 5) = 8 Thus, no. of males having a drives's licence all
Then, 2 corresponds to 4000 implies that 8 50 females must having a drives's licencse will
be 80 – 50 = 30. The max. possible no. of males
4000 having a driving license is 70.
corresponds to ´ 8 = 16000
2 So, ratio between the min. and max. is 30 : 70 or 3
37. (d); Cat Dog : 7.
5 4 40. (b); Given ratio of employees = 9 : 13 : 18
4 3 A.T.Q.
15 : 16 18x = 54
38. (a); Total of max. marks of all subjects = 105 × 5 x=3
= 525 No. of employees of type A = 9 × 3 = 27
Similarly, wages of every employee of type A
525 ´ 80
80% of 525 = = 420
100 10
= ´ 1400 = 2000
Obtained marks of 4 subjects = 360 7
So, marks obtained in science = 420 – 360 = 60 Req. wages = 27 × 2000 = 54000

101 @BEST300MCQ For More Study Material


Visit: studyiq.com
Chapter
Mixture and Alligation
5
Mixture: When two or more then two substances are mixed in any ratio to produce a product is known as Mixture.
Mean price: The cost price of a unit quantity of the mixture is called the mean price.
Concept 1. Alligation Rule
Let the cost price of a unit of cheaper article is Rs. c and that of a unit of costly article is d and the average
(mean) price of mixture is m, then
Quantity of cheaper article Cost price of a unit of costly article – Mean price

Quantity of costly article Mean price – Cost price of a unit of a cheaper article
The above relation is represended as

CP of cheaper CP of dearer
(c) (d)
Mean price
(m)

(d – m) (m – c)

Quantity of cheaper dm


Hence,  d  m  : m  c 
Quantity of dearer mc
Alligation rule is also used to find the ratio in which two or more ingredients at their respective prices
should be mixed to produce a mixture at a given price.
Example: In what proportion must tea at Rs. 14 per kg be mixed with tea at Rs. 18 per kg, so that the mixture be worth
Rs. 17 a kg?
Sol. CP of 1 kg cheaper tea CP of 1 kg dearer tea
14 18
Mean price
17
1 3

Quantity of cheaper 18  17 1
   1:3
Quantity of dearer 17  14 3
Concept 2. A container has milk and water in the ratio a:b, a second container of same capacity as first one has milk
and water in the ratio c:d. If both the mixture are emptied into a third container, then the ratio of milk to
water in third container is given by
 a c   b d 
  :  
a  b c  d  a  b c  d 

Example: There are two containers of equal capacity. The ratio of milk to water in the first container is 3 : 1, in the
second container is 5 : 2. If they are mixed up, then the ratio of milk to water in the mixture will be?
3 3
Sol. Part of milk in first container  
3 1 4

1 1
Part of water in first container  
3 1 4

102 @BEST300MCQ For More Study Material


Visit: studyiq.com
5 5
Similarly, part of milk in second container = =
5+2 7

2 2
Part of water in second container = =
5+2 7

3 5 1 2 41 15
\ Required =
+ : + = : = 41 : 15
4 7 4 7 28 28
Concept 3: Suppose a container contains x units of a liquid from which y units are taken out and replaced by water.
n
æ yö
After n operation, quantity of pure liquid = x ç 1 - ÷ units
è xø
Example: A container contains 40 litres of milk. From this container 4 litres of milk was taken out and replaced by
water. This process was repeated further two times. How much milk is now contained by the container?

é æ 4ö ù
3
æ 9 9 9ö
Sol. Amount of milk left after 3 operations = ê 40 ç 1 - ÷ ú litres = ç 40 ´ ´ ´ ÷ = 29.16 litres.
ë è ø
40 û è 10 10 10 ø

1. In what ratio must water be mixed with milk to gain 3. A dishonest milkman professes to sell his milk at
20% by selling the mixture at cost price? cost price but he mixes it with water and thereby
Sol. Let CP of milk be Rs. 1 per litre. gains 25%. The percentage of water in the mixture is:
Then; SP of 1 litre of mixture = Rs. 1, Gain = 20%. Sol. Let CP of 1 litre milk be Rs. 1.
Then, SP of 1 litre of mixture = Rs. 1, Gain = 25%.
æ 100 ö 5
\ CP of 1 litre of mixture = Rs. çè ´ 1÷ = Rs. .
æ 100 ö 4
120 ø 6
CP of 1 litre mixture = Rs. çè ´ 1÷ = Rs. .
By the rule of alligation, we have: 125 ø 5
CP of 1 litre of milk CP of 1 litre of water
CP of 1 litre of water CP of 1 litre of milk 0
Rs. 1 Rs. 1
0 Mean price
4
5 Rs.
Rs. 5
6 4 1
5 5
5 1 5 5
1– = –0 = 4 1
6 6 6 6 \ Ratio of milk to water = : = 4 : 1.
5 5
1 5 Hence, percentage of water in the mixture
\ Ratio of water and milk = : = 1 : 5.
6 6
æ1 ö
Shortcut: = ç ´ 100÷ % = 20%.
è5 ø
Ratio = Profit% : 100 = 20 : 100 = 1 : 5
2. In what ratio must tea at Rs. 62 per kg be mixed with 4. One quality of wheat at Rs. 9.30 per kg is mixed with
tea at Rs. 72 per kg so that the mixture must be worth another quality at a certain rate in the ratio 8:7. If the
Rs. 64.50 per kg? mixture so formed be worth Rs. 10 per kg, what is the
rate per kg of the second quality of wheat?
Sol. Cost of 1 kg tea of Cost of 1 kg tea of Sol. Let the rate of the second quality be Rs. x per kg.
1st kind 2nd kind By the rule of alligation, we have:
7200 p CP of 1 kg wheat of CP of 1 kg wheat of
6200 p
1st kind 2nd kind
Mean price 930 p (100x)p
6450 p
Mean price
750 250 1000 p
\ Required ratio = 750 : 250 = 3 : 1. (100x – 1000) p 70 p

103 @BEST300MCQ For More Study Material


Visit: studyiq.com
Sol. Amount of milk left after 3 operations
100x - 1000 8
\ = Þ 700x - 7000 = 560
70 7 é æ 10 ö ù
3

700x = 7560 Þ x = Rs. 10.80. = ê100 ç 1 - ÷ ú litres


ëê è 100 ø ûú
5. A container contains 100 litres of spirit. From this
container 10 litres of spirit was taken out and æ 9 9 9ö
replaced by water. This process was repeated further = ç 100 ´ ´ ´ ÷ = 72.9 litres.
è 10 10 10 ø
two times. How much spirit is now contained by the
container?

Ques ti ons (a) 2 : 3 (b) 3 : 2


(c) 4 : 9 (d) Can be determined
1. In a mixture of milk and water of the volume of 60 litre 9. In what ratio must sugar at Rs. 2 per kg be mixed with
the ratio of milk to water is 7 : 5. How much quantity sugar at Rs. 3.50 per kg so that the mixture be worth
of water will be added to make mixture of equal ratio? Rs. 2.50 per kg?
(a) 8 Litre (b) 10 Litre (a) 2 : 1 (b) 1 : 5
(c) 15 Litre (d) 20 Litre (c) 3 : 1 (d) 2 : 3
2. In a mixture of milk and water of the volume of 30 10. 600 gm of Sugar solution has 40% sugar in it. How
Litre, the ratio of milk and water is 8 : 7. How much much sugar should be added to make it 50% in the
water should be added in mixture to make ratio 4 : 5? solution?
(a) 6 Litre (b) 4.5 Litre (a) 60 gm. (b) 90 gm.
(c) 5 Litre (d) 5.5 Litre (c) 120 gm. (d) 150 gm.
3. In what ratio must oil at Rs. 62 per kg be mixed with 11. Gold is 21 times heavy as compared to water and
oil of Rs. 72 per kg, so that the mixture must be worth copper is 11 times heavy as compared to water. In
Rs. 64.50 per Kg? what ratio should these metal be mixed so that the
mixture may be 17 times as heavy as water?
(a) 1 : 3 (b) 1 : 2
(a) 1 : 2 (b) 2 : 1
(c) 3 : 1 (d) 4 : 1
(c) 3 : 1 (d) 3 : 2
4. A mixture of milk and water measures 60 ltr. It
contains 20% water. How many litres of water should 12. I have 100 books of English. I sold some of these at a
be added to it so that water may be 25%? profit of 12% and rest at 8% loss. On the whole I got a
profit of 11%. How many books were sold at 12%
(a) 6 ltr (b) 4 ltr profit by me?
(c) 8 gallons (d) 10 ltr (a) 50 (b) 95
5. In what ratio must a grocer mix two varieties of pulses (c) 57 (d) 60
costing Rs. 15 and Rs. 20 per kg respectively so as to
get a mixture worth Rs. 16.5 per kg? 13. There are 3 tub which contains mixtures of Milk and
Water in the ratio of 5 : 2, 4 : 3 and 3 : 1 respectively. If
(a) 3 : 7 (b) 5 : 7 the mixtures be poured in a single tub. Find the Ratio
(c) 7 : 3 (d) 7 : 5 Milk and Water?
6. A dishonest milkman professes to sell his milk at cost (a) 13 : 3 (b) 19 : 9
price but he mixes it with water and thereby gains
(c) 17 : 9 (d) Can’t be determine
20%. The percentage of water in the mixture is?
14. In what proportion must be Sugar of Rs. 17 per kg
(a) 4% (b) 16.66% mixed with Sugar of Rs. 29 per kg to make mixture of
(c) 20% (d) 25% Rs. 20 per kg?
7. Find the ratio in which rice at Rs. 7.20 a kg be mixed (a) 2 : 3 (b) 17 : 29
with rice at Rs. 5.70 a kg to produce a mixture worth (c) 29 : 17 (d) 3 : 1
Rs. 6.30 a kg?
15. In what proportion must pulse of Rs. 70 per kg be
(a) 1 : 3 (b) 2 : 3 mixed with pulse of Rs. 45 per kg to get a mixture of
(c) 3 : 4 (d) 4 : 5 Rs. 60 per Kg?
8. Aditya and Sanjay started a business investing 45000 (a) 9 : 13 (b) 13 : 9
and 30000 respectively. What will be the ratio (c) 14 : 9 (d) 3 : 2
between their profit?

104 @BEST300MCQ For More Study Material


Visit: studyiq.com
16. A shopkeeper has Rs. 50 kg of rice, of part of which he
1
sells at 10% profit and the rest at 5% loss. He gains 24. If 2 kg of metal, of which is zinc and the rest is
7% on the whole. What is the quantity sold at 10% 3
gain? 1
copper, be mixed with 3 kg of metal, of which is
(a) 50 (b) 40 4
(c) 10 (d) Can’t be determined zinc and the rest is copper, what is the ratio of zinc to
17. A man buys 2 Cow for Rs. 2700 and sells one for loss copper in the mixture?
of 6% and on other he gains 7.5%. On the whole he (a) 13 : 42 (b) 17 : 43
neither gained nor lost. What does 2 nd Cow cost? (c) 19 : 43 (d) 15 : 42
(a) 1100 (b) 1000 25. 50 g of an alloy of gold and silver contains 80% gold
(c) 1200 (d) 6075 (by weight). The quantity of gold, that is to be mixed
18. Aditya, Sonal and Nutan contributed Rs. 50,000 for up with this alloy, so that it may contain 95% gold, is:
business. Aditya contributed Rs. 4000 more than (a) 200 g (b) 150 g
Sonal and Sonal contributed Rs. 5,000 more than (c) 50 g (d) 10 g
Nutan. Out of total profit of Rs. 70,000 how much did
26. A sum of Rs. 6.40 is made up of 80 coins which are
Aditya receive?
either 10-paise or 5-paise coins. How many are coins
(a) Rs. 16,000 (b) Rs. 18,000 of 5-paise are there?
(c) Rs. 20,000 (d) Rs. 29400 (a) 24 (b) 28
19. The ratio in which two sugar solutions of the (c) 32 (d) 36
concentrations 15% and 40% are to be mixed to get a
27. How much tea at Rs. 4 a kg should be added to 15 kg
solution of concentration 30% is:
of tea at Rs. 10 per kg so that the mixture be worth
(a) 2 : 3 (b) 3 : 2 Rs.6.50 a kg?
(c) 8 : 9 (d) 9 : 8 (a) 15 kg (b) 35 kg
20. Nikita bought 30 kg of wheat at the rate of Rs. 9.50 per (c) 25 kg (d) 21 kg
kg and 40 kg of wheat at the rate of Rs. 8.50 per kg and
28. Gold is 19 times as heavy as water and copper is 9
mixed them. She sold the mixture at the rate of Rs.
times as heavy as water. In what ratio should these
8.90 per kg. Her total profit or loss in the transaction
metals be mixed so that the mixture may be 15 times
was:
as heavy as water?
(a) Rs. 2 loss (b) Rs. 2 profit
(a) 2 : 3 (b) 3 : 2
(c) Rs. 7 loss (d) Rs. 7 profit
(c) 2 : 4 (d) 4 : 2
21. A mixture of a certain quantity of milk with 16 litres
of water is worth 90 P per litre. If pure milk be worth 1
Rs. 1.08 per litre. How much milk is there in the 29. A pot contains 81 litres of pure milk. of the milk is
3
mixture?
1
(a) 40 litres (b) 50 litres replaced by the same amount of water. Again of the
3
(c) 60 litres (d) 80 litres
mixture is replaced by that amount of water. The ratio
22. A person has a chemical of Rs. 25 per litre. In what
of the milk and water in the new mixture is?
ratio should water be mixed in the mixture so that by
selling it at Rs. 20 per litre he may get a profit of 25%? (a) 1 : 2 (b) 1 : 1
(a) 14 : 9 (b) 16 : 9 (c) 2 : 1 (d) 4 : 5
(c) 9 : 14 (d) 9 : 16 30. A man has 60 pens. He sells some of these at a profit
of 12% and the rest at 8% loss. On the whole, he gets a
23. A mixture of 40 litres of milk and water contains 10%
profit of 11%. How many pens were sold at 12% profit:
water. How much water must be added to make 20%
water in the new mixture? (a) 47 (b) 52
(a) 3 litres (b) 4 litres (c) 55 (d) 57
(c) 5 litres (d) 6 litres

105 @BEST300MCQ For More Study Material


Visit: studyiq.com
1. In what ratio must water be mixed with milk to gain (a) 400 (b) 500
16% on selling the mixture at cost price? (c) 600 (d) 700
(a) 1 : 6 (b) 4 : 25 10. 400 gm of salt solution has 40% salt in it. How much
(c) 2 : 3 (d) 4 : 3 salt should be added to make it 50% in the solution?
2. In what ratio must a grocer mix two varieties of tea (a) 60 (b) 70
worth Rs. 58 per kg and Rs. 67 per kg so that by selling (c) 65 (d) 80
the mixture at Rs. 68.20 per kg he may gain 10%?
11. Chocolates at Rs. 12.00 per dozen is mixed with
(a) 3 : 2 (b) 3 : 4
chocolate at Rs. 10.00 per dozen in the ratio 3 : 5. Find
(c) 3 : 5 (d) 5 : 4 the price per dozen of the mixture?
3. A milk vendor has 2 cans of milk. The first contains (a) Rs. 11.00 (b) Rs. 9.90
25% water and the rest milk. The second contains
50% water. How much milk should he mix from each (c) Rs. 11.50 (d) Rs. 10.75
of the containers so as to get 12 litres of milk such that 12. Pencils at Rs. 4.20 per dozen is mixed with pencils at
the ratio of water to milk is 3 : 5? Rs. 5.40 per dozen in the ratio 3 : 5. Find the price per
(a) 4 litres, 8 litres (b) 6 litres, 6 litres dozen of the mixture?
(c) 5 litres, 7 litres (d) 7 litres, 5 litres (a) Rs. 4.95 (b) Rs. 4.50
4. How many kilogram of sugar costing Rs. 9 per kg (c) Rs. 5.00 (d) Rs. 5.05
must be mixed with 27 kg of sugar costing Rs. 7 per kg 13. A mixture of 48 litre of milk and water contains 10%
so that there may be a gain of 10% by selling the mixture of water. How much water must be added to make
at Rs. 9.24 per kg? 20% water in the new mixture?
(a) 36 kg (b) 42 kg (a) 2 (b) 6
(c) 54 kg (d) 63 kg (c) 4 (d) 5
5. A container contains 50 litres of milk. From this 14. A vessel of 160 litre is filled with Milk and Water.
container 5 litres of milk was taken out and replaced 70% of Milk and 30% of Water is taken out of the
by water. This process was repeated further two times. vessel. After this task, it is found that vessel is now
How much milk is now contained by the container? filled by 55% quantity of Milk and Water. What is the
(a) 26.34 litres (b) 27.36 litres original quantity of milk and water in the vessel
(c) 28 litres (d) 36.45 litres respectively?
6. In a mixture of milk and water of Volume 30 Litre the (a) 60, 100 (b) 100, 60
ratio of water and milk is 3 : 7. How much quantity of (c) 70, 90 (d) 90, 70
water will be added to the mixture to make the ratio of 15. A mixture of 80 litre of milk and water contains 10%
milk and water 1 : 2 water. How much water must be added to make 20%
(a) 23 Litre (b) 21 Litre water in the new mixture?
(c) 12 Litre (d) 33 Litre (a) 5 litre (b) 8 litre
7. The cost of Type 1 rice is Rs. 15 per kg and Type 2 rice
(c) 10 litre (d) 15 litre
is Rs. 20 per kg. If both Type 1 and Type 2 are mixed in
the ratio of 2 : 3, then the price per kg of the mixed 16. A tub contains a mixture of two liquid P and Q in the
variety of rice is? ratio of 4 : 1. When 10 Litre of the mixure is taken out
and 10 Litre of Liquid Q is poured in the jar, the ratio
(a) Rs. 18 (b) Rs. 18.50
becomes 2 : 3. How many litre of liquid P was
(c) Rs. 19 (d) Rs. 19.50 contained in the jar?
8. A merchant has 1000 kg of sugar, part of which he (a) 10 litre (b) 15 litre
sells at 8% profit and the rest at 18% profit. If he gains
14% on the whole then the quantity sold at 18% profit (c) 20 litre (d) 16 litre
is: 1
(a) 400 kg (b) 560 kg 17. A bottle contains 27 litres of pure milk. of the milk
3
(c) 600 kg (d) 640 kg
1
9. In an Examination out of 900 students 85% of the is replaced by the same amount of water. Again of
boys and 70% of girls passed. How many girls 3
appeared in the examination, if total pass percentage the mixture is replaced by that amout of water. The
was 75%? ratio of the milk and water in the new mixture is:

106 @BEST300MCQ For More Study Material


Visit: studyiq.com
(a) 1 : 2 (b) 1 : 1 24. A dishonest milkman professes to sell his milk at cost
(c) 2 : 1 (d) 4 : 5 price but he mixes it with water and thereby gains 40
18. Aditya and Manish continued in a joint business for % The percentage of water in the mixture is:
36 months. Aditya contribute Rs. 300 for certain time (a) 25% (b) 20%
and Manish invested Rs. 500 for remaining time. If (c) 45% (d) 28.56%
out of total profit of Rs. 1020, Aditya gets Rs. 495, then 25. 200 litres of mixture contains 15% water and the rest
for how long Aditya kept his money? is milk. The amount of milk that must be added so
(a) 16 months (b) 14 months that the resulting mixture contains 87.5% milk is:
(c) 8 months (d) 22 months (a) 30 litre (b) 35 litre
19. A jar contained a mixture of two liquids A and B in
(c) 40 litre (d) 45 litre
the ratio 3 : 2. When 5 litres of the mixture was taken
out and 5 litres of liquid B was poured in the jar, this 26. In a zoo, there are rabbits and pigeons. If the heads
ratio became 2 : 3. The quantity of liquid A contained are counted, there are 200 and if legs are counted,
in the jar initially was: there are 580. How many pigeons are three?
(a) 4 litres (b) 8 litres (a) 105 (b) 110
(c) 9 litres (d) 32 litres (c) 115 (d) 120
20. A man has 40 kg of tea, a part of which he sells at 5% 27. The ratio of milk and water in 66 litre of adulterated
loss and the rest at the cost price. In this business he milk is 5 : 1. Water is added to it to make the ratio 5 : 3.
incurs a loss 3%. Find the quantity which he sells at The quantity of water added is:
the cost price? (a) 20 litres (b) 22 litres
(a) 12 kg (b) 14 kg (c) 24 litres (d) 28 litres
(c) 16 kg (d) 18 kg 28. A sum of Rs. 41 was divided among 50 children. Each
21. In a mixture of 75 litres, the ratio of milk to water is boy gets 90 paise and each girl 65 paise. The number
2:1. The amount of water to be further added to the of boys is?
mixture so as to make the ratio of the milk to water 1 :
(a) 32 (b) 34
2 will be:
(a) 45 litres (b) 60 litres (c) 36 (d) 38
(c) 75 litres (d) 80 litres 29. Milk and water are mixed in vessel A in the ratio of
22. How many kg. of salt at 42 P per kg. must a man mix 5:2 and in vessel B in the ratio of 8 : 5. In what ratio
with 25 kg of salt at 24 P per kg, so that he may, on should quantities be taken from the two vessels so as
selling the mixture at 40 P per kg, gain 25% on the to form a mixture in which milk and water will be in
outlay? the ratio of 9 : 4?
(a) 20 (b) 30 (a) 7 : 2 (b) 5 : 2
(c) 40 (d) 50 (c) 2 : 7 (d) 2 : 5
23. 300 gm of sugar solution has 40% sugar in it. How 30. In a bag, there are three types of coins 1 rupee, 50-
much. sugar should be added to make it 50% in the paise and 25-paise in the ratio 3 : 8 : 20. There total
solution? value is Rs. 372. The total number of coins is:
(a) 40 gm (b) 50 gm (a) 1200 (b) 961
(c) 60 gm (d) 80 gm (c) 744 (d) 612

1. 8 litres are drawn from a cask full of wine and is then (a) 10 (b) 20
filled with water. This operation is performed three (c) 21 (d) 25
more times. The ratio of the quantity of wine now left 3. A container contained 80 litre of milk. From this
in cask to that of water is 16 : 65. How much wine did container 8 litre of milk was taken out and replaced
the cask hold originally? by water. This process was further repreated three
times. How much of milk is now contained by the
(a) 18 litres (b) 24 litres container?
(c) 32 litres (d) 42 litres (a) 54.23 litre (b) 54.26 litre
2. A can contains a mixture of two liquids A and B in (c) 56.26 litre (d) 52.48 litre
the ratio 7 : 5. When 9 litres of mixture are drawn off 4. Vessels A and B contain mixtures of milk and water
and the can is filled with B with same amount the in the ratios 4 : 5 and 5 : 1 respectively. In what ratio
ratio of A and B becomes 7 : 9. How many litres of should quantities of mixture be taken from A and B to
liquid A was contained by the can initially? form a mixture in which milk to water is in the ratio
5:4?

107 @BEST300MCQ For More Study Material


Visit: studyiq.com
(a) 2 : 5 (b) 4 : 3 12. If 5 kg of potato costing Rs. 5 per kg and 3 kg of potato
(c) 5 : 2 (d) 2 : 3 costing Rs. 4 per kg are mixed, find the average cost of
5. A can contains a mixture of two liquids A and B in the mixture per kg?
proportion 4 : 7. When 6 litres of mixture are drawn (a) Rs. 4.40 (b) Rs. 4.45
off and the can is filled with B, the proportion of A (c) Rs. 4.63 (d) Rs. 4.95
and B becomes 3 : 7. How many litres of liquids A was 13. A 20 litre mixture of milk and water contains milk
contained in the can initially? and water in the ratio 3 : 2. 10 litres of the mixture is
(a) 15.4 litres (b) 12.46 litres removed and replaced with pure milk and the
(c) 15.7 litres (d) 16.4 litres operation is repeated once more. At the end of the two
6. 100 litres of mixture contains 20% water and the rest removals and replacement, what is the ratio of milk
is milk. The amount of milk that must be added so and water in the resultant mixture?
that the resulting mixture contains 87.5% milk is: (a) 17 : 3 (b) 9 : 1
(a) 30 litres (b) 35 litres (c) 3 : 17 (d) 5 : 3
(c) 60 litres (d) 45 litres 14. A merchant mixes three varieties of rice costing Rs. 20
per kg, Rs. 24 per kg and Rs. 30 per kg and sells the
7. In what ratio must a person mix three kinds of wheat
mixture at a profit of 20% at Rs. 30 per kg. How many
costing him Rs. 1.20, Rs. 1.44 and Rs. 1.74 per kg so
kgs of the second variety will be in the mixture if 2 kgs
that the mixture may be worth Rs. 1.41 per kg?
of the third variety is there in the mixture?
(a) 11 : 77 : 7 (b) 25 : 45 : 8
(a) 1 kg (b) 3 kgs
(c) 27 : 23 : 6 (d) None of these
(c) 5 kgs (d) 6 kgs
8. Chaiwala mixes three qualities of tea T1, T2 and T3
15. In what ratio must a person mix three kinds of tea
priced at Rs. 74 per kg, Rs. 68 per kg, and Rs. 63 per kg
costing Rs. 60 per kg, Rs. 75 per kg and Rs. 100 per kg
in the ratio of 1 : 2 : 4. He added some more quantity of
so that the resultant mixture when sold at Rs. 96 per
T1 to 4 kg of this mixture. He sold this new mixture for
kg yields a profit of 20%?
Rs. 84 per kg, there by making a profit of 20%. How
much of T1 did he mix with the mixture? (a) 1 : 2 : 4 (b) 3 : 7 : 6
(a) 1 kg (b) 0.5 kg (c) 1 : 4 : 2 (d) 1 : 3 : 5
(c) 0.35 kg (d) 4 kg 16. 4 kg of potato at Rs. 5 per kg is mixed with 8 kg of
potato at Rs. 6 per kg. Find the average price of the
9. Nine litres of water are drawn from a container full of
mixture?
water and it is then filled with milk. Nine litres of
mixture are drawn and the container is again filled (a) Rs. 5.66 per kg (b) Rs. 6.50 per kg
with milk. The quantity of water now left in the (c) Rs. 7.25 per kg (d) None of these
container is to that of the milk in it is 16 : 9. How much 17. A shopkeeper bought 20 kg of onion at Rs. 6.50 per kg
liquid does the container hold? and 30 kg of onion at Rs. 7 per kg. He sold the mixture
(a) 34 litres (b) 38 litres at a profit of Rs. 60. At what price did he sell per kg of
(c) 43 litres (d) 45 litres the mixture of onions?
(a) Rs. 12 (b) Rs. 8
10. A cylinder which has the capacity of eight litres
contains a mixture of oxygen and nitrogen. The (c) Rs. 7.50 (d) Rs. 11
volume of oxygen in the cylinder is 16% of total 18. Two casks of 48 and 42 litres are filled with mixtures
volume. A few litres of the mixture is released and an of wine and water, the proportions in the two casks
equal amount of nitrogen is added. Then the same being respectively 13 : 7 and 18 : 17. If the contents of
amount of the mixture as before is released and the two casks be mixed, and 20 litres of water added
replaced by nitrogen for the second time. As a result, to the whole what will be the proportion of wine to
the oxygen content becomes 9% of total volume. Find water in the resultant mixture?
the amount of mixture released each time? (a) 13 : 12 (b) 12 : 13
(a) 2 litres (b) 2.5 litres (c) 21 : 31 (d) 31 : 21
(c) 1 litres (d) 0.5 litres 19. Three glasses of capacity 2 litres, 5 litres and 9 litres
11. Two liquids are mixed in the ratio of 3 : 2 and the contain mixture of milk and water with milk
mixture is sold at Rs. 11 per litre at a 10% profit. If the concentrations 90%, 80% and 70% respectively. The
1st liquid cost Rs. 2 more per litre than the second, contents of three glasses are emptied into a large vessel.
what is the cost per litre of the second liquid? Find the ratio of milk to water in the resultant mixture?
(a) Rs. 8.80 (b) Rs. 10.80 (a) 121 : 39 (b) 131 : 49
(c) Rs. 12.80 (d) Rs. 14.80 (c) 39 : 121 (d) 49 : 131

108 @BEST300MCQ For More Study Material


Visit: studyiq.com
20. A 12-litres cylinder contains a mixture of oxygen and replaced by nitrogen for the second time. As a result,
nitrogen, the volume of oxygen being 40% of total the oxygen content becomes 10% of the total volume.
volume. A few litres of the mixture is released and an How many litres of mixture is released each time?
equal amount of nitrogen is added. Then the same (a) 3 litres (b) 9 litres
amount of the mixture as before is released and (c) 6 litres (d) 4 litres

1. In an alloy there is 12% of copper. To get 69 kg of


2 1
copper. How much alloy will be required? (a) 6 (b) 6
3 2
(a) 424 kg (b) 575 kg
2 3
(c) 828 kg (d) 1736 kg (c) 99 (d) 6
3 4
2. 40 litres of a mixture of milk and water contains 10% 10. A jar contained a mixture of two liquids A and B in
of water, the water to be added, to make the water the ratio 7 : 2. When 18 litres of mixture was taken out
content 20% in the new mixture is: and 18 litres of liquid B was poured into the jar. This
(a) 6 litres (b) 6.5 litres ratio became 2 : 3. The quantity of liquid A contained
in the jar initially was:
(c) 5.5 litres (d) 5 litres
3. How much pure alcohol has to be added to 400 ml of 450 490
a solution containing 15% of alcohol to change the (a) litres (b) litres
17 19
concentration of alcohol in the mixture to 32%?
(a) 60 ml (b) 100 ml 490 450
(c) litres (d) litres
(c) 128 ml (d) 68 ml 17 19
4. In what ratio must a grocer mix tea at Rs. 60 a kg, and 11. In a mixture of 30 litres, the ratio of milk to water 2 : 1.
Rs. 65 a kg, so that by selling the mixture at Rs. 68.20 The amount of water to be further added to the mixture
a kg, he may gain 10%. so as to make the ratio of the milk to water 1 : 2 will be
(a) 3 : 2 (b) 3 : 4 (a) 45 litres (b) 60 litres
(c) 3 : 5 (d) 4 : 5 (c) 30 litres (d) 40 litres
5. In 50 gm alloy of gold and silver. The gold is 80% by 12. A and B are two alloys of gold and copper prepared
weight. How much gold should be mixed to this alloy by mixing metals in the ratio 5 : 3 and 5 : 11
so that the weight of gold would become 95%? respectively. Equal quantities of these alloys are melted
(a) 200 gm (b) 150 gm to form a third alloy C. The ratio of gold and copper in
(c) 50 gm (d) 10 gm the alloy C is:
6. A person has a milk of Rs. 50 per litre. In what ratio (a) 25 : 13 (b) 33 : 15
should water be mixed with milk so that on selling (c) 15 : 17 (d) 17 : 15
the mixture at Rs. 40 litres he may get profit of 25%.
13. An alloy contains zinc, copper and tin in the ratio 2 :
(a) 14 : 9 (b) 16 : 9 3 : 1 and an other contains copper, tin and lead in the
(c) 9 : 14 (d) 9 : 16 ratio 5 : 4 : 3. If equal weights of both alloys are melted
7. 7 kg of tea costing Rs. 280 per kg is mixed with 9 kg of together to form a third alloy, then the weight of lead
tea costing Rs. 240 per kg. The average price per kg of per kg in the new alloy will be
the mixed tea is:
(a) Rs. 255.80 (b) Rs. 257.50 1 1
(a) kg (b) kg
(c) Rs. 267. 20 (d) Rs. 267.50 2 8
8. In what ratio must a mixture of 30% alcohol strength 3 7
be mixed with that of 50% alcohol strength so as to (c) kg (d) kg
14 9
get a mixture of 45% alcohol strength:
(a) 1 : 2 (b) 1 : 3 14. The ratio in which tea costing Rs. 192 per kg is to be
mixed with tea costing Rs. 150 per kg so that the mixed
(c) 2 : 1 (d) 3 : 1
tea, when sold for Rs. 194.40 per kg. gives a profit of
9. In a 40 litres mixture of milk and water the ratio of 20% is
milk to water is 7 : 1. In order to make the ratio of milk
and water 3 : 1, the quantity of water (in litres) that (a) 2 : 5 (b) 23 : 5
should be added to the mixture will be: (c) 5 : 3 (d) 5 : 2

109 @BEST300MCQ For More Study Material


Visit: studyiq.com
15. In one glass, milk and water are mixed in the ratio 3 : 23. A can contains a mixture of two liquids A and B in
5 and in another glass they are mixed in the ratio 6 : 1. the ratio 7 : 5. When 9 litres of mixture are drained off
In what ratio should the content of the two glasses be and the can is filled with B, the ratio of A and B
mixed together so that the nex mixture contains milk becomes. 7 : 9. How many litres of liquid A was
and water in the ratio 1 : 1? contained by the can initially?
(a) 20 : 7 (b) 8 : 3 (a) 10 (b) 20
(c) 27 : 4 (d) 25 : 9 (c) 21 (d) 35
16. In what ratio Darjeeling Tea costing Rs. 320 per kg be 24. A shopkeeper bought 15 kg of rice at the rate of Rs. 29
mixed with Assam Tea costing Rs. 250 per kg so that per kg and 25 kg of rice at the rate of Rs. 20 per kg. He
there is a gain of 20% by selling the mixture at Rs. 324 sold the mixture of both types of rice at the rate of Rs.
per kg? 27 per kg. His profit in this transaction is:
(a) 1 : 2 (b) 2 : 3 (a) Rs. 125 (b) Rs. 150
(c) 3 : 2 (d) 2 : 5 (c) Rs. 140 (d) Rs. 145
17. The ratio of the volumes of water and glycerine in 25. The proportion of acid and water in three samples is
240cc of mixture is 1 : 3. The quantity of water (in cc) 2 : 1, 3 : 2 and 5 : 3. A mixture containing equal
that should be added to the mixture so the volumes of quantities of all three samples is made. The ratio of
water and glycerine becomes 2 : 3 is: water and acid in the mixture is
(a) 55 (b) 60 (a) 120 : 133 (b) 227 : 133
(c) 62.5 (d) 64 (c) 227 : 120 (d) 133 : 227
18. Two vessels A and B contains acid and water in the 26. Nikita bought 20 kg of wheat at the rate of Rs. 9.50 per
ratio 4 : 3 and 5 : 3 respectively. Then the ratio in kg and 30 kg of wheat at the rate of Rs. 8.50 per kg and
which these mixture to be mixed to obtain a new mixed them. She sold the mixture at the rate of Rs.
mixture in vessel C containing acid and water in the 8.90 per kg. Her total profit or loss in the transaction
ratio 3 : 2 is was:
(a) 5 : 8 (b) 7 : 8 (a) Rs. 2 loss (b) Rs. 2 profit
(c) 7 : 5 (d) 4 : 7 (c) Rs. 7 loss (d) No profit no loss
19. Acid and water are mixed in a vessel A in the ratio of 27. A mixture of a certain quantity of milk with 16 litres
5 : 2 and in the vessel B in the ratio 8 : 5. In what of water is worth Rs. 1.5 per litre. If pure milk be worth
proportion should quantities be taken out from the Rs. 3 per litre. How much milk is there in the mixture?
two vessels so as to form a mixture in which the acid (a) 40 litres (b) 16 litres
and water will be in the ratio of 9 : 4? (c) 60 litres (d) 80 litres
(a) 7 : 2 (b) 2 : 7 28. Three vessles whose capacities are 3 : 2 : 1 are
(c) 7 : 4 (d) 2 : 3 completely filled with milk mixed with water. The
20. The ratio of spirit and water in two mixtures of 20 ratio of milk and water in the mixture of vessels are 5
liters and 36 litres is 3 : 7 and 7 : 5 respectively. Both : 2, 4 : 1 and 4 : 1 respectively. Taking 1/3 of first, 1/2
the mixtures are mixed together. Now the ratio of the of second and 1/7 of third mixtures, a new mixtures,
spirit and water in the new mixture is kept in a new vessel is prepared. The percentage of
(a) 25 : 29 (b) 9 : 10 water in the new mixture is:
(c) 27 : 29 (d) 27 : 31 (a) 32 (b) 28
21. In two types of stainless steel, the ratio of chromium (c) 30 (d) 24
and steel are 2 : 11 and 5 : 21 respectively. In what 29. In 2 kg mixture of copper and aluminium, 30% is
proportion should the two types be mixed so that the copper. How much aluminium powder should be
ratio of chromium to steel in the mixed type becomes added to the mixture so that the quantity of copper
7 : 32? becomes 20% ?
(a) 2 : 3 (b) 3 : 4 (a) 900 gms (b) 800 gms
(c) 1 : 2 (d) 1 : 3 (c) 1000 gms (d) 1200 gms
22. A and B are two alloys of gold and copper in the ratio 30. The liquids X and Y are mixed in the ratio of 3 : 2 and
7 : 2 and 7 : 11 respectively. If equal quantities of these the mixture is sold at Rs. 11 per litre at a profit of 10%.
two alloys are melted to from a new alloy C, then the If the liquid X costs Rs. 2 more per litre than Y, the cost
ratio of gold and copper in C is: of X per litre is (in Rs.):
(a) 6 : 5 (b) 9 : 4 (a) 9.50 (b) 10.80
(c) 12 : 7 (d) 7 : 5 (c) 11.75 (d) 11

110 @BEST300MCQ For More Study Material


Visit: studyiq.com
S olut ions water milk
7 0 1
1. (b); In the mixture quantity of milk are ´ 60 = 35 L
12 5
5 6
In mixture quantity of water are ´ 60 = 25 litre
12
Quantity of water is added by = 35 – 25 = 10 L. 5 1 5
1– =
6 6 6
æ 30 ö æ 30 ö
2. (a); Quantity of milk & water be ççç ´ 8÷÷÷ , ççç ´7 ÷÷÷
è 15 ø è 15 ø
1 5
Milk = 16 Litre, Water = 14 Litre \ Ratio of water and milk = : =1:5
6 6
Let x litre water be added
16 4 1
= Þ 56 + 4x = 80 Þ x = 6 Required percentage = ´ 100% = 16.66%
14 + x 5 6
3. (c); By mixture Alligation method: 7. (b); By the rule of alligation

62 kg 72 kg 7.20 5.70

64.50 6.30

7.50 2.50 0.6 0.9


3 : 1
Required Ratio = 6 : 9 = 2 : 3
4. (b); Quantity of water in a mixture of 60 litre is 8. (b); The recquired ratio will be same as the ratio of
20 amount of their investment.
= ´ 60 = 12 litre
100 \ Required Ratio = 45000 : 30000 = 3 : 2
quantity of milk = 60 lt. – 12 lt. = 48 lt. 9. (a); By the rule of alligation
In new mixture Water = 25%, Milk = 75%
Let total quatity of new mixture be x litre 2 3.50

75
x´ = 48 Þ x = 64 2.50
100
So, quantity of water = 64 – 48 = 16 litre 1.00 0.50
So, 4 ltr. water must be added
5. (c); By the rule of alligation: \ Required ratio = 1 : 0.50 = 2 : 1
10. (c); Quantity of sugar in solution = 240 gram
Cheaper price Dearer price Let quantity added = x gram
15 20
240 + x 1
16.50 \ = Þ x = 120 grams
Mean price 360 1
11. (d); By the rule of alligation
y = 1.50
Gold Copper
\ Required Ratio = 3.50 : 1.50 = 70 : 30 = 7 : 3 21 11
6. (b); Let CP of milk be 1 per litre.
Then SP of 1 litre of mixture = 1 Rs.
Gain obtained = 20% 17

100 5 6 4
CP of 1 litre of mixture = ´1 =
120 6
Required ratio = 6 : 4 = 3 : 2
By the rule of alligation, we have

111 @BEST300MCQ For More Study Material


Visit: studyiq.com
18. (d); Let Nutan invested x Rs.
12. (b); 12% (–8%)
\ Sonal’s contribution = 5000 + x
\ Aditya’s contribution = 9000 + x
11% \ x + (5000 + x ) + (9000 + x) = 50000
3x + 14000 = 50000
19% 1%
3x = 36000 Þ x = 12000
books sold at 12% profit 19 \ Ratio = 21000 : 17000 : 12000 = 21 : 17 : 12
=
books sold at 8% loss 1 21
Amount that Aditya receive = ´ 70000
19 50
\ Required quantity of Book = ´ 100 = 21 × 1400 = 29400
20
= 19 × 5 = 95 19. (a); 15% 40%
5 4 3 20 + 16 + 21 57
13. (b); Milk = + + = =
7 7 4 28 28 30%

2 3 1 8 + 12 + 7 27 10% 15%
Water = + + = =
7 7 4 28 28 \ Required ratio = 10 : 15 = 2 : 3
57 27 20. (a); CP of 30 kgs = 30 × 9.50 = 285
milk : water = : = 57 : 27 = 19 : 9 CP of 40 kgs = 40 × 8.50 = 340
28 28
Total CP of 70 kgs = 285 + 340 = 625
14. (d); 17 29 SP of 70 kgs = 70 × 8.90 = 623
\ Loss = 625 – 623 = Rs. 2
20
21. (d); The mean value is 90 paisa and the price of water
9 3 is zero paisa
\ Required proportion = 9 : 3 = 3 : 1 milk water
15. (d); 70 45 108 0

60 90

15 10 90 18
Required ratio = 15 : 10 = 3 : 2 Ratio of milk and water = 5 : 1
16. (b); 10% profit (–5%) loss Quantity of milk in the mixture = 16 × 5 = 80 ltr.
100 100
22. (b); CP of mixture = ´ 20 = ´ 20 = 16
7% mean ( 100 + 25) 125

12 3 chemical water
25 0
quantity of rice sells at 10% profit 12 4
= =
quantity of rice sells at 5% loss 3 1
16
4
\ Required quantity = ´ 50 = 40 kg. 16 9
5
\ Required ratio = 16 : 9
17. (c); – 6% 7.5% 23. (c); Quantity of milk in the mixture
90
0% = ´ 40 = 36 ltr.
100
7.5% 6% Quantity of water in the mixture
\ Ratio = 7.5 : 6 = 75 : 60 = 5 : 4 10
= ´ 40 = 4 ltr.
4 100
\ 2nd cow cost = ´ 2700 = Rs. 1200
Let x ltr. of water be added
9

112 @BEST300MCQ For More Study Material


Visit: studyiq.com
27. (d); By the rule of alligation
36 80
= Þ (36) = 4 (4 + x)
4 + x 20 4 10
36 = 16 + 4x Þ 4x = 20 Þ x = 5 ltr.
6.50
1 2
24. (b); Quantity of zinc in metal 1st = ´2 =
3 3 3.50 2.50
2 4 \ Ratio of tea = 7 : 5
Quantity of copper in metal 1st = ´2 =
3 3 \ Required quantity = 7 × 3 = 21 kg.
28. (b); By the rule of alligation
2 3 8 + 9 17
Quantity of zinc in mixture = + = =
3 4 12 12 Gold Copper
19 9
2 3 mixture
Quantity of copper in mixture = ´2+ ´3
3 4 15
4 9 16 + 27 43
= + = = 6 4
3 4 12 12
Required ratio = 6 : 4 = 3 : 2
17 43 29. (d); By alligation formula:
\ Required ratio = : = 17 : 43
12 12 Quantity of milk left after 2 operations
25. (b); Let quantity of gold to be mixed = x g
2 2
æ yö æ 27 ö 2 2
80 = x ç 1 - ÷ = 81 ç 1 - ÷ = 81 ´ ´
´ 50 + x 40 + x 19 è xø è 81 ø 3 3
100 95
= Þ =
20 5 10 1 = 4 × 9 = 36 ltr.
´ 50
100 and water quantity in mixture = 81 – 36 = 45 ltr.
40 + x = 190 Þ x = 150 g recquired ratio = 36 : 45 = 4 : 5
26. (c); Let, 5–paise coin = x, 10–paise coin = y
x + y = 80 ...(i) 30. (d); profit loss
112 92
5 10 x y
´x+ ´ y = 6.40 Þ + = 6.40
100 100 20 10 111
x + 2y
= 6.40 Þ x + 2y = 128 ... (ii) 19 1
20
From (i) and (ii) 19
x + y – x – 2y = 80 – 128 no. of pens were sold at 12% profit = ´ 60 = 57
20
– y = – 48 Þ y = 48 Þ x = 80 – 48 = 32

1. (b); Let cost price of 1 litre of milk = Rs. 1


4 25
SP of 1 litre of mixture = Rs. 1; Gain = 16% Recquired ratio = : = 4 : 25
29 29
æ 100 ÷ö 25 Shortcut:
CP of 1 litre of mixture = ççç ÷´ 1 =
è 100 + 16 ÷ø 29 Ratio = Profit% : 100 = 16 : 100 = 4 : 25
100
water milk 2. (d); CP of mixture = ´ 68.20 = 62
110
0 1
58 67
25
29 62

25 = 4 : 25 5 : 4
1–
29 29 29 Required ratio = 5 : 4

113 @BEST300MCQ For More Study Material


Visit: studyiq.com
3. (b); Let cost of 1 litre of milk be Rs. 1
x - 15 3
\ = Þ 2x – 30 = 60 – 3x
3 20 - x 2
Milk in 1 litre mix in lst can =
4 5x = 90 Þ x = 18
1 8. (c); By the rule of alligation
Milk in 1 litre mix in 2nd can =
2 8% 18%
5
Milk in 1 litre of mix solution = 14%
8

5 4% 6%
Mean price = \ ratio = 4 : 6 = 2 : 3
8
3 1 3
2 recquired quantity = ´ 1000 = 600 kg
4 5
5 9. (c); By the rule of alligation
8
1 1 85% 70%
8 8
1 1 75%
Ratio = : = 1:1
8 8 5% 10%
1 \ ratio of boys and girls = 1 : 2
quantity of milk from each can = ´ 12 = 6 litre
2
2
\ no. of girls passed = ´ 900 = 2 × 300 = 600
100 42 3
4. (d); CP of mixture = ´ 9.24 = = 8.4
110 5 10. (d); Quantity of salt in 400 gm of salt solution
(By rule of alligation)
40
9 7 = ´ 400 = 160 gm
100
8.4 Let quantity added = x

: 160 + x 1
1.4 0.6 \ = Þ 160 + x = 240 Þ x = 80
\ Ratio = 14 : 6 = 7 : 3 ( 400 - 160) 1
Quantity to be added = 9 × 7 = 63 kg. 11. (d); Let price per dozen of the mixture = x
5. (d); By rule: \ By the rule of alligation
n
æ yö 12 10
Quantity of milk = x ç 1 - ÷
è xø
x
3 3
æ 5ö æ 9ö 50 ´ 9 ´ 9 ´ 9 :
= 50 ç 1 - ÷ = 50 ç ÷ = 3 5
è 40 ø è 10 ø 10 ´ 10 ´ 10
= 36.45 litres 12 - x 5
\ = Þ 36 – 3x = 5x – 50 Þ 8x = 86
6. (d); Let quantity to be added = x litre x - 10 3
9+x 2 x = 10.75
\ = Þ 9 + x = 42 Þ x = 42 – 9 12. (a); Let mean price = x
21 1
x = 33 litre 5.40
4.20
7. (a); Let the price per kg of the mixture = x
\ By the rule of Alligation
x
15 20
5.40 – x x – 4.20
x
x - 4.2 5
2 3 = Þ 8x = 39.60 Þ x = 4.95
5.4 - x 3

114 @BEST300MCQ For More Study Material


Visit: studyiq.com
13. (b); Let quantity to be added = x 9x – 9 = 4x + 6 Þ 5x = 15 Þ x = 3
\ quantity of A initially = 3 × 3 = 9 litres.
43.2 80
\ =
4.8 + x 20 20. (c); At loss At cost
86.4 = 38.4 + 8x Þ 8x = 48 Þ x = 6 95 100
14. (b); By the rule of alligation
97
Milk Water
70% 30% 3 2

55% 3
\ Ratio =
2
25% 15%
ratio = 25 : 15 = 5 : 3 2
Quantity sold at cost price = ´ 40 = 16kg.
5
5
\ Quantity of milk = ´ 160 = 100 ltr. 21. (c); Let amount of water to be added = x
8
50 1
3 \ = Þ 100 = 25 + x Þ x = 75 ltr.
and, Quantity of water = ´ 160 = 60 ltr. 25 + x 2
8
15. (c); Let amount of water to be added = x 100 4
22. (a); CP of the mixture = ´ 40 = ´ 40 = 32
125 5
72 80
\ = Þ 72 = 32 + 4x By the rule of alligation
8 + x 20
4x = 40 Þ x = 10 42 24
16. (d); Let initially the amount of liquid P in the tub = 4x
32
initially the amount of liquid Q in the tub = x

4 8 10
4x -´ 10 4x - 8 2
5 2 ratio of salts = 4 : 5
\ = Þ =
1 3 x - 2 + 10 3 \ required quantity = 5 × 4 = 20
x - ´ 10 + 10
5 23. (c); Let sugar to be added = x
12x – 24 = 2x + 16 Þ 10x = 40 Þ x = 4 120 + x 1
\ Quantity of liquid P = 4 × 4 = 16 litre \ = Þ x = 60
180 1
17. (d); Qantity of milk in new mixture-
24. (d); When mix 40 litres of water with 100 litres of
æ yö
x
æ 9ö
2
2 2 milk.
= xç 1 – ÷ = 27 ç 1 – ÷ = 27 ´ ´ =12 litre. \ Required proportion = 40 : 100 = 2 : 5
è x ø è 27 ø 3 3
Quantity of water in new mixture 2
\ % water = × 100 = 28.56%
= 27 – 12 = 15 litre 2+5
Required Ratio = 12 : 15 = 4 : 5
15
495 495 33 25. (c); Water in 200 litre mixture = ´ 200 = 30 L
18. (d); Share in profit = = = 100
1020 - 495 525 35
milk in 200 litre mixture = (200 – 30) = 170 litre
Let Aditya invested his money for x month then
Manish will invest (36 – x). 875
170 + x = 100 ´ 10 ´ (200 + x)
300 ´ x 33 x 11
\ = Þ = Þ x = 22
500 ( 36 - x) 35 36 - x 7 7
170 + x = (200 + x) Þ 1360 + 8x = 1400 + 7x
19. (c); Let Initial quantity of A and B = 4x and x 8
x = 40 litre
3
3x - ´5 3x - 3 2 26. (b); Let no. of rabbits = x, no. of pigeons = y
5 2
\ = Þ = \ x + y = 200 ... (i)
2 3 2x + 3 3
2x - ´ 5+ 5 4x + 2y = 580 ... (ii)
5

115 @BEST300MCQ For More Study Material


Visit: studyiq.com
equation (i) × (ii) – equation (ii)
x 17 17
\ = Þ no. of boys = ´ 50 = 34
2x + 2y = 400 y 8 25
4x + 2y = 580
29. (a); A B
– 2x = – 180
5 8
x = 90 Þ y = 110. 13
7
55 5 9
27. (b); Let x be the quantity to added; = 13
11 + x 3 1 2 5 9
165 = 55 + 5x Þ 5x = 110 Þ x = 22 13 91 7 13
4100
28. (b); average sum = = 82 paisa 1 2
50 Required Ratio = : =7:2
13 91
Let no. of boy = x, no. of girls = y
30. (b); Let no. of coins of 1 rupee = 3x
By the rule of alligation
no. of coins of 50 paisa = 8x
90 65 no. of coins of 25 paisa = 20x

82 8x 20x
\ 3x + + = 372 Þ 3x + 4x + 5x = 372
2 4
x y 12x = 372 Þ x = 31
\ Total no. of coins = 31 × 31 = 961

1. (b); Let original quatity = x ltr. 4


Amount of wine left after 4 operations  8 
3. (d); Milk contained in container  80 
 1  

4
 80 
æ 8ö
xç1 - ÷ 4 4
16 è xø æ 8ö æ 2ö  72 4  9 4
= Þ ç1- ÷ = ç ÷  80   80
 
   = 80 × (0.9)
 4
81 x è xø è 3ø  
 80   10 
8 2 8 2 1 = 80 × 0.6561 = 52.48 ltr.
1- = Þ = 1- = Þ x = 24 Ltr.
x 3 x 3 3
4
2. (c); Let us suppose that the Can initially contains 7x 4. (c); Quantity of milk in vessel A =
and 5x litres of mixture A and B respectively 9

æ 7 ö 5
\ Quantity of A in mixture left = ç 7x - ´ 9÷ Quantityof milk in vessel B =
è 12 ø 6

21 5
= 7x - ltr. Quantity of milk in mixture =
4 9
æ 5 ö
Quantity of B in mixture left = ç 5x - ´ 9÷ 4 5
è 12 ø
9 6
15
= 5x - ltr. 5
4
9
21
7x - 28x - 21 7
4 7 5 1
\ = Þ =
æ 15 ö 9 20x + 21 9 18 9
ç 5x - + 9
è 4 ÷ø
252x – 189 = 140x + 147 Þ 112x = 336 Þ x = 3 5 9
\required ratio = : =5:2
\ quantity contained by A initially = 7x = 21 ltr. 18 1

116 @BEST300MCQ For More Study Material


Visit: studyiq.com
5. (b); According to question
70 63
4
4x - ´6
11 3 240 y
= Þ x=
7 7 77
7x - ´ 6 + 6
11
3 4

960 70 - y 4
4x = = 12.46 = Þ 210 – 3y = 4y – 252
77 y - 63 3
7y = 462 Þ y = 66
80 Now Let us assume that z quantity of T1 is mixed
6. (c); Milk in 100 ltr. of mixture = ´ 100 = 80 ltr.
100 with mixture
Water quantity = 20 ltr. 100
CP = 84 ´ = 70
120
80 + x 87.5
\ =
20 12.5 74 66

80 + x 70
= 7 Þ 80 + x = 140 Þ x = 60 ltr.
20

7. (d); Using by allegations z 4

74 - 70 4 4 4
= Þ = Þ z = 4 kg.
70 - 66 z 4 z
9. (d); Let initially container contain x litre of water
2
æ 9ö 2 2
xç1 - ÷ æ 9ö æ 4ö
\ è x ø 16 Þ ç 1 - ÷ = ç ÷
= è xø è 5ø
x 25

9 4 9 4 1
1- = Þ = 1- = Þ x = 45 litres
x 5 x 5 5
10. (a); Let each time y amount of mixture releases

2
\ Required ratio = 12 : 7 : 7 æ yö
8ç1 - ÷ 2 2
è 8ø 0.09 æ yö æ 3ö
8. (d); Let first take T1 and T2 \ = Þ ç1- ÷ = ç ÷
8 0.16 è 8ø è 4ø

74 68 y 1
= Þ y = 2 litres.
8 4
x 11. (a); Let second liquid costs = x per litre
Let first liquid costs = (x + 2) per litre
1 2 100
CP = 11 ´ = Rs. 10
110
74 - x 2
\ = Þ 74 – x = 2x – 136 (x + 2) x
x - 68 1

3x = 210 Þ x = 70 10
Now take T3
3 2

117 @BEST300MCQ For More Study Material


Visit: studyiq.com
above two cases, i.e; QC = 1 + 1 = 2
x + 2 - 10 2 x-8 2
\ = Þ = Therefore, the required ratio = 1 : 5 : 2
10 - x 3 10 - x 3
If there are 2 kgs of the third variety in the mixture,
3x – 24 = 20 – 2x Þ 5x = 44 Þ x = 8.8 per litre. then there will be 5 kgs. of the second variety in
12. (c); Let average cost = x per kg. the mixture.
5 4 15. (c); The resultant mixture is sold at a profit of 20% at
Rs. 96/kg.

x 96
Þ Cost price = = Rs. 80 /kg.
1.2
5 3 Let the three varieties be A, B and C costing Rs.
60, Rs. 75 and Rs. 100 respectively.
5-x 3 The mean price falls between B and C
= Þ 25 – 5x = 3x – 12
x-4 5 Hence the following method should be used:
8x = 37 Þ x = 4.625 = 4.63 (Approximate) Step 1: Find out the ratio of QA : QC using alligation
13. (b); The 20 litre mixture contains milk and water in rule
the ratio of 3 : 2. Therefore, there will be 12 litres of Q A 100 - 80 1
milk in the mixture and 8 litres of water in the = =
QC 80 - 60 1
mixtrue
Step 1: When 10 litres of the mixture is removed, Step 2: Find out the ratio of Q B : Q C using
6 litres of milk is removed and 4 litres of water is alligation rule
removed. Therefore, there will be 6 litres of milk Q B 100 - 80 4
and 4 litres of water left in the container. It is then = =
QC 80 - 75 1
replaced with pure milk of 10 litres. Now the
container will have 16 litres of milk and 4 litres of Step 3: QC can be found by adding the value of QC
water. in step 1 and step 2 = 1 + 1 = 2.
Step 2: When 10 litres of the new mixture is So, QA : QB : QC = 1 : 4 : 2
removed, 8 litres of milk and 2 litres of water is 16. (a); Let average price = x per kg.
removed. The container will have 8 litres of milk
and 2 litres of water in it. Now 10 litres of pure 5 6
milk is added. Therefore, the container will have
18 litres of milk and 2 litres of water in it at the x
end of the second step.
So, required ratio = 18 : 2 or 9 : 1.
4 8
14. (c); If the selling price of mixture is Rs. 30/kg and the
merchant makes a profit of 20%, then the cost x-5 8
\ =
30 6-x 4
price of the mixture = = Rs. 25 /kg.
1.2 4x – 20 = 48 – 8x Þ 12x = 68 Þ x = 5.66
We need to find out the ratio in which the three
varieties are mixed to obtain a mixture costing 17. (b); 6.5 7
Rs. 25/kg.
Let Variety A costs Rs. 20/kg, Variety B costs Rs. x
24/kg and variety C costs Rs. 30/kg. The mean
desired price falls between B and C.
20 30
Find out the ratio QA : QC using alligation rule
Q A 30 - 25 1 x - 6.5 30
= = = Þ 2x – 13 = 21– 3x
Q C 25 - 20 1 7-x 20

Find out the ratio QB : QC using alligation rule 5x = 34 Þ x = 6.8 per kg.
\ Total value after mixture = 6.8 × 50 = 340 Rs.
Q B 30 - 25 5 Sold the mixture in (340 + 60) = 400 rs.
= =
Q C 25 - 24 1
400
QC is found by adding the value of QC in the \ required value per kg = = 8 per kg.
50
118 @BEST300MCQ For More Study Material
Visit: studyiq.com
19. (a); The quantity of milk in the (2 + 5 + 9 = 16) ltr. of
7
18. (b); Water quantity in first case = ´ 48 litre 90 80 70
20
mixture = ´2+ ´5+ ´9
100 100 100
17
water quantity in second case = ´ 42 litre = 1.8 + 4 + 6.3 = 12.1
35 Water quantity = 16.0 – 12.1 = 3.9
Total water quantity in 90 litre of mixture
121
7 ´ 48 17 ´ 42 186 \ Required Ratio = = 121 : 39
= + = litre 39
20 35 5 20. (c); Let y litre of mixture is released every time
After adding of 20 litre of water the quantity will
2
186 286 æ yö
12 ç 1 - ÷ 2
be = + 20 = è 12 ø 0.10 æ y ö 1
5 5 \ = Þ ç1- ÷ =
12 0.40 è 12 ø 4
quantity of wine in 110 litre of mixture
286 264 2 2
= 110 - = æ y ö æ1ö y 1
5 5 ç1- ÷ =ç ÷ Þ = Þ y = 6 litres
è 12 ø è2ø 12 2
264 286
\ required ratio = : = 12 : 13
5 5

1. (b); Let the weight of alloy is x kg.


60 65
12
x´ = 69
100 62
100
x = 69 ´ = 575 kg 65 – 62 : 62 – 60
12
=3 : =2
2. (d); milk water required ratio = 3 : 2
Let we add x litre of 36 4 80
water 36 4+x 5. (b); amount of gold in alloy = ´ 50 = 40 gm
100
amount of Silver = 50 – 40 = 10 gm.
4+x Let we added xgm of gold.
´ 100 = 20 , 20 + 5x = 40 + x
40 + x
40 + x
4x = 20, x = 5 litres ´ 100 = 95
50 + x
3. (b); Amount of alcohol in original solution 40 × 100 + 100x = 95 × 50 + 95x
15 100x – 95x = 4750 – 4000
= ´ 400 = 60 ml 5x = 750 Þ x = 150 gm.
100
6. (b); While selling at Rs. 20/litres he got 25% profit
Let we added x ml of pure alcohol to solution.
100
60 + x Cost Price = ´ 40 = 32 Rs./kg
´ 100 = 32 125
400 + x
60 × 100 + 100x = 32 × 400 + 32x Milk Water
100x – 32x = 32 × 400 – 60 × 100 50 0
68x = 400[32 – 15]
400 ´ 17 32
x= = 100 ml
68
100 32 – 0 : 50 – 32
4. (a); Cost pirce of Tea after mixing = ´ 68.2 = 32 : = 18
110
= 62 Rs./kg required ratio = 16 : 9

119 @BEST300MCQ For More Study Material


Visit: studyiq.com
Type I Type II 70
7. (b); 17x = 70, x=
280 240 17
70 490
Initial amount of A = 7x = 7 × = litre
Av 17 17

11. (c); milk water


Av – 240 : 280 – Av 20 10
7kg : 9kg Let we added x litre
20 10 + x
of water
Av - 240 7
=
280 - Av 9 20 1
= , 40 = 10 + x, x = 30 litre
9 Av – 2160 = 1960 – 7 Av 10 + x 2
16 Av = 4120
5
4120 12. (c); amount of gold in A =
Av = = 257.5 Rs. /kg 8
16
5
amount of gold in B =
8. (b); Type I Type II 16
30% 50% amount of gold in C = Av.
5 5
45% 8 16

Av
50 – 45 : 45 – 30
=5 : = 15
5 5
Av – – Av
5 1 16 8
required ratio = =
15 3
5
Av - 5 5
9. (a); Milk + Water = 40 litres 16 = 1 , Av - = - Av
5 1 16 8
7 1 - Av
´ 40 ´ 40 8
8 8
= 35 =5 5 5 10 + 5 15 15
2Av = + = = , Av =
8 16 16 16 32
35 3
Let we added x litres of water = 15 17
5+x 1 amount of gold = 1 - =
35 = 3 × 5 + 3 × x, 35 = 15 + 3x 32 32
3x = 20 15 17
The ratio of gold and copper = : = 15 : 17
20 2 32 32
x= =6
3 3 13. (b); Let 12 kg of both alloys are melted together
10. (c); Let the volume of Jar is 9x
Zinc Copper Tin Lead
2
A B st
1 Alloy 12 ´ = 4 kg 6 kg 2 kg —
6
7x 2x 5
2 nd Alloy — ´ 12 = 5 kg 4 kg 3 kg
Amount left after taking 12
7x - 14 2x - 4 After melting 4 kg 11 kg 6 kg 3 kg
10 litre mixture out
After adding 10 litre of B 7x - 14 2x - 4 + 18 3
The weight of lead per kg =
4 + 11 + 6 + 3
7x - 14 2
= 3 1
2x + 14 3 = = kg
21x – 42 = 4x + 28 24 8

120 @BEST300MCQ For More Study Material


Visit: studyiq.com
14. (a); The cost price of tea sold at 194.4 kg
17. (b); water(1) + glycerine(3) = 240cc
100 60cc 180cc
= ´ 194.4 = 162 Rs/kg
120 Let we add x cc water in the mixture
192 150 60 + x 2
= , 180 + 3x = 360
180 3
162 3x = 360 – 180 = 180, x = 60 cc.
4
18. (b); amount of acid in A =
162 – 150 : 192 – 162 7
= 12 : = 30 5
amount of acid in B =
required ratio = 12 : 30 = 2 : 5 8
3
3 amount of acid in C =
15. (a); In first glass amount of milk = 5
8 A B
6 4 5
In second glass amount of milk = 7 8
7 C
1 3
The amount of milk in resultant mixture = 5
2

3 6 5 3 : 3 4
– –
8 7 8 5 5 7
25 –24 21 – 20
1 = : =
40 35
2
1 1
= : =
40 35
6 1 1 3 1
– –
7 2 2 8 A 40 35 7
= = =
B 1 40 8
12 - 7 4-3
= = 35
14 8
5
5 1 19. (a); Amount of acid in A =
= = 7
14 8
8
Amount of acid in B =
5 13
14 5 8 20 9
required ratio = = ´ = = 20 : 7 Amount of acid in resultant mixture =
1 14 1 7 13
8 A B
5 8
100 7 13
16. (d); Cost price of mixture = ´ 324 = 270 Rs /kg
120
9
Darjeeling Tea Assam Tea 13
320 250
9 8 : 5 9
– –
270 13 13 7 13
1 : 2
= =
270 – 250 : 320 – 270 13 91
1
= 20 : = 50
A 13 91 7
= = =
Darjeeling Tea 20 2 B 2 2 ´ 13 2
= =
Assam Tea 50 5 91

121 @BEST300MCQ For More Study Material


Visit: studyiq.com
20. (c); sprit water 7 7 14 + 7 21 7
2Avg = + = = =
mixture of 20 litres 6 14 9 18 18 18 6
mixture of 36 litres 21 15 7
Avg =
27 29 12

required ratio = 27 : 29 7 5
Amount of copper = 1 - =
12 12
11
21. (c); Amount of steel in Ist type = 7 5
13
ratio of gold to copper = : =7:5
12 12
21
Amount of steel in IInd type = 23. (c); Let be volume of can is
26
48v
32
Amount of steel in resultant mixture =
39 28v 20v
Ist type IInd type A B
11 21 After operation ratio of A and B is 7 : 9
13 26
A B
32 28v 20v
39
21 15
Amount left After 28v - 20v -
4 4
32 21 : 11 32 taking out 9L
– – 21 15
39 26 13 39 After adding 9L of B 28v - 20v - + 9
4 4
64 – 63 33 – 32
= : =
78 39
1 1 21
= = 28v -
78 : 39 4 =7 3
21 9 , v=
1 20v + 4
Ist type 4
39 1
= 78 = =
nd
II type 1 78 2 3 7
39 Initial Amount of A = ´ 48 ´ = 21L
4 12
7
22. (d); Amount of gold in A = 24. (d); 29 Rs./kg. 20 Rs./kg.
9
7
Amount of gold in B = Avg
18
Let the amount of gold in C is Avg.
7 7 Avg – 20 29 – Avg
9 18 15 kg 25 kg

Avg Avg - 20 15 Avg - 20 3


= =
29 - Avg 25 , 29 - Avg 5
7 7 5 Avg – 100 = 87 – 3 Avg
Avg – – Avg
18 9 8 Avg = 187
7
Avg - 187
18 = 1 Avg = = 23.375
7 8
1
- Avg profit = 27 – 23.375 = 3.625
9
profit for whole transaction = 3.625 × (15 + 25)
7 7
Avg - = - Avg = 3.625 × 40 = Rs. 145
18 9

122 @BEST300MCQ For More Study Material


Visit: studyiq.com
25. (d); Let the volume of each mixture is 120x
milk water
acid water first (7 v)
1st mixture 80x 40x 5v 2v
(milk : water = 5 : 2)
nd
2 mixture 72x 48x sec ond (7v) 4 1
´ 7v ´ 7v
3rd mixture 75x 45x (milk : water = 4 : 1) 5 5
227x 133x third (v) 4 1
´v ´v
The ratio of water and acid in the resultant (milk : water = 4 : 1) 5 5
mixture = 133 : 227 mixture (15v) 11.4v 3.6v
26. (d); 9.5 Rs./kg. 8.5 Rs./kg.
3.6v
percentage of water = ´ 100 = 24%
15v
Avg
29. (c); mixture (2kg) copper aluminium
0.6kg 1.4kg
Avg – 8.5 9.5 – Avg
20 kg 30 kg Let we added x kg aluminium powder

Avg - 8.5 20 0.6


= ´ 100 = 20
9.5 - Avg 30 2+x
3 Avg – 25.5 = 19 – 2 Avg 60 = 40 + 20x, 20 = 20x
5 Avg = 44.5, Avg = 8.9 Rs./kg x = 1 kg = 1000 gm
So, no profit no loss
27. (b); Price of pure milk = 3 per litre 100
Price of mixture = 1.5 per litre 30. (b); cost price of liquid = ´ 11 = 10 Rs per litre
110
So in mixture of 1 litre 500 ml will be pure milk
and 500 ml will be water Let we take 100 litre of mixture
milk : water = 1 : 1
amount of milk in mixture = amount of water 100 litre mixture
= 16 litres.
28. (d); Let the capacity of vessels are 21v, 14v, 7v X Y
1 1 1 60 Litre 40 Litre
we take of first, of second and of third
3 2 7 Price liquid Y is a Rs per litre so price of X will be
and mix it amount taken from first mixture a + 2 Rs per litre
1 100 × 10 = 60 × (a + 2) + 40 × (a)
= ´ 21v = 7v
3 1000 = 60a + 120 + 40a
1 880 = 100a
second mixture = ´ 14v = 7v
2 a = 8.8
1 cost price of X = 8.8 + 2 = 10.8 Rs./litre
third mixture = ´ 7v = v
7

123 @BEST300MCQ For More Study Material


Visit: studyiq.com
Chapter
Percentage
6
Percent: The term 'percent' is derived from the Latin word 'Per centum'. It implies ''out of every hundred''.
The symbol ‘%’ is used to denote percentage. For example, 15% means 15 out of 100. Each ratio can be expressed as a
percentage.
1 1
For example, can be expressed as a percentage by multiplying by 100; ´ 100 = 50%
2 2
A given percentage value can be converted to corresponding fraction by dividing by 100.
75 3
Example: 75% = 75 out of hundred = =
100 4
Percentage fraction conversion chart:
1 5 1 2 2
= 50% = 83 % = 22 %
2 6 3 9 9
1 1 1 2 1
= 33 % = 14 % = 10%
3 3 7 7 10

2 2 2 4 1 1
= 66 % = 28 % =9 %
3 3 7 7 11 11
1 3 6 2 2
= 25% = 42 % = 18 %
4 7 7 11 11
3 1 1 1 1
= 75% = 12 % =8 %
4 8 2 12 3

1 3 1 5 2
= 20% = 37 % = 41 %
5 8 2 12 3
2 5 1 1 2
= 40% = 62 % =6 %
5 8 2 15 3
3 7 1 1
= 60% = 87 % = 5%
5 8 2 20
4 1 1 1
= 80% = 11 % = 4%
5 9 9 25
1 2
= 16 %
6 3
æ y ö÷
Formula to calculate percentage value: y% of x = çç ÷× x
èç 100 ø÷
From the above formula, we have the following result: x% of y = y% of x.
whenever we have two numbers x and y, one number can be expressed as a percentage of the other as follows:
x y
x as a percent of y = ´ 100 , y as a percent of x = ´ 100.
y x
Percentage increase or decrease:
increase in the quantity
Percentage increase = ´ 100
original quantity

124 @BEST300MCQ For More Study Material


Visit: studyiq.com
decrease in the quantity
Percentage decrease = ´ 100
original quantity
For example, if the net profit of a company grew from 50 crore in 2003 to 75 crore in 2004, then the percentage increase
in the net profit from 2003 to 2004 is calculated as follows:
increase in the net profit = (75 – 50) crore = 25 crore
This increase is on Rs. 50 crore.
increase in profit from 2003 to 2004 25
So, Percentage increase = ´ 100 = ´ 100 = 50%
Net profit in 2003 50
When a quantity increases or decreases by some percent, the new value of the quantity can be directly calculated as
follows:
If the original quantity is 120 and it increases by 25% , then the new quantity is: 1.25 × 120 = 150
(Here, 1.25 = 1 + 0.25, where 0.25 is equivalent to 25%)
Similarly, if there is a decrease by 25% on 120, then the new quantity is: 0.75 × 120 = 90
(Here, 0.75 = 1 – 0.25, where 0.25 is equivalent to 25%)
Some important conclusions:
æ a ö
(i) If x is a% more than y, then y is ç ´ 100 ÷% less than x.
è 100 + a ø
æ a ö
(ii) If x is a% less than y, then y is ç ´ 100 ÷ % more than x.
è 100 - a ø
Example: If in an examination, the marks secured by Prerna are 20% less than that of Vinita, then marks secured by
Vinita are how much percent more than prerna’s marks?
Solution: a = 20%
æ a ö 20
According to the above formula; Required percentage = ç ´ 100 ÷ % = ´ 100 = 25%
è 100 - a ø 80
(iii) If a number is first increased by a% and then decreased by a% then the net effect is always a decrease which
a2
%
is equal to ‘a% of a’ i.e.,
100
Example: The salary of a worker is first increased by 5% and then it is decreased by 5%. What is the change in his
salary?
Solution: Here a = 5%
a2 52
There will be a net decrease; Percent decrease = %= %= 0.25%
100 100

(iv) If a quantity is first changed (increased or decreased) by a% and then changed (increased or decreased) by
b%, then

é
Net change = ê ± a ± b +
( ± a)( ± b) ù %
ú
ëê 100 ûú
Net change is an increase or a decrease according to the positive or negative sign, respectively of the final
result.
Example: The price of an article is first increased by 20% and then decreased by 25% due to reduction in sales. Find the
net percent change in the final price of the article.
Solution: a = 20%, b = 25%
æ 20 ´ ( -25 ) ö
Required percentage change = ç 20 - 25 + ÷ % = (– 5 – 5)% = – 10%
è 100 ø
So, there is a net decrease of 10% in the final price of the article as the final result is negative.

125 @BEST300MCQ For More Study Material


Visit: studyiq.com
(v) If the price of a commodity increases or decreases by a%, then the decrease or increase in consumption, so as
æ a ö
not to increase or decrease the expenditure is equal to ç ÷ ´ 100%
è 100 ± a ø
(vi) If the population of a town is P and it increases (or decreases) at the rate of R% per annum, then
n
æ R ö
(i) Population after n years = P ç 1 ± ÷
è 100 ø

P
(ii) Population n years ago = n
æ R ö
ç1± ÷
è 100 ø
('+' sign for increment; '–' sign for decrement).
Some tricks to calculate faster:
(i) Splitting the percentage into parts
Example: Find 51% of 128.
Soltion: 51% of 128 = (50 + 1)% of 128 = 50% of 128 + 1% of 128 = 64 + 1.28 = 65.28
(ii) Interchanging the percentage value and the number
Example: Find 39% of 12.5
1
Solution: 39% of 12.5 = 12.5% of 39 = ´ 39 = 4.875
8

1. Nikhil spent 20% of his monthly income on food and Sol. Here, a = 40%; According to the formula,
15% on house rent. 40% of the remaining he spent on Reduction in Consumption
transport and 30% on entertainment. He is left with
an amount of Rs. 8775 after all the expenditures. What æ 40 ö 40
is Nikhil’s monthly income? = çç 100 + 40 ´ 100 ÷÷ % = ´ 100
è( ) ø 140
Sol. Let the income be Rs. 100
200 4
100 = % = 28 %
7 7
20% 3. The population of a town is 352800. If it increases at
15% the rate of 5% per annum, then what will be its popu-
lation 2 years hence. Also, find the population 2 years
Rs. 65
ago.
Rs. 20 Rs. 15
(Food) (House Rent) (Remaining) Sol. P = 352800, R = 5%, n = 2
Population after 2 years
40% 30% n 2
æ R ö æ 5 ö
= Pç1+ ÷ = 352800 ´ ç 1 + ÷
è 100 ø è 100 ø
26 19.5
(transport) (entertainment) 21 21
= 352800 ´ ´ = 388962
20 20
Final Saving = 100 – 20 – 15 – 26 – 19.5 = Rs. 19.5; But
his final saving is Rs. 8775 Population 2 years ago
19.5 8775 P 352800
(× 450) = =
n 2
So, Income = Rs. 100 × 450 = Rs. 45000. æ R ö æ 5 ö
ç1+ ÷ ç1+ ÷
2. If the price of a Commodity be raised by 40%, by how è 100 ø è 100 ø
much percent must a householder reduce his con-
sumption of that commodity, so as not to increase his 20 20
= 352800 ´ ´ = 320000
expenditure? 21 21

126 @BEST300MCQ For More Study Material


Visit: studyiq.com
4. There are 3 contestants P, Q and R in an election. P 5. Rahul answered 40% of the first 75 questions cor-
secured 20% of the votes and Q secured 70% of the rectly in an examination consisting of 150 questions.
remaining votes. If R secured 4800 votes, by how many Find the percentage of the remaining 75 questions
votes has the winner won the election? that he needs to answer corrrectly in order to answer
Sol. Let the total number of votes be 100. 60% of the total number of questions correctly?
P secured 20% = 20 votes. Sol. 60% of total number of question = 60% of 150 = 90
Remaining votes = 100 – 20 = 80 questions
Q secured 70% of 80 = 56 votes. Number of questions that Rahul answered correctly
R secured (80 – 56) = 24 votes. 40 ´ 75
in the first 75 questions = = 30
But R secured 4800 votes. 100
24 4800 Number of questions from the remaining 75 questions
(× 200)
that he should answer correctly = 90 – 30 = 60
So, winner Q won the election by 56 – 24 = 32 votes
32 6400 6400 votes 60 4
(× 200) Required percentage = ´ 100 = 60 ´ = 80%
75 3

Ques ti ons 9. How many litres of pure acid are there in 8 litres of a
20% solution?
(a) 1.4 (b) 1.5
1. 3.5 can be expressed in terms of percentage as:
(c) 1.6 (d) 2.4
(a) 0.35% (b) 3.5%
10. Which one of the following shows the best
(c) 35% (d) 350%
percentage?
2. What is 15 percent of Rs. 34.?
(a) Rs. 3.40 (b) Rs. 3.75 384 425
(a) (b)
(c) Rs. 4.50 (d) Rs. 5.10 540 500
3. 88% of 370+24% of 210-? = 118
570 480
(a) 256 (b) 258 (c) (d)
700 660
(c) 268 (d) 358
4. 860% of 50+50% of 860 = ? 1
11. 0.15% of 33 % of Rs. 10,000 is?
(a) 430 (b) 516 3
(c) 860 (d) 960 (a) Rs. 0.05 (b) Rs. 5
5. 60% of 264 is the same as: (c) Rs. 105 (d) Rs. 150
(a) 10% of 44 (b) 15% of 1056 12. 45% of 1500+35% of 1700 = ?% of 3175
(c) 30% of 132 (d) 17% of 544 (a) 30 (b) 35
6. 270 candidates appeared for an examination, of (c) 40 (d) 50
which 252 passed. The pass percentage is? 13. An agent gets a commission of 2.5% on the sales of
1 cloth. If on a certain day, he gets Rs. 12.50 as
(a) 80% (b) 83 % commission, the cloth sold through him on that day
2
is worth?
1 1 (a) Rs. 250 (b) Rs. 500
(c) 90 % (d) 93 %
3 3 (c) Rs. 750 (d) Rs. 1250
7. What percent of Rs. 2650 is Rs. 1987.50? 2
(a) 60% (b) 75% 14. If Rs. 2800 is th of the value of a house, the worth of
7
(c) 80% (d) 85% the house (in Rs.) is?
8. What percent of a day is 3 hours? (a) 8,00,000 (b) 9800
1 1 (c) 10,00, 000 (d) 12,00,000
(a) 12 % (b) 16 % 15. If 35% of a number is 175, then what percent of 175 is
2 3
that number?
2 1 (a) 35% (b) 65%
(c) 18 % (d) 22 %
3 2 (c) 285.71% (d) 420%

127 @BEST300MCQ For More Study Material


Visit: studyiq.com
16. If 35% of a number is 12 less than 50% of that number, 26. An agent sells goods of value of Rs. 15000. The
then the number is:
1
(a) 40 (b) 50 commission which he receives is 12 % , is what
2
(c) 60 (d) 80 amount ?
17. What percentage of numbers from 1 to 70 have squares (a) Rs. 1875 (b) Rs. 2125
that end in the digit 1?
(c) Rs. 2000 (d) Rs. 2700
(a) 1% (b) 14%
27. The monthly income of a person is Rs. 5000. If his
(c) 20% (d) 21%
income is increased by 30%, then what is his monthly
18. If 75% of a number is added to 75, then the result is income now?
the number itself. The number is:
(a) Rs. 7000 (b) Rs. 5500
(a) 50 (b) 60
(c) Rs. 4500 (d) Rs. 6500
(c) 300 (d) 400
28. The price of a certain article is Rs. 15000. But due to
19. The sum of two numbers is 2490. If 6.5% of one number slump in the market, its price decrease by 8%. Find
is equal to 8.5% of the other, then the numbers are: the new price of the article?
(a) 989, 1501 (b) 1011, 1479 (a) Rs. 14000 (b) Rs. 13800
(c) 1401, 1089 (d) 1411, 1079 (c) Rs. 16500 (d) Rs. 12600
20. When any number is divided by 12, then dividend 29. A man losses 20% of his money. After spending 25%
1 of the remainder, he has Rs. 480 left. What is the
comes th of the other number. By how much percent amount of money he originally had?
4
first number is greater than the second number? (a) Rs. 600 (b) Rs. 720
(a) 150 (b) 200 (c) Rs. 720 (d) Rs. 800
(c) 300 (d) Data inadequate 30. 15% of 10% of 20% of 1000 is?
(a) 1.50 (b) 67
21. If one number is 80% of the other and the sum of their (c) 150 (d) 3
square is 656, then the numbers are: 31. If the numerator of a fraction is increased by 120%
(a) 4, 5 (b) 8, 10 and denominator is also increased by 350% then the
(c) 16, 20 (d) 14, 12 11
22. A person’s salary has increased from Rs. 7200 to Rs. fraction become what was fraction ?
27
8100. What is the percentage increase in his salary?
(a) 25% (b) 18% 4 5
(a) (b)
5 6
2 1
(c) 16 % (d) 12 %
3 2 6 5
(c) (d)
23. A’s salary is 20% less than B’s salary. Then B’s salary 5 4
is more than A’s salary by: 32. If 40% of a number is equal to two-third of another
1 2 number, what is the ratio of first number to the second
(a) 33 % (b) 16 % number?
2 3
(a) 2 : 5 (b) 3 : 7
(c) 20% (d) 25%
(c) 5 : 3 (d) 7 : 3
24. If the price of petrol is increased by 20%, by what
33. A batsman scored 110 runs which included 3
percentage should the consumption be decreased by
boundaries and 8 sixes. What percent of his total
the consumer, if the expenditure on petrol remains
score did he make by running between the wickets?
unchanged?
5
2 2 (a) 45% (b) 45 %
(a) 16 % (b) 6 % 11
3 3
(c) 8% (d) 15% 6
(c) 54 % (d) 55%
25. The price of an article of Rs. 100. Its price is increased 11
by 10%, then again its price is increased by 10%. How 34. If 50% of (x-y)=30% of (x+y), then what percent of x is
much is increased in total price? y?
(a) 20 (b) 21 (a) 20% (b) 25%
(c) 110 (d) 121 (b) 30% (d) 40%

128 @BEST300MCQ For More Study Material


Visit: studyiq.com
35. In expressing a length 81.472 km as nearly as possible 38. An inspector rejects 0.08% of the meters as defective.
with three significant digits, find the percentage error? How many will he examine to reject 2?
(a) 0.1% (b) 0.034% (a) 2000 (b) 2200
(c) 0.0020% (d) 0.4% (c) 2500 (d) 2800
36. Difference of two numbers is 1600. If 7.5% of one 39. Which is greatest number?
number is 12.5% of the other number, find the two
2 2
numbers? (a) 16 % (b)
3 15
(a) 2490, 4000 (b) 2400, 4000
(c) 2490, 4150 (d) 2500, 4200 1
(c) (d) 0.17
37. Sixty-five percent of a number is 21 less than four- 11
fifth of that number. What is the number?
(a) 100 (b) 120 2 1
40. 16 % of 600 gm – 33 % of 180 gm
(c) 140 (d) 160 3 3
(a) 20 gm (b) 30 gm
(c) 40 gm (d) 60 gm

1. In an examination it is required to get 35% of the


1 1
aggregate marks to pass Rishu got 216 marks and (a) 12 % (b) 13 %
declared failed by 5% marks then what was the total 2 3
marks? (c) 14% (d) 15%
(a) 620 (b) 720 8. Aditya's salary was increased by 40% and then
(c) 820 (d) 710 decreased by 25%. Find out net effect in Aditya’s
2. The price of Petrol went up 20%. In order to keep salary.
expenses same Ram must reduce travel by: (a) 4% increase (b) 3% increase
(a) 25% (b) 30% (c) 5% increase (d) 6% increase
(c) 33.33% (d) 16.67% 9. In a certain school, 20% of students are below 8 years
3. The length and breadth of a rectangle field are of age. The number of students above 8 years of age is
increased, the area increased by 50%. If the length 2/3 of the number of students of 8 years age which is
increased by 20%, by what percentage was breadth 48. What is the total number of students in the school?
increased? (a) 72 (b) 80
(a) 25% (b) 30% (c) 100 (d) 150
(c) 20% (d) Data inadequate
4. Aditya's salary is 80% of Amit's salary and 120% of 3 5
10. A student multiplied a number by instead of .
Rajiv's salary. What is Amit's salary if Rajiv's salary 5 3
is 30,000? What is the percentage error in the calculation?
(a) 40000 (b) 45000 (a) 34% (b) 44%
(c) 50000 (d) 55000 (c) 54% (d) 64%
5. In a class 60% of the students pass in Hindi and 45% 11. In an election between two candidates, one got 55%
pass in Sanskrit. If 25% of them pass in both subjects, of the total valid votes, 20% of the votes were invalid.
what percentage of the student fails in both subjects? If the total number of votes was 7500, the number of
(a) 80% (b) 20% valid votes that the other candidate got, was?
(c) 25% (d) 75% (a) 2700 (b) 2900
6. An arc contains 23% copper. To get 69% of copper the (c) 3000 (d) 3100
quantity of the arc required is:
12. Three candidates contested an election and received
(a) 200 (b) 250
1136, 7636 and 11628 votes respectively. What
(c) 300 (d) 400 percentage of the total votes did the winning candidate
7. Rice is now being sold at Rs. 30 per kg. During last get?
month its rate was Rs. 26 per kg. Find by how much (a) 57% (b) 60%
percentage must a family reduce its consumption to
keep the expenditure fixed? (c) 65% (d) 90%

129 @BEST300MCQ For More Study Material


Visit: studyiq.com
13. Rajeev buys good worth Rs. 6650. He gets a rebate of 22. In an examination it is required to multiply a number
6% on it. After getting the rebate, he pay sales tax @ with 7 but by mistake a Student divide it by 7 then
10%. Find the amount he will have to pay for the what is error percent?
goods? (a) 7% (b) 97.96%
(a) Rs. 6876.10 (b) Rs. 6999.20 (c) 91% (d) 49%
(c) Rs. 6654 (d) Rs. 7000 23. 405 toffees were distributed equally among children
14. The population of a town increased from 1,75,000 to in such a way that the number of toffees received by
2,62,500 in a decade. The average percent increase of each child is 20% of total number of children. How
population per year is? many toffees did each child got?
(a) 4.37% (b) 5% (a) 40 (b) 30
(c) 6% (d) 8.75% (c) 42 (d) 9
15. In an examination, A candidate obtains 25% marks 24. Ram scored 30% marks and failed by 15 marks. Aditya
and and fails by 45 marks while another candidate score 40% marks and obtained 35 marks more than
obtains 46% marks and passed by 15% marks. What those required to pass. The pass percentage is?
are the passing marks? (a) 33% (b) 38%
(a) 600 (b) 675 (c) 43% (d) 46%
(c) 700 (d) 750 25. The price of sugar is reduced by 2%. How many kg of
16. Aditya has some amount out of which 25% is stolen sugar can now be bought for the money which was
in bus, 10% he gave to his friend, 50% of remainder is sufficient to buy 49 kg of sugar earlier?
spent in party and rest Rs. 26 gave to his mother. (a) 1 kg less (b) 1 kg more
What was his initial amount? (c) 2 kg more (d) 2 kg less
(a) 1230 (b) 80 26. Aditya's salary is 125% of Ram's salary. Sanjay's
(c) 160 (d) 90 salary is 80% of Ram's salary. If the total of all the
17. If A’s salary is 20% higher than B then how much B three salaries is Rs. 61000. What is Sanjay's salary?
salary is less than A’s salary? (a) 10000 (b) 12000
(a) 20% (b) 25% (c) 15000 (d) 16000
27. If A is 150% of B then B is what percent of A+B?
1
(c) 16.67% (d) 33 % (a) 30% (b) 35%
3
(c) 40% (d) 45%
18. If a number is increased by 20% and then reduced by
28. If in the examination 20% failed in Hindi, 25% failed
20%. After this operation the number?
in English and 7% failed in both subject then the
(a) does not change (b) increase by 1% percentage of student who failed in atleast one of the
(c) 4% increase (d) 4% decrease subjects?
3 (a) 50% (b) 45%
19. If of a number is 23 more than 50% of the same (c) 55% (d) 38%
5
number, then what will be 80% of the number? 29. If 30% students failed in English, 45% failed in Math
and 25% failed in both then the student who pass in
(a) 230 (b) 174
both subject is?
(c) 23 (d) 184
(a) 38% (b) 55%
20. Two numbers are respectively 20% and 30% less than
(c) 70% (d) 50%
third number. What is the second number as a
percentage of the first? 30. Population of Delhi increases by 10% every year. If
the current population of Delhi is 1,331,000 then what
(a) 87.5% (b) 88%
was its population 3 years ago?
(c) 77.5% (d) 87%
(a) 1000000 (b) 25000
23 (c) 10000000 (d) 1543200
21. The sum of two number is of the first number. The
20 31. A, B, C shared Rs. 18500 so that A received 25% more
second number is what percent of the first? than B and B received 20% more than C then what
(a) 5% (b) 10% amount did A received?
(c) 15% (d) 20% (a) 7000 (b) 5000
(c) 7500 (d) 8000

130 @BEST300MCQ For More Study Material


Visit: studyiq.com
32. If the price of a shirt is increased by 25% and then (a) 7084 (b) 70804
decreased by 30% then what will be net effect? (c) 52360 (d) Can’t determine
(a) 12% increase (b) 12% decrease 37. If the length of rectangle is increased by 20% and
(c) 12.5% increase (d) 12.5% decrease breadth is decreased by 10% then what will be impact
33. Nutan got 456 marks in an exam. Aditya got 54% on area?
marks in same exam which is 24 less than Nutan. (a) 5% (b) 10% increase
The minimum passing marks in exam is 35%. Then (c) 20% decrease (d) 8% increase
how much marks more than passing marks did nutan 38. If the length of rectangle is increased by 20% and
get? breadth is decreased by 10% then what will be impact
(a) 280 (b) 456 on perimeter?
(c) 180 (d) 176 (a) Can't be determined (b) 20% decrease
34. 1200 boy and 650 girls appeared in examination. If (c) 5% increase (d) 8% increase
70% of boy and 40% of girls failed. Find the 39. Ram gets 20% marks more than Girish. Girish get
approximate percentage of passed students? 20% more than Sanjay. Sanjay gets 20% less than
(a) 30% (b) 41% Aditya. If Ram got 576 marks and total marks were
(c) 50% (d) Can’t determine 800 then what marks did Aditya get?
35. Aditya's monthly salary is 15% more than Sanjay's (a) 600 (b) 480
Monthly salary if Aditya's monthly salary is 17250 (c) 500 (d) 600
than what is Sanjay's annual salary? 40. The product of one third of a number and 150% of
(a) 150000 (b) 170000 another number is what percent of product of the given
(c) 180000 (d) 250000 numbers?
36. The population of Delhi two years ago was 55000. It (a) 30% (b) 35%
increased by 12% in the first year and decreased by (c) 39% (d) 50%
15% in the second year. What was the population of
town at the end of 2 years?

1. Ram borrowed Rs. 725 from Shyam at the beginning 4. Sweta invested Rs. 10,000 in a scheme exactly three
of a year at interest. After 8 months, he again years ago. The value of the investment increased by
borrowedRs. 362.50 at a rate of interest double that 10% during the first year, increased by 5% during the
the former sum bears. At the end of the year, the sum second year, and decreased by 10% during the third
of interest on both loans is Rs. 43.50. Find the first year. What is the value of the investment today?
rate of interest per annum? (a) Rs. 10,500 (b) Rs. 10,395
(a) 4.5% (b) 4.75% (c) Rs. 10,342 (d) Rs. 10,230
(c) 6.25% (d) 7.2% 5. In Mumbai, 60% of the registered voters are BJP-
2. The cost price of goods with a bankrupt is Rs. 25500 supporters and the rest are Congress-supporters. In
and if the goods had realised in their full value, his a mayoral race, if 75% of the registered voters who
creditiors would have received 85 paise in the rupee. are BJP-supporters and 20% of the registered voters
But 2/5 of the goods were sold at 17% and the who are Congress-supporters are expected to vote
remainder at 22% below their cost price. How many for candidate X, what percent of the registered voters
paise in a rupee was received by the creditors? are expected to vote for candidate X?
(a) 72 paise (b) 68 paise (a) 53% (b) 55%
(c) 55 paise (d) 52 paise (c) 57% (d) 59%
3. A Shopkeeper undertakes to supply 2000 tables at 6. A pharmaceutical company received Rs. 3 million in
Rs. 1725 each. He estimates that if 10% are defective royalties on the first Rs. 20 million in sales of the
which will be sold at 50%, then the profit will be 15% generic equivalent of one of its products and then
on his whole outlay. When the tables were supplied, Rs.9 million in royalties on the next Rs. 108 million
70% of the tables were found defective. What loss did in sales. By approximately what percent did the ratio
the Shopkeeper incur? of royalties to sales decrease from the first Rs. 20
million in sales to the next Rs. 108 million in sales?
(a) Rs. 607500 (b) Rs. 557500
(a) 10.27% (b) 20.63%
(c) Rs. 550500 (d) Rs. 80680
(c) 38.6% (d) 44.44%

131 @BEST300MCQ For More Study Material


Visit: studyiq.com
7. In Jamshedpur, only two newspapers Dainik Jagran 12. Visions Pvt. Ltd. Appoints a sales representative on
and Prabhat Khabar are published. It is known that the basic salary of Rs. 1200 per month and the
25% of the city population reads Dainik Jagran and condition that for every sales of Rs. 10000 above
20% reads Prabhat Khabar while 8% reads both the Rs.10000, he will get 50% of basic salary and 10% of
newspapers. It is also known that 30% of those who the sales as a reward. There is no incentive for the
read Dainik Jagran but not Prabhat Khabar look into first Rs. 10000 of sales. What should be the value of
advertisement and 40% of those who read Prabhat sales if the sales representative wants to earn Rs. 7600
Khabar but not Dainik Jagran look into advertisement in a particular month?
while 50% of those who read both the newspapers (a) Rs. 120000 (b) Rs. 50000
look into advertisements. What is the percentage of
the population who read an advertisement? (c) Rs. 80000 (d) Rs. 45000
(a) 13.9% (b) 15.8% 13. Neha has a watch which gain 2% per hour when the
(c) 17.2% (d) 21.4% temperature is in the range of 40°C 50°C and it loses
8. In my office, at least 50% of the people read an e- at the same rate when the temperature is in the range
newspaper. Among those who read an e-newspaper, of 20°C – 30°C. The watch runs on times in all other
at most 25% read more than one e-paper. Only one of temperature ranges. On a sunny day, the temperature
the following statements follows from the statements started soaring up from 8 a.m. in the morning at the
given below. Which one is it? uniform rate of 2°C per hour and during the afternoon
(a) At the most 37.5% read exactly one e-paper. it stated coming down at the same rate. Find what
time will it be by the watch at 7 p.m. if at 8 a.m. the
(b) At least 37.5% read exactly one e-paper.
temperature was 32°C and at 4 p.m, it was 40°C?
(c) At the most 19.8% read exactly one e-paper.
(a) 5 : 12 : 42 p.m. (b) 6 : 28 : 33 p.m.
(d) At least 19.8% read exactly one e-paper.
9. In Convent Model School, 60% of the students are (c) 7 : 04 ; 48 p.m. (d) None of these
boys. In an aptitude test, 80% of the girls scored more 14. In laptop market, only three competitors (Lenovo,
than 40 marks (out of a maximum possible 150 marks). Apple and Samsung) exist. Last year the sales of
If 60% of the total students scored more than 40 marks Apple laptops were 10% more than Lenovo. In year,
in the same test, find the fraction of the boys who both the firms Lenovo and Apple increased their
scored 40 marks or less? respective sales by 20%. This year, the sales of the
firm Apple are five times that of Samsung. How much
3 6 were the sales of the firm (approx.) Samsung last year,
(a) (b)
5 7 if the total sales remained constant over the two years
period?
5 7
(c) (d) (a) 25% (b) 32%
7 15
(c) 38% (d) 41%
10. In a recent opinion poll held during April, 60% of the
respondents favoured India Against Corruption 15. Two jars contain equal quantities of 40% alcohol.
(IAC) while the rest favoured Indian political parties Swati changed the concentration of the first jar to
(IPP). It was found in May polls that 10% of IAC 50% by adding extra quantity of pure alcohol. Sonali
supporters switched their preference to IPP, while changed the concentration of the second jar to 50%
the same percentage of IPP’s supporters also switched replacing a certain quantity of the solution with pure
their preference to IAC. What percentage of the alcohol. By what percentage is the quantity of alcohol
electorate should now switch their preference from added by Swati more than that replaced by Sonali?
IAC to IPP so that they are at par? (a) 10% (b) 20%
(a) 14% (b) 19% (c) 30% (d) 40%
(c) 24% (d) 29% 16. For admission in a post graduate program of Calcutta
11. Suman’s project report on ‘Development with University, 90% of the candidates who appeared for
dignity’, consists of 25 pages each of 60 lines with 75 the written test were males and the rest were females,
characters on each line. In case the number of lines is 60% of the males and 80% of the females passed in
reduced to 55 but the number of characters is the written test. What is the total number of students
increased to 90 per lines, what is the percentage who appeared for the written test, if the total number
change in the number of pages. (Assume the number of passed candidates was 1240?
of pages to be a whole number.)
(a) 1380 (b) 1560
(a) – 8% (b) + 8%
(c) 2000 (d) 2500
(c) + 12% (d) 80%

132 @BEST300MCQ For More Study Material


Visit: studyiq.com
17. In a gram panchyat meeting, 1000 people voted on a year. A invests his share in the profit in a scheme
resolution with 10% of the votes being invalid. After that gives her 10% interest compounded annually
some discussion 1000 people voted again. This time and B invests his share in a scheme that gives her
there were 20% invalid votes. The opponents were 20% interest compounded annually. A gets Rs. 2520
increased by 50% while the motion was now rejected as interest at the end of 2 years and B gets an interest
by a majority, which is 300% more than it was of Rs. 4200 at the end of one year. Find C investment
formerly passed by. How many people voted against in the business?
the resolution before the discussion? (a) Rs. 10000 (b) Rs. 15000
(a) 700 (b) 600 (c) Rs. 20000 (d) Rs. 25000
(c) 500 (d) 400 20. Sashi has Rs. 90000 with him. He purchases a mobile,
18. An index of 12 shares contains, among others, the an i-pad and a laptop for Rs. 15000, Rs. 13000 and
shares of Vision Power, Vision Infra and Vision Rs. 35000 respectively and puts the remaining money
Communication with weightage of 7%, 13% and 15% in his bank account which pays 15% per annum
respectively. What is the increase in the prices of other compound interest. After 2 years he sells off the three
shares, if these three rise by 9%, 10% and 4% items at 80% of their original price and also
respectively, while the index rises by 6%? withdraws his entire money from the bank by closing
(a) 5.34% (b) 5.94% the account. What is the total change in his asset?
(c) 6.23% (d) Can’t be determine (a) 5.31% (b) 4.31%
19. A, B and C start a business by investing Rs. 70000 (c) 4.32% (d) – 4.32%
that earns them a profit of Rs. 42000 at the end of the

1. In a test a student got 30% marks and failed by 25 6. 8% of the voters in an election did not cast their votes.
marks. In the same test another student got 40% marks In this election, there were only two candidates. The
and secured 25% marks more than the essential mini- winner by obtaining 48% if the total votes, defeated
mum pass marks. The maximum pass marks for the his contestant by 1100 votes. The total number of
test were : voters in the election was?
(a) 400 (b) 480 (a) 21000 (b) 23500
(c) 500 (d) 580 (c) 22000 (d) 27500
2. In an examination, 80% of the boys passed in English 7. A district has 64000 inhabitants. If the population
and 85% passed in Mathmatics, while 75% passed
1
in both. If 45 boys failed in both. The number of boys increases at the rate of 2 % per annum, the number
who sat for the examination was: 2
(a) 400 (b) 450 of inhabitants at the end of 3 years will be :
(c) 200 (d) 150 (a) 70000 (b) 69200
3. A student scored 32% marks in science subjects out (c) 68921 (d) 68911
of 300. How much should he score in language 8. The value of a machine is Rs 6,250. It decreases by
papers out of 200 if he is to get overall 46% marks? 10% during the first year, 20% during the second
(a) 72% (b) 67% year and 30% during the third year. What will be the
(c) 66% (d) 60% value of the machine after 3 years ?
4. Ram spends 40% of his salary on food, 20% on house (a) Rs 2,650 (b) Rs 3,050
rent, 10% on entertainment and 10% on conveyance. (c) Rs 3,150 (d) Rs 3,510
If his savings at the end of a month are Rs 1500, then 9. 75 gm of sugar solution has 30% sugar in it. Then the
his salary per month (in Rs) is: quantity of sugar that should be added to the solution
(a) Rs 8000 (b) Rs 7500 to make the quantity of the sugar 70% in the solution
(c) Rs 6000 (d) Rs 10000 is :
5. A man had a certain amount with him. He spent 20% (a) 125 gm (b) 100 gm
of that to buy an article and 5% of the remaining on (c) 120 gm (d) 130 gm
transport. Then he gifted Rs 120. If he is left with Rs 10. The expenses of rice, fish and oil of a family are in the
1400 the amount he spent on transport is : ratio 12 : 17 : 3. The price of these articles are increased
(a) Rs 76 (b) Rs 61 by 20%, 30% and 50% respectively. The total expenses
(c) Rs 95 (d) Rs 80 of family on these articles are increased by :

133 @BEST300MCQ For More Study Material


Visit: studyiq.com
1 1 1 1
(a) 14 % (b) 7 % (c) 9 % (d) 11 %
8 8 11 11

1 1
(c) 28 % (d) None of these 18. If 30% of A = 0.25 of B = of C, then A : B : C is equal
8 5
to:
1
11. A spider climbed 62 % of the height of the pole in (a) 5 : 6 : 4 (b) 5 : 24 : 5
2
(c) 6 : 5 : 4 (d) 10 : 12 : 15
1
one hour and in the next hour it covered 12 % of 2A - 3B
2 19. If 40% of (A + B) = 60% of (A - B) then is.
A+B
the remaining height. If pole’s height is 192 m, then
the distance climed in second hour is: 7 6
(a) 3 m (b) 5 m (a) (b)
6 7
(c) 7 m (d) 9 m
12. A and B are two fixed points 5 cm apart and C is a 5 6
(c) (d)
point on AB such that AC is 3 cm. If the length of AC 6 5
is increased by 6%, the length of CB is decreased by: 20. 25% of 120 + 40% of 380 = ? of 637:
(a) 6% (b) 7%
2 1
(c) 8% (d) 9% (a) (b)
7 7
13. In an examination in which full marks were 500. A
got 25% more than C, C got 20% less than D. If A got 4 3
360 marks. What percentage of full marks was (c) (d)
7 7
obtained by D?
(a) 72% (b) 80% 1
21. If 20% of A = 30% of B = of C, then A : B : C is :
(c) 50% (d) 60% 6
14. The population of a town is 3,11,250. The ratio of (a) 2 : 3 : 16 (b) 3 : 2 : 16
women to men is 43:40. If there are 24% literate among (c) 10 : 15 : 18 (d) 15 : 10 : 18
women and 10% illiterate, among men, the total
number of literate persons in the town is : 2
22. A person who spends 66 % of his income is able to
(a) 1,70,700 (b) 1,73,700 3
(c) 1,75,700 (d) 1,73,200 save Rs 1200 per month. His monthly expenses (in
15. The population of a village is 25,000. One-fifth are Rs) is:
females and the rest are males, 5% of males and 40% (a) 1,200 (b) 2,400
of females are uneducated. What percentage on the (c) 3,000 (d) 3,200
whole are educated? 23. The monthly income of a person was Rs 13,500 and
(a) 75% (b) 88% his monthly expenditure was Rs 9,000. Next year his
(c) 55% (d) 85% income increased by 14% and his expenditure
16. In two successive years, 80 and 60 students of a school increased by 7%. The percentage increase in his
appeared at the final examination, of which 60% and savings was:
80% passed respectively. The average rate of students (a) 7% (b) 21%
passed (in percent) is: (c) 28% (d) 35%
24. X has twice as much money as that of Y and Y has
4
(a) 68% (b) 68 % 50% more money than that of Z. If the average money
7 of all of them is Rs 110, then the money, which X has
(c) 32% (d) 36% is:
1 (a) Rs 55 (b) Rs 60
17. If the income of Ram is 12 % more than that of (c) Rs 90 (d) Rs 180
2
Shyam, the income of Shyam is less than that of Ram 25. Tulsiram’s salary is 20% more than that of Kashyap.
by: If tulsiram saves Rs 720 which is 4% of his salary,
then Kashyap’s salary is.
1 1 (a) Rs 15,000 (b) Rs 12,000
(a) 11 % (b) 12 %
9 8 (c) Rs 10,000 (d) Rs 22,000

134 @BEST300MCQ For More Study Material


Visit: studyiq.com
26. One-third of a number is 96. What will 67% of that 34. In a school 40% of the students play football and
number be ? 50% play cricket. If 18% of the students neither play
(a) 192.96 (b) 181.44 football nor cricket, the percentage of the students
(c) 169.92 (d) 204.88 playing both is :
27. A number is divided into two parts in such a way (a) 40% (b) 32%
that 80% of 1st part is 3 more than 60% of 2nd part (c) 22% (d) 8%
and 80% of 2nd part is 6 more than 90% of the 1st 35. The percentage of metals in a mine of lead ore is 60%.
part. Then the number is:
3
(a) 125 (b) 130 Now the percentage of silver is of metals and the
4
(c) 135 (d) 145 rest is lead. If the mass of ore extracted from this mine
28. The price of an article is decreased by 10%. To restore is 8000 kg, the mass (in kg.) of lead is:
its former value the new price must be increased by : (a) 4763 (b) 4764
(a) 10% (b) 11% (c) 4762 (d) 4761
1 1 36. If the numerator of a fraction is increased by 20% and
(c) 11 % (d) 11 % the denominator is decreased by 5%, the value of the
8 9
29. A number is increased by x%; to get back to the 5
new fraction becomes . The original fraction is:
original number, it is to be reduced by? 2
10x 100 24 3
(a) % (b) % (a) (b)
100 + x 10 + x 19 18
x 95 48
(c) x% (d) % (c) (d)
100 + x 48 95
30. The price of an article was increased by r%. Later the 37. A man invests a part of Rs 10,000 at 5% and the
new price was decreased by r%. If the latest price remainder at 6%. The 5% investment yields annually
was Rs 1, then the original price was : Rs 76.50 more than the 6% investment. The amount
2 invested at 6% is:
1-r
(a) Rs 1 (b) Rs (a) Rs 3,600 (b) Rs 3, 550
100
(c) Rs 3,850 (d) Rs 4,000
1 - r2 æ 10000 ö 38. The allowance of an employee constitutes 165% of
(c) Rs (d) Rs ç 2 ÷ his basic pay. If he receives Rs 11925 as gross salary,
100 è 10000 - r ø
then his basic pay is (in Rs):
31. The Government reduced the price of sugar by 10 per (a) 4000 (b) 5000
cent. By this a consumer can buy 6.2 kg more sugar
(c) 4500 (d) 5500
for Rs 837. The reduced price per kg of sugar is:
39. The price of sugar is increased by 20%. If the
(a) Rs 12.50 (b) Rs 13.00
expenditure on sugar has to be kept the same as
(c) Rs 13.50 (d) Rs 14.00 earlier, the ratio between the reduction in
32. A reduction of 20% in the price of wheat enables consumption and the original consumption is:
Bhuvnesh to buy 5 kg more wheat for Rs 320. The (a) 1 : 3 (b) 1 : 4
original rate (in rupees per kg) of wheat was:
(c) 1 : 6 (d) 1 : 5
(a) 16 (b) 18
40. If the price of a commodity is increased by 50%. By
(c) 20 (d) 21 what fraction must its consumption be reduced so as
33. When the price of cloth was reduced by 25%, the to keep the same expenditure on its consumption:
quantity of cloth sold increased by 20%. What was
the effect on gross receipt of the shop? 1 1
(a) (b)
(a) 5% increase (b) 5% decrease 4 3
(c) 10% increase (d) 10% decrease 2
1
(c) (d)
2 3

135 @BEST300MCQ For More Study Material


Visit: studyiq.com
S o lut i on 12 ´ 100
x= Þ x = 80
15
1. (d); 3.5 can be expressed as (3.5 × 100)% = 350% 17. (c); Total such numbers = 14
15 14
2. (d); ´ 34 = 5.10 Rs. ´ 100 = 20%
100 Required percentage =
70
88 24 18. (c); Let the number be x.
3. (b); ´ 370 + ´ 210 - x = 118
100 100 75 3
x + 75 = x Þ x - x = 75
x = 325.6 + 50.4 - 118 = 376 - 118 = 258 100 4
860 50 860
4. (c); ´ 50 + ´ 860 = ( 50 + 50 ) = 860 x
= 75 Þ x = 300
100 100 100 4
60 15 19. (d); Let the two numbers be x and y.
5. (b); ´ 264 = 158.40 Þ ´ 1056 = 158.40
100 100 65 85
x + y = 2490 Þ x= y
252 1 1000 1000
6. (d); Passing percentage = ´ 100 = 93 %
270 3 17 17
x= y Þ y + y = 2490
1987.50 13 13
7. (b); Required percentage = ´ 100 = 75%
2650 30y 2490 ´ 13
= 2490 Þ y =
3 1 13 30
8. (a); Percentage = ´ 100 = 12 % y = 1079 Þ x = 2490 – 1079 Þ x = 1411
24 2
20. (b); Let the First and Second number be x and y
20
9. (c); Litres of pure acid = ´ 8 = 1.6 litres x y x 3
100 = Þ =
12 4 y 1
425
10. (b); Best percentage = 3-1
500 Required percentage = ´ 100 = 200%
1
15 100 21. (c); Let the Numbers be x and y.
11. (b); ´ ´ 10000 = Rs. 5
100 ´ 100 300
80 4x
45 35 x´ =y Þ =y
12. (c); ´ 1500 + ´ 1700 = x ´ 3175 100 5
100 100 100
x 5 é x = 5k ù
1270 ´ 100 = ê y = 4k ú
Þ x= = 40 y 4 ë û
3175
x + y = 656 Þ 25k + 16k2 = 656
2 2 2
13. (b); Let the worth of cloth be x
656
2.5 12500 k2 = Þ k = 4 Þ x = 4 × 5 = 20
x´ = 12.50 Þ x = Þ x = 500 Rs. 41
100 25
y = 4 × 4 = 16
7 Numbers are 16 and 20
14. (b); Worth of the house = 2800 ´ = 9800 Rs.
2 900
22. (d); Percentage increase in salary = ´ 100
175 7200
15. (c); 35% is = 175 Þ 100% = ´ 100 = 500
35 100 1
= = 12 %
500 8 2
Required percentage = ´ 100 = 285. 71%
175 23. (d); Let B’s salary be = 100, A’s salary be = 80
16. (d); Let the number is x
50 35 15x 20
x- x = 12 Þ = 12 Required percentage = ´ 100 = 25%
100 100 100 80

136 @BEST300MCQ For More Study Material


Visit: studyiq.com
24. (a); Let the price of petrol be 100 33. (b); Run scores from boundaries and sixes
Increase Price = 120 = 3 × 4 + 8 × 6 = 60
120 - 100 Required pecentage by Running between the
Required percentage = ´ 100
120 110 - 60 500 5
wickets = ´ 100 = = 45 %
110 11 11
100 2
= = 16 %
6 3 50 30
34. (b); (x - y) = (x + y)
110 110 100 100
25. (b); Increased Price = ´ ´ 100 = 121
100 100 y 1
5x - 5y = 3x + 3y Þ 2x = 8y Þ =
Required value = 121 – 100 = 21 x 4
25 15000
26. (a); Commission = 15000 ´ = = 1875 Rs. 1
200 8 Required Percentage = ´ 100 = 25%
4
130 35. (b); Error = (81.5 - 81.472 ) = 0.028
27. (d); Increase Monthly income = 5000 ´ = 6500 Rs.
100
0.028
Required percentage = ´ 100 = 0.034%
92 81.472
28. (b); New Price of the article = 15000 ´
100 36. (b); Let the number be x and y
= 13800 Rs.
75 125
x – y = 1600 Þ x= y
25 1000 1000
29. (d); Money left = 100% - 20% - [100% - 20%]
100
5 5
= 100% – 20% – 20% = 60% x= y Þ y – y = 1600
3 3
480
\ 60% = 480 Þ 100% = ´ 100 = Rs. 800 2y
60 = 1600 Þ y = 2400
3
15 10 20
30. (d); ´ ´ ´ 1000 = 3 x = 1600 + 2400 = 4000
100 100 100
37. (c); Let the number be x
x 4x 13x
31. (b); Let the number will be 65 4
y x + 21 = x Þ - = 21
100 5 5 20

120 220x 16x - 13x 3x


x+ x = 21 Þ = 21 Þ x = 140
Increased number = 100 = 11 Þ
100 20 20
350 27 450y
y+ y 38. (c); It means that 0.08% of x = 2
100 100
8 2 ´ 100 ´ 100
x 11 45
´x = 2 Þ x =
Þ = ´ 100 ´ 100 8
y 27 22
Required Number = 2500
x 5 39. (d); Greatest Number = 0.17
Original Fraction Þ y = 6
2 1 1 1
40. (c); 16 ´ ´ 600 – 33 ´ ´ 180
32. (c); Let the Numbers be x and y. 3 100 3 100
40 2 2 2 1 1
x= y Þ x= y Þ ´ 600 - ´ 180 = 100 – 60 = 40 gm
100 3 5 3 6 3
x 5
= Þ x:y=5:3
y 3

137 @BEST300MCQ For More Study Material


Visit: studyiq.com
1. (b); If Rishu failed by 5% marks which means he got 10. (d); Let number is 15
30% marks.
5
216 Original result = 15 × = 25
´ 100 3
30% marks = 216 Þ 100% marks =
30
3
Total marks = 720 Wrong result = 15 × =9
5
2. (d); If the price of petrol has increase by 20%, it has
25 - 9
1 Percentage error = ´ 100 = 64%
gone up th of its earlier price. 25
5
\ The percentage of reduction in petrol that will 80
maintain the amount of money spent on petrol 11. (a); No of valid votes = 7500 × = 6000
100
1 1 45
= = No. of votes that other candidate got = 6000 ´
1+ 5 6 100
= 16.67% Þ Reduction in travel = 16.67% = 2700
3. (a); Let the increased breath be x
12. (a); Total no of votes = 1136 + 7636 + 11628 = 20400
20x mn
20 + x + = 50 [% increase = m + n + ] 11628
100 100 required percentage = ´ 100 = 57%
20400
x 6x
x + = 30 Þ = 30 6
5 5 13. (a); Rebate = 6650 ´ = 399 Rs.
100
30 ´ 5
x= Þ x = 25% Amount after Rebate = 6650 – 399 = Rs. 6251
6
4. (b); Let amit's salary be x 110
Amount paid for the goods = 6251 ×
120 100
Aditya salary = ´ 30, 000 = 36,000 Rs.
100 = 6876.10 Rs.
x ´ 80 36000 ´ 100 87500
= 36000 Þ x = = 45,000 Rs. 14. (b); Percentage increase in decade = ´ 100
100 80 175000
5. (b); The percentage of students fails in both subjects
= 50%
= 100 - [(60 + 45) - 25] = 20%
\ 50% increase in 10 years.
100
6. (c); Required quantity of arc = 69 ´ = 300 \ Per years % increase = 5%
23
15. (d); Passing percentage = 46% – 15% = 31%
4
7. (b); Percentage consumption Reduced = ´ 100 45
30 \ 6% = 45 Þ 100 % = ´ 100
6
40 1
= = 13 % \ Total marks = 750
3 3
16. (b); Let the initial amount be 100%
8. (c); Let Aditya's a salary was 100 Rs.
Increased salary = 140 Rs. Total amount stolen and gave to friend = 35%
Remaining amount = 65%
75
Decreased salary = 140 ´ = 105 Rs. 65
100
Amount spend on party = = 32.5%
Net effect = 105 –100 = 5% increase 2
9. (c); Student of 8 years = 48 32.5% = 26
Total number of Students of age more than 8 years
26
2 100 % = ´ 100 Þ Initial amount = Rs. 80
= 48 × = 32 32.5
3
17. (c); Let B's salary be 100, A's salary be 120
Total student 8 years old or above = 32 + 48 = 80
Now 20% of students of is below 8 years old, so 20 100
Required pecentage = ´ 100 = = 16.67%
80% of student = 80 120 6
So total student = 100
138 @BEST300MCQ For More Study Material
Visit: studyiq.com
18. (d); Let the number be 100, Increased number = 120 26. (d); Let Ram's salary is 100
Sanjay's salary is 80 and Aditya's salary is 125
80
Decreased number = 120 ´ = 96 Total of all salary = 305
100
After this number is decreased by 4%. 61000
\ Sanjay's salary = ´ 80 = 16000 Rs.
19. (d); Let the number be x 305
3 50x 3 1 27. (c); let B = 100, then A = 150
\ x = 23 + Þ x - x = 23 Þ x = 230
5 100 5 2 100
Required pecentage = ×100 = 40%
80 250
= ´ 230 = 184 28. (d); Required percentage = 20 + 25 – 7 = 38%
100
20. (a); Let the third number be 100 29. (d); percentage of students passed in both the subjects
= 100 –(30 + 45 – 25) = 50%
Second number = 70, First number = 80
70 A
Required percentage = ´ 100 = 87.5% 30. (a); Population x years ago = n
80 é r ù
21. (c); Let the two number be x and y êë1 + 100 úû

23 23x 3x
x+y= of x Þ y = -x Þ y= 1331000 1331000
20 20 20 = 3 = = 1000000
é 10 ù 1331
3x êë1 + 100 úû 1000
20 ´ 100
Required percentage = = 15% 31. (c); Let C received = 100
x
\ B received = 120
22. (b); Let the Number = 7
So, correct answer = 7 × 7 = 49 125
A received = 120 × = 150
100
7
But answer obtained = =1
7 18500
Amount A received = ´ 150 = Rs. 7500
48 370
% error = ´ 100 = 97.96%
49 32. (d); Let the price of a shirt be 100
23. (d); Let the total number of children be x then 25% increase = 125
405 20 405 x 70
= x´ Þ = Þ x2 = 2025 30 % decrease = 125 × = 87.5%
x 100 x 5 100
x = 45 \ % decrease = 100 – 87.5 = 12.5%
405 33. (d); Marks of Aditya = 456 – 24 = 432
\ number of toffies each students get = =9 \ 54% = 432
45
432
100% = ´ 100 = 800
24. (a); 10% marks = 35 + 15 = 50, 100 % marks = 50 × 10 54
\ Total marks = 500
35
30 Passing marks = 800 × = 280
Passing marks = 500 ´ + 15 = 165 100
100
Required marks = 456 – 280 = 176
165 34. (b); Total no of boy and girl appered in the
Required percentage = ´ 100 = 33% examination = 1200 + 650 = 1850
500
25. (b); Let the price per kg is 100 rs. 30
No. of boy passed = 1200 × = 360
The total money used for 49 kg = 4900 Rs. 100
4900 60
After Price decrease =
98
= 50kg No. of girl passed = 650 ´ = 390
100
Means 1 kg is the answer. Total no. of boys and girl passed = 750

139 @BEST300MCQ For More Study Material


Visit: studyiq.com
39. (c); Let Sanjay gets 100 marks
750
Required % = ´ 100 = 40.54% @ 41% \ Girish gets 120 marks
1850
120
17250 \ Ram gets = 120 ´ = 144 marks
35. (c); Sanjay’s monthly salary = ´ 100 100
115
= Rs. 15000 576
Sanjay marks = ´ 100 = 400
Sanjay’s Annual salary = 15000 × 12 144
= Rs. 180000 400 ´ 100
36. (c); Population at the end of 2 years. Aditya marks = = 500 marks
80
112 85 40. (d); Let the numbers be x and y
55000 × ´ = 52360
100 100 So product = xy
37. (d); Impact on area 1 150y 1 3y xy
given, x´ Þ x´ Þ
3 100 3 2 2
éUsing formula ù
20 ´ ( -10) ê ú
= 20 + (– 10) + mn xy
100 êm + n + ú
ë 100 û
Required percentage = 2 ´ 100
= 10 – 2 = 8% increased xy
38. (a); Can't be determined since impact on perimeter = 50%
can't be determined as changes in different length
and breadth give different net effect on perimeter.

725 ´ R ´ 1 362.5 ´ 4 ´ 2R = Rs. (172500 + 3105000) = Rs. 3277500


1. (a); 43.5 = + Now, Rs. 3277500 includes 15% profit. Therefore,
100 12 ´ 100
100
cost price of 2000 tables = ´ 3277500
43.5 × 300 = 2175R + 362.5 × 2R = 2900 R 115
R = 4.5 = Rs. 2850000
100 Now the actual selling price
2. (b); Total debt = 22500 ´ = Rs. 30000
85
30 70 1725
Money received by selling the goods = 2000 × ×1725+2000× ×
100 100 2
æ 2 83 3 78 ö 25500
= 25500 ç ´ + ´ ÷= (166 + 234) æ 30 35 ö
è 5 100 5 100 ø 500 = 2000 × 1725 ç + ÷
= 51 × 400 = Rs. 20400 è 100 100 ø
Therefore, money received by the creditors for a = 20 ×1725 × 65 = Rs. 2242500
æ 20400 ö \ Loss = Cost Price – Selling Price
rupee = Rs. ç ÷ = Rs. 0.68 = 68 paise Hence, the Shopkeeper incurs a loss of
è 30000 ø
Hence, the creditor received 68 paise in a rupee. = Rs. 607500.
4. (b); The first year’s increase of 10% can be expressed
3. (a); 10% of 2000=200 as 1.10; the second year’s increase of 5% can be
Selling price of 200 tables at 50% expressed as 1.05; and the third year’s decrease
of 10% can be expressed as 0.90. Now, multiply
æ 1725 ö the original value of the investment account by
= Rs. ç 200 ´ ÷ = Rs. 172500
è 2 ø each of these yearly changes.
Selling price of remaining 1800 tables 10,000 × 1.10 × 1.05 × 0.90 = 10395 Rs.
= Rs. (1800 × 1725) = Rs. 3105000 Hence, the value of the investment today is
Total revenue from selling 2000 tables = Rs. 10,395.

140 @BEST300MCQ For More Study Material


Visit: studyiq.com
5. (a); Let y be the number of registered voters in 9. (d); Let the total number of students by y. Then,
Mumbai. Then, the information that 60% of the
3y 2y
registered voters are from BJP can be expressed Number of boys = , Number of girls =
as 0.60y. From this, it can be stated that 1.00y – 5 5
0.60y = 0.40y are from Congress. The percentage Number of girls scoring more than 40 marks
of BJP-supporters and the percentage of
4 2y 8y
Congress-supporters who are expected to vote = ´ =
for candidate X can then be expressed as: 5 5 25
0.75 × 0.60y + 0.20 × 0.40y Total number of students scoring more than 40
Simplify the expression to determine the total 3y
percentage of voters expected to vote for candidate marks =
5
X= 0.75 × 0.60y + 0.20 × 0.40y
= 0.45y + 0.08y = 0.53y 7 5 7
\ Required fraction = ´ =
Hence, 53% of the registered voters are expected 25 3 15
to vote for candidate X. Hence, the fraction of the boys who scored 40
6. (d); The ratio of royalties to sales for the first Rs. 20
7
3 marks or less is .
million in sales is , and the ratio of royalties 15
20
10. (a); Let the total number of respondents = 100
to sales for the next Rs. 108 million in sales is
People favoured IAC = 60
9 1 People favoured IPP = 40
= . The percent decrease in the royalties
108 12 New no. of people facouring IAC = 58
to sales ratios is 100 times the quotient of the New no. of people favouring IPP = 42
difference in the ratios divided by the ratio of Required percentage = 14%
royalties to sales for the first Rs. 20 million in 11. (a); Total characters in the report = 25 × 60 × 75
sales, i.e., Let the new number of pages be n. Then,
1 3 n × 55 × 90 = 25 × 60 × 75
-
12 20 ´ 100 = æ 1 - 3 ö ´ 20 ´ 100 25 ´ 60 ´ 75
3 ç ÷ \ n= = 22.72 » 23
è 12 20 ø 3 55 ´ 90
20
This means that Suman would need 23 pages for
æ 5 - 9 ö 20 -4 20 -4 writing his report, which is a drop of 8% in terms
=ç ÷ ´ ´ 100 = ´ ´ 100 = ´ 100 of the pages.
è 60 ø 3 60 3 9
12. (b); Income of the sales representative
= – 0.4444 × 100 = – 44.44 = 44.44% decrease
= 1200 + 1600 × y
7. (a); Let the population of the city be 100. Then,
Where y is the number of Rs. 10000 sales he
People reading Dainik Jagran = 25
achieves over the initial Rs. 10000.
People reading Prabhat Khabar = 20
Therefore,
People reading both = 8
1200 + 1600 × y = 7600
People reading only Dainik Jagran = 17
People reading only Prabhat Khabar = 12 ( 7600 - 1200 ) 6400
=4
Or, y = Þ Or, y =
Therefore, required percentage of people who read 1600 1600
an advertisement Thus, the total sales value must be Rs. 50000.
æ 30 12 ´ 40 8 ´ 50 ö 13. (c); At 12 noon, the watch would show the correct
= ç 17 ´ + + ÷ = 13.9% time, because till then the temperature range is
è 100 100 100 ø
below 40°C. The watch would gain 2% every hour
8. (b); Let the number of people in my office = 100
between 12 noon and 4 p.m. An hour having
At least 50 people read an e-newspaper.
3600 seconds, it would gain 72 seconds in each
At most 12.5 people read more than one e- of these hours. Thus, at 7 p.m. the watch would
newspaper. be 72 × 4 = 288 seconds ahead. The time exhibited
Therefore, at least 37.5 people read only one e- by the watch would be 7 : 04 : 48 p.m.
newspaper. 14. (a); For last year; Lenovo = 100, Apple = 110
Hence, at least 37.5% read exactly one e- For this year; Lenovo = 120, Apple = 132
newspaper.
Samsung = 26.4

141 @BEST300MCQ For More Study Material


Visit: studyiq.com
Last year, Samsung = 68.4 In the first voting, resolution was passed by 100
votes and in the second voting, resolution was
68.4
Required percentage = ´ 100 = 25% defeated by 400 votes. And it is 300% more than
278.4 100.
Hence, last year the sales of Samsung laptops 18. (a); Let the value of index = 100
were 25% of the total market for the laptops. Value of Vision Power share = 7
15. (b); Let solutions =100 ml and therefore, alcohol Value of Vision Infra = 13
= 40 ml. Value of Vision Communication = 15
Value of remaining = 65
40 + y 1
For first jar; = New value of Vision Power = 7.63
100 + y 2 New value of Vision Infra = 14.3
\ 80 + 2y = 100 + y Þ y = 100 – 80 = 20 ml New value of Vision Communication = 15.6
New value of remaining = 68.47
2
40 - y + y 100 68.47 - 65
5 1
For second jar; = , y= Hence, required percentage = ´ 100
3 1 6 65
60 - y
5
3.47
Hence, required percentage = ´ 100 = 5.34%
65
100 Hence, the price of other shares have increased
20 - by 5.34%
= 6 ´ 100 = 20%
100 19. (b); Let x be A's profit and y be B's profit. Then,
6 æ 10 ö
2
121
x + 2520 = x ç 1 + ÷ Þ x + 2520 = x ´ 100
16. (c); Let the total number of students appeared for the è 100 ø
test be y. Then, 100x + 252000 = 121x Þ 121x – 100x = 252000
9y
The number of male students appeared = 252000
10 \ x= = 12000
21
y
And the number of female students = y ´ 20 ´ 1
10 Also, 4200 = Þ 20y = 4200 × 100
100
3 9y 4 y
Now, according to question = ´ + ´ 420000
5 10 5 10 \ y= = 21000
20
= 1240
C's profit = Rs. 9000
27y + 4y Ratio of their profit = 12000 : 21000 : 9000
= 1240 Þ 31y=1240 × 50 = 62000
50 = 12 : 21 : 9 = 4 : 7 : 3
62000 3
\ y= = 2000 Therefore,C's share = ´ 70000 = 3 ´ 5000
31 14
Hence, the total number of candidates who = Rs.15000
appeared for the test is 2000. 20. (d); Total money spent in buying goods = Rs. 63000
17. (d); The best way of solving this question is through Total money kept in the bank = Rs. 27000
the method of elimination, i.e., going through the Total money received after selling the goods
options. = Rs. 50400
Obviously, options (A), (B) and (C) cannot be the Total money received from bank after 2 years
answer, because if 500 or more then 500 people = Rs. 35707.5
are voting in against the resolution, then the Therefore,
motion cannot be passed. Loss = Rs. 90000 – Rs. (50400 + 35707.5)
Now, let us check the option (D). If 400 people = Rs. 90000 – Rs. 86107.5 = Rs. 3892.5
are voting against the resolution for the first time,
3892.5
then 500 are voting in favour. Now, in the second Hence, percentage loss = ´ 100 = 4.325%
voting, persons voting against the resolution 90000
would be 600, so persons voting in favour=200. Hence, the total assets of Sashi decreases by
4.325%
142 @BEST300MCQ For More Study Material
Visit: studyiq.com
1. (a); Let total marks = x 6. (d); Let total no of voters = 10,000

30x 40x 25 æ 30x ö 92


\ + 25 = - ´ç + 25 ÷ Casting votes = ´ 10, 000 = 9200
100 100 100 è 100 ø 100
Winnig candidates votes = 4800
30x 40x 30x 25
+ 25 = - - \ 9200 – 4800 = 4400
100 100 400 4
\ 400 ® 1100
30x 40x 30x -25 1100
- + = - 25 10,000 = ´ 10, 000 = 27500
100 100 400 4 400
120x - 160x + 30x -25 - 100 7. (c); No. of inhabitants after 3 years
=
400 4 3
æ 5 ö
= 64000 ç 1 + ÷ = 68921
-10x -125 40 ´ 125 è 200 ø
= , x= , x = 1250
400 4 4 8. (c); The value the machine 3 years
30 æ 10 ö æ 20 ö æ 30 ö
\ Maximum passing marks = ´ 1250 + 25 = 6250 ç 1 - ÷ç1 - ÷ç1 - ÷
100 è 100 ø è 100 ø è 100 ø
= 400
9 4 7
= 6250 ´ ´ ´ = 3150 Rs.
2. (b); 10 5 10
45
E M 9. (b); Sugar : Water
75% 3 : 7 Þ 9 : 21
40
80% 85% 7 : 3 Þ 49 : 21
\ 30 ® 75
Failed in both = 45
75
éë 100 - ( 80 + 85 - 75 )ùû % = 45 40 ® ´ 40 = 100 gm
30
10% ® 45 10. (c); Let total of all items = 1200 + 1700 + 300
45 = 3200
1% ® Þ 100% ® 450
10 120
After increment, total expense = ´ 1200
46 32 x 100
3. (b); ´ 500 = ´ 300 + ´ 200
100 100 100 130 150
+ ´ 1700 + ´ 300
230 = 96 + 2x, 2x = 134, x = 67% 100 100
4. (b); éë 100 - ( 40 + 20 + 10 + 10 ) ùû % ® 1500 = 1440 + 2210 + 450 = 4100
20% ® 1500 4100 - 3200
Required = ´ 100
1% ® 75 3200
100% ® 7500
9 1
é 5 ù = ´ 100 = 28 %
5. (d); ê 100 - 20 - ´ 80 ú % ® 1400 + 120 32 8
ë 100 û
1
76% ® 1520 11. (d); 62 % of 192 = 120
2
1520 Remaining hight = 192 – 120 = 72
1% ®
76
1
1520
\ 12 % of 72 = 9 m
4% ® ´ 4 ® 80 Rs. 2
76
143 @BEST300MCQ For More Study Material
Visit: studyiq.com
5 cm 20. (a); 30 + 152 = x × 637
12. (d); A 3 cm C 2 cm
B 182 = 637x
2
106 x=
\ ´ 3 = 3.18 7
100
20 30 1
0.18 21. (d); A= B= C
\ Re quired% = ´ 100 = 9% 100 100 6
2
100 100 10
A:B:C= : :6 = 5: : 6 = 15 : 10 : 18
13. (a); A C D 20 30 3
100 80 100
æ 2ö 1
22. (b); ç 100 – 66 ÷ = 33 % = 1200
360 è 3 ø 3
Required% = ´ 100 = 72%
500 \ 100% = 3600
14. (b); Given ratio
Women : Men = 43 : 40 2
\ Required expenses = 66 % of 3600
Total person = 3,11,250 3
No. of women = 43 × 3750 = 1,61,250 = 2400 Rs.
No. of Man = 40 × 3750 = 1,50,000 23. (c); Income Expenditure Saving
No. of literate person
13500 9000 4500
24 90 15390 9630 5760
= 161250 × + 150000 ×
100 100
= 38700 + 135000 = 1,73,700 5760 - 4500
% increase in his savings = ´ 100
15. (b); Total population = 25000 4500
Female = 5000, Male = 20,000 = 28%
Uneducated persons = 1000 + 2000 = 3000 24. (d); X : Y : Z
25000 - 3000 300 : 150 : 100
\ Required% = ´ 100 = 88%
25000
16. (b); Total students = 140 300 + 150 + 100 550
\ average = =
Passed students = 48 + 48 = 96 3 3

96 550 110
\ Passing% = ´ 100 \ ® 110 , 300 ® ´ 3 ´ 300 = 180 Rs.
140 3 550

5 480 4 25. (a); Tulsi Ram Kashyap


= ´ 96 = = 68 %
7 7 7 120 : 100
1 720
12 Tulsi Ram Salary = ´ 100 = 18000
2 ´ 100 4
17. (a); Required% =
1 18000
112 120 ® 18000, 100 ® ´ 100 = 15000 Rs.
2 120
25 5 ´ 100 100 1 26. (a); No. = 96 × 3 = 288
= ´ 100 = = = 11 %
225 45 9 9 67
67% of the no = ´ 288 = 192.96 Rs.
18. (d); 30A = 25B = 20C 100
A : B : C = 10 : 12 : 15 27. (c); Let first part = x, second part = y
19. (a); 40 (A + B) = 60 (A – B) 80x 60y
= + 3 Þ 4x – 3y = 15 ... (i)
A+B 3 100 100
=
A -B 2 80y 90x
Let A = 2.5x , B = 0.5x And, = + 6 Þ 9x – 8y = –60 ... (ii)
100 100
2A – 3B 5 - 1.5 3.5 7 From (i) and (ii)
\ = = =
A+B 3 3 6 x + y = 135

144 @BEST300MCQ For More Study Material


Visit: studyiq.com
3 3
10 100 1 Silver = % of metal = of metal
28. (d); Required % = ´ 100 = = 11 % 4 400
90 9 9
397
x \ metal = ´ 4800 = 4764
29. (d); Required % = % 400
100 + x
30. (d); 100 : (100 + r)% x
100 : (100 – r)% 36. (c); Let original fraction =
y

1000 : 10000 – r2 120x 5


\ 10000 – r2 ® 1 \ =
95y 2
\ 10,000 ® 10, 000
10000 - r 2 24x 5 x 19 ´ 5 95
= , = =
19y 2 y 24 ´ 2 48
83.7
31. (c); Reduced price per kg of sugar = = 13.5 Rs.
6.2 37. (c); x ´ 5 ´ 1 - (10000 - x) ´ 6 ´ 1 = 76.50
100 100
64
32. (a); Reduced Price = = 12.8 Rs/kg
5 5x – 60,000 + 6x = 7650

100 128 11x = 67650 Þ x = 6150 Rs.


Original rate = ´ 12.8 = = 16
80 8 \ Required amount = (10,000 – 6150) = 3850 Rs.
33. (d); 100 –25% +20% 38. (c); Let his basic pay = x Rs.
75 90
effect = – 10% 165x 265x
\ x+ = 11925 , = 11925
34. (d); 100 100
100%
x = 4500 Rs.

Football Cricket 39. (c); Pr ice 5 : 6


Consumption 6 : 5
40% 50%
1
\ Required Ratio = =1:6
6
Required% = (40 + 50) – (100 – 18) = 8%
35. (b); Given mass of one = 8000 kg. 40. (b); 100 +50%
150
60 50 1
\ Metal = ´ 8000 = 4800 kg. \ Required Fraction = =
100 150 3

145 @BEST300MCQ For More Study Material


Visit: studyiq.com
Chapter
Profit and Loss
Theory:
7
Cost Price (CP): The money paid by the shopkeeper to the manufacturer or whole -seller to buy goods is called the
cost price (cp) of the goods purchased by the shopkeeper.
Selling Price (SP): The price at which the shopkeeper sells the goods is called selling price (s.p) of the goods sold by the
shopkeeper to the customer.
Profit: If the selling price of an article is more than its cost price, then the dealer (or shopkeeper) makes a
profit (or gain)
i.e., Profit = SP – CP; SP > CP
Loss: If the selling price of an article is less than its cost price, the dealer suffers a loss
i.e., Loss = CP – SP; CP > SP
Some Important Formulae:
(i) Profit = SP – CP
(ii) Loss = CP – SP
 Profit 
(iii) Profit percentage = 
  100 %
 CP 
 Loss 
(iv) Loss percentage = 
  100 %
 CP 
æ (100 + Profit%)´ CP ö÷ æ (100 - Loss%)´ CP ö÷
(v) S.P = ççç ÷ = çç ÷
è 100 ø÷ çè 100 ø÷

æ 100´ SP ö÷ æ 100 ´ SP ö÷
(vi) C.P = ççç ÷ = çç ÷
è 100 + Profit% ø÷ çè 100 - Loss%ø÷
(vii) SP = (100 + x)% of CP; when Profit = x% of CP
(viii) SP = (100 – x)% of CP; when Loss = x% of CP
Example 1: A man purchases an item for Rs. 120 and he sells it at a 20 percent profit, find his selling price

æ 100 + Profit% ö÷ 100 + 20 120


Sol. SP = çç ÷÷´ CP = ´ 120 = ´ 120 = Rs. 144
çè 100 ø 100 100

Note: Profit /Loss percentage is always calcualated on C.P. unless otherwise stated.

Example 2: Find the cost price of an article which is sold for Rs. 200 at a loss of 20%
100 100
Sol. CP = ´ SP = ´ 200 = Rs. 250
100 - Loss% 100 - 20
Concept 1:
MARK UP AND DISCOUNT
Marked Price: To avoid loss due to bargaining by the customer and to get profit over the cost price, the trader increases
the cost price. This increase is known as markup and the increased price (i.e., cp+markup) is called the
marked price or printed price or list price of the goods.
Marked Price = CP + markup

Marked Price = CP +
(% marked) ´ CP
100
Generally goods are sold at marked price. If there is no further discount, then in this case selling price
equals marked price.

146 @BEST300MCQ For More Study Material


Visit: studyiq.com
Discount: Discount means reduction of marked price to sell at a lower rate or literally discount means concession.
Basically, it is calculated on the basis of marked price.
Selling price = Marked price – Discount

Selling price = MP –
(% Discount ) ´ MP
100

Example: If the cost price of an article is Rs. 300 and the percent markup is 30%. What is the marked price?
30
Sol. MP = CP + (%markup on CP) = 300 + ´ 300 = Rs. 390
100
Concept 2:
Dishonest Dealer Case: If a trader professes to sell his goods at cost price, but uses false weights, then
Error True weight - False weight
% gain = ´ 100 Þ %gain = ´ 100
True value - Error False weight
Example: A shopkeeper sold an article at cost price but use the weight of 960 gm in place of 1 kg weight. Find his
profit%?
True weight - False weight 1000 - 960 40 25 1
Sol. Profit% = ´ 100 = ´ 100 = ´ 100 = =4 %
False weight 960 960 6 6
Concept 3:
Where two articles are sold at same price but one of them at a profit and another at a loss and the percentage profit is the
same as the percentage loss, In this case there is always a loss.
2 2
æ Common Profit or Loss% ö æ %value ö
Loss% = ç ÷ø = çè 10 ÷ø
è 10
Example: Each of two car is sold for Rs. 1000. The first one is sold at 25% profit and the other one at 25% loss. What is
the percentage loss or gain in the deal?
Sol. Total s.p = 1000 ´ 2 = Rs. 2000
100 ´ 1000
CP of 1st car =
125 ëéQ Profit = 25%ûù
= Rs. 800
100 ´ 1000
CP of 2nd car =
75 ëéQ Loss = 25%ûù

1
= Rs. 1333
3
1
2133 - 2000
1 CP - SP 3
Total CP = Rs. 2133 Þ Loss% = ´ 100 = ´ 100 = 6.25%
3 CP 1
2133
3
or, Using Shortcut Formula
2 2
æ %value ö æ 25 ö
Loss% = ç ÷ = ç ÷ = 6.25 %
è 10 ø è 10 ø
Concept 4:
æ xy ö
When two successive discounts on an article are x% and y% resp. then net discount: çè x + y - %
100 ÷ø
Example: A shopkeeper gives two sucessive discount of 50% and 50%. Find the real (equivalent) discount?
Sol. Let MP = Rs. 100
Cost after 1st discount of 50% = 100 – 50% of 100 = Rs. 50

147 @BEST300MCQ For More Study Material


Visit: studyiq.com
Cost after 2nd discount of 50% = 50 – 50% of 50 = Rs. 25
Price after both discount = Rs. 25
100 - 25
% discount = ´ 100 = 75%
100
or, Using Shortcut Formula
xy
% discount = x + y - [where x = 50%, y = 50%]
100
50 ´ 50
= 50 + 50 - = 100 – 25 = 75%
100

1. There is a profit of 20% on the cost price of an article. Sol. In these types of question, we have to take the LCM of
Find the profit percent when calculated on selling number of individual things.
price? Number of fruits of each type be bought
Sol. Let the cost price of an article be Rs. 100 = LCM of 11 and 9 = 99
then, Profit = 20% of 100 = Rs. 20 Total number of fruits = 99 × 2 = 198
Selling price = Cost price + profit
= 100 + 20 = Rs. 120 10 10
CP of 198 fruits = ´ 99 + ´ 99
Profit% when calculated on SP 11 9
20 100 2 = 90 + 110 = Rs. 200
= ´ 100 = = 16 %
120 6 3 SP = 198 × 1 = Rs. 198
2. By selling a bicycle for Rs. 2850, a shopkeeper gains CP – SP 200 - 198
14%. If the profit is reduced to 8%, find the selling Loss% = ´ 100 = ´ 100
CP 200
price of bicycle?
SP ´ 100 2850 ´ 100 2
Sol. CP = = = ´ 100 = 1%
100 + Profit% 100 + 14 200
5. A book vendor sold a book at a loss of 10%. Had he
2850 ´ 100 sold it for Rs. 108 more, he would have earned a profit
= = Rs. 2500
114 of 10%. Find the cost of the book.
SP of article for 8% Profit Sol. Let the CP article = x
CP ´ ( 100 + Profit%) 2 5 00 ´ 1 0 8 x ( 100 - 10) 90x 9x
SP = = SP = = = éLoss of 10%ùû
100 100 100 100 10 ë
= 25 × 108 = Rs. 2700
3. The selling price of 12 articles is equal to the cost 9x 110x
+ 108 = éIf vendor sold for Rs. 108 more ûù
price of 15 articles. Find the gain percent? 10 100 ë
Sol. Let the CP of 1 article = Rs. x
110x 9x 11x 9x
Cost Price of 15 article = Rs. 15x - = 108 Þ - = 108
Selling Price of 12 article = Rs. 15x 100 10 10 10
2x = 1080 Þ x = Rs. 540
15
SP of 1 article = Rs. x 6. A person bought some articles at the rate of 5 per
12 rupee and the same number at the rate of 4 per rupee.
15x 3x x He mixed both the types and sold at the rate of 9 for
Gain = -x = = Rs. 2. In this business he suffered a loss of Rs. 3. Find
12 12 4
th total no. of articles bought by him?
x Sol. Let the person buys 10 articles
Gain ´ 100 4 ´ 100
Gain%= = = 25% 1 5 ´ 1ö
CP x æ æ 5ö 9
Total CP = Rs. ç 5 ´ + ÷ = Rs. ç 1 + ÷ = Rs.
4. A fruit seller buys some fruits at the rate of 11 for Rs. è 5 4 ø è 4ø 4
10 and the same number at the rate of 9 for Rs. 10. If
2 20
all the fruits are sold for Rs. 1 each. Find the gain or SP of 10 articles = ´ 10 = Rs.
loss percent? 9 9

148 @BEST300MCQ For More Study Material


Visit: studyiq.com
æ 9 20 ö æ 81 - 80 ö 1 1 1
Loss = Rs. ç - ÷ = ç = Rs. Sol. Initial sales tax = 3 % , Final sales tax = 3 %
è 4 9 ø è 36 ÷ø 36 2 3
Difference in percentage of sales tax
1
If loss is Rs. , then number of articles = 10
36 æ 1 1ö 1
= ç3 - 3 ÷ % = %
If loss is Rs. 3, number of articles = 36 × 10 × 3 = 1080 è 2 3ø 6
7. A man buys a field of agricultural land for Rs. 360000.
1 1 1
1 2 Req. diff. = % ´ 8400 = ´ ´ 8400 = Rs. 14
He sells rd at a loss of 20% and th at a gain of 6 6 100
3 5
10. A man sells two cycles for Rs. 1710. The cost price of
25%. At what price must he sell the remaining field
the first is equal to the selling price of the second. If
so as to make an overall profit of 10%?
the first is sold at 10% loss and the second at 25%
æ 110 ö gain, what is his total gain or loss?
Sol. SP of total agricultural field = Rs. çè 360000 ´ ÷
100 ø
Sol. 1st Cycle 2 nd Cycle Total
= Rs. 396000 [overall profit of 10%]
æ 100 ö
1 CP 100 100 ç = 80 180
SP of rd of the field è 125 ÷ø
3
æ 90 ö
SP 100 ç = 90 100 190
1
= ´ 360000 ´
80
éLoss of 20%ùû Þ Rs. 96000 è 100 ÷ø
3 100 ë
Total CP = (CP of 1st Cycle) + (CP of 2nd Cycle)
2
SP of th of the field = 100 + 80 = Rs. 180
5
Total SP = (SP of 1st Cycle) + (SP of 2nd Cycle)
2 125 = 90 + 100 = Rs. 190
= ´ 360000 ´ [Gain of 25%] Þ Rs. 180000
5 100 CP : SP = 180 : 190 = 18 : 19
SP of the remaining field
19 - 18
= Rs. (396000 – 96000 – 180000) = Rs. 120000 Profit = ´ 1710 = Rs. 90
8. One trader calculates the percentage of profit on the 19
buying price and another calculates on the selling 11. Ashish bought an article with 20% discount on the
price. When their selling price are the same, then labelled price. He sold the article with 30% profit on
difference of their actual profit is Rs. 85 and both the labelled price. What was his percent profit on the
claim to have made 20% profit. What is the selling price he bought?
price of each? Sol. Let the labelled price of the article be Rs. x
Sol. For first trader,
Let the CP of the article of Rs. 100, SP = Rs. 120 æ 100 - 20 ö 4x
Cost Price = x ç = Rs.
For second trader, SP of the article = Rs. 120 è 100 ÷ø 5
Gain = 20% [For both the traders]
Let the CP be x æ 100 + 30 ö 13
Selling Price = x ç = Rs. x
è 100 ÷ø 10
120 - x 20
´ 100 = 20 Þ 120 – x = ´6
120 5 13 4 13x - 8x x
Profit = x- x = = [SP – CP]
Þ 120 – x = 24 Þ x = 120 – 24 = Rs. 96 10 5 10 2
Gain = Rs. 24 [SP – CP]
Difference of gain = 24 – 20 = Rs. 4 x
%Profit = 2 ´ 100 = 5 ´ 100 = 125 = 62.5%
If the difference of gain be Rs. 4, then
4x 8 2
SP = Rs. 120 5
When the difference be Rs. 85, then 12. A shopkeeper sold an article for Rs. 400 after giving
120 20% discount on the labelled price and made 20%
SP = ´ 85 = Rs. 2550
4 profit on cost price. What was the percentage profit,
had he not given the discount?
1 1
9. If the sales tax be reduced from 3 % to 3 %. What 400 ´ 100
2 3 Sol. Labelled Price = [Before discount of 20%]
difference does it make to person who purchases an 80
article whose marked price is Rs. 8400? = Rs. 500

149 @BEST300MCQ For More Study Material


Visit: studyiq.com
Cost Price of article
8500 ´ 100
400 ´ 100 1000 Sol. Original Company price = = Rs. 10000
= = Rs. 100 - 15
[20% profit on CP]
120 3 Let the total selling price be Rs. x.
Now, according to the question,
1000 1500 - 1000
500 - x - 10000
3 ´ 100 = 3 ´ 100 ´ 100 = 10 [Profit of 10%]
Profit% =
1000 1000 10000
3 3 100x – 1000000 = 100000 Þ x = Rs. 11000
Total selling price = Rs. 11000
500 15. A publisher published 5000 books in 5 lakh rupees.
= ´ 100 = 50%
1000 2
13. A reduction of 20% in the price of mangoes enables a He gives 500 books in free, rd of the rest he sell on
3
person to purchase 12 more for Rs. 15. Find the price
of 16 mangoes before reduction? 1
Sol. Let the price of 1 mango be x paise 20% discount and remaining rd on M.P. He also
3
Number of mangoes for gives 20% commission of the total selling. Find the
1500 profit% of the publisher if market price of each book
Rs. 15 = [Rs. 1 = 100 paise] is Rs. 200?
x
Sol. Total number of books = 5000
New price of one mango = ( 80% of x) paise he gives free book = 500
80 4 4
= ´ x = x paise SP of Ist part = 3000 ´ 200 ´ = Rs. 480000
100 5 5

æ 1500 ´ 5 ö 2
Number of mangoes for Rs. 15 = ç [20% Discount on rd of rest]
è 4x ÷ø 3
SP or IInd part = 1500 × 200 = Rs. 300000
7500 1500
- = 12 [Diff. as mentioned in the Ques.] 1
4x x [Price is MP of
rd of the rest]
3
x = 31.25
Total SP = 480000 + 300000 = Rs. 780000
Cost of 16 mangoes before reduction
80
31.25 ´ 16 Total SP after Commission = ´ 780000
= = Rs. 5 100
100
[20% Commission]
14. A garment company declared 15% discount for
= Rs. 624000
wholesale buyers. Mr. Hemant bought garments from
Total CP = Rs. 5,00,000, Total SP = Rs. 6,24,000
the company for Rs. 8500 after getting discount. He
Net profit = 6,24,000 – 5,00,000 = 1,24,000
fixed up selling price of garments in such a way that
he earned a profit of 10% on original company price. 124000
Profit% = ´ 100 = 24.8%
What is the total selling price? 500000

Ques ti ons 3. Find SP when CP = Rs. 56.25 and Gain = 20%?


(a) Rs. 72 (b) Rs. 67.5
1. A man buys an article for Rs. 27.50 and sells it for Rs. (c) Rs. 50 (d) Rs. 75
28.60. Find the gain percent? 4. Find SP when CP = Rs. 80.40, loss = 5%?
(a) 4% (b) 3% (a) Rs. 81 (b) Rs. 84.72
(c) 5% (d) 10% (c) Rs. 76.38 (d) Rs. 82.9
2. If a radio is purchased for Rs. 490 and sold for Rs. 5. Find CP when SP = Rs. 40.60, gain = 16%?
465.50. Find the loss%?
(a) Rs. 35 (b) Rs. 50
(a) 6% (b) 5%
(c) Rs. 75 (d) Rs. 89
(c) 4% (d) 3%

150 @BEST300MCQ For More Study Material


Visit: studyiq.com
6. Find CP when SP = Rs. 51.70, loss = 12%? (a) Rs. 400 (b) Rs. 300
(a) Rs. 58.75 (b) Rs. 62.25 (c) Rs. 200 (d) Rs. 500
(c) Rs. 65 (d) Rs. 69.27 17. By selling a book for Rs. 360, 20% profit was earned.
7. A person incurs 5% loss by selling a watch for Rs. What is the CP of the book?
1140. At what price should the watch be sold to earn (a) Rs. 300 (b) Rs. 200
5% profit? (c) Rs. 250 (d) Cannot be determined
(a) Rs. 1380 (b) Rs. 1160 18. Profit earned by selling an article at Rs. 1630 is same
(c) Rs. 1260 (d) Rs. 1400 as the loss incurred by selling the article for Rs. 1320.
8. If the cost price is 96% of the selling price, then what What is the CP?
is the profit percent? (a) Rs. 1475 (b) Rs. 1300
(a) 5.72% (b) 3.72% (c) Rs. 1350 (d) Rs. 1275
(c) 8.92% (d) 4.17% 19. If the CP of 50 items is equal SP of 40 items then what
9. A dishonest dealer professes to sell his goods at cost is the profit or loss%?
price but uses a weight of 960 gms instead of a Kg (a) 20% (b) 15%
weight. Find his gain%?
(c) 25% (d) 35%
27 8 20. If a banana's cost is Rs. 1.25 and apple's cost is Rs.
(a) % (b) %
4 3 1.75 what will be the cost of 2 Dozen of Banana and
3 Dozen of apple?
25 21 (a) Rs. 93 (b) Rs. 83
(c) % (d) %
6 4 (c) Rs. 85 (d) Rs. 70
10. A man sold two cows at Rs. 1995 each. On one he lost 21. Nutan bought a watch with 24% discount. If she pays
10% and on the other he gained 10%. What his gain Rs. 779 for that watch then what is the marked price
or loss percent? of watch?
(a) 4% (b) 2% (a) Rs. 950 (b) Rs. 975
(c) 0.5% (d) 1% (c) Rs. 1000 (d) Rs. 1025
11. Two discounts of 40% and 20% equal to a single 22. Nutan pays Rs. 2140 for 3 calculator and 4 Pen while
discount of? he pay Rs. 1355 for an additional calculator and 5
(a) 48% (b) 53% Pen. Then what he paid for Calculator only?
(c) 52% (d) 60% (a) Rs. 175 (b) Rs. 480
12. Amit buys 5 watches for Rs. 9450 and later sells them (c) Rs. 655 (d) Can't be determined
for Rs. 9700. How much profit does Amit make per
23. A vendor bought toffees at 6 for a rupee. How many
watch?
for a rupee must he sell to gain 20%?
(a) Rs. 75 (b) Rs. 80
(a) 3 (b) 4
(c) Rs. 60 (d) Rs. 50
(c) 5 (d) 6
13. The price of 12 chair and 8 table is Rs. 676. What is
24. A man sold his two horses for Rs. 770 each, on one he
the price of 21 chair and 14 table?
gained 10% & on the other he lost 10%. The average
(a) Rs. 1183 (b) Rs. 4732
gain or loss percentage is
(c) Rs. 1180 (d) Cannot be determine
(a) 100% (b) 0.96%
14. Aditya sold TV to Sanjay at 12% more than the CP. If
(c) 4% (d) 1%
Sanjay paid Rs. 17696 for that TV then what was the
original price of the TV? 4
25. If the selling price of an article is rd of its cost price
(a) Rs. 15,500 (b) Rs. 15,820 3
(c) Rs. 15,520 (d) Rs. 15,800 the profit in the transaction is
15. Amit purchased 13 chair of Rs. 115 each and sold all
2 1
at Rs. 1220. Then find the profit or Loss on the (a) 16 % (b) 20 %
transaction 3 2
(a) Rs. 280 Loss (b) Rs. 275 Loss 1 1
(c) Rs. 325 Profit (d) Rs. 350 Profit (c) 25 % (d) 33 %
2 3
16. Aditya purchase a book with a 20% discount on the 26. A sells his house worth Rs. 10 lakh to B at a loss of
marked price. How much did he pay if the book 10%. Later B sells it back to A at 10% profit. The result
marked was Rs. 500? of the two transactions is

151 @BEST300MCQ For More Study Material


Visit: studyiq.com
(a) A neither loses nor gains 33. The cost price of 12 tables is equal to the selling price
(b) A loses Rs. 10,000 of 16 tables. The loss percent is
(c) A loses Rs. 2,00,000 (a) 15% (b) 20%
(d) B gains Rs. 1,10,000 (c) 25% (d) 30%
27. A fair price shopkeeper takes 10% profit on his goods. 34. Two continuous discounts of 4% on any thing should
He lost 20% goods during theft his loss% is be equal to
(a) 8% (b) 10% (a) 8% (b) 7.92%
(c) 7.84% (d) 8.08%
(c) 11% (d) 12%
35. A sells a bicycle to B at a profit of 20% and B sells it to
28. Aditya bought 200 dozen orange at Rs. 10 per dozen
C at a profit of 25%. If C pays Rs. 1500, what did A
and he spent Rs. 500 on transportation. He sold each
pay for it?
orange at Rs. 1 each. What was his profit or loss%?
(a) Rs. 825 (b) Rs. 1000
(a) 5% (b) 6%
(c) Rs. 1100 (d) Rs. 1125
(c) 4% (d) Can't be determine
36. Ram purchases a chair at Rs. 70 and spent Rs. 17 on
29. If 11 Mango are bought for Rs. 10 and sold at 10 for
its repair and 50 paise on cartage. If he sold the chair
Rs. 11. What was Gain or Loss%?
at Rs. 100 then his approximate margin of profit will
(a) 24% (b) 21% be?
(c) 26% (d) 25% (a) 13.30% (b) 11.25%
30. The cost price of 20 articles is the same as the selling (c) 12.5% (d) 14.3%
price of x articles. If the profit is 25%, then the value 37. A shopkeeper marks his goods 20% above CP but
of x is: allows 30% discount for cash. His net loss is?
(a) Rs. 15 (b) Rs. 16 (a) 8% (b) 20%
(c) Rs. 18 (d) Rs. 25 (c) 10% (d) 16%
31. If selling price is doubled, the profit triples. Find the 38. A single discount, equivalent to a successive discount
profit percent: of 40% and 30% is?
2 (a) 55% (b) 56%
(a) 66 % (b) 100%
3 (c) 57% (d) 58%
39. If the CP of 13 bats is Rs. 390. What is the price when
1 it is sold at 10% loss?
(c) 105 % (d) 120%
3 (a) Rs. 200 (b) Rs. 300
32. Some articles were bought at 6 articles for Rs. 5 and (c) Rs. 350 (d) Rs. 351
sold at 5 articles for Rs. 6. Gain percent is:
40. If an item is sold for Rs. 924 then there is a profit of
1 10%. What is the cost price?
(a) 30% (b) 33 %
3 (a) Rs. 840 (b) Rs. 860
(c) 35% (d) 44% (c) Rs. 880 (d) Rs. 900

1. The cost of an article including the sales tax is Rs. 3. A tradesman gives 4% discount on the marked price
616. The rate of sales tax is 10%, if the shopkeeper and gives one article free for buying every 15 articles
has made a profit of 12%, then the cost price of the and thus gains 35%. The marked price is appox. how
article is? much percent above the CP?
(a) Rs. 350 (b) Rs. 500 (a) 20% (b) 30%
(c) Rs. 650 (d) Rs. 800 (c) 40% (d) 50%
2. Two third of consignment was sold at a profit of 5% 4. When a producer allows 36% concession on the retail
and the remainder at a loss of 2%. If the total profit price of his product, he earns a profit of 8.8%. What
was Rs. 400, the value of the consignment was? would be his profit percent if the commission is
reduced by 24%?
(a) Rs. 12000 (b) Rs. 14000
(a) 48.2% (b) 49%
(c) Rs. 15000 (d) Cannot be determined
(c) 49.6% (d) 51%

152 @BEST300MCQ For More Study Material


Visit: studyiq.com
5. A person earns 15% on investment but loses 10% on (a) Rs. 72.5 (b) Rs. 71
another investment. If the ratio of the two investments (c) Rs. 72 (d) Rs. 70
be 3 : 5, what is the gain or loss on the two investments
14. A person sells 36 apple per rupee and suffers a loss
taken together?
of 4%. Find how many apple per rupees to be sold to
(a) 0.625% (b) 0.8% have a gain of 8%.
(c) 0.9% (d) 1.2% (a) 32 (b) 16
6. The profit earned by selling an article for Rs. 900 is
(c) 4 (d) 15
double the loss incurred when the same article is
sold for Rs. 450. At what price should the article be 15. Ram sells an article to Girish at a gain 20%, Girish
sold to make 25% profit? sells it to Sanjay at a gain of 10% and Sanjay sells it to
(a) Rs. 400 (b) Rs. 500 1
Aditya at a gain of 12 % . If aditya pay Rs. 59.40.
(c) Rs. 700 (d) Rs. 750 2
7. A shopkeeper sold some article at the rate of Rs. 35 What did it cost Ram?
per article and earned profit of 40%. At what price (a) Rs. 40 (b) Rs. 22
each article should have been sold so that profit of (c) Rs. 24 (d) Rs. 18
60% was earned.
(a) Rs. 45 (b) Rs. 42 3
16. A fruit seller sells th part of fruit at a profit of 10%
(c) Rs. 39 (d) Rs. 40 5
8. Due to a 20% rise in price of sugar, a bachelor is able and remaining at a loss of 5%. If the total profit is Rs.
to buy 1.5 kg less for Rs. 135. What is the increased 1500 then what is the total cost price of fruit?
price of sugar per kg? (a) Rs. 37500 (b) Rs. 37000
(a) Rs. 15 (b) Rs. 21 (c) Rs. 36500 (d) Rs. 36000
(c) Rs. 18 (d) Rs. 24 17. A shopkeeper buys a toy at Rs. 100 and sells it at Rs.
9. A trader mixes 26 kg of rice at Rs. 20 per kg with 30 125. Another shopkeeper buys the same toy at Rs.
kg of rice of other variety at Rs. 36 per kg and sells the 125 but sells it at Rs. 100. What are the respective
mixture at Rs. 30 per kg. His profit percent is: profit and loss percentages for the two shopkeepers.
(a) No profit, no loss (b) 5% (a) 20%, 20% (b) 25%, 20%
(c) 8% (d) 10% 2
10. A TV set is being sold for Rs. x in Chandigarh. A dealer (c) 25%, 25% (d) 25%, 16 %
3
went to Delhi and bought the TV at 20% discount (from
the price of Chandigarh). He spent Rs. 600 on 18. The marked price of a bed is Rs. 2400. The shopkeeper
transport. Thus he sold the set in Chandigarh for Rs. x gives successive discounts of 10% and X% to
customer. If the customer pays Rs. 1836 for the bed
2 then find the value of X?
making 14 % profit. What was x?
7 (a) 15% (b) 18%
(a) Rs. 9600 (b) Rs. 8800 (c) 12% (d) 10%
(c) Rs. 8000 (d) Rs. 7200 19. Three successive discounts of 10%, 12% and 15%
11. Sanjay purchased a chair marked at Rs. 800 at 2 will amount to a single discount of?
successive discount of 10% and 15% respectively. (a) 36.28% (b) 32.68%
He spent Rs. 28 on transportation and sold the chair
for Rs. 800. How much is his gain percentage? (c) 34.68% (d) 37%
(a) 14% (b) 30% 20. A loss of 19% gets converted into profit of 17% when
the selling price is increased by Rs. 162. Find the cost
(c) 25% (d) 40%
price of the article.
12. When a book is sold at its Marked Price it gives a
(a) Rs. 300 (b) Rs. 350
profit of 40%. What will happen if it is sold at half
the marked Price? (c) Rs. 400 (d) Rs. 450
(a) 30% profit (b) 25% loss 21. A shopkeeper sold an article offering discount of 5%
and earn a profit of 23.5%. What would have been
(c) 30% loss (d) 40% profit
the percentage of profit earned if no discount has
13. Aditya purchased 14 shirt & 25 pants at Rs. 45 and been offered?
Rs. 55 each respectively. What should be the
(a) 23% (b) 30%
approximate overall average selling price of shirt and
pant so that 40% profit is earned? (c) 33% (d) Cannot be determined

153 @BEST300MCQ For More Study Material


Visit: studyiq.com
22. If Aditya sells an article to Nutan at 10% gain, while 30. An electric pump was sold at a profit of 15%. Had it
Nutan sells it to Manish at 20% gain at Rs. 1914 then been sold for Rs. 600, the profit would have been
what is the Cost Price? 20%. The former selling price is
(a) Rs. 1450 (b) Rs. 1340 (a) Rs. 500 (b) Rs. 540
(c) Rs. 1560 (d) Rs. 1780 (c) Rs. 575 (d) Rs. 600
23. Rita buys an article for Rs. 9600. She sells it at 12% 31. On a Rs. 10,000 payment order, a person has a choice
loss and gets some money and from that money she between three successive discounts of 10%, 10% and
again buys an article and this time she gets 12% profit. 30% and three successive discounts of 40%, 5% and
What is the profit or loss she gets from this 5%. By choosing the better offer, he can save?
transaction? (a) Rs. 200 (b) Rs. 225
(a) Rs. 130 (b) Rs. 138 (c) Rs. 400 (d) Rs. 255
(c) Rs. 138.24 (d) Rs. 138.42 32. A man sells each of his 2 articles for Rs. 99. On one he
gains 10% and on the other he incurred a loss of 1%.
24. Nutan bought 30 dozens of oranges for her juice stall What is his total gain?
in the school fair. She paid Rs. 8 per dozen of oranges.
She also had to pay Rs. 500 as the stall fee to the 4
(a) 9% (b) 4 %
school authorities. She calculated that each glass of 19
juice would need 3 oranges. How much should she (c) 4.5% (d) 5.5%
charge per glass of juice so as to make 20% profit?
33. The market price of an article is Rs. 100. If the article
(a) Rs. 7.20 (b) Rs. 7.40 is sold at a discount of 10%, then 35% profit is realised.
(c) Rs. 7.60 (d) Rs. 7.80 How much loss or profit will be made if it is sold for
25. Aditya bought a scooter for a certain sum of Money. Rs. 30 less than market price?
He spend 15% of cost price on repair and sold it for a (a) 5% loss (b) 8% gain
profit on Rs. 1104. What is the CP of Scooter? (c) 5% gain (d) 8% loss
(a) Rs. 600 (b) Rs. 700 34. A shopkeeper sold an article offering discount of 24%
(c) Rs. 800 (d) Rs. 900 and earn a profit of 23.5%. What would have been
the percentage of profit earned if no discount had
26. In a certain store, the profit is 320% of the cost. If the
been offered?
cost increases by 25% but the selling price remains
constant, approximately what percentage of the (a) 63% (b) 62.50%
selling price is the profit? (c) 60% (d) Cannot be determined
(a) 30% (b) 70% 35. On selling an article for Rs. 500 the loss accrued is
20%. To make a profit of 20% the article must be sold
(c) 100% (d) 250% at?
27. A shopkeeper sells one transistor for Rs. 840 at a (a) Rs. 700 (b) Rs. 750
gain of 20% and another for Rs. 960 at a loss of 4%.
(c) Rs. 800 (d) Rs. 900
His total gain or loss percent is:
36. The CP of 19 article is equal to the selling price of 15
15 15 article. Gain % is?
(a) 5 % loss (b) 5 % gain
17 17 1
(a) 26% (b) 26 %
2 1 3
(c) 6 % gain (d) 6 % Loss
3 2 2
(c) 12% (d) 26 %
28. A dealer buys a table listed at Rs. 600 and gets 3
successive discount of 10% and 20%. What is his 37. The cash difference between the selling price of an
profit or loss percent if he sells it at Rs. 540? article at a profit of 4% and 6% is Rs. 3. The ratio of
(a) 25% (b) 20% the 2 selling price is?
(a) 50 : 53 (b) 52 : 53
1
(c) 15% (d) 17 % (c) 51 : 53 (d) Cannot be determined
2
38. A shopkeeper has to sell 24 Kg of sugar. He sells a
29. Sonal buys mangoes at the rate of 3 kgs for Rs. 21 and part of these at a gain of 20% and the rest at a loss of
sells them at 5 kgs for Rs. 50. To earn a profit of Rs. 5%. If on the whole he earns a profit of 10%, the
102, he must sell how many mangoes? amount of Sugar sold at a loss is?
(a) 34 kgs (b) 52 kgs (a) Rs. 7.5 (b) Rs. 9.6
(c) 26 kgs (d) 32 kgs (c) Rs. 10 (d) Cannot be determined

154 @BEST300MCQ For More Study Material


Visit: studyiq.com
39. If a shopkeeper sells 25 articles at Rs. 45 per article 40. The price of a TV is Rs. 10,000. If successive discount
after giving 10% discount and earns 50% profit. If the of 15%, 10% and 5% allowed. Then at what price
discount is not given the profit gained is? does a customer buy?
(a) 66.67% (b) 32% (a) Rs. 7267.50 (b) Rs. 7000
(c) 35% (d) Cannot be determined (c) Rs. 7200 (d) Cannot be determined

1. Sarita sells a Phone at a profit of 20%. If she had (a) 7.1% (b) 7.5%
bought it at 20% less and sold it for Rs. 180 less, she (c) 7.9% (d) 8.1%
would have gained 25%. Find the cost price of the 7. Paras Health Care made 3000 strips of vitamin tablets
Phone. at a cost of Rs. 4800. The company gave away 1000
(a) Rs. 800 (b) Rs. 850 strips of tablets to doctors as free samples. A discount
(c) Rs. 900 (d) Rs. 1000 of 25% is given on the printed price. Find the ratio of
profit if the price is raised from Rs. 3.25 to Rs. 4.25
2. Ravi purchases 90 pens and sells 40 pens at a gain of
per strip and if at the latter price, samples to doctors
10% and 50 pens at a gain of 20%. Had he sold all of
were done away with.
them at a uniform profit of 15% he would have got
Rs. 40 less. Find the cost price of each pen. (a) Rs. 36.7 (b) Rs. 49.3
(c) Rs. 63.5 (d) Rs. 71.7
(a) Rs. 80 (b) Rs. 75
8. APD printed 3000 copies of 'Career Power' at a cost
(c) Rs. 90 (d) Rs. 100 of Rs. 240000. It gave 500 copies free to different
3. Savita buys 5 shirts and 10 pants for Rs. 1600. She philanthropic institutions. It allowed a discount of
sells shirts at a profit of 15% and pants at a loss of 25% on the published price and gave one copy free
10%. If her over all profit was Rs. 90, what was the for every 25 copies bought at a time. It was able to sell
cost price of a shirt and a pant? all the copies in this manner. If the published price is
(a) Rs. 175, Rs. 50 (b) Rs. 200, Rs. 50 Rs. 325, then what is its overall gain or loss
(c) Rs. 200, Rs. 60 (d) Cannot be determined percentage in the whole transaction?
(a) 89% gain (b) 120% loss
4. At a cost of 60 paise per article, Sarika produces 750
articles. She puts the selling price such that if only (c) 140% loss (d) 143.75% gain
600 articles are sold, she would have made a profit of 9. Surbhi bought a combined total of 25 i-pads and i-
40% on the outlay. However, 120 articles got spoilt phones. She marked up the i-pad by 20% on the cost
and she was able to sell 630 articles at this price. price, while each i-phone was marked up by Rs. 2000.
Find her actual profit or loss percent as the percentage She was able to sell 75% of the i-pads and 2 i-phones
of total outlay assuming that the unsold articles are and make a profit of Rs. 49000. The remaining i-pads
useless. and 3 i-phones could not be sold by her. Find her
overall profit or loss if she gets no return on unsold
(a) 47% profit (b) 51% profit
items and it is known that an i-phone costs 50% of an
(c) 36% loss (d) 28% loss i-pad.
5. Kritika bought 25 i-pads and i-phones for Rs. 205000. (a) Gain of Rs. 48500 (b) Loss of Rs. 48500
She sold 80% of the i-pads and 12 i-phones for a profit (c) Gain of Rs. 51400 (d) no profit, no loss
of Rs. 40000. Each i-pad was marked up by 20% over 10. A merchant buys 4000 kg of wheat, one-fifth of which
cost and each i-phone was sold at a profit of Rs. 2000. he sells at a gain of 5 per cent, one-fourth at a gain of
The remaining i-pads and 3 i-phones could not be 10%, one-half at a gain of 12 percent, and the
sold. What is Kritika's overall profit/loss? remainder at a gain of 16 percent. If he had sold the
(a) Rs. 500 profit (b) Rs. 1000 loss whole at a gain of 11 percent, he would have made
(c) Rs. 1500 profit (d) no profit, no loss Rs. 72.80 more. What was the cost price of the crop
6. Sasha goes to a shop to buy a sofa set and a center per kg?
table. She bargins for a 10% discount on the center (a) Rs. 2 (b) Rs. 2.60
table and 25% discount on sofa set. However, the (c) Rs. 2.50 (d) Rs. 2.80
manager, by mistake, interchanged the discount 11. Ajit calculates his profit percentage on the selling
percentage figures while making the bill and Sasha price whereas Rohit calculates his on the cost price.
paid accordingly. When compared to what she They find that the difference of their profits is Rs.
should pay for her purchases, what percentage did 100. If the selling price of both of them are the same
Sasha pay extra given that the center table costs 40% and both of them get 25% profit, find their selling
as much as the sofa set. price?

155 @BEST300MCQ For More Study Material


Visit: studyiq.com
(a) Rs. 1200 (b) Rs. 1500 16. Divyam bought an article with 15% discount on the
(c) Rs. 1800 (d) Rs. 2000 labelled price. He sold the article with 10% profit on
the labelled price. What was his percent profit on the
12. A pen was sold for a certain sum and there was a
price he bought?
loss of 20%. Had it been sold for Rs. 12 more, there
would have been a gain of 30%. What would be the 7 7
profit if the pen was sold for Rs. 4.80 more than what (a) 28 % (b) 29 %
17 17
it was sold for?
(a) 15% (b) 23% 5
(c) 29 % (d) Data inadequate
(c) 29% (d) no profit, no loss 17
13. A white goods dealer pays 10% custom duty on an i- 17. A shopkeeper sold Chairs at Rs. 2139 each after giving
phone that costs Rs. 25000 in UK. For how much 7% discount on labelled price. Had he not given the
should he mark it, if he desires to make a profit of discount, he would have earned a profit of 15% on
20% after giving a discount of 25% to the buyer? the cost price. What was the cost price of each chairs?
(a) Rs. 32000 (b) Rs. 38000 (a) Rs. 2500 (b) Rs. 2100
(c) Rs. 44000 (d) Cannot be determined (c) Rs. 2000 (d) Rs. 1900
14. A cab driver makes a profit of 20% on every trip when 18. A shopkeeper sold decks at Rs. 166 each after giving
he carries 3 passengers and the price of petrol is Rs.30 17% discount on labelled price. Had he not given the
a litre. Find the percentage profit for the same journey discount, he would have earned a profit of 25% on
if he goes with four passengers per trip and the price the cost price. What was the cost price of each deck?
of petrol reduces to Rs. 24 a litre? (a) Rs. 165 (b) Rs. 155
(Assume that revenue per passenger is the same in (c) Rs. 160 (d) Rs. 164
both the cases.) 19. A garment company declared 15% discount for
(a) 100% (b) 76% wholesale buyers. Mr. Ashish bought garments from
(c) 54% (d) 43% the company for Rs. 25000 after getting discount. He
fixed up the selling price of garments in such a way
1 that he earned a profit of 8% on original company
15. Anil bought an item with 12 % discount on the
2 price. What is the approximate total selling price?
1 (a) Rs. 28000 (b) Rs. 29000
labelled price. He sold the item with 17 % profit on
2 (c) Rs. 31700 (d) Rs. 28500
the labelled price. What was his percent profit on the 20. A garment company declared 14% discount for
price he bought? wholesale buyers. Mr. Swami bought garments from
the company for Rs. 860 after getting discount. He
1 fixed up the selling price of garments in such a way
(a) 35% (b) 34 %
7 that he earned a profit of 6% on original company
price. What is the approximate total selling price?
2 2
(c) 34 % (d) 35 % (a) Rs. 1060 (b) Rs. 1160
7 7
(c) Rs. 960 (d) Cannot be determined

1. A man buys 10 articles for Rs. 8 and sells them at the (a) Rs. 693 (b) Rs. 707
rate of Rs. 1.25 per article. His gain percent is: (c) Rs. 683 (d) Rs. 673
1 3. A reduction of 10 per cent in the price of potatoes
(a) 50 (b) 56 % enables me to obtain 25 kg more for Rs. 225. What is
4
the reduced price per kg? Find also the original price
1 per kg.
(c) 19 % (d) 20%
2 (a) Rs 0.9, Re 1 (b) Re 1, Rs 2
2. The profit earned by selling an article for Rs. 515 is (c) Rs. 0.8, Rs. 2 (d) None of these
equal to the loss incurred when the same article is 4. A 25% hike in the price of tea forces a person to
sold for Rs. 475. What should be the selling price of purchase 2 kg less for Rs. 75. Find the original price of
the article for making 40% per cent profit? the tea.

156 @BEST300MCQ For More Study Material


Visit: studyiq.com
(a) Rs. 7 (b) Rs. 8 (a) Rs. 550 (b) Rs. 450
(c) Rs. 7.5 (d) Rs. 8.5 (c) Rs. 650 (d) Rs. 500
5. A person buys 12 eggs for Rs. 15 and sells them at 10 14. A person bought two horses for Rs. 960. He sold one
for Rs. 14. Wht does he gain or lose per cent? at a loss of 20% and the other at a gain of 60% and he
(a) 10% loss (b) 12% profit found that each horse was sold at the same price.
(c) 12% loss (d) 15% profit Find the cost price of two horses:
6. A dishonest dealer sells the goods at 44% loss on cost (a) Rs. 640, Rs. 320 (b) Rs. 540, Rs. 420
price but uses 30% less weight. What is his percentage (c) Rs. 440, Rs. 520 (d) Rs. 650, Rs. 310
profit or loss?
5
(a) 20% gain (b) 28% gain 15. of a consignment was sold at 16% loss and the rest
8
(c) 20% loss (d) 25% loss at a profit of 8%. If there was an overall loss of Rs. 140,
7. A seller uses 1250 gm in place of one kg to sell his find the value of the consignment?
goods. Find his actual % profit or loss, when he sells (a) Rs. 2000 (b) Rs. 2500
his article on 25% gain on cost price?
(c) Rs. 1800 (d) Rs. 2400
(a) 25% loss (b) 15% gain
(c) Neither loss nor gain 2
16. If goods be purchased for Rs. 380, and rd be sold at
(d) Can’t be determined 3
8. A person sold a chair at a profit of 14%. Had he sold a loss of 15 per cent, at what gain per cent should the
it for Rs. 15 more, 17% would have been gained. Find remainder be sold so as to gain 10 per cent on the
the cost price. whole transaction.
(a) Rs. 550 (b) Rs. 650 (a) 60% (b) 70%
(c) Rs. 600 (d) Rs. 500 (c) 90% (d) 40%
9. I sold a book at a loss of 8%. Had I sold it for Rs. 120 3
more, 7% would have been gained. Find the cost price. 17. A man buys coffee for Rs. 7200. He sells th of it at a
8
(a) Rs. 900 (b) Rs. 800 profit of 24%. At what per cent gain should he sell
(c) Rs. 600 (d) Rs. 750
5
10. An article is sold at a profit of 15%. If both the cost remaining th so as to make an overall profit of 21%
8
price and selling price are Rs. 56 less, the profit would
on the whole transaction?
be 7% more. Find the cost price.
(a) Rs. 176 (b) Rs. 186 1 1
(a) 18 % (b) 20 %
(c) Rs. 156 (d) Rs. 196 5 5
11. An article is sold at 30% profit. If its cost price and
1
selling price are less by Rs. 25 and Rs. 19 respectively (c) 19 % (d) Data inadequate
the percentage profit increases by 20%. Find the cost 5
price. 18. Oranges are bought at 12 for a rupee and an equal
(a) Rs. 97 (b) Rs. 92.5 number more at 8 for a rupee. If these are sold at 15 for
(c) Rs. 98.5 (d) Rs. 98 a rupee, find the loss or gain per cent.
12. An article is sold at 45% profit. If its cost price is (a) 36% loss (b) 36% profit
increased by Rs. 80 and at the same time if its selling (c) 38% loss (d) 38% profit
price is also increased by Rs. 70, the percentage of 19. A person bought two tables for Rs. 860. He sold one at
profit becomes 30%. Find the cost price: a gain of 5% and the other at a gain of 10% and he
found that each table was sold at the same price. Find
2 the cost prices of the two tables.
(a) Rs. 246 (b) Rs. 226
3 (a) Rs. 440, Rs. 420 (b) Rs. 460, Rs. 400
1 1 (c) Rs. 480, Rs. 380 (d) Data inadequate
(c) Rs. 293 (d) Rs. 226 20. Raman buys 5 pens and 30 pencils for Rs. 1000. He
3 3
sells the pens at a profit of 15% and pencils at a profit
1 1 of 10% and makes a total profit of Rs. 120. Find the
13. of a commodity is sold at 25% profit, is sold at
5 10 cost of a pen and of a pencil.
40% profit and the rest at 30% profit. If a profit of Rs. (a) Rs. 70, Rs. 30 (b) Rs. 85, Rs. 15
150 is earned, then find the value of the commodity: (c) Rs. 80, Rs. 20 (d) Data inadequate

157 @BEST300MCQ For More Study Material


Visit: studyiq.com
21. A person bought some oranges at the rate of 12 per 29. What will be the percentage profit on selling an article
rupee. He bought the same number of oranges at the at a certain price if there is a loss of 25% when the
rate of 8 per rupee. He mixes both the types and sells article is sold at half of the previous selling price?
at 20 for rupees 2. In this business he bears a loss of (a) 45% (b) 60%
Rs. 8. Find out how many oranges he bought in all? (c) 55% (d) 50%
(a) 1680 (b) 1820 30. Jeevan bought an article with 30 per cent discount on
(c) 1920 (d) 1290 the labelled price. He sold the article with 12 per cent
22. A man buys two horses for Rs. 1550. He sells one so profit on the labelled price. What was his per cent
as to lose 23% and the other so as to gain 27%. On the profit on the price he bought?
whole he neither gains nor loses. What does each (a) 40 (b) 50
horse cost? (c) 60 (d) Data inadequate
(a) Rs. 807, Rs. 743 (b) Rs. 817, Rs. 733 31. A shopkeeper sold an article for Rs. 210 after giving
(c) Rs. 827, Rs. 723 (d) Rs. 837, Rs. 713 16% discount on the labelled price and made 5% profit
on the cost price. What would have been the
23. A trader bought a car at 5% discount on its original percentage profit, had he not given the discount?
price. He sold it at 10% increase on the price he bought
(a) 50% (b) 25%
it. What percentage of profit did he make on the
original price? (c) 30% (e) 20%
(a) 5.5% (b) 6.5% 32. Alok bought 25 kg of rice of Rs. 6.00 per kg and 35 kg
of rice at the rate of Rs. 7.00 per kg. He mixed the two
(c) 4.5% (d) 3.5% and sold the mixture at the rate of Rs. 6.75 per kg.
24. 20 kg of potato costs as much as 5 kg of tomato, 12 kg What was his gain or loss in this transaction?
of tomato costs as much as 30 kg of onion, 15 kg of (a) Rs. 16.00 gain (b) Rs. 16.00 loss
onion costs as much as 18 kg of cabbage. If 10 kg of (c) Rs. 20.00 gain (d) Rs. None of these
cabbage costs Rs. 50. find the cost of 24 kg of potato.
33. Prateek sold a music system to Kartik at 20% gain
(a) Rs. 90 (b) Rs. 72 and Kartik sold it to Swastik at 40% gain. If Swastik
(c) Rs. 108 (d) Rs. 96 paid Rs. 10500 for the music system. What did Prateek
25. A man sells two articles, each for the same price Rs. pay for the music system?
640. He earns 20% profit on the first and 10% profit (a) Rs. 8240 (b) Rs. 7500
on the second. Find his overall per cent profit. (c) Rs. 6250 (d) Cannot be determined
(a) 14.78% (b) 14.08% 34. The owner of a toy shop charges his customer 33%
(c) 14.58% (d) None of these more than the cost price. If the customer paid Rs. 4921
for a toy, then what was the cost price of the toy?
26. Miheer calculates his profit percentage on the selling
price whereas Satya calculates his profit on the cost (a) Rs. 3850 (b) Rs. 3700
price. They find that the diffference of their profits is (c) Rs. 3550 (d) Rs. 3900
Rs. 110. If the selling price of both of them are the 35. Rani bought a piece of cloth for Rs. 950 and spent Rs.
same, and Miheer gets 10% profit and Satya gets 5% 300 on designing it. At what price should she sell it to
profit, then find their selling price. make 30% profit?
(a) Rs. 2100 (b) Rs. 2250 (a) Rs. 1650 (b) Rs. 1550
(c) Rs. 1750 (d) Rs. 2200 (c) Rs. 1525 (d) Rs. 1625
27. A man sells two horses for Rs. 11900. The cost price of 36. Naresh purchased a TV set for Rs. 11250 after getting
the first is equal to the selling price of the second. If discount of 10% on the labelled price. He spent Rs.
the first is sold at 30% loss and the second at 25% 150 on transport and Rs. 800 on installation. At what
gain, what is his total gain or loss (in rupees)? price should it be sold so that the profit earned would
have been 15%, if no discount was offered?
(a) Rs. 600 loss (b) Rs. 700 loss
(a) Rs. 12938 (b) Rs. 14030
(c) Rs. 750 gain (d) Rs. 700 gain
(c) Rs. 13450 (d) Rs. 15467
28. A dealer sells a table for Rs. 405, making a profit of 37. The owner of an electronics shop charges his customer
35%. He sells another table at a loss of 30%, and on 22% more than the cost price. If a customer paid Rs.
the whole he makes neither profit nor loss. What did 10980 for a DVD player, then what was the cost price
the second table cost him? of the DVD player?
(a) Rs. 360 (b) Rs. 350 (a) Rs. 8000 (b) Rs. 8800
(c) Rs. 340 (d) Rs. 300 (c) Rs. 9500 (d) Rs. 9000

158 @BEST300MCQ For More Study Material


Visit: studyiq.com
38. A company pays a rent of Rs. 25000 per month for profit of 10% on his purchase price. What is his
office space to its owner. But if the company pays the percentage profit on actual price of the mobile phone?
annual rent at the beginning of the year, the owner (a) 2% (b) 3%
gives a discount of 5% on the total annual rent? What
(c) 4.5% (d) 5%
is the annual amount the company pays to the owner
after the discount? 40. In a sale, a perfume is available at a discount of 15%
on the selling price. If the perfume’s discounted selling
(a) Rs. 285000 (b) Rs. 275000
price is Rs. 3675.4, what was the original selling price
(c) Rs. 300000 (d) Rs. 295000 of the perfume?
39. Nandu purchased a mobile phone and got a disocunt (a) Rs. 4324 (b) Rs. 4386
of 5% from dealer. He sold that mobile phone at a
(c) Rs. 4400 (d) Rs. 4294

S olut ions 7. (c); Let the new SP be Rs. x then

100 - loss% 100 + gain%


=
1. (a); CP = Rs. 27.50, SP = Rs. 28.60 1st SP 2nd SP
Then Gain = SP – CP = 28.60 – 27.50 = Rs. 1.10 (CP is same in both case)
gain ´ 100 100 - 5 100 + 5 105 ´ 1140
Since, Gain% = % =
CP , x= = Rs. 1260
1140 x 95
1.10 ´ 100 8. (d); Let SP = Rs. 100 then CP = Rs. 96
Þ Gain% = = 4%
27.50 Profit = SP – CP = 100 – 96 = Rs. 4
2. (b); CP = Rs. 490, SP = Rs. 465.50 profit 4
Loss = CP – SP = 490 – 465.50 = Rs. 24.50 Profit% = ´ 100% = ´ 100 = 4.17%
CP 96
loss ´ 100 24.50 ´ 100 9. (c); Here, True weight = 1000g.
Loss% = % = = 5%
CP 490 False weight = 960g.
Error change = (1000 – 960)g. = 40g.
é 100 + gain% ù
3. (b); SP = ê ú ´ CP
ë 100 û Error change
Þ Gain% = ´ 100%
True weight – Error
é 100 + 20 ù
Þ SP = ê 56.25 = Rs. 67.50
ë 100 úû =
40 25
´ 100% = %
1000 - 40 6
100 - loss% ù
4. (c); SP = éê úû ´ CP
10. (d); Here, since both gain and loss percent is same,
ë 100 hence the resultant value would be loss percent
only.
é 100 - 5 ù
Þ SP = ê ´ 80.40 = Rs. 76.38
ë 100 úû Þ Loss% =
a2
éëwhere a = 10%ùû
100
100 ´ SP = 1%
5. (a); CP =
100 + gain%
11. (c); Using net discount formula
100 ´ 40.60 é ab ù
Þ CP = = Rs. 35 Þ
100 + 16 êa + b - 100 ú %
ë û
Here, a = 40%, b = 20%
6. (a); CP = 100 ´ SP
100 - loss% Applying both values in above formula:

100 ´ 51.70 é 40 ´ 20 ù
Þ CP = = Rs. 58.75 Þ ê 40 + 20 - 100 ú % = 52%
100 - 12 ë û

159 @BEST300MCQ For More Study Material


Visit: studyiq.com
12. (d); Using simple formula of 20. (a); Cost of 1 banana = Rs. 1.25
Profit = SP – CP = 9700 – 9450 = Rs. 250 Cos of 1 apple = Rs. 1.75
[Total 5 watches] Cost of 2 dozen banana = Rs. 24 × 1.25 = Rs. 30
Cost of 3 dozen apple = Rs. 36 × 1.75 = Rs. 63
Rs. 250
Profit on 1 watch = = Rs. 50 Total cost = Rs. (30 + 63) = Rs. 93
5
21. (d); The formula to determine MP of watch if we are
13. (a); Here, cost of 12 chairs and 8 tables = Rs. 676 given SP and discount% is:
On dividing above equation by 4
é SP ù
1 Þ êë 100 - D% ´ 100 = MP úû
Þ Cost of 3 chairs and 2 tables = Rs. 676 ×
4
Now multiply it by 7 779
MP = ´ 100 = Rs. 1025
Þ Cost of 21 chairs and 14 tables 76
22. (b); Let cost of 1 calculator be Rs. 'C'
7
= Rs. 676 × = Rs. 1183 and cost of 1 pen be Rs. 'P'
4 According to question:
14. (d); Let CP be Rs. 100 3C + 4P = 2140 ...(i)
Then SP = Rs. 112. (12% more than CP) 1C + 5P = 1355 ...(ii)
Þ Now if SP = Rs. 17696 Solving (i) and (ii)
Then by unitary method: We get C = Rs. 480 [cost of 1 calculator]
100 1
Þ CP = ´ 17696 = Rs. 15800 23. (c); CP of 6 toffees = Rs. 1, CP of 1 toffee = Rs.
112 6
15. (b); Total SP given = Rs. 1220 SP of x toffees = Rs. 1
Total CP of 13 chairs = Rs. 13 × 115 = Rs. 1495 (where x is no. of toffees to sell)
Þ Hence, CP > SP 1
Þ Loss = CP – SP = Rs. 1495 – 1220 = Rs. 275 SP of 1 toffee = Rs.
x
16. (a); Here, MP = Rs. 500
Now since we need discount of 20% 1 1
-
20 1 1 1 1
é 20 ù Gain% = = x 6 Þ ´ = - Þ x=5
Þ Amount paid = Rs. ê 500 - 500 ´ 100 1 5 6 x 6
ë 100 úû = Rs. 400
6
17. (a); Here, profit% = 20%
a2
é SP - CP ù 20 360 - CP 24. (d); Using Net Loss formula = % [when P% = L%]
Þ P% = ê ´ 100% Þ = 100
ë CP û ú 100 CP
(10) 2
Þ CP = Rs. 300 = % = 1% Loss
100
18. (a); Here, profit = loss ...(i)
25. (d); Let CP = Rs. 1
Þ Here, profit = (SP)1 – (CP)
and, Loss = (CP) – (SP)2 4 4 1
SP = Rs. (Given) , Pr ofit = - 1 =
Now putting these values in (i) 3 3 3
(SP)1 – (CP) = (CP) – (SP)2
1 1
Pr ofit% = ´ 100 = 33 %
(SP)1 + (SP) 2 1630 + 1320 3 3
CP = = Rs. = Rs. 1475
2 2 2
æ aö
19. (c); As, CP of 50 items = SP of 40 items 26. (b); Here, Net Loss% = ç ÷ % = 1% Loss
è 10 ø
Þ 50 × (CP of 1 item) = 40 × (SP of 1 item)
Þ So, Loss of 1% on Rs. 10 lakh = Rs. 10,000
CP of 1 item 40 4 27. (d); Using net effective formula,
Þ = =
SP of 1 item 50 5
10 ´ 20
Þ 10 - 20 - = 12% Loss
SP - CP 5 - 4 100
Pr ofit% = = ´ 100 = 25%
CP 4 Hence, 12% Loss

160 @BEST300MCQ For More Study Material


Visit: studyiq.com
28. (c); Total CP = 200 × 10 + 500 = Rs. 2500
é ab ù
Total SP = 1 × 200 × 12 = Rs. 2400 34. (c); Using net discount formula = êa + b - úû %
ë 100
100 where a = b = 4%
%loss = ´ 100 = 4%
2500
é 4 ´ 4ù
29. (b); CP of 11 Mangoes = Rs. 10 Þ êë 4 + 4 - 100 úû % = 7.84%
é 10 ù 100
Þ CP of 10 Mangoes = Rs. ê10 ´ ú = Rs. 35. (b); Let CP for A be Rs. 100
ë 11 û 11 A sells it to B at 20% profit
SP of 10 Mangoes = Rs. 11 Rs. [100 + 20% of 100] = Rs. 120
100 Now B sells it to C at 25% profit
11 - Rs. [120 + 25% of 120] = Rs. 150
%profit = 11 ´ 100% = 21%
100 If C buys at Rs. 150, A bought at Rs. 100
11 Hence, by unitary method,
30. (b); CP of 20 articles = SP of x articles é 100 ù
If C bought at Rs. 1500, A paid = Rs. ê ´ 1500 ú
20 × CP of 1 article = x × SP of 1 article ë 150 û
CP of 1 article x = Rs. 1000
Þ =
SP of 1 article 20 36. (d); Total CP = Rs. [70 + 17 + 0.5] = Rs. 87.50
SP = Rs. 100
25 SP - CP 1 20 - x
Pr ofit% = = Þ =
100 CP 4 x 12.50
Pr ofit% = ´ 100 = 14.3%
x = Rs. 16 87.50
31. (b); Let profit be P 37. (d); Let CP = Rs. 100, MP = Rs. 120
Now, SP – CP = P ...(i) 70
In given question, when SP is doubled, P get The SP after discount = 120 ´ = 84 Rs.
100
tripled
[30% discount]
2SP – CP = 3P ...(ii)
So loss = 16% [CP – SP]
On solving (i) and (ii)
38. (d); Using net discount formula
We get CP = P and SP = 2P
é 40 ´ 30 ù
Pr ofit% =
2P - P
´ 100 = 100% êë 40 + 30 - 100 úû = 58%
P
39. (d); Given Loss% = 10%
25
32. (d); CP of 6 articles = Rs. 5, CP of 5 articles = Rs. æ CP - SP ö
6 Þ 10% = çè ÷ ´ 100
CP ø
SP of 5 articles = Rs. 6

25 10 390 - SP
6- = , SP = Rs. 351
6 ´ 100 = 11 ´ 100 = 44% 100 390
%gain =
25 25 40. (a); Here, Profit% = 10%
6
10 SP - CP 1 924 - CP
Þ = Þ =
33. (c); CP of 12 tables = SP of 16 tables 100 CP 10 CP
CP of 1 table 16 4 11CP = 9240 Þ CP = Rs. 840
= =
SP of 1 table 12 3

4-3
%Loss = ´ 100 = 25%
4

161 @BEST300MCQ For More Study Material


Visit: studyiq.com
1. (b); 110% of SP = 616 (Rate of sales tax = 10%) 5. (a); Let the investments be Rs. 3x and Rs. 5x
616 ´ 100 100 ´ SP Then total investment = 8x
SP = = Rs. 560 , CP = Total receipt = 115% of 3x + 90%of 5x
110 100 + gain%
3x 5x
100 ´ 560 = 115 ´ + 90 ´ = 7.95x
CP = = Rs. 500 100 100
100 + 12
Loss = CP – SP = 8x – 7.95x = 0.05x
2. (c); Let the total value be Rs x
2 2x 100
then value of rd = Rs. , loss% = 0.05x ´ = 0.625%
3 3 8x
6. (d); Let CP be Rs. x
1 x
value of rd = Rs. then, 900 – x = 2 (x – 450) [Profit = 2 Loss]
3 3
3x = 1800 Þ x = Rs. 600
According to question
CP = Rs. 600, gain required = 25%
2 æ 5 ö 1 æ 2 ö
xç ÷ - xç ÷ = 400 CP
3 è 100 ø 3 è 100 ø SP = ( 100 + gain%) ´
100
x x
- = 400 600
30 150 SP = ( 100 + 25) ´ = Rs. 750
100
5x - x 7. (d); Here initially SP of some article = Rs. 35
= 400 Þ x = Rs. 15000
150 Profit% = 40%
3. (d); Let the CP of each article be Rs. 100 Now, finally SP of articles = Rs. x
Then CP of 16 articles = Rs. (100 × 16) = 1600 Profit% = 60%
135 Here, CP is same in each case
SP of 16 articles = 1600 ´ = Rs. 2160
100 Þ (CP)1 = (CP)2
(1 article free)
(SP)1 (SP)2 35 x
Þ = , = , x = Rs.
2160 100 + P1 % 100 + P2 % 140 160
SP of each article = = Rs. 144
15
40
If SP is Rs. 96, marked price = Rs. 100 8. (c); Let price of sugar per kg is x so
If SP is Rs. 144, marked price
135 135
100 - = 1.5 [as given in question]
= ´ 144 = Rs. 150 x 1.2x
96
Therefore marked price » 50% above CP æ 0.2 ö 135 ´ 0.2
4. (c); Let retail price = Rs. 100 135 ç ÷ = 1.5 , x = = Rs. 15 per kg
è 1.2x ø 1.2 ´ 1.5
then, Commission = Rs. 36
SP = retail price – concession = 100 – 36 = Rs. 64 Increased price = 15 × 1.2 = Rs. 18 per kg
But profit = 8.8% 9. (b); Total CP of mixture = 26 × 20 + 30 × 36
520 + 1080 = Rs. 1600, SP = 30 × 56 = Rs. 1680
100 ´ SP 100 ´ 64 1000
CP = = = Rs.
100 + gain% 100 + 8.8 17 80
%profit = ´ 100 = 5%
New commission = Rs. 12 then 1600
New SP = 100 – 12 = Rs. 88 10. (c); The SP of TV in chandigarh = Rs. x
The dealer bought it at delhi at = Rs. 0.8x
1000 496
Gain = 88 - = Rs. [Discount of 20%]
17 17
Total CP of TV set (including transportation cost)
496 Rs. 0.8x + 600
100
Gain% = gain × = 17 ´ 100 = 49.6% 2
CP 1000 Given Profit% = 14 %
17 7

162 @BEST300MCQ For More Study Material


Visit: studyiq.com
16. (a); Let CP of whole fruit = Rs. A
100 æ ( x) - ( 0.8x + 600 ) ö
=ç ÷ ´ 100
7 è ( 0.8x + 600) ø He sold
3
th part at 10% profit and remaining
5
1 æ 0.2x - 600 ö 2
=ç ÷ , On solving, x = Rs. 8000 th part at 5% loss
7 è 0.8x + 600 ø 5
11. (c); MP of chair = Rs. 800 Total profit = Rs. 1500
After getting successive discount of 10% and 15%
respectively é3 10 2 5 ù
1500 = ê ´ A ´ - ´A ´
ë5 100 5 100 úû
é 90 85 ù
Þ CP of chair = Rs. ê800 ´ ´ = Rs. 612
ë 100 100 úû On solving above equation we get:
Total CP = A = Rs. 37,500
Total CP (including transportation cost)
17. (b); Case – 1
612 + 28 = Rs. 640
Here, CP = Rs. 100, SP = Rs. 125
800 - 640 Profit of 1st shopkeeper = 25%
Profit% = ´ 100 = 25%
640 Case – 2
12. (c); Let us assume that cost of the book is Rs. 100 Here, CP = Rs. 125, SP = Rs. 100
and Market Price is Rs. 140
If we sell the book at half of MP æ 125 - 100 ö
Loss of 2nd shopkeeper = ç ´ 100%
è 125 ÷ø
140
then selling Price = = Rs. 70 = 20%
2
18. (a); Here, the successive discounts given are 10%
So percent loss = (100 – 70) = 30% loss
and x%
13. (c); Price of 14 shirt = 14 × 45 = Rs. 630
25 pant = 25 × 55 = Rs. 1375 90 ( 100 - x )
2400 ´ ´ = 1836
Total price of 39 items = Rs. 2005 100 100
2005 1836 ´ 100 ´ 100
Pr ice = ´ 1.40 [Overall profit = 40%] 100 - x = = 85
39 2400 ´ 90
= 71.97 = Rs. 72 (Approx)
So discount = 100 – 85 = 15%
14. (a); Here, CP is same in both transactions
19. (b); Three successful discount equivalent to
(CP)1 = (CP)2
é xyz ù
( SP) 1 ( SP) 2 é Where x = 4% Loss ù êx + y + z -
xy + yz + zx
+ ú%
=
100 - x 100 + y ê
ë y = 8% Gain úû êë 100 ( 100) 2 úû
(where x = 10%, y = 12%, z = 15%)
1
(SP)1 of 1 apple = Rs.
36 é 10 ´ 12 + 12 ´ 15 + 15 ´ 10
êë 10 + 12 + 15 - 100
1
36 = ( SP ) 2 , ( SP ) 2 =
108 1 1
Þ ´ = Rs. 10 ´ 12 ´ 15 ù
100 - 4 100 + 8 96 36 32 + ú%
( 100) 2 úû
Hence, in a rupee, the person can sell 32 apples.
15. (a); Here, Aditya paid Rs. 59.40 for article 37 – 4.50 + 0.18 = 32.68%
But before that there were 3 transactions of gain 20. (d); Let the original price be Rs. x
20%, 10% and 12.5% Þ Loss of 19% = Rs. (x – 0.19x)
So, initially Ram would have bought article at = Rs. 0.81x (old price)
Rs. x
Þ Profit of 17% = Rs. (x + 0.17x)
100 + 20 100 + 10 100 + 12.5 = Rs. 1.17x (new price)
Þ x´ ´ ´
100 100 100 (New price) – (Old price) = Rs. 162
= Rs. 59.40 [According to given question]
59.40 ´ 100 ´ 100 ´ 100 162
x= = Rs. 40 1.17x – 0.81x = 162, x = = Rs. 450
120 ´ 110 ´ 112.5 0.36

163 @BEST300MCQ For More Study Material


Visit: studyiq.com
21. (b); Let CP = Rs. 100, Then SP = Rs. 123.5
840
Here discount of 5% is given 27. (b); CP of 1st item = = Rs. 700 [gain of 20%]
1.2
Let Mark Price be x Rs. then
960
123.50 CP of 2nd item = = Rs. 1000 [loss of 4%]
x= ´ 100 = Rs. 130 0.96
95
Total CP = 700 + 1000 = Rs. 1700
So profit on Marked Price = 130 – 100 = 30% SP = 840 + 960 = Rs. 1800
22. (a); Let the original price bought by Aditya = Rs. 100
100 15
Aditya Nutan Manish %profit = ´ 100 = 5 %
1700 17
10% gain 20% gain
100 ¾¾¾¾
® 110 ¾¾¾¾
® 132 28. (a); Given MP = Rs. 600
If Manish bought goods at Rs. 132 Hence on giving successive discounts of 10%
Initial CP = Rs. 100 and 20%,
Now, Manish bought goods at Rs. 1914 90 80
CP = 600 ´ ´ = Rs. 432
é 100 ù 100 100
Initial CP = Rs. ê ´ 1914 ú = Rs. 1450
ë 132 û 108
%profit = ´ 100 = 25%
23. (c); Initial CP = Rs. 9600 432
88 21
SP after selling it at 12% Loss = Rs. 9600 ´ 29. (a); CP of each kg mango = Rs. = Rs. 7
100 3
New CP = Rs. 8448
50
Final SP after selling new CP price at 12% gain SP of each kg mango = Rs. = Rs. 10
5
112 Profit = SP – CP = Rs. 3
Þ Rs. 8448 ´ = Rs. 9461.76
100 Here, Rs. 3 is profit earned for 1 kg
So, Total Loss = Initial CP – Final SP Similarly, Rs. 102 is profit earned for:
= Rs. [9600 – 9461.76] = Rs. 138.24 1
24. (b); Total CP = ´ 102 = 34 kg
3
= [CP of 30 dozen orange] + [CP of stall fee]
Þ 8 × 30 + 500 = Rs. 740 15 ( SP )1 - CP
30. (c); Old Profit% = = ...(i)
Here, Nutan calculated that each glass needs 3 100 CP
oranges and she wants to make 20% profit
20 ( SP )2 - CP
740 New Profit% = = ...(ii)
Per glass price = ´ 3 ´ 1.20 = Rs. 7.40 100 CP
30 ´ 12
[Here, (SP)2 = Rs. 600]
25. (c); Let CP = Rs. 100 = From (ii), we get CP = Rs. 500
then total CP after repair = 100 + 15 = Rs. 115 Divide (i) and (ii):
after getting 20% profit
SP = 115 × 1.20 = 138 Rs. 3 ( SP ) 1 - 500
=
but SP given is Rs. 1104 4 600 - 500
Hence, (SP)1 = Former Selling price = Rs. 575
100
so CP = ´ 1104 = Rs. 800 31. (d); Let us assume payment order be Rs. 100
138 Case – 1: Successive discount of 10%, 10%, 30%
26. (b); Let CP = Rs. 100
90 90 70
After 320% profit SP = Rs. 420 Þ 100 ´ ´ ´ = Rs. 56.7
100 100 100
After increasing cost the,
CP = Rs. 125 [25% cost increase] Case – 2: Successive discount of 40%, 5%, 5%
Profit = 420 – 125 = Rs. 295 60 95 95
Þ 100 ´ ´ ´ = Rs. 54.15
100 100 100
295
´ 100 = 70% (Appx.) For Rs. 100, person can save Rs. (56.7 – 54.15)
420
= Rs. 2.55

164 @BEST300MCQ For More Study Material


Visit: studyiq.com
Hence, for Rs. 10000, he can save 36. (d); Here, CP of 19 article = SP of 15 article
2.55 CP of 1 article 15
= Rs. ´ 10000 = Rs. 255 =
100 SP of 1 article 19
99 19 - 15 2
32. (b); CP of 1st article = = Rs. 90 [Profit of 10%] %gain = ´ 100 = 26 %
1.10 15 3
99 37. (b); Here, we need to determine only ratio of selling
CP of 2nd article = = Rs. 100 [Loss of 1%] price only.
0.99
CP of both article together = 100 + 90 = Rs. 190 SP is directly proportional to profit%
SP of both article together = 99 + 99 = Rs. 198 ( SP ) 1 x + 0.04x 1.04x
= = = = 52 : 53
198 - 190 4 ( SP ) 2 x + 0.06x 1.06x
%profit = ´ 100 = 4 %
190 19 38. (b); Let quantity sold at loss be x kg and let CP per kg
33. (c); Here, (100 + Profit)% of CP be Rs. 1. Total CP = Rs. 24
= Rs. (MP – 10% of MP) Total SP = Rs. [120% of (24 – x) + 95% of x]
(100 + 35)% CP = Rs. (100 – 10)
é6 19x ù é 576 - 5x ù
= Rs. ê ( 24 - x) + ú = Rs. ê
(135%) CP = Rs. 90 Þ CP = Rs.
200 ë 5 20 û ë 20 úû
3
SP of article (at Rs. 30 less than MP) = Rs. 70 576 - 5x
= 110% of 24 , 576 – 5x = 528
20
200 5x = 48 Þ x = Rs. 9.6
70 -
Profit% = 3 ´ 100 = 5% 39. (a); Let Mark Price is Rs. 100
200
Selling Price = Rs. 90
3
90
34. (b); Let CP = Rs. 100 Cost Price is = Rs. 60
1.5
Then SP = Rs. 123.5
[on earning profit of 50%]
123.50 If discount is not given, Percentage Profit
Let Marked Price be x Rs., then x = ´ 100
76
100 - 60
[on discount of 24%] will be = ´ 100 Þ So, Profit = 66.67%
= Rs. 162.50 60
So profit on Marked Price = 162.50 – 100 = 62.50% 40. (a); If we are given 3 successive discounts of 15%,
10% and 5%,
20 CP - 500 So, the new reduced price after applying above
35. (b); Loss = =
100 CP discount on Rs. 10000
CP = Rs. 625 85 90 95
= 10000 ´ ´ ´ = Rs. 7267.50
20 SP - 625 100 100 100
Now Profit = = , SP = Rs. 750
100 625

1. (c); Let us assume the cost price of the phone to be


100
Rs. 100. Then, \ Cost price =´ 180 = Rs. 900
20
sells at
Rs. 100 ¾¾¾¾
20% profit
® 100 ´ 1.2 = Rs. 120 = ( SP ) 1 Hence, the cost price of the book is Rs. 900
If she bought the phone at 20% less, i.e. at Rs. 80 2. (a); Let the CP of each pen be Rs. 100
then, At the profit of 10%, SP of 40 pens
sells at
Rs. 80 ¾¾¾¾
25% profit
® 80 ´ 1.25 = Rs. 100 = ( SP ) 2 = (100 + 10) × 40 = Rs. 4400
At the profit of 20%, SP of 50 pens
So, (SP)1 – (SP)2 = Rs. (120 – 100)
= Rs. 20 when cost price is Rs. 100. = (100 + 20) × 50 = Rs. 6000
But, (SP)1 – (SP)2 = Rs. 180 SP of 90 pens = Rs. (4400 + 6000) = Rs. 10400

165 @BEST300MCQ For More Study Material


Visit: studyiq.com
CP of 90 pens = Rs. (90 × 100) = Rs. 9000 5. (b); Total number of i-phones = 15
At the profit of 15%, SP of 90 pens \ Total number of i-pads = 25 – 15 = 10
= Rs. (90 × 115) = Rs. 10350 Total CP = Rs. 205000
Difference in SP = Rs. (10400 – 10350) = Rs. 50 Since, Kritika sells 80% of both goods at a profit
If the difference is Rs. 50, then CP = Rs. 100 of Rs. 40000, therefore, cost of 80% of the goods
If the difference is Rs. 40, then CP = 0.8 × 205000 = Rs. 164000
Total amount recovered (or SP)
100 ´ 40
= = Rs. 80 = Rs. (164000 + 40000) = Rs. 204000
50
Hence, loss = Rs. (205000 – 204000) = Rs. 1000
Hence, the cost price of each pen is Rs. 80.
Hence, Kritika's overall loss is Rs. 1000
3. (c); Let Rs. p be the cost price of a shirt and Rs. q be
6. (d); Let the cost of sofa set be Rs. 100. Then, the cost of
the cost price of a pant. Then,
centre table is Rs. 40 [40% of sofa set given]
CP of 5 shirts = Rs. 5p
According to Sasha, cost of centre table
CP of 10 pants = Rs. 10q
\ 5p + 10q = 1600 ...(i) 90
= ´ 40 = Rs. 36
100
15 ´ 5p 3p
Profit on the sale of 5 shirts = = Rs. [10% discount on center table]
100 4
75
10q ´ 10 and cost of sofa set = ´ 100 = Rs. 75
Loss on the sale of 10 pants = = Rs. q 100
100
[25% discount on sofa set]
Given,
According to manager, He inter changed
Profit on the shirts – Loss on pants = Rs. 90
discount % so, cost of centre table
3p
Þ - q = 90 75
4 = ´ 40 = Rs. 30
100
\ 3p – 4q = 360 ...(ii)
Multiplying (i) by 3 and (ii) by 5 and then 90
and cost of sofa set = ´ 100 = Rs. 90
subtracting (ii) from (i), we get 100
3000 Extra money = Rs. (90 + 30) – Rs. (36 + 75)
50q = 3000 Þ q = = Rs. 60 = Rs. 120 – Rs. 111 = Rs. 9
50
Puting the value of q in (i) we get 9
% extra = ´ 100 = 8.1%
111
1000
5p = 1000 Þ p = = Rs. 200 Hence, Sasha paid 8.1% extra price.
5
7. (c); Case 1: If Rate is Rs. 3.25
Hence, the cost price of shirt is Rs. 200 each and
Total sales revenue = 2000 × 3.25 × 0.75
the cost price of pant is Rs. 60 each.
= Rs. 4875
4. (a); Total CP of articles = 750 × 0.6 = Rs. 450
[Here 1000 strips are given free of cost]
[CP of 1 article = Rs. 0.6]
Profit = Total sales revenue – Rs. 4800
By selling 600 articles, Sarika should make a 40%
profit on the outlay. This means that the selling = Rs. 4875 – Rs. 4800 = Rs. 75
price for 600 articles should be Case 2: If rate is Rs. 4.25
1.4 × 450 = Rs. 630 Total sales revenue = 3000 × 4.25 × 0.75
Thus, selling price per article = Rs. 9562.5 [Total 3000 strips]
Profit = Total sales revenue – Rs. 4800
630 63 = Rs. 9562.5 – Rs. 4800 = Rs. 4762.5
= = = Rs. 1.05 [SP of 1 article]
600 60 Hence, the ratio of profit is
Since, Sarika sells only 630 articles at this price,
her total recovery = 1.05 × 630 = Rs. 661.5 New profit 4762.5
= = 63.5
Hence, actual profit percent Old profit 75

661.5 - 450 8. (d); Cost price = Rs. 240000 [Total 3000 copies]
= ´ 100 = 47% Published price = Rs. 325 [Published price]
450
Thus, Sarika earns 47% profit on her total 75
Selling price = ´ 325 = Rs. 243.75
investment. 100

166 @BEST300MCQ For More Study Material


Visit: studyiq.com
2500 é 100 ù = Rs. 4 p
No. of free copies = 500 + = 500 + 100 = 600 CP of Rohit = Rs. p ê
25 ë 100 + 25 úû 5
So, total selling price = 2400 × 243.75 = Rs. 585000 (% Profit on CP)
585000 – 240000 3 p
Hence, percentage gain = ´ 100 So, Ajit profit = SP – CP = p - p = Rs.
240000 4 4
345000 4 p
= ´ 100 = 143.75% Rohit profit = SP – CP = p - p = Rs.
240000 5 5
Hence, the overall gain is 143.75%.
9. (b); There were 5 i-phones (2 + 3) and 20 i-pads. p p
Difference of profit = - = Rs. 100 (given)
Surbhi sells 2 i-phones for a profit of Rs. 2000 4 5
each. Hence, profit from i-phone sales = Rs. 4000 p
Then, profit from i-pads sales = Rs. 45000 = = 100 Þ SP = p = Rs. 2000
20
45000 12. (d); Let CP of pen be Rs. x
Thus, profit per i-pad = = Rs. 3000
15 and SP be Rs. y
(Since, 15 i-pads were sold in all.) Initially at loss of 20%,
Hence, CP of i-pad = Rs. 15000 20 x-y 1
CP of i-phone = Rs. 7500 Þ = Þ x = x-y
100 x 5
Total cost = Rs. 15000 × 20 + Rs. 7500 × 5
= Rs. 300000 + Rs. 37500 = Rs. 337500 4
Þ y= x ...(i)
[If only 75% of i-pads and 2 i-phones were sold] 5
Total revenue = Rs. 18000 × 15 + Rs. 9500 × 2 Now if y would change to Rs. (y + 12), then profit
= Rs. 270000 + Rs. 19000 = Rs. 289000 becomes 30%
Since, total revenue is less than total cost, there is
a loss. 4
x + 12 - x
30 y + 12 - x 3
Hence, loss = Rs. 337500 – Rs. 289000 = Rs. 48500 Þ = Þ = 5
Thus, Surbhi has an overall loss of Rs. 48500. 100 x 10 x
10. (b); In the given question, let the total profit% be p% Þ x = Rs. 24 ...(ii)
1 1 96
Þ Total Pr ofit p% = ´ 5 + ´ 10 + y = Rs. or Rs. 19.2 ...(iii)
5 4 5
%profit now if y becomes Rs. (y + 4.8)
1 é æ1 1 1ö ù
´ 12 + ê1 - ç + + ÷ ´ 16
2 ë è5 4 2 ø úû Þ %profit =
SP - CP
´ 100
CP
5 æ 1 ö 103
Þ 1+ + 6 + ç ´ 16÷ Þ % Rs. ( y + 4.8) - 24
2 è 20 ø 10 = ´ 100
24
Now if he would have sold whole wheat at 11%,
he would had made Rs. 72.80 more Rs. ( 19.2 + 4.8 ) - 24
= ´ 100 = 0%
103 7 24
11% - % = Rs. 72.80 , % = Rs. 72.80 13. (c); CP of i-phone to dealer
10 10
(on inclusion of 10% custom duty)
72.80 ´ 10 = Rs. (25000 + 10% of 25000) = Rs. 27500
1% = Rs. [Unitary method]
7 The SP of i-phone at 20% profit
100% = Rs. 10,400 [Total CP of 4000 kg wheat]
é 120 ù
= Rs. ê 27500 ´ = Rs. 33000
CP of crop/kg = Rs.
10400
= Rs. 2.60 ë 100 úû
4000
But Here, Rs. 33000 is price after 25% discount
11. (d); Suppose selling price of both of them be Rs. P. on MP
é 100 - 25 ù = Rs. 3 p Hence, MP × (1 – 0.25) = 33000
CP of Ajit = Rs. p ê ú
ë 100 û 4 33000
MP = = Rs. 44000
(% Profit on SP) 0.75

167 @BEST300MCQ For More Study Material


Visit: studyiq.com
14. (a); Let CP of cab driver be price of petrol
5a
= Rs. 30 per liter 5 7
His, SP would be to carry 3 passengers % profit = 20 ´ 100 = ´ 100 = 29 %
17a 17 17
Let cost of 1 passenger be Rs. x
20
Initially he made profit of 20%
17. (c); Here, the labelled price of chair is
20 SP - CP 20 3x - 30
Þ p% = = = = SP ´ 100 2139 ´ 100
100 CP 100 30 = = = Rs. 2300
x = Rs. 12 ( 100 - D ) % 93
Now, CP of petrol = Rs. 24 per litre Let the CP of chair be Rs. p
SP = 4 (cost of 1 passenger) = Rs. 48 Now according to question:
48 - 24 15 2300 - p
= Pr ofit% = ´ 100 = 100% = Þ 15p = 230000 – 100p
24 100 p
15. (c); Let the labelled price of article be Rs y. p = Rs. 2000
æ 100 - 12.5 ö = Rs. 7 y 18. (c); Here, the labelled price of decks is defined as:
CP of article = y ç ÷
è 100 ø 8 SP ´ 100 166 ´ 100
Þ = = Rs. 200
æ 100 + 17.5 ö 117.5
( 100 - D) % 100 - 17
SP of article = y çè ÷ = Rs. y or Let CP of deck be p
100 ø 100
According to the question:
47
= Rs. y 25 200 - p
40 = Þ P = Rs. 160
100 p
é 47 7 ù 12 19. (c); The original company price
Pr ofit = Rs. ê y - y ú = Rs. y
ë 40 8 û 40
é 25000 ´ 100 ù é 25000 ´ 100 ù
= Rs. ê ú = Rs. ê úû
12
y ë 100 - 15 û ë 85
12 2
%profit = 40 ´ 100 = ´ 100 = 34 % = Rs. 29,411.76
7 35 7
y Let the total SP be Rs. p
8
Now according to question:
16. (b); Let the labelled price of article be Rs. a
p - 29411.76
= ´ 100 = 8 Þ p = Rs. 31,764.70
æ 100 - 15 ö 29411.76
CP of article = Rs. a ç
è 100 ÷ø
é SP ´ 100 ù
20. (a); The original company price = Rs. ê ú
é 85 ù ë ( 100 - D ) % û
17
= Rs. ê ú a or Rs. a
ë 100 û 20
é 860 ´ 100 ù
æ 100 + 10 ö = Rs. ê ú = Rs. 1000
SP of article = Rs. a ç
è 100 ÷ø ë ( 100 - 14) % û
Let the total SP be Rs. q
é 110 ù 11 Now according to equation:
= Rs.ê a or Rs. a
ë 100 úû 10
SP - CP
= ´ 100 = ( profit ) %
é 11 17 ù 5a CP
Pr ofit = Rs. ê a - a ú = Rs.
ë 10 20 û 20 q - 1000
Þ ´ 100 = 6 Þ q = Rs. 1060
1000

168 @BEST300MCQ For More Study Material


Visit: studyiq.com
7. (c); Let CP = 100
8
1. (b); CP = = 0.8 Actual CP = 125
10
SP = 100 + 25 = 125
SP = 1.25
\ Neither loss nor gain
1.25 - 0.8 225 1 8. (d); CP : SP
gain% = ´ 100 = = 56 %
0.8 4 4 100 : 114
2. (a); Let CP = x Rs. 100 : 117
\ 515 – x = x – 475 \ 3 units = 15
2x = 990 Þ x = 495 100 units = 500 Rs.
140 9. (b); CP : SP
\ Required SP = ´ 495 = 693 Rs.
100 : 92
100
100 : 107
10
\ 15 units = 120
3. (a); + 1 unit – 10%
9 120
100 units = ´ 100 = 800 Rs.
Re duced price 9 15
= 10. (a); Let CP 100x Þ (100x – 56) New CP
Original price 10
SP 115x Þ (115x – 56) New SP
225
\ Required Reduced price = = Rs. 0.9 122
25 ´ 10 ( 100x - 56 ) = 115x - 56
100
225 122x – 1.22 × 56 = 115x – 56
Required original price = = Rs. 1
25 ´ 9 x = 1.76
4 \ Required CP = 1.76 × 100 = 176 Rs.
4. (c); – 1 unit + 25% 11. (b); Let CP = 100x Þ 100x – 25 New CP
5 SP = 130x Þ 130x – 19 New SP
increased price 5 150
= ( 100x - 25 ) = 130x - 19
original price 4 100

75 150
\ Required original price = = 7.5 Rs. 150x - = 130x - 19
2´5 4

15 5 14 7 150
5. (b); CP = = Þ SP = = 20x = - 19 Þ x = 0.925
12 4 10 5 4
\ Required CP = 92.5 Rs.
7 5 12. (b); Let CP = 100x Þ 100x + 80 New CP
-
\ Required profit% = 5 4 ´ 100 SP = 145x = 145x + 70 New SP
5
4 130
\ ( 100x + 80 ) = 145x + 70
100
3
= ´ 100 = 12% 130 ´ 80
25 130x + = 145x + 70
6. (c); Let CP = 100 100
actual CP = 70 15x = 13 × 8 – 7
SP = 100 – 44 = 56 x = 2.2666

70 - 56 14 2
\ loss% = ´ 100 = ´ 100 = 20% \ Required CP = 226 Rs.
70 70 3

169 @BEST300MCQ For More Study Material


Visit: studyiq.com
13. (d); Let CP is Rs. x 17. (c); Let Required% = x
According to question
+ 24% X%
1 25 1 40 7 30
´ ´x + ´x´ + ´ x´ = 150
5 100 10 100 10 100 21%
5x 4x 21x
+ + = 150
100 100 100 (21 – x)% 3%
150 ´ 100
= = 500 Given, (21 – x) : 3 = 3 : 5
30
\ x = 19.2
14. (a); Let CP of horse sold at gain = A
1
CP of horse sold at loss = B x = 19 %
5
A 1 18. (a); Let oranges bought = 240
160% of A = 80% of B Þ =
B 2
120 120
1 \ Total CP = + = 10 + 15 = 25 Rs.
12 8
CP of A = ´ 960 = 320
3
240
SP = = 16 Rs.
2 15
CP of B = ´ 960 = 640
3
25 - 16
15. (a); Let value of consignment = 800 \ loss% = ´ 100 = 36%
25
5 19. (a); Let CP of two tables be A and B
of consignment = 500
8
A 21
110% A = 105% of B Þ =
16 B 22
loss = ´ 500 = 80 Rs.
100
21
rest value = 800 – 500 = 300 Rs. CP of A = ´ 860 = 420
43
8
profit = ´ 300 = 24 Rs. 22
´ 860 = 440
100 CP of B =
43
\ Overall loss = 80 – 24 = 56 Rs.
\ 56 ® 140 120
20. (c); Total profit = ´ 100 = 12%
1000
140
1® Pen Pencils
56 15% 10%
140
800 ® ´ 800 = 2000 Rs.
56 12%
16. (a); Let Required% = x
2 3
– 15% + x%
2
Total Cost price of Pen = ´ 1000 = 400 Rs.
5
+ 10%
3
Total Cost price of Pencils = ´ 1000 = 600 Rs.
5
(x – 10)% (10 + 15) = 25%
400
Given (x – 10) : 25 :: 2 : 1 \ Cost of a Pen = = 80 Rs.
5
\ x – 10 = 50
x = 60% 600
Cost of a Pencil = = 20 Rs.
30
170 @BEST300MCQ For More Study Material
Visit: studyiq.com
21. (c); Let no. of oranges bought = 240 132 ® 640
120 120 640
\ Total CP = + = 10 + 15 = 25 Rs. 110 ® ´ 110 = 533.33
12 80 132
640
240 120 ® ´ 120 = 581.8181
Total SP = = 24 Rs. 132
10
Total CP = 1115.148
loss = 25 – 24 = 1 Rs.
\1®8 164.85
\ Required profit% = ´ 100 = 14.78%
240 ® 8 × 240 = 1920 1115.14
26. (a); Miheer Satya
22. (d); 1st 2nd
CP 90 20 1890 2000
– 23% 27%
SP 100´21 21´100 2100 2100

0% difference of their profit = 110


So, Selling price = 2100
27. (b); First Second
27 : 23
CP 100 : 80
27 SP 70 : 100
\ Cost price of 1st horse = ´ 1550 = 837 Rs.
50 Given total SP = 11900
170 ® 11900
23
Cost price of 2nd horse = ´ 1550 = 713 Rs. 1 ® 11900
50
23. (c); Let original price = 100 170
180 ® 11900
110 ´ 180
SP = ´ 95 = 104.5 170
100
Total CP = 12600
\ Required % profit = 4.5%
Loss = 12600 – 11900 = 700 Rs.
Patato 5 1
24. (a); = = 28. (b); + 35% – 30%
Tomato 20 4

Tomato 30 5 O
= =
Onion 12 2

Onion 18 6 30 : 35
= =
Cabbage 15 5 Ratio = 6 : 7

50 100
1 kg of Cabbage = = 5 Rs./kg. CP of first table = ´ 405 = 300 Rs.
135
10
\ Potato : Tomato : Onion : Cabbage 300
\ CP of second table = ´ 7 = 350 Rs.
15 : 60 : 24 : 20 6
\ Potato : Cabbage = 3 : 4 29. (d); Let CP = 100, SP = x

5 x
100 -
\ Required Price = ´ 3 ´ 24 = 90 Rs. 2 ´ 100
4 25 =
100
25. (a); 1st : 2nd x x
Cost price 10 : 10 110 : 120 25 = 100 - , = 75
2 2
SP 12 ´11 : 11´12 132 : 132 x = 150 Rs.
Given, SP = 640 Rs. 50
Required % profit = ´ 100 = 50%
total SP = 1280 Rs. 100

171 @BEST300MCQ For More Study Material


Visit: studyiq.com
30. (c); CP LP SP 130
35. (d); Required SP = ( 950 + 300 ) = 1625 Rs.
70 100 112 100
36. (d); Total CP when discount not offered
42
Required% = ´ 100 = 60%
100
70 = ´ 11250 + 150 + 800 = 13450 Rs.
90
31. (b); CP MP SP
200 250 210 115
Required SP = ´ 13450 = 15467.5 Rs.
100
50
Required% = ´ 100 = 25% 100
200 37. (d); CP = ´ 10980 = 9000 Rs.
32. (d); Total CP = 25 × 6 + 35 × 7 122
= 150 + 245 = 395 Rs. 95
SP = 60 × 6.75 = 405 Rs. 38. (a); Required amount = ´ ( 25000 ) ´ 12
100
gain = 10 Rs. = 285000 Rs.
100 100 39. (c); 100 ¾¾¾ - 5% + 10%
33. (c); Required price = ´ ´ 10500 = 6250 Rs. ® 95 ¾¾¾
® 104.5
140 120
Required% = 4.5%
100
34. (b); CP = ´ 4921 = 3700 Rs. 100
133 40. (a); original price of perfume = ´ 3675.4 = 4324
85

172 @BEST300MCQ For More Study Material


Visit: studyiq.com
Chapter Simple Interest and
8
Simple Interest
Compound Interest
If Principal = Rs. 'P', Time = 'T' years, Rate = 'R%' per annum,
P ´R ´ T
Simple Interest (SI) =
100
Amount = Principal + Simple Interest
P ´R ´ T  RT 
A= P+  P 1  
100  100 
Example: Find the simple interest on Rs. 200 for 5 years at 6% per annum?
Sol. Here, P = Rs. 200, T = 5 years, R = 6%
P ´ R ´ T 200´ 5´ 6
\ SI = = = Rs. 60
100 100

æ Rö
(i) If rate of interest is half-yearly, Rate = ç ÷ % and Time = 2T
è 2ø

æ Rö
(ii) If rate of interest is quarterly, Rate = çè ÷ø % and Time = 4T
4

æRö
(iii) If rate of interest is monthly, Rate = çè ÷ø % and Time = 12T
12
Installments
When the borrower paid total money in some equal parts, then we can say that he is paying in installments.
For simple interest,
  x R  1   x R  2  
A  x   x     x    .........
  100   100  
where A = Total amount paid
x = value of each installment
Example: A scooty is sold by an automobile agency for Rs. 19200 cash or for Rs. 4800 cash down payment together
with five equal monthly instalments. If the rate of interest charged by the company is 12% per annum, then
find each instalment?
Sol. Balance of the price to be paid through instalments
P = 19200 – 4800 = 14400
Now, according to the formula,
  x R 1   x R  2   x  R  4 
A  x   x     x    ...   x  
  100   100   100  

P  n R
where, A  P
100

 14400  12  5    12x   12x  2   12x  4 


Þ  14400    x   x     x    ...   x  

 100  12    12  100   12  100   12  100  

x 151200
Þ 15120  5x  Þ x Þ x = Rs. 2964.70
10 51

173 @BEST300MCQ For More Study Material


Visit: studyiq.com
Compound Interest
If Principal = Rs. P, Time = n years, Rate = r% per annum and interest compounded annually
n
é r ù
(i) When interest compounded annually Þ Amount = P ê 1 +
ë 100 úû

( )
2n
é r ù
(ii) When interest compounded half yearly Þ Amount = P ê 1 + 2 ú
ê 100 ú
ë û

( )
4n
é r ù
(iii) When interest compounded quarterly Þ Amount = P ê 1 + 4 ú
ê 100 ú
ë û

( )
12 n
é r ù
ê
(iv) When interest compounded monthly Þ Amount = P 1 + 12 ú
ê 100 ú
ë û

(v) When time is in fraction of a year, say 3


4 é
years Þ Amount = P ê1 +
3 é
r ù ê
1 +
4 r
5 ( ) ùú
ú
5 ë 100 û ê 100 ú
ëê ûú
(vi) When rate of interest is r1% durring first year, r2% durring 2nd year, r3% durring 3rd year.
é r ùé r ùé r ù
Amount = P ê1 + 1 ú ê1 + 2 ú ê1 + 3 ú
ë 100 ûë 100 ûë 100 û
n n  n 
 r   r  r 
Concept 2: A  P  1   , CI  A  P  P 1    P , CI  P 1    1
 100   100   100  
 
Concept 3: A sum of money, placed at compound interest, becomes n times in t years and m times in x years
1 1
then, n t = m x
Example: A sum of money at compound interest amounts to thrice itself in 3 years, in how money years will it be 9
times it self?
1 2
= 9 x , 3 3 = 3 x , 3 = x , x = 6 years
11 1 2
Sol. 3 3

Concept 4: Relationship between CI and SI for two years


CI 200 + r
=
SI 200
Example: The SI on a certain sum of money for 2 years at 10% per anuum is Rs. 400, find CI at the same rate and for the
same time.
210
Sol. CI =
´ 400 = Rs. 420
200
Concept 5: When difference between the CI and SI on a certain sum of money for 2 years at r%, is given by,
2
 r 
Difference = P  
 100 
Example: The difference between the CI and SI on a certain sum of money at 5% per annum for 2 years is Rs. 1.50, Find
the sum

P ( 5)
2

Sol. 1.5 = , P = Rs. 600


100 2

174 @BEST300MCQ For More Study Material


Visit: studyiq.com
Concept 6: The difference between CI and SI on a certain sum for 3 years is given by,
Pr 2 300  r
Difference 
100 3
Example: If the difference between CI and SI on a certain sum of money for 3 years at 5% per annum is Rs. 122 find the
sum.
2
P 5 300  5
Sol. 122  , P = Rs. 16000
100 3
Concept 7: If a sum 'A' becomes 'B' in t1 years at compound rate of interest, then after t2 years the sum becomes
t2

B t1

t2
1
A t1

Example: Rs. 4800 becomes Rs. 6000 in 4 years at a certain rate of CI. What will be the sum after 12 years.
12
3
6000 6000
Required Amount  
4

12
 2
 Rs. 9375
4800 4
1
4800
Concept 8: If a sum of money Rs. x is divided among 'n' parts in such a manner that when placed at CI, amount
obtained in each case remains equal while the rate of interest on each part is r1, r2, r3 ....... rn respectively and
time period for each part is t1, t2, t3 ....... tn respectively. Then the divided parts of sum will be in the ratio of:

1 1 1 1
t1
: t2
: t3
: .......... : t
 r   r   r   rn  n
1  1  1  2 
 1  3   1
 100 
 
 100   100   100   
Example: A sum of Rs. 3903 is divided between A and B, so that A's share at the end of the 7 years be equal to B's share
at the end of 9 years, CI being 4% find A's share.

1 1 1 625
A 's share : B's share  7
: 9
 1: 2
1:  676 : 625
 4   4   4  676
1  
 1 
 
 1 
 

 100   100   100 

676
A 'share = ´ 3903 = Rs. 2028
( 676 + 625)

1. A sum of money becomes four times in 20 years at SI. 2. A sum becomes two times in 5 years at a certain rate
Find the rate of interest. of interest. Find the time in which the same amount
Sol. Given, T = 20 years, will be 8 times at the same rate of interest.
Sol. Let sum = P
Let sum = P
Then, for 5 years,
Then, the sum after 20 years = 4P SI = 2P – P = P
\ SI = 4P – P = 3P
P´R´T
SI =
PRT P ´ R ´ 20 100
Now, 3P = =
100 100
P ´ R ´ 5 PR
\ P= =
20R R 100 20
Þ 3= =
100 5 Þ R = 20%
Again, for another time (T),
\ R = 15%
SI = 8P – P = 7P

175 @BEST300MCQ For More Study Material


Visit: studyiq.com
6. Raviraj invested an amount of Rs. 10000 at
´ ´ 20TP TP
\ 7P = = = compound interest rate of 10% per annum for a period
100 100 5 of three years. How much amount will Raviraj get
\ T = 7 × 5 = 35 years after 3 years?
1 n
3. Ravi lent out a certain sum. He lent part of his sum é r ù
3 Sol. Amount = P ê 1 +
ë 100 úû
1
at 7% SI, part at 8% SI and remaining part at 10% where P ® Principal, r ® Rate, n ® time
4
3
SI. If Rs. 510 is his annual total annual interest, then æ 10 ö
Amount = 10000 ç 1 + ÷ = Rs. 13310
find the money lent out. è 100 ø
Sol. Let entire sum = P
7. Seema invested an amount of Rs. 16000 for two years
Now, according to the question,
at compound interest and received an amount of Rs.
1 1 é æ 1 1ö ù 17640 on maturity. What is the rate of interest?
P ´ 7% + P ´ 8% + ê 1 - ç + ÷ ú ´ P ´ 10% = 510
3 4 ë è 3 4ø û A æ r ö
n

Sol. = ç1+ ÷
1 1 5 P è 100 ø
P´7 P´8 P ´ 10
Þ 3 + 4 + 12 = 510 2
100 100 100 17640 æ r ö 1764
= ç1+ ÷ =
16000 è 100 ø 1600
7P 25P
Þ + 2P + = 510 ´ 100 2 2
3 6 æ 42 ö æ r ö
çè ÷ø = çè 1 + ÷ , r = 5%
510 ´ 100 ´ 6 40 100 ø
\ P= = Rs. 6000
8. Rs. 7500 is borrowed at CI at the rate of 4% per
51
4. A sum of Rs. 7700 is lent out in two parts in such a annum. What will be the amount to be paid after 1
way that the interest on one part at 20% for 5 years is year, if interest is compounded half-yearly?
equal to that on another part at 9% for 6 years. Find 2t
the two sums. é r ù 2 ´1
ê ú é 4 ù
Sol. Let the first sum be Rs. x. Then, second sum Sol. A = P ê1 + 2
ú = 7500 ê1 + 2 ú = Rs. 7803
= Rs. (7700 – x) ê 100 ú ê 100 ú
ë û
Now, according to the question, ë û

x ´ 20 ´ 5 ( 7700 – x ) ´ 9 ´ 6 9. Rs. 7500 is borrowed at CI at the rate of 4% per


= annum. What will be the amount to be paid after 6
100 100
Þ 50x = (7700 – x) × 27 months, if interest is compounded quarterly?
Þ 50x = 7700 × 27 – 27x 4´
6
4t
Þ 77x = 7700 × 27 é r ù é 4 ù 12

Þ x = Rs. 2700 Sol. A = P ê1 + 4 ú = 7500 ê 1 + 4 ú = Rs. 7650.75


ê 100 ú ê 100 ú
\ The second part = (7700 – x) = 7700 – 2700 ë û ë û
= Rs. 5000 10. Find the compound interest on Rs. 2000 at 5% per
5. What annual payment will discharge a debt of Rs.
848 in 8 years at 8% per annum? 1
annum, compounded yearly for 2 years?
Sol. Here, P = Rs. 848, T = 8 years, R = 8% 2
According to the formula,
r ù é
2 r ù
100P é ê1 + 2 ú
Annual payment = Sol. A = P ê1 +
RT ( T - 1) ë 100 úû ê 100
100T + ë û
2
2
é 5 ù é 5 ù
100 ´ 848 = 2000 ê 1 + 1+ = 2260.125
= ë 100 úû êë 200 úû
8 ´ 8 ( 8 - 1)
100 ´ 8 + CI = 2260.125 – 2000 = Rs. 260.125
2
11. An amount of money grows upto Rs. 4840 in 2 years
848 ´ 100 84800 and upto Rs. 5324 in 3 years on compound interest.
= = = Rs. 82.8125
800 + 32 ´ 7 1024 Find the rate per cent.

176 @BEST300MCQ For More Study Material


Visit: studyiq.com
Sol. P + CI of 3 years = Rs. 5324 14. What sum of money at compound interest will
P + CI of 2 years = Rs. 4840 amount to Rs. 2249.52 in 3 years, if the rate of interest
CI for 3rd year = 5324 – 4840 = Rs. 484 is 3% for the first year, 4% for the second year and 5%
for the third year?
484 ´ 100
Re quired r% = = 10%
4840 æ r öæ r öæ r ö
12. Find the ratio of CI to SI on a certain sum at 10% per Sol. A = P ç 1 + 1 ÷ ç 1 + 2 ÷ ç 1 + 3 ÷
è 100 ø è 100 ø è 100 ø
annum for 2 years?
CI 200 + r 210 æ 3 öæ 4 öæ 5 ö
Sol. = = = 21 : 20 2249.52 = P ç 1 + ÷ ç1+ ÷ ç1+ ÷
SI 200 200 è 100 ø è 100 ø è 100 ø
13. Rs. 1600 becomes Rs. 2000 in 2 years at a certain rate
P = Rs. 2000
of compound interest. What will be the sum after 4
years? 15. Find the difference between CI and SI on Rs. 8000 for
Sol. If a sum 'A' becomes 'B' in t1 years at CI, then after t2 3 years at 2.5% pa.
t2
Pr 2 ( 300 + r )
years, the sum becomes
( B) t 1
Sol. Difference = [only possible for 3 years]
t2 100 3
(A) t1
-1

8000 ( 2.5) ( 300 + 2.5)


2

4 = = Rs. 15.125
Required Amount =
( 2000)
=
(
2 2000)
2

= Rs. 2500
100 3

( 1600 ) 2 - 1 ( 1600)
4 1

Ques ti ons 7. What would be the C.I. obtained on an amount of Rs.


4800 at the rate of 5 p.c.p.a for 3 years?
1. What is S.I. of Rs. 800 on 5% per annum for 3 years? (a) Rs. 448.7 (b) Rs. 817.8
(a) Rs. 120 (b) Rs. 140 (c) Rs. 623.5 (d) Rs. 756.5
(c) Rs. 210 (d) Rs. 230 8. What would be the C.I. obtained on an amount of
2. How many years will it take for amount of Rs. 600 to 12500 at the rate of 12 p.c.p.a. after 2 years?
yield Rs. 120 as interest at 10% per annum of S.I.? (a) Rs. 3180 (b) Rs. 3360
(a) 1 year (b) 3 year (c) Rs. 3540 (d) Rs. 3720
(c) 2 year (d) 4 year
9. The difference between simple and compound interest
3. A sum of Rs. 15000 amount gave Rs. 4500 as interest on sum of 10000 is 64 for 2 years. Find the rate of
in 5 year. What is rate of interest? interest.
(a) 3% (b) 4%
(a) 8 (b) 64
(c) 5% (d) 6%
4. A sum of money becomes Rs. 1100 in 2 years and Rs. (c) 4 (d) 2
1400 in 6 years at S.I. Find the rate percent. 10. The difference between simple and compound interest
compounded annually on a certain sum of money for
17 7 2 years at 4% per annum is Rs. 1. The sum (in Rs.) is:
(a) 7 (b) 1
19 13 (a) 625 (b) 630
4 2 (c) 640 (d) 650
(c) 2 (d) 5
3 3 11. Find the compound interest on Rs. 12500 at 8% per
5. What would be the C.I. on Rs. 17500 at the rate of 12 annum for 9 months compounded quarterly.
p.c.p.a. after 2 years? (a) Rs. 1020 (b) Rs. 1428
(a) Rs. 4442 (b) Rs. 4452 (c) Rs. 765 (d) Rs. 550
(c) Rs. 4462 (d) Rs. 4482 12. Find the compound Interest on Rs. 32000 at 20% per
6. What would be the C.I. obtained on an amount of Rs. annum for 1 year, compounded half yearly.
12000 at the rate of 9 p.c.p.a for 3 years?
(a) Rs. 6320 (b) Rs. 6720
(a) Rs. 3840 (b) Rs. 3740.75
(c) Rs. 3540 (d) Rs. 3640 (c) Rs. 6400 (d) Rs. 6500

177 @BEST300MCQ For More Study Material


Visit: studyiq.com
13. The difference between C.I. & S.I. on Rs. 700 in 2 years (a) 130% (b) 140%
at 5% per annum is: (c) 125% (d) 150%
(a) Rs. 10 (b) Rs. 5 25. Amit invested an amount of Rs. 25000 in fixed deposit
(c) Rs. 1.75 (d) Rs. 2.5 @ C.I. 8% per annum for 2 years. What amount Amit
14. The difference between the compound and simple will get?
1 (a) Rs. 28240 (b) Rs. 28540
interest on a sum of money for 2 years at 6 % per
4 (c) Rs. 29240 (d) Rs. 29160
annum is Rs. 10. The sum is 26. Find the ratio of S.I. & C.I. on a certain sum of money
(a) Rs. 2000 (b) Rs. 2200 at 5% per annum for 2 years
(c) Rs. 2560 (d) Rs. 2600 (a) 50 : 51 (b) 40 : 41
15. If a sum of money doubles itself in 8 years at S.I. then (c) 30 : 31 (d) 45 : 46
the rate of interest per annum is: 27. A sum of money invested at C.I. accured to Rs. 800 in
(a) 10.5 (b) 12.5 3 years and to Rs. 840 in 4 years. The rate of interest
(c) 11.5 (d) 13.5 per annum is.
16. What sum lent at 10% per year on S.I. will amount to
Rs. 450 in 2 years? 1
(a) 2 (b) 4%
(a) Rs. 350 (b) Rs. 375 2
(c) Rs. 240 (d) Rs. 280 2
17. Nutan invest 22400 on S.I. at rate 12 p.c.p.a. How (c) 5% (d) 6 %
3
much amount she will get after seven year?
(a) 41,116 (b) 41,216 28. What is difference between C.I. and S.I. of Rs. 12000
(c) 42,116 (d) 42, 216 on 5% per annum for 2 years?
18. What time taken by sum of Rs. 7000 to became 10500 (a) 35 (b) 25
at the rate of 5% per annum? (c) 30 (d) 40
(a) 8 years (b) 10 years 29. Find the difference between C.I. & S.I. on Rs. 15000 at
(c) 5 years (d) 15 years 3% per annum for 3 years?
19. A sum fetched a total simple interest of Rs. 4016.25 at (a) 40.91 (b) 73.91
the rate of 9% p.a. in 5 years. What is the sum? (c) 39.91 (d) 30
(a) Rs. 4462.50 (b) Rs. 8032.50 30. Find the difference between C.I. & S.I. on Rs. 13000 at
(c) Rs. 8900 (d) Rs. 8925 4% per annum for 3 years?
20. If the difference between the C.I. compounded half (a) Rs. 63.23 (b) Rs. 73.25
yearly and simple interest on a sum at 10% per annum
(c) Rs. 68.25 (d) Rs. 70.29
for one year is Rs. 25, the sum is:
(a) Rs. 9000 (b) Rs. 9500 31. A certain sum of money gives Rs. 500 at 5% S.I. for 1
year. What is the C.I. at the same rate at the end of 2
(c) Rs. 10,000 (d) Rs. 10,500
years?
21. The difference between compound and simple interest
at a certain rate on Rs. 2000 at the end of two years is (a) Rs. 1050 (b) Rs. 1015
Rs. 12.8. The rate of interest per annum is (c) Rs. 1035 (d) Rs. 1025
(a) 6.8% (b) 8% 32. The difference between the S.I. and C.I. obtained on
(c) 12.8% (d) 16.8% principal amount at 5 p.c.p.a. after 2 years is Rs. 35.
22. If a sum of money at simple interest doubles itself in 6 What is principal amount.
years, it will become four times in: (a) Rs. 15000 (b) Rs. 10000
(a) 14 years (b) 12 years (c) Rs. 14000 (d) Rs. 13000
(c) 18 years (d) 16 years 33. The C.I. on Rs. 10000 at 20% per annum at the end of
23. A sum of money doubles in 3 years at compound 1 year 6 month, if the interest is calculated half yearly
interest, compounded annually. It will become 4 times will be
of itself in: (a) 4340 (b) 5320
(a) 12 years (b) 9 years (c) 3310 (d) 4590
(c) 8 years (d) 6 years
34. The S.I. is 7200 on 12 p.c.p.a. for 6 years on a sum.
1 What is C.I. on 5 p.c.p.a for 2 years?
24. If the amount is 6 times the sum after 2 years at
4 (a) Rs. 1020 (b) Rs. 1055
compound interest, the rate of interest per annum is: (c) Rs. 1050 (d) Rs. 1025

178 @BEST300MCQ For More Study Material


Visit: studyiq.com
35. If the difference between S.I. and C.I. is Rs. 25 on 10000 38. What will be CI on an amount of Rs. 10000 at the rate
for 2 years then what is rate of interest? of 20 p.c.p.a. in two years if the interest rate is
(a) 6 (b) 4 compounded half yearly?
(c) 5 (d) 3 (a) Rs. 4525 (b) Rs. 4232
36. If the ratio of CI & SI interest for the same principal (c) Rs. 4641 (d) Rs. 4461
and rate of interest for 2 years is 1.2, find the rate of 39. Find the difference between SI and CI at 12% for 2
interest. year on Rs. 10000.
(a) 300% (b) 15% (a) Rs. 169 (b) Rs. 12
(c) 40% (d) 3% (c) Rs. 144 (d) Rs. 24
37. What is CI on Rs. 7500 for 4 years if the rate of interest
40. Find the difference between SI and CI at 14% for 2
is 10% p.a. for the first 2 year and 20% p.a. for the next
years on Rs. 10000.
2 year?
(a) Rs. 144 (b) Rs. 169
(a) Rs. 5273 (b) Rs. 5568
(c) Rs. 5724 (d) Rs. 5325 (c) Rs. 124 (d) Rs. 196

1. Prabhat took a certain amount as a loan from bank at (a) 3.6% (b) 4.5%
the rate of 8% Simple interest per annum and gave the (c) 3.46% (d) 6%
same amount to Ashish as a loan at the rate of 12%
7. Aditya invest Rs. 50,000 in a fixed deposit at 10% C.I.
p.a. on S.I. If at the end of 12 yrs, he made a profit of
for 2 years. At the end of 2 years she put the money in
Rs. 320 in the deal, What was the original amount?
another deposit at 12% S.I. for 3 years. What was the
(a) Rs. 500 (b) Rs. 600 final value of the initial Investment?
(c) Rs. 666.67 (d) Rs. 750.27
(a) Rs. 80000 (b) Rs. 81280
2. What annual instalment will discharge a debt of Rs.
1092 due in 3 years at 12% Simple interest? (c) Rs. 82280 (d) Rs. 83280
(a) Rs. 250 (b) Rs. 275 8. There is 60% increase in an amount in 6 years at simple
(c) Rs. 300 (d) Rs. 325 interest. What will be the compound interest of Rs.
3. A Man invests a certain sum of money at 6% p.a. 12,000 after 3 years at the same rate?
Simple interest and another sum at 7% p.a. Simple (a) Rs. 2160 (b) Rs. 3120
interest. His income from interest after 2 years was (c) Rs. 3972 (d) Rs. 6240
Rs. 354. One fourth of the first sum is equal to one fifth 9. A bank offers 5% compound interest calculated on
of the second sum. Find the total sum he invested? half-yearly basis. A customer deposits Rs. 1600 each
(a) Rs. 1500 (b) Rs. 1200 on 1st January and 1st July of a year. At the end of the
(c) Rs. 2700 (d) Rs. 5400 year, the amount he would have gained by way of
4. Mr. Thomas invested an amount of Rs. 13,900 divided interest is:
in two different schemes A and B at the simple interest (a) Rs. 120 (b) Rs. 121
rate of 14% p.a. and 11% p.a. respectively. If the total
amount of simple interest earned in 2 years be Rs. (c) Rs. 122 (d) Rs. 123
3508, what was the amount invested in Scheme B? 10. Aditya invested certain amount in two different
(a) Rs. 6400 (b) Rs. 6500 schemes A and B. Scheme A offer S.I @ 12 p.c.p.a. and
(c) Rs. 7200 (d) Rs. 7500 scheme B offer C.I. @ 10 p.c.p.a. Interest accured on
the amount invested in scheme A in 2 years was Rs.
5. An automobile financier claims to be lending money
3600 and the total amount invested was Rs. 35000.
at simple interest, but he includes the interest every
What was the interest accured on the amount invested
six months for calculating the principal. If he is
in scheme B?
charging an interest of 10%, the effective rate of interest
becomes: (a) Rs. 4500 (b) Rs. 4200
(a) 10% (b) 10.25% (c) Rs. 4000 (d) Cannot be determine
(c) 10.5% (d) 15% 11. The population of a town in 2009 was 125000. It
6. A sum of Rs. 725 is lent in the beginning of a year at a increase 10% per year. What is the population after 3
certain rate of interest. After 8 months, a sum of Rs. years?
362.50 more is lent but at the rate twice the former. At (a) 166375 (b) 170000
the end of the year, Rs. 33.50 is earned as interest from
(c) 125000 (d) 10000
both the loans. What was the original rate of interest?

179 @BEST300MCQ For More Study Material


Visit: studyiq.com
12. A sum of Rs. 1500 amounts to Rs. 3000 in five years at 20. Which of the statement given below are sufficient to
a certain rate of simple interest. If the rate of interest is answer the following question.
increased by 1% the same sum in the same time would What is the rate of interest per annum on an
amount to: investment of Rs. 800?
(a) Rs. 3,288 (b) Rs. 3,312 (1) The income from S.I. at the end of 3 years at the
(c) Rs. 3,340 (d) Rs. 3075 same rate of interest is 19
13. Abhinav invested a certain amount at the rate of 8 (2) C.I. is 15.76 more than S.I. on same sum & same
p.c.p.a. for 5 years and obtained a SI of Rs. 3800. Had rate in 2 years
he invested the same amount at the same interest for 2 (a) Only I (b) Only II
years, how much amount would she have obtained
(c) Either I or II (d) neither I nor II
as CI at the end of 2 years?
(a) Rs. 1520 (b) Rs. 1550.5 21. Which information given below is sufficient to know
the amount if the difference between the C.I. & S.I. for
(c) Rs. 1550 (d) Rs. 1580.8
2 years is 18?
14. Nutan borrowed some money at rate of 4 p.c.p.a. for
(1) The rate is same at which an amount of Rs. 1000
the first three year, at the rate of 8 p.c.p.a. for the next
become 1120 for 2 years in S.I.
2 years and at the rate of 9 p.c.p.a. for the period beyond
5 years. If she pay a total SI of Rs. 19550 at the end of (2) The principal given is Rs. 2000.
7 years. How much money did she borrow? (a) Only I (b) Only II
(a) Rs. 41500 (b) Rs. 42500 (c) Both I or II (d) Neither I nor II
(c) Rs. 39500 (d) Rs. 40500 22. A sum of Rs. 1521 is lent out in two parts in such a
15. Girish invested some money in a bank. After eight way that the simple interest on one part at 10% for 5
years his amount became triple. How many times of yr is equal to that of another part at 8% for 10 yr. What
the original sum will the money become in 20 years at will be the two parts of sum?
Simple Interest? (a) Rs. 926 and Rs. 595 (b) Rs. 906 and Rs. 615
(a) 5 times (b) 6 times (c) Rs. 916 and Rs. 605 (d) Rs. 936 and Rs. 585
(c) 7 times (d) 8 times 23. Rashmi lent Rs. 600 to Geeta for 2 yr and Rs. 150 to
16. Ram divided 2189 in three parts such that interest on Seeta for 4 yr at same rate of interest and received
them after 1, 2 and 3 years respectively be equal. The altogether Rs. 80 as simple interest from both. Find
rate of SI is 4% per annum in all cases. The smallest the rate of interest.
part is
4 4
(a) Rs. 702 (b) Rs. 398 (a) 3 % (b) 2 %
(c) Rs. 425 (d) Rs. 756 9 9
17. Ravi invested a total amount of Rs. 65000 in three 4 4
different schemes A, B, C with rate of interest 12 (c) 5 % (d) 4 %
9 9
p.c.p.a., 16 p.c.p.a. and 18 p.c.p.a. respectively and
earned a total interest of Rs. 10,180 in one year. If the 24. Hemant makes a fixed deposit of Rs. 20000 in Bank of
amount invested in A is 72% of the amount invested India for a period of 3 yr. If the rate of interest be 13%
in C then what was the amount invested in scheme B. SI per annum charged half-yearly, what amount will
(a) Rs. 15000 (b) Rs. 25000 he get after 42 months?
(c) Rs. 22000 (d) Rs. 18000 (a) Rs. 27800 (b) Rs. 28100
18. The population of Mumbai increase 5% per annum. (c) Rs. 29100 (d) Rs. 30000
It’s population was 100000 in 2004. What was it’s 25. Gaurav borrowed Rs. 800 at 6% and Naresh borrowed
population after 3 year? Rs. 600 at 10%. After how much time, will they both
(a) 115760 (b) 123575 have equal debts?
(c) 132557 (d) 11000 1 1
19. Which of the statement given below are sufficient to (a) 15 yr (b) 14 yr
3 2
answer the following question.
What is the rate of interest p.c.p.a. on the amount of 1 2
(c) 18 yr (d) 16 yr
Rs. 15000 taking loan? 3 3
(1) The difference between C.I. & S.I. is Rs. 170 26. What annual payment will discharge a debt of Rs.
(2) The S.I. of 2 years is Rs. 2500 1092 due in 2 yr at 12% simple interest?
(a) Only I (b) Only II (a) Rs. 725 (b) Rs. 325
(c) Either I or II (d) neither I or II (c) Rs. 515 (d) Rs. 900

180 @BEST300MCQ For More Study Material


Visit: studyiq.com
27. The annual payment of Rs. 160 in 5 yr at 5% per 34. Income of Hemant was Rs. 4000. In the first 2 years,
annum simple interest will discharge a debt of his income decreased by 10% and 5% respectively
(a) Rs. 980 (b) Rs. 880 but in the third year, the income increased by 15%.
What was his income at the end of third year?
(c) Rs. 440 (d) Rs. 220
(a) Rs. 3933 (b) Rs. 4000
28. The population of a city increase at the rate of 5% pa.
(c) Rs. 3500 (d) Rs. 3540
If the present population of the city is 370440, then
what was its population 3 yr ago? 35. A man borrows Rs. 5100 to be paid back with
compound interest at the rate of 4% pa by the end of 2
(a) 3 lakh (b) 3.2 lakh
years in two equal yearly instalments. How much
(c) 3.4 lakh (d) 3.6 lakh will be each instalment?
29. The population of a particular area of a city is 5000. It (a) Rs. 2704 (b) Rs. 2800
increases by 10% in 1st yr. It decreases by 20% in the (c) Rs. 3000 (d) Rs. 2500
2nd yr because of some reason. In the 3rd yr, the 36. Divide Rs. 2602 between X and Y, so that the amount
population increases by 30%. What will be the of X after 7 yr is equal to the amount of Y after 9 yr, the
population of area at the end of 3 years? interest being compounded at 4% pa.
(a) 5120 (b) 5300 (a) Rs. 1352, Rs. 1250 (b) Rs. 1400, Rs. 1350
(c) 5720 (d) 5620 (c) Rs. 1215, Rs. 1300 (d) Rs. 1500, Rs. 1450
30. The cost price of a car is Rs. 400000. If its price 37. If Rs. 3000 amounts to Rs. 4320 at compound interest
decreases by 10% every year, then what will be the in a certain time, then Rs. 3000 amount to what in
cost of car after 3 years? half of the time?
(a) Rs. 3,00,000 (b) Rs. 2,91,700 (a) Rs. 3400 (b) Rs. 3600
(c) Rs. 2,91,600 (d) Rs. 2,50,000 (c) Rs. 38000 (d) Rs. 3520
31. Suneeta borrowed certain sum from Reena for 2 yr at 38. A sum of money lent at compound interest for 2 yr at
simple interest. Suneeta lent this sum to Venu at the 20% pa would fetch Rs. 964 more, if the interest was
same rate for 2 yr at compound interest. At the end of payable half-yearly than if it was payable annually.
2 yr, she received Rs. 110 as compound interest but What is the sum?
paid Rs. 100 as simple interest. Find the sum and the (a) Rs. 40000 (b) Rs. 60000
rate of interest? (c) Rs. 90000 (d) Rs. 500000
(a) Rs. 250, rate 10% pa (b) Rs. 250, rate 20% pa 39. A sum of Rs. 8448 is to be divided between X and Y
who are respectively 18 and 19 yr old, in such a way
(c) Rs. 250, rate 25% pa (d) Rs. 250, rate 30% pa
that if their shares be invested at 6.25% per annum at
32. Find the least number of complete years in which a compound interest, they will receive equal amounts
sum of money put out at 20% compound interest will on attaining the age of 21 yr. The present share of X is
be more than double? (a) Rs. 4225 (b) Rs. 4352
(a) 3 yr (b) 4 yr (c) Rs. 4096 (d) Rs. 4000
(c) 5 yr (d) 8 yr 40. During the first year, the population of a village is
33. The population of a country is 10 crore and it is the increased by 5% and in second year it is diminished
possibility that the population will become 13.31 by 5%. At the end of the second year, its population
crore in 3 years. What will be the annual rate per cent was 47880. What was the population at the beginning
of this growth? of the first year?
(a) 8% (b) 12.7% (a) 45500 (b) 48000
(c) 10% (d) 15% (c) 43500 (d) 53000

1. According to a census report, the population growth (a) 2016 (b) 2014
rate of Faridabad is going to be an increasing A.P. (c) 2013 (d) 2019
with first year’s rate as 5% and common difference as 2. Monica deposited a total of Rs. 10500 with a bank in
5%, but simultaneously the migration rate is an two different deposit schemes at 10% p.a., interest
increasing G.P. with first term as 1% and common being compounded annually. As per the schemes, she
ratio of 2. If population on 31 December 2010 is 1 gets the same amount after 2 years on the first deposit
million, then find in which year will Faridabad as she gets after 3 years on the second deposit. How
much money did she deposit for 3 years?
witness its first fall in population?

181 @BEST300MCQ For More Study Material


Visit: studyiq.com
(a) Rs. 4500 (b) Rs. 5000 (a) Rs. 28800 (b) Rs. 29586
(c) Rs. 6500 (d) Rs. 7200 (c) Rs. 31776 (d) Rs. 32846
3. A sum of money invested at simple interest for two
11. A certain sum of money invested at compound
years at 5% p.a. amounted to Rs. 2750. The rate at
which the sum should have been invested to get Rs. interest becomes 1.44 times of itself in 2 years. If twice
300 as interest after two years would be? this sum were lent at simple interest, in how many
(a) 6% (b) 6.9% years would it double itself?
(c) 7.6% (d) 8.1% (a) 2 years (b) 2.5 years
4. A sum of Rs. 1000 after 3 years at compound interest (c) 3 years (d) 5 years
becomes a certain amount that is equal to the amount 12. Arun gives his wife Asha a birthday gift, giving her
that is the result of a 3 year depreciation from Rs. each year a number of rupees equal to the number of
1728. Find the difference between the rates of C.I. and years of her age. If her birthday falls on August 8,
depreciation? (Given C.I. is 10% p.a.) what sum must be placed at simple interest at 7% on
(a) 1.7% (b) 2.9% January 1, before she is 42, in order to raise the required
(c) 4% (d) 7.6% sum?
5. When the rate of interest in a bank is reduced from 5% (a) Rs. 1000 (b) Rs. 1200
p.a. to 4.5% p.a., Bani deposited Rs. 4000 in her saving (c) Rs. 1500 (d) Rs. 1900
account. She found that the new interest income 13. Bindu lends Rs. 10000 in four parts to her friends. If
remained the same as before. Find her original deposit? she gets 8% on Rs. 2000, 7.5% on Rs. 4000 and 8.5 %
(a) Rs. 3600 (b) Rs. 3150 on Rs. 1400, what percent must she get for the
(c) Rs. 2980 (d) Rs. 2768 remainder, if the average interest is 8.13%.
6. Two equal sums were lent at simple interest for 4 years (a) 6.3% (b) 8.6%
and for 3 years respectively. The rate of interest in the (c) 9% (d) 9.2%
latter case was 3% higher than that of the former case, 14. A sum of money at compound interest amounts in
but the amount in each was Rs. 1088. Find the rate of two years to Rs. 2809, and in three years to Rs. 2977.54.
interest of the former case? Find the rate of interest and the original sum.
(a) 7% (b) 9% (a) 4%, Rs. 2500 (b) 6%, Rs. 1800
(c) 11% (d) 13%
(c) 4%, Rs. 1800 (d) 6%, Rs. 2500
1 1 15. Sapna borrowed a certain sum of money from Kavita
7. Kavita invested rd of her money at 4% p.a., th at
3 4 under the following repayment scheme based on
3% p.a. and the rest at 5% p.a. She received an annual simple interest. 8% p.a. for the initial 2 years, 9.5%
interest of Rs. 500 on her money. Find the total money p.a. for the next 4 years, 11% p.a. for the next 2 years,
invested. 12% p.a. after the first 8 years. Find the amount which
(a) Rs. 21500 (b) Rs. 16480 a sum of Rs. 9000 taken for 12 years becomes at the
(c) Rs. 12000 (d) Rs. 11200 end of 12 years.
8. Shruti borrowed Rs. 2500 from her two friends. For (a) Rs. 20160 (b) Rs. 22350
one loan she paid 8% p.a. and for the other 6% p.a. (c) Rs. 23470 (d) Rs. 24567
simple interest. If she paid Rs. 180 as total interest for 16. If Rs. 1200 amounts to Rs. 1323 in 2 years at compound
one year, how much did she borrow at 8% p.a.? interest compounded annually, what will Rs. 1600
(a) Rs. 1500 (b) Rs. 1700 amount to in 3 years at the same rate?
(c) Rs. 1950 (d) Rs. 2120 (a) Rs. 1780.45 (b) Rs. 1852.50
9. A sum of Rs. 2540 is lent out in two parts, one at 12% (c) Rs. 1945.55 (d) Rs. 2166.76
1 17. If the compound interest on a certain sum for 3 years
and the other at 12 % . If the total annual return on at 5% p.a exceeds the simple interest on the same sum
2
the total money invested is Rs. 312.42, find the money and for the same time and at the same rate by Rs. 183,
lent at 12% p.a? find the sum.
(a) Rs. 996 (b) Rs. 1016 (a) Rs. 17560 (b) Rs. 21680
(c) Rs. 1265 (d) Rs. 1295 (c) Rs. 24000 (d) Rs. 26780
10. A part of Rs. 38800 is lent out at 72% for six months. 18. If the compound interest on a certain sum of money
The rest of the amount is lent out at 5% p.a. after one for 3 years at 5% p.a. is Rs. 504.40, find the
year. The ratio of interest after 3 years from the time corresponding simple interest.
when first amount was lent out is 5 : 4. Find the second (a) Rs. 329.24 (b) Rs. 384.78
part that was lent out at 5%. (c) Rs. 435.45 (d) Rs. 480.00

182 @BEST300MCQ For More Study Material


Visit: studyiq.com
19. Find the least number of complete years in which a 20. If the compound interest on a certain sum of money
sum of Rs. 1500 at 25% compound interest will be for 2 years is Rs. 164 whereas the simple interest for 3
more than doubled. years at the same rate is Rs. 240 on the same sum, find
(a) 4 years (b) 5 years the sum.
(c) 6 years (d) 7 years (a) Rs. 1200 (b) Rs. 1400
(c) Rs. 1600 (d) Rs. 1800

7
(a) 312 (b) 312.50
1. A sum of money becomes of itself in 3 years at a (c) 3120 (d) 3120.50
6
8. Ramesh deposited Rs. 15600 in a fixed deposit at the
certain rate of simple interest. The rate of interest per
rate of 10% per annum simple interest. After every
annum is:
second year, he adds his interest earnings to the
5 5 principal. The interest at the end of fourth year is
(a) 5 % (b) 6 %
9 9 (a) Rs. 1716 (b) Rs. 1560
(c) 18% (d) 25% (c) Rs. 3744 (d) Rs. 1872
2. The simple interest on a certain sum at 5% per annum 9. In how many years will a sum of money double itself
for 3 years and 4 years differ by Rs. 42. The sum is:
1
(a) Rs. 210 (b) Rs. 280 at 6 % simple interest per annum?
(c) Rs. 750 (d) Rs. 840 4
3. A sum of Rs. 10,000 is lent partly at 8% and remaining (a) 24 years (b) 20 years
at 10% per annum. If the yearly interest on the average (c) 16 years (d) 12 years
is 9.2%, the two parts are: 10. A sum of money was invested at a certain rate of simple
(a) Rs. 4000, Rs. 6000 (b) Rs. 4500, Rs. 5500 interest for 2 years. Had it been invested at 1% higher
(c) Rs. 5000, Rs. 5000 (d) Rs. 5500, Rs. 4500 rate, it would have fetched Rs. 24 more interest. The
4. The simple interest on a sum of money is 4/9 of the sum of money is:
principal and the number of years is equal to the rate (a) Rs. 1200 (b) Rs. 1050
percent per annum. The rate per annum is: (c) Rs. 1000 (d) Rs. 9600
2 11. The population of a village decreases at the rate of
(a) 5% (b) 6 % 20% per annum. If its population 2 years ago was
3
10,000, the present population is:
1 (a) 4600 (b) 6400
(c) 6% (d) 7 %
5 (c) 7600 (d) 6000
5. Rs. 500 was invested at 12% per annum simple interest 12. Rs. 12000 is divided into two parts such that the
and a certain sum of money invested at 10% per simple interest on the first part for 3 years at 12% per
annum simple interest. If the total interest on both the annum may be equal to the simple interest on the
sum after 4 years is Rs. 480, the latter sum of money is:
1
(a) Rs. 450 (b) Rs. 750 second part for 4 years at 16% per annum. The ratio
(c) Rs. 600 (d) Rs. 550 2
6. A money lender finds that due to reducing the interest of the first part to the second part is:
(a) 2 : 1 (b) 1 : 2
3
rate decrease from 8% to 7 % , his yearly income (c) 2 : 3 (d) 3 : 2
4 13. If x, y, z, are three sum of money such that y is the
diminishes by Rs. 61.50. His capital is simple interest on x and z is the simple interest on y
(a) Rs. 22400 (b) Rs. 23800 for the same time and at the same rate of interest, then
(c) Rs. 24600 (d) Rs. 26000 we have:
7. If the simple interest on a certain sum of money for 15 (a) z2 = xy (b) xyz = 1
1 (c) x2 = yz (d) y2 = zx
months at 7 % per annum exceeds the simple interest
2 14. A sum of Rs. 7,930 is divided into 3 parts and given
on loan at 5% simple interest to A, B and C for 2, 3 and
1
on the same sum for 8 months at 12 % per annum 4 years respectively. If the amounts of all three are
2 equal after their respective periods of loan, then A
by Rs. 32.50, then the sum of money (in Rs.) is: received a loan of

183 @BEST300MCQ For More Study Material


Visit: studyiq.com
(a) Rs. 2,800 (b) Rs. 3,050 23. The compound interest on Rs. 16,000 for 9 months at
(c) Rs. 2,750 (d) Rs. 2,760 20% per annum, interest being compounded
quarterly, is:
15. The principal which gives Rs. 1 interest per day at a
rate of 5% simple interest per annum is (a) Rs. 2,520 (b) Rs. 2,524
(a) Rs. 5000 (b) Rs. 355000 (c) Rs. 2,522 (d) Rs. 2,518
(c) Rs. 7300 (d) Rs. 3650 24. The compound interest of Rs. 2000 in 2 years if the
16. A boy aged 12 years is left with Rs. 100,000 which is rate of interest is 4% per annum for the first year and
under a trust. The trustees invest the money at 6% per 3% per annum for the second year, will be:
annum and pay the minor boy a sum of Rs. 2500, for (a) Rs. 142.40 (b) Rs. 140.40
his pocket money at the end of each year. The expenses (c) Rs. 141.40 (d) Rs. 143.40
of trust come out to be Rs. 500 per annum. Find the 25. A money-lender borrows money at 4% per annum
amount that will be handed over to the minor boy and pays the interest at the end of the year. He lends
after he attains the age of 18 years. it at 6% per annum at compound interest compounded
(a) Rs. 125000 (b) Rs. 118000 half yearly and receives the interest at the end of the
(c) Rs. 150000 (d) Rs. 120000 year. In this way, he gains Rs. 104.50 a year. The
17. Tushar borrowed a sum of Rs. 12,000 at 15% per amount of money he borrows, is:
annum from a money-lender on 13th January, 1987 (a) Rs. 6,000 (b) Rs. 5,000
and return the amount on 8th June, 1987 to clear his (c) Rs. 7,500 (d) Rs. 8000
debt. Then the amount paid by Tushar to the money- 26. A sum of money at compound interest amounts to
lender to clear his debt was. thrice itself in 3 years. In how many years will it be 9
(a) Rs. 15,000 (b) Rs. 11,394 times itself?
(c) Rs. 12,720 (d) Rs. 13,650 (a) 9 years (b) 27 years
18. The compound interest on Rs. 10,000 in 2 years at 4% (c) 6 years (d) 3 years
per annum, the interest being compound half-yearly, 27. The difference between the compound interest and
is: the simple interest on a certain sum at 5% per annum
(a) Rs. 636.80 (b) Rs. 824.32 for 2 years is Rs. 1.50. The sum is:
(c) Rs. 912.86 (d) Rs. 825.82 (a) Rs. 600 (b) Rs. 500
19. The sum of money invested at compound interest (c) Rs. 400 (d) Rs. 300
amounts in 3 years to Rs. 2,400 and in 4 years to Rs. 28. What sum will give Rs. 244 as the difference between
2,520. The interest rate per annum is: simple interest and compound interest at 10% in
(a) 5% (b) 6%
1
(c) 10% (d) 12% 1 years compounded half yearly?
2
20. On a certain sum of money the compound interest for
(a) Rs. 40,000 (b) Rs. 36,000
2 years is Rs. 282.15 and the simple interest for the
same period of time is Rs. 270. The rate of interest per (c) Rs. 32,000 (d) Rs. 28,000
annum is: 29. An amount of money appreciates to Rs. 7,000 after 4
(a) 6.07% (b) 10% years and to Rs. 10,000 after 8 years at a certain
compound interest compounded annually. The initial
(c) 9% (d) 12.15%
amount of money was
21. In what time will Rs. 1000 becomes Rs. 1331 at 10%
(a) Rs. 4700 (b) Rs. 4900
per annum, if compounded annually?
(c) Rs. 4100 (d) Rs. 4300
1 30. What does Rs. 250 amounts to in 2 years with
(a) 3 years (b) 2 years
2 compound interest at the rate of 4% in the 1st year
and 8% in the second year?
1
(c) 2 years (d) 3 years (a) Rs. 280 (b) Rs. 280.80
2
(c) Rs. 468 (d) Rs. 290.80
22. A sum of money on compound interest amounts to
31. Two years ago, the value of my motorbike was Rs.
Rs. 10648 in 3 years and Rs. 9680 in 2 years. The rate
62500. If the value depreciates by 4% every year, now
of interest per annum is:
its value is:
(a) 5% (b) 10%
(a) Rs. 56700 (b) Rs. 57600
(c) 15% (d) 20%
(c) Rs. 57500 (d) Rs. 55700

184 @BEST300MCQ For More Study Material


Visit: studyiq.com
32. A sum of Rs. 210 was taken as a loan. This is to be 36. A sum of money placed at compound interest doubles
paid back in two equal installments. If the rate of itself in 5 years. It will amount to eight times itself at
interest be 10% compounded annually, then the value the same rate of interest in
of each installment is (a) 15 years (b) 10 years
(a) Rs. 127 (b) Rs. 121
(c) 12 years (d) 20 years
(c) Rs. 210 (d) Rs. 225
37. On what sum of money will the difference between
33. A certain amount of money earns Rs. 540 as simple
simple interest and compound interest for 2 years at
interest in 3 years. If it earns a compound interest of
5% per annum be equal to Rs. 63
Rs. 376.20 at the same rate of interest in 2 years, find
the amount (in Rupees). (a) Rs. 23200 (b) Rs. 29200
(a) 1600 (b) 1800 (c) Rs. 25200 (d) Rs. 31200
(c) 2100 (d) 2000 38. The amount on Rs. 25000 in 2 years at annual
34. In what time will Rs. 64,000 amount to Rs. 68,921 at compound interest, if the rates for the successive years
5% per annum interest being compounded half be 4% and 5% per annum respecively is:
yearly? (a) Rs. 26800 (b) Rs. 27300
1 (c) Rs. 28500 (c) Rs. 30000
(a) 1 years (b) 2 years
2 39. At what rate of compount interest per annum will a
1 sum of Rs. 1200 become Rs. 1348.32 in 2 years?
(c) 3 years years.
(d) 2 (a) 6.5% (b) 4.5%
2
35. A sum of money is paid back in two annual (c) 6% (d) 7.5%
installments of Rs. 17,640 each, allowing 5% 40. The compound interest on 12000 for 9 months at 20%
compound interest compounded annually. The sum per annum, interest being compounded quarterly is:
borrowed was
(a) Rs. 1891.50 (b) Rs. 2089.70
(a) Rs. 32,800 (b) Rs. 32,400
(c) Rs. 2136 (d) Rs. 1750
(c) Rs. 32,000 (d) Rs. 32,200

S olut ions éæ 12 ö
2
ù
= 17500 êç 1 + ÷ - 1 ú = Rs. 4452
êëè 100 ø úû
PRT 800 ´ 5 ´ 3
1. (a); SI = = = Rs. 120
100 100 éæ R ö
n
ù
6. (c); CI = P êç 1 + ÷ - 1ú
SI ´ 100 120 ´ 100 ëêè 100 ø úû
2. (c); T = = = 2 years
PR 600 ´ 10 Here, P = Rs. 12000, R = 9% p.a, n = 3 years

SI ´ 100 4500 ´ 100 éæ 109 ö 3 ù


3. (d); R = = = 6% = 12000 êç ÷ - 1ú = Rs. 3540 (Appx.)
PT 15000 ´ 5 êëè 100 ø úû
4. (a); Total SI difference = 1400 – 1100 = Rs. 300
Total Time = 6 – 2 = 4 years éæ R ö
n
ù
7. (d); CI = P êç 1 + ÷ - 1ú
êëè 100 ø úû
300
SI = = Rs. 75
4
éæ 5 ö
3
ù
For 2 year SI = 75 × 2 = Rs. 150 = 4800 êç 1 + ÷ - 1ú = Rs. 756.6
So, P = 1100 – 150 = Rs. 950 ëêè 100 ø ûú

75 17 éæ R ö
n
ù
R= ´ 100 = 7 % 8. (a); CI = P êç 1 + - 1ú
950 19 è ÷
ø
êë 100 úû
éæ R ö
n
ù
5. (b); CI = P êç 1 + ÷ - 1ú éæ 12 ö
2
ù
ëêè 100 ø úû = 12500 êç 1 + ÷ - 1 ú = Rs. 3180
ëêè 100 ø úû

185 @BEST300MCQ For More Study Material


Visit: studyiq.com
PR 2 P´R´T
9. (a); Difference between CI and SI for 2 years = 16. (b); Amount = P +
100 2 100

10000 ´ R 2 P ´ 2 ´ 10
64 = Þ R2 = 64 Þ R = 8% 450 = P + Þ P = Rs. 375
100 ´ 100 100
P´R´T
PR 2 17. (b); Amount = P +
10. (a); Difference between CI and SI for 2 years = 100
100 2
22400 ´ 12 ´ 7
P ´ 16 = 22400 + = Rs. 41216
Þ = 1 Þ P = Rs. 625 100
100 2
P´R´T
11. (c); Because amount compound quarterly 18. (b); Amount = P +
100
rate
So, Year = year × 4, Rate = 7000 ´ 5 ´ T
4 10500 = 7000 + Þ T = 10 years
100
9 8
\ year = ´ 4 = 3 years , rate = = 2% PRT P´9´5
12 4 19. (d); SI = Þ 4016.25 = Þ P = Rs. 8925
100 100
éæ 2 ö
3
ù 20. (c); Year = 2 years,
\ CI = 12500 êç 1 + ÷ - 1ú = Rs. 765
ëêè 100 ø ûú R=
10
= 5% [Q CI is compounded half-yearly]
2
12. (b); Because the amount is compounded half–yearly
PR 2
rate Difference between CI and SI for 2 years =
So, year = year × 2, rate = 100 2
2
\ year = 2 years, rate = 10% P ´ 25
25 = ; P = Rs. 10000
100 ´ 100
éæ R ö
n
ù
Þ CI = P êç 1 + ÷ - 1ú PR 2
ëêè 100 ø úû 21. (b); Difference between CI and SI for 2 years =
1002

éæ 10 ö
2
ù 2000 ´ R 2
= 32000 êç 1 + - 1ú = Rs. 6720 12.8 = Þ R = 8% per annum
êë è 100
÷
ø úû ( 100) 2
22. (c); Here, Amount = Principal + SI
PR 2 Now, since amount is given as 2P, hence SI must
13. (c); Difference between CI and SI for 2 years =
100 2 be equal to P

700 ´ 5 ´ 5 P´R´6
= = Rs. 1.75 Þ P= [Using SI formula]
100 ´ 100 100
50
PR 2 ÞR= % per annum
14. (c); Difference between CI and SI for 2 years = 3
1002
Now, Amount = 4P (Need to find)
25 Hence, SI = 3P
Here, Difference = Rs. 10, R = % per annum
4 50
P´ ´T
2 3 50
é 25 ù 3P = Þ 300 = ´ T or T = 18 years
Þ 10 = P ´ ê Þ P = Rs. 2560 100 3
ë 4 ´ 100 úû
23. (d); Let the sum of money be Rs. 100
15. (b); Here, amount = 2P (Where P = Principal) 3 years 3 years
Rs. 100 ¾¾¾ ® Rs. 200 ¾¾¾ ® Rs. 400
100 ´ SI P ´ 100
Hence, SI = P, Rate = = = 12.5% Here, Rs. 100 becomes 4 times i.e. Rs. 400 in total
P´T P´8 6 years

186 @BEST300MCQ For More Study Material


Visit: studyiq.com
24. (d); Let amount be 100 30. (a); The difference between CI and SI for 3 years
25 PR 2 ( 300 + R )
Then 100 ´ = Rs. 625 (Final amount) Þ Difference =
4 ( 100) 3
éæ r ö
2
ù
13000 ´ ( 4 ) ( 304 )
2
So, 625 = 100 êç 1 + ÷ ú
êëè 100 ø úû = = Rs. 63.23 (appx.)
( 100)3
25 æ 100 + r ö 5 100 + r
Þ =ç ÷Þ = Þ r = 150% SI ´ 100
10 è 100 ø 2 100 31. (d); Principal =
Rate ´ Time
éæ R ö
2
ù 500 ´ 100
25. (d); Here, Amount = P êç 1 + ÷ ú = = Rs. 10000
êë è 100 ø úû 5´1
Þ Now CI at 5% rate
éæ 8 ö ù
2

= 25000 êç 1 + ÷ ú éæ 5 ö
2
ù
ëêè 100 ø ûú = 10000 êç 1 + ÷ - 1 ú = Rs. 1025
êëè 100 ø úû
éæ 27 ö 2 ù
= 25000 êç ÷ ú = Rs. 29160 PR 2
è ø 32. (c); Difference between CI and SI for 2 years =
ëê 25 ûú 1002

P´R´T P ( 5)
2

SI 100 Þ 35 = Þ P = Rs. 14000


26. (b); Here,
CI
=
éæ 2
ù
( 100) 2
R ö
P êç 1 + ÷ - 1ú
ëêè 100 ø ûú 20 3
33. (c); Rate = = 10% , Time = ´ 2 = 3 years
2 2
100 ´ 5 ´ 2 10 [\ CI is calculated half yearly]
= 100 = 100 = 40 : 41
éæ 5 ö
2
ù 41 éæ 10 ö
3
ù
100 êç 1 + ÷ - 1ú 400 CI = 10000 êç 1 + ÷ - 1ú
êëè 100 ø úû ëêè 100 ø ûú

27. (c); CI becomes Rs. 800 in 3 years and becomes Rs. éæ 11 ö 3 ù


840 in 4 years = 10000 êç ÷ - 1ú = Rs. 3310
è
êë 10 ø úû
Þ Hence, CI of 1 year = Rs. (840 – 800) = Rs. 40

40 ´ 100 SI ´ 100 7200 ´ 100


Þ Rate = [Here, P = Rs. 800] 34. (d); P = = = Rs. 10000
800 ´ 1 R ´T 12 ´ 6
= 5% per annum
éæ R ö
2
ù
28. (c); Here, difference between SI and CI for 2 years Þ CI = P êç 1 + ÷ - 1ú
ëêè 100 ø úû
12000 ´ ( 5)
2
PR 2
= = = Rs. 30
( 100) 2 ( 100) 2 éæ
= 10000 êç 1 +
5 ö
2
ù
- 1ú = Rs. 1025
÷
29. (a); Difference between CI and SI for 3 years ëêè 100 ø úû

PR 2 ( 300 + R ) PR 2
Þ Difference = 35. (c); Difference between CI and SI for 2 years =
( 100) 3 ( 100) 2

( 15000)( 3) 2 ( 303) = Rs. 40.91 (appx.) Þ 25 =


( 10000 ) R 2 Þ R = 5% per annum
=
( 100) 3 ( 100 ) 2

187 @BEST300MCQ For More Study Material


Visit: studyiq.com
38. (c); Here, the amount is compounded half-yearly
éæ R ö
2
ù
36. (c); Here, CI = P êç 1 + ÷ - 1ú R
è 100 ø Hence, Rate = % = 10% per annum
ëê úû
2
éæ ù PR R Time = 2T = 2 × 2 = 4 years
R2 2R ö é ù
= P êç 1 + + ÷ - 1 ú= +2
( 100) 2 100 ø úû 100 êë 100 úû éæ ù
n
êëè R ö
Þ CI = P êç 1 + ÷ - 1ú
ëêè 100 ø úû
PR é R + 200 ù
P´2´R 100 êë 100 úû éæ ù
4
CI 10 ö
Þ SI = Þ = 1.2 = = 10000 êç 1 + ÷ - 1ú = Rs. 4641
100 SI 2PR è 100 ø
ëê úû
100
PR 2
CI 200 + R 200 + R 39. (c); Difference between SI and CI for 2 years =
Þ
SI
=
200
, 1.2 =
200
, R = 40% ( 100) 2

éæ 10 ö æ
2
20 ö
2
ù
Þ Difference =
( 10000)( 12 )
2

= Rs. 144
37. (b); CI = 7500 êç 1 + ÷ ´ ç 1 + ÷ - 1ú
ëê
è 100 ø è 100 ø úû ( 100) 2
[Here, R = 10% per annum for first 2 years and R Pr 2
= 20% per annum for next 2 years] 40. (d); Difference between SI and CI for 2 years =
1002
éæ 11 ö 2 æ 12 ö 2 ù
= 7500 êç ÷ ´ ç ÷ - 1ú = 5568 10000 ´ 14 2
è ø è ø Difference = = Rs. 196
êë 10 10 úû 10000

1. (c); Let the original amount be Rs. P. 4. (a); Let 14% amount = Rs. x
T = 12, R1 = 8%, R2 = 12%, Profit = 320 11% amount = Rs. (13900 – x)
P ´ T ´ R2 P ´ T ´ R1
- = 320 x ´ 2 ´ 14 ( 13900 - x) ´ 11 ´ 2
100 100 + = 3508
100 100
P ´ 12 ´ 12 P ´ 8 ´ 12 2000 6x = 350800 – 13900 × 22 Þ x = 7500
- = 320 Þ P =
100 100 3
Scheme B = 13900 – 7500 = Rs. 6400
P = Rs. 666.67
2. (d); Let each instalment be Rs. A 5. (b); Let the sum be Rs. 100 and interest is calculated
half-yearly.
100P 100 ´ 1092
A= = = Rs. 325 10
RT ( T - 1 ) 100 ´ 3 + 36 Þ Then, Rate = = 5% , Time = 2 years
100T + 2
2
3. (c); Let the sums be A and B 5´5
R1 = 6, R2 = 7, T = 2 Effective rate of interest = 5 + 5 + = 10.25%
100
P1 ´ R 1 ´ T P2 ´ R 2 ´ T 6. (c); Let the original rate = R%
+ = 354
100 100 New rate = (2R)%
A ´6´2 B´7 ´2
Þ + = 354 Þ 6A + 7B = 17700 725 ´ R ´ 1 362.50 ´ 2R ´ 1
100 100 + = 33.50
100 3 ´ 100
also one fourth of the first sum is equal to one
fifth of the second sum Þ (2175 + 725) R = 33.50 × 100 × 3
Þ (2900) R = 10050 Þ R = 3.46%
A B
= = 5A - 4B = 0 7. (c); CI for first 2 years
4 5
By solving the equations we get, éæ 2
ù
10 ö
A = 1200, B = 1500 = 50000 êç 1 + ÷ - 1ú = Rs. 10500
è 100 ø
Total sum = Rs. (1200 + 1500) = Rs. 2700 ëê úû

188 @BEST300MCQ For More Study Material


Visit: studyiq.com
Amount = P + CI = 50000 + 10500 = Rs. 60500 12. (d); Here, diference = SI = 3000 – 1500 = Rs. 1500
P ´ R ´ T 60500 ´ 12 ´ 3 1500 ´ 100
SI = = = Rs. 21780 Þ Rate = = 20%, (P = Rs. 1500)
100 100 1500 ´ 5
Total amount = Rs. 82280
Þ Now, Rate of interest = (20 + 1)% = 21%
60
8. (c); Let P = 100, A = 100 ´ + 100 = Rs. 160 1500 ´ 5 ´ 21
100 Þ Hence, new SI = = Rs. 1575
100
60 ´ 100 Þ Amount = Principal + SI
I = 160 – 100 = Rs. 60, R = = 10%
100 ´ 6 = Rs. [1500 + 1575] = Rs. 3075

331 3800 ´ 100


CI = ´ 12000 = Rs. 3972 13. (d); Principal = = Rs. 9500
1000 8´5
9. (b); Here, CI is calculated on half-yearly basis éæ R ö
n
ù
Þ CI = P êç 1 + ÷ - 1ú
Hence, Amount would be defined as: êëè 100 ø úû
é æ 5 ö
2
æ 5 öù
A = ê1600 ç 1 + ÷ + 1600 ç 1 + ÷ú éæ 8 ö
2
ù
ëê è 2 ´ 100 ø è 2 ´ 100 ø ûú = 9500 êç 1 + ÷ - 1ú = Rs. 1580.8
è 100 ø
ëê úû
æ Rate 5 ö 14. (b); Total SI = Rs. 19550
çè Where Rate = = %÷
2 2 ø P ´ R 1 ´ T1 P ´ R 2 ´ T2 P ´ R 3 ´ T3
19550 = + +
100 100 100
é 41 æ 41 ö ù
A = Rs. ê 1600 ´ ç + 1÷ = Rs. 3321
ë 40 è 40 ø úû é4´ 3 8´ 2 9´ 2ù
19550 = P ê + +
ë 100 100 100 úû
CI = A – P = Rs. (3321 – 3200) = Rs. 121
é 12 16 18 ù
19550 = P ê + + Þ P = Rs. 42500
10. (b); Let the money in first installment = Rs. x ë 100 100 100 úû
Second installment = Rs. (35000 – x) 15. (b); Let his money be 100 Rs.
The principal is defined as: After eight year it becomes 300
\ interest = 200
3600 ´ 100
Þ P= = Rs. 15000 200 ´ 100
12 ´ 2 So, Rate = = 25%
(Using SI in scheme A) 100 ´ 8
The money in first installment = Rs. x = Rs. 15000 100 ´ 25 ´ 20
In 20 year, SI = = 500
Scheme B = Rs. (35000 – 15000) = Rs. 20000 100
So, amount = 500 + 100 = Rs. 600
éæ R ö
2
ù Hence, from Rs. 100 to Rs. 600, it becomes 6 times
Þ CI = P êç 1 + ÷ - 1ú
êëè 100 ø úû 16. (b); Let amount be a, b, c respectively then,
a´4´1 b´4´2 c´4´3
éæ 2
ù = =
10 ö 100 100 100
= 20000 êç 1 + ÷ - 1ú = Rs. 4200
ëêè 100 ø úû Þ a = 2b = 3c = 6K(let)
\ a = 6K, b = 3K, c = 2K
11. (a); Let population P be 100
so the part are in ratio 6:3:2
+ 10%­ + 10%­ + 10% ­
100 ¾¾¾® 110 ¾¾¾® 121 ¾¾¾® 133.1 2189
\ Smallest part = ´ 2 = 199 ´ 2 = Rs. 398
For P = 100, increase in 3 years = 133.1 11
For P = 125000, increase in 3 years 17. (c); Suppose amount invested in scheme A = x
Suppose amount invested in scheme B = y
133.1
= ´ 125000 = 166375 Suppose amount invested in scheme C = z
100
\ x + y + z = 65000 ...(i)

189 @BEST300MCQ For More Study Material


Visit: studyiq.com
From question 23. (d); Given, T1 = 2 years and T2 = 4 years,
18 P1 = Rs. 600, P2 = Rs. 150 and SI1 + SI2 = Rs. 80
x = 72% of z = z ...(ii) According to the question,
25
So, 12x + 16y + 18z = 1018000 600 ´ R ´ 2 150 ´ R ´ 4
Þ 6x + 8y + 9z = 509000 ...(iii) + = 80
100 100
From (i), (ii) and (iii) get
Þ 120R + 60R = 800 Þ 180R = 800
43
z + y = 65000 ...(iv) 800 80 40 4
25 \ R= = = =4 %
180 18 9 9
108 24. (c); Given, time = 42 months
z + 8y + 9z = 509000
25
42 1
333 Þ years = 3 years
Þ z + 8y = 509000 ...(v) 12 2
25
7 13
344 Þ ´ 2 = 7 half-yr, rate = % half-yearly
From (iv) we have z + 8y = 520000 ...(vi) 2 2
25
20000 ´ 13 ´ 7
11 SI = = Rs. 9100
\ (vi) – (v) Þ z = 11000 100 ´ 2
25
\ Amount (A) = 20000 + 9100 = Rs. 29100
\ 11z = Rs. 275000, z = Rs. 25000
25. (d); Given, R1 = 6%, R2 = 10%
\ y = Rs. 22000
18. (a); Here, Population P = 100000 According to the question,

100000 ¾ 5%­
¾® 105000 ¾ 5%­
¾® 110250 ¾5%­
¾® 115762.5 800 ´ 6 ´ T 600 ´ 10 ´ T
800 + = 600 +
(2004) (2005) (2006) (2007) 100 100
Hence Population after 3 years is almost 115760 800 + 48T = 600 + 60T Þ 12T = 200 Þ 3T = 50
19. (c); This question can be solved either using statement
(1) or (2) 50 2
\ T= = 16 years
In statement (1), it is simple formula of difference 3 3
PR 2 26. (c); According to the formula,
=
( 100) 2
100P
Annual payment =
In statement (2), it is simple formula of SI R T ( T - 1)
20. (c); This question can be solved either using statement 100T +
2
(1) or (2)
In statement (1), we can apply formula of SI in 1092 ´ 100 1092 ´ 100
statement (2), we need to determine difference = =
24 ( 2 - 1) 212
100 ´ 2 +
PR 2 2
formula of =
( 100 ) 2 = Rs. 515.09 » Rs. 515
21. (c); We can solve this question using both 27. (b); Given, annual payment = Rs. 160
information together. R = 5%, T = 5 years, debt (P) = ?
Because, in statement I, we can find rate of interest According to the formula,
and in statement II, we know the Principal.
22. (d); Given, T1 = 5 yr, R1 = 10% and 100P
Annual payment =
T2 = 10 yr and R2 = 8% R T ( T - 1)
100T +
Let the first part = x 2
Then, second part = (1521 – x)
Now, according to the question, 100P 100P
Þ 160 = Þ 160 =
5´4´5 550
x ´ 5 ´ 10 ( 1521 - x ) ´ 10 ´ 8 5 ´ 100 +
= 2
100 100
Þ 5x = 12168 – 8x Þ 13x = 12168, x = Rs. 936 550 ´ 160
and second part = 1521 – 936 = Rs. 585 \ P= = 55 ´ 16 = Rs. 880
100
190 @BEST300MCQ For More Study Material
Visit: studyiq.com
28. (b); Given that, P = 370440, R = 5% and n = 3 years 32. (b); Let the sum be Rs. P
According to the formula, According to given question:
n n
P æ R ö æ 20 ö
Population n years ago = n Þ Pç1+ ÷ > 2P Þ P ç 1 + ÷ > 2P
æ R ö è 100 ø è 100 ø
çè 1 + 100 ÷ø
n
æ 6ö
Þ ç ÷ > 2 By hit and tril method, n > 4 years
370440 370440 è 5ø
Population 3 years ago = 3
= 3
æ 5 ö æ 21 ö 33. (c); Given, P = 10 crore
çè 1 + ÷ çè ÷ø
100 ø 20 and population after 3 years = 13.31 crore
According to the formula,
370440 ´ 20 ´ 20 ´ 20 370440 ´ 8000
= = n
21 ´ 21 ´ 21 9261 æ R ö
Population after n year = P ç 1 + ÷
= 40 × 8000 = 320000 è 100 ø
29. (c); Population of particular area P of a city = 5000
3 3
In 1st year population got increased by 10% æ R ö 1331 æ R ö
Þ 13.31 = 10 ç 1 + ÷ø Þ = ç1+ ÷
\ Population P1 = 5000 + 10% of 5000 = 5500 è 100 1000 è 100 ø
In 2nd year population decreased by 20%
3 3
\ Population P2 = 5500 – 20% of 5500 æ 11 ö æ R ö R 11
Þ ç ÷ = ç1+ ÷ Þ 1+ =
è 10 ø è 100 ø 100 10
20
= 5500 - × 5500 = 4400
100 R 11 1
Þ = -1= \ R = 10%
In 3rd year population increased by 30% 100 10 10
Population P3 = 4400 + 30% of 4400 34. (a); Initial income of Hemant, P = Rs. 4000
30 Let r1(rate of decrease) = 10%,
= 4400 + × 4400 = 5720
100 r2(rate of decrease) = 5%, r3(rate of growth) = 15%
30. (c); Given that cost of a car, P = Rs. 400000 So, Net final
Þ Decreasing rate = 10%, year = 3 years
æ r öæ r öæ r ö
Income = P ç 1 - 1 ÷ ç 1 - 2 ÷ ç 1 + 3 ÷
æ R ö
3
è 100 ø è 100 ø è 100 ø
Þ Price of car after 3 years = P ç 1 - ÷
è 100 ø
æ 10 ö æ 5 öæ 15 ö
= 4000 çè 1 - ÷ø çè 1 - ÷ø çè 1 + ÷
æ 10 ö
3
9 9 9 100 100 100 ø
= 400000 ç 1 - ÷ = 400000 ´ ´ ´
è 100 ø 10 10 10
9 19 23
= Rs. 291600 = 4000 ´ ´ ´ = Rs. 3933
10 20 20
31. (b); Let the sum borrowed be Rs. P
35. (a); Let the installment in each part be Rs. P
Rate of interest = R%
According to question:
CI = Rs. 110, SI = Rs. 100, Time (t) = 2 years
CI – SI = 110 – 100 = 10 P P
Þ + 2
= Rs. 5100
æ R ö æ R ö
PR 2 PR 2 çè 1 + 100 ÷ø
Þ Difference = Þ 10 = ...(i) çè 1 + ÷
100 ø
( 100) 2 ( 100) 2
SI ´ R P P
Þ Difference = Þ + 2
= 5100
200 æ 4 ö æ 4 ö
çè 1 + ÷ çè 1 + ÷
100 ø 100 ø
100 ´ R
10 = ; R = 20% ...(ii)
200 25P 625
+ P = 5100 Þ 1275P = 5100 × 676
Put value (ii) in (i) 26 676
P ( 20)
2
5100 ´ 676
Þ 10 = ; P = Rs. 250 P= = Rs. 2704
( 100) 2
1275

191 @BEST300MCQ For More Study Material


Visit: studyiq.com
36. (a); Let the first part be Rs. a
R
Second part Rs. (2602 – a) (where R = % p.a. and T = 2T years)
According to question: 2
Amount after 7 years = Amount after 9 years éæ 11 ö4 ù 4641 P
7 9 Þ CI = P êç ÷ - 1ú =
æ r ö æ r ö 10000
Þ aç1 + ÷ = ( 2602 - a ) ç 1 + ÷ ëêè 10 ø ûú
è 100 ø è 100 ø
If CI is compounded annually,
2
a æ 4 ö éæ 20 ö
2
ù 11P
Þ = 1+ - 1ú =
( 2602 - a) çè 100 ÷ø Þ CI = P êç 1 +
ëêè
÷
100 ø úû 25

a 26 26 676 According to question:


Þ = ´ =
( 2602 - a ) 25 25 625 4641P 11
Þ - P = 964
Þ 625a = 676 (2602 – a) 10000 25
676 ´ 2602 964 ´ 10000
Þ a= = Rs. 1352 Þ P= = Rs. 40000
1301 241
Second part = Rs. (2602 – a) = Rs. 1250 39. (c); Let shares of X and Y be Rs. x and Rs. (8448 – x),
37. (b); Let rate = R% and time = n year respectively.
n Amount got by X after 3 years = Amount got by Y
æ R ö
Then, 4320 = 3000 ç 1 + ÷ after 2 years
è 100 ø
3 2
æ 6.25 ö æ 6.25 ö
æ R ö
n
4320
xç1 +
è ÷ = ( 8448 - x) ´ çè 1 + ÷
100 ø 100 ø
Þ ç1+ ÷ = = 1.44
è 100 ø 3000
6.25 8448 - x 1 8448 - x
n Þ 1+ = Þ1+ =
æ R ö 2 100 x 16 x
\ ç1+ ÷ = 1.44 = 1.2
è 100 ø 17 8448 - x
Þ = Þ 17x = 135168 – 16xÞ
n 16 x
\ Required amount for year x = Rs. 4096
2
40. (b); Let the population at beginning of first year be 'a'
æ R ö
n
2 So, according to question:
= 3000 ç 1 + ÷ = 3000 × 1.2 = Rs. 3600
è 100 ø é 5 ù é 5 ù
Þ a ê1 + ú ´ ê1 - úû = 47880
38. (a); Let the principal be Rs. P ë 100 û ë 100
If CI is compounded half-yearly,
é 21 ù é 19 ù
éæ 4
ù Þ a ´ ê ú ´ ê ú = 47880
10 ö ë 20 û ë 20 û
Þ CI = P êç 1 + ÷ - 1ú
êëè 10 ø úû 47880 ´ 20 ´ 20
Þ a= = Rs. 48000
21 ´ 19

1. (a); Here from 2010 on wards we would determine Growth rate Migration rate
the population of Faridabad. Year (A.P. where a (G.P. where a
Here, we would be creating 2 tables, one for = 5%, d = 5%) = 1%, r = 2%)
population growth rate and another for decrease
due to migration. On comparing both the tables, 2011 5% 1%
if the migration rate would become greater than 2012 10% 2%
population growth rate, then there would be a
2013 15% 4%
fall in population.
2014 20% 8%
2015 25% 16%
2016 30% 32%

192 @BEST300MCQ For More Study Material


Visit: studyiq.com
From 2016 on wards, Migration rate > Growth 5. (a); Here, we assume original deposit of Bani be Rs. x.
rate Þ The SI obtained in both cases is same
2. (b); Let the amount invested by Monica in first
scheme be x´ 5´ T 4000 ´ 4.5 ´ T
=
Rs. x and in another be Rs. (10500 – x) Þ 100 100
Rate of interest = 10% p.a. [R = 5% p.a.] [R = 4.5% p.a.]
According to CI Þ 5x = 4000 × 4.5 Þ x = Rs. 3600
The amount in 1st scheme after 2 years
6. (b); Here, Amount = Rs. 1088
= Rs. 1.21x
Let principal invested be Rs. x and Let Rate of
The amount in 2nd scheme after 3 years
interest be R% in first case and (R + 3)% in later
= Rs. 1.331 (10500 – x)
case.
Now since, she got same amount from both
scheme. Hence, æ RT ö
Þ A = Pç1+ ÷
Þ 1.21x = 1.331 (10500 – x) è 100 ø
Þ1.21x + 1.331x = 10500 × 1.331
Þ 2.541x = 13975.5 æ 4R ö
Þ 1088 = x ç 1 + ÷ ...(i)
è 100 ø
13975.5
x= = 5500
2.541 æ 3 ( R + 3) ö
Þ Hence, her amount in 3 years scheme is Þ 1088 = x ç 1 + ...(ii)
è 100 ÷ø
Þ (10500 – 5500) = Rs. 5000
P´R´T æ 4R ö æ 3 ( R + 3) ö
3. (a); Here, SI = Hence, x ç 1 + ÷ = xç1+
100 è 100 ø è 100 ÷ø
é 10 ù
Þ A - P = P ´ 5 ´ 2 Þ 2750 = P ê1 + 4R 3R + 9
100 ë 100 úû Þ = Þ R = 9% p.a.
100 100
Þ Principal P = Rs. 2500
7. (c); Let the total money invested be Rs. P.
Now, we need SI for Rs. 300 after 2 years.
According to question:
2500 ´ R ´ 2
Þ SI = 300 = Þ R = 6% æP ö æP ö
100
çè ´ 4 ´ 1÷ø çè ´ 3 ´ 1÷ø
4. (a); The amount of Rs. 1000 at 10% p.a. after 3 years Þ 500 = 3 +
4
+
would becomes: 100 100
3 3
æ R ö æ 10 ö éæ æ P Pöö ù
Þ Pç1+
è ÷ = A Þ 1000 çè 1 + ÷ =A êçè P - çè 3 + 4 ÷ø ÷ø ´ 5 ´ 1ú
100 ø 100 ø
ë û
Þ A = Rs. 1331 100
Here, now the principal of Rs. 1728 depreciated
at rate of R% per annum and amounts to Rs. 1331 4P 3P 25P
Þ 500 = + +
3 300 400 1200
éæ R öù
Þ P êçè 1 - ÷ = 1331
ë 100 ø úû 16P 9P 25P
Þ 500 = + + Þ P = Rs. 12000
1200 1200 1200
3
éæ R öù 8. (a); Let Shruti borrowed Rs. x from one of her friends.
Þ 1728 êçè 1 - ÷ = 1331
ë 100 ø úû Then remainning amount borrowed from another
friend be Rs. (2500 – x)
3
æ R ö 1331 R 11 Þ Total SI for 1 year = Rs. 180
Þ ç1- ÷ = Þ 1- =
è 100 ø 1728 100 12
x ´ 8 ´ 1 ( 2500 - x) ´ 6 ´ 1
Þ 180 = +
R 1 100 100
Þ = or R = 8.33%
100 12
8x 6 ( 2500 - x)
Þ Hence rate difference between CI and Þ 180 = +
100 100
depreciation is almost = (10 – 8.33)% = 1.7%.

193 @BEST300MCQ For More Study Material


Visit: studyiq.com
8x 15000 - 6x
Þ 180 = + Thus Arun must invest certain sum for
100 100
3
8x - 6x th of year at 7% to get 42 Rs.
Þ 180 = + 150 5
100
(at 42 years of Asha's age)
2x P ´7 ´ 3
Þ 30 = x = Rs. 1500 Þ SI = 42 = Þ P = Rs. 1000
100 100 ´ 5
9. (b); Let us evaluate overall rate of interest. 13. (c); Here, Let us assume the rate of interest overall be
2540 ´ R ´ 1 8.13% (average interest)
Þ SI = Rs. 312.42 =
100 10000 ´ 8.13 ´ 1
= = SI , SI = Rs. 813
31242 100
Þ R= = 12.3%
Now, the interest levied on different parts must
2540
Now using alligation method, we can determine be combined to get resultant interest of Rs. 813.
ratio of sum invested in both parts. Þ 8% of 2000 + 7.5% of 4000 + 8.5% of 1400
+ y% of (10000 – 7400). = 813
12% 12.5% (where y% = Rate on remainder money)
y
Þ 8 × 20 + 75 × 4 + 8.5 × 14 + × 2600 = 813
12.3% 100
Þ y = 9%
14. (d); The difference in amount of money will determine
0.2% 0.3%
compound interest of one year
Þ 2:3 Þ Hence, Rs. [2977.54 – 2809] = Rs. 168.54
2 This is the CI levied on Rs. 2809, in one year.
Hence, sum lent at 12% p.a. = ´ 2540 = Rs. 1016
5 é R ù
Hence, CI = P ê1 + - 1ú
10. (a); Let the first part be x. Then the second part be ë 100 û
(38800 – x)
é R ù 16854
x ´ 0.72 ´ 1 Þ 168.54 = 2809 ê ú Þ 2809 = R Þ R = 6%
2 5 ë 100 û
Now, =
( 38800 - x) ´ 0.05 ´ 2 4 The amount at CI for 2 years is:
1.44x = 19400 – 0.5x Þ x = 10000 2
é 6 ù é 106 106 ù
The second part = 38800 – 10000 = Rs. 28800 Þ 2809 = P ê1 + Þ 2809 = P ê ´
11. (d); Let the principal (sum of money) be Rs. 100 ë ú
100 û ë 100 100 úû
Þ Amount after 2 years = Rs. 144 Þ P = Rs. 2500
15. (a); The SI obtained after 12 years would be:
éæ R ö ù
2
æ R ö
2
144
Þ 100 êç 1 + ÷ ú = 144 Þ ç 1 + ÷ = 9000 ´ 8 ´ 2 9000 ´ 9.5 ´ 4
êëè 100 ø úû è 100 ø 100 Þ + +
100 100
R 12 9000 ´ 11 ´ 2 9000 ´ 12 ´ 4
Þ 1+ = ; R = 20%. +
100 10 100 100
Now, we have to make amount from Rs. 200 to Þ 90 [16 + 38 + 22 + 48] = Rs. 11160
Rs. 400. So, SI would be Rs. 200. Þ Final amount = P + SI
200 ´ 20 ´ T = Rs. (9000 + 11160) = Rs. 20160
Þ 200 = Þ T = 5 years
100
éæ R ö
2
ù
12. (a); No. of days between 1st January and 8th August 16. (b); Here, Amount = P êç 1 + ÷ ú
(take non-leap year) êëè 100 ø úû
(30 + 28 + 31 + 30 + 31 + 30 + 31 + 8) = 219 days
éæ R ö ù
2
æ 219 ö 3 Þ 1323 = 1200 êç 1 + ÷ ú
Here, 219 constitutes ç or th of the year
è 365 ÷ø 5 ëê
è 100 ø ûú

194 @BEST300MCQ For More Study Material


Visit: studyiq.com
2
1323 æ R ö éæ 21 ö 3 ù é 1261 ù
Þ = ç1+ ÷ Þ 504.40 = P êç ÷ - 1ú Þ 504.40 = P ê
1200 è 100 ø
ëê
è 20 ø
ûú ë 8000 úû
On solving above equation we get Rate of interest
Þ P(Principal) = Rs. 3200
R = 5%
Hence, amount of principal = Rs. 1600 in 3 years ´ ´ 3200 ´ 5 ´ 3
would become: Þ SI = = = Rs. 480
100 100
éæ 5 ö ù
3
é 21 21 21 ù 19. (a); Here, P = Rs. 1500, R = 25% p.a.
Þ A = 1600 êç 1 + ÷ ú = 1600 ê ´ ´
ëêè 100 ø ûú ë 20 20 20 úû n = ? (No. of years)
Now according to question:
1600 1
= [ 21]3 = ´ 9261 = Rs. 1852.50 éæ R ö ù
n
æ 25 ö
n
8000 5 Þ P êç 1 + ÷ ú > 2P Þ çè 1 + ÷ >2
è 100 ø ûú 100 ø
17. (c); According to the question ëê
Þ CI – SI = 183 ...(i)
n
æ 5ö
P ´ 5 ´ 3 3P Þ ç ÷ >2
Here, SI = = ...(ii) è 4ø
100 20
By hit and trial method, we get Þ n = 4 years
éæ 5 ö
3
ù éæ 21 ö 3 ù 20. (c); Here, SI for 3 years = Rs. 240
Þ CI = P êç 1 + ÷ - 1ú = P êç ÷ - 1ú
êë è 100 ø úû è
êë 20 ø úû
240
SI for 1 year = Rs.
é 1261 ù 3
=Pê ...(iii)
ë 8000 úû 240
SI for 2 years = Rs. ´ 2 = Rs. 160
Putting values of (ii) and (iii) in (i): 3
1261 3P 1261P 1200P P´R´2
Þ P- = 183 Þ - = 183 Þ 160 = Þ PR = 8000
8000 20 8000 8000 100
Þ 61P = 183 × 8000 Þ P = Rs. 24000
2
æ R ö
éæ 3
ù Hence, ( 164 - 160) = P ç
18. (d); Here, CI = P êç 1 +
R ö
- 1ú è 100 ÷ø
÷
ëêè 100 ø ûú
2
é 8000 ù P ´ 8000 ´ 8000
Þ 4 =Pê ú Þ 4=
éæ ù P 2 ´ ( 10000)
3
5 ö ë P ´ 100 û
Þ 504.40 = P êç 1 + ÷ - 1ú
êëè 100 ø úû Þ P = Rs. 1600

3. (a);
7 PRT
1. (a); SI = P–P=
6 100 8% 10%
1 PRT 1 R´3
P= Þ =
6 100 6 100 9.2%
100 5
R= =5 % 0.8% 1.2%
18 9
2. (d); SI4 - SI3 = 42
A 8 0.8 2
PR ´ 4 PR ´ 3 = =
- = 42 A 10 1.2 3
100 100
2
PR P´5 A8 = ´ 10000 = 4000
= 42 Þ = 42 5
100 100
P = 42 × 20 = 840 A10= 10000 – 4000 = 6000

195 @BEST300MCQ For More Study Material


Visit: studyiq.com
4. (b); Let the interest rate = number of years = x 12. (a); P1+P2 = 12000
PRT 4 P´x´x 9
SI = , P= P2 ´ ´ 16
100 9 100 P1 ´ 3 ´ 12 2
=
100 100
400 20
= x2 Þ x =
9 3 P1 9´8 P 2
= Þ 1 =
P2 3 ´ 12 P2 1
2
rate of interest = 6 % 13. (d); Let rate of interest is r and time is t
3
x´r´ t
500 ´ 12 ´ 4 P ´ 10 ´ 4 y=
5. (c); + = 480 100
100 100
y r´ t
2P 2P = ...(i)
240 + = 480 Þ = 240 Þ P = Rs. 600 x 100
5 5
y´ r´ t
æ 31 ö z=
6. (c); ç 8 - ÷%of P = 61.5 100
è 4 ø
z r´ t
1 = ...(ii)
y 100
4 ´ P = 61.5 Þ P = Rs. 24600
100 from equation (i) and (ii)
y z
15 15 25 8 = Þ y2 = xz
P´ ´ P´ ´ x y
7. (c); 2 12 - 2 12 = 32.5
100 100 14. (d); If amount received is equal, then the ratio of dis-
tributed amount
3P P 9P - 8P
- = 32.5 Þ = 32.5 1 1 1
32 12 96 M1 : M2 : M3 = 100 + r t : 100 + r t : 100 + r t
1 1 2 2 3 3
P = 32.5 × 96 = 3120
15600 ´ 10 ´ 2 1 1 1
= : :
8. (c); Interest after two year = =Rs.3120 100 + 5 ´ 2 100 + 5 ´ 3 100 + 5 ´ 4
100
Principal becomes after two year = 15600 + 1 1 1 1 1 1
= : : = : :
3120 = 18720 110 115 120 22 23 24
Interest at the end of fourth year = = 276 : 264 : 253
18720 ´ 2 ´ 10 M1 = 2760 Þ M2 = 2640 Þ M3 = 2530
= Rs. 3744
100 Amount received by A = M1 = Rs. 2760
15. (c); Annual interest = 1 × 365 = 365
PRT
9. (c); SI = P ´ 5´ 1
100 365 =
100
25 P = 365 × 20 Þ P = Rs. 7300
P´ ´T
P= 4 Þ T = 16 years 16. (b); After 6 year boy will attain age of 18
100
Interest earned from the trust in 6 year
P ´ (r + 1) ´ 2 P ´ r ´ 2 100000 ´ 6 ´ 6
10. (a); - = 24 = = 36000
100 100 100
P´2 Money paid to boy for his pocket money in 6 year
= 24 Þ P = Rs. 1200 = 2500 × 6 = 15000
100
Expenses of trust in 6 year = 500 × 6 = 3000
-20% -20% total amount paid to boy after 6 year
11. (b); 10000 8000 6400
1st year 2nd year = 100000 + 36000 - 15000 – 3000 = Rs. 118000

196 @BEST300MCQ For More Study Material


Visit: studyiq.com
17. (c); No. of days = 2
æ r ö
18 + 28 + 31 + 30 + 31 + 8 22. (b); 9680 = P ç 1 + ÷ ... (i)
è 100 ø
­ ­ ­ ­ ­ ­
3
Jan Feb Mar Apr May June æ r ö
10648 = P ç 1 + ÷
= 146 è 100 ø
2
146 æ r ö æ r ö
12000 ´ ´ 15 10648 = P ç 1 + ÷ ç1+ ÷
Simple interest for 146 days = 365 è 100 ø è 100 ø
100
æ r ö
= 720 10648 = 9680 ç 1 + ÷ ... [from (i)]
Amount to be paid by Tusar to the money lender è 100 ø
= 12000 + 720 = Rs. 12720 10648 r 968 r
-1= , = , r = 10%
4 9680 100 9680 100
18. (b); r = = 2%
2 9
æ 20 ö 3
4 ö
4 ç 4 ÷
æ
ç ÷ 23. (c); Amount = 16000 ç 1 + 100 ÷
C.I. = 10000 ç 1 + 2 ÷ –10000 çç ÷÷
çç 100 ÷ è ø
÷
è ø 3
æ 105 ö 9261
= 16000 ç ÷ = 16000 ´ 8000
éæ 102 ö4 ù è 100 ø
= 10000 êç ÷ - 1ú Amount = 18522
êëè 100 ø úû
Compound Interest = 18522 – 16000 = Rs. 2522
= 10000 × 0.08243216 = Rs. 824.3216
æ 4 öæ 3 ö
2520 - 2400 24. (a); Amount = 2000 ç 1 + ÷ç1 + ÷
´ 100 è 100 ø è 100 ø
19. (a); rate of interest =
2400
103 103
= 20 ´ 104 ´ = 104 ´ = 2142.4
120 100 5
= ´ 100 = 5%
2400 CI = 2142.4 – 2000 = Rs. 142.4
20. (c); 25. (b); Let the amount of money borrowed is P.
Amount paid by money lender as a interest at the
r 270 P´ 4´1 P
yea = 135
1st 2 end of the year = =
Simple Interest 100 25
2nd 270 Interest received by money lender at the end of
yea = 135
r 2 the year on the amount which he lend

r é æ 6 ö
2
ù
yea 135 ê ç ú
1st ÷
ê P ç 1 + 2 ÷ - P ú é æ 103 ö ù
2
Compound Interest =ê ç = ê P ç ÷ - Pú
2nd 100 ÷ ú è 100 ø
yea 135 + x% of 135 ê çè ÷
ø ú êë úû
r ë û
= 282.15 - 135

x% of 135 = 282.15 – 270 é æ 103 ö2 ù P


his gain = êP ç ÷ - Pú -
x 1215 ëê è 100 ø úû 25
´ 135 = 12.15 Þ x= = 9%
100 135 2
æ 103 ö 26
104.50 = P ç - P
è 100 ÷ø
t
æ 10 ö 25
21. (a); 1331 = 1000 ç 1 + ÷
è 100 ø
é 10609 - 10400 ù
3 t 104.50 = P ê ú , 1045000 = P × 209
æ 11 ö æ 11 ö ë 10000 û
ç ÷ =ç ÷
è 10 ø è 10 ø 1045000
t = 3 years P= Þ P = Rs. 5000
209

197 @BEST300MCQ For More Study Material


Visit: studyiq.com
26. (c); If a certain amount becomes N times in T years 32. (b); If x is the amount of each instalment and amount
then same amount becomes Nn times in T × n borrowed is P
years
According to question é 100 100 2 100 t ù
P = xê + + ...... ú
ë (100 + r) (100 + r) (100 + r)t û
2
The sum of money on compound interest becomes
3 times in 3 years then same amount becomes 9 =
32 times in 3 × 2 = 6 years é 100 100 2 ù
210 = x ê + 2 ú
PR 2 ë 100 + 10 (100 + 10) û
27. (a); Difference between CI and SI =
100 2 100 é 100 ù
210 = x ´ 1+
P´5 2
15000 110 êë 110 úû
1.5 = 2 Þ P= Þ P = Rs. 600
100 25 10x é 210 ù
210 = , x = 121
28. (c); 10% interest rate compounded half yearly for 11 êë 110 úû
1 33. (d); SI for 3 years ® 540
1 year
2
540
= 5% interest rate compounded annually for 3 SI for 1 year ®
3
year
Difference between CI and SI for three year Let the Rate of interest is r.

PR2 300 + R P ´ 52 + { + 180 + r% of180


180
= ´ , 244 = ´ CI for 2 year = 1st year 1442443
1002 100 1002 100 2nd year

244 ´ 400 ´ 100 r


P= , P = Rs. 32000 376.20 = 360 + ´ 180
305 100
29. (b); Let the amount is P r
4 376.20 – 360 = ´ 180
æ r ö 100
7000 = P ç 1 + ÷ ...(i)
è 100 ø r
16.20 = ´ 180
æ r ö
8
100
10000 = P ç 1 + ÷
è 100 ø 1620
= r Þ r = 9%
4 4 180
æ r ö æ r ö
10000 = P ç 1 + ÷ ç1 + ÷
è 100 ø è 100 ø P ´ 9´ 3
SI for 3 year =
4 100
æ r ö
10000 = 7000 ç 1 + ÷ P ´ 9´ 3
è 100 ø
540 = Þ P = 2000
4 100
æ r ö 10
ç1+ ÷ = ...(ii)
æ r ö
t
è 100 ø 7
34. (a); A = P ç 1 + ÷
from equation (i) and (ii) è 100 ø
10 t = number of half – year
7000 = P × Þ P = Rs. 4900 P = principal amount
7
t
æ 4 öæ 8 ö æ 5 ö
30. (b); Amount = 250 ç 1 + ÷ç1+ ÷ ç ÷ 68921 æ 2.5 ö
t
è 100 ø è 100 ø 68921 = 64000 ç 1 + 2 ÷ , = ç1+ ÷
100 ÷ 64000 è 100 ø
104 108 çç ÷
250 ´ ´ = 280.80 è ø
100 100
31. (b); Value of Bike after deprication for two years 3
æ 41 ö æ 102.5 ö
t 3
æ 41 ö æ 41 ö
t

2 2 ç ÷ =ç ÷ , ç ÷ =ç ÷
æ r ö æ 4 ö è 40 ø è 100 ø è 40 ø è 40 ø
= Pç1- ÷ = 62500 ´ ç 1 - ÷
è 100 ø è 100 ø
1
96 96 t = 3 half – years = 1 years
= 62500 ´ ´ = 57600 2
100 100
198 @BEST300MCQ For More Study Material
Visit: studyiq.com
35. (a); borrowed amount = 2
æ r ö
39. (c); Amount = P ç 1 + ÷
é 100 100 2 100t ù è 100 ø
xê + + ...... ú
êë 100 + r ( 100 + r ) ( 100 + r ) úû
2 t
2
æ r ö
x = amount paid in each installment 1348.32 = 1200 ç 1 + ÷
è 100 ø
r = rate of interest
t = time 1348.32 æ r ö
2

= ç1+ ÷
é 100 ( 100) 2 ù 1200 è 100 ø
borrowed amount = 17640 ê + 2 ú
êë 100 + 5 ( 100 + 5) úû 2
134832 æ r ö
= ç1+ ÷
120000 è 100 ø
100 é 100 ù
= 17640 ´ 1+
105 ë 105 úû
ê
2
44944 æ r ö
= ç1+ ÷
100 205 40000 è 100 ø
= 17640 ´´
105 105
2 2
borrowed amount = Rs. 32800 æ 212 ö æ r ö
çè ÷ = çè 1 + ÷
36. (a); If a certain amount becomes N times in T years 200 ø 100 ø
then same amount becomes Nn times in T × n
years 212 æ r ö
= ç1+ ÷
According to question, 200 è 100 ø
Sum of money becomes, two times in 5 years then
same amount becomes 8 = 23 times in 5 × 3 = 15 12 r
years = Þ r = 6%
200 100
PR 2 t
37. (c); Difference between CI and SI for two years = æ r ö
100 2 40. (a); Amount = P ç 1 + ÷
è 100 ø
P ´ 52 63 ´ 100 ´ 100
63 = Þ P= 9
100 2 5´ 5 t = number of quarters = =3
Þ P = Rs. 25200 3

æ r1 ö æ r ö æ 20 ö
3

38. (b); Amount = P ç 1 + ÷ ç 1+ 2 ÷


ç ÷
è 100 ø è 100 ø Amount = 12000 ç 1 + 4 ÷
çç 100 ÷
÷
æ 4 öæ 5 ö è ø
Amount = 25000 ç 1 + ÷ç 1 + ÷
è 100 øè 100 ø
3

104 105 æ 105 ö


´ Amount = 12000 ç ÷
= 25000× è 100 ø
100 100
Amount = Rs. 13891.50
26 21
= 25000 ´ ´ = Rs. 27300 CI = 13891.50 – 12000 = Rs. 1891.50
25 20

199 @BEST300MCQ For More Study Material


Visit: studyiq.com
Chapter
Average
9
Average: Average is defined as the sum of different data (terms) divided by total number of terms
Sum of given terms(S)
Average =
Total number of terms(N)
Example: Find the average of given terms 2, 3, 4, 5, 6
Sol. Total number of terms = 5
2+3+4+5+6
Average = =4
5
Some Basic Formulae
(n + 1)
1. Average of ‘n’ natural number =
2
2. Average of ‘n’ even numbers = (n + 1)
3. Average of ‘n’ odd numbers = n
First number + Last number
4. Average of ‘n’ consecutive natural numbers =
2
(n + 1)(2n + 1)
5. Average of sum of squares of first ‘n’ natural numbers =
6
Some Important Points:
1. If the value of each number is increased by the same value ‘a’, then the average of all numbers will also increase
by ‘a’.
2. If the value of each number is decreased by the same value ‘a’, then the average of all numbers will also
decrease by ‘a’.
3. If the value of each number is multiplied by the same value ‘a’, then the average of all numbers will also get
multiplied by ‘a’.
4. If the value of each numbers is divided by the same value ‘a’, then the average of all numbers will also get
divided by ‘a’.
Some useful Formulae
xa + yb
1. If the average of ‘x’ numbers is a and that of ‘y’ numbers is b, then the average of (x + y) numbers = x + y

Example: The average of 10 numbers is 15 and that of 15 numbers is 20. Find the average of all 25 numbers?
10 ´ 15 + 15 ´ 20 150 + 300 450
Sol. Average = = = = 18
10 + 15 25 25
2. If the average of ‘n’ quantities is equal to ‘x’ when a quantity is removed the average becomes ‘y’. Then the
value of the removed quantity is = [n (x – y) + y]
Example: The average age of 24 men and 1 woman is equal to 35 years. If 1 woman left, the average becomes 34
years. Find the age of woman who left the class?
Sol: Age of women = [25(35 – 34) + 34] = 59 years
3. If the average of marks obtained by ‘n’ students in an exam is ‘M’. If the average marks of passed students in ’P’
n(P - M)
and that of failed students is ‘F’. Then the number of students who failed in exam is .
P -F

200 @BEST300MCQ For More Study Material


Visit: studyiq.com
Total students(Passedaverage - Total average)
No. of failed students =
Passed average - Failed average
Example: The average marks obtained by 125 students in an exam is 29. If the average marks of passed
students is 36 and that of failed students is 11. Find the numbers of failed students?
125(36 - 29) 125 ´ 7
Sol. No of failed students = = = 35
36 - 11 25
4. If a batsman in his nth innings makes a score of ‘s’ and thereby increased his average by ‘t’ then the average
after ‘n’ innings is ‘s – t(n – 1)]
Example: A bastman in his 44th innings makes a score of 86 and thereby increases his average by 1. Find the
average after 44 innings?
Sol.Average after 44th innings = (86 – 1(44 – 1)) = 86 – 43 = 43

1. A bastman scores 34, 36, 38, 40, 42, in his five innings 5. The average of the first and the second of three
respectively. Find the average runs in the five innings? numbers is 10 more than the average of the second
and the third of these numbers. What is the difference
34 + 36 + 38 + 40 + 42 190
Sol. Average runs = = = 38. between the first and third of these three numbers?
5 5
Sol. Average of the first and the second numbers
2. The average of 10 quantities is 12. The average of 6 of
them is 8. What is the average of remaining four First + Second
numbers. =
2
10 ´ 12 – 6 ´ 8 Average of the second and the third numbers
Sol. The required average = = 18.
10 – 6 Second + Third
3. Average of marks obtained by 120 candidates in a =
2
certain examination is 35. If the average marks of According to the question,
passed candidates is 39 and that of the failed
candidates is 15, what is number of candidates who First + second Second + Third
– = 10
passed the examination? 2 2
Sol. Let the number of passed candidates be x. \ First – Third = 20
Then, total marks = 120 × 35 = 39x + (120 – x) × 15 6. The average height of 50 students of a class is 152 cm.
or, 4200 = 39x + 1800 – 15x If 10 among them whose average height is 148 cm left
or, 24x = 2400 the class and 10 new boys of average height 150 cm
\ x =100 are included in the class, then what will be the new
\ number of passed candidates = 100. average height of the students of the class?
4. The average salary of the entire staff in a office is Rs. Sol. The total height of 50 students = 152 × 50 = 7600cm
120 per month. The average salary of officers is Rs. Total decrease in the height when 10 students left the
460 and that of non-officers is Rs 110. If the number of class
officers is 15, then find the number of non-officers in
= 148 × 10 = 1480 cm
the office.
Total increase in the height after 10 new students came
Sol. Let the required number of non-officers = x
Then, 110x + 460 × 15 = 120 (15 + x) = 10 × 150 = 1500 cm
or, 120x – 110x = 460 × 15 – 120 × 15 = 1 (460 – 120) Now, Total height of 50 students = 7600 – 1480 +
or, 10x = 15 × 340; \ x = 15 × 34 = 510 1500 = 7620cm
No. of non-officers = No. of officers × 7620
New average = = 152.4 cm
æ Av. salary of officers – Mean average ö 50
çè Mean average – Av. Salary of non - officers ÷ø 7. The sum of the present ages of Hari and Mohan is
double the difference of their present ages. Four years
æ 460 – 120 ÷ö ago this ratio was one and half times. Find the ratio of
= 15 ççç ÷ = 510. their ages after 12 years.
çè 120 – 110 ÷÷ø

201 @BEST300MCQ For More Study Material


Visit: studyiq.com
Sol. Let the present ages of Hari and Mohan be their father is twice the sum of their ages. At the time
respectively x years and y years then of Mahesh’s birth their father’s age was 8 times of the
x + y = 2(x – y), x = 3y ... (i) age of Geeta. Find their present ages?
Sol. Let the present age of Mahesh is x years then the
3 present age of Geeta will be x + 5 and the present age
From the question, (x – 4 + y – 4) = {(x – 4) – (y – 4)}
2 of their father will be 2(x + x + 5) = 4x + 10 years.
or, 2x + 2y – 16 = 3x – 3y The age of Geeta at the time of Mahesh’s birth will be
or, 5y – x = 16 ... (ii) x + 5 – x = 5 years while the age of the father will be 4x
Solving eqn (i) and (ii), + 10 – x = 3x + 10 years
x = 24 and y = 8 From the question,
Hari’s age after 12 years = 24 + 12 = 36 years 3x + 10 = 5 × 8
Mohan’s age after 12 years = 8 + 12 = 20 years or, 3x = 40 – 10 = 30 or, x = 10 years
The ratio of their ages = 36 : 20 = 9 : 5 and x + 5 = 15 and 4x + 10 = 50 years
8 The present age of Geeta is 5 year more than the So, the ages of Mahesh, Geeta and their father are 10,
present age of his brother Mahesh. The present age of 15 and 50 years respectively.

Ques ti ons 5. The average of the price per kg. of rice at 10 different
places was Rs. 4.85. After a week, the price per kg.
was increased by 20 paise at 3 places and decreased
1. A motorist covered the first 20 km of his journey in 30 by 10 paise at one place. The new average of price per
minutes and the remaining 29 km in 40 minutes. His kg is:
average speed in km/hr is: (a) Rs. 4.88 (b) Rs. 4.86
(c) Rs. 4.90 (d) Rs. 5.35
3
(a) 41 (b) 42 6. The average weight of a group of 20 boys was
4
calculated to be 89.4 kg and it was later discovered
that one weight was misread as 78 kg instead of the
1
(c) 49 (d) 57 correct one of 87 kg. The correct average weight is :
6
(a) 88.95 kg (b) 89.25 kg
2. A cyclist goes from a place to another and returns by (c) 89.55 kg (d) 89.85 kg
the same route. He pedals his way uniformly with 7. A class has 20 boys and 30 girls. The average age of
speed u while going and with speed v while boys is 11 years and that of girls is 12 years. What is
returning. The average speed of his journey is : the average age of the whole class
u+v u-v (a) 11.6 years (b) 12 years
(a) (b) (c) 10 years (d) 11.2 years
2 2
8. Find the average of the following set of scores 253,
1 1 124, 255, 534, 836, 375, 101, 443, 760
(c) (d) (a) 427 (b) 413
1 æ 1 1ö 1 æ 1 1ö
ç + ÷ ç - ÷ (c) 141 (d) 409
2 è u vø 2 è u vø
9. What will be the average of even numbers between 11
3. The mean of 19 observations is 4. If one more to 63?
observation of 24 is added to the data, the new mean (a) 37.5 (b) 47
will be: (c) 42 (d) 37
(a) 4 (b) 5 10. Find average of all prime numbers between 60 and
(c) 6 (d) 7 90.
(a) 72 (b) 74.7
4. A student bought 4 books for Rs. 120 from one book
shop and 6 books for Rs. 150 from another. The average (c) 74 (d) 73.6
price (in rupees), he paid per book was : 11. Average age of 5 boys is 16 yr, of which that of 4 boys
is 16 yr 3 months. The age of the 5th boy is
(a) 27 (b) 27.50
(a) 15 yr (b) 15 yr 6 months
(c) 135 (d) 138 (c) 15 yr 4 months (d) 15 yr 2 months

202 @BEST300MCQ For More Study Material


Visit: studyiq.com
12. The average age of 30 girls is 13 yr. The average of 22. If the average of m numbers is n2 and that of n numbers
first 18 girls is 15 yr. Find out the average age of is m2, then the average of (m + n) numbers is -
remaining 12 girls?
m
(a) 12 yr (b) 10 yr (a) (b) m + n
n
(c) 16 yr (d) 10.5 yr
(c) mn (d) m – n
13. The average of 13 results is 60. If the average of first 7
results is 59 and that of last 7 results is 61, what will 23. A, B, C, D & E are five consecutive even numbers.
be the seventh result? Average of A and E is 46. What is the largest number?
(a) 52 (b) 42
(a) 90 (b) 50
(c) 50 (d) 48
(c) 75 (d) 60
24. Of the three numbers whose average is 60, the first is
14. The average of nine numbers is 50. The average of the
one-fourth of the sum of the others. The first number
first five numbers is 54 and that of the last three
is
numbers is 52. Then, the sixth number is?
(a) 30 (b) 36
(a) 34 (b) 24
(c) 42 (d) 45
(c) 44 (d) 30
25. The average salary of 15 persons is Rs. 5,500. If the
15. The average age of 30 women decreases by 3 months salary of one person is added, the average increase to
if a new person Neha is included in place of a 25 yr Rs. 5,700. What is the salary of this one person?
old woman. Calculate the age of Neha?
(a) Rs. 8,700 (b) Rs. 9,500
(a) 17.5 yr (b) 20 yr
(c) Rs. 7,800 (d) Rs. 8,500
(c) 30 yr (d) 22 yr
26. The average weight of 24 students is 35 kgs. If the
16. The average salary per head of all workers of an weight of teacher is added the average increased by
institution is Rs. 60. The average salary per head of 400 gms. What is the weight of the teacher?
12 officers is Rs. 400. The average salary per head of
(a) 45 (b) 36
the rest is Rs. 56. Then, the total number of workers in
the institution is? (c) 54 (d) 63
(a) 1030 (b) 1032 27. The average of runs of a cricket player of 10 innings
was 32. How many runs must he score in his next
(c) 1062 (d) 1060
innings so as to increase his average of runs by 4?
17. A cricketer scored some runs in his continuous 9 (a) 76 (b) 70
innings. He scored 100 runs in his 10th innings and
(c) 4 (d) 2
this increased his average by 8 runs. What was the
average of his runs at the end of 10th innings? 28. 10 kg of rice priced at Rs. 12 per kg is mixed with 6 kg
of rice priced at Rs. 16 per kg. Find the average price
(a) 20 (b) 24
of the whole mixture?
(c) 28 (d) 32
(a) Rs. 14 per kg (b) Rs. 14.50 per kg
18. The average age of three boys is 15 yr. If the ratio of
(c) Rs. 13 per kg (d) Rs. 13.50 per kg
their ages is 3 : 5 : 7, what is the age of the oldest boy?
29. The average of 5 quantities is 6. The average of three
(a) 7 yr (b) 14 yr
of them is 4. What is the average of remaining two
(c) 20 yr (d) 21 yr quantities?
19. Nine friends have a dinner in a hotel. Eight of them (a) 2 (b) 5
spent Rs. 12 each on their meals and the ninth spent (c) 9 (d) 7
Rs. 16 more than the average expenditure of all the
30. The average of five numbers is 6.9. If one of the
nine. Find out the total money spent by them?
numbers is deleted, the average of the remaining
(a) Rs. 126 (b) Rs. 135 numbers is 4.4. What is the value of the number
(c) Rs. 111 (d) Rs. 141 deleted?
20. The average of certain first consecutive even number (a) 12.5 (b) 17.2
is 101. Find their sum? (c) 16.9 (d) 16.8
(a) 25,000 (b) 33,600 31. The average age of Seema, Sapna, Asha, Kavita and
(c) 10100 (d) 24,960 Atrye is 40 years. The average age of Seema and Sapna
21. The average of certain first consecutive natural is 35 years and the average age of Asha and Kavita is
numbers is 20.5. Find their sum. 42 years. The age of Atrye is?
(a) 282 (b) 450 (a) 48 (b) 47
(c) 518 (d) 820 (c) 46 (d) 32

203 @BEST300MCQ For More Study Material


Visit: studyiq.com
32. The average of 11 numbers is 10.9. If the average of 37. The average expenditure of a man for the first five
the first six is 10.5 and the average of last six is 11.4, months is Rs. 120 and for the next seven months is
find the middle (sixth) number. Rs. 130. His monthly average income if he saves Rs.
(a) 11 (b) 10.6 290 in that year, is
(c) 11.5 (d) 10.45 (a) Rs. 160 (b) Rs. 170
33. The average temperature from Monday to Thursday (c) Rs. 150 (d) Rs. 140
is 48°C and from Tuesday to Friday, the average 38. The average salary of 20 workers in an office is Rs.
temperature is 52°C. If the temperature on Monday is 1900 per month. If the manager’s salary is added, the
42°C, what was it on Friday? average becomes Rs. 2000 per month. The manager’s
(a) 46°C (b) 58°C annual salary (in Rs) is:
(c) 50°C (d) 45°C (a) Rs. 24000 (b) Rs. 25200
34. Find the average of first 25 multiples of 5. (c) Rs. 45600 (d) Rs. 48000
(a) 65 (b) 60 39. A club has 15 men and 25 women. The average age of
(c) 75 (d) 80 men is 10 years and the average age of women is 12
35. What was the average daily expenditure of a man in year. What is the average age of the whole club?
1999 who spent Rs. 76535 in the first half year and (a) 11.40 years (b) 11.60 years
Rs. 88165 in the last?
(c) 11.25 years (d) 12.25 years
(a) Rs. 450 (b) Rs. 451.32
40. The average of 12 results is 15, and the average of the
(c) Rs. 451.23 (d) Rs. 450.23
first two is 14. What is the average of the rest?
36. The average of 6 numbers is 8. What is the 7th number
so that average becomes 10? (a) 15.2 (b) 13.2
(a) 22 (b) 18 (c) 15 (d) 16
(c) 21 (d) 20

1. An aeroplane flies along the four sides of a square 5. The average weight of 21 boys was recorded as 64 kg.
field at speeds of 200, 400, 600 and 800 km/hr. The If the weight of the ‘teacher was added, the average
average speed of the plane in the flight around the increased by one kg. What was the teacher’s weight?
field in km/hr is: (a) 86 kg (b) 64 kg
(a) 384 (b) 400 (c) 72 kg (d) 98 kg
(c) 500 (d) 284 6. Roshan bought 5 pants at Rs. 25 each, 10 shirts at Rs.
2. The average speed of a cyclist who covers first, second 50 each and 15 ties at Rs. 35 each. Find the average
and third km at 20, 16 and 12 km/hr respectively (in price of all the articles.
km/hr) is: (a) Rs. 38.33 (b) Rs. 45
(a) 16.24 (b) 16 (c) Rs. 60 (d) Rs. 45.33
(c) 15.66 (d) 15.32 7. The mean weight of 150 students in a class is 60 kg.
3. In a class of 100 students, the mean marks obtained The mean weight of boys is 70 kg and that of girls is
in a subject is 30 and in another class of 50 students 55 kg. What is the number of boys in the class?
the mean marks obtained in the same subject is 60. (a) 50 (b) 60
The mean marks obtained by the students of two (c) 75 (d) 100
classes taken together is : 8. The average of the test scores of a class of ‘m’ students
(a) 40 (b) 45 is 70 and that of ‘n’ students is 91. When the scores of
(c) 48 (d) 50 both the classes are combined, the average is 80. What
4. The mean weight of 34 students of a school is 42 kg. If is n/m?
the weight of the teacher be included, the mean rises 11 13
by 400 g. Find the weight (in kg) of the teacher? (a) (b)
10 10
(a) 66 (b) 56
10 10
(c) 55 (d) 57 (c) (d)
13 11

204 @BEST300MCQ For More Study Material


Visit: studyiq.com
9. The average marks of 120 students are 35. If average 2 1
of passed students was 39 and failed students was (a) 15 (b) 18
3 2
15, then Find the number of students who have
passed? 1 1
(c) 21 (d) 16
3 2
(a) 80 (b) 100
17. The average age of a man and his two twin born on
(c) 120 (d) 140
the same day is 30 yr. The ratio of the ages of father
10. A person covers 9 km with a speed of 3 km/h, 25 km
and one son is 5 : 2, what is the father’s age?
with a speed of 5 km/h and 30 km with a speed of 10
km/h. Find out the average speed of person? (a) 50 yr (b) 30 yr
(c) 45 yr (d) 20 yr
9 5
(a) 5 km / h (b) 11 km / h 18. If the average of the ages of Rakesh and Mohan is 15,
11 9
average of the ages of Mohan and Ramesh is 12 and
5 5 the average of the ages of Rakesh and Ramesh is 13,
(c) 9 km /h (d) 5 km / h
11 11 then the age of Mohan, is?
11. Five years ago, the average age of P and Q was 15 yr. (a) 16 (b) 13
Now, average age of P, Q and R is 20 yr. What whould (c) 14 (d) 12
be the age of R after 10 yr?
19. The average of three numbers is 28. If first number is
(a) 35 yr (b) 40 yr half of the second and the 3rd number is twice the
(c) 30 yr (d) 50 yr second, tthen the third number is?
12. The average salary of the entire staff in an office is Rs. (a) 48 (b) 36
500 per day. The average salary of officers is Rs. 750
(c) 24 (d) 18
and that of non-officers is Rs. 250. If the number of
officers is 15, then find the number of non-officers in 20. The average temperature from Monday to Wednesday
the office? is 37°C while the average temperature from tuesday
(a) 25 (b) 10 to thursday is 34°C. The temperature of thursday is
4
times to that of Monday. Find the temperature of
(c) 15 (d) 20 5
thursday?
13. Three year ago, the average age of a family of five
(a) 36°C (b) 33°C
members was 27 yr. The present average age is also
27 yr after the inclusion of a child into the family. (c) 37°C (d) 34°C
Find the age of the child? 21. There are 6 consecutive odd numbers in increasing
(a) 16 yr (b) 12 yr order. The difference between the average of squares
(c) 24 yr (d) 20 yr of the last four numbers and the first four numbers is
14. The average age of 5 sisters is 20 yr. If the age of the 64. Then the average of all the six numbers is?
youngest sister be 4 yr, what was the average age of (a) 8 (b) 7
the group of sisters at the birth of the youngest sister? (c) 6 (d) 5
(a) 25 yr (b) 15 yr 22. The average of marks obtained by 150 candidates in a
(c) 18 yr (d) 20 yr certain examination is 50. If the average marks of
15. The average weight of 4 men A, B, C and D, is 67 kg. passed candidates is 55 and that of the failed
The 5th man E is included and the average weight candidates is 25, what is the number of candidates
decreases by 2 kg. A is replaced by F. The weight of F who passed the examination?
is 4 kg more than E. Average weight decreases because (a) 125 (b) 120
of the replacement of A and now the average weight
(c) 140 (d) 160
is 64 kg. Find the weight of A?
23. The average age of 24 boys and their teacher is 15
(a) 78 kg (b) 66 kg
years. When the teacher’s age is excluded, the average
(c) 75 kg (d) 58 kg
age decreases by 1 year. The age of the teacher is?
16. Average of a, b and c is 11; average of c, d and e is 17;
(a) 38 years (b) 39 years
average of e and f is 22 and average of e and c is 17.
Find out the average of a, b, c, d, e and f? (c) 40 years (d) 41 years

205 @BEST300MCQ For More Study Material


Visit: studyiq.com
24. The average age of 8 members of a group is 40 years. (a) 10 (b) 51
The age of 1 member is 55 yrs. After his retirement a (c) 56 (d) 86
new member comes in his place of age 39. What is the
32. The average age of x and y is 20 years. If z were to
effect of his retirement on the average age of group?
replace x, the average would be 19 and if z were to
(a) 2 (b) 0
replace y, the average would be 21. Find the ages of x,
(c) 3 (d) 5 y and z.
25. The average age of 11 players of a cricket team is
(a) 22, 18, 20 (b) 18, 22, 20
increased by 2 months when two of them aged 18
years and 20 years are replaced by two new players. (c) 20, 18, 20 (d) 18, 20, 22
The average age of the new players is? 33. The average age of a family of 6 members is 22 years.
(a) 19 years 1 month (b) 19 years 6 months If the age of the youngest member be 7 years, then
(c) 19 years 11 months (d) 19 years 5 months what was the average age of the family at the birth of
the youngest member?
26. In a school, the average age of students is 6 years, and (a) 18 (b) 15
the average age of 12 teachers is 40 years. If the average (c) 13 (d) 26
age of the combined group of all the teachers and
34. The average age of 10 men is increased by 2 years
students is 7 years, then the number of students is?
when one of them of 25 years age, is replaced by new
(a) 396 (b) 400 man. What is the age of new man?
(c) 408 (d) 416
(a) 50 (b) 45
27. A batsman has a certain average in 11 innings. In the
(c) 35 (d) 25
12th innings, he scores 90 runs and his average
decreases by 5. After the 12th innings what is his 35. The mean of the marks obtained by 100 students is
average? 60. If the marks obtained by one of the students was
(a) 145 (b) 130 incorrectly calculated as 75, whereas the actual marks
obtained by him were 65, what is the correct mean of
(c) 135 (d) 140
the marks obtained by the students?
28. A cricketer gives 12.4 runs per wicket. He gives 26
runs and takes 5 wicket in a match after which his (a) 59.9 (b) 59
average becomes 12 per wicket. How many wickets (c) 58.5 (d) 59.4
had been taken till the last match? 36. A, B, C and D are four consecutive even numbers
(a) 85 (b) 90 respectively and their average is 65. What is the
(c) 65 (d) 72 product of A and D?
29. The average of a batsman after 25 innings was 56 (a) 3968 (b) 4092
runs per innings. If after the 26th inning his average
(c) 4216 (d) 4352
increased by 2 runs, then find his score in the 26th
inning? 37. The average weight of 36 students is 50 kg. If was
(a) 100 (b) 102 found later that the weight of one of the student was
misread as 73 kg. whereas his actual weight was 37
(c) 108 (d) 98
kg. Find the correct average.
30. The average age of a class of 30 students and a teacher
reduce by 0.5 years if we exclude the teacher. If the (a) 47 (b) 49.5
initial average is 14 years, find the age of the class (c) 49 (d) 51
teacher? 38. The average score of Sachin after 25 innings is 46
(a) 29 years (b) 30 years runs per innings. If after the 26th innings, his average
(c) 31 years (d) 32 years runs increased by 2 runs, then what is his score in the
31. The average marks of a group of 20 students on a test 26th innings?
is reduced by 4 when the topper who scored 90 marks (a) 48 (b) 84
is replaced by a new students. Find the marks of the
(c) 98 (d) 89
new student.

206 @BEST300MCQ For More Study Material


Visit: studyiq.com
39. 3 years ago the average age of a family of 5 members 40. The average marks of a student in four subjects is 75. If
was 17 years, with the birth of a new baby, the average the student obtained 80 marks in the 5th subject then
age of six members remains the same even today. Find the new average is?
the age of the new baby. (a) 80 (b) 76
(a) 2 years (b) 3 years (c) 92 (d) 95
(c) 5 years (d) can't be determined

1. The average weight of A, B and C is 84 kgs. If D joins, 7. Ashish has earned an average of 4200 dollars for the
the average weight now is 80 kgs. If another person E first eleven months of the year. If he justifies his staying
who is 3 kgs heavier than D replaces A then the on in the US on the basis of his ability to earn at least
average weight of B, C, D and E becomes 79 kgs. what 5000 dollars per month for the entire year, how much
is the weight of A? should he earn (in dollars) in the last month to achieve
(a) 70 kg (b) 75 kg his required average for the whole year?
(c) 65 kg (d) 55 kg (a) 14,600 (b) 5,800
2. A batsman has an average of 30 runs in his 42 inning. (c) 12,800 (d) 13800
The difference between his maximum and minimum
8. The sum of three numbers is 98. If the ratio between
score is 100. If these two innings are removed his
the first and second is 2 : 3 and that between the
average for 40 innings comes down to 28. What is his
second and the third is 5 : 8, then the second number
minimum score?
is
(a) 20 (b) 30
(c) 45 (d) 40 (a) 30 (b) 20
3. Five years ago the average age of husband and wife (c) 58 (d) 48
was 23 years. Today the average age of husband, wife 9. The average salary per head of all the workers in a
and child is 20 yrs. How old is child? company is Rs. 95. The average salary of 15 officers is
(a) 3 yrs (b) 4 yrs Rs. 525 and the average salary per head of the rest is
(c) 12 yrs (d) 2 yrs Rs. 85. Find the total number of workers in the
4. The average age of a husband and wife, who were workshop.
married 7 years ago, was 25 years at the time of their (a) 660 (b) 580
marriage, Now, the average age of the family, (c) 650 (d) 460
including husband, wife and a child, born during the
10. A batsman makes a score of 270 runs in the 87th
interval, is 22 years. What is the present age of the
inning and thus increases his average by a certain
child?
number of runs that is a whole number. Find the
(a) 4 yrs. (b) 2 yrs. possible values of the new average.
(c) 3 yrs. (d) 5 yrs.
(a) 98 (b) 184
5. The average age of mother, father and son was 42 yrs.
(c) 12 (d) All of these
at the time of the marriage of the son. After one year
an infant was born in the family and after 6 yrs of 11. 19 persons went to a hotel for a combined dinner party.
marriage the average age of the family is 36 years. 13 of them spent Rs. 79 each on their dinner and the
What was the age of the bride at the time of marriage. rest spent Rs. 4 more than the average expenditure of
all the 19. What was the total money spent by them.
(a) 25 yrs. (b) 23 yrs.
(c) 22 yrs. (d) 24 yrs. (a) 1628.4 (b) 1536.07
6. There were 35 students in a hostel. If the number of (c) 1492 (d) 1496
students increases by 7, the expenses of the mess 12. The average age of a group of persons going for a
increase by Rs. 42 per day while the average picnic is 16.75 years. 20 new persons with an average
expenditure per head decreases by Rs. 1. Find the age of 13.25 years join the group on the spot due to
original expenditure of the mess? which the average of the group becomes 15 years.
(a) Rs. 420 (b) Rs. 520 Find the number of persons initially going for the
(c) Rs. 450 (d) Rs. 550 picnic?

207 @BEST300MCQ For More Study Material


Visit: studyiq.com
(a) 24 (b) 20 Rs. 1500 in July. The average expenditure for the
(c) 15 (d) 18 month of February to July is:
13. The average salary of workers in AMS careers is Rs. (a) Rs. 4250 (b) Rs. 4520
2,000, the average salary of faculty being Rs. 4,000 (c) Rs. 4060 (d) Rs. 4200
and the management trainees being Rs. 1,250. The 19. The average price of 80 computers in an electronic
total number of workers could be shop is Rs. 30,000. If the highest and lowest price
(a) 450 (b) 300 computers are sold out then the average price of the
(c) 110 (d) 500 remaining 78 computers is Rs. 29,500. The cost of the
14. 10 years ago the average age of all the 25 teachers of highest price computer is Rs. 80,000. The cost of lowest
the Girls college was 45 years. 4 years ago, the price computer is:
principal has retired from her post at the age of 60 (a) Rs. 19,000 (b) Rs. 20,000
year. So after one year a new principal whose age (c) Rs. 29,000 (d) can’t be determined
was 54 years recruited from outside. The present
20. Eleven years earlier the average age of a family of 4
average age of all the teachers is, if principal is also
members was 28 years. Now the age of the same family
considered as a teacher:
with six members is yet the same, even when 2
18 17 children were born in this period. If they belong to the
(a) 54 years (b) 55 years
same parents and the age of the first child at the time
25 25
of the birth of the younger child was same as there
1 2 were total family members just after the birth of the
(c) 49 years (d) 49 years
2 3 youngest member of this family, then the present age
15. The average weight of all the 11 players of Indian of the youngest member of the family is :
cricket team is 50 kg. If the average of first six lightest (a) 3 years (b) 5 years
weight players is 49 kg and that of the six heaviest
(c) 6 years (d) 4 years
players is 52 kg. The average weight of the player
which lies in the sixth position in the list of players 21. Mr. Patel walked 6 km to reach the station from his
when all the 11 players are arranged in the order of house, then he boarded a train whose average speed
increasing or decreasing weights: was 60 km/hr and thus he reached his destination.
In this way he took total of 3 hours. If the average
(a) 56 kg (b) 52 kg
speed of the entire journey was 32 km/hr then the
(c) 51 kg (d) 50 kg average speed of walking is :
16. The average age of Sachin and Ganguli is 35 years. If
(a) 3 km/hr (b) 4.5 km/hr
Kaif replaces Sachin, the average age becomes 32 years
and if Kaif replaces Ganguli, then the average age (c) 4 km/hr (d) 5 km/hr
becomes 38 years. If the average age of Dhoni and 22. In a set of prime and composite numbers, the composite
Irfan be half of the average age of Sachin, Ganguli numbers are twice the number of prime number and
and Kaif, then the average age of all the five people is: the average of all the numbers of the set is 9. If the
(a) 28 years (b) 32 years number of prime numbers and composite numbers
are exchanged then the average of the set of numbers
(c) 25 years (d) 26 years
is increased by 2. If during the exchange of the
17. The average age of a family of 6 members 4 year ago numbers the average of the prime numbers and
was 25 years. Meanwhile a child was born in this composite numbers individually remained constant,
family and still the average age of the whole family is then the ratio of the average of composite numbers to
same today. The present age of the child is: the average of prime numbers (initially) was:
1 7
(a) 2 years (b) 1 years 13
2 (a) (b)
13 7
(c) 1 year (d) data insufficient
9 7
18. Amitabh’s average expenditure for the January to June (c) (d)
11 11
is Rs. 4200 and he spends Rs. 1200 in January and

208 @BEST300MCQ For More Study Material


Visit: studyiq.com
23. The average marks of Sameer decreased by 1, when 25. There are 6 consecutive odd numbers in increasing
he replaced the subject in which he has scored 40 order. The difference between the average of the
marks by the other two subjects in which he has just squares of the first 4 numbers and the last four
scored 23 and 25 marks respectively. later he has also numbers is 64. If the sum of the squares of the first
included 57 marks of Computer Science, then the and the last element (i.e., odd numbers) is 178, then
average marks increased by 2. How many subjects the average of all the six numbers is:
were there initially? (a) 7 (b) 8
(a) 6 (b) 12 (c) 9 (d) 10
(c) 15 (d) can’t be determined 26. The average age of board of directors of a company,
24. The average age of Donald, his wife and their two having 10 directors was 48 years. Coincidentally
children is 23 years. His wife is just 4 year younger when a director aged 53 resigned from the board of
than Donald himself and his wife was 24 years old directors, another director died on the same day. So a
when his daughter was born. He was 32 years old new director joined the board of directors aged 34.
when his son was born. Th average age of Donald Next year in the same month the average age of all the
and his daughter is : 9 directors was found to be 46 years. The age of the
(a) 25 years (b) 22.5 years late (i.e., dead) director at the time of his death was :
(c) 26 years (d) can’t be determined (a) 56 years (b) 53 years
(c) 57 years (d) 61 years

1. In a cricket team, the average age of 11 players and 5. The average runs scored by a cricketer in 42 innings,
the coach is 18 yr. If the age of the coach is not is 30. The difference between his maximum and
considered, then the average decreases by 1 yr. Find minimum scores in an innings is 100. If these two
out the age of the coach? innings are not taken into consideration, then the
(a) 29 yr (b) 21 yr average score of remaining 40 innings is 28. Calculate
(c) 25 yr (d) 30 yr the maximum runs scored by him in an innings?
2. The average salary of the entire staff in an office is Rs (a) 125 (b) 120
200 per day. The average salary of officers is Rs 550 (c) 110 (d) 100
and that of non-officers is Rs 120. If the number of
officers is 16, then find the numbers of non-officers 6. A class is divided into two sections A and B. Passing
in the office? average of 20 students of section A is 80% and passing
average of 30 students of section B is 70%. What is
(a) 70 (b) 30
the passing average of both the sections?
(c) 50 (d) 40
3. A person covers 20 km distance with a speed of 5 (a) 72% (b) 74%
km/h, then he covers the next 15 km with a speed of (c) 75% (d) 77%
3 km/h and the last 10 km is covered by him with a
7. The arithmetic mean of the scores of a group of
speed of 2 km/h. Find out his average speed for the
students in a test was 52. The brightest 20% of them
whole journey.
secured a mean score of 80 and the dullest 25%, a
3 2 mean score of 31. The mean score of remaining 55%
(a) 3 km /h (b) 3 km /h is
14 14
(a) 54.6% (b) 45%
3 3
(c) 1 km /h (d) 1 km / h (c) 50% (d) 51.4%
10 12
4. Mr. Sharma travels by car and covers 25% of his 8. A person bought some oranges worth Rs 36 from
journey with a speed of 10 km/h, 45% of his journey each of the five markets at Rs 1, Rs 1.50, Rs 1.80, Rs 2
with a speed of 5 km/h and remaining 30% of his and Rs 2.25 per orange, respectively. What is the
journey with a speed of 15 km/h. What will be the average price of an orange?
average speed of Mr. Sharma for the whole journey? (a) Rs 1.91 (b) Rs 2.00
(a) 5.40 km/h (b) 7.40 km/h (c) Rs 1.58 (d) Rs 1.80
(c) 2.40 km/h (d) 8.40 km/h

209 @BEST300MCQ For More Study Material


Visit: studyiq.com
9. The average weight of the students in four sections 18. The average height of 50 students of a class is 152
A, B, C and D is 60 kg. The average weight of the cm. If 10 among them whose average height is 148
students of A, B, C and D individually are 45 kg, 50 cm left the class and 10 new boys of average height
kg, 72 kg and 80 kg, respectively. If the average weight 150 cm are included in the class, then what will be
of the students of sections A and B together is 48 kg the new average height of the students of the class?
and that of B and C together is 60 kg, what is the ratio (a) 162.4 cm (b) 112.5 cm
of the number of students in sections A and D? (c) 152.4 cm (d) 122.6 cm
(a) 12 : 7 (b) 4 : 3 19. The average age of 5 boys is 12 years. The average
(c) 3 : 2 (d) 8 : 5 age of other 3 boys is 16 years. What will be the
10. The average age of students of a class is 15.8 yr. The average of all 8 boys?
average age of boys in the class is 16.4 yr and that of
1 1
the girls is 15.4 yr. Find out the ratio of the number of (a) 13 years (b) 12 years
boys to the number of girls in the class. 2 2
(a) 3 : 1 (b) 5 : 2 (c) 14 years (d) 13 years
(c) 2 : 3 (d) 3 : 7 20. The average income of 40 persons is Rs. 4200 and the
11. What will be the average of first five prime numbers? average income of other 35 persons is Rs. 4,000. What
is the average income of the entire group (in Rupees)?
(a) 5.6 (b) 7.1
(c) 7.6 (d) 4.6 1
(a) 4100 (b) 4106
12. A cricket player scored 53, 52,11, 0, 5 not out, 104 not 3
out and 35 runs in 7 innings of a series. What is the
average of run scored by player in that series? 2 1
(c) 4106 (d) 4108
(a) 62 (b) 52 3 3
(c) 70 (d) 80 21. In a T.V. factory an average of 60 pieces of T.V. is
13. A joint family consists of 5 males, 4 females and some produced in first 25 days of any month. Due to the
children. The average consumption per person per illness of few workers the average production per
month is 1.5 kg. If the average consumption for male, day lower downs to 58 for whole month. What is the
female and children is respectively 1 kg, 1.25 kg and average production of last 5 days?
2 kg then what is the number of children in the family? 2
(a) 9 (b) 5 (a) 59 (b) 60
3
(c) 7 (d) 8
(c) 48 (d) 53
14. The average temperature of Monday, Tuesday and
22. The average annual income (in Rupees) of some
Wednesday was 32°C. The average temperature of
agriculture labours is S and the average annual
Tuesday, Wednesday and Thursday was 34°C. If the
income of some other labours is T. The number of
temperature of Monday be 29°C. Find the temperature
agriculture labours is 11 times more than that of other
on Thursday?
labours. Then what will be the average monthly
(a) 37° (b) 26° income (in Rupees) of all the labours?
(c) 25° (d) 35°
15. The average of 40 observations was 28. It was later S + 11T S+T
(a) (b)
found that in two observations, 42 was taken instead 12 2
of 24 and 12 instead of 62. What is the correct
11S + T 11S + T
average? (c) (d)
(a) 28.5 (b) 28.8 12 144
(c) 26.8 (d) 23.5 23. The average consumption of rice per person per
month in a family of 8 adults and some kids is 10.8
16. The average of 15 numbers is 27 in which the average
kg, where the average consumption per person for
of first 7 is 25 and the average of last seven is 31. Find
adult is 15 kg and for kids is 6 kg. What is the number
the 8th number?
of kids in the family?
(a) 15 (b) 11
(a) 8 (b) 6
(c) 13 (d) 14
(c) 7 (d) 9
17. The average age of 9 boys is 15 years and when the
24. The average salary of all the workers in a workshop
teacher’s age is included the average age becomes
in Rs. 8000. Among them the average salary of 7
17. What is the age of the teacher?
technicians is Rs. 12000 and the average salary of
(a) 35 years (b) 16 years rest labours is Rs. 6000. What is the number of all the
(c) 40 years (d) 15 years workers in the workshop?

210 @BEST300MCQ For More Study Material


Visit: studyiq.com
(a) 14 (b) 21 (a) 296 (b) 320
(c) 23 (d) 22 (c) 324 (d) 285
25. The average marks achieved by 120 aspirants in any 33. There were 35 students in hotel. If the number of
exam is 35. The average marks achieved by passed students increases by 7 then the expense of fooding
student is 39 and the average marks achieved by failed increases by Rs. 42 per day while average
students is 15. What is the number of passed student expenditure of the boys decreases by Rs. 1. What was
in the exam ? the initial expenditure on fooding per day ?
(a) 100 (b) 120 (a) Rs. 400 (b) Rs. 442
(c) 150 (d) 140 (c) Rs. 432 (d) Rs. 420
26. The average wages of 500 workers is Rs. 200. But it 34. The average of first 5 natural numbers is m. If next 3
was later found that for two workers the wages was natural numbers are included, then the average of
wrongly put as Rs. 180 and Rs 20 instead of Rs. 80 these 8 number will how much more than m ?
and Rs. 220 respectively. Then what is the correct (a) 2 (b) 1
average wages?
(c) 1.4 (d) 1.5
(a) Rs. 200.10 (b) Rs. 200.20
35. If the average of m numbers is n2 and n numbers is m2
(c) Rs. 200.50 (d) Rs. 201.00
then the average of (m + n) numbers will be.
27. In any year the monthly average income of a person
is Rs. 3400. The average income of first eight month m
is Rs. 3160 and the average income of last 5 months (a) (b) m + n
n
is Rs. 4120. What is the income in 8th month of the
(c) mn (d) m - n
year?
36. There are 50 students in a class. One boy among them,
(a) Rs. 5080 (b) Rs. 6080
whose weight is 51 kg leaves the class and a new boy
(c) Rs. 25180 (d) Rs. 3880 admits in the class. Due to this the average weight of
28. A cricket player whose bowling average was 24.85
runs per wicket, takes 5 wicket for 52 runs in a match. 1
the class increases by kg. The weight of newly
Due to this his average decreases by 0.85. What will 2
be the number of wickets taken by him till the last admitted student is?
match? (a) 73 kg (b) 76 kg
(a) 64 (b) 72 (c) 74 kg (d) 75 kg
(c) 85 (d) 96 37. The average marks of a student in the first three test
29. A batsman has a certain average of runs for his 19 is ‘N’. If in the fourth test he gets 20 marks more than
innings. In 20th innings he makes 100 runs and his that of the average of previous tests then what is the
average increases by 2 runs then what is the average average of first four test?
of 20 innings? (a) N + 20 (b) N + 10
(a) 60 (b) 61 (c) N + 4 (d) N + 5
(c) 62 (d) 64 38. A student finds the average of ten number of two
30. In a city the rainfall recorded in inch for five digits. However he wrongly copies one number by
consecutive years was 20.54 inch, 33.10 inch, 11.62 exchaning its digits. Therefore he gets the answer 1.8
inch, 19.20 inch and 21.74 inch. What was the average less than the correct answer. What will be the
rainfall ? difference of the digit he wrongly put?
(a) 21.44 inch (b) 21.24 inch (a) 2 (b) 3
(c) 20.24 inch (d) 17.94 inch (c) 4 (d) 5
31. The difference between the average of three 39. The average of 8 continuous numbers is 6.5.What is
consecutive odd number and 2 consecutive even the average of the largest and the smallest number?
numbers thereafter is 4. What is the first even
(a) 4 (b) 6.5
number?
(c) 7.5 (d) 9
(a) 10 (b) 12
40. The average of 25 observations is recorded as 13. Later
(c) 14 (d) Can not be determined
it was found that one observation with value 73 was
32. The sum of 8 consecutive numbers of set -A is 376. entered wrongly as 48. The new average is
What is the sum of 5 consecutive numbers of another
set whose smallest number is 15 more than that of (a) 12.6 (b) 14
the average of set-A ? (c) 15 (d) 13.8

211 @BEST300MCQ For More Study Material


Visit: studyiq.com
S olut ions 8. (d); Average
253 + 124 + 255 + 534 + 836 + 375 + 101 + 443 + 760
1. (b); Total distance covered = (20 + 29) km = 49 km =
9
Total time taken = (30 + 40) min = 70 min
= 409
æ 49 ö 63 - 11 52
Average speed = ç ´ 60÷ = 42 km/h 9. (d); Total even no = = = 26
è 70 ø 2 2
2. (c); Let one way distance be 'x'. Sum of the even number between is 11 to 63
\ Total distance = 2x 26
= [2 × 12 + (26 – 1) × 2] = 13 [24 + 50]
æ x xö 2
Total time taken = çè + ÷ø
u v = 13 × 74 = 962
Average of even numbers between 11 to 63
2x 1
\ Average speed = = 962
x x 1 æ 1 1ö = = 37
+ ç + ÷ 26
u v 2 è u vø
10. (b); Average of all prime numbers between 60 to 90
19 ´ 4 + 24 ö 100 61 + 67 + 71 + 73 + 79 + 83 + 89 523
3. (b); New mean = æç ÷ø = =5 = = = 74.7
è 20 20 7 7
4. (a); Cost of 10 books= Rs (120 + 150) = Rs 270 11. (a); Total age of 5 boys = 16 × 5 = 80 years

æ 270 ö æ 3ö
Average cost = Rs. ç = Rs. 27 Total age of 4 boys = çè 16 ÷ø ´ 4 = 16.25 × 4 = 65
è 10 ÷ø 12
The age of the 5th boy = 80 – 65 = 15 years
5. (c); The sum of price of per kg Rice at 10 different
12. (b); Total age of 30 girls = 30 × 13 = 390 years
places = 4.85 × 10 = 48.5
Total age of 18 girls = 18 × 15 = 270 years
Increased price = 20 × 3 – 10 × 1 = 50 paise
\ Sum of the increased price of rice at 10 different 390 - 270
\ Average Age of remaining 12 girls =
places = 48.50 + 0.50 = 49.0 Rs. 12
49 120
New Average = = 4.90 Rs. = = 10 years.
10 12
6. (d); Total weight of 20 boys = 89.4 × 20 = 1788 kg 13. (d); Sum of the first 13 Results = 13 × 60 = 780
Increase in weight due to misreading Sum of the first 7 Results = 59 × 7 = 413
= 87 – 78 = 9 kg Sum of the last 7 Results = 7 × 61 = 427
Correct weight of 20 boys = 1788 + 9 = 1797 kg \ 7th Result = Sum of first 7 Results + sum of last
1797 7 Results – sum of 13 Results = 413 + 427 – 780
Correct average weight = = 89.85 kg = 840 – 780 = 60
20
14. (b); The sum of 9 numbers = 9× 50 = 450
Shortcut:
Sixth number = sum of 9 numbers – (Sum of first
Increase in weight = 9 kg. 5 numbers + sum of last 3 numbers)
9 = 450 – (54 × 5 + 52 × 3) = 450 – (270 + 156)
New average = 89.4 + = 89.85 kg
= 450 – 426 = 24
20
7. (a); Sum of the Age of 20 boys = 20 × 11 = 220 years 3
15. (a); Average decreases by 3 months or years.
Sum of the Age of 30 girls = 30 × 12 = 360 years 12
Total age of 20 boys and 30 girls = 220 + 360
3 15
= 580 years. Sum of the age decreased = 30 × =
12 2
580
Average age of the whole class = = 11.6 years = 7.5 years
50 \ Age of Neha = 25 – 7.5 = 17.5

212 @BEST300MCQ For More Study Material


Visit: studyiq.com
16. (b); Let the total number of workers = x 24. (b); Let the First no = x
According to the question Sum of the other numbers = 4x
60x = 12 × 400 + 56 (x – 12) According to the question
60x – 56x = 4800 – 672
x + 4x
4x = 4128 Þ x = 1032 = 60 Þ 5x = 180 Þ x = 36
3
17. (c); Let average of 9 innings = x, Total run = 9x
25. (a); Sum of the Salary of 15 person is = 5500 × 15
According to the question
= 82500
9x + 100 Sum of the Salary of 16 person is = 5700 × 16
=x+8
10 = 91200
9x + 100 = 10x + 80 Þ x = 20 Salary of this one person is = 91200 – 82500
Average after 10 innings = 20 + 8 = 28 = 8700
Shortcut: 26. (a); Total weight of 24 students = 24 × 35 = 840 kg
Average at the end of 10th innings Total weight after teacher is added = 25 × 35.4
= (100 – 9 × 8) = 28 runs. = 885 kg
18. (d); Total age of all the three boys = 3 × 15 = 45 years Weight of teacher = 885 – 840 = 45 kg
7 27. (a); Let the run scored in 11th innings = x
Given ratio of ages = 3 : 5 : 7 = × 45 \ According to the question
3+ 5+7
= 21 years. 10 × 32 + x = 11 × (32 + 4)
19. (a); Let average expenditure of 9 person = x 320 + x = 36 × 11 Þ x = 396 – 320 = 76
\ According to the question Shortcut:
12 × 8 + (x + 16) = 9x Þ 8x = 112 He must score 32 runs and some extra runs.
So, in the next innings he should score:
112
Þ x= = 14 32 + 11 × 4 = 76
8
28. (d); Total price of 10 kg Rice = 12 × 10 Þ 120
Total money spent = 9x = 9 × 14 = 126 Rs. Total price of 6 kg Rice = 16 × 6 Þ 96
20. (c); Let the total consecutive even number = x
\ Average of consecutive even no = x + 1 120 + 96 216
\ Average = = = 13.50 kg
x + 1 = 101 Þ x = 100 16 16
\ Sum = 101 × 100 = 10100 29. (c); Let the average of remaining two quantities is y.
21. (d); Let the total consecutive natural number = N According to the question
\ Average of consecutive natural number
3´4+ 2´y
= 6 Þ 12 + 2y = 30 Þ y = 9
N+1 N+1 5
= Þ = 20.5 Þ N + 1 = 41 Þ N = 40
2 2 Hence the average of remaining two quantities is
\ Sum = 40 × 20.5 = 820 9.
22. (c); Sum of m numbers = n2 × m 30. (c); Average of five numbers
Sum of n numbers = m2 × n
a+b+c+d+e
Þ = 6.9 Þ a + b + c+ d + e = 34.5
2
n ´m+m ´n 2 5
Average of (m + n) =
m+n Let the number is deleted be e. The average of
remaining 4 number is
mn(n + m)
= a +b+ c+ d
(n + m) = mn Þ = 4.4 Þ a + b + c + d = 17.6
4
23. (c); Let A is x
Value of deleted number = 34.5 – 17.6 = 16.9
\ Then E = x + 8
31. (c); Total age of Seema + Sapna + Asha + Kavita +
According to the question Atrye = 40 × 5 = 200
x+x+8 Seema+ Sapna = 35 × 2 = 70
= 46 Þ 2x = 84 Þ x = 42
2 Asha + Kavita = 42 × 2 = 84
Largest number is = 42 + 8 Þ 50 \ Age of Atrye = 200 – (70 + 84) = 46 years

213 @BEST300MCQ For More Study Material


Visit: studyiq.com
32. (c); Total first 6 number = 10.5 × 6 = 63 Total expenditure for next 7 months = 130× 7
Total of last 6 number = 11.4 × 6 = 68.4 = 910 Rs.
Total of all 11 number = 11 × 10.9 = 119.9 Total income = Total expenditure of the whole
\ Sixth number = 63 + 68.4 – 119.9 = 11.5 year + saving
Hence the middle number is = 11.5 = 600 + 910 + 290 = 1800 Rs
33. (b); Total temp of mon + tues + wed + thus 1800
= 4 × 48°C = 192°C Monthly average income = = 150 Rs.
12
Temp on mon = 42°C
38. (d); Total monthly salary of 20 workers in an office
temp on Tue + Wed + Thus = 192° – 42° = 150°C
= 1900 × 20 = Rs. 38000
Total temp for tue + wed + thus + Fri
Total monthly salary of 20 workers and manager
= 52° × 4 = 208°C
= 2000 × 21 = Rs. 42000
Tem on Friday = 208° – 150° = 58°C
Manager monthly salary = 42000 – 38000
34. (a); Sum of first 25 multiples of 5 = 5 (1 + 2 + 3....25)
= Rs. 4000
5 ´ n ( n + 1) 25 ´ 26 Manager annual salary = 12 × 4000 = Rs. 48000
=5× = 1625
2 2 39. (c); Total age of men = 15 × 10 = 150
Total age of women = 25 × 12 = 300
1625
Average = = 65 150 + 300
25 Average age of the whole club =
40
35. (c); Total expenditure of a man in year 1999
= 76535 + 88165 = 164700 450
= = 11.25 years.
164700 40
Average = = 451.23 40. (a); Let the average of rest results = x
365
36. (a); Let the 7th number be x. According to the question
According to the question 14 × 2 + x × 10 = 15 × 12 Þ 10x = 180 – 28
6 × 8 + x = 7 × 10 Þ x = 70 – 48 = 22 152
x= Þ x = 15.2
37. (c); Total expenditure for first 5 months = 120 × 5 10
= 600 Rs. Hence average of the rest is 15.2

1. (a); Let the side of square be a 3. (a); total marks obtained by 150 students.
and average speed of aeroplane x = 100 × 30 + 50 × 60 = 6000
a a a a 4a 6000
\ + + + = mean marks = = 40
200 400 600 800 x 150
4. (b); Let the weight of teacher = x kg
12a + 6a + 4a + 3a 4a 25 4
Þ = Þ = and average weight of students and teacher
2400 x 2400 x
= 42 + .40 = 42.40 kg
\ x = 384
\ 34 × 42 + x = 35 (42.4) Þ x = 1484 – 1428 = 56 kg
Hence average speed of a aeroplane = 384 km/hr
Hence weight of the teacher is 56 kg.
2. (d); Let the average speed of a cyclist = x
Shortcut:
Total distance = 3 km
The mean rises by 0.4 kg and total number of
1 1 1 3 12 + 15 + 20 3 persons = 35
\ + + = Þ =
20 16 12 x 240 x Weight of teacher = 42 + 0.4 × 35 = 56 kg
5. (a); Let the weight of teacher = x kg
47 3
= x = 15.32 km/hr According to the question
240 x
\ 21 × 64 + x = 22 × 65
Average speed of cyclist is 15.32 km/hr x = 1430 – 1344 Þ x = 86 kg

214 @BEST300MCQ For More Study Material


Visit: studyiq.com
Hence weight of the teacher is 86 kg. 13. (b); Total Present age of family of 5 members
Shortcut: = 27 × 5 + 3 × 5 = 135 + 15 = 150 years
Teacher's weight = 64 + 22 × 1 = 86 kg Total present age of a family including child
6. (a); Total price of all the articles.
= 27 × 6 = 162 years
= 5 × 25 + 10 × 50 + 15 × 35
age of a child = 162 – 150 = 12 years.
= 125 + 500 + 525 = 1150 Rs.
14. (d); Total age of 5 sisters = 100years.
1150
Average price = = 38.33 Rs. Total age of a group of sister at the birth of
30
youngest sister = 100 – 4 × 5 = 80 years.
7. (a); Let the no of boys in a class = x
\ x × 70 + (150 – x) 55 = 150 × 60 80
Average age of group of sisters = 20 years.
70x – 55x = 9000 – 8250 Þ 15x = 750 4
x = 50 Þ No. of boys = 50 15. (b); Sum of the weight of A, B, C and D = 67 × 4
8. (d); Total score of a class of m students = 70 × m = 268 kg
Total score of a class of n students = 91 × n
Sum of the weight of A, B, C, D, E = 65 × 5 = 325 kg
\ 70 m + 91n = (m + n) 80
70 m + 91n = 80m + 80 n weight of E = 325 – 268 = 57 kg
Weight of F = 57 + 4 = 61 kg
n 10
11n = 10 m Þ = Total weight of F, B, C, D, E = 64 × 5 = 320 kg
m 11
Total weight of B, C, D, E = 320 – 61 = 259 kg.
9. (b); Let the no of passed student = x
\ Weight of A = 325 – 259 = 66 kg.
According to the question
x × 39 + 15 (120 – x) = 120 × 35 Shortcut:
39x – 15x = 4200 – 1800 Weight of E = (67 – 2) – (4 × 2) = 57
24x = 2400 Þ x = 100 Weight of F = 61 kg
Hence no of passes student is 100. Average weight decreases by 1 kg on replacing
10. (a); Here a = 9 km, b= 25 km, c = 30 km A.
P= 3 km/hr Q = 5 km/hr R = 10 km/hr So, weight of A = 61 + 5 × 1 = 66 kg
\ Requried average speed
16. (a); Total of a + b + c = 33, c + d + e = 51
a+b+ c 9 + 25 + 30 e + f = 44, e + c = 34, d = 51 – 34 = 17
= =
a b c 9 25 30 \ a + b + c + d + e + f = 33 + 17 + 44 = 94
+ + + +
P Q R 3 5 10
94 2
Average of a, b, c, d, e, f = = 15
64 64 9 6 3
= = =5 km/hr
3 + 5 + 3 11 11 17. (a); Total age of a man and his twin born son
9 = 30 × 3 = 90 years
Hence average speed of a person = 5 km/h given the ratio of father and his twin son
11
11. (c); Total present age of Pand Q = 15 × 2 + 10 =5:2:2
= 40 years. 90
Total present age of P, Q and R Father's age = ´ 5 = 50 years
9
= 20 × 3 = 60 years.
P + Q + R = 60 18. (c); Total age of Rakesh and Mohan = 30 years
(Present age) R = 20 years. Mohan + Ramesh = 24 years
R age after 10 years = 20 + 10 = 30 years. Rakesh + Ramesh = 26 years
12. (c); Let the total no of non officer staff in an office = x
80
According to the question (Rakesh + Mohan + Ramesh) = = 40 years.
750 × 15 + 250x = 500 (x + 15) 2
500x – 250x = 11250 – 7500 age of Mohan = 40 – 26 = 14 years.
3750 19. (a); Let the second number = x
250x = 3750 Þ x = = 15
250 x x + 2x + 4x
+ x + 2x = 84 Þ = 84
Hence no of non-officer staff = 15 2 2

215 @BEST300MCQ For More Study Material


Visit: studyiq.com
84 ´ 2 25. (c); Sum of the age of 2 new players
7x = 84 × 2 Þ x = Þ x = 24
7 2
= 18 + 20 + 11 ×
Third number = 24 × 2 = 48 12
20. (a); Total temp of Mon + tues + wed = 111°C
tues + wed + thus = 34 × 3 = 102°C 22 478
= 38 + = years
Mon – Thus = 9°C 12 12

4 M average age of 2 new players


M– M = 9°C Þ = 9°C Þ M = 45 °C
5 5 478 478 22
= = = 19
4 12 ´ 2 24 24
Temp on thursday = 45 × = 36°C
5 = 19 years 11 months.
21. (a); let the number be (a – 5), (a – 3), (a – 1), (a + 1), (a 26. (a); Let the number of students = x
+ 3), (a + 5) According to the question
Then the average of all six consecutive odd
x × 6 + 12 × 40 = (x + 12) 7
numbers
6x + 480 = 7x + 84 Þ x = 396
( a - 5 ) + (a - 3 ) + ( a - 1) + ( a + 1) + ( a + 3 ) + ( a + 5 )
= =a 27. (a); Let the average of 11 innings = x
6
Total run of 11 innings = 11x
According to question,
After 12 innings
( a - 1) + ( a + 1 ) + ( a + 3) + ( a + 5 )
2 2 2 2
11x + 90 = (x – 5)12
4 12x – 11x = 90 + 60 Þ x = 150
( a - 5 )2 + ( a - 3 )2 + ( a - 1 )2 + ( a + 1 )2 Average after 12th innings = 150 – 5 = 145
- = 64
4
Shortcut:
a=8 Average after 12 innings = (90 + 12 × 5) – 5
22. (a); Let the number of candidates who passed = x = 145
According to question
28. (a); Let the no of wicket taken by the cricketer = x
55 × x + 25 (150 – x) = 150 × 50
According to the question
3750
55x – 25x = 7500 – 3750 = Þ x = 125 12.4 ´ x + 26
30 = 12 Þ 12.4x + 26 = 12x + 60
Hence no of candidate who passed the exam
x+5
= 125 34 340
23. (b); Total age of 24 boys and teacher = (24 +1) × 15 x= = = 85
.4 4
= 375 years
Total age of all boys excluding teacher 29. (c); Let his score in 26th inning = x
= 24 × (15 – 1) = 336 years. According to the question
age of teacher = 375 – 336 = 39 years 25 × 56 + x = 26 (56 + 2)
Shortcut: x = 1308 – 1400 Þ x = 108
Age of teacher = 15 + 24 × 1 = 39 years
Shortcut:
24. (a); Total age of 8 members = 8 × 40 = 320 years.
Total age after replacement of 1 member Score in the 26th innings = 56 + 2 – 26 = 108
= 320 – 55 + 39 = 304 years. 30. (a); Let the age of a class teacher = x
304 According to question
New average = = 38 years 30 × 13.5 + x = 14 × 31 Þ x = 434 – 405= 29 years
8
Hence average decreased by 2 years. Shortcut:
Shortcut: Age of the teacher = 14 + 0.5 × 30 = 29 years
Difference in ages = 55 – 39 = 16 years its effect 31. (a); Let the marks of New student = x
16 x = 90 – 20 × 4 Þ x = 80
= =2
8 Marks of New student = 10

216 @BEST300MCQ For More Study Material


Visit: studyiq.com
32. (a); Sum of Age of x and y = 40 years 4x = 248 Þ x = 62
y and z = 38 years, z and x = 42 years A × D = 62 × 68 = 4216
37. (c); Total correct weight of 36 students
40 + 38 + 42 120
x+y+z= = = 60 = 50 × 36 – 73 + 3 = 1800 – 36 = 1764 kg.
2 2
x = 60 – 38 = 22 years, y = 60 – 42 = 18 years 1764
Correct average = = 49 kg.
z = 60 – 40 = 20 years 36
33. (a); Total age of a family of 6 members = 22 × 6 Shortcut:
= 132 years æ 73 - 37 ö
Correct average = 50 - ç = 49 kg
Total age of the family at birth of youngest member è 36 ÷ø
= 132 – 7 × 6 = 132 – 42 = 90 years. 38. (c); Sachin score in 26th innings.
90 = 26 × (46 + 2) – 25 × 46 = 1248 – 1150 = 98
Average age = = 18 years.
5 Shortcut:
34. (b); Age of a new man = 25 + 10 × 2 = 45 years. Sachin's score in 26th innings = 46 + 2 × 26 = 98
35. (a); Total correct marks obtained by students 39. (a); total present age of 5 members = 17 × 5 + 3 × 5
= 100 × 60 – 75 + 65 = 5990 = 100
total present age including new baby = 17 × 6
5990
mean marks of 100 students = = 59.9 = 102
100
Age of new baby = 102 – 100 = 2 years.
Shortcut:
40. (b); Total sum of all the 5 subjects = 75 × 4 + 80
The actual marks are 10 less then the previous
marks. = 380

380
æ 10 ö Average = = 76
So, correct mean = 60 - ç = 59.9
è 100 ÷ø 5
Shortcut:
36. (c); Let the consecutives even numebr
= x, x + 2, x + 4, x + 6 æ 80 - 75 ö
New average = 75 + ç = 76 marks
\ x + x + 2 + x + 4 + x + 6 = 260 è 5 ÷ø

1. (b); Total weight of A + B + C = 84 × 3 = 252 kgs Sum of the present age of Husband, wife and child
A + B + C + D = 80 × 4 = 320 kgs. = 20 × 3 = 60
D = 68, E = 68 + 3 = 71 kgs Age of child = 60 – 56 = 4 years.
B + C + D + E= 79 × 4 = 316 kg 4. (b); Present age of Husband and wife = 25 × 2 + 7 × 2
B + C + D = 316 – 71 = 245 kg = 64 years
A = 320 – 245 = 75 kgs. Present age of Husband wife and child
2. (a); Total runs of 42 innings = 42 ×30 = 1260 = 22 × 3 = 66 years
Total run excluding min and max inning Age of a child is = 66 – 64 = 2 years.
= 40 × 28 = 1120 5. (a); Sum of the present age of Mother, Father and Son
max + min = 1260 – 1120= 140 ...(i) = 42 × 3 + 6 × 3 = 126 + 18 = 144 years.
max – min = 100 ...(ii) Sum of present age of the family = 36 × 5 = 180
On solving (i) & (ii) Present age of the bride = 180 – 144 – 5 = 31 years.
max = 120, min = 20 Age of the bride at the time of marriage
Hence, minimum score is 20. = 31 – 6 = 25 years.
3. (b); Sum of the present age of Husband and wife 6. (a); Let the average expenditure of a man = x Rs.
= 23 × 2 + 5 × 2 = 56 years \ According to the question

217 @BEST300MCQ For More Study Material


Visit: studyiq.com
\ 35 × x + 42 = 42 (x – 1) = 51 × 25 = 1275
\ 7x = 84 Þ x = 12 Rs. and the total age of remaining 24 teachers when
Original expenditure = 35 × 12 = 420 Rs. just principal has retired = 1275 – 60 = 1215 years
7. (d); Ashish total earnings of 11 months 1 year later (i.e. 3 years ago from present)
= 4200 × 11 = 46,200$ total age of 24 teacher (just before the recruitment
Total earnings to be needed for 12 months of new principal) = 1215 + (1 × 24) = 1239 years.
= 5000 × 12 = 60,000$ and the total age of 25 teacher including new
Last month earnings = 60,000 – 46,200 = 13,800$ principal just after the recuitment = 1239 + 54
= 1293 years.
Shortcut:
Thus the present age of all the 25 teachers
Required earning in the last month
= 1293 + 3 × 25 = 1368 years.
= 5000 + (5000 – 4200) × 11 = $ 13800
Hence the present average age of the 25 teacher.
8. (a); Let the three no is x, y, z
x : y Þ (2 : 3) × 5 Þ y : z = (5 : 8) × 3 1368 18
= = 54 years.
Þ x : y : z = 10 : 15 : 24 25 25

15 15. (a); Sum of six lightest weight players = 49 × 6


Second number y = ´ 98 = 30 = 294 kgs.
49
sum of six heaviest players weight = 52 × 6
9. (a); Let the total number of workers = x
= 312 kg
\ 15 × 525 + (x – 15) 85 = x × 95
\ weight of the player at 6th position
\ 15 × 525 – 15 × 85 = 10x
= 312 + 294 – 550 = 56 kg.
\ 7875 – 1275 = 10x Þ 10x = 6600 Þ x = 660
16. (a); Total age of S + G = 35 × 2 = 70 years
10. (d); Let his Avg be A K + G = 32 × 2 = 64 years
And increased average = A + x S + K = 38 × 2 = 76 years
(270 - A) 70 + 64 + 76 210
86A + 270 = 87 (A + x) Þ x = S+G+K= = = 105 years
87 2 2
if x be a whole number possible value of A
105
A = 9, 96, 183, 270 for x = 3, 2, 1, 0 Average of S + G + K = = 35 years
3
So new value of A = 12, 98, 184, 270
11. (b); Let the Average expenditure of the party is x 35
D+I= Þ Sum of age of D + I = 35
\ 13 × 79 + 6(x + 4) = 19 × x 2
1027 + 24 105 + 35
13x = 13 × 79 + 24 Þ x = = 80.84 Average of S, G, K, D, I = = 28 years
13 5
Total expenditure of the party = 80.84 × 19 17. (c); Total present age of a family of 6 members
= 1536.07 = 25 × 6 + 4 × 6 = 174
12. (b); Let the person initially going for picnic is x Total present age of a family including child = 25
\ 16.75 × x + 20 × 13.25 = 15 (x + 20) × 7 = 175 years.
1.75x = 15 × 20 – 20 × 13.25 Age of child is = 175 – 174 = 1 year.
1.75x = 35 Þ x = 20 18. (a); Total expenditure of Jan to June = 4200 × 6
13. (c); Let the total no of faculty = x = Rs. 25200
and management trainees = y Total expenditure from Feb to July
4000x + 1250y = (x + y)2000 = 25200 – 1200 + 1500 = Rs. 25500
8x = 3y Þ x : y = 3 : 8 25500
So the possible value could be = 110 Average = = Rs. 4250
6
14 (a); 10 years ago average age of 25 teachers = 45 years. 19. (a); The price of the costliest and cheapest computers
4 years ago (just before the retirement of principal) =(80 × 30,000) – (78 × 29,500) = 99,000
average age of 25 teachers = 45 + 6 = 51 Therefore the price of cheapest computer
and at the same time total age of 25 teachers = 99,000 – 80,000 = 19,000

218 @BEST300MCQ For More Study Material


Visit: studyiq.com
24. (a); Let Donald be denoted by H (Husband)
20. (a); No. of family His wife be denoted by W, His daughter be
Average Total denoted by D, His son be denoted by S
member
Eleven years The average age of 4 persons
4 28 112
Earlier H+W+D+ S
= = 23
Presently if 4 39 156 4
6 28 168 H + W + D + S = 92 Þ H = W + 4
H W D S
Since it is obvious that the youngest member (i.e. At the time when ® ( +4 )
daughter is born 28 ¬¾ 24 0 ´
child) was the 6th family member in the family.
¯
Therefore at the time of the birth of the youngest At the time when ( -4 )
child the elder child's age was 6 years. Son is born ® 32 ¬¾ 28 4 0
Now the sum of their ages So at the time of birth of his Son, total age of his
Let age of youngest member = x family = 64 years. (32 + 28 + 4 + 0 = 64) and
x + x + 6 = (168 – 156) Þ 2x + 6 = 12 Þ x = 3 presently the total age of his family = 92 years.
Then the present age of the youngest member of It means total increase in age of the whole family
the familly is 3 years. = 28 years.
21. (c); let the average of walking = x km/hr 28
Thus average increase in age = = 7 years.
Total distance = 32 × 3 = 96 4
6 90 6 3 It means the age of Donald = 39 years
\ + = 3, \ + = 3 and age of his daughter = 11 years
x 60 x 2
Therefore the average age of Donald and his
12 + 3x Daughter is 25 years.
= 3 Þ 3x = 12 Þ x = 4 km/hr.
2x 25. (b); let the number be (a – 5), (a – 3), (a – 1), (a + 1)
22. (a); Let the average of prime number be P and average (a + 3) (a + 5)
of composit number be C. Then this average of all six consecutive odd
numbers
Again the number of prime number be x,
then the number of composite number be 2x (a - 5) + (a - 3) + (a - 1) + (a + 1) + (a + 3) + (a + 5)
= =a
6
Px + 2Cx
= 9 Þ P + 2C = 27 Again the value of 'a' can be found by using the
3x last statement
2Px + Cx (a – 5)2 + (a + 5)2 = 178 Þ a2 = 64 Þ a = 8
= 11 Þ 2 P + C = 33 Average of all six number is 8.
3x
On adding eq. (i) and (ii) 26. (a); No of Average
we get P + C = 20 Total age
Directors Age
and on Subtracting eq. (i) from (ii) Just before
we get P – C = 6 death and 10 48 480
C 7 resignation
Therefore P = 13 and C = 7 Þ thus = = Just after
P 13 (480 - (53 + x)
death and 9
23. (c); Let the number of subjects be n and average marks + 34
resignation
be x, the total marks = nx
Again (n + 1) (x – 1) = (nx – 40) + (23 + 25) one year later 9 46 414
x–n=9 ...(i)
Further (n + 2) (x + 1) = (nx – 40) + (23 + 25) + 57 So one year later, after the incident
nx + 2x + n + 2 = nx + 65 total age = {480 – (53 + x) + 34} + 9 × 1 = 414
2x + n = 63 ...(ii) x = 56
Solving equation I and II we get where x is the age of the dead person at the time
n = 15 & x = 24 of his death.

219 @BEST300MCQ For More Study Material


Visit: studyiq.com
1. (a); Total age of 11 player and coach = 12 × 18 9. (b); Let no. of students in section A, B, C and D be w,
= 216 yr. x, y, z.
Total age of 11 player = 11 × 17 = 187 yr So, Average weight of students of section A and
Age of coach = 216 – 187 = 29 yrs. B = 48 kg
2. (a); Non-officers Officers 45w + 50x
= 48
120 550 w+x
15w = 10x ...(i)
200 Avg. weight of students of sections B and C =
60kg.
350 50x + 72y = 60 (x + y)
80
10x = 12y ...(ii)
Ratio = 35 : 8
Avg. weight of student A, B, C and D is 60 kg.
No. of officers 8x ® 16, x ® 2 45w + 50x + 72y + 80z = 60 (w + x + y + z)
So, no. of non-officers = 35 × 2 = 70 15w + 10x – 12y – 20z = 0
D1 + D2 + D 3 15w = 20z
3. (a); Required Avg. speed = T + T + T
1 2 3 w 4
=
20 + 15 + 10 45 3 z 3
= = =3 km / h
20 15 10 14 14
+ + 10. (c); Girls Boys
5 3 2
15.4 16.4
1
4. (b); Req. Avg. speed = P + P + P
1 2 3 15.8
1 9 3
P1 = , P2 = , P3 = 0.6 0.4
4 ´ 10 20 ´ 5 10 ´ 15

1 2
So, = = 7.40 km / h =
1 9 3 3
+ +
4 ´ 10 20 ´ 5 10 ´ 15
2 + 3 + 5 + 7 + 11
5. (b); Let min. score = x 11. (a); Required Average = = 5.6
Max. score = 100 + x 5
So, x + (x + 100) = 30 × 42 – 40 × 28 12. (b); When we calculating the Average of run in cricket
x = 20 matches, not out innings are not considered.
Thus, the max. score = x + 100 = 120 So, Total runs
20 ´ 80 + 30 ´ 70 = 53 + 52 + 11 + 0 + 5 + 104 + 35 = 260
6. (b); Total Average = = 74%
20 + 30 260
7. (d); Let the total no. of student be 100 Average of the player inserees = = 52 runs
5
Also, let the mean score = x
13. (c); Let the no. of children in the family is x.
According to the question
20 × 80 + 25 × 31 + (100 – 20 – 25) x = 100 × 52 3 5
x = 51.36 = 51.4% ( 5 + 4 + x) ´ = 5´1+ 4´ + x´ 2
2 4
8. (c); No. of orange bought by the person
36 36 36 36 36
( 9 + x) ´ 3 = 2 ( 5 + 5 + 2x)
= + + + + = 114 x=7
1 1.50 1.80 2 2.25
Total expenditure = 36 × 5 = 180 14. (d); Temp. on thursday = Temp. on Monday + Avg.
increase in the temp. of three days.
180
Avg. of each orange = = 1.58 = 29° + 3 × 2° = 35°
114

220 @BEST300MCQ For More Study Material


Visit: studyiq.com
28. (c); Let the player took x wikets at the average of 24.85
= 28 +
( 24 - 42 ) + ( 62 - 12 ) then 24.85x + 52 = 24 (x + 5)
15. (b); New Average
40 x = 80
32 total wickets = 80 + 5 = 85
= 28 +
= 28.8 29. (c); A.T.Q.
40
16. (c); Required number = 15 × 27 – 7 × 25 – 7 × 31 = 13 19x + 100
Þ = ( x + 2) , x = 60
17. (a); Sum of ages of 9 boys = 15 × 9 = 135 years 20
Sum of ages of 9 boys + 1 teacher = 17 × 10 So, Avg. runs in 20 innings = 60 + 2 = 62
= 170 years 30. (b); Avg. rainfall
The age of teacher = 170 – 135 = 35 years
20.54 + 33.10 + 11.62 + 19.20 + 21.74
18. (c); New Average =
5
æ Change of avg. of going ö = 21.24 inch
çè and coming student ÷ø × 10 x+5+x +7 x+ x+ 2+ x+ 4
= Avg. height + 31. (d); - =4
50 2 3
( 2x + 12 ) ( 3x + 6 )
( 150 - 148) ´ 10 So, - = 4, 4=4
= 152 + = 152.4 cm 2 3
50
This is correct for any no. so, cannot be
5 ´ 12 + 3 ´ 16 determined. Since x is cancelled out
19. (a); Average of all 8 boys =
8 376
32. (b); The average for set A = = 47
1 8
= 13years
2 The smallest no. of second set = 47 + 15 = 62
20. (c); Average of the entire group Required total = 62 + 63 + 64 + 65 + 66
= 64 × 5 = 320
40 ´ 4200 + 35 ´ 4000 2 33. (d); Let initial expenditure on food perday = x
= = 4106
75 3
x + 42 x
21. (c); Average production of last 5 days = -1
35 + 7 35
60 ´ 25 + 5 ´ x Þ 35x + 1470 = 42x – 1470, x = 420
Þ = 58 is x,
30 34. (d); The total of first 5 natural no. = 15
30 ´ 58 - 60 ´ 25 the total of first 8 natural no. = 36
So, x = = 48 36 15
5 Increase in Average = - = 1.5
22. (d); Let the no. of another labour = x 8 5
The no. of agriculture labour = 11x 35. (c); The total of m numbers = n×n2
Total annual income = 11x × S + x × T and the total of n numbers = n×m2
Required Avg. = (m + n) avg. of the numbers.
( 11S + T ) x 11S + T
Average annual income = = mn 2 + nm 2
12x 12 = = mn
( m + n)
11S + T 11S + T
Average monthly income = = 36. (b); Weight of new student = Original value + no. ×
12 ´ 12 144
increase
23. (c); 8 × 15 + x × 6 = (8 + x) × 10.8
x=7 1
= 51 + 50 ´ = 76 kg.
24. (b); A.T.Q. 2
7 × 12000 + x × 6000 = (7 + x) × 8000 3N + N + 20
x = 14 37. (d); Average marks of all 4 sets =
4
Total workers = 14 + 7 = 21 =N+5
25. (a); A.T.Q. 38. (a); The total difference of the total of the no.
x × 39 + (120 – x) × 15 = 120 × 35 Þ (10x + y) – (10y + x) = 1.8 × 10, x – y = 2
x = 100 39. (b); The Average of first and the last no. will be same
26. (b); Correct average wages as the average of second no. and the second last
80 + 220 - 180 - 20 number. so, Average is = 6.5
= 200 + = 200.20
500 73 - 48
40. (b); New Average is = 13 + = 14
27. (a); 8 × 3160 + 5 × 4120 – 3400 × 12 = 5080 25
221 @BEST300MCQ For More Study Material
Visit: studyiq.com
Chapter
Time and Work
10
Concept 1:
1
(1) If a person can complete a work in 'D' days, then the work done by him in 1 day is
D
Efficiency is inversely proportional to the time taken (T) when the work done is constant.
1
Ea
T
1
(2) If P is 'n' times more efficient than Q, than P will take time of the total time taken by Q to complete the same
n
amount of work.
Example: Ram can do a work in 40 days. Hari is 4 times more efficient than Ram. In how many days Hari can finish
the work?
Sol. Ram can do a work in 40 days
1
Hari can complete the work in = ´ 40 days = 10 days
4
Concept 2:
If M1 persons can do W1 work in D1 days working H1 hours and M2 persons can do W2 work in D2 days working H2
hours, then relation between them is
M1 D1 H1 M2 D 2 H2
=
W1 W2
Example: 24 men working 8 hours a day make a road in 15 days. In how many days 48 men working 6 hours a day
will make the three times long road?
M 1 D 1 H1 M 2 D 2 H 2 24 ´ 15 ´ 8 48 ´ D 2 ´ 6 24 ´ 15 ´ 8 ´ 3
Sol. = Þ = Þ D2 = = 30 days
W1 W2 1 3 48 ´ 6
Concept 3:
ab
If A does a work in 'a' days and B in 'b' days then both can complete the work in days.
ab
Example: A complete the work in 10 days and B in 15 days. In how many days A + B can complete the work?
10 ´ 15 150
Sol. (A + B) complete the work in = = 6 days
10 + 15 25
Concept 4:
If A and B can complete a work in x days and A alone can finish that work in y days, then number of days B takes to
xy
complete the work is = days
yx
Example: A and B can complete a work in 20 days and A alone can finish that work in 30 days. In how many days B
can complete the work?
20 ´ 30 600
Sol. B complete the work = = = 60 days
30 - 20 10
Concept 5:
xyz
A, B, C can do a work in x, y and z days respectively. They will finish the work in days
xy  yz  zx

222 @BEST300MCQ For More Study Material


Visit: studyiq.com
Example: A, B and C can do a work in 10, 12 & 15 days respectively. In how many days all of them together will finish
the work?
10  12  15 1800
Sol. (A + B + C) complete the work in = = 4 days
10  12  12  15  15  10 120  180  150
Concept 6:
If A and B can do a piece of work in x days, B and C can do the same work in y days and A and C can do it in z days,
2xyz
then, working together they can complete that work in days
xy  yz  zx
Example: A and B can complete a work in 20 days. B and C can complete the same work in 30 days. C and A can
complete the same work in 40 days. In how many days work together they can complete the work?

2  20  30  40 48000 48000 240 6


Sol. (A + B + C) complete the work in = = = = 18 days
20  30  30  40  20  40 600  1200  800 2600 13 13
Concept 7:
If A takes 'a' days more to complete a work than the time taken by (A+B) to do some work and B takes 'b' days more than
the time taken by (A+B) to do same work. Then (A + B) do the work in ab days .
Example: A takes 4 days more to complete a work than the time taken by (A + B) to do the same work and B takes 9
days more than the time taken by (A + B) to do the same work. In how many days A + B complete the work?

Sol. (A + B) complete the work in 4 ´ 9 = 36 = 6 days


Concept 8:
A can do a certain piece of work in d1 days and B in d2 days. Then, the ratio of wages of A and B are:
1 1
A's share : B's share = : = d 2 : d1
d1 d2
A, B and C can do a piece of work in d1, d2 and d3 days. Then the ratio of wages of A, B and C are
1 1 1
A's share : B's Share : C's share = : :
d1 d2 d3
Multiplying each equation by (d1d2d3 )
Then the ratio is A's share : B's share : C's share = d2d3 : d1d3 : d1d2
Concept 9:
If A, B and C can do a piece of work in x, y and z days respectively. The contract for the work is Rs. r and all of them work
together.
Then,
 ryz   rzx   rxy 
Share of A  Rs.   , Share of B  Rs. 
 
 , Share of C  Rs. 
 


 xy  yz  zx   xy  yz  zx   xy  yz  zx 
Example: A, B and C can do a work in 20 days, 25 days and 30 days respectively. They finished together that work
and gained Rs. 3700 as wage. Find the wages of A, B and C respectively

 3700  25 30  2775000 2775000


Sol. Share of A = Rs. 
  = Rs. = Rs. = Rs. 1500
 20  25  25 30  20  30  500  750  600 1850

 3700  20  30  2220000
Share of B = Rs. 
  = Rs. = Rs. 1200
 500  750  600  1850

 3700  20  25  1850000
Share of C = Rs. 
  = Rs. = Rs. 1000
 500  750  600  1850

223 @BEST300MCQ For More Study Material


Visit: studyiq.com
Concept 10:
A can do a piece of work in x days. With the help of B, A can do the same work in y days. If they get Rs. a for that work
Then,

æ ay ö æ a ( x - y) ö
Share of A = Rs. ç ÷ , And Share of B = Rs. ç
è xø è x ÷ø

Example: A can do a piece of work in 20 days. With the help of B, A can do the same work in 15 days. If A + B gets Rs.
1500 for the work, find the share of A and B respectively
æ 1500 ´ 15 ö æ 1500 ´ 5 ö
Sol. Share of A = Rs. ç ÷ = Rs. 1125, Share of B = Rs. çè ÷ = Rs. 375
è 20 ø 20 ø

1. A can do a piece of work in 8 hours and B can do the 2. Ram, Rohit and Rahul can do a work in 10 days, 12
same work in 12 hours. In how much time both can days and 15 days respectively. In how many days
finish the whole work working together? will they finish the work?

1 1
Sol. Work done by A in 1 hour = Sol. Work done by Ram in 1 day =
8 10

1 1
Work done by B in 1 hour = Work done by Rohit in 1 day =
12 12

1 1 5 1
Work done by (A + B) in 1 hour = + = Work done by Rahul in 1 day =
8 12 24 15
Work done by Ram, Rohit and Rahul in 1 day
24 4
Time taken to finish the work = = 4 hours
5 5 1 1 1 6+5+4 1
= + + = =
10 12 15 60 4
ab 8 ´ 12
Direct formula ® , Time taken by them to complete the work = 4 days
a + b 8 + 12
xyz
8 ´ 12 24 4 Direct formula: = days
= = = 4 hours xy + yz + zx
20 5 5
Shortcut: 10 ´ 12 ´ 15
= = 4 days
10 ´ 12 + 12 ´ 15 + 15 ´ 10
A 8h 3 units/h
(LCM) Shortcut:
24
Ram 10 6 units/day
B (Total work) 2 units/h (LCM)
12h
Rohit 12 60 5 units/day
Take L.C.M. as total work.
Rahul 15 (Total work) 4 units/day
24
One hour work of A is = 3 units and one hour Work completed by Ram, Rohit and Rahul in 1 day
8
= 6 + 5 + 4 = 15 units
24
work of B is = 2 units. Combined work of (A + B)
12 60
Total work completed by them in = = 4 days
in 1 hour is (3 + 2) units = 5 units. 15
Therefore total work will be completed by both (A + 3. A and B do a work in 10 days, B and C do it in 15
days while C and A in 20 days. In how many days A,
24 4 B and C all working together can finish the same
B) in hours = 4 hours
5 5 work?

224 @BEST300MCQ For More Study Material


Visit: studyiq.com
1 1
Sol. Work done by (A + B) in 1 day = Efficiency a
10 Time taken
1 The difference between time taken by A & B is 20
Work done by (B + C) in 1 day = days
15
The difference between ratio of A & B is 2
1 Therefore, 2 = 20 Þ 1 = 10
Work done by (C + A) in 1 day = Therefore A completed the work in 10 days and B in
20
= 3 ×10 = 30 days
1 1 1 5. Ram does 40% more work than Shiv. If Shiv can do it
Work done by 2(A + B + C) in 1 day = + +
in 14 days, then in how many days Ram can do the
10 15 20
same work?
6 + 4 + 3 13
Work done by (A + B + C) in 1 day = = 1
60 ´ 2 120 Sol. Shiv's 1 day work =
14
120
Time taken by them to complete the work = days 1 140 1
13 Ram's 1 day work = ´ =
14 100 10
2xyz Ram alone can finish the work = 10 days
Direct Formula ®
xy + yz + zx Shortcut:

2 ´ 10 ´ 15 ´ 20 120 Ram Shiv


= = days Efficiency 140 : 100
10 ´ 15 + 15 ´ 20 + 10 ´ 20 13
Shortcut: 7 : 5
Time taken 5 : 7
A+B 10 6 units/day
LCM The time taken by Shiv to complete the work is 14
B+C 15 60 4 units/day days
(Total work) Therefore, 7 º 14 Þ 1 º 2
C+A 20 3 units/day Time taken by Ram to complete the work = 2 × 5
Work done by A + B in 1 day = 6 units = 10 days
Work done by B + C in 1 day = 4 units 6. A can do a certain work in 12 days. B is 20% more
Work done by C + A in 1 day = 3 units efficient than A. How many days will A and B
Work done by 2(A + B + C) in 1 day = 6 + 4 + 3 together take to do the same job?
= 13 units 1
Sol. A's 1 day work =
60 120 12
Total work completed by them in = = days
13 13 1 120 1
2 B's 1 day work = ´ =
12 100 10
4. A is thrice as good as B. A is able to finish the work in
æ 1 1 ö 5 + 6 11
20 days less than B. In how many day B will finish (A+B)'s 1 day work = ç + ÷ = =
the work? è 12 10 ø 60 60
Sol. A's efficiency = 3 × B’s efficiency. 60
(A+B) completed the work in days
A's efficiency 3 11
= Þ 3:1
B's efficiency 1 Shortcut:
Ratio of Time taken by A and B = 1 : 3 A B
Let A and B do the work in x and 3x days respectively. Efficiency 100 : 120
3x – x = 20 Þ 2x = 20 Þ x = 10 5 : 6
Time taken by B to finish the work = 3x Time taken 6 : 5
= 3 × 10 = 30 days
6 º 12, \ 1 º 2
Shortcut:
Time taken by B to complete the work in 2 × 5 = 10
A:B days
Efficiency 3:1 12 ´ 10 60
Both can complete the work in = days
Time Taken 1 : 3 12 + 10 11

225 @BEST300MCQ For More Study Material


Visit: studyiq.com
7. The ratio of efficiency of two workers is 3 : 2. Working
æ 6 + 4 + 3 ö æ 4 + 3 ö 26 7 33 11
together if they complete a work in 6 days then in = 2 ´ç + = + = =
how many days will they do it respectively? è 60 ÷ø çè 60 ÷ø 60 60 60 20
Sol. Ratio of efficiency of two workers = 3 : 2
11 9
Ratio of Time taken by them = 2 : 3 Remaining work = 1 - =
20 20
They complete the work in 2x days and 3x days.
Time taken by C to finish the remaining work
1 1 1 5 1
\ + = Þ = Þx=5
2x 3x 6 6x 6 9
\ They will complete the work in 2x days and 3x Remaining work 20
= = = 9 days.
days i.e 10 days and 15 days respectively. C's efficiency 1
8. A and B complete a work in 10 days and 15 days 20
respectively. They started together but A leaves the Shortcut:
work after 2 days and B alone continues the work. In
how many days B complete the rest of the work? A 10 6 units/day
LCM
1 B 15 60 4 units/day
Sol. A's work done in 1 day =
10
C 20 (Total work) 3 units/day
1
B's work done in 1 day = Work done by (A + B + C) in 2 days = 2(6 + 4 + 3) = 26
15
units
1 1 5 1 Work done by (B + C) in 1 day = 7
(A+B)'s work done in 1 day = + = =
10 15 30 6 Remaining work = 60 – (26 + 7) = 27 units
1 1 27
(A+B)'s work done in 2 days = 2 ´ = Remaining work completed by C in = 9 days.
6 3 3
1 2 10. A and B can separately make a wall in 10 days and
Remaining work = 1 - = 15 days respectively. How many days before will B
3 3
leave the work so that the wall would be finished in
Time taken to finish remaining work by B total 8 days?
2 Sol. Let B will leave the work x days before its completion.
Remaining work 2 15 \ A's 8 day's work + B's (8 – x) day's work = 1
= = 3 = ´ = 10 days
B's efficiency 1 3 1
1 1 8-x 4
15 \8´ + ( 8 - x) ´ =1Þ = 1-
10 15 15 5
Shortcut:
( 8 - x ) 1
\ = Þ 8 - x = 3 Þ x = 8 - 3 = 5 days
A 10 3 units/day 15 5
LCM
30 Shortcut:

B (Total work) 2 units/day A 10 3 units/day


15 LCM
Work done by (A + B) in 2 days = 2(3 + 2) = 10 units 30
Remaining work = 30 – 10 = 20 units (Total work)
B 15 2 units/day
20
Remaining work completed by B in = 10 days Work done by A in 8 days = 8 × 3 = 24 units
2
9. A, B, C can do a piece of work in 10 days, 15 days and Remaining work = 30 – 24 = 6 units
20 days respectively. They began to work together 6
but A leaves the work after 2 days and B after 3 days. Therefore B work for = 3 days
2
In how many days will C alone finish the remaining
work? B leaves the work before 5 days.
Sol. Two days' work + third day's work 11. 15 men working 8 hours make a wall in 10 days. In
how many days will 12 men working 10 hours will
æ 1 1 1 ö æ 1 1 ö make it?
= 2ç + + ÷ +ç + ÷
è 10 15 20 ø è 15 20 ø
226 @BEST300MCQ For More Study Material
Visit: studyiq.com
Sol. 15 men working 8 hours make a wall in 10 days
51 99
Let 12 men working 10 hours make that wall in x days \ 12 men + 24 women = + 24 = women
2 2
By using MDH formula
15 × 8 × 10 = x × 12 × 10 M 1D1 M 2 D2 17 ´ 33 99 ´ D 2
By = Þ =
15 ´ 8 W1 W2 1 2´3
x= = 10 days
12
17 ´ 33 ´ 6
12. If 10 masons working 7 h daily can build 28 pumps D2 = = 34 days
99
in 16 days, how many days will 12 masons working
6 h daily take to build 36 pumps? 15. 20 men or 24 women can complete a piece of work in
Sol. Given, 20 days. If 30 men and 12 women undertake to
M1 = 10, H1 = 7, D1 = 16, W1 = 28 complete the work, then the work will be completed
in
And M2 = 12, H2 = 6, D2 = ?, W2 = 36
We know that, Sol. Given,
20 men = 24 women Þ 5 men = 6 women
M 1 D 1 H1 M 2 D 2 H 2 Or 10 men = 12 women
=
W1 W2 \ 30 men + 12 women = 40 men
Or, M1D1H1W2 = M2D2W1H2 By M1D1 = M2D2
M 1 D 1H 1 W2 10 ´ 7 ´ 16 ´ 36 20 ´ 20
\ D2 = = = 20 days 20 × 20 = 40 × D2 Þ D 2 = = 10 days
M 2 W1 H 2 12 ´ 6 ´ 28 40
Number of days taken = 20 days \ Work will be completed in 10 days.
13. A can do a piece of work in 40 days and B can do it in 16. Hemant contracted to finish a work in 62 days. He
50 days. They do the work alternatively. In how many 1
days will they finish the work, if A begins? employed 32 men to work. But rd of work was
Sol. Work done by A and B in first 2 days
3
done in 30 days. How many additional men would
1 1 5+ 4 9 be employed so that remaining work be finished in
= + = =
40 50 200 200 given duration?
Sol. Total time = 62 days and total work = 1
9 99
\ work done in 44 days = 22 ´ = Remaining time = 62 – 30 = 32 days
200 100
1 2
99 1 Remaining work = 1 - =
remaining work after 44 days = 1 - = 3 3
100 100
On 45 day A's turn
th 1
32 men do rd of a work in 30 days
\ Time taken to finish remaining work by 3

1 2
Let (32 + x) men will do rd of work in 32 days
Remaining work 100 1 40 2 3
A= = = ´ =
A's efficiency 1 100 1 5
40 32 ´ 30 ( 32 + x) ´ 32 32 ´ 30 ( 32 + x) ´ 32
= Þ =
1 2 1 2
2 2 3 3
\ Total days to finish the work = 44 + = 44 days.
5 5 32 + x = 60 Þ x = 60 – 32 = 28 men
14. Either 8 men or 17 women can paint a wall in 33 17. A and B together can complete a work in 12 days. A
days. The number of days required to paint three such alone can complete it in 20 days. If B does the work
walls by 12 men and 24 women working at the same only half a day daily, then in how many days A and
rate is B together will complete the work?
Sol. Given,
1
17 Sol. (A+B)'s 1 day's work = and
8 men = 17 women Þ 1 man = women 12
8
1
17 51 A's 1 day's work =
12 men = ´ 12 = women 20
8 2

227 @BEST300MCQ For More Study Material


Visit: studyiq.com
Shortcut:
1 1 5- 3 1
B's 1 day's work = - = =
12 20 60 30 A 30 2 units/day
LCM
1 1 B 20 60 3 units/day
\ B's day's work =
2 60
C 10 (Total work) 6 units/day
1 1 3+1 1 (A + B)'s 1 day work = (2 + 3) = 5 units
\ (A+B)'s 1 day's work = + = =
20 60 60 15 (A + C)'s 1 day work = (6 + 2) = 8 units
\ The work will be completed in 15 days. 2 days work = (8 + 5) = 13 units
Shortcut: Days Work
A+B 12 5 units/day 2 13
LCM
×4 ×4
60 8 52
(Total work) 3 units/day Remaining work = 8 units
A 20
(A + B) does 5 units in next day
A's 1 day work = 3 units
3
B's 1 day work = 2 units (A + C) finish the remaining work in days
8
when B works half a day daily then B's work = 1 unit
then, (A + B)'s 1 day work = 3 + 1 = 4 units 3 3
Total time taken = 8 + 1 + = 9 days
8 8
60
work will be completed in = = 15 days 19. A, B and C undertake to complete a piece of work for
4 Rs. 1200. A can complete the work in 8 days, B in 12
18. A, B and C can do a piece of work in 30, 20 and 10 days and C in 16 days. They complete the work with
days, respectively. A is assisted by B on one day and the help of D in 3 days. What does the D get?
by C on the next day, alternately. How long would Sol. Total money = Rs. 1200
the work take to finish? Time taken by A = 8 days
Sol. Work done in first two days
1
One day work of A =
2 1 1 1 1 1 4 + 3 + 6 13 8
= + + = + + = =
30 20 10 15 20 10 60 60 Similarly,

52 1 1
Work done in first 8 days = One day work of B = , One day work of C =
60 12 16
Let D does the work in x days.
52 8 2 Then, according to the question.
Remaining work = 1 - = =
60 60 15
1 1 1 1 1 1 1 1 1 1
Now, it is the turn of A and B + + + = Þ = - - -
8 12 16 x 3 x 3 8 12 16
1 1 2+3 1 1 16 - 6 - 4 - 3 1 3
(A+B)'s 1 day's work = + = = = Þ = Þ x = 16
30 20 60 12 x 48 x 48
2 1 8-5 3 1 Now, ratio of work done by A, B, C and D
\ Remaining work = - = = =
15 12 60 60 20 1 1 1 1
= : : : = 6 : 4 : 3 : 3 (taking LCM)
Now, it is the turn of A and C 8 12 16 16

1 1 1+3 2 3
(A+C)'s 1 day's work = + = = So, amount got by D = ´ 1200 = Rs. 225
30 10 30 15 16
Shortcut:
1 15 3
\ Time taken = ´ = days A 8 6 units/day
20 2 8 LCM
B 12 4 units/day
48
3 3 C 16 3 units/day
Total time = 9 + = 9 days
8 8 A+B+C+D 3 (Total work) 16 units/day

228 @BEST300MCQ For More Study Material


Visit: studyiq.com
One day work of D = 16 - (6 + 4 + 3) = 16 - 13 = 3 Sol. Total salary for 20 days = Salary of one day × number
of days = Rs. (75 × 20) = Rs. 1500
3 3
D can get = ´ 1200 = ´ 300 = Rs. 225 Actual salary received = Rs.1140
16 4
Difference = Rs. (1500 – 1140) = Rs. 360
20. A worker was appointed by a contractor on the
condition that he would be paid Rs. 75 for each day of Money deducted for 1 day's absence from work
his work but would be fined at the rate of Rs. 15 per = Rs. (15 + 75) = Rs. 90
day for his absence, apart from losing his wages. After
20 days, the contractor paid the worker Rs. 1140. Find 360
Number of days he absented = = 4 days
the number of days the worker absented from the 90
work.

Ques ti ons 8. 5 men can prepare 10 toys in 6 days working 6 hours


a day. Then in how many days can 12 men prepare
16 toys working 8 hrs a day?
1. 18 boys can do a piece of work in 24 days. In how
(a) 5 days (b) 3 days
many days can 27 boys do the same work?
(c) 4 days (d) 6 days
(a) 16 (b) 32
9. If A and B together can complete a work in 18 days. A
(c) 23 (d) 48
and C together in 12 days, and B and C together in 9
2. How many days will 1648 persons take to construct
days, then B alone can do the work in
a dam, if 721 persons can build the same in 48 days?
(a) 18 days (b) 24 days
(a) 21 days (b) 20 days
(c) 30 days (d) 40 days
(c) 23 days (d) 24 days
10. P, Q and R contract a work for Rs. 550. Together, P
3. If 10 persons can do a job in 20 days, then 20 persons
with twice the efficiency can do the same job in : 7
and Q are supposed to do of the work. How much
(a) 5 days (b) 10 days 11
(c) 20 days (d) 40 days does R get?
4. A and B can separately do a piece of work in 6 days (a) Rs. 200 (b) Rs. 300
and 12 days respectively. How long will they together (c) Rs. 150 (d) Rs. 250
take to do the work? 11. P, Q and R undertake to do a work for Rs. 707. P and
(a) 9 days (b) 18 days 5
Q together do of the work and rest is done by R
(c) 6 days (d) 4 days 7
5. A job can be completed by 12 men in 12 days. How alone. How much should R get?
many extra days will be needed to complete the job if, (a) Rs. 202 (b) Rs. 200
6 men leave after working for 6 days? (c) Rs. 102 (d) Rs. 150
(a) 3 (b) 6 12. A and B can complete a piece of work in 15 days and
(c) 12 (d) 24 10 days respectively. They contracted to complete the
6. A and B can do a piece of work in 12 days and 15 work for Rs. 30000. The share of A in the contracted
days, respectively. They began to work together but money will be
A left after 4 days. In how many more days would B (a) Rs. 18000 (b) Rs. 16500
alone complete the remaining work?
(c) Rs. 12500 (d) Rs. 12000
20 25 13. A daily-wage labourer was engaged for a certain
(a) (b)
3 3 number of days for Rs 5,750 but being absent on some
(c) 6 (d) 5 days he was paid only Rs 5,000. What was his
maximum possible daily wage?
7. Working efficiencies of A and B for completing a piece
of work are in the ratio 3 : 4. The number of days to be (a) Rs 125 (b) Rs 250
taken by them to complete the work will be in the (c) Rs 375 (d) Rs 500
ratio 14. A can finish a work in 24 days, B in 9 days and C in
(a) 3 : 2 (b) 2 : 3 12 days. B and C start the work but are forced to leave
(c) 3 : 4 (d) 4 : 3 after 3 days. The remaining work was done by A in:

229 @BEST300MCQ For More Study Material


Visit: studyiq.com
(a) 5 days (b) 6 days 23. A can do piece of work in 12 days and B in 15 days.
They work together for 5 days and then B left. The
1 days taken by A to finish the remaining work is
(c) 10 days (d) 10 days
2 (a) 3 (b) 5
15. If 3 men or 6 women can do a piece of work in 16 days, (c) 10 (d) 12
in how many days can 12 men and 8 women do the 24. A and B can do a work in 12 days. B and C can do the
same piece of work? same work in 15 days, C and A can do the same work
(a) 4 days (b) 5 days in 20 days. The time taken by A, B and C to do the
(c) 3 days (d) 2 days same work to
(a) 5 days (b) 10 days
16. A can do a work in 15 days and B in 20 days. If they
together work on it for 4 days, then the fraction of the (c) 15 days (d) 20 days
work that is left is: 25. 18 women can complete a work in 12 days and 12
men can complete the same work in 9 days. In how
8 7 many days will 8 men and 8 women complete that
(a) (b) work?
15 15
(a) 9 days (b) 6 days
1 1 (c) 12 days (d) 8 days
(c) (d)
4 10 26. A and B can do a piece of work in 12 days, B and C in
17. If a job is to be completed in 10 days, it requires 270 8 days, and C and A in 6 days. How long would B
persons. If 180 persons take up the same job, they take to do the same work alone?
will finish it in (a) 24 days (b) 32 days
(a) 27 days (b) 12 days (c) 40 days (d) 48 days
(c) 15 days (d) 18 days 27. A group of men decided to do a work in 10 days, but
18. 4 mat-weavers can weave 4 mats in 4 days. At the five of them absented themselves. If the rest of the
same rate how many mats would be woven by 8 mat- group finished the work in 12 days, find the original
weavers in 8 days? number of men?
(a) 4 (b) 8 (a) 20 men (b) 30 men
(c) 12 (d) 16 (c) 40 men (d) 50 men
19. A and B can do a job in 6 and 12 days, respectively. 28. To complete a work, A takes 50% more time than B. If
They began the work together but A leaves after 3 together they take 18 days to complete the work, how
days. Then, the total number of days needed for the much time shall B take to do it?
completion of the work is (a) 30 days (b) 35 days
(a) 4 (b) 5 (c) 40 days (d) 45 days
(c) 6 (d) 9 29. A complete a piece of work in 4 days and B completes
20. A alone can complete a work in 18 days and B alone it in 6 days. If they both work on it together, then the
in 15 days. B alone worked at it for 10 days and then number of days required to complete the same work
left the work. In how many more days, will A alone is
complete the remaining work?
2 2
1 (a) 2 days (b) 3 days
(a) 5 (b) 5 5 5
2
(c) 6 (d) 8 5 5
(c) 3 days (d) 2 days
21. How many men will be required to plough 100 acres 2 2
of land in 10 days if 10 men require 8 days to plough 30. A and B can do a piece of work in 18 days, B and C in
20 acres of land? 24 days, A and C in 36 days. In what time can they do
(a) 30 (b) 40 it all working together?
(c) 60 (d) 50 (a) 12 days (b) 13 days
22. A and B can do a piece of work in 20 days and 12 (c) 16 days (d) 26 days
days, respectively. A started the work alone and then 31. A and B can separately complete a piece of work in
after 4 days B joined him till the completion of the 20 days and 30 days respectively. They worked
work. How long did the work last? together for some time, then B left the work. If A
(a) 10 days (b) 20 days completed the rest of the work in 10 days, then B
(c) 15 days (d) 6 days worked for?

230 @BEST300MCQ For More Study Material


Visit: studyiq.com
(a) 6 days (b) 8 days (a) 5 days (b) 4 days
(c) 12 days (d) 16 days (c) 6 days (d) 7 days
32. A can do a work in 6 days and B can do the same 37. If 10 men or 20 boys can make 260 shirts in 20 days,
work in 5 days. The contract for the work is of Rs. 220.
then how many shirts will be made by 8 men and 4
How much shall B get if both of them work together?
boys in 20 days?
(a) Rs. 110 (b) Rs. 120
(a) 260 (b) 240
(c) Rs. 130 (d) Rs. 150
33. P, Q and R undertake to do a work for Rs. 660. P and (c) 280 (d) 520
38. A can finish a work in 12 days and B can do it in 15
8
Q together do of the work and rest is done by R days. After A had worked for 3 days, B also joined A to
11
finish the remaining work. In how many days, the
alone. How much should R get?
remaining work will be finished?
(a) Rs. 200 (b) Rs. 160
(c) Rs. 180 (d) Rs. 190 1 1
(a) 5 (b) 4
34. A farmer can plough a field working 6 h per day in 2 2
18 days. The worker has to work how many hours
(c) 5 (d) 6
per day to finish the same work in 12 days?
39. A man and a boy received Rs. 800 as wages for 5 days
(a) 7 (b) 9
for the work they did together. The man's efficiency in
(c) 11 (d) 13
the work was three times that of the boy. What are the
35. A can do a job in 6 days and A and B can do it together daily wages of the boy?
in 2 days. How many days will be taken by B to do the (a) Rs. 76 (b) Rs. 56
job alone?
(c) Rs. 44 (d) Rs. 40
(a) 5 days (b) 3 days
40. If a work can be completed by A in 30 days and by B
(c) 4 days (d) 8 days in 60 days, then the number of days taken by them to
36. If 6 men and 8 boys can do a piece of work in 10 days finish the work, working together, is:
and 26 men and 48 boys can do the same work in 2 (a) 20 (b) 25
days, the time taken by 15 men and 20 boys to do the (c) 45 (d) 50
same type of work will be:

1. Some staff promised to do a job in 18 days, but 6 of


1
them went on leave. So, the remaining men took 20 5. P can complete of a work in 10 days, Q can
days to complete the job. How many men were there 4
originally? complete 40% of the same work in 15 days, R can
(a) 55 (b) 62 1
complete of the work in 13 days and S can
(c) 56 (d) 60 3
2. 5 men and 5 women earn Rs. 660 in 3 days. 10 men
1
and 20 women earn Rs. 3500 in 5 days. In how many complete of the work in 7 days, Who will be able to
days can 6 men and 4 women earn Rs. 1060? 6
(a) 5 days (b) 10 days complete the work first?
(c) 6 days (d) 12 days (a) P (b) Q
3. A track of 100 m can be built by 7 men or 10 women in (c) R (d) S
10 days. How many days will 14 men and 20 women 6. A can do a work in 20 days and B in 40 days. If they
take to build a track of 600 m? work on it together for 5 days. Then fraction of the
(a) 15 (b) 20 work that is left, is:
(c) 25 (d) 30
5 5
4. A and B together can complete a work in 3 days. (a) (b)
8 15
They start together but after 2 days, B left the work. If
the work is completed after two more days, B alone 7 1
would do the work in (c) (d)
15 10
(a) 5 days (b) 6 days
(c) 9 days (d) 10 days

231 @BEST300MCQ For More Study Material


Visit: studyiq.com
7. A is 50% as efficient as B. C does half of the work (a) Rs. 11,600 (b) Rs. 11,700
done by A and B together in same time. If C alone (c) Rs. 16,100 (d) Rs. 17,100
does the work in 20 days, then A, B and C together
can do work in: 16. A, B and C together can finish a piece of work in 4
days, A alone can do it in 12 days and B alone in 18
2 2 days. How many days will be taken by C to do it
(a) 5 days (b) 6 days
3 3 alone?
(c) 6 days (d) 7 days (a) 21 (b) 16
8. If 12 men or 18 women can make a wall in 14 days, (c) 14 (d) 9
then working at the same rate, 8 men and 16 women 17. A and B can do a piece of work in 20 days and 12
can make the same wall in: days respectively. A started the work alone and then
(a) 9 days (b) 5 days after 4 days B joined him till the completion of the
(c) 7 days (d) 8 days work. How long did the work last?
9. A certain number of men complete a work in 160 (a) 10 days (b) 20 days
days. If there were 18 men more, the work could be (c) 15 days (d) 6 days
finished in 20 days less. How many men were
18. A man, a woman and a boy can together complete a
originally there?
piece of work in 3 days. If a man alone can do it in 6
(a) 116 (b) 122
days and a boy alone in 18 days, how long will a
(c) 124 (d) 126 woman alone take to complete the work?
10. A team of 30 men is supposed to do a work in 38 (a) 9 days (b) 21 days
days. After 25 days, 5 more men were employed and
(c) 24 days (d) 27 days
the work was finished one day earlier. How many
days would it have been delayed if 5 more men were 19. A certain job was assigned to a group of men to do in
not employed? 20 days. But, 12 men did not turn up for the job and
(a) 1 day (b) 2 days the remaining men did the job in 32 days. The original
(c) 3 days (d) 4 days number of men in the group was:
11. 8 children and 12 men complete a certain piece of (a) 32 (b) 34
work in 9 days. If each child takes twice the time (c) 36 (d) 40
taken by a man to finish the work, in how many days 20. If 16 men or 20 women can do a piece of work in 25
will 12 men finish the same work? days, in what time will 28 men and 15 women do it?
(a) 8 days (b) 10 days
2 1
(c) 11 days (d) 12 days (a) 14 days (b) 33 days
7 3
12. A, B and C together earn Rs. 2700 in 18 days. A and C
together earn Rs. 940 in 10 days. B and C together
3
earn Rs. 1520 in 20 days. Find the daily earning of C? (c) 18 days (d) 10 days
4
(a) Rs. 20 (b) Rs. 40
21. A certain number of men can do a piece of work in 40
(c) Rs. 10 (d) Rs. 15
days. If there were 45 men more the work could have
13. Two women, Sita and Gita working separately can been finished in 25 days. Find the original number of
mow a field in 8 and 12 hrs respectively. If they work men employed in the work.
in stretches of one hours alternatively, Sita beginning
(a) 70 (b) 85
at 9 a.m., when will the mowing be finished?
(c) 65 (d) 75
(a) 6:30 pm (b) 5:30 pm
22. 7 men can complete a piece of work in 12 days. How
(c) 7:00 pm (d) 6:30 am many additional men will be required to complete
14. If the wages of 45 women amount to Rs. 46575 in 48 double the work in 8 days?
days, how many men must work 16 days to receive (a) 28 (b) 21
Rs. 17250, the daily wages of a man being double
(c) 14 (d) 7
than those of a woman?
23. 6 men or 12 women can do a piece of work in 20 days.
(a) 20 men (b) 25 men
In how many days can 8 men and 16 women do twice
(c) 30 men (d) 15 men as big as this work?
15. 5 men or 7 women can earn Rs. 5,250 per day, how (a) 2 (b) 5
much would 7 men and 13 women earn per day? (c) 15 (d) 10

232 @BEST300MCQ For More Study Material


Visit: studyiq.com
24. A and B working separately can do a piece of work in 31. P, Q and R contracted to do a work for Rs. 4200. P can
9 and 12 days, respectively. If they work for a day do the work in 6 days, Q in 10 days and R in 12 days.
alternatively with A beginning, the work would be If they work together to do the work, what is the share
completed in: of R?
(a) Rs. 2000 (b) Rs. 1200
2 1
(a) 10 days (b) 10 days (c) Rs. 1000 (d) Rs. 1500
3 2
32. If 8 men or 10 women can do a piece of work in 50
1 1 days, in what time will 28 men and 15 women do it?
(c) 10 days (d) 10 days
4 3 1
2
25. If 10 men or 18 boys can do a work in 15 days, then (a) 14 days (b) 33 days
7 3
the number of days required by 15 men and 33 boys
to do twice the work is 3
(c) 18 days (d) 10 days
(a) 9 (b) 36 4
1 33. If 3 men with 4 boys can earn Rs. 2100 in 7 days and
(c) 4 (d) 8 11 men with 13 boys can earn Rs. 8300 in 8 days, in
2
what time will 7 men with 9 boys earn Rs. 11000?
26. 20 women together can complete a work in 16 days.
(a) 16 days (b) 18 days
16 men together can complete the same work in 15
days. The ratio of the working capacity of a man to (c) 14 days (d) 20 days
that of a woman is 34. There is sufficient food for 400 men for 31 days. After
(a) 3 : 4 (b) 4 : 3 28 days 280 men leave the place. For how many days
(c) 5 : 3 (d) 4 : 5 will the rest of the food last for the rest of the men?
27. Ram can finish a work in 15 days working 8 hrs a (a) 5 days (b) 10 days
(c) 12 days (d) 15 days
2
day. Hari can finish it in 6
days at 9 hrs a day. Find 7
3 35. If 28 men complete of a piece of work in a week,
in how many days both can finish it together, if both 8
work 10 hrs a day? then the number of men, who must be engaged to get
(a) 2 days (b) 3 days the remaining work completed in another week is
(c) 4 days (d) Can't be determined (a) 5 (b) 6
28. 3 men and 2 women can do a piece of work in 15 (c) 4 (d) 3
days. 2 men and 3 women can do the same work in 36. A contractor undertakes to dig a canal 12 km long in
18 days. The number of days to be taken by 1 man 350 days and employs 45 men. After 200 days he
and 1 woman to do the work is. find that only 4.5 km of the canal has been completed.
Find the number of extra men he must employ to finish
10 10
(a) 20 (b) 30 the work in time.
11 11 (a) 45 men (b) 55 men
10 10 (c) 65 men (d) 75 men
(c) 40 (d) 45
11 11 37. A and B undertook to do a piece of work for Rs. 4500.
29. A takes twice as much time as B and C takes thrice as A alone could do it in 8 days and B alone in 12 days.
much time as B to finish a piece of work. Working With the assistance of C they finished the work in 4
together they can finish the work in 12 days. The days. Then, C's share of the money is
number of days needed for A to do the work alone is (a) Rs. 2250 (b) Rs. 1500
(a) 20 (b) 22 (c) Rs. 750 (d) Rs. 375
(c) 33 (d) 44 38. A can do a piece of work in 16 days and B in 24 days.
30. Hemant decided to build a farmhouse in 40 days. He They take the help of C and three together finish the
employed 100 men in the beginning and 100 more work in 6 days. If the total renumeration for the work
after 35 days and completed the construction in is Rs. 400. The amount (in Rs.) each will receive, in
stipulated time. If he had not employed the additional proportion, to do the work is
men, how many days behind schedule would it have (a) A : 150, B : 100, C : 150
been finished? (b) A : 100, B : 150, C : 150
(a) 5 days (b) 6 days (c) A : 150, B : 150, C : 100
(c) 8 days (d) 10 days (d) A : 100, B : 150, C : 100

233 @BEST300MCQ For More Study Material


Visit: studyiq.com
39. A, B and C completed a work costing Rs. 1800. A 40. A man can do a work in 10 days. With the help of a
worked for 6 days, B for 4 days and C for 9 days. If boy he can do the same work in 6 days. If they get Rs.
their daily wages are in the ratio of 5 : 6 : 4, how much 50 for that work , what is the share of that boy?
amount will be received by A? (a) Rs. 20 (b) Rs. 40
(a) Rs. 800 (b) Rs. 600. (c) Rs. 50 (d) Rs. 60
(c) Rs. 900 (d) Rs. 750

1. A 10 hectare field is reaped by 2 men, 3 women and 4 7. 2 men and 1 woman can do a piece of work in 14
children together in 10 days. If working capabilities days, while 4 women and 2 men can do the same
of a man, a woman and a child are in the ratio 5 : 4 : 2, work in 8 days. If a man gets Rs. 90 per day, what
then a 16 hectare field will be reaped by 6 men, 4 should be the wages per day of a women?
women and 7 children in
(a) Rs. 48 (b) Rs. 60
(a) 5 days (b) 6 days
(c) Rs. 72 (d) Rs. 135
(c) 7 days (d) 8 days
8. Heena can do a work in 20 days, while Himani can
2. A can do a certain work in the same time in which B
do the same work in 25 days. They started the work
and C together can do it. If A and B together could do
it in 10 days and C alone in 50 days, then B alone jointly. Few days later Mayuri also joined them and
could do the work in: thus all of them completed the whole work in 10 days.
(a) 15 days (b) 20 days All of them were paid total Rs. 700. What is the share
(c) 25 days (d) 30 days of Mayuri?
3. One man, 3 women and 4 boys can do a work in 96 (a) Rs. 130 (b) Rs. 185
hrs, 2 men and 8 boys can do it in 80 hrs and 2 men (c) Rs. 70 (d) Rs. 80
and 3 women can do it in 120 hrs. In how many 9. A and B can complete a piece of work in 12 and 18
hours can it be done by 5 men and 12 boys? days respectively. A begins to do the work and they
5 7 work alternatively one at a time for one day each. The
(a) 41 hrs (b) 43 hrs whole work will be completed in
11 11
5 7 1 2
(c) 43 hrs (d) 42 hrs (a) 14 days (b) 15 days
11 11 3 5
4. 40 men can complete a work in 40 days. They started 1 2
the work together. But at the end of each 10th day, 5 (c) 16 days (d) 18 days
3 3
men left the job. The work would have been completed
in 10. A, B and C can do a piece of work individually in 8,
10 and 15 days respectively. A and B start working
2 1 but A quits after working for 2 days. After this, C
(a) 56 days (b) 56 days
3 3 joins B till the completion of work. In how many days
(c) 52 days (d) 50 days will the work be completed?
5. 3 women and 18 children together take 2 days to 53 34
complete a piece of work. How many days will 9 (a) days (b) days
9 7
children alone take to complete the piece of work, if 6
women alone can complete the piece of work in 3 85 53
(c) days (d) days
days? 13 10
(a) 9 (b) 7 11. A can complete a work in 10 days, B can complete the
(c) 5 (d) 6 same work in 20 days and C in 40 days. A starts
6. A can do a piece of work in 120 days and B can do it working on the first day, B works for second day and
in 150 days. They work together for 20 days. Then B C works for third day. Again A works for fourth day
leaves and A alone continues the work. After 12 days and B for fifth day and so on. If they continued
C joins A and the work is completed in 48 days more. working in the same way, in how many days will the
In how many days can C do it if he works alone? work be completed?
(a) 230 days (b) 225 days (a) 15 days (b) 16.5 days
(c) 240 days (d) 220 days (c) 15.5 days (d) 17 days

234 @BEST300MCQ For More Study Material


Visit: studyiq.com
12. A can complete a work in 10 days, B in 12 days and C
1 1
in 15 days. All of them began the work together; but (a) 6 days (b) 6 days
A had to leave the work after 2 days of the start and B 2 4
also left 3 days before the completion of the work.
3 1
How long did the work last? (c) 6 days (d) 12 days
4 2
(a) 7 days (b) 8 days
(c) 10 days (d) 12 days 19. A contract is to be completed in 92 days and 234 men
were set to work, each working 16 h a day. After 66
13. A piece of work was to be completed in 40 days. A
number of men employed upon it did only half the 4
days, of the work is completed. How many
work in 24 days. 16 more men were then set on, and 7
the work was completed in the specified time, how additional men may be employed, so that the work
many men were employed at first? may be completed in time, each man now working 18
(a) 16 men (b) 32 men h a day?
(c) 24 men (d) 48 men (a) 162 (b) 234
14. A contract is to be completed in 50 days and 105 men (c) 262 (d) 81
were set to work, each working 8h a day. After 25 20. A, B and C can complete a piece of work in 15, 30 and
40 days respectively. They started the work together
2
days, th of the work is finished. How many but A left 2 days before the completion of the work
5 and B left 4 days before the completion of the work.
additional men be employed, so that the work may be In how many days was the work completed?
completed on time, each man now working 9 h a day?
3 2
(a) 34 (b) 36 (a) 7 days (b) 10 days
10 15
(c) 35 (d) 37
15. A can do a work in 5 days less than the time taken by 7 7
(c) 10 days (d) 10 days
1 30 15
B to do it. If both of them together take 11 days,
9 21. A , B and C can do a piece of work individually in 8,
then the time taken by 'B' alone to do the same work 12 and 15 days, respectively. A and B start working
(in days) is but A quits after working for 2 days. After this, C joins
(a) 15 (b) 20 B till the completion of work. In how many days will
(c) 25 (d) 30 the work be completed?
16. A and B can complete a piece of work in 45 and 40 8 6
(a) 5 days (b) 4 days
days respectively. Both started to work together, but 9 7
after some days A left and B alone completed the rest
work in 23 days. For how many days did A work? 7 3
(c) 6 days (d) 3 days
(a) 12 days (b) 10 days 13 4
(c) 8 days (d) 9 days 22. A piece of work can be completed by 10 men and 6
17. 4 men and 6 women together can complete a work in women in 18 days. Men work 9 hours per day while
8 days while 3 men and 7 women together can women work 7.5 hours per day. Per hour efficiency
complete it in 10 days. 20 women working together
2
will complete it in of a woman is rd of a man's efficiency. In how many
3
(a) 36 days (b) 32 days
days 10 men and 9 women complete the work?
(c) 24 days (d) 20 days
(a) 16 days (b) 20 days
18. Two workers A and B working together completed a
(c) 30 days (d) 25 days
job in 5 days. If A worked twice as efficiently as he
23. 4 men and 6 women get Rs. 1600 by doing a piece of
1 work in 5 days. 3men and 7 women get Rs. 1740 by
actually did and B worked as efficiently as he
3 doing the same work in 6 days. In how many days, 7
actually did, the work wouldhave completed in 3 days. men and 6 women can complete the same work getting
Find the time for A to complete the job alone? Rs. 3760?

235 @BEST300MCQ For More Study Material


Visit: studyiq.com
(a) 6 days (b) 8 days (a) Rs. 400 (b) Rs. 250
(c) 10 days (d) 12 days (c) Rs. 150 (d) Rs. 120
24. The total number of men, women and children 25. If 5 men and 3 boys can reap 23 hectares in 4 days
working in a factory is 18. They earn Rs. 4000 in a and if 3 men and 2 boys can reap 7 hectares in 2 days,
day. If the sum of the wages of all men, all women how many boys must assist 7 men in order that they
and all children is in the ratio of 18 : 10 : 12 and if the may reap 45 hectares in 6 days?
wages of an individual man, woman and child is in (a) 2 boys (b) 6 boys
the ratio 6 : 5 : 3, then how much a woman earn in a (c) 4 boys (d) 5 boys
day?

6. If 4 men or 8 women can do a piece of work in 15 days,


1
1. X does of a job in 6 days. Y completes rest of the job in how many days can 6 men and 12 women do the
4 same piece of work?
in 12 days. Then X and Y could complete the job
(a) 5 days (b) 10 days
together in.
(c) 45 days (d) 15 days
1
(a) 9 days (b) 8 days 7. A contractor undertook to complete a project in 90
8 days and employed 60 men on it. After 60 days, he
3 1 found that 3 of the work has already been completed.
(c) 9 days (d) 7 days
5 3 4
How many men can he discharge so that the project
2
2. Janardan completes of his work in 10 days. Time may be completed exactly on time?
3
(a) 40 (b) 20
3 (c) 30 (d) 15
he will take to complete of the same work, is:
5 8. If 90 men can do a certain job in 16 days, working 12
(a) 4 days (b) 8 days hours/day, then the part of that work which can be
(c) 6 days (d) 9 days completed by 70 men in 24 days, working 8 hours/
3. A and B together can do a given piece of work in 8 day is:
days, B and C can do the same work in 12 days and A,
5 2
B and C together complete it in 6 days. no. of days (a) (b)
required to finish the work by A and C together is: 8 3
(a) 24 days (b) 8 days
1 7
(c) 16 days (d) 12 days (c) (d)
3 9
1
4. In two days A, B and C together can finish of a 9. 20 men can do a piece of work in 18 days. They worked
2 together for 3 days, then 5 men joined. In how many
work and in another 2 days B and C together can more days is the work completed?
3 (a) 12 days (b) 14 days
finish part of the work. Then A alone can complete
10 (c) 13 days (d) 15 days
the whole work in: 10. A, B and C can complete a piece of work in 10, 12 and
(a) 15 days (b) 10 days 15 days respectively. A left the work 5 days before the
(c) 12 days (d) 14 days work was completed and B left 2 days after A had left.
5. 4 men and 6 women complete a work in 8 days, 2 men Number of days required to complete the whole work
and 9 women also complete it in 8 days. The number was:
of days in which 18 women complete the work is :
2
1 2 (a) 8 days (b) 6 days
3
(a) 5 days (b) 5 days
3 3
2
1 2 (c) 6 days (d) 7 days
(c) 4 days (d) 4 days 3
3 3

236 @BEST300MCQ For More Study Material


Visit: studyiq.com
11. If p men working p hours per day for p days produce (a) 5 hours (b) 8 hours
p units of work, then the units of work produced by n (c) 4 hours (d) 6 hours
men working n hours a day for n day is 19. Dinesh and Rakesh are working on an Assignment,
2 3 Dinesh takes 6 hours to type 32 pages on a computer,
p p
(a) (b) while Rakesh takes 5 hours to type 40 pages. How
n2 n2 much time will they take working together on two
different computers to type an assignment of 110 page?
n2 n3
(c) 2 (d) 2 (a) 7 hrs, 30 min. (b) 8 hrs,
p p (c) 8 hrs, 15 min. (d) 8 hrs, 25 min.
12. P and Q together can do a job in 6 days. Q and R can 20. A labourer was appointed by a contractor on the
condition he would be paid Rs 75 for each day of his
60
finish the same job in days. P started the work work but would be fined at the rate of Rs 15 per day
7 for his absence. After 20 days, the contractor paid the
and worked for 3 days then Q and R continued for 6 labourer Rs 1140. The number of days the labourer
days and completed the work. Then the difference of absented from work days is:
days in which R and P can complete the job alone is (a) 3 days (b) 5 days
(a) 10 days (b) 8 days (c) 4 days (d) 2 days
(c) 12 days (d) 15 days 21. A takes three times as long as B and C together to do a
13. A’s 2 days work is equal to B’s 3 days work. If A can job. B takes four times as long as A and C together to
complete the work in 8 days, then to complete the do the work. If all the three, working together can
work B will take: complete the job in 24 days, then the number of days,
(a) 15 days (b) 12 days A alone will take to finish the job is:
(c) 16 days (d) 14 days (a) 100 (b) 96
14. A can do half of a piece of work in 1 day, whereas B (c) 95 (d) 90
can do full. B can do half the work as C in 1 day. The 22. A man, a woman and a boy can complete a job in 3, 4
ratio of their efficiencies of work is: and 12 days. How many boys must assist 1 man and
(a) 1 : 2 : 4 (b) 2 : 1 : 4 1
1 woman to complete the job in of a day?
(c) 4 : 2 : 1 (d) 2 : 4 : 1 4
15. A can do in one day three times the work done by B in (a) 1 (b) 4
2 (c) 19 (d) 41
one day. They together finish of the work in 9 days. 23. A can do a piece of work in 4 hours; B and C can do it
5
in 3 hours. A and C can do it in 2 hours. How long
The number of days by which B can do the work alone
will B alone take do it ?:
are:
(a) 10 hours (b) 12 hours
(a) 120 days (b) 100 days (c) 8 hours (d) 24 hours
(c) 30 days (d) 90 days 24. A and B can do a piece of work in 30 days while B and
16. x can copy 80 pages in 20 hours, x and y together can C can do the same work in 24 days and C and A in 20
copy 135 pages in 27 hours. Then y can copy 20 pages days. They all work together for 10 days. How long
in: will A take to finish the remaining work?
(a) 20 hours (b) 24 hours (a) 30 days (b) 24 days
(c) 3 hours (d) 12 hours (c) 18 days (d) 36 days
17. Work done by (x + 4) men in (x + 5) days is equal to the 25. While working 7 hours a day, A alone can complete a
work done by (x - 5) men in (x + 20) days. Then the piece of work in 6 days and B alone in 8 days. In what
value of x is: time would they complete it together working 8 hours
(a) 15 (b) 20 a day?
(c) 16 (d) 30 (a) 3 days (b) 4 days
(c) 2.5 days (d) 3.6 days
18. Two workers A and B are engaged to do a piece of
work. A working alone would take 8 hours more to 26. A skilled, a half skilled and an unskilled labourer
complete the work that when work together. If B work for 7, 8 and 10 days respectively and they
together get Rs 369 for their work. If the ratio fo their
1
worked alone, would take 4 hours more than when 1 1 1
2 each day’s work is : : , then how much does
3 4 6
work together. The time required to finish the work
together is: trained labourer get (in rupees):

237 @BEST300MCQ For More Study Material


Visit: studyiq.com
(a) 164 (b) 102.50 34. A does half as much work as B in one-third of the time
(c) 201.50 (d) 143.50 taken by B. If together they take 10 days to complete a
work, then the time taken by B alone to do it would
27. If there is a reduction in the number of workers in a
have been:
factory in the ratio 15 : 11 and an increment in their
wages in the rate 22 : 25, then the ratio by which the (a) 30 days (b) 25 days
total wages of the workers should be decreased is: (c) 6 days (d) 12 days
35. A is 30% more efficient than B and can alone do a
(a) 6 : 5 (b) 5 : 6
piece of work in 23 days. In how many days A and B
(c) 3 : 7 (d) 3 : 5 working together, can finish the job ?
28. A can do a piece of work in 8 days which B can destroy (a) 11 days (b) 13 days
in 3 days. A has worked for 6 days, during the last 2 (c) 20 days (d) 21 days
days of which B has been destroying. How many days 36. If 4 men or 6 women can do a piece of work in 12 days
must A now work alone to complete the work? working 7 hours a day how many days will it take to
1 complete a work twice as large with 10 men and 3
(a) 7 days (b) 7 days women working together 8 hours a day?
3
(a) 6 days (b) 7 days
2 (c) 8 days (d) 10 days
(c) 7 days (d) 8 days 37. A and B can do a piece of work in 28 and 35 days
3
respectively. They began to work together but A leaves
29. A wall of 100 metres can be built by 7 men or 10 women after sometime and B completed remaining work in
in 10 days. How many days will 14 men and 20 women 17 days. After how many days did A leave ?
take to build a wall of 600 metres:
(a) 15 (b) 20 2
(a) 14 days (b) 9 days
(c) 25 (d) 30 5
30. One man, 3 women and 4 boys can do a piece of work 5
in 96 hours, 2 men and 8 boys can do it in 80 hours, 2 (c) 8 days (d) 7 days
9
men and 3 women can do it in 120 hours. 5 men and
12 boys can do it in 38. A man and a boy can complete a piece of work together
in 24 days. If for the last six days boy alone does the
1 7 work then it is completed in 28 days. How long the
(a) 39 hours (b) 42 hours man will take to complete the work alone?
11 11
(a) 72 days (b) 20 days
7 (c) 24 days (d) 36 days
(c) 43 hours (d) 44 hours
11 39. A and B together can complete a job in 8 days. Both B
31. A contractor undertakes to make a road in 40 days and C, working alone can finish the same job in 12
and employees 25 men. After 24 days, he finds that days. A and B commence work on the job, and work
only one-third of the road is made. How many extra for 4 days, where upon A leaves, B continues for 2
men should he employ so that he is able to complete more days, and then he leaves too, C now starts
the work 4 days earlier? working, and finishes the job. How many days will C
(a) 100 (b) 60 require to complete the job?
(c) 75 (d) None of these (a) 5 days (b) 8 days
32. A does half as much work as B in one sixth of the time (c) 3 days (d) 4 days
taken by B. If together they take 10 days to complete a 40. 16 women take 12 days to complete a work which can
work, how much time shall B take to do it alone? be completed by 12 men in 8 days. 16 men started
working and after 3 days 10 men left and 4 women
(a) 70 days (b) 30 days
joined them. How many days will they take to
(c) 40 days (d) 50 days complete the remaining work?
33. A 10 hectare field is reaped by 2 men, 3 women and 4 (a) 4 days (b) 6 days
children together in 10 days. If working capabilities (c) 8 days (d) 10 days
of a man, a woman and a child are in the ratio 5 : 4 : 2,
41. A and B together can do a piece of work in 12 days
then a 16 hectare field will be reaped by 6 men, 4
which B and C together do in 16 days. If A works for
women and 7 children in:
5 days, B works for 7 days then C complete the
(a) 5 days (b) 6 days remaining work in 13 days. In how much time B alone
(c) 7 days (d) 8 days do the whole work.

238 @BEST300MCQ For More Study Material


Visit: studyiq.com
(a) 48 days (b) 24 days 10 days. The time taken by A to complete the work
(c) 16 days (d) 12 days separately is.
42. To do a certain work, B would take time thrice as long (a) 22 days (b) 24 days
as A and C together and C twice as long as A and B (c) 30 days (d) 20 days
together. The three men together complete the work in

S olut ions 1
6. (c); 1 day's work of A =
12
1. (a); Here, M1 = 18, M2 = 27, D1 = 24
By the formula 1
1 day's work of B =
M1D1 = M2D2 Þ 18 × 24 = 27 × D2 15
Part of the work done by A and B in 4 days
18 ´ 24
\ D2 = = 16 days
27 æ 1 1 ö æ 5+ 4 ö 9 3
= 4ç + ÷ = 4ç ÷ = 4´ =
2. (a); Let 1648 persons can construct a dam in x days è 12 15 ø è 60 ø 60 5
Given, M1 = 1648 M2 = 721
3 2
D1 = x days, D2 = 48 days Remaining work = 1 - =
\ M1D1 = M2D2 Þ 1648 × x = 721 × 48 5 5
\ Time taken by B to complete the remaining
721 ´ 48
x = = 21 days 2
1648 work = ´ 15 = 6 days
\ 1648 persons require 21 days to construct a 5
dam. Shortcut:
3. (a); Efficiency is inversely proportional to time. A 12 5
LCM
Efficiency 2 : 1 ü 60
ý : : 20 : x
Persons 20 : 10 þ
(Total work) 4
B 15
1 ´ 10 ´ 20
\x = = 5 days. In 4 days, work done by (A + B) = 4(5 + 4) = 36
20 ´ 2 units
1 1 2+1 1 24
4. (d); (A + B)’s 1 day’s work = + = = Remaining work done by B = = 6 days
6 12 12 4 4
\ A and B together will complete the work in 4 7. (d); Working efficiencies are in the ratio 3 : 4
days. \ ratio of time taken by them = 4 : 3
1 8. (b); By using MDH formula
5. (b); Work done by 12 men in 6 days = M1D1H1W2 = M2D2H2W1
2
Here, 5 × 6 × 6 × 16 = 12 × D2 × 8 × 10
1 1
Remaining work = 1 - = 5 ´ 6 ´ 6 ´ 16
2 2 \ D2 = = 3 days
12 ´ 8 ´ 10
By the formula
M 1D1 M 2 D 2 12 ´ 12 6 ´ x 1
= , = 9. (b); (A + B)'s 1 day's work =
W1 W2 1 1 18
2 1
12 × 12 = 12 × x Þ x = 12 (B+C)'s 1 day's work =
9
Extra days to complete the work = 12 – 6 = 6 days
Shortcut: 1
(A+C)'s 1 day's work =
12 × 12 = 12 × 6 + 6 × x 12
6x = 12 × 6 Þ x = 12 Adding all the above three,
No. of extra days = 12 – 6 = 6 days 2(A + B + C)’s 1 day’s work

239 @BEST300MCQ For More Study Material


Visit: studyiq.com
1 1 1 2+4+3 9 1 15. (c); 3 m = 6w
= + + = = = \ 1m = 2w
18 9 12 36 36 4
12m + 8w = (12 × 2w) + 8w = 32w
1 Since, 6 women can do the work in 16 days,
\ (A + B + C)' s 1 day's work =
8 16 ´ 6
\ B's 1 day's work = (A + B + C)’s 1 day’s work – \ 32 women can do the work in = 3 days.
32
1 1 3-2 1
(A + C)’s 1 day’s work = - = = 1
8 12 24 24 16. (a); A’s work per day =
15
Hence, B alone can do the work in 24 days.
10. (a); Let the total work be 11 units 1
B’s work per day =
According to question (P + Q) finish 7 units works 20
means R will finish the remaining 4 units works
1 1 7
4 (A+B)’s work per day = + =
\ R will get an amount = ´ 550 = Rs. 200 15 20 60
11
11. (a); Let the total work be 7 units 7 7
\ (A+B)’s work in 4 days = 4 ´ =
60 15
Then, P + Q ® 5 units

R does ® ( 7 - 5) = 2 units 7 8
Þ fraction of work left = 1 - =
15 15
2 17. (c); Here, M1 = 270, M2 = 180, D1 = 10, D2 = x
So, R will get =´ 707 = Rs. 202
7 By the formula
12. (d); Total amount = 30000 M1D1 = M2D2 Þ 270 × 10 = 180 × x

1 1 270 ´ 10
A's 1 day's work = , B's 1 day's work = x= =15 Days
15 10 180
18. (d); Given, M1 = 4, D1 = 4, W1 = 4
1 1
Ratio = : = 2 : 3 ( taking LCM ) M2 = 8, D2 = 8, W2 = ?
15 10
M 1 D1 M 2 D 2 4´4 8´8
Sum of the ratios = 2 + 3 = 5 = Þ =
W1 W2 4 W2
2 4W2 = 64 Þ W2 = 16
\ A's share = ´ 30000 = Rs. 12000
5
1
13. (b); It is required to find the H.C.F. of 5750 and 5000, 19. (c); A's one day's work =
6
because his daily wage is their common factor.
Hence, the daily wage is Rs. 250. 1
B's one day's work =
14. (c); Work done by (B+C) in 3 days 12

æ1 1 ö 1 1 4+3 7 1 1 2+1 1
= 3´ ç + ÷ = + = = (A + B)'s one day's work = + = =
è 9 12 ø 3 4 12 12 6 12 12 4
3
7 5 (A + B)'s three day's work =
Remaining work = 1 - = 4
12 12 \ Required number of days = Remaining work ×
This part of work is done by A alone. 1
B' s time = ´ 12 = 3
1 4
Now, part of work is done by A in 1 day.
24 Hence, total days = 3 + 3 = 6
Shortcut:
5 A 6 2
\ part of work will be done in LCM
12
12
5 (Total work)
= 24 × = 10 days. 1
12 B 12

240 @BEST300MCQ For More Study Material


Visit: studyiq.com
In 3 days, (A + B) done = 9 units
2
Remaining work = 12 – 9 = 3 unit Now, work is done by (A + B) in 1 day
15
3 4
B done remaining work = = 3 days 15 4
1 \ work is done in = ´ = 6 days
5 2 5
Total work completed in = 3 + 3 = 6 days Hence, the work lasted for 4 + 6 = 10 days.
1 Shortcut:
20. (c); A's one day's work =
18 A 20 3
LCM
1 60
B's one day's work =
15
(Total work) 5
B 12
1 2
Part of work done by B in 10 days 10 ´ = In 4 days A done = 4 × 3 = 12
15 3
Remaining work = 48
2 1
Remaining work = 1 - = 48
3 3 (A + B) done remaining work = = 6 days
5+3
1 Work completed in (4 + 6) = 10 days
\ Time taken by A = ´ 18 = 6 days.
3 1
23. (a); 1 day's work of A =
Shortcut: 12
A 18 5 1
LCM 1 day's work of B =
90 15

(Total work) 1 1
B 15 6 1 day's work of A and B = +
12 15
Work done by B 10 in days = 10 × 6 = 60 units
5+4 9 3
Remaining work = (90 – 60) = 30 units = = =
60 60 20
30
Remaining work completed by A in = 6 days. 15
5 5 day's work of A and B =
20
21. (b); 10 men require 8 days to plough 20 acres of land
Let x men require 10 days to plough 100 acres of 15 20 - 15 5 1
Remaining work = 1 - = = =
land 20 20 20 4

( 10 ´ 8) x ´ 10 Time taken by A to complete


1
work
= Þ x = 40
20 100 4

1 1
22. (a); Given, A's 1 day's work = = ´ 12 = 3 days
20 4
Shortcut:
4 1
A's 4 day's work = = A 12 5 units/day
20 5 LCM
1 4 60
Remaining work = 1 - =
5 5 (Total work) 4 units/day
B 15
This part is completed by A and B together.
In 5 days,
1 1 (A + B) completed = 5(5 + 4) = 45 units
Now, (A + B)'s 1 day's work = +
20 12 Remaining work done by A only

3+ 5 8 2 60 - 45
= = = = = 3 days
60 60 15 5

241 @BEST300MCQ For More Study Material


Visit: studyiq.com
1 26. (d); A + B 12 2 units/day
24. (b); (A + B)'s 1 day's work = ...(i) LCM
12
B+C 8 24 3 units/day
1 (Total work)
(B + C)'s 1 day's work = ...(ii) C+A 6 4 units/day
15
2 (A + B + C)'s 1 day work = 4 + 3 + 2
1
(C + A)'s 1 day's work = ...(iii) 9
20 (A + B + C)'s 1 day work =
2
By eq. (i) + eq. (ii) + eq. (iii) (A + B + C) completed the work in
We get 2(A + B + C)'s 1 day's work
24 ´ 2 16
= =
1 1 1 5 + 4 + 3 12 1 9 3
= + + = = =
12 15 20 60 60 5
A+C 6 8
LCM
1 1 48
\ (A + B + C)'s 1 day's work = =
5 ´ 2 10
16 (Total work) 9
\ The time taken by A, B and C together to do the A+B+C
3
same work = 10 days
1 day work of B = 1 unit
Shortcut: B completed the work in 48 days.
A+B 12 5 27. (b); Suppose there were x men originaly
LCM
Then by using formula, M1D1 = M2D2
B+C 15 60 4 we have 10x = 12(x – 5)
(Total work)
C+A 20 3 ( 12 ´ 5)
\ x= = 30 men
2(A + B + C) 1 day work = 5 + 4 + 3 = 12 12 - 10
28. (a); Let B alone can do the work in x days.
12
(A + B + C) 1 day work = =6 3x
2 \ A can do the work in days.
2
Total work completed by (A + B + C) in
According to the question,
60 1 2 1 18 ´ 5
= = 10 days + = Þ 3x = 18 ´ 5 Þ = 30 days
6 x 3x 18 3
1 1 1
25. (a); 18 w's one day's work = ...(i) 29. (a); A's one day's work = , B's one day's work =
12 4 6
\ Required no. of days to complete the work by
1
12 m's one day's work = ...(ii) 12 2
9 A and B together = = 2 days.
Now dividing equation (i) by equation (ii), we 5 5
have Shortcut:

18w 1 9 9 A 4 LCM 3
= ´ =
12m 12 1 12 12
Þ 2w = 1m (Total work)
B 6 2
Or, 1 man = 2 women Þ 8 men = 16 women
\ 8 men and 8 women = 16 + 8 = 24 women 12 12 2
Work completed in = = = 2 days
Since, 18 women can complete the work in 12 3+2 5 5
days
1
\ 24 women can complete the work in 30. (c); Work done by (A + B) in 1 day = ...(i)
18
12 ´ 18
= = 9 days. 1
24 Work done by (B + C) in 1 day = ...(ii)
24
242 @BEST300MCQ For More Study Material
Visit: studyiq.com
34. (b); Given, H1 = 6h, D1 = 18, H2 = ?, D2 = 12
1
Work done by (C + A) in 1 day = ...(iii) By the Formula
36
On adding equation (i), (ii) and (iii) 6 ´ 18
H 1D 1 = H 2D 2 Þ H 2 = = 9h
Work done by 2(A + B + C) in 1 day 12
35. (b); B's 1 day's work = [(A + B)'s 1 day's work] – [A's
1 1 1 4+3+2 9 1
= + + = = = 1 days's work]
18 24 36 72 72 8
æ 1 1ö 2 1
\ Work done by (A + B + C) in 1 day =
1
=
1 çè - ÷ø = =
2 6 6 3
8 ´ 2 16
\ A, B and C together finish the work in 16 days \ B alone can finish the job in 3 days.
Shortcut: 36. (b); According to question.
(6M+8B) x 10 = (26M + 48B) x 2
A+B 18 4
LCM \ 60M + 80B = 52M + 96B or, 1M = 2B
B+C 24 72 3 \ 15M + 20B = (30 + 20) B = 50 boys
(Total work) 6M + 8B = (12 + 8) boys = 20 boys
A+C 36 2
20 boys can finish the work in 10 days.
2(A + B + C)'s 1 day work = 4 + 3 + 2 = 9 units \ 50 boys can finish the work in
9 20 ´ 10
(A + B + C)'s 1 day work = = = 4 days
2 50
72 37. (a); 10 men = 20 boys
Total work completed by (A + B + C) in 9 \ 1 man = 2 boys Þ 8 men = 16 boys
2 (8M + 4B) = (16 + 4) boys = 20 boys
Hence 8 men and 4 boys will make 260 shirts in
72 ´ 2 20 days.
= = 16 days
9
31. (a); Let A and B worked together for x days æ 1ö 1
38. (c); Work done by A in 3 days = ç 3 ´ ÷ = .
è 12 ø 4
According to the question,
Part of work done by A for (x + 10) days + part of æ 1ö 3
work done by B for x days = 1 Remaining work = çè 1 - ÷ø = .
4 4
x + 10 x 3x + 30 + 2x
+ =1Þ =1 Now , (A + B)’s 1 day's work
20 30 60
æ 1 1ö 9 3
30 = çè + ÷ø = =
5x + 30 = 60 Þ 5x = 30 Þ x = = 6 days 12 15 60 20
5
Remaining work will be finished in
32. (b); A can complete the work in 6 days.
B can complete the work in 5 days. 3 20
x= ´ = 5 days.
We know that there is inverse relation between 4 3
time and work and amount of Rs. 220 will be 39. (d); Man : boy = 3 : 1
distributed in the ratio of their work.
1
\ Boy's share = ´ 800 = Rs. 200
6 4
B will get an amount = ´ 220 = Rs. 120 .
11
æ 200 ö
33. (c); Let the total work be 11 units \ The daily wages of boy = Rs. ç = Rs. 40
è 5 ÷ø
Then, P + Q ® 8 units
æ 1 1 ö 1
R does ® ( 11 - 8) units = 3 units 40. (a); (A + B)’s 1 day's work = ç + ÷=
è 30 60 ø 20
3 \ Both working together can finish the work in
R's share = ´ 660 = Rs. 180
11 20 days.

243 @BEST300MCQ For More Study Material


Visit: studyiq.com
1. (d); Let M1 = x and D1 = 18 days = 2 × 3 = 6 days
But 6 men went on leave and it took 20 days to 1 1 2-1 1
complete the work. \ B's 1 days' work = - = =
3 6 6 6
\ M2 = (x – 6) and D2 = 20 days
\ B alone can do the work in 6 days.
By the formula
Shortcut:
M1D1 = M2D2 Þ (x × 18) = (x – 6) × 20
1
18x = 20x – 120 Þ 20x – 18x = 120 A + B ® 3®3
2x = 120 Þ x = 60 men. In 2 days (A + B) done 2 unit of work Remaining
2. (a); 3(5men + 5 women) = 660 work done by A in 2 days 1 days work of
660 1
(5 men + 5 women) = = 220 ...(i) A= unit
3 2
5(10 men + 20 women) = 3500 1
1 days work of B = unit
3500 2
(10 men + 20 women) = = 700 ...(ii)
5 3
On solving 1 and 2 Work is completed by B in = = 6 days
1
10w = 700 – 440 = 260 Þ 1w = 26 2
Similarly, 5. (b); Time taken by P in completing whole work
5m + 5× 26 = 220 = 10 × 4 = 40 days.
220 - 130 90 Time taken by Q in completing whole work
1m = = = Rs. 18
5 5 15 ´ 5 75 1
= = days = 37 days
Required no. of days 2 2 2
1060 1060 Time taken by R in completing 1 work
= =
( 18 ´ 6 + 26 ´ 4 ) 212 = 5 days = 13 × 3 = 39 days
Time taken by S in completing 1 work
10 = 7 × 6 = 42 days
3. (a); 7 men = 10 women, 1 man = women
7 Clearly, Q took the least time
14 men + 20 women 1
i.e., 37days.
æ 10 ´ 14 ö 2
=ç + 20 ÷ = 40 women Work done time ratio of P, Q, R, S to complete the
è 7 ø
whole work will be (total time/one day's work)
By the formula
10 15 13 7
M 1 D1 M 2 D 2 10 ´ 10 40 ´ D 2 P :Q :R :S = : : :
= Þ = 1 æ 40 ö 1 1
W1 W2 100 600 4 çè 100 ÷ø 3 6
600
D2 = Þ D2 = 15 days 1
40 = 40 : 37
: 39 : 42
2
1 2 Hence, Q will be able to complete the work first.
4. (b); Work done by A and B in 2 days = ´2 =
3 3 1 1 2+1 3
6. (a); (A + B)'s 1 day's work = + = =
2 1 20 40 40 40
Remaining work = 1 - =
3 3 3 3
\ (A + B)'s 5 day's work = 5 ´ =
1 40 8
Now, work is done by A in 2 days
3 3 5
\ Remaining work = 1 - =
\Whole work will be done by A in 8 8

244 @BEST300MCQ For More Study Material


Visit: studyiq.com
Shortcut: 10. (a); 35 men do the rest of the job in = (38 – 25 – 1) days
= 12 days
A 20 2
LCM \ 30 men can do the rest of the job in
40 12 ´ 35
= = 14 days
(Total work) 1 30
B 40
Thus the work would have been finished in 25 +
5 days work of (A + B) = 5(2 + 1) = 15 14 = 39 days, that is 39 – 38 = 1 days after the
25 5 scheduled time.
Remaining work = = 11. (d); If each child takes twice the time taken by a man,
40 8
8 children = 4 men.
1 \ 8 children + 12 men = 16 men do the work in 9
7. (b); C's efficiency = days
20
9 ´ 16
1 1 \ 12 men finish the work in = 12 days.
\ The efficiency of ( A + B ) = 2 ´ = 12
20 10
12. (a); 18 ( A + B + C) ® Rs. 2700
1
\ A's efficiency + B's efficiency = A + B + C ® 150 ...(i)
10
10 ( A + C ) ® 940
1
\ A's efficiency + 2 × A's efficiency =
10 A + C ® 94 ...(ii)

1 20 ( B + C) ® 1520
3 × A's efficiency =
10 B + C ® 76 ...(iii)
On solving (i) and (ii)
1 1
\ A's efficiency = = (A + B + C) – (A + C) = 150 – 94
10 ´ 3 30
Daily wages of B = Rs. 56
1 1 3-1 2 1 B + C = 76 Þ C = Rs. 20
\ B's efficiency = - = = =
10 30 30 30 15 1
13. (a); In the first hour Sita mows of the field.
\ (A + B + C)'s efficiency 8
1 1 1 2+4+3 9 3 1
= + + = = = In the second hour Gita mows of the field.
30 15 20 60 60 20 12
\ A, B and C together can do the work in
æ1 1 5ö
\ In first 2 hrs çè + = ÷ of the field is mowed.
20 2
= 6 days. 8 12 24 ø
3 3
5 5
8. (a); 12 men = 18 women Þ 2 men = 3 women \ In 8 hrs ´ 4 = of the field is mowed
24 6
Þ 8 men = 12 women
Field remained to be mowed = 1 – 5/6 = 1/6
\ 8 men and 16 women = 12 women + 16 women
= 28 women 1
In the 9th hour Sita mows of the field.
\ 18 women can make a wall in 14 days 8
Let 28 women can make the same wall in x days
æ 1 1ö 1
\ Gita will finish the mowing of çè - ÷ø = of
18 ´ 14 6 8 24
\ 18 × 14 = 28 × x Þ x = = 9 days
28
æ 1 1 ö 1
9. (d); Let the original number of men be x the field in ç ¸ ÷ or of an hour.
è 24 12 ø 2
x × 160 = (x + 18)(160 – 20)
Original number of men, x æ 1ö 1
\ the total time required is ç 8 + 1 + ÷ or 9 hrs.
18 ´ ( 160 - 20 ) è 2ø 2
= = 126
20 Thus, the work will be finished at 6:30 pm.

245 @BEST300MCQ For More Study Material


Visit: studyiq.com
14. (b); By formula 18. (a); Work done by 1 woman in 1 day

M 1 D1E1 M 2 D2 E2 1 1 1 6-3-1 1
= = – - = =
W1 W2 3 6 18 18 9
Let the number of men be x \ 1 Woman will do the work in 9 days.
19. (a); Let there be x men originally.
45 ´ 48 ´ 17250 ´ 1
x= = 25 men Now, x men do the job in 20 days.
46575 ´ 16 ´ 2
15. (d); 5 men = 7 women 20x
\ (x – 12) men can do it in = 32 days
( x - 12)
7 49
\ 7 men = ´7 = women hence, x = 32
5 5
20. (d); 16 men = 20 women Þ 4 men = 5 women.
49 114 Now, according to question,
and 7 men + 13 women = + 13 = women
5 5 16 men complete the work in 25 days.
Now,
1
Q 7 women get Rs. 5250 \ 1 man one day’s work =
25 ´ 16
114 5250 114
Q women get ´ = Rs. 17100 1 1
5 7 5 \ 4 men one day’s work = =
25 ´ 4 100
16. (d); C's 1 day's work
= [(A + B + C)'s 1 day's work] – [(A's 1 day's work) 5 1
\ 5 women one day’s work= =
+ (B's 1 day's work)] 25 ´ 20 100

1 æ 1 1 ö æ1 5 ö 4 1 28
\ 28 men = ´ 5 = 35 women
- + - = =
4 çè 12 18 ÷ø çè 4 36 ÷ø 36 9
= = 4

C complete the work in 9 days 50 1


\ 50 women one day’s work = =
25 ´ 20 10
1
17. (a); A’s 1 day’s work = Therefore, 28 men and 15 women can complete
20 the whole work in 10 days.
4 1 21. (d); Let number of persons in beginning be x
A’s 4 days' work = = M1 = x, D1 = 40, M2 = (x + 45) and D2 = 25
20 5
By the formula
1 4 M1D1 = M2D2 Þ x × 40 = (x + 45) × 25
Remaining work = 1 - =
5 5 x × 8 = (x + 45) × 5 Þ 8x – 5x = 225
This part is completed by A and B together. 225
x= = 75
3
1 1
Now, (A+B)’s 1 day’s work = + 22. (c); Given, M1 = 7, D1 = 12, W1 = 1
20 12
And M2 = ?, D2 = 8, W2 = 2
3+5 8 2 By the Formula
= = = M1D1W2 = M2D2W1 Þ 7 × 12 × 2 = M2 × 8 × 1
60 60 15
7 ´ 12 ´ 2
2 M2 = = 21
Now, work is done by (A+B) in 1 day. 8
15
\ Number of additional men = 21 – 7 = 14
4 15 4 23. (c); 6 men = 12 women
\ work is done in = ´ = 6 days. \ 1 man = 2 women
5 2 5
Now, 8 men + 16 women = (8 × 2 + 16)
Hence, the work lasted for 4 + 6 = 10 days.
= 32 women

246 @BEST300MCQ For More Study Material


Visit: studyiq.com
According to the question, 28. (c); (3 men + 2 women) × 15 = (2 men + 3 women) × 18
(3 men + 2 women) × 5 = (2 men + 3 women) × 6
M 1D1 M 2 D2 12 ´ 20 32 ´ D 2
= Þ = 15 men + 10 women = 12 men + 18 women
W1 W2 1 2
\ 15 men – 12 men = 18 women – 10 women
12 × 20 = 16D2 Þ D2 = 3 × 5 Þ D2 = 15 days
8
24. (c); A 9 \ 3 men = 8 women Þ 1 man = women
4 3

3 36 æ8 ö
\ 1 man + 1 women = çè + 1÷ø women
3
B 12
A does 4 work in 1 day out of total 36 work 11
= women
B does 3 work in 1 day out of total 36 work 3
In 2 days A and B do — 7 work and 3 men + 2 women = (8 + 2) women = 10
In 10 days A and B do — 35 work women
\ 10 women can do a work in 15 days
1
Remaining 1 work A will do in days. 11
4
Let women can do that work in x days
3
1
Thus total no. of days = 10 11 450 10
4 \ 10 ´ 15 = ´xÞx = = 40 days.
3 11 11
M 1 D1 H 1 M 2 D 2 H 2
25. (a); By = 29. (d); If B takes x days to finish the work
W1 W2
\ A takes 2x days and C takes 3x days to do the
10 M × 15 = 15 × 18 B work
Þ 5M=9B ... (i) 1 1 1 1 3+6+ 2 1
\ + + = = =
(15M + 33B) 2x x 3x 12 6x 12
and, 10M ´ 15 = ´D
2
11 1
Þ (18 B × 15)2 = (27 B + 33 B) × D \ = Þ 6x = 11 ´ 12 Þ x = 22 days.
6x 12
18 B ´ 15 ´ 2 \ The no. of days taken by A = 2x = 2 × 22 = 44
D= = 9 days days
60 B
30. (a); Let 100 men only complete the work in x days
M 1 D1 H 1 M 2 D 2 H 2 Work done by 100 men in 35 days + work done
26. (b); By =
W1 W2 by 200 men in 5 days = 1
20 W × 16 = 16 M × 15 35 200 ´ 5 35 10
+ =1, + =1
Þ 4W=3M x 100x x x

M 4 45
Þ = = 1, x = 45 days
W 3 x
27. (c); Change the time into hours Therefore, if additional men were not employed,
Ram finish in 15 × 8 = 120 hrs the work would have lasted (45 – 40) = 5 days
behind schedule time.
20
Hari finish in ´ 9 = 60 hrs 31. (c); P 6 10
3 (LCM)
Both of them working together finish the work in Q 10 60 6
120 ´ 60 (Total work) 5
= = 40 hrs R 12
120 + 60
5
40 The share of R = ´ 4200 = Rs. 1000
\ number of days = = 4 days. 21
10

247 @BEST300MCQ For More Study Material


Visit: studyiq.com
32. (d); Given, 8 men = 10 women 37. (c); Total money = Rs. 4500
4 men = 5 women
1 1
Now, according to the question A's one day's work = , B's one day's work =
8 12
8 men complete the work in 50 days.
1
28 (A + B + C)'s 1 day's work =
\ 28 men = ´ 5 = 35 women 4
4
1 æ 1 1 ö 1 æ 3 + 2ö
50 1 C's 1 day work = -ç + ÷ = -ç ÷
\ 50 women one day's work = = 4 è 8 12 ø 4 è 24 ø
50 ´ 10 10
Required days = 10 1 5 6-5 1
= - = =
33. (a); 7 (3m + 4b) = 2100 4 24 24 24
2100 \ Ratio of their one day's work
3m + 4b = = 300 ...(i)
7 1 1 1
8 (11m + 13b) = 8300 A:B:C = : : = 3 : 2 : 1 (taking LCM)
8 12 24
8300 Sum of the ratio = 3 + 2 + 1 = 6
11m + 13b = = 1037.50 ...(ii)
8
æ1 ö
On solving (i) and (ii) C's share = ç ´ 4500÷ = Rs. 750
è6 ø
44b – 39b = 187.5 Þ 5b = 187.5
1b = Rs. 37.5 1
38. (a); Work done by A in 1 day =
Similarly, 3m + 150 = 300 16
3m = 300 – 150 Þ 1m = Rs. 50
1
Work done by B in 1 day =
11000 24
Required no. of days =
7 ´ 50 + 9 ´ 37.5
1
11000 Work done by A, B and C in 1 day =
6
= = 16 days
687.5 Then, work done by C in 1 day = (A + B + C)'s 1
34. (b); The rest of the food will last for 31 – 28 = 3 days day work – (A + B)'s 1 day work – A's 1 day work
if nobody leaves the place.
Number of men eating everyday = 400
1 1 1 1 1 8- 3- 2 3 1
= - - Þ = = =
C 6 16 24 C 48 48 16
So, 1200 men could eat the remaining food
completely in one day. \ Ratio of wages of A, B and C
But there are 120 men left, so they can eat the 1 1 1 1 1 1
remaining food in 10 days. = : : = : :
A B C 16 24 16
M 1 D1H1 M 2 D2 H 2 = 3 : 2 : 3 (taking LCM)
35. (c); By using MDH formula = =
W1 W2
3 3
Share of A = ´ 400 = ´ 400 = Rs. 150
28 ´ 1 x 3+2+3 8
= Þ x = 4 men
7 1
2
8 8 Share of B = ´ 400 = Rs. 100
3+ 2 + 3
36. (b); By Direct Formula:
M1D1W2 = M2D2W1 3
Share of C = ´ 400 = Rs. 150
Or, 45 × 200 × 7.5 = M2 × 150 × 4.5 (3 + 2 + 3)

45 ´ 200 ´ 7.5 39. (b); Total money = Rs. 1800


\ M2 = = 100
150 ´ 4.5 Ratio of wages of A, B and C, respectively
\ required number of persons to be added = 5 × 6 : 6 × 4 : 4 × 9 = 30 : 24 : 36 = 5 : 4 : 6
= 100 – 45 = 55 men. \ Amount received by A

248 @BEST300MCQ For More Study Material


Visit: studyiq.com
Let the total work be 30 units
A
= ´ Total money One day work of man = 3 units
A+B+C
One day work of (man + child) = 5 units
5 5 So, one day work of child = 2 units
= ´ 1800 = ´ 1800 = Rs. 600
5+4+6 15
2
40. (a); A man can complete a work in 10 days Boy will get an amount = ´ 50 = Rs. 20
5
(Man + child) can do it in 6 days

1. (d); Ratio of the working capabilities of a man, a 3. (b); 1M + 3W + 4B in 96 hrs ...(i)


woman and a child = 5 : 4 : 2 2M + 8B in 80 hrs ...(ii)
\ Ratio of man, woman and child equivalence Or, 1M + 4B in 160 hrs ...(iii)
1 1 1 1 1 1 2M + 3W in 120 hrs ...(iv)
= : : = ´ 20 : ´ 20 : ´ 20 = 4 : 5 : 10 From (i) and (iii), we have,
5 4 2 5 4 2
3W do the work in
Or 4 men = 5 women = 10 children
4 men = 10 children 160 ´ 96
= 240 hrs ...(v)
\ 2 men = 5 children Þ and 6 men = 15 children 160 - 96
5 women = 10 children From (iv) and (v), we have
\ 3 women = 6 children Þ 4 women = 8 children 2M do the work in
\ 2 men + 3 women + 4 children = 15 children 240 ´ 120
6 men + 4 women + 7 children = 30 children = 240 hrs ...(vi)
240 - 120
By the Formula
2
M 1 D1 M 2 D 2 15 ´ 10 30 ´ x \ 5M do the work in 240 ´ = 96 hrs ...(vii)
= Þ = 5
W1 W2 10 16 From (ii) and (vi) we have,
15 ´ 10 ´ 16 120 ´ 8
x= Þ x = 8 days 12B do the work in = 80 hrs ...(viii)
10 ´ 30 12
2. (c); A's 1 day's work = (B + C)'s 1 days' work Now, from (vii) and (viii) we have,
1 5M + 12B do the work in
(A + B)'s 1 days' work =
10 96 ´ 80 480 7
= = 43 hrs
96 + 80 11 11
1
C's 1 days' work = 4. (a); Remaining part of work after 50 days
50
æ1 7 3 5 2 ö 1
1 1 5+1 = 1-ç + + + + ÷=
(A + B + C)'s day's work = + = è 4 32 16 32 16 ø 16
10 50 50
Similarly, part of work done by 15 men in 10
6 3
= = 15 ´ 10 3
50 25 days = =
1600 32
3
(A+B + C)'s 1 day's work = \
1
25 part of the work is done in
16
3
A's 1 days' work = 10 ´
32 1 20
´ =
2
= 6 days
50
3 16 3 3
1 3 5-3 2 1 \ Total number of days to complete the work
B's 1 days work = - = = =
10 50 50 50 25 2 2
= 50 + 6 = 56 days
Hence, B alone will complete the work in 25 days. 3 3

249 @BEST300MCQ For More Study Material


Visit: studyiq.com
Shortcut: (A + B) work together for 20 days Hence, (A + B)'s
Total work = 40 × 40 = 1600 20 day's work
After every 10 days, 5 men left the job
3 3
Therefore, = 20 ´ =
Work done in first 10 days = 40 × 10 200 10
Next 10 days = 35 × 10 After 20 days B leaves, and A alone works for 12
Next 10 days = 30 × 10 days
Next 10 days = 25 × 10
1 1
Next 10 days = 20 × 10 \ A's 12 day's work = ´ 12 =
120 10
Work done in 50 day = 1500
Remaining work = 1600 – 1500 = 100 Now, after 12 days, C joins A and the work is
15 men × No. of days = 100 finished in 48 days. It means A works for 48 days
more.
100 10 2
No. of days = =6 =6 1 2
15 15 3 \ A's 48 day's work = ´ 48 =
120 5
æ 2ö
\ Total work completed in ç 50 + 6 ÷ days \ Total work done by A and B together
è 3ø
3 1 2 3+1+ 4 8 4
2 = + + = = =
= 56 days 10 10 5 10 10 5
3
5. (d); 3 women + 18 children complete the work in 2 4 1
days. Therefore, (3 × 2) women + (18 × 2) children \ remaining work = 1 - =
5 5
complete the work in 1 day
6 women + 36 children complete the work in 1 1
This part of work, i.e., is done by C in 48 days
day. 5

1 2 1
Work of 36 children for 1 day = 1 - = \ C's 48 day's work =
3 3 5

é 1ù 1 1
êëQ work of 6 women for 1 day = 3 úû
\ C's 1 day's work = =
5 ´ 48 240
Hence, C alone can finish the work in 240 days.
2
\ 36 children do part of the work in 1 day. 7. (b); Let man be represented by m and woman be
3
represented by w.
3
or, 36 children can do the work in days. 1
2 Q 2m + 1w =
14
æ3 ö Þ 14 (2m + 1w) = 1 ...(i)
or, 9 children can do the work in ç ´ 4÷ = 6 days
è2 ø
1
and 4w + 2m =
1 8
6. (c); A's 1 day's work =
120 or, 8 (4w + 2m) = 1 ...(ii)
On equating Eqs. (i) and (ii), we get
1
B's 1 day's work = 14 (2m + 1w) = 8 (4w + 2m)
150
28m + 14w = 32w + 16m
1 1 28m – 16m = 32w – 14w Þ 12m = 18w
(A + B)'s 1 day's work = +
120 150
m 18 3
\ = =
5+ 4 9 3 w 12 2
= = =
600 600 200 So, efficiency of 1 man and 1 woman is 3 : 2. So,

250 @BEST300MCQ For More Study Material


Visit: studyiq.com
\ The days taken by B and C to complete the
90 3
their wages must be in the same ratio i.e = remaining work
x 2
11
[here, x = wages of a woman]
remaining work 20 11 6
= = = ´
90 ´ 2 working efficiency of B and C 1 20 1
\ x= = Rs. 60
6
3
8. (c); Efficiency of Heena = 5% 33
=
Efficiency of Himani = 4% 10
Thus, in 10 days working together they will \ Total days to complete the whole work
complete only 90% of the work.
33 20 + 33 53
[(5 + 4) × 10] = 90 =2+ = = days
10 10 10
Hence, the remaining work will be surely done Shortcut:
by Mayuri, which is 10%.
A 8 15
Thus, Mayuri will get 10% of Rs. 700, which is LCM
Rs. 70. B 10 120 12
1 (Total work) 8
9. (a); A’s 1 day’s work = C 15
12 2 days work of (A + B) = 2(27) = 54
Remaining work = 120 – 54 = 66
1
B’s 1 day’s work = Remaining work completed by (B + C) in
18
66 33
Part of work done by A and B in first two days = days
20 10
1 1 3+ 2 5
= + = = 33 53
12 18 36 36 Total work completed in 2 + = days
10 10
35
Part of work done by A and B in 14 days = 1
36 11. (b); A's work for the first day =
10
35 1
Remaining work = 1 – = 1
36 36 B's work for the second day =
20
Now A will work for 15th day.
1
1 C's work for the third day =
1 1 40
A will do the work in ´ 12 = day.
36 36 3
1 1 1
Work done in 3 days by them = + +
1 10 20 40
\ Work will be done in 14 days.
3
4+2+1 7
10. (d); (A + B)'s 2 day's work = =
40 40
æ1 1 ö 5+4 9 9
2´ç + ÷ = 2´ = 2´ = 7
è 8 10 ø 40 40 20 Hence, part of work will be completed in 3
40
9 11 days.
\ remaining work = 1 - =
20 20
7´5 35
\ i.e part of work will be completed in
1 1 5 1 40 40
(B + C)'s 1 day's work = + = =
10 15 30 6 = 3 × 5 or 15 days.
1 35 5 1
\ Working efficiency of (B + C) = Remaining work = 1 - = =
6 40 40 8

251 @BEST300MCQ For More Study Material


Visit: studyiq.com
Now, A will work on 16th day. The remaining
2 3
and W2 = 1 - =
1 1 5-4 1 5 5
work after 16 days = - = =
8 10 40 40 On putting these values in the above formula,
Again, B will work on 17th day.
105 ´ 25 ´ 8 ( 105 + x) ´ 25 ´ 9
Q B completes the work in 20 days. =
2 3
1 5 5
\ B will complete part of work in
40
105 ´ 8 ( 105 + x) ´ 9
1 1 Þ =
= 20 ´ = day 2 3
40 2
105 × 4 = (105 + x) × 3 Þ 105 × 4 = 105 × 3 + 3x
\ Total time taken in completion of work
3x = 105 Þ x = 35 men
1 1 15. (c); Let A can do in (x – 5) days and B in x days.
= 15 + 1 + = 16 days
2 2 1 1 9 é 1 100 ù
\ + = ê since, 11 = days ú
12. (a); Let the total work be completed in x days. x - 5 x 100 ë 9 9 û
Then, as per question
Work of A for 2 days + work of B for (x – 3) days x+x-5 9 2x - 5 9
or, = Þ 2 =
+ work of C for x days = 1 x ( x - 5) 100 x - 5x 100

1 1 1 or, 9x2 – 45x = 200x – 500 Þ 9x2 – 245x + 500 = 0


2´ + ( x - 3) ´ +x´ =1
10 12 15 or, 9x2 – 225x – 20x + 500=0 Þ(9x – 20)(x – 25)= 0
or, 9x – 20 ¹ 0 but x = 25
5 (x - 3) + 4 ´ x 1 4
= 1- Þ 9x - 15 = ´ 60 = 48 \ time taken by B alone to do the same work
60 5 5 = 25 days.
( 48 + 15) 16. (d); Let A worked for x days.
x= = 7 days
9 1
A's 1 day's work =
13. (b); Let x men are employed at first. 45
1 x
x men do of the work in 24 days \ A's x day's work =
2 45
\ 1 man do the whole work in
1
24 × 2 × x = 48x days. B's 1 day's work =
40
Now, (x + 16) men do the remaining work
1 x
æ 1 1ö \ B's x day's work = ´x=
çè 1 - = ÷ø in (40 – 24 = 16) days. 40 40
2 2
(A + B) together worked for x days.
\ 1 man do the whole work in
16 × 2 (x + 16) days. x x
\ (A + B)'s x day's work = +
or, 48x = 32 (x + 16) \ x = 32 men. 45 40
14. (c); According to the formula, 8x + 9x 17x
= =
M 1 D 1H1 M 2 D 2 H 2 360 360
=
W1 W2
17x 360 - 17x
\ Remaining work = 1 - =
360 360
2
Given, M1 = 105, D1 = 25, H1 = 8, W1 =
5 360 - 17x
This part of work, i.e., is completed by
Now, let the additional men be x. 360
Then, M2 = 105 + x, H2 = 9, D2 = 25 B alone in 23 days.

252 @BEST300MCQ For More Study Material


Visit: studyiq.com
360 - 17x 4
\ = 23 ´ B's 1 day's work W1 = , M 2 = ( 234 + x) , D 2 = 26,
360 7

360 - 17x 1 23 3
= 23 ´ = Þ 360 – 17x H 2 = 18, W2 =
360 40 40 7
According to the question,
23 M 1W2H1D1 = M2W1H2D2
= ´ 360 = 207 Þ 17x = 360 – 207 = 153
40
3 4
234 ´ ´ 16 ´ 66 = ( 234 + x) ´ ´ 18 ´ 26
153 7 7
Þ x= = 9 days
17
3 ´ 66 ´ 16 ´ 234
Hence, A worked for 9 days. Þ 234 + x =
4 ´ 26 ´ 18
17. (d); 8(4m + 6w) = 10 (3m + 7w)
234 + x = 36 × 11 = 396 Þ x = 396 – 234 = 162
32m + 48w = 30m + 70w
Additional men to be employed = 162
2m = 22w Þ 1m = 11w Þ 4m + 6w = 50w 20. (b); Shortcut:
\ M1D1 = M2D2 Þ 50 × 8 = 20 × D2
A 15 8 units/day
50 ´ 8 LCM
D2 = = 20 days.
20 B 30 120 4 units/day
18. (b); Shortcut: C 40 (Total work) 3 units/day

A+B 5 3 units Here, Total work = 120 units


LCM
Let A and B do not left the job and add the work
15
done by A in 2 days and by B in 4 days.
B 3 (Total work) 5 units \ Now, Total work = 120 + 8(2) + 4(4)
2A+
3
= 120 + 32 = 152 units
Therefore, Total work = 15 units 152 2
A+B=3 ... (i) Total work completed in = = 10 days
15 15
B 1 1 5
2A + =5 ... (ii) 21. (a); Work done by A and B in 1 day = + =
3 8 12 24
12 10
On solving (i) and (ii), A = 2 day's work of A and B =
5 24
After 2 day's A left the work
12
One day work of A is units
5 10 14
\ Remaining work = 1 - =
24 24
15
\ Total work will be completed by A in =
12 1 1 9
One day work of B and C together = + =
5 12 15 60
So, the number of days required by B and C to
15 ´ 5 25 1
= = = 6 days 14
12 4 4
14 60 35
finish work = 24 = ´ =
æ 4ö 3 9 24 9 9
19. (a); Remaining work = ç 1 - ÷ =
è 7ø 7 60

Remaining period = (92 – 66) = 26 days \ Total days to complete the work
Let the number of additional men = x 35 53 8
= 2+ = = 5 days
Given, M1 = 234, D1 = 66, H1 = 16, 9 9 9

253 @BEST300MCQ For More Study Material


Visit: studyiq.com
From Eq. (i), in 1 day,
3
22. (a); Work of a man for 1 hour = × women's work
2 2
(4 men + 6 women) = (4 men + 6 × men)
for 1 hour. Again, work of a man for 1 day 5

æ3 9 ö 32
=ç ´ = men get Rs. 320
è 2 7.5 ÷ø × women's work for 1 hour 5

9 320 ´ 5
Þ Work of a man for 1 day = × women's \ In 1 day, 1 man get = = Rs. 50
5 32

9 2
work for 1 day Þ 1 man = women \ In 1 day, 1 woman get = 50 ´ = Rs. 20
5 5
\ In 1 day, (7 men + 6 women) get
9 7 × 50 + 6 × 20 = Rs. 470
\ 10 men + 6 women = 10 ´ + 6 = 24 women
5
3760
9 \ Required number of days = = 8 days
\ 10 men + 9 women = 10 ´ + 9 = 27 women 470
5
24. (b); Ratio of number of men, women and children
Now, M1 × D1 = M2 × D2 Þ 18 × 24 = D2 × 27
18 10 12
Þ D2 = 16 days = : : = 3x : 2x : 4x
6 5 3
23. (b); In 5 days, (4 men + 6 women) get Rs. 1600
\ In 1 day, (4 men + 6 women) get (3x + 2x + 4x) = 18 Þ x = 2
Therefore, number of women = 4
1600
= Rs. 320 ...(i) 10
5 Share of all women = ´ 4000 = Rs. 1000
40
In 1 day, number of persons to get Rs. 1
(Since, 18 + 10 + 12 = 40)
320
= ...(ii) 1000
4 men + 6 women \ Share of each woman = = Rs. 250
4
Similarly, in second codition,
25. (a); (5M + 3B) can reap 23 hectares in 4 days
In 1 day, number of persons to get Rs. 1
(3M + 2B) can reap 7 hectares in 2 days
1740
= 4 ( 5M + 3B) 2 ( 3M + 2B)
6 ( 3 men + 7 women ) = Þ 1M = 4B
23 7
290 Now, 5M + 3B = 23B
= ...(iii)
+
3 men 7 women i.e 23 boys can reap 23 hectares in 4 days.
or, 1 boys can reap 1 hectare in 4 days.
From Eqs. (ii) and (iii), we get
or, 4 boys can reap 1 hectare in 1 day.
320 290
= 4 ´ 45
4 men + 6 women 3 men + 7 women or, boys can reap 45 hectares in 6 days.
6
96 men + 224 women = 116 men + 174 women or, 30 boys can reap 45 hectares in 6 days.
Þ 20 men = 50 women or, 30B = 28B + 2B = 7M + 2B
Men 5 2 \ 2 boys must assist 7 men
Þ = Þ 1 woman = man
Woman 2 5

254 @BEST300MCQ For More Study Material


Visit: studyiq.com
1. (c); X = 6 × 4 = 24 days
3 5
9. (a); Remaining work after 3 days = 1 - =
4 18 6
Y = ´ 12 = 16 days
3
5
1 3 \ Required no of days = = 12 days
Required days = = 9 days 6
1 1 5
+ 25
16 24
20 ´ 18
10 ´ 3
2. (d); Janardon ® = 15 days 10. (d); Let Required no of days = x
2
x-5 x–3 x
3 + + =1
Required days = ´ 15 = 9 days 10 12 15
5
6x - 30 + 5x - 15 + 4x
1 1 1 = 1 Þ x = 7 days
3. (b); C's one day work = - = 60
6 8 24
11. (d); p men working p hours/day for p days
1 1 1
A's one day work = - = produce p units of work.
6 12 12
1 man working 1 hour/day for 1 day produce
24 ´ 12
(A + C) will finish work in = = 8 days p 1
36 3
= 2 units of work.
p p
4. (b); A + B + C = 4 days
n men working n hours a day for n days produce
10 20
B+C= ´2 = days
3 3 n3
units of work.
p2
1 1 20
A= = = = 10 days
1 3 5-3 2 1 7
- 12. (a); Given, P + Q = and Q + R =
4 20 20 6 60
5. (a); (4M + 6W)8 = (2M + 9W)8
2M = 3W 7 ´ 6 42
Work done by Q and R for 6 days = =
60 60
1 1
\ 12W = Þ W=
8 96 42 18 3
Remaining work = 1 - = =
96 1 60 60 10
\ Required no of days = = 5 days
18 3 \ No of days taken by P to complete the work
6. (a); 4 men = 8 women 10
1 man = 2 women, 6 men = 12 women = ´ 3 = 10 days
3
8´ 15
So, 24 women can do work in = = 5 days \ No of days taken by Q to complete the work
24
1
60 ´ 60 = = 15 days
7. (b); = 1 1
3 -
6 10
4
\ No of days taken by R to complete the work
\ x = 20
90 ´ 16 ´ 12 70 ´ 24 ´ 8 1
8. (d); = = = 20 days
1 x 7 1
-
60 15
70 ´ 24 ´ 8 7
x= , x= \ Required no of days = 20 – 10 = 10 days
90 ´ 16 ´ 12 9

255 @BEST300MCQ For More Study Material


Visit: studyiq.com
13. (b); A : B
20. (c); 75 –15
Time 2 : 3
\2 ® 8
1140
8 = 57
1® = 4 20
2
3 ® 4 × 3 = 12 days
14. (a); A : B : C 72 : 18
Time 4x : 2x : x 1
1 1 1 \ No of days the labour absented
\ Ratio of their efficiencies = : :
4x 2x x
1
=1:2:4 = ´ 20 = 4 days
5
15. (d); A : B
Efficiency 3 : 1 21. (b); A : B+C B : A+C
Time 1 : 3
Time 3 : 1´5 4 : 1´4
\No of days taken by both to complete the work
Time 15 : 5 16 : 4
5 45 Efficiency 5 : 15 4 : 16
= ´9 = days
2 2
Total work = 24 × (20)
1 1 2
\ + = , x = 30 days 24 ´ 20
x 3x 45 \ Time taken by A = = 96 days
5
\ No of days by B to complete the work
= 30 × 3 = 90 days 1 1 x 4+3+x
16. (a); x = 4 pages/hr 22. (d); + + =4 Þ = 4 Þ x = 41
3 4 12 12
x + y = 5 pages/hr
\ y = 5 – 4 = 1 pages/hr 23. (b); A : B+C : A+C
\ Required time = 20 hours Time 4 : 3 : 2
17. (b); (x + 4) (x + 5) = (x – 5) (x + 20)
Efficiency 3 : 4 : 6
x2 + 9x + 20 = x2 + 20x – 5x – 100
Let total efficiency = 7 units
9x + 20 = 15x – 100
6x = 120, x = 20 \ efficiency of A = 3 units
18. (d); A = 8h extra \ efficiency of C = 3 units
\ efficiency of B = 1 units
1 9 A ® 4 hours
B = 4 h = h extra
2 2 \ B ® 4 × 3 = 12 hours
9 1 1 1
\ Required time = ab = 8´ =6h 24. (c); A + B = , B +C = , C +A =
2 30 24 20

32 16 1 æ 1 1 1 ö 1 æ 15 ö 1
19. (c); Dinesh = pages/hr = pages/hr (A + B + C) = ç + + ÷= ç ÷=
6 3 2 è 30 24 20 ø 2 è 120 ø 16

40 10
Rakesh = = 8 pages/hr 10 days work =
5 16
16 40 10 6 3
Dinesh + Rakesh = +8 = pages/hr Remaning work = 1 - = =
3 3 16 16 8
110 1 1 1
\ Required time = ´3 And, A = – =
40 16 24 48
1 3
=8 hr = 8 hrs. 15 min. \ Required No of days = ´ 48 = 18 days
4 8

256 @BEST300MCQ For More Study Material


Visit: studyiq.com
1 1 1 1 5-3
25. (a); A = ,B= \ 3W = – =
42 56 96 160 480

8+6 14 1 1
\A + B = = = W=
7 ´6´ 8 7 ´6´ 8 24 720

24 1 1 1 1
\ Required time = = 3 days 2M = - = Þ M=
8 120 240 240 480
26. (d); Ratio of their efficiency = 4 : 3 : 2 1 1 2 1 1
Total work ratio = 7 × 4 : 3 × 8 : 2 × 10 8B = – = = Þ B=
80 240 240 120 960
= 28 : 24 : 20
1 480 7
28 \ Required time = = = 43
\ Required wages = ´ 369 = 143.50 Rs. 5 12 11 11
72 +
480 960
27. (a); Initial wages = 15 × 22
Wages after reduction = 11 × 25 24 ´ 25 (25 + x) 12
\ Required Ratio = 3 : 22 : : 11 : 5 = 6 : 5 31. (c); =
1 2
4 1 3 3
28. (b); A's 4 days work = =
8 2 25 + x = 4 × 25 Þ x = 75
32. (c); A : B
1 1
Remaining work = 1 - = Time 1 : 6
2 2 Work 1 : 2
1 1 8-3 5 Total time 2 : 6 = 1 : 3
(A + B)'s 1 day destruction = - = =
3 8 24 24 1 1 1 40
\ + = Þ x=
10 x 3x 10 3
(A + B)'s is 2 days destrution = \ Required days = 3x
24
= 40 days
11 33. (d); Ratio of their efficiency = 5 : 4 : 2
\ Remaning work after destruction =
12 one day work of 2 men = 10 units
one day work of 3 women = 12 units
11
one day work of 4 children = 8 units
22 1
\ Required days = 12 = = 7 days Let time taken is D
1 3 3
8 (10 + 12 + 8) ´ 10 éë( 6 ´ 5 ) + ( 4 + 4 ) + ( 7 ´ 2 ) ûù ´ D
=
29. (a); 7M = 10W ; 14M = 20W 10 16

10 ´ 10 40 ´ x 60 ´ D
\ = Þ x = 15 30 = Þ D = 8 days
100 600 16
34. (b); A : B
1
30. (c); M + 3W + 4B = ...(i) Time 1 : 3, work 1 : 2, Time 2 : 3, Work 3 : 2
96
\ Total work = 10 × 5 = 50 units
1 \ B alone complete the whole work
2M + 8B = ...(ii)
80 50
= = 25 days
1 2
2M + 3W = ...(iii) 35. (b); A : B
120
Work 13 : 10, Time 10 : 13
5M + 12B = ?
\ (A + B) will complete the work in
From eqn (ii)
Total work 13 ´ 23
1 = = = 13 days
M+4B= Efficiency of(A + B) 10 + 13
160

257 @BEST300MCQ For More Study Material


Visit: studyiq.com
36. (b); 4M = 6W, 2M = 3W
1 1
40. (b); Men = , Women =
4M ´ 12 ´ 7 12M ´ 8 ´ D 12 ´ 8 12 ´ 16
=
1 2
16 ´ 3 1
D = 7 days \ 16 men's work of 3 days = =
12 ´ 8 2
37. (c); Let Required days = x
1 1
x 17 + x 5x + 68 + 4x Remaining work = 1 - =
\ + =1 Þ =1 2 2
28 35 7´4´5
9x + 68 = 140, x = 8 days 1
22 \ Required no. of days = 2
38. (d); In 22 days, work completed = 6 4
24 +
12 ´ 8 12 ´ 16
22 1 = 6 days
Remaining work = 1 – =
24 12
41. (a); A + B 12
1 4
part of the work is done by boy in = 6 day
12 3 48
\ Boy = 72 days
B+C 16
1 72 A.T.Q.
\ Man = = = 36 days A does for 5 days
1 1 2
- and B does for 7 days
24 72 Þ (A + B) does 20 work in 5 days
Now C does for 13 days the remianing work
4 1 and (B + C) does 6 work in 2 days
39. (d); (A + B) 4 days work = =
8 2 Þ C does 22 work in 11 days
Þ Efficiency of C = 2 and of B = 1
2 1
B's 2 days work = = 48
12 6 Þ B will alone do in = = 48 days.
1
æ 1 1ö 1 42. (b); Let efficiency of B =x
\ Remaining work = 1 - ç + ÷ = Þ efficiency of (A + C) = 3x
è2 6ø 3 and let efficiency of C = x
Þ efficiency of (A + B) = 2x
1 Equating the efficiencies and balancing them
\ 3 = 4 days that C require. Efficiency of A = 5, B = 3, C = 4
1 Total work = 12 × 10 = 120
12 120
Time taken by A alone = = 24 days
5

258 @BEST300MCQ For More Study Material


Visit: studyiq.com
Chapter
Pipe and Cistern
11
Pipes and Cistern problems generally consist of a cistern (tank) to which one or more pipes fill the cistern or empty the
cistern. These problems of pipes and cisterns can be solved by using the same method used in time and work. And we
changes our formulae according to the requirement of the pipes and cisterns.
(i) A pipe connected with a tank or a` cistern that fill the tank is known as inlet.
(ii) A pipe connected with a tank that empty it is known as outlet.
Important Points:
1
1. If a pipe can fill a tank in x hours, then the part filled in 1 hour =
x
1
2. If a pipe can empty a tank in y hours, then the part of the full tank emptied in 1 hour =
y
3. If a pipe can fill a tank in x hours and another pipe can empty the full tank in y hours (y > x), then the net part
æ1 1ö y-x
filled in 1 hour, when both the pipes are opened = ççç - ÷÷÷ =
çè x y ÷ø xy

xy
Time taken to fill the tank =
y-x
4. If a pipe can fill a tank in x hours and another pipe can fill the same tank in y hours, the part of the tank filled
æ 1 1ö x + y
in 1 hour when both pipes are opened simultaneously = ççç + ÷÷÷ =
çè x y ø÷ xy

xy
\ Time taken to fill completely the tank when both pipes are open simultaneously =
x+y
5. If three pipes can fill a tank separately in x, y and z h respectively, then time taken to fill the tank by working
xyz
together = h.
xy + yz + zx
6. If a pipe fills a tank in x hours and another fills the same tank in y hours, but a third pipe empties the full tank
æ 1 1 1ö
in z hours and all of them are opened together, the net part filled in 1 hour = ççç + - ÷÷÷ .
çè x y z ÷ø

xyz
Time taken to fill the tank = hours
yz + xz - xy
Example: Pipe A can fill a water tank in 25 minutes, Pipe B can fill the tank in 40 minutes and Pipe C can empty the
tank in 30 minutes. If all the 3 pipes are opened together, then in how many minutes will the tank be
completely filled?
25 ´ 40 ´ 30 600 11
Sol. Time taken to fill the tank = = = 31 minutes
40 ´ 30 + 25 ´ 30 - 25 ´ 40 19 19
7. Two pipes A and B can fill a tank in x minutes and y minutes respectively. If both the pipes are opened
simultaneously, Then the time after which pipe B should be closed, so that the tank is full in 't' minutes is
é æ t öù
ê y çç1 - ÷÷ú minutes.
êë èç x ø÷úû

259 @BEST300MCQ For More Study Material


Visit: studyiq.com
Example: Two pipes A and B can fill a tank is 12 minutes and 16 minutes respectively. If both the pipes are opened
simultaneously, after how much time should B be closed so that the tank is full is 9 minutes?
æ 9ö 3
Sol. Pipe B should be closed after = ç 1 - ÷ ´ 16 = ´ 16 = 4 min .
è 12 ø 12

1. Pipe A can fill the tank in 80 minutes and pipe B in Shortcut:


120 minutes. Then after how much time both the pipe
can together fill the tank? A 10 1 units/hour
LCM
Sol. Part filled by two taps in 1 minutes 10
1 1 3+2 5 1 – 2 units/hour
= + = = = B 5
80 120 240 240 48
Take negative sign when a pipe empty the tank
\ Time taken to fill the tank = 48 minutes
Shortcut: 10
therefore tank will be empty in = = 10 hours
1–2
A 80 3 units/min
LCM 4. Two pipe P and Q can fill a cistern in 12 and 15 min,
240 respectively. If both are opened together and at the
end of 3 min the first is closed. How much extra time
B 120 (Total Capacity) 2 units/min will Q take to fill the cistern?
\ Take LCM as the total capacity of the tank. Sol. Given, time taken by P to fill the tank = 12 min.
And time taken by Q to fill the tank = 15 min.
240
Time taken to fill the tank = = 48 minutes 1 1
3+2 Part filled by both pipes in 1 min = +
12 15
2. Two pipes A and B can empty a full tank in 20 hours
and 25 hours respectively. In how many hours will 5+ 4 9
tank be emptied when they are opened together? = =
60 60
1 1 5+ 4 9
Sol. Efficiency of both pipes = + = = 3 ´ 9 27 9
20 25 100 100 Now, part filled in 3 min = = =
60 60 20
100 1
\ Tank will be emptied in = 11 hours 9 11
9 9 \ Remaining part = 1 - =
20 20
Shortcut:
Now, the remaining part is filled by pipe Q in x min.
A 20 5 units/hour 11 3 ´ 11 33 1
LCM x= ´ 15 = = = 8 min
100 20 4 4 4
Shortcut:
B 25 4 units/hour
P 12 5 units/min
A can empty 5 units/hour and B can empty 4 units/ LCM
hours. 60
100 1 4 units/min
Time taken to empty the tank = = 11 hours Q 15
9 9
Total capacity=60
3. A can fill a cistern in 10 hours and B can empty it in 5
hours. If they are opened together, in how many hours Part filled by (P+Q) in 3 min = 3(5 + 4) = 27 units
will cistern be empty? Remaining part is filled by Q only

10 ´ 5 10 ´ 5 60 - 27 33 1
Sol. Required time = = = 10 hours. = = = 8 min
10 – 5 5 4 4 4

260 @BEST300MCQ For More Study Material


Visit: studyiq.com
5. A tank can be filled by two pipes A and B in 20 min 7. Pipe A can fill a cistern in 5 hours and B can fill it in
and 30 min, respectively. When the tank was empty 20 hours. Both pipes are turned on but there is a
the two pipes were opened. After some time, the first leakage in the bottom of the cistern. So the cistern is
pipe was stopped and the tank was filled in 18 min. filled in 30 minutes more. In how many time will
After how much time of the start was the first pipe leakage emptie the full cistern?
stopped?
1 1 4 + 1 15 1
Sol. Given, tank filled by A=20 min Sol. Efficiency of pipe A and B = + = = =
5 20 20 20 4
And tank filled by B=30 min
\ Pipe A and B working together will fill the cistern
Let the first pipe be closed after x min. in 4 hours
x But time taken with leakage = 4 hours + 30 minutes
Then, work done by first pipe in x min =
20 æ 1ö 9
= ç 4 + ÷ hours = hours
According to the question, è 2ø 2
x 18 x 18 x 3 2 1 1 1 2 1 1 2
+ =1 Þ = 1- Þ = 1- = + - = Þ + -
20 30 20 30 20 5 5 5 20 x 9 5 20 9

2 1 36 + 9 - 40 5 1
\ x= ´ 20 = 8 min = = = Þ x = 36hours
5 x 180 180 36
Shortcut: \ Leakage will empty the cistern in 36 hours
Shortcut:
A 20 3 units/min
LCM A 5 4
LCM
60
20
B 30 (Total Capacity) 2 units/min
B 20 (Total Capacity) 1
B work all time, therefore part filled by B in 18 min
= 2 × 18 = 36 units 20
A and B fill the cistern in = 4 hours
5
24
Remaining part is filled by A in = 8 min But when there is a leakage in the bottom then the
3
æ 1ö 9
Therefore, after 8 min the first pipe is closed. tank is filled in ç 4 + ÷ = hours
è 2ø 2
6. Pipe A can fill a cistern in 20 minutes and pipe B can
fill it in 30 minutes. While pipe C can empties the full A+B 4 x
cistern in 1 hours. If all the pipes be turned on at the
same time, in how much time will the cistern be full? 4x
Sol. Filled part of cistern in 1 minute, when A, B and C be
Leakage x –4
1 1 1 3+2-1
turned on at same time = + - =
20 30 60 60 4x 9
= Þ x = 36 hours
4 1 x-4 2
= = 8. Two pipes can fill a cistern in 20 minutes and 25
60 15
minutes respectively. These are opened alternatively
\ time taken to fill the cistern = 15 minutes
for 1 minutes each, beginning with first. In how much
Shortcut: time will the cistern be full?
A 20 3 Sol. Filled cistern in 1 round (or 2 minutes)
LCM
1 1 5+ 4 9
B 30 60 2 = + = =
20 25 100 100
C 60 (Total Capacity) –1 æ 9 ö
çè Þ 100 ¸ 9 gives 11 round complete÷
100 ø
Therefore Cistern will be filled in
\ Filled cistern in 11 round ( or 22 minutes)
60 60 9 99
= = 15 minutes = ´ 11 =
3+2-1 4 100 100

261 @BEST300MCQ For More Study Material


Visit: studyiq.com
99 1 1
\ Empty part of cistern = 1 - = Part of pool emptied in 1 h by A =
100 100 x
On 23rd min., it is first pipe's is opened. \ Time taken by pipe
\ Time taken to fill empty part by first pipe
2 2 4x
C= = = h
1 1 1 1+ 2 3
+
empty part 100 2x x 2x
= =
efficiency of first pipe 1 Now, part of the tank filled by all together
20
1 1 3 4+2+3 1
= + + = =
1 20 1 x 2x 4x 4x 2
= ´ = minutes 6+
100 1 5 3

1 1 4+2+3 3 9 ´ 20
\ Total time = 22 + = 22 minutes = = = 9 ´ 20 = 4x ´ 3 Þ x = = 15 h
5 5 4x 20 4´3
Shortcut: 10. A tank has a leak which would empty the completely
filled tank in 10 h. If the tank is full of water and a tap
A 20 5
is opened which admits 4 L of water per minutes in
100 the tank, the leak takes 15 h to empty the tank. How
many litres of water does the tank hold?
B 25 4 Sol. Let the capacity of the tank=x L
A and B opened alternatively. According to the question,
\ In 2 minutes filled is 9 units. x
Quantity of water emptied by the leak in 1h = L
Minutes Tank Filled 10
2 9 1
×11 ×11 Quantity of water emptied with inlet pipe in 1 h =
15
22 99
Let Inlet pipe fills tank in x h
Remaining part = 100 – 99 = 1 unit
1 1 1 1 1 1 3-2 1
1 So, - =- Þ = - Þ =
1 unit is filled by A in = minutes x 10 15 x 10 15 30 30
5
So, Inlet pipe fills tank in 30 h
1 1 Capacity of tank = 30 × 60 × 4 = 7200 L.
Total time = 22 + = 22 minutes
5 5 Shortcut:
9. A swimming pool has 3 drain pipes. The first two A 10 –3
pipes A and B, operating simultaneously can empty LCM
the pool in half the time that C, the 3rd pipe, alone 30
takes to empty it. Pipe A, working alone, takes half
the time taken by pipe B. Together they take 6 h 40 A+B 15 –2
min to empty the pool. Time taken by pipe A to empty
Therefore B admit 1 unit/hour and originally B admit
the pool, in hours, is
4 L/min. Therefore
Sol. Let the time taken by pipe B = 2x h
1 = 4 × 60 Þ 1 º 240
1 Total Capacity = 30 × 240 = 7200 L
Part of the pool emptied by B in 1 h =
2x

262 @BEST300MCQ For More Study Material


Visit: studyiq.com
Ques ti ons 8. Three taps A, B and C together can fill an empty cistern
in 10 min. The tap A alone can fill it in 30 min and the
tap B alone can fill it in 40 min. How long will the tap
1. If a pipe fills a tank in 6 h, then what part of the tank C alone take to fill it?
will the pipe fill in 1 h?
(a) 16 min (b) 24 min
1 1 (c) 32 min (d) 40 min
(a) (b) 9. Two pipes A and B can fill a tank in 1 h and 75 min,
3 6
respectively. There is also an outlet C. If all the three
1 1 pipes are opened together. The tank is full in 50 min.
(c) (d)
4 5 How much time will be taken by C to empty the full
tank?
1 (a) 100 min (b) 150 min
2. An inlet pipe fills part of a tank in 1 h. How much
8 (c) 200 min (d) 125 min
time will the pipe take to fill the empty tank? 10. A, B and C are three pipes connected to a tank. A and
(a) 4h (b) 2h B together fill the tank in 6 h, B and C together fill the
(c) 6h (d) 8h tank in 10 h and A and C together fill the tank in 12 h.
3. An outlet pipe can empty a cistern in 3 h. In what time In how much time A, B and C fill up the tank together?
will the pipe empty two-third part of the cistern? 3
(a) 4h (b) 2h (a) 9 h (b) 5 h
7
(c) 3h (d) 5h
4. There are two taps A and B to fill up a water tank. The 2 5
(c) 5 h (d) 5 h
tank can be filled in 40 min, if both taps are on. The 7 7
same tank can be filled in 60 min, if tap A alone is on. 11. Inlet A is four times faster than inlet B to fill a tank. If
How much time will tap B alone take, to fill up the A alone can fill it in 15 min, how long will it take if
same tank? both the pipes are opened together?
(a) 64 min (b) 80 min (a) 10 min (b) 12 min
(c) 96 min (d) 120 min (c) 15 min (d) 14 min
5. A pipe can fill a tank in 10 h, while an another 12. There are two inlets A and B connected to a tank. A
pipe can empty it in 6 h. Find the time taken to empty and B can fill the tank in 16 h and 10 h, respectively.
the tank, when both the pipes are opened If both the pipes are opened alternately for 1 h, starting
simultaneously? from A, then how much time will the tank take to be
(a) 11h (b) 15h filled?
(c) 18h (d) 16h 1 6
6. Three taps are fitted in a cistern. The empty cistern is (a) 13 h (b) 11 h
4 8
filled by the first and the second taps in 3 and 4h,
respectively. The full cistern is emptied by the third 2 1
tap in 5 h. If all three taps are opened simultaneously, (c) 12 h (d) 12
h
5 4
the empty cistern will be filled up in?
13. A pipe can empty a cistern in 27 hours. Find the time
14 14 2
(a) 1 h (b) 2 h
23 23 in which part of the cistern will be emptied?
3
(c) 2 h 40 min (d) 1 h 56 min (a) 9 hours (b) 12 hours
7. Pipe A can fill a tank in 30 min, while pipe B can fill (c) 15 hours (d) 18 hours
the same tank in 10 min and pipe C can empty the full
tank in 40 min. If all the pipes are opened together, 2
14. A water tank is rd full. Pipe A can fill the tank in 10
how much time will be needed to make the tank full? 3
minutes and the pipe B can empty it in 6 minutes. If
3 4
(a) 9 h (b) 9 h both the pipes are open, how long will it take to empty
13 13 or fill the tank completely?
7 9 (a) 6 minutes to empty (b) 6 minutes to fill
(c) 9 h (d) 9 h (c) 9 minutes to empty (d) 10 minutes to empty
13 13

263 @BEST300MCQ For More Study Material


Visit: studyiq.com
15. A tap can fill a cistern in 8 hours and another can (a) 110 min (b) 70 min
empty it in 16 hours. If both the taps are opened (c) 120 min (d) 90 min
simultaneously, the time (in hours) to fill the tank is?
(a) 8 (b) 10 1
24. Two pipes A and B can fill a tank in 37 min and 45
(c) 16 (d) 24 2
16. A pipe can empty a tank in 15 hrs and another pipe min respectively. If both the pipes are opened together,
can empty it in 10 hours. If both the pipes are opened then after how much time should B be closed so that
simultaneously, find the time in which a full tank is the tank is full in half hours?
emptied? (a) 15 min (b) 10 min
(a) 8 hrs (b) 6 hrs (c) 21 min (d) 9 min
(c) 4 hrs (d) 5 hrs 25. A big tanker can be filled by two pipes A and B in 60
17. Two pipes A and B can fill a cistern in 20 minutes and 40 min respectively. Then what time it will take
and 25 minutes respectively. Both are opened together, to fill an empty tanker if tap B is used half of time and
but after 5 minutes, B is turned off. How much longer tap A and B together are used rest half of time?
will the cistern take to fill?
(a) 24 min (b) 30 min
(a) 16 minutes (b) 18 minutes
(c) 18 min (d) 32 min
(c) 11 minutes (d) C.N.D.
18. 12 pumps working 6 hours a day can empty a 26. Two taps A and B can fill an oil tank in 15 min and 18
completely filled reservoir in 15 days. How many such min respectively while a third tap is used to empty
pumps working 9 hours a day will empty the same the tank. The third tap is opened after 6 min of
reservoir in 12 days? 1
opening of tap A and B. If the tank empties in 16
(a) 15 (b) 9 2
(c) 10 (d) 12 min after opening the third tap then in how much
19. A tank has a leak which would empty it in 8 hrs. A time the third tap can empty the full tank?
tap is turned on which admits 6 litres a minutes into (a) 6 min (b) 12 min
the tank, and it is now emptied in 12 hrs. How many
(c) 10 min (d) 15 min
litres does the tank hold?
(a) 8260 litres (b) 8450 litres 27. Three pipes A, B and C can fill a tank in 6 hours. After
running them together for two hours the tap C is
(c) 8640 litres (d) 8660 litres
closed and A and B fills it completely in 7 hours. How
20. A tap can fill the cistern in 8 hours and another can
much time would C take to fill the tank alone?
empty it in 16 hours. If both the taps are opened
simultaneously, the time (in hours) to fill the tank is? (a) 7 hours (b) 10.5 hours
(a) 16 hrs (b) 8 hrs (c) 14 hours (d) 21 hours
(c) 10 hrs (d) 12 hrs 28. A water tank has three taps A, B, C. Tap A, when opened,
21. A pipe can fill a tank with water in 5 hours and an can fill the water tank alone in 4 hours. Tap B, when
another pipe can empty the same tank in 4 hours. If opened, can filled the water tank alone in 6 hours and
the tank is completely filled and both the taps are tap C, when opened, can empty the water tank alone in
opened together than the tank will be empty in 3 hours. If taps A, B and C are opened simultaneously
following time (hour)? how long will it take to fill the tank completely?
(a) 9 (b) 18 (a) 10 hours (b) 8 hours
1 (c) 18 hours (d) 12 hours
(c) 20 (d) 20 29. A water tank has two taps A and B. A can fill it in 6
2
hours, and tap B empty it in 5 hours. If Both the taps
22. Two pipes can fill a tank respectively in 15 and 12
open simultaneously the time required to empty the
hours while a third pipe can empty it in 20 hours. If
tank?
the tank is empty and all three pipes are opened
together then in how much time will it take to fill the (a) 20 hours (b) 30 hours
tank completely? (c) 25 hours (d) 35 hours
(a) 10 hours (b) 5 hours 30. 18 pumps can fill reservoir of capacity 1440 kilo liter
(c) 15 hours (d) 20 hours in 5 hours. Then how many hours required to fill a
23. 2 pipes A and B can fill a tank separately in 60 min and reservoir of capacity 1920 kilo liter by 8 pumps?
70 min respectively. There is a third pipe attached to (a) 10 hours (b) 16 hours
the bottom of the tank to empty it. The tank is filled in (c) 15 hours (d) 20 hours
60 min when all three pipes are opened. In how much
time (min) the third pipe alone take to fill the tank?

264 @BEST300MCQ For More Study Material


Visit: studyiq.com
1. Two pipes can fill a cistern in 14 h and 16 h, 8. Two taps A and B can fill a tank in 25 min and 20
respectively. The pipes are opened simultaneously min, respectively. But taps are not opened properly,
and it is found that due to leakage in the bottom, it
5 2
took 92 min more to fill the cistern. When the cistern so the taps A and B allow th and rd part of water,
is full, in what time will the leak empty it? 6 3
respectively. How long will they take to fill the tank?
15 17 (a) 12 min (b) 13min
(a) 43 h (b) 43 h
23 23 (c) 14 min (d) 15 min
13 19 9. Three taps A, B and C fill a tank in 20 min, 15 min and
(c) 43 h (d) 43 h 12 min, respectively. If all the taps are opened
23 23
simultaneously, how long will they take to fill 40% of
2. A pipe P can fill a tank in 12 min and another pipe R the tank?
can fill it in 15 min. But, the 3rd pipe M can empty it in (a) 1 min (b) 2 min
6 min. The 1st two pipes P and R are kept open for (c) 3 min (d) 4 min
5 min in the beginning and then the 3rd pipe is also
10. A pipe can fill a cistern in 4 min and another pipe can
opened. In what time is the tank emptied?
fill it in 5 min, but a third pipe can empty it in 2 min.
(a) 30 min (b) 25 min The first two pipes are kept open for 2 min in the
(c) 45 min (d) 35 min beginning and then the third pipe is also opened.
3. A tap having diameter ‘d’ can empty a tank in 40 min. Time taken to empty the cistern is?
How long another tap having diameter ‘2d’ take to (a) 20 min (b) 22 min
empty the same tank? (c) 42 min (d) 18 min
(a) 5 min (b) 20 min 11. Taps A, B and C attached with a tank and velocity of
(c) 10 min (d) 40 min water coming through them are 42 L/h, 56 L/h and
4. If two pipes function together, the tank will be filled 48 L/h, respectively. A and B are inlets and C is outlet.
in 12 h. One pipe fils the tank in 10 h faster than the If all the taps are opened simultaneously, tank is filled
other. How many hours does the faster pipe take to in 16 h. What is the capacity of the tank?
fill up the tank? (a) 2346 L (b) 1600 L
(a) 20 h (b) 60 h (c) 800 L (d) 960 L
(c) 25 h (d) 35 h 12. A boy and girls together fill a cistern with water. The
5. Two pipes A and B can fill a cistern in 15 and 20 min, boy pours 4 litres of water every 3 minutes and the
respectively. Both the pipes are opened together, but girl pours 3 litres every 4 minutes. How much time
after 2 min, pipe A is turned off. What is the total time will it take to fill 100 litres of water in the cistern?
required to fill the tank? (a) 36 minutes (b) 42 minutes
46 52 (c) 48 minutes (d) 44 minutes
(a) min (b) min 13. A tap takes 36 hours extra to fill a tank due to a leakage
3 3
equivalent to half of its inflow. The inlet pipe alone
43 41 can fill the tank in how many hours?
(c) min (d) min. (a) 36 (b) 24
3 3
6. Two pipes A and B can fill a tank in 24 and 32 min, (c) 30 (d) 18
respectively. If both the pipes are opened together, 14. A tank be filled by three pipes whose diameter are 1
after how much time pipe B should be closed so that cm, 3 cm and 4 cm running together if all three pipes
the tank is full in 9 min? fill the tank in 16 minutes. find out in what time the
(a) 40 min (b) 30 min third pipe alone will fill the tank. If amount of water
(c) 10 min (d) 20 min flowing from each pipe being proportional to square
of its diameter?
7. Two taps A and B can fill a tank in 20 min and 30
min, respectively. An outlet pipe C can empty the full (a) 26 minutes (b) 416 minutes
tank in 15 min. A, B and C are opened alternatively, (c) 24 minutes (d) 312 minutes
each for 1 min. How long will the tank take to be 15. Two pipes A and B can fill a tank in 36 hours and 45
filled? hours respectively. If both the pipes are opened
(a) 105 min (b) 120 min simultaneously, how much time will be taken to fill
the tank?
(c) 167 min (d) 185 min

265 @BEST300MCQ For More Study Material


Visit: studyiq.com
(a) 10 hrs (b) 15 hrs 18. A cistern can be filled by two pipes A and B in 4 hours
(c) 20 hrs (d) 25 hrs and 6 hours respectively. When full, the tank can be
16. In what time would a cistern be filled by three pipes emptied by a third pipe C in 8 hours. If all the taps be
turned on at the same time, the cistern will be full in?
1
whose diameters are 1 cm, 1 cm, 2 cm, running 2 3
3 (a) 2 hrs (b) 3 hrs
together, when the largest alone will fill it in 61 7 7
minutes, the amount of water flowing in by each pipe 3 5
being proportional to the square of its diameter? (c) 4 hrs (d) 5 hrs
7 7
(a) 32 minutes (b) 34 minutes
19. If two pipes function simultaneously, the reservoir
(c) 36 minutes (d) 38 minutes
will be filled in 6 hours. One pipe fills the reservoir 5
17. Two pipes can separately fill a tank in 20 hrs and 30 hours faster than the other. How many hours does
hrs respectively. Both the pipes are opened to fill the the faster pipe take to fill the reservoir?
1 (a) 8 hrs (b) 10 hrs
tank but when the tank is full a leak develops in the
3 (c) 12 hrs (d) C.N.D.
20. Three pipes A, B and C can fill cistern in 6 hrs. After
1
tank through which of the water supplied by both working together for 2 hours, C is closed and A and B
3 fill the cistern in 8 hrs. Then find the time in which
the pipes per hour leak out. What is the total time the cistern can be filled by pipe C?
taken to fill the tank? (a) 6 hrs (b) 12 hrs
(a) 12 hrs (b) 14 hrs (c) 14 hrs (d) 20 hrs
(c) 16 hrs (d) 18 hrs

1. Two pipes A and B can fill a tank in 24 minutes and (a) 20 min (b) 30 min
32 minute respectively. If both the pipes are opened (c) 45 min (d) 40 min
simultaneously, after how much time should B be
6. A cistern has three pipes A, B and C. Pipes A and B
closed so that the tank is full in 18 minutes?
can fill it in 3 h and 4 h, respectively, while pipe C can
(a) 2 min. (b) 4 min. empty the completely filled cistern in 1 h. If pipes are
(c) 6 min. (d) 8 min. opened in order at 3:00 pm, 4:00 pm and 5:00 pm,
2. If three taps are opened together, a tank is filled in 12 respectively, at what time will the cistern be empty?
hrs. One of the taps can fill it in 10 hrs and another in (a) 6 : 15 pm (b) 7 : 12 pm
15 hrs. How many hours does the third tap take alone
(c) 8 : 12 pm (d) 8 : 35 pm
to empty it completely?
(a) 8 hrs (b) 10 hrs 7. Three pipes A, B and C can fill a tank in 30 min, 20
min and 10 min, respectively. When the tank is empty,
(c) 12 hrs (d) 16 hrs
all the three pipes are opened. If A, B and C discharge
3. A tank can be filled with water by two pipes A and B chemical solutions P, Q and R respectively, then the
together in 36 minutes. If the pipe B was closed after part of solution R in the liquid in the tank after 3 min
30 minutes, the tank is filled in 40 minutes. The pipe is:
B can alone fill the tank in?
(a) 45 minutes (b) 60 minutes 8 5
(a) (b)
(c) 75 minutes (d) 90 minutes 11 11
4. A tank has two pipes. The first pipe can fill it in 45
minutes and the second can empty it in 1 hour. In 6 7
(c) (d)
what time will the empty tank be filled if the pipes be 11 11
opened one at a time in alternate minutes? 8. Three pipes A, B and C can fill a cistern in 6 hrs. After
(a) 2 hrs 55 min (b) 3 hrs 40 min working together for 2 hours, C is closed and A and B
(c) 4 hrs 48 min (d) 5 hrs 53 min fill the cistern in 8 hrs. Then find the time in which
5. A tank can be filled by a tap in 20 min and by another the cistern can be filled by pipe C?
tap in 60 min. Both the taps are kept open for 5 min (a) 14 hrs. (b) 12 hrs.
and then the 1st tap is shut off. After this, how much (c) 16 hrs. (d) 18 hrs.
time the tank will be completely filled?

266 @BEST300MCQ For More Study Material


Visit: studyiq.com
9. A, B and C are three pipes connected to tank. A and B 15. Three taps A, B and C can fill a tank in 12, 15 and 20
together fill the tank in 6 hrs. B and C together fill the hours respectively. If A is open all the time and B and
C are open for one hour each alternately, the tank will
1
tank in 10 hrs. A and C together fill the tank in 7 hrs. be full in?
2
In how much time will A, B and C fill the tank together? 2
(a) 6 hrs (b) 6 hrs
(a) 36 hrs. (b) 32 hrs. 3
(c) 30 hrs. (d) 5 hrs. (c) 5 hrs (d) 7 hrs
10. Two pipes A and B can fill a tank in 15 hours and 20 16. A booster pump can be used for filling as well as for
hours respectively while a third pipe C can empty the emptying a tank. The capacity of the tank is 2400 m3.
full tank in 25 hours. All the three pipes are opened in The emptying capacity of the tank is 10 m3 per minute
the beginning. After 10 hours C is closed. Find, in higher than its filling capacity and the pump needs 8
how much time will the take be full? minutes lesser to empty the tank than it needs to fill it.
(a) 12 hrs (b) 8 hrs What is the filling capacity of the pump?
(c) 10 hrs (d) 14 hrs (a) 50 m3/min (b) 60 m3/min
11. Three pipes A, B and C can fill a cistern in 10 hours, (c) 72 m3/min (d) 32 m3/min
12 hours and 15 hours rerspectively. First A was 17. A leak in the bottom of a tank can empty the full tank
opened. After 1 hour, B was opened and after 2 hours in 8 hours. An inlet pipe fills water at the rate of 6
from the start of A, C was also opened. Find the time litres a minute. When the tank is full, the inlet pipe is
in which the cistern is just full? opened and due to the leak, the tank is empty in 12
(a) 2 hrs (b) 4 hrs hours. How many litres does the tank hold?
(c) 2 hrs 52 min (d) 4 hrs 52 min (a) 7580 (b) 7960
12. A, B, C are pipes attached to a cistern. A and B can fill (c) 8290 (d) 8640
it in 20 and 30 minutes respectively, while C can 18. Two pipes can fill a tank in 20 and 24 minutes
empty it in 15 minutes. If A, B, C be kept open respectively and a waste pipe can empty 3 gallons
successively for 1 minute each, how soon will the per minute. All the three pipes working together can
cistern be filled? fill the tank in 15 minutes. The capacity of the tank is?
(a) 167 min (b) 160 min (a) 60 gallons (b) 100 gallons
(c) 166 min (d) 164 min (c) 120 gallons (d) 180 gallons
13. A tank is filled in 5 hours by three pipes A, B and C. 19. Three pipes A, B and C can fill a tank in 6 hours. After
The pipe C is twice as fast as B and B is twice as fast working at it together for 2 hours, C is closed and A
as A. How much time will pipe A alone take to fill the and B can fill the remaining part in 7 hours. The
tank? number of hours taken by C alone to fill the tank is?
(a) 20 hrs (b) 25 hrs (a) 10 (b) 12
(c) 35 hrs (d) 15 hrs (c) 14 (d) 20
14. Two pipes A and B can fill a tank in 15 minutes and 20. Two pipes A and B can fill a tank in 24 min. and 48
20 minutes respectively. Both the pipes are opened min. respectively. If both the pipes are opened
together but after 4 minutes, pipe A is turned off. What simultaneously, after how much time B should be
is the total time required to fill the tank? closed so that the tank is full in 20 minutes?
(a) 10 min 20 sec (b) 11 min 45 sec (a) 8 (b) 12
(c) 12 min 30 sec (d) 14 min 40 sec (c) 14 (d) 16

1. Three taps A, B and C can fill a tank in 12, 15 and 20 2. A pipe of diameter ‘d’ can drain a certain water tank
hours respectively. If A is open all the time and B and in 40 minutes. The time taken by a pipe of diameter
C are open for one hour each alternatively, the tank ‘2d’ for doing the same job is:
will be full in: (a) 5 minutes (b) 10 minutes
(c) 20 minutes (d) 80 minutes
1
(a) 6 hours (b) 6 hours 3. A pipe can fill a tank with water in 3 hours. Due to a
2
1
1 leakage in bottom, it takes 3 hours to fill it. In what
(c) 7 hours (d) 7 hours 2
2 time the leak will empty the fully filled tank?

267 @BEST300MCQ For More Study Material


Visit: studyiq.com
(a) (x – y) hours (b) (y – x) hours
(a) 12 hours (b) 21 hours
xy xy
1 1 (c) x - y hours (d) y - x hours
(c) 6 hours (d) 10 hours
2 2
4. Two pipes A and B can separately fill a cistern in 60 11. 12 pumps working 6 hours a day can empty a
minutes and 75 minutes respectively. There is a third completely filled reservoir in 15 days. How many such
pipe in the bottom of the cistern to empty it. If all the pumps working 9 hours a day will empty the same
three pipes are simultaneously opened, then the reservoir in 12 days?
cistern is full in 50 minutes. In how much time the (a) 15 (b) 9
third pipe alone can empty the cistern? (c) 10 (d) 12
(a) 110 minutes (b) 100 minutes 12. A tap takes 36 hours extra to fill a tank due to a leakage
(c) 120 minutes (d) 90 minutes equivalent to half of its inflow. The inflow can fill the
5. A tap can fill a tank in 6 hours. After half the tank is tank in how many hours?
filled, three more similar taps are opened. What is the (a) 36 hrs (b) 24 hrs
total time taken to fill the tank completely? (c) 30 hrs (d) 18 hrs
(a) 4 hrs (b) 4 hrs 15 min. 13. A tank can be filled with water by two pipes, A and B
(c) 3 hrs 15 min. (d) 3 hrs 45 min. together in 36 minutes. If the pipe B was stopped after
6. One pipe can fill a tank three times as fast as another 30 minutes, the tank is filled in 40 minutes. The pipe
pipe. If together the two pipes can fill the tank in 36 B can alone fill the tank in:
minutes, the slower pipe alone will be able to fill tank (a) 45 minutes (b) 60 minutes
in (c) 75 minutes (d) 90 minutes
(a) 81 minutes (b) 108 minutes 14. Two pipes A and B fill a water tank in 20 and 24
(c) 144 minutes (d) 192 minutes minutes respectively and a third pipe C can empty at
7. A cistern has two pipes. One can fill it with water in 8 the rate of 3 gallons per minute. If A, B and C are
hours and other can empty it in 5 hours. In how many opened together to fill the tank in 15 minutes, find the
hours will the cistern be emptied if both the pipes are capacity of tank?
(a) 180 (b) 150
3
opened together when of the cistern is already full (c) 120 (d) 60
4
15. Three pipes P, Q and R can separately fill a cistern in
of water?
4, 8 and 12 hours respectively. Another pipe S can
1 empty the completely filled cistern in 10 hours. Which
(a) 13 hours (b) 10 hours of the following arrangements will fill the empty
3
cistern in less time than others?
1 (a) Q alone is open (b) P, R and S are open
(c) 6 hours (d) 3 hours
3 (c) P and S are open (d) P, Q and S are open
8. A tank is fitted with two taps. The first tap can fill the 16. A tank has a leak which would empty the completely
tank completely in 45 minutes and the second tap filled tank in 10 hours. If the tank is full of water and
can empty the full tank in one hour. If both the taps a tap is opened which admits 4 litres of water per
are opened alternately for one minute, then in how minute in the tank, the leak takes 15 hours to empty
many hours the empty tank will be filled completely? the tank. How many litres of water does the tank hold?
(a) 2 hours 55 minutes (b) 3 hours 40 minutes (a) 2400 L (b) 4500 L
(c) 4 hours 48 minutes (d) 5 hours 53 minutes (c) 1200 L (d) 7200 L
9. Two pipes can fill a tank with water in 15 and 12 17. A boy and girl together fill a cistern with water. The
hours respectively and a third pipe can empty it in 4 boy pours 4 litres of water every 3 minutes and the
hours. If the pipes be opened in order at 8, 9 and 11 girl pours 3 litres of water every 4 minutes. How much
a.m. respectively, the tank will be emptied at time will it take to fill 100 litres of water in the cistern?
(a) 11 : 40 a.m. (b) 12 : 40 p.m. (a) 36 minutes (b) 42 minutes
(c) 1 : 40 p.m. (d) 2 : 40 p.m. (c) 48 minutes (d) 44 minutes
10. A pipe can fill a tank ‘x’ hours and another pipe can 18. Three pipes A, B and C can fill a cistern in 6 hours.
empty it in ‘y’ (y > x) hours. If both the pipes are open. After working at it together for 2 hours, C is closed
In how many hours will the tank be filled? and A and B fill it in 7 hours more. The time taken by
C alone to fill the cistern is:

268 @BEST300MCQ For More Study Material


Visit: studyiq.com
(a) 14 hours (b) 15 hours 25. Two pipes X and Y can fill a cistern in 24 minutes
(c) 16 hours (d) 17 hours and 32 minutes respectively. If both the pipes are
19. A tap can fill an empty tank in 12 hours and another opened together, then after how much time (in
tap can empty half the tank in 10 hours. It both the minutes) should Y be closed so that the tank is full in
taps are opened simultaneously, how long would it 18 minutes?
take for the empty tank to be filled to half its capacity? (a) 10 (b) 8
(a) 10 hrs (b) 30 hrs (c) 6 (d) 5
(c) 15 hrs (d) 20 hrs 26. Two pipes A and B can fill a tank in 36 minutes and
20. Two pipes, P and Q can fill a cistern in 12 and 15 45 minutes respectively. Another pipe C can empty
minutes respectively. Both are opened together, but at the tank in 30 minutes. First A and B are opened.
the end of 3 minutes, P is turned off. In how many After 7 minutes, C is also opened. The tank is filled up
more minutes will Q fill the cistern? in:
(a) 39 minutes (b) 46 minutes
1 (c) 40 minutes (d) 45 minutes
(a) 7 minutes (b) 7 minutes
2 27. Having the same capacity 9 taps fill up a water tank
1 in 20 minutes. How many taps of the same capacity
(c) 8 minutes (d) 8 minutes are required to fill up the same water tank in 15
4
minutes?
21. Pipes P and Q can fill a tank in 10 hours and 12 hours (a) 10 (b) 12
respectively and C can empty it in 6 hours. If all the
three are opened at 7 am, at what time will one-fourth (c) 15 (d) 18
of the tank be filled? 28. A cistern is provided with two pipes A and B. A can
(a) 10 am (b) 10 pm fill it in 20 minutes and B can empty it in 30 minutes.
If A and B be kept open alternatively for one minute
(c) 11 pm (d) 11 am each, how soon will the cistern be filled?
3 (a) 121 minutes (b) 110 minutes
22. If th of a cistern is filled in 1 minute, the time needed
5 (c) 115 minutes (d) 120 minutes
to fill the rest is: 29. Pipe A can fill an empty tank in 6 hours and pipe B is
(a) 40 sec (b) 30 sec 8 hours. If both the pipes are opened and after 2 hours
(c) 36 sec (d) 24 sec pipe A is closed, how much time B will take to fill the
remaining tank?
23. A cylindrical cistern of diameter 25 cm is full of water.
If 11 litres of water is drawn off, the water is drawn 1 3
off, the water level in the cistern will drop by? (a) 7 hours (b) 4 hours
2 5
æ 22 ö
çè use p = ÷ø 2 1
7 (c) 2 hours (d) 3 hours
5 3
1 6 30. A water reservoir has two inlets and one outlet.
(a) 10 cm (b) 12 cm Through the inlet it can be filled in 3 hours and 3
2 7
hours 45 minutes respectively. It can be emptied
2 2 completely in 1 hour by the outlet. If the two inlets are
(c) 22 cm (d) 20 cm
5 5 opened at 01:00 pm and 02:00 pm respectively and
24. If two pipes function simultaneously, a tank is filled the outlet at 03:00 pm then it will be emptied at?
in 12 hours. One pipe fills the tank 10 hours faster (a) 05 : 55 pm (b) 05 : 00 pm
than the other. How many hours does the faster pipe (c) 05 : 20 pm (d) 05 : 30 pm
alone take to fill the tank?
(a) 20 hrs (b) 18 hrs
(c) 15 hrs (d) 12 hrs

269 @BEST300MCQ For More Study Material


Visit: studyiq.com
S olut ions 120 3
Required time = =9 h
13 13
1. (b); Time taken to fill the tank = 6 h.
Part of tank filled in 6 h = 1 1 1 1 1 1 1 1 1
8. (b); Given, + + = Þ = , =
1 a b c 10 a 30 b 40
Part of tank filled in 1 h =
6
1 1 1 1 12 – 4 – 3 5
= - - Þ =
1 c 10 30 40 120 120
2. (d); Time taken to fill part of tank = 1 h
8 c = 24 min
\ Time taken to fill the empty tank = 8 h.
9. (a); Given, a = 60, b = 75
3. (b); Time taken to empty a cistern = 3h
2 1 1 1 1 1 1 1 1
Time taken to empty part of the cistern + - = Þ = + -
3 a b c 50 c 60 75 50
2 5+ 4-6 1 3
= 3´ =2h = Þ = Þ c = 100
3 300 c 300
4. (d); Given time taken by tap A to fill the tank = 60 min 10. (d); Let a, b, c be the time taken to fill the tank by
Let Time taken by tap B to fill the tank = x min
pipes A, B and C respectively
1 1 1 1 1 1 3-2
There, + = Þ = - = 1 1 1 1 1 1 1 1 1
60 x 40 x 40 60 120 Given, + = , + = , + =
a b 6 b c 10 a c 12
1 1
= Þ x = 120 min
x 120 æ1 1 1ö 1 1 1 10 + 6 + 5
Then, 2ç + + ÷ = + + =
5. (b); Given pipe A can fill a tank = 10 h è a b c ø 6 10 12 60
Pipe B can empty it = 6 h
1 1 1 1 21
+ + =
Time taken to empty the full tank = 1 1 a b c 120
-
6 10
120 40 5
1 Required time = = =5 h
1 21 7 7
= = = 15 h
5-3 5-3 11. (b); Let 4x is the time taken to fill the tank by inlet B
30 30 Then x will be the time taken to fill tank by inlet A
6. (b); Given,
Þ x = 15
Time taken by first tap to fill the tank (A) = 3 h
Time taken by second tap to fill the tank (B) = 4 h 4+1 1
Part of the tank will fill in 1 min = = min
And time taken to empty the full tank by third 60 12
tap (C) = 5hr Required time = 12 min
\ Part of the tank will be filled by all there taps in 12. (c); A = 16 h, B = 10 h
1 1 1 20 + 15 - 12 23 1 1
1 hr = + - = = h Part of the tank filled in (2h) = +
3 4 5 60 60 16 10
60 14 5 + 8 13
Required time = h= 2 h = =
23 23 80 80
7. (a); Given,
78
A = 30 min, B = 10 min, C = 40 min Part of the tank filled in (12 h) =
80
1 1 1
Part of the tank filled in 1 h = + - 78 2 1
30 10 40 Remaining part = 1 - = =
80 80 40
4 + 12 - 3 13
= = 1
120 120 part of the tank will be filled by inlet
40

270 @BEST300MCQ For More Study Material


Visit: studyiq.com
18. (c); By the formula
1
40 = 16 = 8 4 2 M 1D1 H1 M 2 D 2 H 2
A= = = hr =
1 40 20 10 5 W1 W2
16 Here, D1 = 15, H1 = 6, M1 = 12
2 2 M2 = ?, D2 = 12, H2 = 9
Required time = 12 + = 12 hr 15 × 6 × 12 = x × 12 × 9
5 5
13. (d); 1 part of cistern will be empty by pipe A = 27 h 5´ 6
x= = 10
3
2
part of cistern will be empty by pipe 19. (c); Time taken by second pipe to fill the tank
3
2 1 1 1
A = 27 ´ = 18 hr. = = = 1 = 24 hrs
3 æ 1 1 ö 3 - 2
ç - ÷ 24 24
14. (d); Given, A = 10 min, B = 6 min è 8 12 ø

1 1 Capacity of tank = 24 × 60 × 6 = 8640 litres


Part of tank will be empty in 1 min = -
6 10 20. (a); Given,
First tap to fill the tank (A) = 8 hr
5-3 1
= = Second tap can empty the tank (B) = 16 hr
30 15
\ Full tank will be empty in 15 min. 1 1
Part of the tank wil be filled in 1 h = -
8 16
2
\ rd part of the tank will be empty in
3 2 -1 1
= = Þ Required time = 16 hr
2 16 16
= 15 ´ = 10 minutes to empty the tank
3 21. (c); Given,
First tap to fill the tank (A) = 5 hr
1 1 1
15. (c); Part of the tank will be filled in 1 h = - = Second tap to empty the tank (B) = 4 hr
8 16 16
Part of the tank will be empty in 1 hr
Required time = 16 h
1 1 5-4 1
1 1 = - = =
16. (b); Part of the tank that will be empty in 1 h = + 4 5 20 20
15 10
Required time = 20 hr
2+3 5 1 22. (a); Part of the tank will be filled in 1 hr
= = = Þ Required time = 6 hrs
30 30 6 1 1 1 4+5-3 6 1
17. (c); Given, A = 20 min, B = 25 min = + - = = =
15 12 20 60 60 10
In 1 min, part of the tank will be filled Required time = 10 hr
1 1 5+4 9 23. (b); Part of tank will be emptied in 1 hr by 3rd pipe
= + = =
20 25 100 100 1 1 1 1
= + - =
45 60 70 60 70
In 5 min part of the tank will be filled =
100 Required time = 70 min
24. (d); Let after x min the pipe be closed so that the tank
45 55 11 will be filled full in half hour
Remaining part = 1 - = =
100 100 20
30 x 180 + 5x
\ ´2+ =1 Þ =1
11 75 45 225
It will take pipe A to fill the part of the tank
20 180 + 5x = 225 Þ 5x = 45 Þ x = 9 min
25. (b); Let the required time = x
11
Part of the tank will be filled by tap (A and B)
20
= 1 = 11 min
1 1 5 1
= + = =
20 60 40 120 24
271 @BEST300MCQ For More Study Material
Visit: studyiq.com
Then according to question
1 2
Remaining part = 1 - =
x x x x 3 3
+ =1 Þ + = 1 Þ x = 30
2 ´ 40 2 ´ (24) 80 48 Let a and b be time to fill the tank tap A and B
respectively.
Required time = 30 min

1 1 6 + 5 11 2
26. (a); Part of tank filled in 1 min = + = = 2 1 2
15 18 90 90 7= 3 Þ 1+1 = ´ =
1 1 a b 3 7 21
Part of the tank that will be filled in 6 min +
a b
66 11 Part of the tank will be fill by the tap C 1 hr
= =
90 15
1 2 7-4 3 1
Let x be the time taken to empty the tank by third = - = = =
6 21 42 42 14
tap.
Required time = 14 hr.
11 28. (d); Part of the tank will be filled by all of the taps in 1
15 33 90 – 11x 11 2
= ´
1 11 = 2 Þ 90x 15 33 min =
3+2-4 1
=
- 12 12
x 90
Required time = 12 hour
4050
180x = 4050 – 495x Þ x = = 6 min 1 1 1
675 29. (b); Part of the tank will be empty in 1 hr = - =
5 6 30
Required time = 30 hr
27. (c); Part of the tank filled by all the three tap A, B, C in
18 ´ 5 x´8
1 30. (c); By formula; =
1 hr = 1440 1920
6
18 ´ 5 ´ 1920
1 x= Þ x = 15 hour
In 2 hr part of the tank will be fill = 1440 ´ 8
3

1. (d); Part of the cistern that can be filled in 1 hr \ Part of the tank that will be filled after 5 min
1 1 8 + 7 15 45 3
= + = = = =
14 16 112 112 60 4
112 6 min is the time to empty the tank by tap M.
Time taken to fill the cistern = hr
15 3 1
\ Required time = ´
112 4 æ1 9 ö
= ´ 60 min = 448 min çè - ÷ø
6 60
15
Due to leakage, time taken to fill the cistern 3 60
= 448 + 92 = 540 min = 9 hr = ´ = 45 min
4 1
Time required to empty the tank
40
1 112 ´ 9 19 3. (c); Required time = min = 10 min
= = = 43 h 4
15 1 23 23
- 4. (a); Let first pipe fill the tank in (x) h then
112 9
Second pipe will fill it in (x + 10)h
2. (c); Part of the tank that will be filled when pipe
1 1 1 x + 10 + x 1
1 1 9 + = Þ =
P and R are opened for 1 min = + = x x + 10 12 x(x + 10) 12
12 15 60
272 @BEST300MCQ For More Study Material
Visit: studyiq.com
2x + 10 1 1 1
= Þ 24x + 120 = x2 + 10x \ Part of the tank will be filled in 1 min = +
2 30 30
x + 10x 12
x2 – 14x – 120 = 0 1
x2 – 20x + 6x – 120 = 0 =
15
x(x – 20) + 6(x – 20)= 0 Þ x = 20
\ Required time = 15 min
5. (b); Let total time = x
9. (b); Part of the tank will be filled in 1 min
2 x x 2
\ + =1 Þ = 1- 1 1 1 3 + 4 + 5 12 1
15 20 20 15 = + + = = =
20 15 12 60 60 5
x 13 52
= Þ x= min 40
20 15 3 \ Required time = 5 ´ = 2 min
100
6. (d); Part of the tank that will be filled by both pipe A
10. (d); Part of the tank will be filled in 1 min by tap
1 1 4+3 7
and B in 1 min = + = = 1 1 5+4 9
24 32 96 96 A and B = + = =
4 5 20 20
Let after x min, pipe B should be closed
In 2 min. part of the tank will be filled
7 (9 - x)
\ x´ + =1 9 9
96 24 = 2´ =
20 10
7x + 36 - 4x Time Required by third pipe to empty the tank
=1
96
9 1 9
3x + 36 = 96 Þ 3x = 60 Þ x = 20 min = ´ = ´ 20 = 18 min
10 æ 1 1 1 ö 10
7. (c); Part of the tank will be filled when all of the pipes ç - - ÷
A, B and C are opened alternatively, each of 1 è2 4 5ø
min (3 min) 11. (c); In 1 hr the amount of water that will be filled in
the tank = 42 + 56 – 48 = 50 L
1 1 1 3+2-4 1
= + - = = \ Capacity of the tank = 50 × 16 = 800 L
20 30 15 60 60
4
é æ 1 1 öù 12. (c); In 1 min the boy will pour water in tank = L
\ Time taken to fill ê 1 - ç + ÷ ú Or 3
ë è 20 30 øû
3
55 In 1 min the girl will pour water in tank = L
4
th part of the tank filled in = 3 × 55 = 165 min
60
1 12
55 \ Both will pour water in 1 min = =
1 4 3 25
\ Remaining part = 1 - = +
60 12 3 4
1 12
Tank A fills part in 1 min \ Required time = ´ 100 = 48 min
20 25
1 1 1 13. (a); Let inlet pipe fill the tank in x hr
\ Remaining part = - = \ Outlet pipe will empty the tank in 2x hr
12 20 30

1 1 1 1 1 1
\ - = Þ = Þ x = 36 hr
i.e., th part is filled by B in 1 min x 2x x + 36 2x x + 36
30
\ Total time = (165 + 1 + 1) = 167 min 14. (a); Diameter Þ 1 3 4
8. (d); When taps are not opened properly Efficiency Þ 1 : 9 : 16 ® 26 ® 16 min
6 ¯
Tap A will fill the tank = 25 ´ = 30 min
5 16 ´ 26
16
3
Tap B will fill the tank = 20 ´ = 30 min Thus, third pipe alone will fill the tank in 26 min.
2

273 @BEST300MCQ For More Study Material


Visit: studyiq.com
15. (c); Part of the tank will be filled in 1 hr
2 1 2 1
= ´ = ´
1 1 9 1 3 1 1 1 3 9+6-5
= + = = + -
36 45 180 20 20 30 36 180
\ Required time = 20 hr
2 180
16. (c); In 1 minute the pipe of 2 cm fills = ´ = 12 hr
3 10
1 1
´ of the cistern \ Total time = (12 + 4) hr = 16 hr
61 4
18. (b); Part of tank will be full in 1 hr
1 1 4 1 1 1 6+ 4-3 7
In 1 min the pipe of 1 cm diameter fill ´ of = + - = =
3 61 9 4 6 8 24 24
the cistern
24 3
1 \ Required time = = 3 hrs.
In 1 min the pipe of 1 cm diameter fill of the 7 7
61
19. (b); Let first pipe can fill the reservoir in x hr
cistern.
\ Second pipe will fill it in (x + 5) hr
1 1 4 1 9 + 16 + 36
\ + ´ + = 1 1 1 x+ 5+ x 1
61 ´ 4 61 9 61 9 ´ 4 ´ 61 \ + = Þ =
x x+5 6 x(x + 5) 6
61 1 6(2x + 5) = x2 + 5x Þ 12x + 30 = x2 + 5x
= =
36 ´ 61 36 x2 – 7x – 30 = 0 Þ x2 – 10x + 3x – 30 = 0
\ The whole is filled with 36 minutes. (x + 3) (x – 10) = 0 Þ x = 10 hrs
17. (c); Part of the tank will be filled, by both pipe in 1 hr 20. (b); Let x, y and z be time to fill the cistern by pipes, A,
B and C respectively. The part of the tank will be
1 1 5 1 filled in 2 hr by all the pipe
= + = =
20 30 60 12
æ 1ö 1
\ In 1 hr the leak will empty the part of tank = 2´ç ÷ =
è 6ø 3
1 1 1
= ´ = 1 2
12 3 36 Remaining part = 1 - =
3 3
1 \ Time taken by pipe A and B together to fill the
And to fill of the tank, both pipe will take
3
3
tank = 8 ´ = 12 h
12 2
= = 4 hr.
3 1 1 1
\ by pipe C alone to fill the tank = - =
2 6 12 12
to fill other part of the tank, it will take \ Required time = 12 hr
3

1. (d); Let x be the time, after that B be closed so that the 3. (d); In 30 min the part of the tank will be filled by
tank is full in 18 min.
30 5
both tap = =
18 x 72 + 3x 36 6
\ + =1 Þ =1
24 32 96
5 1
72 + 3x = 96 Þ 3x = 24 Þ x = 8 min \ Required part = 1 - =
6 6
2. (c); Part of the tank will be empty in 1 hr
1
1 1 1 6+4-5 5 1 \ part of the tank will be filled by pipe A in 10
= + - = = = 6
10 15 12 60 60 12
min
\ Required time = 12 hr \ tap A will take 60 min. to fill the tank

274 @BEST300MCQ For More Study Material


Visit: studyiq.com
\ Pipe B will take time to fill the tank 3
60 6
=
1

1 5–3
= =
2
=
1
; 3 min = 10 = =
36 60 180 180 90 11 110 11
20
Hence B will take 90 min to fill the tank.
8. (b); Part of tank will be filled by both pipe A and B in
4. (d); The part of tank that will filled in
2 1 1
1 1 4-3 1 1 hr = ´ =
2 min = - = = 3 8 12
45 60 2 180
12
\ Required time = = 12 hr
æ 1 44 ö 2 -1
Part of the tank ç 1 - = ÷ will be filled in
è 45 45 ø 9. (d); Part of the tank that will fill the tank in 1 hr by all

176 1æ1 1 2 ö 1
= ´ 180 ´ 2 = 352 min these pipes = ç + + ÷=
180 2 è 6 10 15 ø 5
So, tank will be filled in 5 h
1 10. (a); Let tank is filled in x hours
And of the tank will be filled in 1 min
45
x x 10 20x + 15x - 120
\ Total time = (352 + 1) min + - =1 Þ =1
15 20 25 300
= 353 min = 5 hr 53 min
5. (d); Let time = x 420
35x = 300 + 120 Þ x = = 12
35
5 (x + 5) 15 + x + 5 Therefore tank is filled in 12 hours
\ + =1 Þ =1
20 60 60 11. (d); (A’s 1 hour work) + (A + B)’s 1 hour work
20 + x = 60 Þ x = 40 min 1 æ 1 1 ö 17
6. (b); (A's 1 hour work) + (A + B)'s 1 hour work = +ç + ÷ =
10 è 10 12 ø 60
1 æ 1 1 ö 11 17 43
+ç + ÷ = Remaining part = 1 - =
3 è 3 4 ø 12 60 60
Now (A + B + C)’s 1 hour work
11
At 5'o' clock part of the cistern is filled. 1 1 1 1
12 = + + =
10 12 15 4
Total discharge if all the pipes opened
1 æ 43 ö
æ 1 1ö 12 – (3 + 4) 5 part is filled by then in ç 4 ´ ÷ = 2 hr 52 min
=1– ç + ÷ Þ = 4 è 60 ø
è 3 4ø 12 12 \ Total time = 2 + 2 hr 52 min = 4 hr 52 min
12. (a) Part of the tank that will filled when all the pipes
11
Time required to empty part of the tank 1 1 1 3+2-4 1
12 are open in 3 min = + - = =
20 30 15 60 60
11 éæ 1 1 ö 55 ù
12 11 êç 1 - 20 - 30 ÷ = 60 ú part of the tank will be filled
= = = 2.2 hr. ëè ø û
5 15 in = 55 × 3 = 165 min
12
55 1
Hence required time = 7.12 PM Remaining part = 1 - =
60 12
7. (c); Part of the tank will be filled in 1 min
1
Tap A fills part in 1 min
1 1 1 2 + 3 + 6 11 20
= + + = =
30 20 10 60 60 1 1 1
Remaining part = - =
12 30 30
33 11
In 3 min the part of the tank will be filled = =
60 20 1
th part is filled by B in 1 min.
Part of the solution R in the liquid after 30
\ Total time = (165 + 1 + 1) = 167 min.

275 @BEST300MCQ For More Study Material


Visit: studyiq.com
13. (c); Let A can fill the tank = x 16. (a); Let the filling capacity of the tank = x m3/min
x Emptying capacity of the tank = (x + 10) m3/min
\ B will fill the tank =
2 2400 2400
\ - = 8 Þ x2 + 10x – 3000 = 0
x x (x + 10)
\ C will fill the tank =
4 (x – 50) (x + 60) = 0 Þ x = 50 m3/min
4 2 1 1 7 1 17. (d); Work done by the inlet in 1 hr
\ + + = Þ = Þ x = 35 hrs
x x x 5 x 5 1 1 1
14. (d); Let total time = x = - =
8 12 24
4 x 16 + 3x Work done by the inlet in 1 min
\ + =1 Þ = 1 Þ 3x = 60 – 16
15 20 60 1 1 1
= ´ =
44 44 24 60 1440
x= min Þ = ´ 60 sec = 880
3 3 1
= 14 min 40 sec Volume of part = 6 litres
1440
15. (d); In one hour the part of the tank will be filled
Volume of tank = 6 × 1440 = 8640 litres
1 1 5+4 9 18. (c); Part of the tank will be empty by waste pipe in
= + = =
12 15 60 60
1 1 1 6 + 5-8 3 1
In second hour part of the tank will be filled 1 min = + - = = =
20 24 15 120 120 40
1 1 5+3 8 capcity of tank = 40 × 3 = 120 gallons.
= + = =
12 20 60 60
2 1
\ In two hour part of the tank will be filled 19. (c); Part of the tank will be filled in = 2 hr = =
6 3
9 8 17
= + = 1 2
60 60 60 \ Remaining part = 1 - =
3 3
51
In 6 hour part of the tank will be filled = 7 21
60 \ together (A + B) can fill the tank = ´3=
2 2
51 9 3
Remaining part = 1 - = = 1 2 1
60 60 20 \ C’s 1 hour work = - =
6 21 14
3
Required time = 14 hrs
\ Remaining part will be filled in = 20
20. (a); Let after x minute the pipe B should be closed
9
60 20 x 40 + x
+ =1 Þ =1
24 48 48
3 60
= ´ = 1 hr ; \ Total time = (6 + 1) = 7 hr. 40 + x = 48 Þ x = 8
20 9

1. (c); A 12 In 6 hour, part of tank that can be filled


5
= 17 × 3 = 51 units
4 left part = (60 – 57) = 9 units
B 15 60 unit
3 Which will be filled in 1 hour bg pipe A and B.
C 20 \ Total time = 7 hours
In 1 hour, part of tank that can be filled 2. (b); 'd ' diameter : '2d' diameter
= 5 + 4 = 9 units Efficiency 1 : 4
In 2 hour, part of tank that can be filled
= 5 + 3 = 8 units 40
\ Required time = = 10 min .
4

276 @BEST300MCQ For More Study Material


Visit: studyiq.com
In 2 minutes part of tank will be filled = 4.3
1 1
3. (b); Required time = = = 21 hours = 1 units
1 2 7-6
- To fill (180 – 4 = 176) units of tank, time
3 7 21
= 176 × 2 = 352 units.
1 \ Required time = 352 + 1 = 353 minutes
4. (b); Required time = = 5 hr 53 minutes
1 1 1
+ -
60 75 50 9. (d); A 15
4
300 5
= = 100 minutes B 12 60 units
5+ 4-6 – 15
1 1 1 C 4
5. (d); Remaining part after hour = 1 - =
2 2 2 At 11 A.M. past of tank filled = 4 × 3 + 5 × 2
= 22 units
1 In 1 hr, all the three taps will empty the tank
1 3
to fill part, time taken = 2 = hr = 15 – 5 – 4 = 6 units
2 4 4
6 22
To empty 6 units, time = = 3 hr 40 units
6
3 \ Required time = 02 : 40 PM
\ Required time = 3 hr
4
1 xy
3 hr 45 min. 10. (d); Required time = = hours
1 1 y-x
-
6. (c); Faster : Slower x y
Efficiency 3 : 1 11. (c); 12 × 6 × 15 = x × 9 × 12
Time 1 : 3 x = 10
12. (a); Efficiency 2 : 1
1 1 1
= + = Time 1 : 2
x 3x 36
2x = x – 36 Þ x = 36 hours
3+1 1
= 1 1 1
3x 36 13. (d); + = ...(i)
A B 3C
x = 48 minutes
\ Required time = 48 × 3 = 144 minutes 40 30
+ =1 ...(ii)
A B
1
7. (b); both pipes will empty the tank in = equation (i) × 40 – equation (ii)
1 1
-
5 8 40 40 40
+ =
A B 36
1 40 30
40 + = 1
= 8-5 = hrs A B
40 3 - - -
10 40
3 = - 1
B 36
\ Required time = 4 = 10 hours
3 10 4
40 = Þ B = 90 minutes
B 36
8. (d); A 45 14. (c); Time taken to empty the tank by pipe
+4
1 120
c= = = 40 minutes
180 units 1 1 1 6+ 5- 8
–3 + -
20 24 15
B 60
\ Capacity of tank = 40 × 3 = 120 gallons

277 @BEST300MCQ For More Study Material


Visit: studyiq.com
15. (d); Efficiency é 1 ù

P 4 20 19. (c); Required time = 1 1 ú
2ê - ú
15 ë 12 20 û
Q 8
10 120
1 60
R 12 – 12 = ´ = 15 hrs.
2 5-3
S 10 3 3+ x 15 + 12 + 4x
20. (d); + =1 Þ =1
(A) Q efficiency 15 units/hr 12 15 60
(B) (P + R – S) efficiency = 30 + 10 – 12 = 28 unit / 4x = 60 – 27 Þ 4x = 33
hr 33 1
(C) (P + S) efficiency = 30 – 12 = 18 unit/hr x= Þ x = 8 minutes
4 4
(D) (P + Q – S) efficiency = 30 + 15 – 12 = 33 unit/
hr é 1 ù

\ efficiency of D is highest 21. (b); Time = 1 1 1ú
4ê + - ú
ë 10 12 6 û
1 1 1
16. (d); - =
10 x 15 1 60
= ´ = 15 hrs.
1 1 1 3-2 4 6 + 5 - 10
= - = \ Required time = 10 PM
x 10 15 30
x = 30 hours 3
22. (a); th ® 1
\ Required capacity of tank 5
= 30 × 4 × 60 = 7200 L
5 2 5 2 2
1® Þ ® ´ = minutes
3 4 3 5 3 5 3
17. (c); Boy = , Girl =
4 3 = 40 sec.
16 + 9 25 23. (c); Let it will drop by h cm
Boy + Girl = = litres
12 12 22 25 25
\ ´ ´ ´ h = 11000
7 2 2
25
\ litre ® 1 minute
12 2
h = 22 cm
5
1
100 litre ® ´ 12 ´ 100 = 48 minutes
25 1 1 1
24. (a); + =
x x + 10 12
2 1
18. (a); After 2 hours, part filled = =
6 3 x + 10 + x 1
=
x ( x + 10) 12
1 2
Remaining part = 1 - = (2x + 10)12 = x2 + 10x
3 3
24x + 120 = x2 + 10x
2 x2 – 14x – 120 = 0
\ 3 =7 x2 – 20x + 6x – 120 = 0
1 1
+ x(x – 20) + 6(x – 20) = 0
A B
(x – 20)(x + 6) = 0
1 1 2 x = 20 hrs.
+ =
A B 21 18 x 72 + 3x
25. (b); + =1 Þ =1
42 24 32 96
1
\ Required time = = = 14 hours 72 + 3x = 96
1 2 7-4
- 3x = 24 Þ x = 8 minutes
6 21

278 @BEST300MCQ For More Study Material


Visit: studyiq.com
x x (x - 7) 14
26. (b); + - =1 29. (d); After 2 hours, Remaining part = 1 -
36 45 30 24

5x + 4x - 6x + 42 10 5
=1 = =
180 24 12
3x = 138 5
40
x = 46 minutes \ Required time = 12 =
1 12
27. (b); 9 × 20 = x × 15
8
9´4
x= = 12 20 10 1
3 = = = 3 hours
6 3 3
28. (c); A 20 +3 30. (c); Inlet 180 + 5

60 units +4
–2 Inlet 225 900 unit
– 15
B 30 Outlet 60
In 2 minutes, tank filled = 3 – 2 = 1 units At 3 PM, tank filled = 5 × 2 + 4 = 14 units
In 114 minutes tank filled = 57 units 14
14 units of tank can be embtied in =
Remaining = 60 – 57 = 3 units 6
Which will be filled in 1 minute by tap A = 2 hrs 20 min
\ Required time = 5 : 20 PM
\ Total time = 114 + 1 = 115 minutes

279 @BEST300MCQ For More Study Material


Visit: studyiq.com
Chapter
Speed, Time and Distance
12
The concepts of time distance are most important in the terms of competitive exams. The basic concept of time and
distance is used in solving the question based on motion in a straight line. The applications of time & distance are used
to solve the problems related to trains and races.
The relation between time, distance and speed is
Distance = Time × Speed
Distance (D) Distance (D)
i.e, D = T × S Þ Time (T) = Speed (S)
Þ Speed (S) =
Time (T)
Example: A car covers 200 km in 4 hours, then find the speed of the car.
Distance (D)
Sol. We know that,  Speed (S) 
Time (T)

200
Required speed = = 50 km/h
4
5 5
Conversion of Units: (i) When we convert km/h into m/s, we multiply the speed by . i.e, 1km/h = m/s.
18 18
18 18
(ii) When we convert m/s into km/h, we multiply the speed by . i.e, 1m/s = km/h
5 5
Example: Convert 72 km/h into m/s.
 5
Sol. We know that, 72 km/h = 
72   m /s = 4 × 5 = 20 m/s
 18 
Concept 1:
Average speed: A certain distance is covered at ‘x’ km/h and the same distance is covered at ‘y’ km/h then the average
speed during the whole journey.
2xy
Average speed = km /h
x+y
Example: Rohit covers a certain distance by car driving at speed of 40 km/h and he returns back to the starting point
riding on a scooter with a speed of 10 km/hr. Find the average speed of the whole journey?
2 ´ 40 ´ 10 2 ´ 400
Sol. Average speed = = = 16 km / hr
40 + 10 50
Concept 2: A person covers a distance in T hours and the first half at S1 km/h and the second half at S2 km/h, then the
total distance covered by the person.
2 ´ T ´ S1 ´ S2
D=
S1 + S 2
Example: A car covers a distance in 10 hrs, the first half at 40 km/h and the second half at 20 km/h. Find the distance
travelled by car?
2 ´ 10 ´ 40 ´ 20 2 ´ 10 ´ 40 ´ 20
Sol. Distance = km /h = = 266.67 km
40 + 20 60
Concept 3: If two persons P and Q start at the same time in opposite directions from two points and after passing each
they complete their journeys in 'a' and 'b' hrs respectively then
P 's speed b
=
Q 's speed a

280 @BEST300MCQ For More Study Material


Visit: studyiq.com
Example: Shivam sets out to cycle from Delhi to Ghaziabad and at the same time Hemant starts from Ghaziabad to
Delhi, After passing each other they complete their journeys in 4 and 16 hours respectively. At what rate
does Hemant cycle if Shivam cycle at 18 km per hour?

Shivam 's speed 16 18 4


Sol. = Þ = Þ Hemant’s speed = 18 = 9 km / h
Hemant 's speed 4 Hemant 's speed 2 2
Concept 4: If a man travelled a certain distance by bus at a rate of x km/h and walked back at the rate of ‘y’ km/h. If the
æ xy ö
whole journey took ‘t’ hours, then the distance he travelled is ç t km.
è x + y ÷ø
Example: A man travelled a certain distance by train at a rate of 15 km/h and walked back at the rate of 12 km/h. The
whole journey took 9 hours. Find the distance he travelled?
æ xy ö æ 15 ´ 12 ö 15 ´ 12 ´ 9
Sol. Required distance = ç ÷t =ç ÷ø 9 = = 60 km
è x + y ø è 15 + 12 27

a
Concept 5: If a person changes his speed to of its usual speed and late by T minutes, then the usual time taken by him
b

T éa ù éa ù T
is
b êë b < 1úû and when ê b > 1ú then the usual time taken by him is b
.
-1 ë û 1-
a a

4
Example: Walking of his usual speed, a man is 16 minutes late. Find the usual time taken by him to cover that
5
distance?
T 16 16 ´ 4 16 ´ 4
Sol: Usual time = = = = = 64 minutes
b 5 5- 4 1
-1 -1
a 4
Concept 6: (i) If speed is constant, then distance is directly proportional to the time; D µ T
(ii) If time is constant, then distance is directly proportional to the speed; D µ S

1
(iii) If Distance is constant, then speed is inversely proportional to the time; S µ
T
Example A person covers a certain distance with a speed of 54 km/h in 15 min. If he wants to cover the same distance
in 30 min, what should be his speed?
15 9 27
Sol. We know that, Distance = Speed × Time = 54 ´ = ´ 15 = km
60 10 2

27 27 27
Speed to cover km in 30 min = = ´ 2 = 27 km/h
2 2 2
30
60
Concept 7: (i) When a train passes a pole or any other object, the distance covered by train is equal to the length of the
train.
(ii) If a train passes a bridge, platform etc, then distance travel by train is equal to the sum of the length of
train and the stationary object through which the train is passing.
Example: A 100 m long train passes a platform of 200 m long. Find the distance covered by the train in passing the
platform?
Sol. Required distance = length of train + length of platform = 100 + 200 = 300 m
Concept 8: (i) When two trains are moving is opposite directions, then their relative speed is equal to the sum of the
speed of both trains.

281 @BEST300MCQ For More Study Material


Visit: studyiq.com
(ii)
When two trains are moving is same directions, then their relative speed is equal to the difference of
the speed of both trains.
Example: Two trains are moving in the same direction with speed of 40 km/h and 50 km/h respectively. Find the
relative speed?
Sol. Required relative speed = (50 – 40) km/h = 10 km/h
Concept 9: Two trains start at the same time from P and Q and proceed towards each other at the rate of x km/h and y
km/h respectively. When they meet it is found that one train has travelled D km more than the other. The
x y  Sum of speeds
Distance between P and Q is  D ,
 Distance = × Difference in distance
x  y  Difference of speed
Example: Two trains start at the same time from Kanpur and Delhi and proceed towards each other at the rate of 74
km/h and 47 km/h respectively. When they meet it is found that the train has travelled 13 km more than the
other. Find the distance b/w Kanpur and Delhi?

Sum of speeds ( 73 + 47) ´ 13 = 120 ´ 13 = 60 km


Sol. Required Distance = ´ Difference in Distance =
Difference of speeds ( 73 - 47) 26
Concept 10: When the speed of two trains are in the ratio x : y. They are moving in opposite directions on parallel
tracks. The first train crosses a telegraph pole in ‘t 1’seconds where as the second train crosses the pole in
‘t 2 ’ seconds. Time taken by the trains to cross each other completely is given by
t 1x  t2 y
Time taken  seconds.
xy
Example: The speed of two trains are in the ratio 4 : 5. They are moving in opposite directions along the parallel tracks.
If each takes 3 seconds to cross a pole. Find the time taken by the train to cross each other completely?
æ t x + t 2 y ö 3  4  3  5 27
Sol. Time taken = ç 1    3 seconds
è x + y ÷ø 45 9

1. A man covers a certain distance between his house Sol. Let the required distance be x km.
and office on bike. When his speed is 30 km/h, he is
x
late by 20 min. However, with a speed of 40 km/h, he Then time taken when he goes to school = h
reaches his office 10 min earlier. Find the distance 6
between his house and office? x
Sol. Let the distance be x km. Time taken when he goes to his house = h
4
x x x
Time taken to cover x km at 30 km/h = h Therefore, + =5
30 6 4
x 2x + 3x
Time taken to cover x km at 40 km/h = h Þ =5
40
12
1
Difference between time taken = (20 + 10) m = h 5
2 \ x = ´ 12 = 12 km
5
x x 1 30 ´ 40 3. The distance between two stations A and B is 450 km.
\ - = Þ 40x – 30x =
30 40 2 2 A train starts from A and moves towards B at an
average speed of 20 km/h. Another train starts from
30 ´ 40
x= = 60 km B, 20 minutes earlier than the train at A, and moves
20 towards A at an average speed of 30 km/h. How far
Hence, the required distance is 60 km. from A will the two trains meet?
2. A boy goes to school at a speed of 6 km/h and return Sol. Let the trains meet at a distance of x km from A.
to his house at a speed of 4 km/h. If he takes 5 hrs in
all, what is the distance between his house and the 450 - x x 20 20 (450 - x) - 30x 1
- = ; =
school? 30 20 60 20 ´ 30 3

282 @BEST300MCQ For More Study Material


Visit: studyiq.com
20 ´ 30 Train takes 6 hours in second case
9000 – 20x – 30x = , 9000 – 50x = 200 5 hours in original case
3
8800 60
8800 = 50x Þ x= = 176 km Original speed = = 12 km/h
50 5
4. Two men P and Q start from a place a walking at 6 km 8. A thief is spotted by a policemen at a distance of 200
and 8 km an hour respectively. How many km will metres. When the policeman starts to chase, the theif
they be apart at the end of 2 hours, if they walk in also starts running. The speed of thief and policemen
same direction? are 10 km/h and 14 km/h respectively. How far will
Sol. Distance travelled by P in 2 hours = 2 × 6 = 12 km have the thief run before he caught?
Distance travelled by Q in 2 hours = 2 × 8 = 16 km Sol. Relative Speed = (14 – 10) = 4 km/h
Distance between P and Q = (16 – 12) km = 4 km 200 1
5. Two runners cover the same distance at the rate of 10 \ The thief will be caught after = = h
1000 ´ 4 20
km/h and 15 km/h respectively. Find the distance
The distance covered by thief before he caught
travelled when one takes 16 minutes longer than the
other? 1
= 10 ´ = 0.5 km = 500 m
Sol. Let the distance be x km. 20
x 9. The ratio between the speeds of Hemant and Nitish is
Time taken by the first runner = h 6 : 7. If Hemant takes 30 minutes more than Nitish to
10
cover a distance. Find the actual time taken by Hemant
x and Nitish ?
Time taken by the second runner = h
15 Sol. Let the speed of Hemant and Nitish be x1 km/h and
x2 km/h respectively.
x x 16 3x - 2x 16 and the time taken by Hemant and Nitish be t1 and t2
Now, - = Þ =
10 15 60 30 60 respectively.
16 x1 t 2 t 6
x= ´ 30 = 8 km Distance = x1 t1 = x2 t2 , = or 2 =
60 x2 t1 t1 7
6. Without any stoppage a person travels a certain
distance at an average speed 80 km/h and with 30
t1 - t2 7 - 6 t 1 - t 2 1 1
stoppages he covers the same distance at an average = , = , 60 =
speed of 60 km/h. How many minutes per hour does t1 7 t1 7 t1 7
he stop?
Sol. Let the total distance be x km. 7 7 6
t1 = hours t 2 = ´ = 3 hours
x 2 2 7
Time taken at the speed of 80 km/h = h
80 7
Actual time taken by Hemant = hours
x 2
Time taken at the speed of 60 km/h = h Actual time taken by Nitish = 3 hours
60
10. A man tries to ascend a greased pole 101 metres high.
æ x xö 20x x He ascends 10 metres in first minute and slips down
\ he rested for ç - ÷ h = = h
è 60 80 ø 60 ´ 80 240 1 metre in the alternate minute. If he continues to
x x 1 ascends in this way. How long does he takes to reach
He rested per hour = ¸ = h = 15 minutes the top?
240 60 4
Sol. In every 2 minutes, he is able to ascend (10 – 1) = 9
7. A train leaves the station 1 hour before the scheduled
metres
time. The driver decreases its speed by 2 km/h. At the
next station 60 km away, the train reached on time. In 22 minutes, he ascends upto 9 × 11 = 99 metres
Find the original speed of the train? 2 1
Sol. Let it takes x hours in second case For remaining distance he takes = = min = 12
10 5
60 60 30 30 seconds
- =2, - =1
x-1 x x-1 x Total time taken = 22 min 12 seconds.
30 x – 30 (x – 1) = x2 – x Þ 30 x – 30 x + 30 = x2 – x 11. A train running at 25 km/h takes 18 seconds to pass
x2 – x – 30 = 0 a platform. Next, it takes 12 seconds to pass a man
x2 – 6x + 5x – 30 =0 Þ x (x – 6) + 5 (x – 6) = 0 walking at 5 km/h in the opposite direction. Find the
(x + 5) (x – 6) = 0 Þ x = – 5, 6; \ x = 6 length of the train and platform?

283 @BEST300MCQ For More Study Material


Visit: studyiq.com
Sol. Speed of train relative to man = (25 + 5) = 30 km/h
æ 5ö æ 55 5 ö
5 25 length of the train = ç x – ÷ 9 = ç – ÷ 9 = 50m.
= 30 ´ = m/sec è 9ø è 9 9ø
18 3
14. A train after travelling 50 km meets with an accident
25 3
Distance travelled in 12 seconds = ´ 12 = 100 m
3 and then proceeds at of its former speed and arrives
4
Length of train = 100 m at its destination 35 minutes late. Had the accident
5 125 occurred 24 km further, it would have reached the
Speed of train = 25 km/h = 25 ´ = m/sec destination only 25 minutes late. Find the speed of
18 18
the train and the distance which the train travels?
125 Sol. Let the speed of the train be x km/hr and the distance
Distance travelled in 18 seconds = ´ 18 = 125 m
18 D km.
Length of train + length of platform = 125 m From the question we have,
Length of platform = (125 – 100) = 25 m 50 (D – 50)4 D 35
12. Two trains of length 200 m and 250 m respectively + = +
x 3x x 60
with different speeds passes a static pole in 8 sec and
14 sec respectively. In what time will they cross each 150 + 4D – 200 12D + 7x
other when they are moving in same direction. or, =
3x 12x
200 or, 4D – 7x = 200 ... (i) and
Sol. Speed of first train = = 25 m/s
8
74 (D – 74)4 D 25
250 125 + = +
Speed of second train = = m/s x 3x x 60
14 7
Total distance to be travelled = 200 + 250 = 450 m 222 + 4D – 296 12D + 5x
or, =
Relative speed when they are moving in the same 3x 12x
direction 4D – 5x = 296 ... (ii)
125 Now, subtracting equation (i) from the equation (ii),
= 25 – we have
7
2x = 96
450 ´ 7 \ x = 48 km/hr
Required time = = 63 sec
50 \ Speed of the train = 48 km/hr
13. A train overtakes two persons who are walking in the To find the distance, put the value of x in equation (ii)
same direction as the train is moving, at the rate of 2 4D – 5x = 296
km/hr and 4 km/hr and passes them completely in 9 or, 4D – 5 × 48 = 296
and 10 seconds respectively. Find the speed and the
length of the train? 536
or, D = = 134km
Sol. Speed of two men are: 4
5 5 \ Distance which the train travel = 134 km.
2 km/hr = 2 ´ = m / s and 15. In a race of 2500 m, A beats B by 500 m and in a race of
18 9
2000 m, B beats C by 800 m. By what distance A gives
5 10 start to C so that they will end up at same time in 3 km
4 km/hr = 4 ´ = m /s
18 9 race. Also, find by what distance A will win over C in
Let the speed of the train be x m/s. Then relative speed 1 km race?
Sol. Ratio of speeds of A : B = 2500 : 2000 = 5 : 4
æ 5ö æ 10 ö
are ç x – ÷ m / s and çè x – 9 ÷ø m / s Ratio of speeds of B : C = 2000 : 1200 = 5 : 3
è 9ø Ratio of speeds of A : B : C = 25 : 20 : 12
Now, length of train = Relative speed × Time taken to In 3 km race A run 3000 m,
pass a man
3000 ´ 20
æ 5ö æ 10 ö B run = = 2400 m
\ length of train = ç x – ÷ ´ 9 = ç x – ÷ ´ 10 25
è 9 ø è 9ø
3000 ´ 12
100 45 55 C run = = 1440 m
\ x= – = m/s 25
9 9 9 So to end up the race at same time A should gives C
55 18 the start of 1560 m.
\ speed of the train = ´ = 22 km / hr and In 1 km race, A will win over C by 520 m.
9 5

284 @BEST300MCQ For More Study Material


Visit: studyiq.com
Ques ti ons (a) 4 hours (b) 3 hours
3 1
(c) 3 hours (d) 4 hours
1. A man travels first 50 km at 25 kmph, next 40 km at 20 4 2
kmph and then 90 km at 15 kmph. His average speed
for the whole journey (in kmph) is: 7
9. A train running at of its normal speed reached a
(a) 25 (b) 20 11
(c) 18 (d) 40 place in 22 hours. How much time could be saved if
2. A man walks at the rate of 5 km/hr for 6 hours and at the train would have run at its normal speed?
4 km/hr for 12 hours. The average speed of the man (a) 14 hours (b) 7 hours
(in km/hr) is: (c) 8 hours (d) 16 hours
10. Walking at three-fourth of his usual speed, a man
1
(a) 4 (b) 4 covers certain distance in 2 hours more than the time
2 he takes to cover the distance at his usual speed. The
1 2 time taken by him to cover the distance with his usual
(c) 4 (d) 4 speed is:
3 3
(a) 4.5 hours (b) 5.5 hours
1 (c) 6 hours (d) 5 hours
3. If a person travels 10 km in 3 hours, then the
5 11. A man goes from a place A to B at a speed of 12 km/
distance covered by him in 5 hours will be : hr and returns from B to A at a speed of 18 km/hr.
(a) 18 km (b) 17 km The average speed for the whole journey is:
(c) 16 km (d) 15 km 2
4. If a train 110m long passes a telegraph pole in 3 (a) 14 km/hr (b) 15 km/hr
5
seconds, then the time taken (in seconds) by it to cross
a railway platform 165 m long, is: 1
(c) 15 km/hr (d) 16 km/hr
(a) 3 (b) 4 2
(c) 5 (d) 7.5 12. Two trains started at the same time, one from A to B
5. A train 700 m long is running at the speed of 72 km/ and the other from B to A. If they arrived at B and A
hr. If it crosses a tunnel in 1 minute, then the length of respectively 4 hours and 9 hours after they passed
the tunnel (in metres) is : each other, the ratio of the speeds of the two trains
(a) 700 (b) 600 was:
(c) 550 (d) 500 (a) 2 : 1 (b) 3 : 2
6. If a 200 m long train crosses a platform of the same (c) 4 : 3 (d) 5 : 4
length as that of the train in 20 seconds, then the speed 13. A starts from a place P to go to a place Q. At the same
of the train is : time B starts from Q to P. If after meeting each other A
(a) 50 km/hr (b) 60 km/hr and B took 16 and 25 hours more respectively to reach
(c) 72 km/hr (d) 80 km/hr their destinations, the ratio of their speeds is:
7. Two trains, each of length 125 metre, are running in (a) 3 : 2 (b) 5 : 4
parallel tracks in opposite directions. One train is (c) 9 : 4 (d) 9 : 13
running at a speed 65 km/hour and they cross each 14. A train of 320 m cross a platform in 24 seconds at the
other in 6 seconds. The speed of the other train is: speed of 120 km/h. while a man cross same platform
(a) 75 km /hour (b) 85 km/ hour in 4 minute. What is the speed of man in m/s?
(c) 95 km/ hour (d) 105 km/ hour (a) 2.4 (b) 1.5
(c) 1.6 (d) 2.0
3
8. A man with of his usual speed reaches the 15. A car travel first 39 km distance in 45 minute while
5 next 25 km distance in 35 minutes. What is its average
1 speed?
destination 2 hours late. Find his usual time to
2 (a) 45 Km/h (b) 35 Km/h
reach the destination? (c) 48 Km/h (d) 90 Km/h

285 @BEST300MCQ For More Study Material


Visit: studyiq.com
16. A 280 m long train moving with an average speed of 25. A 75 meter long train is moving at 20 kmph. It will
108 km/h cross a platform in 12 seconds. A boy cross a man standing on the platform in
crosses the same platform in 10 seconds. What is the (a) 12 seconds (b) 14 seconds
speed of boy in m/s? (c) 13.5 seconds (d) 15.5 seconds
(a) 5 m/s (b) 8 m/s 26. In what time will a train 100 meter long cross an
(c) 10 m/s (d) Cannot be determined electric pole, if its speed be 144 km/hour?
17. A truck cover 224 km in 4 hours, the average speed of (a) 2.5 seconds (b) 5 seconds
1 (c) 12.5 seconds (d) 3 seconds
a bike is th the average speed of the truck how
4 27. A train running at a speed of 60 kmph crosses a
much distance will the bike cover in seven hour? platform double its length in 32.4 seconds. What is
(a) 96 km (b) 98 km the length of the platform?
(c) 95 km (d) 92 km (a) 180 m (b) 240 m
18. If a person walks at 14 km/h instead of 10 km/h he (c) 360 m (d) 90 m
would have walked 20 km more. The actual distance 28. A train running at the speed of 66 kmph crosses a
travelled by him is: signal pole in 18 seconds. What is the length of the
(a) 85 Km (b) 50 Km train?
(c) 80 Km (d) 70 Km (a) 330 m (b) 300 m
19. Train fare between Patna to Munger for one adult is (c) 360 m (d) 320 m
three times the train fair of one child. If adult fair is 29. Two trains of equal length take 10 seconds and 15
102 then. What will be the fare of 2 adult and 3 seconds respectively to cross a telegraph post. If the
children together for same distance? length of each train be 120 metres, in what time (in
(a) 306 (b) 212 seconds) will they cross each other travelling in
(c) 206 (d) 214 opposite direction?
20. A car travels a distance of 75 km at the speed of 25 (a) 16 sec (b) 15 sec
km/h. It covers the next 25 km of its journey at the (c) 12 sec (d) 10 sec
speed of 5 km/h and the last 50 km of its journey at
30. A man covers a certain distance by car driving at 70
the speed of 25 km/h. What is the average speed of
km/hr and he returns to the starting point riding on a
car?
scooter at 55 km/hr. Find his average speed for the
(a) 15 Km/h (b) 12.5 Km/h whole journey.
(c) 40 km/h (d) 25 km/h
(a) 61.6 km/hr (b) 62.8 km/hr
21. If the length of the train is 700 m and length of
(c) 63.6 km/hr (d) 64.6 km/hr
platform is 500 m find the time taken by the train
moving at 54 km/h to cross platform? 31. A boy walking at a speed of 10 km/hr reaches his
school 15 minutes late. Next time he increases his
(a) 75 sec (b) 80 sec
speed by 2 km/hr, but still he is late by 5 minutes.
(c) 85 sec (d) 90 sec
Find the distance of his school from his house.
22. Amit start to go to Delhi from patna at speed of 50
(a) 10 km (b) 15 km
km/h. Distance between Delhi and Patna is 1000
km. He takes rest of 20 minutes every 3 hours of (c) 20 km (d) 25 km
journey. How much time will he take to arrive Delhi? 32. A motor car does a journey in 10 hrs, the first half at
(a) 20 hours (b) 21 hours 21 km/hr and the second half at 24 km/hr. Find the
(c) 22 hours (d) 23 hours distance?
23. A car cover a distance of 330 km in a certain amount (a) 224 km (b) 225 km
of time at speed of 55 km/h. What is the average (c) 226 km (d) 228 km
speed of bike that cover distance of 15 km less than 33. Two men A and B starts from a place P walking at 3
car in 1 hour less than time taken by car? km and 3.5 km an hour respectively. How many kms
(a) 50 Km/h (b) 60 Km/h will they be apart at the end of 3 hrs, if they walk in
(c) 63 Km/h (d) 65 Km/h opposite directions?
24. A car traveling at a speed of 40 km/hour can complete (a) 13.5 km (b) 15.5 km
a journey in 9 hours. How long will it take to travel (c) 17.5 km (d) 19.5 km
the same distance at 60 km/hour? 34. What is the length of the bridge which a man riding
(a) 6 hours (b) 3 hours 15 km an hour can cross in 5 minutes?
1 (a) 850 m (b) 1050 m
(c) 4 hours (d) 4 hours (c) 1250 m (d) Can’t be determined
4

286 @BEST300MCQ For More Study Material


Visit: studyiq.com
35. A train moving with the speed of 180 km/hr. Its speed 38. A train running at 25 km/hr takes 18 seconds to pass
(in metres per second) is: a platform. Next, it takes 12 seconds to pass a man
(a) 5 (b) 40 walking at 5 km/hr in the opposite direction. Find
(c) 30 (d) 50 the sum of the length of the train and that of the
platform?
36. If a train, with a speed of 60 km/hr, crossed a pole in
30 second, the length of the train (in metres) is: (a) 125 m (b) 135 m
(a) 1000 (b) 900 (c) 145 m (d) 155 m
39. A train is travelling at a rate of 45 km/hr. How many
(c) 750 (d) 500
seconds, it will take to cover a distance of 4/5 km?
37. Two trains are moving in the same direction at 50
(a) 36 second (b) 64 second
km/hr and 30 km/hr. The faster train crossed a man
(c) 90 second (d) 120 second
in the slower train in 18 seconds. Find the length of
the faster train? 40. A 570 metre long train crosses a platform of equal
length in 15 seconds. What is the speed of the train in
(a) 80 m (b) 90 m
metres/second?
(c) 100 m (d) Can’t be determined
(a) 38 (b) 54
(c) 76 (d) 70

1. A boy is running at a speed of p kmph to cover a him. Another train of same length going in the
distance of 1 km. But, due to the slippery ground, his opposite direction takes 5 seconds to pass him. The
speed is reduced by q kmph (p > q). If he takes r hours time taken (in seconds) by the two trains to cross each
to cover the distance, then : other will be:
1 1 1 1 31 40
(a) = + (b) = p-q (a) (b)
r p q r 9 9
(c) r = p + q (d) r = p – q 49 50
(c) (d)
2. A boy goes to the school with the speed of 3 kmph 9 9
and returns with the speed of 2 kmph. If he takes 5 7. A train, 240m long crosses a man, walking along the
hours in all, then the distance (in kms) between the line in opposite direction at the rate of 3 kmph in 10
village and the school is : seconds. The speed of the train is:
(a) 6 (b) 12 (a) 63 km/h (b) 75 km/h
(c) 8 (d) 9 (c) 83.4 km/h (d) 86.4 km/h
3. A student walks from his house at 5 kmph and reaches 8. The distance between two cities A and B is 330km. A
his school 10 minutes late. If his speed had been 6 train starts from A at 8 am. and travels towards B at
kmph he would have reached 15 minutes early. The 60 km/hr. Another train starts from B at 9 am. and
distance of his school from his house is : travels towards A at 75 km/hr. At what time do they
(a) 2.5 km (b) 12.5 km meet?
(c) 5.5 km (d) 3.6 km (a) 10 am. (b) 10 : 30 am.
4. A train takes 18 seconds to pass completely through (c) 11 am. (d) 11 : 30 am.
a station 162 m long and 15 seconds through another 9. Two men are standing on opposite ends of a bridge
station 120 m long. The length of the train (in metres) of 1200 metres long. If they walk towards each other
is : at the rate of 5m/minute and 10m/minute
(a) 70 (b) 80 respectively, in how much time will they meet each
(c) 90 (d) 100 other?
5. Two trains of lengths 120 m and 80 m are running in (a) 60 minute (b) 80 minute
the same direction with velocities of 40 kmph and 50 (c) 85 minute (d) 90 minute
kmph respectively. The time taken by them to cross 10. Two trains one 160 m and the other 140 m long are
each other is : running in opposite directions on parallel track, the
(a) 60 sec (b) 72 sec first at 77 km an hour and the other at 67 km an hour.
(c) 75 sec (d) 80 sec How long will they take to cross each other?
6. A man standing on a railway platform observes that (a) 7 seconds (b) 7.5 seconds
a train going in one direction takes 4 seconds to pass (c) 6 seconds (d) 10 seconds

287 @BEST300MCQ For More Study Material


Visit: studyiq.com
11. Two trains of equal length take 10 seconds and 15 17. The ratio between the speed of bike and train is 15 :
seconds respectively to cross a telegraph post. If the 27 respectively. Also a bus cover a distance of 720 km
length of each train be 120 meters, in what time (in in 9 hours. The speed of the bike is three fourth the
seconds) will they cross each other travelling in same speed of bus. How much distance will the train cover
direction? in 7 hours?
(a) 16 (b) 15 (a) 756 km (b) 760 km
(c) 12 (d) 60 (c) 740 km (d) Cannot be determined
12. A person travels 600 km by train at 80km/hr, 800 km 18. A bus started its journey from Bettiah and reached
by ship at 40 km/hr. 500 km by aeroplane at 400 km/ Motihari in 44 minute with its average speed 50 km/
hr and 100 km by car at 50 km/hr. What is the average h. If the average speed of bus is increased by 5 km/h
speed for the entire distance? how much time will it take to cover the same distance?
(a) 31 min (b) 36 min
5
(a) 65 km./hr. (b) 60 km./hr. (c) 38 min (d) 40 min
123
19. A train can travel 50% faster than a bus. Both start
5 from point A at the same time and reach point B at
(c) 60 km./hr. (d) 62 km./ hr. same time. Point B is 75 km away from point A on the
123
way the train stopping 12.5 minutes. The speed of
13. One third of a certain journey is covered at the rate of bus is:
25 km/hr one-fourth at the rate of 30 km/hr and the
(a) 110 km/h (b) 120 km/h
rest at 50 km/hr. The average speed for the whole
journey is (c) 55 km/h (d) 60 km/h
20. A train crosses a pole in 10 seconds and a platform,
1 which is 40% longer than the length of train in 24
(a) 35 km/hr (b) 33 km/hr
3 seconds. If the length of platform is 140 m, what is
speed of the train?
1 (a) 36 m/s (b) 5 m/s
(c) 30 km/hr (d) 37 km/hr
12
(c) 10 m/s (d) 15 m/s
21. It takes eight hours for a 600 km journey if 120 km is
14. In covering certain distance, the speeds of A and B
done by train and rest by car. It takes 20 minutes
are in the ratio of 3 : 4 A takes 30 minutes more than
more if 200 km is done by train and the rest by car.
B to reach the destination. The time taken by A to
The ratio of the speed of the train to that of car is:
reach the destination is :
(a) 2 : 3 (b) 3 : 2
1 (c) 4 : 3 (d) 3 : 4
(a) 1 hour (b) 1 hours
2 22. Aditya is travelling on his cycle and has calculated
1 that he will reach point A at 2 pm, if he travel at 10
(c) 2 hours (d) 2 hours km/h. He will reach there at 12 noon, if he travel at
2 15 km/h. At what speed must he travel to reach D at
15. A car starts with the speed of 70 km/h with its speed 1 pm?
increasing every two hours by 10 km/h. In how many (a) 10 kmph (b) 11 kmph
hours will it cover 345 km?
(c) 12 kmph (d) 13 kmph
1 23. A train running at a speed of 40 m/s crosses a pole in
(a) 2 hours (b) 4 hours
4 21 seconds less than the time it required to cross a
bridge 3.5 times its length at the same speed. What is
1 the length of the bridge?
(c) 4 hours (d) 3 hour
2 (a) 1080 m (b) 240 m
16. Aditya rides a cycle at the speed of 15 km/h but stops (c) 840 m (d) 560 m
for 10 minute to take rest every 20 kms. How much 24. Two trains A and B start at the same time from Delhi
time will he take to cover a distance of 150 kms? and Patna respectively towards each other they met
(a) 11 hours 10 minutes after 16 hours. If the distance between Delhi and
(b) 11 hours Patna is 1872 km and train B runs 27 km/h faster
than train A what is the speed of train A?
(c) 12 hours 30 minutes
(a) 45 km/h (b) 72 km/h
(d) 15 hours
(c) 48 km/h (d) 60 km/h

288 @BEST300MCQ For More Study Material


Visit: studyiq.com
25. Two trains M and N crosses a pole in 25 seconds and 33. A train 800 meter long is running at the speed of 78
1 min 15 seconds respectively. Length of train M is km/hr. if it crosses a tunnel in 1 minute, then the
half length of train N. What is the respective ratio length of the tunnel (in meter) is :
between speed of train M and train N? (a) 772 (b) 500
(a) 2 : 3 (b) 1 : 2 (c) 1300 (d) 13
(c) 2 : 1 (d) 3 : 2 34. A cyclist, after cycling a distance of 70 km on the
26. A truck cover a distance of 396 km in 5 hour 30 second day, finds that the ratio of distance covered
4 by him on the first two days is 4 : 5. If he travels a
minutes. The average speed of bike is of average distance of 42 km. on the third day, then the ratio of
3
speed of truck. What is the time taken by bike to cover distances travelled on the third day and the first day
distance of 12 km less than the distance which the is:
truck cover? (a) 3 : 4 (b) 5 : 2
(a) 4 Hrs (b) 3 Hrs (c) 9 : 4 (d) 9 : 13
(c) 5 Hrs (d) 4 hour 15 minutes 35. A thief is noticed by a policeman from a distance of
27. Train A starts its journey from Patna to Hazipur while 200m. The thief starts running and the policeman
train B starts from Hazipur to Patna. After crossing chases him. The thief and the policeman run at the
each other they finish their journey in 81 hours and rate of 10 km. and 11 km. per hour respectively. What
121 hours respectively. Then what will be speed of train is the distance between them after 6 minutes?
B if train A speed is 44 km/h? (a) 100 m (b) 190 m
(a) 44 Km/h (b) 55 Km/h (c) 200 m (d) 150 m
(c) 36 Km/h (d) 46 Km/h 36. A boy started from his house by bicycle at 10 a.m. at a
28. Two train start at same time from Delhi and Mumbai speed of 12 km per hour. His elder brother started
and comes towards each other at the rate of 85 km after 1 hr 15 mins by scooter along the same path and
and 65 km per hour respectively. When they meet it caught him at 1.30 p.m. The speed of the scooter will
is found that one train has travelled 20 km more than be (in km/hr):
the other. Find the distance between Delhi and (a) 4.5 (b) 36
Mumbai?
(a) 140 Km (b) 75 Km 2
(c) 18 (d) 9
(c) 150 Km (d) Cannot be determined 3
29. A train start from Delhi on Sunday at 9 am with speed 37. A student walks from his house at a speed of 2 km
of 25 km/h, another train starts from there on same per hour and reaches his school 6 minutes late. The
day at 40 km/h at 3 pm in the same direction. Find at next day he increases his speed by 1 km per hour and
what distance from Delhi both train will meet? reaches 6 minutes before school time. How far is the
(a) 200 Km (b) 300 Km school from his house?
(c) 400 Km (d) Cannot be determined
5 6
30. An aeroplane covers a certain distance at a speed of (a) km (b) km
4 5
240 km per hour in 5 hours. To cover the same
2 9 11
distance in 1 hours, it must travel at a speed of : (c) km (d) km
3 4 4
(a) 300 km./hr. (b) 360 km./hr 38. A moving train crosses a man standing on a platform
and a bridge of 300 metres long in 10 seconds and 25
(c) 600 km./hr (d) 720 km./hr
seconds respectively. What will be the time taken by
31. A boy goes to his school from his house at a speed of
the train to cross a platform 200 metres long?
3 km/hr. If he takes 5 hours in going and coming, the
distance between his house and school is: 2
(a) 16 seconds (b) 18 seconds
(a) 6 km (b) 5 km 3
(c) 5.5 km (d) 7.5 km (c) 20 seconds (d) 22 seconds
32. If a train runs at 40 km/hour, it reaches its destination 39. A, B and C start at the same time in the same direction
late by 11 minutes. But if it runs at 50 km/hour, it is
to run around a circular stadium. A completes a round
late by 5 minutes only. The correct time (in minutes) in 252 seconds, B in 308 seconds and C in 198
for the train to complete the journey is seconds, all starting at the same point. After what
(a) 13 (b) 15 time will they next meet at the starting point again?
(c) 19 (d) 21

289 @BEST300MCQ For More Study Material


Visit: studyiq.com
(a) 46 minutes 12 seconds 40. A and B run a kilometer and A wins by 25 sec. A and
(b) 45 minutes C run a kilometer and A wins by 275 m. When B and
(c) 42 minutes 36 seconds C run the same distance, B wins by 30 sec. The time
taken by A to run a kilometer is:
(d) 26 minutes 18 seconds
(a) 2 min 25 sec. (b) 2 min 50 sec.
(c) 3 min 20 sec. (d) 3 min 30 sec.

1. Aditya covered a certain distance at some speed. Had (a) 4.15 o’clock (b) 5.15 o’clock
he moved 3 km/h faster he would have taken 40 (c) 6.25 o’clock (d) Can’t be determined
minute less. If he had moved 2 km/h slower he would 7. Two trains start at the same time from two stations
have taken 40 minutes more. The distance is: and proceed towards each other at the rates of 20 km
(a) 40 (b) 35 and 25 km per hour respectively. When they meet, it
(c) 49 (d) 45 is found that one train has travelled 80 km more than
2. A bus take 15 hours to go from Delhi to Chandigarh. the other. Find the distance between the two stations.
Bus go one-third time with speed of 80 km/h. 40% of (a) 720 km (b) 740 km
2 (c) 760 km (d) 780 km
rest time with speed of 70 km/h. rd of rest of time 8. A goods train and a passenger train are running on
3
parallel tracks in the same direction. The driver of
with speed of 85 km/h rest with speed of 100 km/h.
the goods train observes that the passenger train
What is the distance that cover by bus?
coming from behind overtakes and crossed his train
(a) 1200 (b) 860
completely in 60 seconds. Whereas a passenger on
(c) 1120 (d) 1220 the passenger train marks that he crossed the goods
3. In a race of one kilometer, A gives B a start of 100 train in 40 seconds. If the speed of the train be in the
metres and still wins by 20 seconds. But if A gives ratio of 1 : 2, find the ratio of their lengths.
B a start of 25 seconds, B wins by 50 metres. The time (a) 1 : 2 (b) 2 : 1
taken by A to run one kilometer is:
(c) 3 : 2 (d) 2 : 3
500 1200 9. A train after travelling 50 km meets with an accident
(a) seconds (b) seconds
29 29 3
and then proceeds at of its former speed and arrives
800 700 4
(c) seconds (d) seconds
29 29 at its destination 35 minutes late. Had the accident
4. A man covers a certain distance between his house occurred 72 km further, it would have reached the
and office on scooter. Having an average speed of 30 destination only 15 minutes late. The normal speed
km/hr, he is late by 10 min. However, with a speed of of the train is
40 km/hr, he reaches his office 5 min earlier. Find (a) 36 km/hr. (b) 38 km/hr.
the distance between his house and office. (c) 46 km/hr. (d) 72 km/hr.
(a) 20 km (b) 25 km 10. Two trains measuring 100 m and 80 m respectively,
(c) 30 km (d) 35 km run on parallel lines of track. When travelling in
5. The distance between two stations, Delhi and opposite directions they are observed to pass each
Amritsar, is 450 km. A train starts at 4 p.m. from Delhi other in 9 seconds, but when they are running in the
and moves towards Amritsar at an average speed of same direction at the same rate as before, the faster
60 km/hr. Another train starts from Amritsar at 3.20 train passed the other in 18 seconds. Find the speed
p.m. and moves towards Delhi at an average speed of the two trains in km per hour.
of 80 km/hr. How far from Delhi will the two trains (a) 12 km/hr, 5 km/hr
meet and at what time? (b) 14 km/hr, 18 km/hr
(a) 5.30 p.m. (b) 5.50 a.m. (c) 16 km/hr, 54 km/hr
(c) 6.50 p.m. (d) 6.30 a.m. (d) 18 km/hr, 54 km/hr
6. A, B and C can walk at the rates of 3, 4 and 5 km per 11. A 300 meters long train is travelling with the speed
hour respectively. They starts from Pune at 1, 2, 3 of 45 km/hr when it passes point A completely. At
o’clock respectively. When B catches A, B sends him the same time, a motorbike starts from point A with
back with a message to C. When will C get the the speed of 70 km/hr. When it exactly reaches the
message? middle point of the train, the train increases its speed

290 @BEST300MCQ For More Study Material


Visit: studyiq.com
to 60 km/hr and motorbike reduces its speed to 65 17. A dog finds a cat at 25 leaps away. The cat sees the
km/hr. How much distance will the motorbike travel dog coming towards it and starts running with the
while passing the train completely? dog in hot pursuit. In every minute, the dog makes 5
(a) 3.8 km (b) 3.2 km leaps and the cat makes 6 leaps and one leap of the
(c) 2.37 km (d) 2.2 km dog is equal to 2 leaps of the cat. Find the time in
12. Two trains, Kanpur Mail and Delhi Mail, start at the which the dog will catch the cat.
same time from stations Kanpur and Delhi (a) 12.5 minutes (b) 13 minutes
respectively towards each other. After passing each (c) 11.5 minutes (d) 10.5 minutes
other, they take 12 hours and 3 hours to reach Delhi 18. Two guns were fired from the same place at an
and Kanpur respectively. If the Kanpur Mail is interval of 13 minutes but a person in a train
moving at the speed of 48 km/hr, the speed of the approaching the place hears the second shot 12
Delhi Mail is: minutes 30 seconds after the first. Find the speed of
(a) 90 km/hr (b) 96 km/hr the train, supposing that sound travels at 330 metres
(c) 86 km/hr (d) 84 km/hr per second.
13. Heena and Renu are competing in a 100 meters race. 13 11
Initially, Heena runs at twice the speed of Renu for (a) 47 km / hr (b) 47 km / hr
25 25
the first fifty meters. After the 50 meters mark, Heena
9 7
1 (c) 47 km / hr (d) 47 km / hr
runs at th her initial speed while Renu continues 25 25
4
to run at her original speed. If Renu catches up with 19. A man walks from A to B and back in a certain time at
Heena at a distance of N meters from the finish line, 1
then N is equal to: the rate of 3 km per hour. But if he had walked
2
(a) 75 meters (b) 55 meters from A to B at the rate of 3 km an hour and back from
(c) 25 meters (d) 30 meters B to A at the rate of 4 km an hour, he would have
14. A horse rider travels on horse back from Bhopal to taken 5 minutes longer. Find the distance between A
Chandigarh at a constant speed. If the horse and B.
increased its speed by 6 km/hr, it would take the (a) 14 km (b) 12 km
rider 4 hours less to cover that distance. And (c) 6 km (d) 7 km
travelling with a speed 6 km/hr lower than the initial 20. A carriage driving in a fog passed a man who was
speed, it would take him 10 hours more than the time walking at the rate of 3 km an hour in the same
he would have taken if he had travelled at a speed 6 direction. He could see the carriage for 4 minutes
km/hr higher than the initial speed. Find the distance and it was visible to him upto a distance of 100m.
between Bhopal and Chandigarh? What was the speed of the carriage?
(a) 720 km (b) 680 km
1 1
(c) 560 km (d) 480 km (a) 5 km per hour (b) 4 km per hour
2 2
15. Seeta and Geeta cycled towards each other, one from
point A and the other from point B, respectively. Seeta 1 1
left point A 6 hours later than Geeta left point B, and (c) 3 km per hour (d) 8 km per hour
2 2
it turned out on their meeting that Seeta had travelled
21. A man leaves a point P and reaches the point Q in 4
12 km less than Geeta. After their meeting, they kept
hours. Another man leaves the point Q, 2 hours earlier
cycling with the same speed, and Seeta arrived at B 8
and reaches the point P in 4 hours. Find the time in
hours later and Geeta arrived at A 9 hours later. Find
which the first man meets to the second man.
the speed of the faster cyclist.
(a) 2 hours (b) 3.5 hours
(a) 3 km/hr (b) 6 km/hr
(c) 1 hours (d) 3 hours
(c) 9 km/hr (d) 12 km/hr
22. A man travels 360 km in 4 hrs, partly by air and
16. Two cyclists started simultaneously towards each
partly by train. If he had travelled all the way by air,
other and meet each other 3 hours 20 minutes later.
How much time will it take the slower cyclist to cover 4
he would have saved of the time he was in train
the whole distance if the first arrived at the place of 5
departure of the second 5 hours later than the second and would have arrived at his destination 2 hours
arrived at the point of departure of the first? early. Find the distance he travelled by train?
(a) 5 hours (b) 10 hours (a) 75 km (b) 90 km
(c) 15 hours (d) 20 hours (c) 85 km (d) 80 km

291 @BEST300MCQ For More Study Material


Visit: studyiq.com
23. One aeroplane started 30 minutes later than the (a) 24 km/hr. (b) 25 km/hr.
scheduled time from a place 1500 km away from its (c) 26 km/hr. (d) 27 km/hr.
destination. To reach the destination at the scheduled
25. A hare sees a dog 100 metres away from her and run
time the pilot had to increase the speed by 250 km/
off in the opposite direction at a speed of 12 km an
hr. What was the speed of the aeroplane per hour
hour. A minute later the dog perceives her and gives
during the journey?
chase at a speed of 16 km per hour. How much
(a) 750 km/hr (b) 755 km/hr distance cover by hare after being spotted?
(c) 760 km/hr (d) 745 km/hr
(a) 750 metres (b) 800 metres
24. A train leaves the station 1 hour before the scheduled
(c) 850 metres (d) 900 metres
time. The driver decreases its speed by 4 km/hr. At
the next station 120 km away, the train reached on
time. Find the original speed of the train.

1. A train 180 m long moving at the speed of 20m/sec. (a) 100 (b) 80
overtakes a man moving at a speed of 10 m/sec in the (c) 75 (d) 120
same direction. The train passes the man in: 8. A man standing on a platform finds that a train takes
(a) 6 sec (b) 9 sec 3 seconds to pass him and another train of the same
(c) 18 sec (d) 27 sec length moving in the opposite direction, takes 4
2. How many seconds will a 500 metre long train take to seconds. The time taken by the trains to pass each
cross a man walking with a speed of 3 km/hr. in the other will be:
direction of the moving train if the speed of the train
is 63 km/hr? 3 3
(a) 2 seconds (b) 3 seconds
(a) 25 sec (b) 30 sec 7 7
(c) 40 sec (d) 45 sec
3 3
3. The length of a train and that of a platform are equal. (c) 4 seconds (d) 5 seconds
If with a speed of 90 km/hr the train crosses the 7 7
platform in one minute, then the length of the train (in 9. A train passes two persons walking in the same
metres) is: direction at a speed of 3 km/hour and 5 km/hour
(a) 500 (b) 600 respectively in 10 seconds and 11 seconds
(c) 750 (d) 900 respectively. The speed of the train is:
4. A train passes two bridges of length 800 m and 400 m (a) 28 km/hour (b) 27 km/hour
in 100 seconds and 60 seconds respectively. The (c) 25 km/hour (d) 24 km/hour
length of the train is: 10. After travelling 3 hours a train meets with an accident
(a) 80 m (b) 90 m due to this it stops for an hour. After this the train
(c) 200 m (d) 150 m moves at 75% speed of its original speed and reaches
5. Two trains are running with speeds 30 km/hr and 58 to destination 4 hours late. If the accident would occur
km/hr in the same direction. A man in the slower at 150 km ahead in the same line then the train reaches
train passes by the faster train in 18 seconds. The only 3.5 hours late. Then find the distance of journey
length (in metres) of the faster train is: and the original speed of the train?
(a) 70 m (b) 100 m (a) 100 km/h, 1200 km (b) 150 km/h, 1200 km
(c) 128 m (d) 140 m (c) 75 km/h, 1000 km (d) 125 km/h, 900 km
6. Two trains are moving on two parallel tracks but in 11. A train starts from A at 7 a.m. towards B with speed
opposite directions. A person sitting in the train 50 km/h. Another train from B starts at 8 a.m. with
moving at the speed of 80 km/hr passes the second speed 60 km/h towards A. Both of them meet at 10
train in 18 seconds. If the length of the second train is a.m. at C. The ratio of the distance AC to BC is:
1000 m, its speed is:
(a) 5 : 6 (b) 5 : 4
(a) 100 km/hr (b) 120 km/hr
(c) 6 : 5 (d) 4 : 5
(c) 140 km/hr (d) 150 km/hr
7. Two trains travel in the same direction at the speeds 12. Two trains start from a certain place on two parallel
of 56 km/h and 29 km/h respectively. The faster train tracks in the same direction. The speed of the trains
passes a man in the slower train in 10 seconds. The are 45 km/hr and 40 km/hr respectively. The distance
length of the faster train (in metres) is: between the two trains after 45 minutes will be:

292 @BEST300MCQ For More Study Material


Visit: studyiq.com
(a) 2 km 500 m (b) 2 km 750 m (a) 20 km/h (b) 60 km/h
(c) 3 km 750 m (d) 3 km 250 m (c) 40 km/h (d) 80 km/h
13. Two trains start from stations A and B and travel 19. Two trains started at the same time, one from A to B
towards each other at speeds of 50 km/hour and 60 and the other from B to A. If they arrived at B and A
km/hour respectively at same time. At the time of their respectively 49 hours and 81 hours after they passes
meeting, the second train has travelled 120 km more each other, the ratio of the speeds of the two trains
than the first. The distance between A and B is: was:
(a) 990 km (b) 1200 km (a) 2 : 1 (b) 9 : 7
(c) 1320 km (d) 1440 km (c) 7 : 9 (d) 5 : 4
14. A train of 24 m length runs with a speed of 250 m/s. 20. Two trains, A and B, start from stations X and Y
A man in train at the tail end of the train runs with a respectively. After passing each other, they take 4
speed of 10 m/s. When he reaches the front end he hours 48 minutes and 3 hours 20 minutes to reach Y
turns back with a speed of 6 m/s and this process and X respectively. If train A is moving at 45 km/hr.,
continues. How many rounds (up and down) he will then the speed of the train B is:
complete if the train runs 8 kms, providing that during
(a) 60 km/hr (b) 64.8 km/hr
running he will not loose contact with the train?
(c) 54 km/hr (d) 37.5 km/hr
(a) 8 rounds (b) 4 rounds
(c) 5 rounds (d) 7 rounds 21. A train travels some distance with a uniform speed. If
the speed of the train is 20 km/hr less then it takes 16
15. A train Aligarh express leaves Delhi for Aligarh at hours more. If its speed is 10 km/hr more then it takes
14:30 pm and goes at the rate of 60 km an hour and an 5 hours less during the whole journey. Then find the
another train, Rajdhani express leaves Delhi for distance ? And also find the speed of the train and
Aligarh at 16 : 30 pm and goes at the rate of 80 km/h. how much time will it take to travel the whole
How far from Delhi will two trains meet if they are journey?
moving in the same direction?
(a) 2800 km, 70 km/h, 40 hours
(a) 120 km (b) 360 km
(b) 3500 km, 70 km/h, 40 hours
(c) 480 km (d) 500 km
(c) 4200 km, 140 km/h, 30 hours
16. Two trains starting at the same time from two stations
650 km apart and going in opposite direction they (d) None of these
meet each other after 10 hours. If one starts 4 hour 20 22. A train 550 m long is running with a speed of 55 km
min late from another train then they meet after 8 hours per hour. In what time will it pass a man who is
from the starting of another train. Then find the walking at 10 km per hour in the same direction in
average speed of trains? which the train is going?
(a) 30 km/h, 35 km/h (b) 45 km/h, 20 km/h (a) 42 sec (b) 33 sec
(c) 32 km/h, 33 km/h (d) 25 km/h, 40 km/h (c) 40 sec (d) 44 sec
17. A man can see only 50 m in fog. He was walking at a 23. Two trains are moving in the opposite direction at 24
speed of 4 km/hr and saw a train was coming behind km and 12 km/hr. The faster train crosses a man in
him. The train was 20 m in length and disappeared the slower train in 2 seconds. Find the length of the
from his eyes after 30 seconds. Find the speed of the faster train:
train? (a) 25 m (b) 30 m
2 3 (c) 20 m (d) Data inadequate
(a) 18 km/hr (b) 6 km/hr
5 5 24. In covering a certain distance, the speeds of A and B
are in the ratio of 3 : 4. A takes 30 minutes more than
1 2 B to reach the destination. The time taken by A to
(c) 8 km/hr (d) 4 km/hr
5 5 reach the destination is:
18. Two trains start from the same point simultaneously (a) 1 hour (b) 1 hour 30 min
and in the same direction. The first train travels at 40 (c) 2 hours (d) 2 hours 30 min
km/h, and the speed of the second train is 25 per cent
more than the speed of the first train. Thirty minutes 25. A man reduces his speed to 2/3, he takes 1 hour more
later, a third train starts from the same point and in in walking a certain distance. The time (in hours) to
the same direction. It overtakes the second train 90 cover the distance with his normal speed is:
minutes later than it overtook the first train. What is (a) 2 hrs (b) 1 hrs
the speed of the third train? (c) 3 hrs (d) 1.5 hrs

293 @BEST300MCQ For More Study Material


Visit: studyiq.com
26. A and B start at the same time with speeds of 40 km/ (a) 6 km (b) 5 km
hr and 50 km/hr respectively. If in covering the (c) 5.5 km (d) 6.5 km
journey A takes 15 minutes longer than B, the total 34. A car completes a journey in 10 hours. If it covers half
distance of the journey is: of the journey at 40 kmph and the remaining half at
(a) 46 km (b) 48 km 60 kmph, the distance covered by car is:
(c) 50 km (d) 52 km (a) 400 km (b) 480 km
1 (c) 380 km (d) 300 km
27. A car can cover a certain distance in 4
hours. If the 35. Two cars start at the same time from one point and
2
speed is increased by 5 km/hour, it would take move along two roads at right angles to each other.
Their speeds are 36 km/hour and 48 km/hour
1 respectively. After 15 seconds the distance between
hour less to cover the same distance. Find the slower
2 them will be:
speed of the car: (a) 400 m (b) 150 m
(a) 50 km/hour (b) 40 km/hour (c) 300 m (d) 250 m
(c) 45 km/hour (d) 60 km/hour 36. A runs twice as fast as B and B runs thrice as fast as C.
28. A and B started at the same time from the same place The distance covered by C in 72 minutes, will be
covered by A in:
5
for a certain destination. B walking at of A’s speed (a) 18 minutes (b) 24 minutes
6
(c) 16 minutes (d) 12 minutes
reached the destination 1 hour 15 minutes after A. B
reached the destination in: 37. The speeds of two trains are in the ratio 6 : 7. If the
second train runs 364 km in 4 hours, then the speed
(a) 6 hours 15 minutes (b) 7 hours 15 minutes
of first train is:
(c) 7 hours 30 minutes (d) 8 hours 15 minutes
(a) 60 km/hr (b) 72 km/hr
29. Two rifles are fired from the same place at a difference
(c) 78 km/hr (d) 84 km/hr
of 11 minutes 45 seconds. But a man who is coming
towards the place in a train hears the second sound 38. A truck covers a distance of 550 metres in 1 minute
after 11 minutes. Find the speed of train (assuming whereas a bus covers a distance of 33 kms in 45
speed of sound = 330 m/s): minutes. The ratio of their speeds is:
(a) 72 km/h (b) 36 km/h (a) 4 : 3 (b) 3 : 5
(c) 81 km/h (d) 108 km/h (c) 3 : 4 (d) 50 : 3
30. An aeroplane covers a certain distance at a speed of 39. Three cars travelled distances in the ratio 1 : 2 : 3. If
240 km/hour in 5 hours. To cover the same distance the ratio of the time of travel is 3 : 2 : 1, then the ratio of
their speeds is:
2 (a) 3 : 9 : 1 (b) 1 : 3 : 9
in 1 hours, it must travel at a speed of:
3 (c) 1 : 2 : 4 (d) 4 : 3 : 2
(a) 30 km./hr. (b) 360 km./hr. 40. If a man walks 20 km at 5 km/hr, he will be late by 40
(c) 600 km./hr. (d) 720 km./hr. minutes. If he walks at 8 km per hr, how early from
31. A car travelling at a speed of 40 km/hour can complete the fixed time will he reach?
a journey in 9 hours. How long will it take to travel (a) 15 minutes (b) 25 minutes
the same distance at 60 km/hour?
(a) 6 hours (b) 3 hours 1
(c) 50 minutes hours
(d) 1
2
1
(c) 4 hours (d) 4 hours 41. In covering a distance of 30 km, Abhay takes 2 hours
2 more than Sameer. If Abhya doubles his speed, then
32. A man travelled a certain distance by train at the rate he would take 1 hour less than sameer. Abhay’s speed
of 25 kmph. and walked back at the rate of 4 kmph. If (in km/hr) is:
the whole journey took 5 hours 48 minutes, the (a) 5 (b) 6
distance was: (c) 6.25 (d) 7.5
(a) 25 km (b) 30 km 42. A bus meets with an auto at 10 : 00 am while going on
(c) 20 km (d) 15 km the same way in the same direction towards
33. A boy goes to his school from his house at a speed of Haridwar. The Bus reach at Haridwar at 12 : 30 p.m.
3 km/hr and returns at a speed of 2 km/hr. If he takes and take 1 hour rest at there. Bus return back on the
5 hours in going and coming, the distance between same way and meet with the same auto half an hour
his house and school is: later. At what time the Auto will reach at Haridwar:

294 @BEST300MCQ For More Study Material


Visit: studyiq.com
(a) 3 pm (b) 4 pm (a) 20 km (b) 35 km
(c) 3 : 30 pm (d) 5 pm 2
(c) 36 km (d) 40 km
43. Two swimmers started simultaneously from the 3
beach, one to the south and the other to the East. Two 47. When a bus stops at different stations then its average
hours later, the distance between them turned out to speed is 40 km/h. But when it travels without
be 100 km. Find the speed of the faster swimmer, stoppage the average speed of the bus is 50 km/h.
knowing that the speed of one of them was 75% of the Then find how many minutes the bus stops in an
speed of the other: hour?
(a) 30 kmph (b) 40 kmph (a) 12 min (b) 15 min
(c) 45 kmph (d) 60 kmph (c) 25 min (d) 10 min
44. A constable is 114 metres behind a thief. The constable 48. A boy rides his bicycle 10 km at an average speed of
runs 21 metres and the thief 15 metres in a minute, IN 12 km per hr and again travels 12 km at an average
what time will the constable catch the thief? speed of 10 km per hr. His average speed for the entire
trip is approximately:
(a) 19 minutes (b) 18 minutes
(a) 10.4 km/hr (b) 10.8 km/hr
(c) 17 minutes (d) 16 minutes (c) 11.0 km/hr (d) 12.2 km/hr
45. A thief, who was chased by a policeman was 100 49. A train moves with a speed of 30 kmph for 12 minutes
metres ahead of the policeman initially. If the ratio and for next 8 minutes at a speed of 45 kmph. Find the
between speeds of the policeman and the thief is 5 : 4, average speed of the train:
then how long thief would have covered before he (a) 37.5 kmph (b) 36 kmph
was caught by the policeman? (c) 48 kmph (d) 30 kmph
(a) 80 m (b) 200 m 50. A man completed a certain journey by a car. If he
(c) 400 m (d) 600 m covered 30% of the distance at the speed of 20 km/hr,
46. Ram covered a certain distance at some speed. Had 60% of the distance at 40 km/hr and the remaining
he moved 3 km per hour faster, he would have taken distance at 10 km/hr his average speed for the whole
40 minutes less. If he had moved 2 km per hour slower, journey was:
he would have taken 40 minutes more. The distance (a) 25 km/hr (b) 28 km/hr
(in km) is: (c) 30 km/hr (d) 33 km/hr

S olut ions 110


4. (d); Speed of the train = m/sec.
3

(c); Average speed =


50 + 40 + 90 ( 165 + 110 )
1. Required time = ´3
50 40 90 110
+ +
25 20 15
275
= ´ 3 = 7.5 sec.
180 180 110
= = = 18 kmph
2+2+6 10 5. (d); (Length of the 1st train + Length of the tunnel)
= speed × time
total distance 5 ´ 6 + 4 ´ 12
2. (c); Average speed= total time = 6 + 12 x + 700 =
72 ´ 5
´ ( 1 ´ 60 )
18
30 + 48 78 13 1 x + 700 = 20 × 60
= = = = 4 km/hr
18 18 3 3 x + 700 = 1200 Þ x = 1200 – 700
x = 500 metres.
1
10 17
distance 51 1 200 + 200
3. (b); Speed = = 5 = ´ = km/hr 6. (c); Speed of the train =
time 3 5 3 5 20
\ Required distance = speed × time
400 18
17 = = 20 m/sec = 20 ´ km/hr
= ´ 5 = 17 km 20 5
5 = 72 km/hr

295 @BEST300MCQ For More Study Material


Visit: studyiq.com
7. (b); Since the trains are moving in opposite direction
2xy 2 ´ 12 ´ 18
\Relative speed = speed of first train + speed of 11. (a); Average speed = =
second train x+y ( 12 + 18)
Let speed of second train = x km/hr 72 2
= = 14 km/hr
5 125 + 125 5 250 5 5
\ ( 65 + x ) ´ = , ( 65 + x ) ´ =
12. (b); From the formula –
18 6 18 6
(65 + x) = 150 Þ x = 150 – 65 Þ x = 85 km/hr. \ Required ratio = b : a = 9 : 4 =3:2
8. (c); Let usual time = t, distance = d, and speed = s 13. (b); Required ratio = b: a
d = 25 : 16 = 5 : 4
\s = ... (i)
t 14. (d); Let length of the platform = x m

3 5 320 + x
s= d \ 120 ´ =
and ... (ii) 18 24
5 1
t+2 800 = 320 + x Þ x = 480 m
2
From equation (i) ÷ equation (ii) 480 120
\ Speed of the man = = = 2.0 m/sec.
4 ´ 60 60
d 5 15. (c); Average speed
5
t+
s
= t Þ = 2 Þ 5t = 3t + 15
3 d 3 t 2 39 + 25 64 ´ 60
s = = = 48 km/hr
5 1 80
5
t+ ( 45 + 35) ´
2 60
16. (b); Let length of the platform = x
15 15 3
2t = Þ t= = 3 hours
\ 108 ´
5 ( 280 + x ) Þ 360 = 280 + x
2 4 4 =
18 12
9. (c); At normal speed, Let usual time = t
x = 80 metres
d 80
\s= ... (i) \ Speed of Boy = = 8 m/sec
t 10
7 d 224
and s = ... (ii) 17. (b); Speed of the truck = = 56 km/hr
11 22 4
From equation (i) ÷ equation (ii)
56
\ average speed of the bike = = 14 km/hr
d 4
s 11 22 \ Required distance = 14 × 7 = 98 km
= t Þ = Þ t = 14 hours.
7 d 7 t 18. (b); When person walks at 14 km/h
s
11 22 d
\ 14 = ... (i)
\ saved time = (22 – 14) hrs = 8 hrs. t
10. (c); Let required time = t hours; Now speed = 10 km/h

d ( d + 20 )
\ s= ... (i) 10 = ... (ii)
t t
\ From eqn(i) ÷ eqn(ii)
3 d d
s = ... (ii)
4 (t + 2) 14 t
=
10 (d + 20)
d t
s t 4 t+2
= Þ = 14 d
3 d 3 t = Þ 14d + 280 = 10d Þ 4d = 280
s 10 d + 20
4 (t + 2)
d = 70km
4t = 3t + 6 Þ t = 6 hours \ Actual distance = 70 – 20 = 50 km

296 @BEST300MCQ For More Study Material


Visit: studyiq.com
19. (a); Given adult fair = 102
distance 5 d
102 28. (a); Speed = Þ 66 ´ =
\ Child fair = = 34 time 18 18
3 d = 330 m
Required fair = 2 ( 102 ) + 3 ( 34 ) = 204 + 102 = 306
120
20. (a); Average speed 29. (c); Speed of first train (S1) = = 12 m/sec.
10
total distance 75 + 25 + 50
= = 120
total time 75 25 50 Speed of second train (S2) = = 8 m/sec.
+ + 15
25 5 25
150 150 120 + 120 240
= = =15 km/hr \ Required time = = = 12 sec
3+5+2 10 ( 12 + 8 ) 20
distance 5 700 + 500
21. (b); Speed = , 54 ´ = 2xy 2 ´ 70 ´ 55
time 18 t 30. (a); Average speed = =
x+y 125
1200
t= sec Þ t = 80 sec.
15 = 61.6 km/hr
31. (a); Let distance = d, time = t
1000
22. (c); Running time = = 20 hrs.
50 d
\ 10 = ...(i)
20 15
No. of times that he will rest = »6 t+
3 60
\ Rest time = 6 × 20 = 120 min = 2 hours
\ Total time = (20 + 2) = 22 hrs. d
12 = ...(ii)
5
330 t+
23. (c); Time taken by car = = 6 hrs. 60
55
From equation (i) ÷ equation (ii)
( 330 - 15 ) =
315
\ Average speed of bike = 1
(6 - 1) 5 10
t+
12 Þ 10 = 4 ´ ( 12t + 1) Þ t = 3 hr
=
= 63 km/hr 12 1 12 12 ( 4t + 1) 4
t+
24. (a); distance = 40 × 9 = 360 km 4
360
Required time = = 6 hrs. æ 3 15 ö 60
60 d = 10 ´ ç + ÷ = 10 ´ = 10km
è 4 60 ø 60
distance 5 75 Shortcut:
25. (c); Speed = Þ 20 ´ =
time 18 t Let distance = d
75 ´ 18 54 d d 10
t= = = 13.5 seconds - = Þ d = 10 km
100 4 10 12 60
26. (a); Let Required time = t
2xy 2 ´ 21 ´ 24
5 1 00 32. (a); Average speed = =
144 ´ = Þ t = 2.5 seconds x+y 45
18 t

27. (c); Speed = distance 112


= km /hr
time 5
Let length of the platform = 2x m
112
60 ´ 5 x + 2x \ Distance = ´ 10 km = 224 km
= Þ 3x = 1.8 ´ 60 ´ 5 5
18 32.4
3x = 90 × 6 33. (d); Relative speed = (3 + 3.5) km/hr = 6.5 km/hr
x = 30 × 6 = 180 m time = 3 hrs
\ Length of the plat form = 2x = 360 m \ required distance = (6.5×3) = 19.5 km

297 @BEST300MCQ For More Study Material


Visit: studyiq.com
34. (c); Let length of the bridge = x 38. (a); Let length of the train = l1 , length of platform = l2
distance x 5 l 150 l 1
\ speed = Þ 15 = ´ 60 \ ( 25 + 5) ´ = 1 Þ = Þ l1 = 100m
time 5 18 12 18 12

75 5 5 5 100 + l 2
x= km = km = ´ 1000 Þ x = 1250 m \ 25 ´ = Þ 100 + l2 = 125 Þ l2 = 25m
60 4 4 18 18
35. (d); Speed in metres per second \ Required length (100 + 25) = 125 m
5 4 1
= 180 ´ = 50 m/sec. 39. (b); Time = ´ hrs
18 5 45

distance 5 d 4 1
36. (d); Speed = Þ 60 ´ = Þ d = 500 m = ´ ´ 60 ´ 60 sec = 64 seconds
time 18 30 5 45
37. (c); Relative speed of the trains
570 + 570
40. (c); Speed of the train =
5 15
= (50 – 30) = 20 km/hr = 20 ´ m / sec .
18
1140
= = 76 m/sec.
5 15
\ length of the faster train = 20 ´ ´ 18 = 100 m
18

1. (b); By the given statements 6. (b); Let length of each train = x metres
Distance 1 1 x
Speed = Þ p –q = Þ = p–q \ speed of first train = m/sec
Time r r 4
2. (a); Let distance = d x
speed of second train = m/sec
5
d d 5d
+ =5 Þ = 5 Þ d = 6 km x x 9x
2 3 6 Relative speed = + = m/sec
3. (b); Let distance = d 4 5 20
Time taken to cross each other
d d 25 = time train to cover 2x metres
- =
5 6 60 æ 9x ö 20 40
at ç ÷ m/sec = 2x ´ = sec.
d 25 è 12 ø 9x 9
= Þ d = 12.5 km
30 60 7. (c); Let the speed of the train = s km/hr
4. (c); Let length of train = L 5 240
( s + 3) ´= Þ 24 × 18 = 5s + 15
18 10
L + 162
\ speed = ... (i) 432 = 5s + 15 Þ 5s = 417 Þ s = 83.4 km/hr
18 8. (c); Let the time taken by train A when it meet = t
L + 120 Let the time taken by train B when it meet = (t – 1)
speed = ...(ii) 60 × t + 75 (t – 1) = 330 Þ 60t + 75t – 75 = 330
15
135 t = 405 Þ t = 3 hr
From equation (i) and equation(ii) \ both train will meet at 11 A.M.
L + 162 L + 120 9. (b); Let they meet in t min.
= 5t + 10t = 1200
18 15
15t = 1200
5L + 810 = 6L + 720 Þ L = 90
t = 80 min
5. (b); Relative speed of two train = (50 – 40) 10. (b); Relative speed = (77 + 67) km/hr = 144 km/hr
5 144 ´ 5
= 10 kmph = 10 ´ m/s = m/sec= 8 × 5 = 40 m/sec.
18 18

Required time =
120 + 80
´ 18 =
200
´ 18 = 72 sec. ( 160 + 140) =
300
\ Required time = = 7.5 sec.
50 50 40 40

298 @BEST300MCQ For More Study Material


Visit: studyiq.com
17. (a); Let speed of bike = 15x
120
11. (d); Speed of first train (S1) = = 12 m/sec Speed of train = 27x
10
720
120 \ speed of bus = = 80 km/hr
Speed of second train (S2) = = 8 m/sec 9
15
3
\ Relative speed = S1 – S2 = 4 m/sec 15x = (80) Þ 15x = 60 Þ x = 4
4
120 + 120 240 Speed of train = 108 km/hr
\ Required time = = = 60 sec
4 4 \ Required distance = (108 × 7) km = 756 km

total distance 44
12. (a); Average speed = 18. (d); Distance = ´ 50 km
total time 60
\ time when speed of bus increased by
2000 44 1 4 4
= 5 km/hr = ´ 50 ´ = hrs = ´ 60 min.
3000 + 8000 + 500 + 800
60 55 6 6
400
= 40 min
2000 ´ 400 8000 5 19. (b); Let speed of bus = 2x
= = = 65 km/hr \ speed of train = 3x
12300 123 123
13. (b); Let total distance = d km 75 75 12.5 125 75 æ 1 1 ö
- = Þ = ç - ÷
2 x 3x 60 600 x è2 3ø
total distance
\ average speed = 75 æ 1 ö 125 75
total time = ç ÷ Þ 100 = x
x è6ø
d
= 75 ´ 100
d 1 d 1 5d 1 x= Þ x = 4 × 15 Þ x = 60 km/hr
´ + ´ + ´ 125
3 25 4 30 12 50
\ speed of bus = 2 × 60 = 120 km/hr
1 1 20. (c); Length of platform = 140 m
= =
1 1 5 8+5+5 Let length of train = x m
+ +
75 120 600 600
40
x + x´ = 140 Þ x = 100 m
600 300 100 1 100
= = = = 33 km/hr
18 9 3 3
100
14. (c); Let speed of A = 3x Speed = = 10 m/sec.
10
Let speed of B = 4x
21. (d); Let speed of train = x
æ 30 ö Let speed of car = y
3x × t = 4x ç t - ÷ Þ 3t = 4 t – 2 Þ t = 2 hrs.
è 60 ø 120 480
+ = 8 hours ... (i)
15. (c); In first two hours the distance covered by x y
car = 70 + 70 = 140 km
In the four hours distance covered by car 200 400 1
+ =8 ... (ii)
= 140 + 160 = 300 x y 3
Remaining 45km is covered by car in equation (i) × 5 – equation (ii) × 3
45 1 1 600 2400 600 1200
= hr = hr; \ Total time = 4 hrs. + = 40 Þ + = 25
90 2 2 x y x y
150 On subtracting (2) from (1)
16. (a); No. of stops that he rests = »7
20 1200 120
=15 Þ y = 80 Þ +6 =8
\ rest time = 7 × 10 = 70 min = 1 hr 10 min y x
150
\ run time = = 10 hrs 120
15 = 2 Þ x = 60
x
\ Total time = 11 hr 10 min \ Require ratio = x : y = 60 : 80 = 6 : 8 = 3 : 4

299 @BEST300MCQ For More Study Material


Visit: studyiq.com
22. (c); Let Aditya takes t hours when he travel at 29. (c); Let time taken by first train = t hrs
10 km/hr 25t = 40(t – 6), 25t = 40t – 240
d d d 15t = 240, t = 16 hrs
- =2 Þ = 2 Þ d = 60 km \ Required distance = 16 × 25 = 400
10 15 30
30. (d); Distance = 240 × 5 = 1200 km
60
\ t= = 6 hr 1200
10 \ required speed = ´ 3 = 240 × 3
5
60
\ to reach A at 1 pm, speed = = 12 kmph = 720 km/hr
5
23. (c); Let length of train = x m 5
31. (d); Distance = 3 ´ km = 7.5 km
\ Let length of bridge = 3.5x m 2
x + 3.5x x d
- = 21 Þ x + 3.5x – x = 40 × 21 32. (c); 40 = ... (i)
40 40 11
t+
60
840
3.5x = 840 Þ x = ´ 10
35 d
50 =
5 ... (ii)
840 7 t+
\ Length of bridge = ´ 2 ´ = 840 m 60
7 2
By (i) and (ii)
24. (a); Let speed of train A = x km/hr
then Let speed of train B = (x + 27) km/hr 5
\ 16x + 16(x + 27) = 1872 4 t + 60 4 60 ( 60t + 5)
= Þ =
16x + 16x = 1872 – 432 5 11 5 60 ( 60t + 11)
t+
32x = 1440 Þ x = 45 km/hr 60
25. (d); Let length of train N = x 240t + 44 = 300t + 25 Þ 60t = 19
x 19 19
\ Let length of train M = t= hr Þ t = ´ 60 min Þ t = 19 min
2 60 60
33. (b); Let length of tunnel = L
x x
\ speed of train M = = m/s 5 800 + L
2 ´ 25 50 \ 78 ´ = Þ 1300 = 800 + L Þ L = 500
18 60
x
Speed of train N = m/s 70
75 34. (a); Distance travelled by A in first day = 4 ´
5
x x 1 1
Required ratio = : = : =3 :2 = 56 km
50 75 50 75 \ Required ratio = 42 : 56 = 6 : 8 = 3 : 4
396 35. (a); Relative speed = (11 – 10) km/hr = 1 km/hr
26. (a); Speed of truck = ´ 2 = 36 × 2 = 72 km/hr
11 1´ 5
4 in 6 min., distance betwen both = ´ 6 ´ 60
Speed of bike = ´ 72 =24 × 4 = 96km/hr 18
3 = 100 m
396 - 12 384 32 \ Required distance = (200 – 100) m = 100 m
Required time = = = = 4 hrs.
96 96 8 36. (c); At 1 : 30 PM., distance covered by boy

27. (c); Ratio of their speed = b : a = 121 : 81 7


= ´ 12 = 42 km
= 11 : 9 2
Given A’s speed = 44 km Let speed of scooter = x km/hr
\ B’s speed = 36 km/hr
æ7 5ö 14 - 5
28. (c); Let distance travelled by second train = d 42 = x ´ ç - ÷ Þ 42 = x ×
è2 4ø 4
d d + 20
= 9 42 ´ 4 56
65 85 42 = x × Þ x = Þ x =
4 9 3
85d = 65d + 65×20
20d = 65×20, d = 65 km 2
x = 18 km/hr
\ Total distance = 65 + 65 + 20 = 150 km 3
300 @BEST300MCQ For More Study Material
Visit: studyiq.com
37. (b); Let distance = x km 39. (a); Time taken by A = 252 = 2 × 2 × 3 × 3 ×7
x x 12 3x - 2x 1 6 Time taken by B = 308 = 2 × 2 ×7 × 11
\ - = Þ = Þ x = km Time taken by C = 198 = 2 × 3 × 3 × 11
2 3 60 6 5 5
\ L.C.M = 2772 sec
38. (c); Let length of train = L
= 46 min 12 sec
L 40. (c); A win by B in 25 sec
\s = ... (i)
10 B win by C in 30 sec
L + 300 A win by C in 55 sec
s= ... (ii) A win by C by 275 m
25
by equation (i) and equation (ii) 275
Speed of A = = 5 m/s
L L + 300 55
= Þ 25L = 10L + 3000
10 25 1000
Time taken to run a kilometre by A = = 200
15L = 3000 Þ L = 200 m Þ s = 20 m/sec 5
200 + 200 400 = 3 min 20 sec
\ Required time = = = 20 seconds
20 20

1. (a); Let distance = d, usual speed = s km/h 4. (c); Let distance = x km


d d 40 x x 1 4x - 3x 1
- = \ - = Þ = Þ x = 30 km
s s + 3 60 30 40 4 120 4
2s (s + 3) = 9d ....(i) 5. (c); Let required time = t
d d 40
- = æ 2ö
s - 2 s 60 60 ç t - ÷ + 80t = 450 Þ
è 3ø
s(s – 2) = 3d ...(ii)
Solving equation (i) and equation (ii) 60t – 40 + 80t = 450 Þ 140t = 490
d = 40 7 1
t = Þ t =3
total distance 2 2
2. (d); Average speed =
total time \ Required time = 3 : 20 + 3 : 30 = 6 : 50 PM
distance = speed × time 6. (b); Let the time to catch A for B = t
\ 3 (t + 1) = 4t Þ t = 3hrs.
15
= ´ 80 + 4 ´ 70 + 4 ´ 85 + 2 ´ 100 \ Distance covered by A = 3 × 4 = 12 km
3
distance covered by C in 2hr = 10 km
= 400 + 280 + 340 + 200 = 1220
3. (a); Let A takes x seconds in covering 1000m 1
\ 2 = 3t + 5t Þ t = hr = 15 min
and B takes y seconds covering the same distance 4
900 10x 200 \ Required = 5 : 15 o’clock
\ x + 20 = yÞ + =y ... (i)
1000 9 9 d d + 80
7. (a); =
20 25
950
x + 25 = y ...(ii) 25d = 20d + 1600 Þ d = 320
1000
From equation (i) and (ii) \ Total distance = 320 + 320 + 80 = 640 + 80
= 720 km
10x 200 950x 8. (b); Let length of good train = L1, speed = S1
Þ + = + 25 (From Eq. (i))
9 9 1000 length of passenger train = L2, speed = S2
200x - 171x 225 - 200 L1 + L2
= (S2 – S1) = ... (i)
180 9 60
25 180 500 L1
x= ´ = seconds (S2 – S1) = ... (ii)
9 29 29 40

301 @BEST300MCQ For More Study Material


Visit: studyiq.com
On solving (i) and (ii)
Speed of Kanpur mail 3
12. (b); =
L1 40 2 L1 2 Speed of Delhi mail 12
= = = = 2:1
L 1 + L 2 60 3 , L2 1 Speed of Delhi mail =
9. (d); 50 km 12 ´ 48
72 km = 2 ´ 48 = 96 km / h
B 3
A C D
13. (c); Let initially speed of Heena = 4x
(i) When accident takes place at C then the Let initially speed of Renu = 2x
distance covered after the accident = CB
\ Time will be same when Renu catches Heena.
(ii) When accidnent taken place at D then
distance covered after the accident = DB 50 d 50 + d
+ =
CB – DB = 72 km 4x x 2x
CD = 72 km 50 + 4d 50 + d
= Þ 50 + 4d = 100 + 2d
Let normal speed = 4x 4x 2x
Speed after accident = 3x 2d = 50 Þ d = 25
72 72 35 15 \ Required distance (N) = 100 – (50 + 25) = 25 m
- = - , x = 18
3x 4x 60 60 d d
14. (a); From the first case, - =4 ... (i)
So, normal speed = 72 km/h s s+6
10. (d); Let speed of two trains are S1 and S2 d d
From the second case, - = 10 ... (ii)
100 + 80 180 s-6 s+6
\ S1 + S2 = = =20 ... (i) If we solve equation(i) and (ii) then it is very long
9 9
and
100 + 80 180
S1 – S 2 = = =10 ... (ii) d d d
18 18 tedious method. So here , , should be
s s-6 s+6
On solving (i) and (ii)
represent integers to satisfy both equation
\ 2S1 = 30
\ D should be the value which have three divisors
18 which are 6 units apart from each other.
S1= 15 m/sec = 15 ´ = 54km/hr
5 So only 720 satisfy the condition which have 3, 9,
15 divisors which are 6 units apart.
18
\ S2 = 5m/sec = 5 ´ = 18 km/hr 15. (b);
5
d
11. (c); When the motorbike reached to the middle point
Distance = 150 m, relative speed t
= (70 – 45) = 25 km/hr.
150
\ Time = ´ 18 sec.
25 ´ 5
\ Distance covered by motorbike
150 5 From the figure it is clear that seeta is faster as she
= ´ 18 ´ 70 ´ = 420 m takes only (t + 2) hours while Geeta has taken
25 ´ 5 18
(t + 9) hours to complete the journey.
When it reaches exactly middle point of the train
after that distance to be covered = 150 m t-6 8
Then, we get = = Þ t2 – 6t = 72
relative speed = (65 – 60) = 5 km/hr 9 t
t2 – 6t – 72 = 0 Þ t = 12, – 6; \ t = 12
150
Time = ´ 18
5´ 5
6 hours 8 hours
\ Distance covered by bike
150 ´ 8 65 ´ 5
= ´ = 1950 m 9 hours 12 hours
5´5 18
\ total distance = 1950 + 420 = 2370 m = 2.37 km A M B

302 @BEST300MCQ For More Study Material


Visit: studyiq.com
\ ratio of the speed = 3 : 2 d d
\ 12 × 2s – 6 × 3s = 12; s = 2 km/hr \ ´ x + (x + 2) = d
4 4
\ Speed of faster person = 6 km/hr
16. (b); Since the two cyclist meet after = 200 minutes x x+2
+ =1 Þ x+x+2=4
they cover 0.5% of the distance per minutes and 4 4
30% per hour. 2x = 2 Þ x = 1hr
This condition is satisfied only if the slower
cyclist takes 10 hour ( there by covering 10% per 22. (b); Given,
hour) and faster cycle takes 5 hours ( there by total time = 4 hrs.
covering 20% per hour. Let time travelled by aeroplane = x
17. (a); Initial distance = 25 dogs leaps \ time travelled by train = (4 – x)
Per minute dog makes 5 dog leaps
Per minute cat makes 6 cat leaps = 3 dog leaps 4 10 5
\2= (4 – x) Þ 4 – x = =
Relative speed = 2 dogs leaps per minutes 5 4 2
Thus, an initial distance of 25 dogs leaps would
8-5 3
be covered in 12.5 minutes. x= =
2 2
18. (a); Distance travelled by train in
1 3
12 min = Distance travelled by sound in 30 sec \ time travelled by aeroplane = hrs.
2 2
= 330 × 30 m/sec In 2 hrs, distance travelled by aeroplane
330 ´ 30 = 360 km
\ Speed of the train = ´ 2 m/sec
25 ´ 60 3
When hrs. distance travelled by aeroplane
330 ´ 30 18 1188 13 2
= ´ 2 ´ km/hr = = 47 km
25 ´ 60 5 25 25 360 3
= ´ = 270 km
19. (d); Let distance = x km 2 2
2 ´ 3 ´ 4 24 and distance travelled by train
In the second case average speed = =
3+ 4 7 = (360 – 270) = 90 km
2x ´ 7 2x ´ 2 5 23. (a); Let it takes x hrs. in the second case
\ - =
24 7 60
1500 1500
\ Speed = = + 250
98x - 96x 5 2x 5 x 1
= Þ = Þ x = 7 km x+
24 ´ 7 60 24 ´ 7 60 2
20. (b); Distance travelled by man in 4 min.
æ 1ö
4 1 1500 ç x + ÷ - 1500x
= ´ 3 = km = 200 m è 2ø
= 250
60 5 æ 1ö
xç x + ÷
Distance travelled by carriage in è 2ø
4 min. = (200 + 100) = 300 min.
æ 1ö x
300 5 750 = 250x ç x + ÷ Þ x2 + - 3 = 0
\ Speed of carriage = m/sec = m/sec è 2ø 2
4 ´ 60 4
4 3 3
5 18 9 1 2x2 + x – 6 = 0 Þ x = - , ; \ x =
= ´ km/hr = = 4 km per hour 2 2 2
4 5 2 2
21. (c); Let distance = d 3
\ aeroplane takes hrs. in second case.
Let A meet B after x hours from point P 2

d 3 1
Speed of the first man = There in normal case it will take = + = 2 hrs.
4 2 2

d 1500
Speed of the second man = \ speed = = 750 km/hr.
4 2

303 @BEST300MCQ For More Study Material


Visit: studyiq.com
24. (a); Let it takes x hrs in the second case. 25. (d); In 1min. the hare will cover distance
120 120 120 120 5
\ speed = = -4 Þ - =4 = 1 × 60 × 12 × = 200 m
x x-1 x-1 x 18
120 ( x - x + 1) \ total distance = (200 + 100) = 300 m
= 4 Þ 120 = 4 x2 – 4 x
x ( x - 1) relative speed = (16 – 12) = 4 km/hr

x2 – x – 30 = 0 Þ x = – 5, 6; \ x = 6 5 300
= 4´ m/sec. Þ time to chase = ´ 18
\ the train take 6 hrs. in second case 18 4´ 5
\ the train will take time in normal case = 15 × 18 = 270 sec.
= (6 – 1) = 5 hrs. In 270 sec. hare cover the distance
120 12 ´ 5
\ original speed = = 24 km/hr = 270 ´ = 900 metres
5 18

1. (c); Relative speed of Train and man 5. (d); Relative speed of train = 28 km/hr
= 20 m/s – 10 m/s = 10 m/s 5
Time taken by train to passes the man 28 ´ m /s
18
180
= = 18 sec. 5
10 Length of the faster train = 28 ´ ´ 18 = 140 m
18
2. (b); Relative speed of train and man 6. (b); Let the speed of the second train be x m/s
= 63 – 3 = 60 km/hrs
5
Speed of first train = 80 km/hr = 80 ´ m /s
5 18
= 60 ´ m / sec.
18
1000
According to question = = 18
500 80 ´ 5
Time = ´ 18 = 30 sec. x+
60 ´ 5 18
1000 = 18x + 400
3. (c); Let the length of train is L.
To cross the the platform train travels total 600 600 18
x= m /s = ´ = 120 km / hr
distance = 2 L 18 18 5
7. (c); Relative speed of trains = 56 – 29 = 27 km/h
5
Speed of train = 90 ´ m / s = 25 m / s
18 5
= 27 ´ m / sec.
2L = speed × time 18
2L = 25 × 1 × 60 Þ 2L = 1500 Þ L = 750 m 5
length of faster train = 27 ´ ´ 10 = 75 m
4. (c); Let the length of Train = L m. 18
and the speed of the train is S m/s 8. (b); Let the length of each train is L
L + 800 L L
= 100 S1 = S2 =
S 3 4
L + 800 = 100S .... (i) Time taken by trains to pass each other
L + 400 = 60S ... (ii) Total distance
Divide eq. (i) by (ii). =
Relative speed
L + 800 10
= L L 7L
L + 400 6 Relative speed = + =
3 4 12
6L + 4800 = 10 L + 4000
2L 24 3
4L = 800, L = 200 m T= ´ 12 = s = 3 s
7L 7 7

304 @BEST300MCQ For More Study Material


Visit: studyiq.com
9. (c); Let the speed of train is S km/hr
11. (b); 7 am 60 km/hr
and the length of the train is L km. 8 am
50 km/hr 8am
According to the question

L A X C B
= 10 (10 am)
S–3
Train from A starts at 7 am with 50 km/hr, in
L = 10S – 30 ... (i)
1 hr it covers 50 km. So at 8 am it is 50 km away
L from A at X.
= 11 These trains meet at C at 10 a.m.
S–5
Distance travelled from B to C in 2 hour = 2 × 60
L = 11S – 55 ... (ii) = 120 km.
Subtract equation 1 from 2. Distance between X and C = 2 × 50 = 100 km
Distance between AC = 50 + 100 = 150 km
L = 10S - 30
AC 150
L = 11S - 55 =
BC 120
- - +
AC : BC = 5 : 4
0 = -S + 25
12. (c); Relative speed of trains = 5 km/hr
S = 25
Distance between two trains after 45 min
S = 25 km/hr 45 15
= 5´ = = 3 km 750 m.
10. (a); After the Accident, trains speed = 75% of orignal 60 4
speed. 13. (c);
A B
S2 3 50 km/hr 60 km/hr
=
S1 4
In 1 hr. second train travels 10 km more than
first.
4
Now it takes time of original time. \ So in 12 hr it travelled 120 km more than the
3 first Relative speed of both trains = 110 km/hr.
ì ü Total Distance between AB
T2 4 ï 1ï
= íS µ ý = Relative speed × time = 110 × 12 = 1320 km
T1 3 ï
ï
î Tï
ï
þ
8000
It train does not stop for an hour then it will be 14. (c); Time period for which he runs = = 32 sec
250
late by 3 hour.
T2 = 4 × 3 = 12 hr 24
Time taken by man to go up = = 2.4 sec
T1 = 3 × 3 = 9 hr 10
Now train takes 12 hr to complete the journey 24
Time take to go down = = 4 sec.
and before it takes 9 hr to complete, the remaining 6
journey. Time take to complete a round = 6.4 sec.
If accident occur 150 km ahead, time taken by 32
train to complete the remaining journey. Total round = = 5 round
6.4
T2 4 2.5 10 15. (c); Distance between two trains at 16:30 = 120 km
= ´ =
T1 3 2.5 7.5 (because aligarh express Runs two hour at a
speed of 60 km/hr.)
Now it covers the 150 km with original speed in
Relative speed = 80 – 60 = 20 km/h
(9 – 7.5) = 1.5
120
150 Time = = 6 hr .
= = 100 km / hr. 20
1.5 \ So, both trains meet 6 hr after 14:30.
Total time of journey = 12 (9 + 3 before Accident) Distance between Delhi and meeting point
Total Distance = 100 × 12 = 1200 km. = 6 × 80 = 480 km.

305 @BEST300MCQ For More Study Material


Visit: studyiq.com
650 km According to question
16. (a);
X Y D D
= + 16 ... (i)
(S – 20) S
Let the speeds of trains are X and Y km/hr.
10 x + 10 y = 650 D D
(x + y) = 65 ... (i) = -5 ... (ii)
(S + 10) S
Distance covered in 8 hours by the trains
= 65 × 8 = 520 km. By solving there two questions
\ Remaining distance is covered by first train We can get the value of D and S and by using
in 4 hours and 20 min. there values we can get time.
By shortcut method:
650 – 520
So speed of first train = = 30 km/hr.
æ ö
çç 4 + 1 ÷÷ Distance =
(S i S f ) (t 2 – t 1 )
çè 3 ø÷ (Different in speed)
From equation (i) speed of IInd train = 65 – 30
Si ® Initial speed
= 35 km/hr.
Sf ® Final speed.
17. (a); Let the speed by train is S km/hr.
Relative speed of man and train = (S – 4) km/hr. (t2 – t1) Þ Difference in time.
Total Distance while he see the train S i (S i – 20) S (S + 10)
= 50 + 50 + 20 = 120 m ´ 16 = i i ´5
20 10
120 2 \ Distance in both cases should be equal.
= 30 , S = 18 km / hr
5 5 8 Si – 160 = 5 Si + 50
(S – 4) ´
18 Si = 70 km/hr.
intital speed = 70
125
18. (b); Speed of second train = 40 ´ = 50 km / hr.
100 70 ´ 50
Distance = ´ 16 = 2800 km.
Let the speed of third train = S km/hr. 20
According to question: Dis tan ce 2800
Actual time = = = 40 hours.
20 25 speed 70
+ 1.5 =
S – 40 S – 50 22. (d); Relative speed of train and man = 55 – 10 = 45
from here we can direct put the value of S from km/hr
the options. 550
\ S = 60 km/hr. Time = ´ 18 sec
45 ´ 5
19. (b); Required Ratio of the speeds of train
= 22 × 2 = 44 sec.
S1 81
= =9:7 23. (c); Relative speed of trains = 36 km/hr.
S2 49
5
A = 36 ´ = 10 m/s.
45 km/hr B 18
20. (c); length of faster train = 10 × 2 = 20 m.
X Y
SA 3
SA 200 100 10 5 24. (c); =
= = = = SB 4
SB 288 144 12 6
5x = 45 1 T 4
Sµ Þ A =
6x = 54 T TB 3
Speed of train B is 54 km/hr. Let this is 4x and 3x
21. (a); Let the speed of train is S km/hr 4x – 3x = 30 min
and the distance = D km x = 30 min
D TA = 4x = 4 × 30 = 120 min = 2 hr.
Initially time taken = hr.
S

306 @BEST300MCQ For More Study Material


Visit: studyiq.com
2 29. (c); From the question it is clear that train travels the
25. (a); Speed Reduces to the intial speed. distance in 11 minutes is equal to the distance
3
covered by sound in 45 second.
2 Distance travel by train = 330 × 45 m
Snew = S initial
3 Time = 11 minutes
Sn 2 Tn 3
= Þ = 330 ´ 45 18
Si 3 Ti 2 Speed = ´ km / hr = 81 km/hr.
11 ´ 60 5
Initial time = 2x.
New time = 3x 30. (d); Total distance travelled by Aeroplane
3x – 2x = 1 Þ x = 1 = 240 × 5 = 1200 km
\ Time to cover the distance with normal speed 1200
= 2x = 2 × 1 = 2 hr. New speed = ´ 3 km / hr = 720 km/hr.
5
SA 4 T 5 31. (a); Total Distance travelled by car = 40 × 9 = 360 km.
26. (c); = Þ A =
SB 5 TB 4 Time taken when this distance travelled by 60
Time difference = 15 min km/hr

TA 5 ´ 15 75 360
= = min . = = 6 hours.
TB 4 ´ 15 60 60
32. (c); Ratio of speed when he goes by train and walking
40 ´ 75 speed
Distance = SA × TA = = 50 km.
60 ST 25
27. (b); Let the speed of car = x km/hr
=
Sw 4
and distance of journey = y km.
Tt 4
9 Ratio of time =
y = x´ ... (i) Tw 25
2
y = (x + 5) × 4 ... (ii) 4x + 25x = 348 min. [\ Total time is given]
On dividing (i) by (ii) 29x = 348
x = 12 min.
9x
9x Time in which he travels by train = 4 × 12
2 =1 Þ 4x + 20 =
4x + 20 = 48 min
2
x = 40 km/hr 48
Distance = 25 ´ = 20 km.
By short cut Method: 60
Ratio of time. T1 T2 S1 3
33. (a); =
9 S2 2
4
2
T1 2
9 : 8 =
Ratio of speed 8 : 9 T2 3
9x – 8x = 5 2x + 3x = 5, x = 1
x=5 he travels 2 hours with speed 3 km/hr.
Initial speed = 40 km/hr Total distance = 3 × 2 = 6 km.
28. (c); Ratio of speeds of A : B 34. (b); Let the total distance = 2d.
6 5
Ratio of time = 5 : 6 d d 5d
+ = 10 Þ = 10
6x – 5x = 60 + 15 40 60 120
x = 75 d = 240
Time taken by B = 6x 2d = 480 km.
= 6 × 75 = 450 min = 7 hour 30 min.

307 @BEST300MCQ For More Study Material


Visit: studyiq.com
5 41. (a); Let the speed of Abhay = x km/hr
35. (d); Speed of each car in m/s = 36 × = 10 m / s , and the speed of sammer = y km/hr
18
5 40 30 30
Þ 48 ´ = m/ s – =2 ... (i)
18 3 x y

Distance covered by 30 30
these in 15 sec
– =1 ... (ii)
y 2x
= 10 × 15 = 150 m 150 D
On adding (i) and (ii) we get.
40
Þ ´ 15 = 200 m
3 30 30
– =3
Total distance between then 200 x 2x
x = 5 km/hr
D= (150)2 + (200)2 = 250 m
36. (d); Let the speed of C is S m/s 42. (a); 2.5 hour (Bus)
Speed of B = 3S 2 hour (Bus)
Speed of A = 6S
Ratio of their time B
A C D
1 1 1 Auto 1
TA : TB : TC = : : = 1 : 2 : 6 4 hour — hour
6 3 1 10 am 2
6x = 72 Þ x = 12
Let C is a point where auto and the bus meet first
A covered the distance in 12 minutes
time and D is a point where the auto and the Bus
364 meets 2nd time.
37. (c); Speed of second train = = 91 km / hr
4 According to the above diagram;
According to the question their speeds are 6S : 7S Distance CD covered by BUS in 2 hour and auto
7S = 91 Þ S = 13 covered the same distance in 4 hours.
Speed of first train = 6 × 13 = 78 km/hr Distance CB covered by Bus in 2.5 hr and by Auto
38. (c); To jind the ratio of their speeds we need to find in double of Bus.
the individuals speed in same unit.
= 2.5 × 2 = 5 hours
So we are going to find the speed in m/s.
Time at which auto will reach at harid was
550 = 10 + 5 = 3 P.M.
Speed of truck =
60
2x × 0.75
33 ´ 1000 43. (b);
Speed of Bus =
45 ´ 60
x km/h

m
S truck 55 ´ 45 ´ 60 5 ´ 45 3 2x
00k
= = = 1
S Bus 6 ´ 33 ´ l000 3´ 100 4
Ratio = 3 : 4
39. (b); Distance Ratio = 1 : 2 : 3
(2x)2 + (1.5 x)2 = (100)2
Time of traveling = 3 : 2 : 1
4x2 + 2.25 x2 = 10000
Distance 1 2 3 x = 40 km/hr.
Speed Ratio = = : : =1:3:9
Time 3 2 1
44. (a); 114 m
20 Constable Thief
40. (c); Fixed time = – 40 min = 3h 20 min.
5 21 m 15 m
Time taken when he travels 8 km/hr. \ Relative speed between Thief and constable
20 = 21 – 15 = 6 m/min
= = 2 hours 30 min.
8
114
So he reached 50 minutes earlier. Time = = 19 minutes
6

308 @BEST300MCQ For More Study Material


Visit: studyiq.com
45. (c); Let their speeds are 5 and 4 m/s
Total distance
Relative speed = 1 m/s 48. (b); Avg speed =
Total time taken
100
Time to caught the thief = = 100 sec
1 10 + 12 22
= =
Distance travelled by thief = 4 × 100 = 400 m 10 12 50 + 72
+
46. (d); Let initial speed = S km/hr 12 10 60
S i Sf 22 ´ 60
D= × (Difference in time) = = 10.8 km/hr.
Difference in speed 122

S(S + 3) 40 12 8
Distance = ´ 49. (b); Total distance = 30 ´ + 45 ´ = 12 km.
3 60 60 60
Total time = (12 + 8) min = 20 min
S(S – 2) 40
2 nd condition D = ´ 12
2 60 Aug speed = ´ 60 = 36 km / hr.
20
S(S + 3) 40 S(S – 2) 40 50. (a); Let the total distance is 100 km.
´ = ´
3 60 2 60 So, 30 km is covered with a speed of 20 km. 1hr.
(Distance is same in both the cases) \ 60 km at a speed of 40 km 1 hr. and the
S = 12 km/h remaining 10 km distance at 10 km 1hr.

12 ´ 15 40 30 60 10
Total time = + +
Distance = ´ = 40 km. 20 40 10
3 60
47. (a); From the question it is clear that Bus stops for a 60 + 60 + 40 160
time in which it travels a distance of 10 km with = = = 4 hr.
40 40
a speed of 50 km/hr.
100
10 10 Avg speed = = 25 km/hr.
time = hr = ´ 60 min = 12 min. 4
50 50

309 @BEST300MCQ For More Study Material


Visit: studyiq.com
Chapter
Boat and Stream
13
The chapter of boat and stream is based on the application of time and distance. Two terms which are frequently used
in this chapter are downstream and upstream.
1. Downstream ® When boat is moving along the direction of the stream.
2. Upstream ® When boat is moving against the direction of the stream.
Basic formula: If 'x' be the speed of boat in still water and 'y' be the speed of stream.
(i) Downstream speed (v) = (x + y) km/h
(ii) Upstream speed (u) = (x – y) km/h

Where downstream Speed is 'v' and upstream speed is 'u'.


1
(iii) Speed of boat in still water, (x) = (u + v)
2
1
(iv) Speed of stream, (y) = (v – u)
2
Note: If the speed of water in river is zero, then the water is considered to be still.
Example: A man row with a speed of 8 km/h in still water. Find the downstream and upstream speed of boat, if the
speed of stream is 4 km/h. ?
Sol. Downstream speed = (x + y)km/h = (8 + 4) = 12 km/ h
Upstream speed = (x – y) km/h = (8 – 4) = 4 km/h
Concept 1: If the speed of boat in still water is x km/h and speed of stream is y km/h. If time taken to go and come back
T (x2 - y 2 )
from a point is T, the distance between both points = km
2x
Example: A man can row 14 km/h in still water. When the stream is running at 2 km/h, it takes him 7 hour to row
to a place and to come back. How far is the place ?

Sol. Required distance =


(
7 14 2 - 2 2 ) km = 7 ´ 192 = 48 km
2 ´ 14 2 ´ 14
Concept 2: A person can row a certain distance downstream is t1 h and returns upstream the same distance in t2 h.
When the stream flows at the rate of ‘s’ km/h.
s (t1 + t2 )
Then the speed of man = km/h
t2 - t1
Example: Sonu can row a certain distance is 8 h and can return the same distance is 12h. If the stream of flows at the
rate of 8 km/h, then find the speed of Sonu is still water ?
8 ( 8 + 12 ) 8 ´ 20
Sol: Speed of man = km/h = = 40 km/h
12 - 8 4
Concept 3: If the speed of boat in still water is x km/h and river is flowing with a speed of y km/h, then average speed

in going to a certain place and coming back to starting point is given by


x  yx  y km/h.
x
Example: A man can row in still water with a speed of 6 km/h to go to a certain place and to come back. Find the
average speed for the whole journey, if the river is flowing with a speed of 3 km/h ?
( 6 + 3 )( 6 - 3 )
9´ 3
Sol. Average speed = km/h =
km/h = 4.5 km/h
6 6
Concept 4: If a man can row d1 km upstream and e1 km downstream in T1 hours. Also, he can row d2 km upstream and
e2 km downstream in T2 hours.

310 @BEST300MCQ For More Study Material


Visit: studyiq.com
æ d e - d2 e1 ö æ d 1 e 2 - d 2 e1 ö
Then, the upstream speed of man = ç 1 2 km/h Þ Downstream speed of man = ç ÷ km/h
è e2 T1 - e 1 T2 ÷ø è d 1 T2 - d 2 T1 ø
Example: A man can row 30 km upstream and 44 km downstream is 10 hrs. Also, he can row 40 km upstream and
55 km downstream is 13 hrs. Find the rate of current and speed of man is still water ?
30 ´ 55 - 40 ´ 44 -110
Sol. Upstream speed of man = = = 5 km/h
55 ´ 10 - 44 ´ 13 -22
30 ´ 55 - 40 ´ 44 -110
Downstream speed of man = = = 11 km/h
30 ´ 13 - 40 ´ 10 -10
5 + 11 11 - 5
Speed of man = = 8 km/h Þ Speed of stream = = 3 km/h
2 2

1. A man can row upstream at 10 km/hr and 4. Ramesh can row a certain distance downstream in 6
downstream at 16 km/hr. Find the speed of man in hours and return the same distance in 9 hours. If the
still water ? stream flows at the rate of 3 km per hour find the
Sol. Let the speed of the man in still water be x km/hr and speed of Ramesh in still water.
speed of the stream be y km/hr. Sol. Let the speed of Ramesh in still water be x km/hr.
According to the question, Then his upstream speed = (x – 3) km/hr
Downstream speed = (x + 3) km/hr.
x + y = 16 ....(i)
Now, we are given that up and down journey are
and x – y = 10 .... (ii) equal, therefore,
Adding eq. (i) with eq. (ii), we have (x + 3)6 = (x – 3)9 or, 6x + 18 = 9x – 27
2x = 26 \ x = 13 km/hr or, 3x = 45; \ x = 15 km/hr
\ Speed of the man in still water = 13 km/hr. 5. The speed of a boat in still water is 11 km/h and the
2. A man can row upstream at 10 km/hr and speed of the stream is 3 km/h. The boat takes a total of
downstream at 16 km/hr. Find the rate of the current? 11 hours to cover a distance up and down of a fixed
destination. What is the fixed distance ?
Sol. Let the speed of the man in still water be x km/hr and
Sol. Let the fixed distance be l km.
the rate of the current be y km/hr
Speed of the boat downstream = 11 + 3 = 14 km/h
According to the question,
Downstream speed = x + y = 16 km/hr ... (i) l
Time taken to cover l km downstream = hours.
Upstream = x – y = 10 km/hr ... (ii) 14
Now the speed of the boat upstream = 11 – 3 = 8 km/h
Subtracting eq. (ii) from eq. (i), we have
2y = 6 km/hr or y = 3 km/hr l
Time taken to cover l km upstream = hours
\ Speed of the current = 3 km/hr 8
3. A stream is running at 2 km/hr. A motorboat goes 10 From the question :
km upstream and comes back to the starting point in l 7 l + 4l
l 56
55 minutes. Find the speed of the motorboat in still + = 11 or, = 11 or, l = 11 × = 56 km
water. 14 8 56 11
6. A steamer takes one hour more to go 48 km upstream
Sol. Let the speed of the motorboat in still water be x
than the time to go 60 km downstream. If the steamer
km/hr.
takes equal time to cover 30 km downstream and 18
10 10 55 km upstream then find the ratio of the speed of the
+ = boat in still water and the speed of the stream.
x + 2 x - 2 60
Sol. Let the speed of the boat in still water = x km/h.
or, 240x = 11x2 – 44
Speed of the stream is y km/h
or, 11x2 – 240x – 44 = 0 So, The speed of steamer downstream = x + y km/h
\ (x – 22) (11x +2) = 0 and the speed of the steamer upstream = x –y km/h
So, x = 22 km/hr (neglect the – ve value) 48 60
\ Speed of the motorboat in still water = 22 km/hr. In first situation, - =1 ... (i)
x-y x+y

311 @BEST300MCQ For More Study Material


Visit: studyiq.com
30 18 48 60
and in second situation = Again from eq. (i), - =1
x+y x-y 12 x + y

60 36 60
or, = ... (ii) or, 4 – 1 = or, x + y = 20 ... (iv)
x+y x-y x+y

48 36 Solving eq. (iii) and (iv), x = 16 and y = 4


from eq. (i) and (ii) - =1
x-y x-y x 16
So required ratio = = = 4 :1
y 4
12
or, = 1 or, x – y = 12 ... (iii)
x-y

Ques ti ons 9. When the speed of a boat in still water is 4 km/h and
the rate of stream is 2 km/h, find upstream speed of
the boat ?
1. A boat goes 14 km upstream in 42 minutes. The speed
(a) 6 km/h (b) 5 km/h
of stream is 4 km/h. The speed of boat in still water is
(c) 2 km/h (d) 7 km/h
(a) 24 km/h (b) 25 km/h
10. If the speed of a swimmer in still water is 9 km/h.
(c) 30 km/h (d) 23 km/h
Find the downstream speed of the swimmer, when
2. The speed of a boat in still water is 7 km/h. If its speed the river is flowing with the speed of 6 km/h ?
in downstream is 10 km/h. Then the speed of stream is
(a) 15 km/h (b) 18 km/h
(a) 7 km/h (b) 3 km/h
(c) 3 km/h (d) 12 km/h
(c) 10 km/h (d) 6 km/h
11. A swimmer’s speed in downstream is 11 km/h and
3. Aditya can row with stream at 10 km/h and against
speed of the stream is 1.5 km/h. Find the upstream
the stream at 6 km/h. His speed in still water is
speed of swimmer ?
(a) 16 km/h (b) 6 km/h
(a) 8 km/h (b) 9.5 km/h
(c) 8 km/h (d) 10 km/h
(c) 9 km/h (d) 6.25 km/h
4. Speed of man is 12 km/h in still water. If the rate of
12. A boatman rows 1 km in 5 min along the stream and 6
current is 4 km/h, then the effective speed of the man
km in 1 h against the stream. The speed of the stream is
upstream is
(a) 3 km/h (b) 6 km/h
(a) 6 km/h (b) 7 km/h
(c) 10 km/h (d) 12 km/h
(c) 5 km/h (d) 8 km/h
13. Keshav can row 60 km downstream and 36 km
5. A man moves downstream at a rate of 14 km/h and
upstream, taking 10 h each time. What is the velocity
upstream at a rate of 6 km/h. Speed of boat in still
of the current?
water is
(a) 4 km/h (b) 10 km/h (a) 3 km/h (b) 2.2 km/h
(c) 16 km/h (d) 2 km/h (c) 4 km/h (d) 1.2 km/h
6. A man moves 20 km down stream in 5 hours and 10 14. What time will be taken by a boat to cover a distance
km up stream in same time. The speed of stream of 128 km along the stream, if speed of a boat in still
water is 24 km/h and speed of stream is 8 km/h?
(a) 4 km/h (b) 1 km/h
(a) 8 h (b) 4 h
(c) 1.6 km/h (d) done
(c) 7 h (d) 6 h
7. A boat travels 18 km upstream in 6 hours. How long
(In hrs) will it take to cover the same distance 15. A man can row against the current three-fourth of a
downstream. The speed of current is one fourth the kilometre in 15 min and returns same distance in 10
speed of boat in still water. min, then ratio of his speed to that of current is –
(a) 3 (b) 4 (a) 3 : 5 (b) 5 : 3
(c) 3.6 (d) 3.4 (c) 1 : 5 (d) 5 : 1
8. A person can row with the stream at 8 km/h and 16. A boat goes 48 km downstream in 20 h. It takes 4 h
against the stream at 4 km/h. The speed of the current more to cover the same distance against the stream.
is (In km/hr) What is the speed of the boat in still water ?
(a) 1 (b) 2 (a) 2.2 km/h (b) 2 km/h
(c) 1.5 (d) 4 (c) 4 km/h (d) 4.2 km/h

312 @BEST300MCQ For More Study Material


Visit: studyiq.com
17. A man can row 15 km downstream in 3 hours and 5 19. A man can row 20 km/h in still water and speed of
1 the stream is 5 m/sec. Ratio of speed of man to current
km upstream in 2 hours. His speed in still water in speed is
2
(km/hr) is (a) 9 : 10 (b) 4 : 1
(a) 4 km/hr (b) 4.5 km/hr (c) 1 : 4 (d) 10 : 9
(c) 3.5 km/hr (d) 5 km/hr 20. A man can swim in still water at 9.6 km/hr, but takes
18. A man can row 44 km downstream in 4 hours. If the twice as long to swim upstream then downstream.
man’s rowing rate in still water is 8 km/hr, then find The speed of stream is
in what time will he cover 25 km upstream? (a) 3.2 km/hr (b) 2.3 km/hr
(a) 5 hours (b) 6 hours (c) 5.6 km/hr (d) 6.5 km/hr
(c) 4.5 hours (d) 4 hours

1. Speed of motorboat in still water is 45 km/h. If the (a) 12 km/h (b) 10 km/h
motorboat travels 80 km along the stream in 1 h 20 (c) 9 km/h (d) 8 km/h
min, then the time taken by it to cover the same distance 7. The ratio of speeds of a motorboat to that of the current
against the stream will be of water is 36 : 5. The motorboat goes along with the
(a) 4 h 20 min (b) 3 h 40 min current in 5 h 10 min. Find the time to come back of
(c) 2 h 40 min (d) 2 h 55 min motorboat.
2. Speed of boat in still water is 5 km/h. The river is (a) 5 h 50 min (b) 6 h
flowing with a speed of 2 km/h and time taken to (c) 6 h 50 min (d) 12 h 10 min
cover a certain distance upstream is 2 h more than
8. In a river, the ratio of the speed of stream and speed of
time taken to cover the same distance downstream.
a boat in still water is 2 : 5. Again, ratio of the speed of
Find the distance?
stream and speed of an another boat in still water is
(a) 10.5 km (b) 11 km 3 : 4. What is the ratio of the speeds of the first boat to
(c) 10.9 km (d) 15 km the second boat in still water?
3. A man can row at 10 km/h in still water. If he takes (a) 10 : 7 (b) 15 : 8
total 5 h to go to a place 24 km away and return, then (c) 4 : 3 (d) 5 : 4
the speed of the water current is –
9. A man can row at the rate of 3.5 km/hr in still water.
(a) 2 km/h (b) 3 km/h If the time taken to row a certain distance upstream is
1 1
(c) km/h (d) 1 km/h 2 times as much as to row the same distance
2 2
4. A steamer goes downstream from one port to another downstream, find the speed of the current.
in 4 h. It covers the same distance upstream in 5 h. If (a) 2.5 km/hr (b) 1.5 km/hr
the speed of the stream is 2 km/h, then find the
(c) 3 km/hr (d) 1.25 km/hr
distance between the two ports.
10. A man can row at the rate of 4 km/hr in still water. If
(a) 50 km (b) 60 km
the time taken to row a certain distance upstream is 3
(c) 70 km (d) 80 km times as much as to row the same distance
5. A boatman takes twice as long to row a distance downstream, find the speed of the current ?
against the stream as to row the same distance with (a) 3 km/hr (b) 1.5 km/hr
the stream. Find the ratio of speeds of the boat in still
(c) 1 km/hr (d) 2 km/hr
water and the stream ?
11. A man swimming in a stream which flows 1.5 km/hr
(a) 2 : 1 (b) 3 : 1
finds that in a given time he can swim twice as far
(c) 1 : 2 (d) 1 : 3 with the stream as he can against it, at what rate does
6. A river is flowing with a steady speed of 4 km/h. One he swim ?
rows his boat downstream in the river and then returns (a) 4 km/hr (b) 4.5 km/hr
by rowing upstream in the same river. When he returns
(c) 5 km/hr (d) 3.5 km/hr
to the starting point, the total distance covered by him
is 42 km. If the return journey takes 2 h more than his 12. A man can row upstream 36 km in 6 hours. If the
onward journey, then the speed of his rowing in still speed of a man in still water is 8 km/hr, find how
water must be: much he can go downstream in 10 hours ?

313 @BEST300MCQ For More Study Material


Visit: studyiq.com
(a) 150 km (b) 80 km 17. A man can row 7 km/hr in still water. If the river is
(c) 90 km (d) 100 km running at 3 km/hr, it takes 6 hours more in upstream
13. The speed of a boat in still water is 4 km/hr and the than to go downstream for the same distance. How
speed of current is 2 km/hr. If the time taken to reach far is the place?
a certain distance upstream is 9 hours, find the time it (a) 48 km (b) 36 km
will take to go to same distance downstream ? (c) 42 km (d) 40 km
(a) 2 hrs (b) 2.5 hrs 18. A man can row 9 km/hr in still water. If the river is
(c) 3.5 hrs (d) 3 hrs running at 3 km/hr, it takes 3 hours more in upstream
14. A person can swim in still water at 4 km/hr. If the than to go downstream for the same distance. How
speed of water is 2 km/hr, how many hours will the far is the place?
man take to swim back against the current for 6 km ? (a) 30 km (b) 36 km
(a) 3 (b) 4 (c) 24 km (d) 42 km
1 19. A boat takes 3 hours to travel from place M to N
(c) 4 (d) 5
2 downstream and back from N to M upstream. If the
15. A man can row at a speed of 4.5 km/hr in still water speed of boat in still water is 4 km/hr, what is the
to a certain upstream point and back to the starting distance between the two places?
point in a river which flows at 1.5 km/hr. Find his (a) 8 km (b) 12 km
average speed for total journey? (c) 6 km (d) Data inadequate
(a) 4 km/hr (b) 6 km/hr 20. P, Q and R are the three towns on a river which flows
(c) 4.5 km/hr (d) 5 km/hr uniformly. Q is equidistant from P and R. I row from P
16. A man can row at a speed of 15 km/hr in still water to to Q and back in 10 hours and I can row downstream
a certain upstream point and back to the starting point from P to R in 4 hours. Compare the speed of my boat
in a river which flows at 3 km/hr. Find his average in still water with that of the river.
speed for total journey ? (a) 5 : 3 (b) 4 : 3
(a) 9 km/hr (b) 6 km/hr
(c) 6 : 5 (d) 7 : 3
(c) 3 km/hr (d) 14.4 km/hr

1. A motorboat travelling at some speed, can cover 25 16


km upstream and 39 km downstream in 8 h. At the 4. The speed of a boat downstream is times the
9
same speed, it can travel 35 km upstream and 52 km
speed of the boat upstream. The speed of the current
downstream in 11 h. The speed of the stream is:
is what percent of the speed of the boat in still water?
(a) 2 km/h (b) 3 km/h (a) 28% (b) 24%
(c) 4 km/h (d) 5 km/h (c) 56.25% (d) 40%
2. A man can row a boat at a speed of 8 kmph in still 5. A river is flowing at a speed of 5 km/h in a particular
water. He was rowing the boat downstream from one direction. A man, who can swim at a speed of 20 km/
point to another. After travelling half of the distance h in still water, starts swimming along the direction
the motor of the boat failed and stopped working. He of flow of the river from point A and reaches another
travelled on the boat along the stream and reached point B which is at a distance of 30 km from the starting
his destination taking six hours more than the usual point A. On reaching point B, the man turns back and
time. The speed of the stream is 2 kmph. What is the starts swimming against the direction of flow of the
distance between the two points? river and stops after reaching point A. The total time
(a) 20 (b) 30 taken by the man to complete his journey is
(c) 24 (d) 28 (a) 2 h 30 min (b) 3 h 12 min
3. A, B and C are situated at the bank of river which is (c) 3 h 30 min (d) 3 h 45 min
flowing at a constant rate. B is at an equal distance 6. A boat covered a certain distance travelling
with A and C. A swimmer Avinash takes 10 h to swim downstream in 45 minutes, while it came back to the
from A to B and B to A. Also, he takes 8 h to swim from starting point in one hour 15 minutes. The speed of
A to C in downstream. What is the ratio of speed of the stream is 2 kmph. What is the speed of the boat in
Avinash in still water and speed of stream? still water?
(a) 5 : 4 (b) 3 : 5 (a) 8 kmph (b) 9 kmph
(c) 5 : 1 (d) 1 : 2 (c) 10 kmph (d) 11 kmph

314 @BEST300MCQ For More Study Material


Visit: studyiq.com
7. In a stream running at 2 km/hr, a water scooter goes 14. A man can row 18 kmph in still water, it takes him
10 km upstream and back again to the starting point thrice as long to row up as to row down the river.
in 55 minutes. Find the speed of the water scooter in Find the rate of stream.
still water. (a) 8 (b) 9
(a) 22 km/hr (b) 26 km/hr (c) 10 (d) 11
(c) 28 km/hr (d) 30 km/hr
15. Rohit can row a certain distance downstream in 8
8. A man can row 40 km upstream and 55 km hours and return the same distance in 12 hours. If the
downstream in 13 hours. Also, he can row 30 km stream flows at the rate of 5 km per hour. Find the
upstream and 44 km downstream in 10 hours. Find
speed of Rohit in still water ?
the speed of the man in still water and the speed of
the current. (a) 20 km/hr (b) 30 km/hr
(a) 8 kmph & 3 kmph (b) 6 kmph & 2 kmph (c) 15 km/hr (d) 25 km/hr
(c) 10 kmph & 4 kmph (d) Can not be determined 16. A man takes 3 hours 45 minutes to row a boat 15 km
9. A person takes total 5 hours to go the destination downstream of a river and 2hours30minutes to cover
downstream and also comes back upstream by boat. a distance of 5km upstream. find the speed of the
If the speed of the boat in still water is 10 km/h and river current in km/hr.
the speed of the stream is 4 km/h, then what is the (a) 1 (b) 2
distance of destination from the starting point ? (c) 3 (d) 4
(a) 16 km (b) 18km
17. A boat goes 6 km upstream and back again to the
(c) 21 km (d) 25 km starting point in 2 hours. If the velocity of the stream
10. A boat covers 24 km upstream and 36 km downstream is 4 km/ hr, what is the speed of the boat in still water?
in 6 hours while it covers 36 km upstream and 24 km
(a) 6.5 km/hr (b) 7.5 km/hr
1 (c) 8 km/hr (d) 8.5 km/hr
downstream in 6 hours. The velocity of the current
2 18. Speed of a boat in standing water is 9 kmph and the
is: speed of the stream is 1.5 kmph. A man rows to a
(a) 2 km/hr (b) 3 km/hr place at a distance of 105 km and comes back to the
(c) 4 km/hr (d) 5 km/hr starting point. The total time taken by him is:
11. The speed of boat in still water is 10 km/hr. If it can (a) 16 (b) 18
travel 26 km downstream and 14 km upstream in the (c) 20 (d) 24
same time, the speed of the stream is?
19. A boat covers 12 km upstreram and 18 km
(a) 2 km/hr (b) 2.5 km/hr
downstream in 3 hours while it covers 36 km upstream
(c) 3 km/hr (d) 4km/hr
12. In a stream running at 2 kmph, a motorboat goes 6 1
and 24 km downstream in 6 hours, what is the
km upstream and back again to the starting point 2
in 33 minutes. Find the speed of the motorboat in velocity of the stream?
still water ? (a) 1.5 km/hr (b) 1 km/hr
(a) 22 (b) 23 (c) 2 km/hr (d) 2.5 km/hr
(c) 20 (d) 25
20. A man rows to a place 48 km distance and come back
1 in 14 hours. He finds that he can row 4 km with the
13. A man row 9 kmph in still water and finds that it stream in the same time as 3 km against the stream.
3
takes him thrice as much time to row up than as to The rate of the stream is:
row down the same the same distance in the river. (a) 1 (b) 1.5
The speed of the current is: (c) 2 (d) 2.5
1 1
(a) 3 km /hr (b) 3 km/hr
3 9
2 1
(c) 4 km/hr (d) 4 km/hr
3 2

315 @BEST300MCQ For More Study Material


Visit: studyiq.com
1. A boat goes 11 km along the stream in 1 hr and 5 km (a) 2 mph (b) 2.5 km/hr
against the stream in 1 hr. The speed of the boat in (c) 3 mph (d) 4 mph
still water (in km/hr) is: 10. At his usual rowing rate, Rahul can travel 12 miles
(a) 3 (b) 5 downstream in a certain river in 6 hours less than it
(c) 8 (d) 9 takes him to travel the same distance upstream. But if
2. A boat running downstream covers distance of 16 km he could double his usual rowing rate for his 24 mile
in 2 hours while for covering the same distance up round trip, the downstream 12 miles would then take
stream, it takes 4 hours. What is the speed of the boat only one hour less than the upstream 12 miles. What
in still water? is the speed of the current in miles per hour?
(a) 4 km/hr (b) 6 km/hr 1 2
(a) 1 (b) 1
(c) 8 km/hr (d) Data inadequate 3 3
3. A boatman goes 2 km against the current of the stream
in 1 hour and goes 1 km along the current in 10 1 2
(c) 2 (d) 2
minutes. How long will it take to go 5 km in stationary 3 3
water? 11. A motorboat went downstream for 28 km and
(a) 40 minutes (b) 1 hour immediately returned. It took the boat twice as long to
(c) 1 hr 15 min (d) 1 hr 30 min make the return trip. If the speed of the river flow were
4. A man takes twice as long to row a distance against twice as high, the trip downstream and back would
the stream as to row the same distance in favour of take 672 minutes. Find the speed of the boat in still
the stream. The ratio of the speed of the boat (in still water and the speed of the river flow:
water) and the stream is: (a) 9 km/h, 3 km/h (b) 9 km/h, 6 km/h
(a) 2 : 1 (b) 3 : 1 (c) 8 km/h, 2 km/h (d) 12 km/h, 3 km/h
(c) 3 : 2 (d) 4 : 3 12. Two friends A and B, on their last day in college,
decided to meet after 20 years on a river. A had to sail
5. A boat running upstream takes 8 hours 48 minutes to
cover a certain distance, while it takes 4 hours to cover 5
42 km to the meeting place and B had to sail 35 per
the same distance running downstream. What is the 7
ratio between the speed of the boat and speed of the cent less. To arrive at the meeting place at the same as
water current respectively? his friend B, A started at the same time as B and sailed
(a) 2 : 1 (b) 3 : 2 with the speed exceeding by 5 km/h the speed of B.
(c) 8 : 3 (d) Cannot be determined Find the speed of A:
6. If a boat goes 7 km upstream in 42 minutes and the (a) 10 kmph (b) 14 kmph
speed of the stream is 3 kmph, then the speed of the (c) 9 kmph (d) 21 kmph
boat in still water is: 13. On the banks of the river Yamuna there are two
(a) 4.2 km/hr (b) 9 km/hr bathing points in Varanasi and Patna. A diya left in
(c) 13 km/hr (d) 21 km/hr the river at Varanasi reaches Patna in 24 hours.
7. A man can row at 5 kmph in still water. If the velocity However, a motorboat covers the whole way to and
of current is 1 kmph and it takes him 1 hour to row to fro in exactly 10 hours. If the speed of the motorboat
a place and come back, how far is the place? in still water increased by 40% then it takes the motor
boat 7 hours to cover the same way (from Varanasi to
(a) 2.4 km (b) 2.5 km
Patna and back again). Find the time necessary for
(c) 3 km (d) 3.6 km
the motorboat to sail from Varanasi to Patna when its
8. A motorboat, whose speed is 15 km/hr in still water speed in still water is not increased:
goes 30 km downstream and comes back in a total of (a) 3 hours (b) 4 hours
4 hours 30 minutes. The speed of the stream (in km/
(c) 4.8 hours (d) None of these
hr) is:
14. A motorboat moves form point A to point B and back
(a) 4 (b) 5
again, both points being located on the riverbank. If
(c) 6 (d) 10 the speed of the boat in still water is doubled, then the
9. A boat takes 90 minutes less to travel 36 miles trip from A to B and back again would take 20% of the
downstream than to travel the same distance time that the motorboat usually spends in the journey.
upstream. If the speed of the boat in still water is 10 How many times is actual speed of the motor boat
mph, the speed of the stream is: higher than the speed of the river flow?

316 @BEST300MCQ For More Study Material


Visit: studyiq.com
17. A boat goes 60 km and back to starting point in 10
3 2 hours. The time taken by the boat to row 3 km
(a) (b)
2 3 downstream is equal to the time taken by the boat to
row 2 km upstream. Find the speed of boat in still
2 3
(c) (d) water and rate of current:
3 2
(a) 12.5 km/hr, 2.5 km/hr
15. The normal speed of a boat in still water is 4 times the
(b) 15 km/hr, 10 km/hr
speed of the river. The boat goes upstream and takes 2
hrs to reach B from A. By what percent should the (c) 15 km/hr, 12 km/hr
boat increase/decrease its speed if it is required to (d) None of these
reach A from B in exactly 1 hr 30 minutes?
(a) increase by 20% (b) increase by 25% 3
18. A man can row against the current of a kilometre
(c) decrease by 20% (d) decrease by 25% 4
in 15 min and returns the same distance in 10 min.
16. Two boats start at the same instant to cross a river W
The ratio of this speed to that of the current is:
metre wide. The faster boat reaches the other bank
and returns immediately. What are the distance (a) 3 : 5 (b) 5 : 3
travelled by them when they meet, where the speeds (c) 1 : 5 (d) 5 : 1
of these boats are b1 & b2? 19. The ratio of speed of a motor-boat to that of the current
2W 2W of water is 36 : 5. The boat goes along with the current
(a) , for 10 minutes. It will come back in:
(b1 + b 2 ) (b 1- b2 )
(a) 12 min (b) 14.5 min
2W 2W (c) 13.22 min (d) 11.6 min
(b) b1 and b2
(b1 + b 2 ) (b1 + b2 ) 20. A boat goes 20 km downstream in 1h and the same
distance upstream in 2h. The speed of the boat in still
2W water is:
(c) b2
(b1 + b 2 ) (a) 15 km/h (b) 10 km/h
(d) data insufficient (b) 5 km/h (d) 7.5 km/h

S olut ions 10
=5Þx–y=2 ... (ii)
x-y
1. (a); Let speed of boat in still water = x
From (i) and (ii), 2x = 6 Þ x = 3
14 42 y = 1 km/h
Then, = Þ x = 24 km/h
x-4 60 7. (c); Let speed of boat in still water = x
2. (b); Let the speed of the stream is y km/h. 1
Then, y + 7 = 10 Þ y = 3 km/h From question, Given y = x
4
3. (c); Let speed of boat = x km/h
Speed of stream = y km/h 18 3x
Then =6 Þ 3= Þx =4
Then, x + y = 10 Þ x – y = 6 x 4
x-
Þ 2x = 16 Þ x = 8 km/h 4
4. (d); Speed of the man upstream (v) = 12 – 4 = 8 km/h Then the time it will take to cover the same
5. (b); Let speed of boat in still water = x km/hr 18 18
Speed of stream = y km/hr distance downstream = = = 3.6 hr.
4+1 5
x + y = 14, x – y = 6, 8. (b); Let the speed of boat = x
2x = 20, x = 10 km/h Speed of stream = y
6. (b); Let speed of boat in still water = x x + y = 8 km/h Þ x – y = 4 km
Speed of stream = y 2x = 12 Þ x = 6 Þ y = 2 km/h
20 9. (c); Given x = 4 km/h y = 2 km/h
=5Þx +y=4 ... (i) Upstream rate = 4 – 2 = 2 km/h
x+y

317 @BEST300MCQ For More Study Material


Visit: studyiq.com
10. (a); Given x = 9, y = 6
x 15 / 4 15 5
Then downstream speed of the swimmer \ = = = Þ x:y=5:1
y 3/4 3 1
= 9 + 6 = 15 km/h
11. (a); Given, x + y = 11 Þ y = 1.5 then x = 9.5 16. (a); Let speed of boat = x, speed of stream = y
\ upstream speed of swimmer = 9.5 – 1.5 48 12
= 8 km/hr = 20 Þ x + y = ... (i)
x+y 5
12. (a); Let speed of boat = x, Speed of steam = y
1 48
5 = 24 Þ x – y = 2 ... (ii)
= Þ x +y = 12 ... (i) x-y
x + y 60
From (i) and (ii)
6
=1 Þ x–y=6 ... (ii) 12 11
x-y 2x = +2 Þx =
5 5
From (i) and (ii)
x = 2.2 km/h
2x = 18 Þ x = 9
Speed of the stream = (12 – 9) = 3 km/h 17. (c); Let speed of boat = x, speed of stream = y
13. (d); Let speed of boat = x, speed of current = y 15
=3 Þ x+y=5 ... (i)
60 x+y
= 10 Þ x + y = 6 ... (i)
x+y 5 5
= Þx –y =2 ... (ii)
36 x-y 2
= 10 Þ x – y = 3.6 ... (ii) From (i) and (ii)
x-y
From (i) and (ii) x = 3.5 km/h
2x = 9.6 Þ x = 4.8 Þ y = 1.2 km/h 18. (a); Let speed of the current= y km/h

128 44
128 Given = 4 Þ y = 3 km/h
14. (b); Time taken = = =4h 8+ y
24 + 8 32
15. (d); Let speed of boat = x, speed of stream = y 25
Time taken in upstream = =5h
3/4 15 (8 - 3)
= Þx –y =3 ... (i)
x-y 60 19. (d); Speed in still water = 20 km/h

3/4 10 3 1 18
= Þ = Speed of the stream = 5 m/sec = 5 ´ km /hr
x+y 60 4(x + y) 6 5
= 18 km/h Þ Ratio = 20 : 18 = 10 : 9
9
Þ x+y= ... (ii) 20. (a); Given speed in still water (x) = 9.6 km/h
2
From (i) and (ii) Let speed of stream = y km/h, distance = d

9 15 15 d 2d
2x = 3 + Þ 2x = Þx = According to question =
2 2 4 (9.6 - y) (9.6 + y)

15 15 - 12 3 2 × 9.6 – 2y = 9.6 + y Þ 3y = 9.6


and y = -3 Þ y= =y=
4 4 4 y = 3.2 km/h

1. (c); Speed of motor boat in still water = 45 km/h 80 4


Time taken to travel along the stream = Þ 60 = 45 + y Þ y = 15 km/h
45 + y 3
20 1 4 80 80
= 1 hr 20 min = 1 = 1 = hr.. Then required time = =
60 3 3 45 - 15 45 - 15
Let speed of current = y km/h
80
Then According to question, = ´ 60 min = 160 min = 2 hr 40 min.
30

318 @BEST300MCQ For More Study Material


Visit: studyiq.com
2. (a); Speed of boat in still water = 5 km/h 7. (c); Let the speed of the motorboat = 36 x
speed of current = 2 km/h Speed of current = 5x
Let distance = d let distance = d
d d d d d 10 31 d 41
= 2+ Þ = 2+ =5 = Þ =
5-2 5+2 3 7 41x 60 6 31x 6
d d 41
– = 2 Þ d = 10.5 km. Means, time to come back of motor boat = hr
3 7 6
3. (a); Speed of boat in still water = 10 km/h 41
Let speed of current = y km/h = ´ 60 min = 410 min = 6 h 50 min
6
24 24 æ 10 + y + 10 - y ö 8. (b); Let speed of first boat = 5a, stream = 2a
+ = 5 Þ 24 ç =5 Speed of stream = 3b, boat = 4b
10 + y 10 - y è (10 + y)(10 - y) ÷ø
3
Stream should be same Þ 2a = 3b Þ a = b
24 ´ 20 2
= 5 Þ 100 – y2 = 96 Þ y2 = 4
100 - y2 5a 5 ´ 3 15
Required Ratio = = =
y = 2 (Neglect – ve sign) 4b 2 ´ 4 8
4. (d); Speed of the stream = 2 km/h 9. (b); x = 3.5 km/h Let speed of current = y
Let speed of the boat in still water = x km/h d 5 d
And distance b/w two boats = d km = ´ Þ 5x – 5y = 2x + 2y
x-y 2 x+y
d 3x = 7y Þ 3 × 3.5 = 7y Þ y = 1.5 km/h
Then = 4 Þ d – 4x = 8 ... (i)
x+2 10. (d); Given x = 4 km/h. Let speed of current = y
d d 3d
= 5 Þ d – 5x = – 10 ... (ii) = Þ 3x – 3y = x + y
x-2 x-y x+y
From eq. (i) and (ii) 2x = 4y Þ 8 = 4y Þ y = 2 km/h
d – 4x – d + 5x = 8 + 10 Þ x = 18 11. (b); Given y = 1.5 km/h
From eq. (i) Let speed of swimmer in still water = x
d – 18 × 4 = 8 Þ d = 80 km d 2d
5. (b); Let speed of boat in still water = x = Þ x + y = 2x – 2y
x-y x+y
speed of current = y Þ distance = d
x = 3y Þ x = 3 × 1.5 Þ x = 4.5 km/hr
d 2d 12. (d); x = 8 km/h, Let speed of current = y
Then = Þ 2x – 2y = x + y
x-y x+y 36 36
=6 Þ =6 Þ 36 = 48 – 6y
x x-y 8-y
3
x = 3y Þ =
y 1 d
6y = 12 Þ y = 2 Þ = 10 Þ d = 100 km.
Required ratio = 3 : 1 8+2
6. (b); Given speed of current = 4 km /h 13. (d); x = 4 km/h, y = 2 km/h
Let speed of rowing in still water = x km/h d 18
= 9 Þ d = 18 km Þ = t Þ t = 3 hr
42 4-2 4+2
distance (d) = = 21 km
2 14. (a); x = 4 km/h y = 2 km/hr

21 21 6
= t Þ t = 3 hr
Then = 2+ 4-2
x-4 x+ 4
15. (a); x = 4.5 km/hr y = 1.5 km/hr
æ 1 1 ö æ 8 ö upstream speed (u) = 4.5 – 1.5 = 3 km/h
21 ç - =2 Þ 21 ç =2
è x - 4 x + 4 ÷ø è x 2 - 16 ÷ø downstream = (v) = 4.5 + 1.5 = 6 km/h
21 × 8 = 2x2 – 32 Þ 200 = 2x2 2uv 2 ´ 3´ 6
x2 = 100 Þ x = 10 km/h Average speed = = = 4 km/hr
u+v 9

319 @BEST300MCQ For More Study Material


Visit: studyiq.com
16. (d); x = 15 km/h, y = 3 km/hr d d
u = 12 km/ v = 18 km/h 20. (a); + = 10 ... (i)
x+ y x-y
2uv 2 ´ 18 ´ 12
Average speed = = 2d
u+v 30
= 4 Þ d = 2(x + y) ... (ii)
= 14.4 km/hr x+y
17. (d); x = 7 km/hr, y = 3 km/hr From equation (i)
d d d d 3d æ x-y+x+ y ö
= 6+ Þ = 6+ Þ =6 dç = 10 Þ 2xd= 10 (x + y) (x – y)
7-3 10 4 10 20 è (x + y)(x - y) ÷ø
d = 40 km xd = 5 (x + y) (x – y)
18. (b); x = 9, y = 3
d
d d d d From equation (ii) put (x + y) =
= 3+ Þ = 3+ Þ d = 36 km 2
9-3 9+3 6 12
19. (d); x = 4 km/h d
xd = 5 ´ (x – y) Þ 2x = 5x – 5y Þ 3x = 5y
2
d d
+ =3 x
4+y 4-y 5
3x = 5y Þ =
We can’t solve this question. y 3

25 39 d d
1. (c); + =8 ... (i) 3. (c); + = 10 ... (i)
x-y x+y x+ y x-y

35 52 2d
+ = 11 ... (ii) = 8 Þ 4(x + y) = d ... (ii)
x-y x+y x+y
Let x – y = A, x + y = B
x-y+x+ y
25 39 From (i) d ´ = 10
+ =8 ... (iii) (x + y)(x - y)
A B
d × 2x = 10 (x + y) (x – y)
35 52
+ = 11 ... (iv) xd = 5 (x + y) (x – y)
A B
Eq. (iii) × 7 – eq. (iv) × 5 d 5 5
From (ii), xd = 5 × × (x – y) Þ x = x – y
4 4 4
æ 175 273 ö æ 175 260 ö
çè + = 56÷ – ç + = 55÷
A B ø è A B ø x 5
x = 5y Þ =
B = 13 y 1
Put it in eq. (iii)
16
4. (a); (x + y) = (x - y) Þ 9x+ 9y = 16x – 16y
25 9
+3 =8ÞA=5 Þ x–y=5 ... (a)
A
7x
x + y = 13 ... (b) 7x = 25y Þ y =
25
From equation (a) and (b)
x = 9, y = 4 7x
Required percentage = ´ 100 = 28 %
Speed of stream = 4 km/hr. 25x
2. (b); Let d be the distance between two points 5. (b); Given, x = 20 km/h y = 5 km/h
(d / 2) (d / 2) d 30 30 6
+ = +6 Total time taken = + = 2+
10 2 10 25 15 5
d d d æ 20 ö 16 16
+ - = 6 Þ d = 6 ç ÷ = 30 km = hr = ´ 60 min = 192 min = 3 h 12 min
20 4 10 è 4ø 5 5

320 @BEST300MCQ For More Study Material


Visit: studyiq.com
d 3 24 36
+
6. (a); = Þ 4d = 3x + 6 ... (i) 10. (a); =6 ... (i)
x+2 4 u v

d 5 36 24 13
= Þ 4d = 5(x – 2) + = ... (ii)
x-2 4 u v 2
Þ 4d = 5x –10 – (ii) ... (ii) eq. (i) × 3 – eq. (ii) × 2
From (i) and (ii) 72 108 72 48 60
+ = 18 Þ + = 13 Þ =5
3x + 6 = 5x – 10 Þ x =8 km/h u v u v v
7. (a); Given y = 2 km/hr v = 12, u = 8 Þ x + y = 12
æ x+y+x-y ö x – y = 8 Þ x = 10, y = 2
10 10 55 11
+ = Þ 10 ç ÷ = Velocity of current = 2 km/hr.
x-y x+y 60 è (x - y)(x + y) ø 12
11. (c); Given, x = 10 km/hr
20x 11 20x 11 26 14
= Þ = = Þ 13x – 13y = 7x + 7y
2 2 12 2
x -4 12 x+y x-y
x -y
240x = 11x2 – 44 Þ 11x2 – 240x – 44 = 0 6x = 20y Þ 20y = 6 × 10 Þ y = 3 km/hr
11x2 – 242x + 2x – 44 = 0 12. (a); Given y = 2 km/h
11x(x – 22) + 2 (x – 22) = 0 6 6 33  2x  33
(11x + 2) (x – 22) = 0 Þ x = 22 km/h
 = Þ 6  =

x2 x2 60 2 60
x  4 
8. (a); Let speed in still water = x, speed of current = y
11x2 – 240x – 44 = 0
55 40 11x2 – 242x + 2x – 44 = 0
Then, + = 13 ... (i)
x+ y x-y (x – 22) (11x + 2) = 0 Þ x = 22 km/hr

44 30 28
+ = 10 ... (ii) 13. (c); Given, x = km/h, speed of current = y km/h
x+y x-y 3

v = x + y, u = x – y d 3d 28
= Þ + y = 28 - 3y
28 28 3
55 40 -y +y
+ = 13 ... (iii) 3 3
v u
56 14 2
44 30 4y = ,y= , y = 4 km/h
+ = 10 ... (iv) 3 3 3
v u
14. (b); Given x = 18 km/h
From eq. (iii) × 3 – eq. (iv) × 4
d 3d
165 120 176 120 = Þ 18 + y = 3(18 – y )
+ = 39, + = 40 18 - y 18 + y
v u v u
18 + y = 54 – 3y Þ 4y = 36, y = 9 km/h
11
= – 1 Þ v = 11, u = 5 Þ x + y = 11 d
v 15. (d); =8
x+ y
x –y=5
Speed of man, x = 8 Þ Speed of current, y = 3 d = 8x + 8y ... (i)
9. (c); Speed of boat = 10 km/h d
= 12
Speed of stream = 4 km/h x-y
d d d d 3d + 7d d = 12x – 12y ... (ii)
+ = 5, + = 5, = 5,
10 + 4 10 - 4 14 6 42 On solving (i) and (ii)
5 ´ 42 8x + 8y = 12x – 12y Þ 4x = 20y, x = 5y
d= = 21 km
10 Here y = 5 Þ x = 5 × 5 = 25 km/h

321 @BEST300MCQ For More Study Material


Visit: studyiq.com
16. (a); Speed of boat = x Let x – y = A, x + y = B
Speed of river current = y
12 18
+ =3 ... (iii)
15 45 15 A B
=3 =
x+y 60 4
36 24 13
x+y=4 ... (i) + = ... (iv)
A B 2
5 1 5 Eq. (i) × 3 – eq. (ii)
=2 =
x-y 2 2
36 54 36 24 13
+ =9 Þ + =
x –y=2 ...(ii) A B A B 2
On solving (i) and (ii)
x = 3 and y = 1 km/h 30 13 30 5
= 9- Þ = Þ B = 12
B 2 B 2
6 6 æ 2x ö From eq. (iv)
17. (c); + = 2 Þ 6ç ÷ =2
x-y x+y çè x 2 - y 2 ÷ø
36 9
= Þ A = 8 Þ x – y = 8 Þ x + y = 12
Put y = 4 km/hr A 2
2x = 20 Þ x = 10 km/hr Þ y = 2 km/hr
6x
= 1 Þ x2 – 16 = 6x Þ x2 – 6x – 16 = 0
x 2 - 16 48 48
+
20. (a); = 14 ... (i)
x2 – 8x + 2x – 16 = 0 Þ x(x – 8) + 2 (x – 8) = 0 u v
(x + 2) (x – 8) = 0 Þ x = 8 km/hr 3 u 3v
18. (d); Here x = 9 km/hr and y = 1.5 km/hr = Þ u=
4 v 4
105 105 105 105 Put it in eq. (i)
+ =T Þ T= +
9 + 1.5 9 - 1.5 10.5 7.5
4 ´ 48 48 7 ´ 48
= 10 + 14 = 24 hrs + = 14 Þ = 14
3v v 3v
12 18
19. (c); + =3 ... (i) 3
x-y x+y v= 8 Þ u= ´ 8 = 6 Þ x – y = 6, x + y = 8
4
36 24 13 x = 7 Þ y = 7 – 6 = 1km/hr
+ = ... (ii)
x-y x+y 2 rate of stream = 1 km/hr

1. (c); Down stream speed SD = Speed of Boat + Speed 3. (c); Boat man goes down stream in 1 hour = 1 × 6
of stream = 6 km
S B + S S = 11 Downstream speed = 6 km/hr
SB - S S = S Upstream speed = 2 km/hr
2S B = 16 Speed of Boat in still water = 4 km/hr
5
Speed of Boat = 8 km/hr Time taken by Boat to cover 5 km = hr.
4
16 = 1 hr 15 min
2. (b); Downstream speed of Boat = = 8 km / h
2 4. (b); This type of question can also be done by option.
16 take (b) option so speeds are 3 and 1 km/hr
Upstream speed = = 4 km /hr
4 downstream distance cover in 1 hour = 4 km.
SB + SS = 8 Þ SB – SS = 4 upstream distance = 2 km
from these two equations By General Method:
SB = 6 Þ SS = 2 Let's Man's upstream's speed is x km/hr
so speed of Boat in still water = 6 km/hr Downstream's speed = 2x km/h

322 @BEST300MCQ For More Study Material


Visit: studyiq.com
In one hour distance covered in downstream
12 12
= 2x + x = 3x \ - =6
(x - y) (x + y)
In one hour upstream distance covered = 2x – x
=x 6(x2 – y2) = 24 y
x2 – y2 = 4y
Downstream 's speed 3x
= =3:1 x2 = (4y + y2) ...(i)
Upstream 's speed x
12 12
5. (c); Let's man upstream speed be x km/hr And - =1
(2x - y) (2x + y)
and downstream = y km/h
Then Distance covered upstream in 8 hr 48 min 24y + y 2
= Distance covered downstream in 4 hrs. x2 = ... (ii)
4
4 44x From (i) and (ii) we have:
x´8 = y´4 Þ = 4y
5 5
24y + y 2
4y + y 2 =
11 y 11 4
y= x Þ =
5 x 5 3y2 = 8y
æ y + xö æ y - xö 8 8 2
Required Ratio = ç :ç = = 8:3 y= \ speed of current = 2 mph
è 2 ÷ø è 2 ÷ø 3 3 3
11. (a); Let the speed of motorboat and stream is x km/h
7 and y km/h respectively.
6. (c); Speed of Boat upstream = ´ 60 = 10 km/hr.
42 According to question
Speed of Boat = 10 + (Speed of stream)
= 13 km/hr æ 28 ö 28
2ç ÷ =
7. (a); Man's upstream speed = 5 – 1 = 4 km/hr è x + yø x - y
Down stream speed = 6 km/hr x = 3y
Let the distance is D km/h when the speed of the stream is doubled
D D 3D + 2D
+ =1 Þ =1 28 28 672
4 6 12 + =
x + 2y x - 2y 60
12
D= = 2.4 km 28 28 672
5 + = éx = 3yùû
5y y 60 ë
8. (b); Let the speed of stream = S km/hr
30 30 9 1 1 24
+ = + =
15 - S 15 + S 2 5y y 60
From here try to use the option. y = 3 km/h, x = 9 km/h
So option (b) i.e. S = 5 km/hr fit correctly. 12. (b); Let the speed of B = x km/hr
9. (a); Let the speed of the stream be x mph, Then the speed of A = (x + 5) km/hr
Downstream speed = (10 + x) mph \ Distance travelled by A to meet his friend
Upstream speed = (10 – x) mph = 42 km
36 36 90 5
\ - =
(10 - x) (10 + x) 60 Distance travelled by B = 35 % less of A
7
x2 + 48x – 100 = 0
é 250 5ù
(x + 50) (x – 2) = 0 =ê = ú
ë 7 ´ 100 14 û
x = 2 mph.
10. (d); Let the speed in still water be x mph and the Distance travelled by A = 14x
speed of the current be y mph. Then by B = 9x (14 – 5 = 9)
Upstream speed = (x – y) 14x = 42, x = 3
Downstream speed = (x + y) 9 × 3 = 27 km Þ travelled by B.

323 @BEST300MCQ For More Study Material


Visit: studyiq.com
Time is equal 15. (c); River : Boat
42 27 Speed v : 4v
= Þ x = 9 km/hr
x+5 x
Upstream speed = 3v
Speed by A = 9 + 5 = 14 km/hr
Distance = t × speed
13. (b); Let the distance between Varansi and Patna is d
km, and the speed of the boat in still water is x = 3v × 2 = 6v
km/h, and the speed of stream is y km/h. Downstream speed = 5v
\ Diya will move at a speed of stream D 6v
Speed = = = 4v
d T æ 1ö
24 = Þ d = 24y km çè 1 + ÷
y 2ø

24y 24y Intial Downstream speed = 5v


\ + = 10 Final Downstream speed = 4v
x+y x-y
When speed of boat increases by 40% 5v - 4v
% decreament = ´ 100 = 20%
5v
24y 24y
+ =7 16. (b);
7 7
x+ y x-y
5 5 A C B
After solving this we get speed of boat A = b1
x = 5y speed of boat B = b2
let they meet at point C
24y étime taken to sail from ù
t=
6y êë Varansi to Patna ú
û total distance 2W
Total time = =
= 4 hours Relative speed b1 + b 2
14. (a); Let the distance between two points A and B is d
2W
km. And the speed of motoboat in still water Distance travelled by A = ´ b1
= x km/hr b1 + b2
speed of stream is y km/h
2W
Distance travelled by B = ´ b2
d d b1 + b 2
From question x + y + x - y = 5
17. (a); Let the speed of Boat in still water = x km/h
Let original time = 5 and that of current = y km/h
20% of 5 = 1 (new time)
d(x + y + x – y) = 5 (x2 – y2) 60 60
+ = 10 ...(i)
x+y x-y
d=
(
5 x -y
2 2
) ...(i)
2x 3 2
=
When the speed of Boat is doubled x+y x-y

d d 3 (x – y) = 2 (x + y)
+ =1
2x + y 2x - y x+y 3
=
d [2x + y + 2x – y] = 4x2 – y2 x-y 2
4x 2 - y 2 put these value in (i)
d= ...(ii)
4x 60 60
d is same in both the cases. + , = 10
3R 2R
5(x 2 - y 2 ) 4x 2 - y 2 R = 5, then x + y = 15
=
2x 4x x – y = 10
6x = 9y
2 2
1
speed of Boat = ( 15 + 10) = 12.5 km / h
x 3 2
=
y 2 speed of current = 2.5 km/h

324 @BEST300MCQ For More Study Material


Visit: studyiq.com
19. (c); speed of Boat : speed of current = 36 : 5
3 1
18. (d); Upstream speed = ´ ´ 60 = 3 km/hr so, downstream speed = 41
4 15
1
3 Distance = 41 ´
Downstream speed = ´ 60 = 4.5 km/h 6
4 ´ 10
upstream speed = 31
3 + 4.5
speed of boat = = 3.75 km / h 41 41
2 Time = = hr. = 13.22 min.
6 ´ 31 186
4.5 - 3 20. (a); Downstream speed = 20 km/h
speed of current = = .75 km /h
2 20
Upstream speed = = 10 km /h
3.75 2
Ratio of his speed to current = = 5:1
.75 20 + 10
Speed of Boat = = 15 km /h
2

325 @BEST300MCQ For More Study Material


Visit: studyiq.com
Chapter
Data Interpretation
14
What is data interpretation: When data is organized into tables and charts it is done with the purpose of making it
meaningful. The objective of data interpretation is to assess whether a student can understand bars and charts and
Answer some questions based on them. This act of organizing and interpreting data to get meaningful information
under a given set of conditions is Data interpretataion.

About data interpretation: This is the calculation intensive portion, it consists of a myrid of graph. charts and tables
and analyze data. The key to crack this area is to quickly Identify the key pieces of information that you will require to
work on.

Basic key that will help you to solve this topic:


• Calculation
• Square
• cube
15 –20 days calculation
• table
• BODMAS
• Percentage
• Profit and loss
• Ratio and proportion
• Average

Types of Data Interpretation:


• Data table
• Line graph
• Pie charts
• Bar graph
• Mixed graph
• Line with pie chart
• Table with Bar
• Table with Line, etc.
• Radar graph
• Triangular graph
• Case study (Puzzle)
• Venn Diagram
• Table format
Approach for data interpretation:
• First you look carefully at the table or graph and the direction. Note the years to which, the data refers to and
the units. Sometimes the figures may be given in thousands. While the Answer may be millions Resulting in
mistakes.
• The level of approximation that can be done is assessed from the choices. If the answer is wide, time should not
be wasted in working out exact figures. If the choice ‘none of the above exists, a close approximation may be
required’.
• Read the question carefully, it will give an indication as to which row and column should be seen. A carefull
reading of the question will reveal exactly what is to be done and the units in which the answer is required.
• There may be one or two very large question requiring calculations. Attempt these at the last.
• Revise bar charts, table and line graphs before attempting D.I. question remember that the D.I. section is a
scoring one and also time saving.

326 @BEST300MCQ For More Study Material


Visit: studyiq.com
Data Table: The numbers in the bracket give the maximum marks in each subject.
The following table gives the percentage of marks obtained by seven students in six different subjects in an examination. Study
it and answer the question given below it.

Students Subject (Maximum Marks)


Maths Chemistry Physics Geography History Computer
(150) (130) (120) (100) (60) Science (40)
Golu 90 50 90 60 70 80

Mithi 100 80 80 40 80 70

Suraj 90 60 70 70 90 70

Gapplu 80 65 80 80 60 60

Mahi 80 65 85 95 50 90

Khushi 70 75 65 85 40 60

Sheetal 65 35 50 77 80 80

1. What are the average marks obtained by all the seven students in physics? (Round off two digits after decimal)
(a) 77.26 (b) 89.14 (c) 91.37 (d) 96.11
2. The number of students who obtained 60% or above marks in all subjects is
(a) 1 (b) 2 (c) 3 (d) 4
3. What was the aggregate of marks obtained by suraj in all the six subjects ?
(a) 409 (b) 419 (c) 429 (d) 449
4. In which subject is the overall percentage the best?
(a) Maths (b) Chemistry (c) Physics (d) History
5. What is the overall percentage of sheetal
(a) 52.5% (b) 55% (c) 60% (d) 63%

1
Solution 1: é( 90% of 120 ) + ( 80% of 120 ) + ( 70% of 120 ) + ( 80% of 120 ) + ( 85% of 120 ) + ( 65%of 120 ) + ( 50%of 120 ) ùû

1
= é( 520%of 120 ) ùû

624
= = 89.14
7
Solution 2: From the table, it is clear tha that Suraj & Gapplu have 60% or more marks in each of the six subjects.
Solution 3: Aggregate marks obtained by suraj
= [(90% of 150)+(60% of 130)+(70% of 120)+(70% of 100)+(90% of 60)+(70% of 40)]
= [135 + 78 + 84+ 70 + 54 + 28] = 449
Solution 4: We shall find the best overall % with respect to each subject: Mathematics
Solution 5: Aggregate marks obtained by sheetal.
= [(65% of 150)+(35%of 130)+(50% of 120)+(77% of 100) + (80% of 60)+(80% of 40)] = 360
Maximum marks = 600

360
Overall % = ´ 100 = 60%
600

327 @BEST300MCQ For More Study Material


Visit: studyiq.com
Line Graph:
Answer the question based on the given line graph
Following line graph shows the ratio of export to import of company A and company B over the year
2
Company A
1.75

1.5 Company B
1.25

.75

.5

.25

0
Years 2005 2006 2007 2008 2009 2010

1. In how many of the given years were the exports more than the imports for company A?
(a) 2 (b) 3 (c) 4 (d) 5
2. If the imports of company A in 2007 were increased by 40%. What would be the ratio of exports to the increased
imports?
(a) 1.50 (b) 1.25 (c) 1.75 (d) 1.35
3. If the exports of company B in 2008 was Rs 237 Crore, what was the amount of imports in that years ?
(a) 189.6 (b) 188.8 (c) 184.6 (d) 182.8
4. In 2005, the export of company A was double that of company B. If the imports of company A during the year was
180 crore. What was the approximate amount of imports of company B during that year?
(a) 190 crore (b) 210 crore (c) 225 crore (d) 200 crore
5. In which year were the exports of company A minimum proportionate to its imports.
(a) 2008 and 2009 (b) 2009 (c) 2008 (d) 2005
Solution 1: required ratio, more than 1 for the years; 2005, 2006 and 2007,ie; for 3 years
Solution 2: In 2007, for company A the ratio of exports to import = 175 : 100
Let exports of company A = 175 x Þ and imports of company A = 100x
New imports of company = 140% of 100x = 140x
175x
There for requied ratio = = 1.25
140x
Solution 3: Let import of company B in 2008 = x
237 237
Then, = 1.25 Þ x = Þ 189.6 crore
x 1.25
Solution 4: In 2005 for company A Þ Export = 1.75 × 180 crore
1.75 ´ 180 Ex
in 2005 for company B export = = 157.5 crore, But = .75
2 Im

157.5
Import of company B = = 210 crore
.75
Solution 5: 2008 and 2009, A option.

328 @BEST300MCQ For More Study Material


Visit: studyiq.com
Pie Charts or Circle Graphs:
Distribution of candidate who were enrolled in MBA and the candidate (out of those enrolled) who passed the exam in
different institutes

X X
16% P 12% P

V 22% V 18%

12% 15%
Q Q
T T
15% 17%
8% 9%
S R S R

17% 10% 16% 13%

Total number of candidates Total number of candidates


enrolled = 8550 who passed the exam = 5700

1. What percentage of candidates passed the exam from institute T out of the total no. of candidate enrollled from the
same institute?
(a) 50% (b) 52.5% (c) 75% (d) 80%
2. Which institute has the highest % of candidates passed to the candidates enrolled?
(a) Q (b) R (c) V (d) T
3. The no. of candidates passed from institute S and P together exceeds the no. of candidates enrolled from institutes
T and R together by?
(a) 228 (b) 279 (c) 399 (d) 407
4. What is % of candidates passed to the candidate enrolled for institutes Q and R together ?
(a) 68% (b) 80% (c) 74% (d) 65%
5. What is the ratio of candidates passed to the candidates enrolled from institute P ?
(a) 9 : 11 (b) 14 : 17 (c) 6 : 11 (d) 9 : 17
9%of 5700
Solution 1: Required % = ´ 100 = 75%
8%of8550
Solution 2: The % of candidates passed to candidates enrolled can be determined for each institute as under.
17% of 5700 13%of 5700
For Q Þ ´ 100 = 75.56% ; For R Þ ´ 100 = 86.67%
15%of 8550 10%of 8550

9%of 5700 15%of 5700


For T Þ ´ 100 = 75% ; For V Þ ´ 100 = 83.33%
8% of 8550 12%of 8550
So the highest of these is 86.67% corresponding to institutes R.
Solution 3: Required Difference = [(16% + 18%) of 5700] – [(8% + 10%) of 8550] = 1938 – 1539 = 399
30%of 5700
Solution 4: 25% of 8550 ´ 100 = 80%

18% of 5700 6
Solution 5: = = 6 : 11
22%of 8550 11

329 @BEST300MCQ For More Study Material


Visit: studyiq.com
Bar Graph: Percentage profit earned by two companies X & Y over the given years.

Income - Expenditure
Pr ofit% = ´ 100
Expenditure
Company X
70
65
Company Y
60
60
55 55
50 50 50 50
50
45 45
Profit%

40
40
35

30

20

2006 2007 2008 2009 2010 2011


1. The income of two company X and Y in 2010 were in the rato of 3 : 4. respectively. What is the respective ratio of
their expenditures in 2010?
(a) 7 : 22 (b) 14 : 19 (c) 15 : 22 (d) 27 : 35
2. If the expenditure of company Y in 2007 was Rs 220 crore, what was its income in 2007?
(a) Rs. 312 crore (b) Rs. 297 crore (c) Rs. 283 crore (d) Rs. 275 crore
3. If the expenditures of company X and Y in 2006 were equal and the total income of the two companies in 2006 was
Rs 342 crore, what was the total profit of the two company together in 2006?
(a) Rs. 240 crore (b) Rs. 171 crore (c) Rs. 120 crore (d) Rs. 102 crore
4. The expanditure of company X in the year 2008 was Rs 200 crore and the income of company X in 2008 was the
same as its expenditure in 2011. The income of company X in 2011 was?
(a) Rs. 465 crore (b) Rs. 385 crore (c) Rs. 335 crore (d) Rs. 295 crore
5. If the income of two company were equal in 2009, then what was the ratio of expenditure of company X to that of
company Y in 2009?
(a) 6 : 5 (b) 5 : 6 (c) 11 : 6 (d) 16 : 15
165%of E 1 3 E 15
Solution 1: = Þ Therefore, 1 = = 15 : 22
150%of E 2 4 E 2 22
Solution 2: 220 + 35% of 220 Þ 220 + 77 = 297 crore
Solution 3: Let the expenditure of each companies x and y in 2006 be 100
342
Then we have 100 + 40 + 100 + 45 = 342 crore Þ Required profit = ×85 = 102 crore
285
Solution 4: Expenditure of Company X in year 2008 is 200 crore, Then income of company X in year 2008
= 200 + 55% of 200 = 310 crore
Hence, the expenditure of company X in year 2011 is 310 crore,
Therefore, the required income of X in year 2011 = 310 + 50% of 310 = 465 crore
150 160
Solution 5: X= Y
100 100
x 160 16
be the incomes of two companies in 2009; = = = 16 : 15
y 150 15

330 @BEST300MCQ For More Study Material


Visit: studyiq.com
Mixed Graph: Pie Chart breakup shows that number of employees in different department of an organization.
Table shows the percentage of men in each department (Rest one woman)
Total number of employess = 1200

Accounts
IT
15%
20%

Distribution Production
12% 25%

HR
Marketing
23% 5%

Departments Percentage
of Men
IT 35
PRODUCTION 87
HR 25
MARKETING 75
DISTRIBUTION 50
ACCOUNTS 65
1. What is the Respective Ratio of the number of men from the marketing department to those from the accounts
department
(a) 23 : 13 (b) 13 : 9 (c) 27 : 19 (d) 17 : 11
2. The number of women from the IT department are what percent of the number of men from the same department
(Rounded off to two deigits after decimal)
(a) 159.38% (b) 190.07% (c) 185.71% (d) 168.23%
3. The total number of men from all departments together forms what percent of the total no. of employees in the
organization?
(a) 67% (b) 63% (c) 55% (d) 58%
4. What is the total number of women from the production department and the HR department together ?
(a) 78 (b) 84 (c) 92 (d) 64
5. Which department has the highest number of women employees
(a) IT (b) marketing (c) Accounts (d) Distribution
75 ´ 23
Solution 1: Required Ratio = = 23 : 13
65 ´ 15
65 ´ 100
Solution 2: Required Percentage = = 185.71%
35

20 ´ 35 ´ 1200
Solution 3: Men in IT = = 89 , Similarly calculate men from all departements; Total men = 756
100 ´ 100
756
Required Percentage = ´ 100% = 63%
1200
Solution 4: Required number of woman = no. of women from production department + number woman from HR
department 39 + 45 = 84
Solution 5: From the table, it is clear that IT department has highest number of women emp.

331 @BEST300MCQ For More Study Material


Visit: studyiq.com
Mixed Graph:
Data Related to human resource of a multinational company (x) which has 145 offices across 8 – countries

28
26 24
24
22 21
no. of offices

22
20
18
18
16 17
16
13
14
14
12
10

Countries A B C D E F G H

Percentage distribution of post-graduate employees Number of employees and respective Ratio male to
across – 8 – countries female across - 8 countries

Total no. of Respective ratio


Countries
Employees of Male and
A Female emp.
H
75%
A 2568 5:7
80%
B
B 2880 11 : 5
65%

G 55% C 2310 10 : 11
40% D 3575 3:2
C
%
F 75 E 2054 7:6
60%
50% F 2788 20 : 21
E D
G 3720 8:7

H 3360 9:5

1. If the number of male post – graduate, employees in country H is 1800 what percent of female employees in that
particular country is post graduate?
(a) 76 (b) 74 (c) 72 (d) 64
2. In which of the given countries is percentage of woman employees to number of employees (both male and female)
in that country the second Lowest?
(a) G (b) B (c) E (d) H
3. What is the respective ratio between total number of male employees in countries B and H together and total
number of female employees. in countries C and D together?
(a) 63 : 52 (b) 51 : 38 (c) 77 : 64 (d) 69 : 44
4. What is the difference between average number of post graduate employees in countries A, B and D together and
Average number of Post-graduate employees in countries F, G, and H together?
(a) 282 (b) 276 (c) 294 (d) 342
5. Which of the given countries has the highest number of average employees per offices?
(a) F (b) H (c) B (d) C

332 @BEST300MCQ For More Study Material


Visit: studyiq.com
3360 ´ 80
Solution 1: Total PG emp = = 2688 Þ Male PG = 1800 Þ Female PG. emp. = 2688 – 1800 = 888
100

3360 ´ 5 888
Total female in country H = = 1200 Þ Required % F% = ´ 100 = 74%
14 1200
Solution 2: Required ratio of country
7 6 5 5
G= × 100 = 46.6%; E = × 100= 46.15%; B = × 100 = 31.1%; H = × 100 = 35.7%; D = 40%
15 13 16 14
Hence, 2nd lowest country H
Solution 3: Total male in B and H or girl C and D
Requird Ratio
æ 2880 ´ 11 3360 ´ 9 ö æ 2310 ´ 11 3575 ´ 2 ö
ç + ÷:ç + ÷ Þ (1980 + 2160) : (1210 + 1430)
è 16 14 ø è 21 5 ø
4140 : 2640 Þ 207 : 132 Þ 69 : 44
75 2880 ´ 65 3575 ´ 60
Solution 4: Total PG. emp in A, B and D is: 2568 ´ + +
100 100 100
5943
1926 + 1872 + 2145 = 5943 Þ Average Pg in AB & D is = = 1981
3
Total PG in F,G and H = 2091+2046+2688 = 6825
6825
Average PG in F.G.H = = 2275 Þ Required Diff = 2275 –1981 = 294
3
Solution 5: Average employee/office is
2788 2880 3360 3575 2310
F= = 164 Þ B = = 160 Þ H= = 160 Þ D = = 162.5 Þ C = = 165
17 18 21 22 14
Hence C is required Answer.
Missing Data Interpretation—

No. Of People A B C D E
Principal

Rate of interest 10%


Simple interest
for 1st year
Compound
interest

1st year

First two year 21000

First three year 33800

First four year

Amount 2nd year


for compound interest 4624
Amount 3rd year 4913
for compound

333 @BEST300MCQ For More Study Material


Visit: studyiq.com
1. If the principal of E is 20% more than the principal of B than , find the principal of E?
(a) 72830 (b) 82320 (c) 76380 (d) 84430
2. Find the principal of (A) person?
(a) 4000 (b) 4096 (c) 3840 (d) 4286
3. For the person D if the Difference between simple interest and compond interest for the fourth year is 507. If rate of
interest of person D is same as person ‘B’. Then the principal of (D)?
(a) 7000 (b) 7120 (c) 7203 (d) 7224
4. Find the compound interest for the person C. For 3 years if the principal of C is the sum of principal of A, E, and D
(a) 24839.212 (b) 28487.312 (c) 30987.889 (d) 32183.789
Solution 1. To find principal of E, we must know B’s principal.

éæ r ö
2
ù
P
for 2 year, C = 21000 = êç 1 + ÷ - 1ú ...(i)
ëêè 100 ø ûú

éæ r ö
3
ù
P
for 3 year, C = 33800 = êç 1 + ÷ - 1ú ...(ii)
ëêè 100 ø ûú
Equation (ii) divide by equation (i)
(ii) and (i)
2
æ r ö
1+ ÷ -1
21000 çè 100 ø r
= 3 ; let = x Þ 210 [(1+x)3-1] = 338 [(1+ x)2–1]
33800 æ r ö 100
ç1+ ÷ -1
è 100 ø

210 [1+x3+3x(1+x)–1] = 338 [1+ x2+2x–1] Þ 105x2 + 315x + 315 = 169x + 338 Þ 105x2 + 146x –23 = 0

+15 -160 æ -161 ö


x= , ç neglect ÷ coz (–)
105 15 è 105 ø

r 15 100 2 éæ 100 ö
2
ù
Now, = r= = 14.28% or 14 % Þ Find P = 21000 = P ê ç 1 + ÷ - 1ú
100 105 7 7 êëè 700 ø úû

( 64 - 49 ) 21000 ´ 49
21000 = P ´ Þ =P
49 15
68600 = P B’s Principal Þ E’s principal is 20% more than B
So B = 68600 Þ E = 82320
2
289 ´ 100 æ 625 ö P ´ 17 ´ 17 Þ 4624 ´ 16 ´ 16 = P
Solution 2: for R = = 6.25% Þ 4624 = P ç 1 + ÷ Þ 4624 =
4624 è 10000 ø 16 ´ 16 17 ´ 17
4096 = P
Solution 3: By question – 1– Rate of interest D = B as we know B = 14.28%

P A
7 4
84 = 4096 ( fourth year amount )
(7 3
83 ) = 3584 ( 3rd year amount )

(73 83) × 7
Comound input only 4th year Amount = 512, Simple interest for the 4th year = 343
Difference = 512 – 343 = 169 Þ 169 = 507
507 507 4
1= Þ 74 = ´ 7 = 7203
169 169

334 @BEST300MCQ For More Study Material


Visit: studyiq.com
Solution 4: Principal of C = A + D + E = 4096 + 7203 + 82320 = 93619

éæ 10 ö
3
ù
Now C.I. for person ‘C’ = 93619 êç 1 + ÷ - 1ú = 30987.889
êëè 100 ø úû
Missing D.I.: A professor keeps data on students in tabular form on performance and sex of the student. The data is
kept on a computer disk, but unfortunately some of its data has been lost because of a virus. Only the following data
has been recovered.

Performance Total
Average Good Excellent
Male — — 10 —
Female — — — 32
Total — 30 — —

An expert committee was formed, which decided that the following facts were self evident.
(1) Half the students were either excellent or good.
(2) 40% of the students were females.
(3) One third of the male students were average.
1. How many students were both female and excellent
(a) 0 (b) 8 (c) 16 (d) 32
2. How many students are both male and good?
(a) 10 (b) 16 (c) 22 (d) 48
3. Among average students, what is the ratio of male to female?
(a) 1 : 2 (b) 2 : 1 (c) 3 : 2 (d) 2 : 3
4. What proportion of female students are good?
(a) 0 (b) 0.25 (c) 0.5 (d) 1.0
5. What proportion of good students are male?
(a) 0 (b) 0.73 (c) 0.4 (d) 1.0
Solution:
• From the table it is given that the number female is 32 and this number is 40% of the students. This means that
the total no. of students is 80 and the number of male is 48.
• Now in the question we are given that half the students were excellent or good, also, it is given that one third
of the male students were average. So the number of male average students is 16.
• (No. of good students + No. of Excellent student) Þ 40
• Total good students = 30
• Therefore the number of Excellent Students Þ 40 – 30 Þ 10
• Also in the question it is mentioned that 1/3 rd of male students were average, therefore total no. of males
students that were good = (48 – 16 – 10) = 22
Now fill the graph

Performance Total
Average Good Excellent
Male 16 22 10 48
Female 24 8 0 32
Total 40 30 10 80

335 @BEST300MCQ For More Study Material


Visit: studyiq.com
Solution 1: (a); No. of student are both, Female and Excellent = 0
Solution 2: (c); No. of students who are both, Male and good = 22
Solution 3: (d); Ratio male to female among Average student = 16 : 24 = 2 : 3
Solution 4: (b); Proportion of female student who are good = 8/32 = 0.25
22
Solution 5: (b); Proportion of good students who are male = Þ 0.73
30
Radar Graph: The radar chart is a chart that consists of a sequence of equi-angular spokes, called radii, with each
spoke representing one of the variables. The radar chart also known as a spider chart because of its appearance, has
its zero starting point in the middle from the middle of each axis of the chart can protrude out like the spokes on a
wheel. There can be one axis for each subject. And there is no limit on the no. of subject that can be used it can also be
described as a radial grid with zero starting point being the middle.
Radar Graph
Number of students (in thousands) in two different universities in six different years.

2007
35

30
25

2012 20
15
2008

10
5
Þ University 1
0

Þ University 2

2011 2009

2010
1. What was the difference between the number of students in university-1 in the year 2010 and the number of
students in university-2 in the year 2012?
(a) zero (b) 5000 (c) 15000 (d) 10000
2. What is the sum of the number of students in university-1 in the year 2007 and the number of students in
university-2 in the years 2011 together?
(a) 40000 (b) 55000 (c) 45000 (d) 57000
3. If 25% of the students in university-2 in the year 2010 were female what was the number of male students in the
university - 2 in the same year?
(a) 11250 (b) 12350 (c) 12500 (d) 11500
4. What was the percentage increase in the number of students in university -1 in the year 2011 as compared to the
previous year?
(a) 135 (b) 15 (c) 115 (d) 25
5. In which year was the difference between the number of students in university-1 and the number of students in
university-2 highest?
(a) 2008 (b) 2009 (c) 2010 (d) 2012

336 @BEST300MCQ For More Study Material


Visit: studyiq.com
Solution 1: Required Þ 20,000 – 20,000 Þ 0 (zero)
Solution 2: Required Þ 30,000 + 10,000 Þ 40,000
75
Solution 3: Boys Þ 15000 ´ Þ11250
100

5000
Solution 4: Required increased Percentage ´ 100 Þ25%
20000
Solution 5: Difference Þ 2012 higher (shows in figure)
Triangular Bar diagram: A survey was conducted in five cities viz. Pune, Kanpur, Raisen, Surat and Trivandrum for
the percentage of people using T (Trains), B (Buses) & C (Cars), as mode of transport. Number of persons surveyed in
cities Pune, Kanpur, Raisen, Surat and Trivandrum are 2000, 4000, 6000, 3000 and 8000 respectively.

Train
T : 100

Pune
T : 75 B : 25

Kanpur Raisen B : 50
T : 50

Surat
T : 25 B : 75

Trivandrum
B : 100
Cars Buses
C: 100 C : 75 C : 50 C : 25
1. The city, where the least number of Persons use buses is,
(a) Surat (b) Raisen (c) Kanpur (d) Pune
2. The average number of persons using trains for transportation in Pune, Kanpur, Raisen and Trivandrum is
(a) 1880 (b) 1875 (c) 1950 (d) 1850
3. The mode of transport used by the least number of persons in all the given cities
(a) trains (b) buses (c) cars (d) cars & buses
4. Out of total number of car users in all the cities together, how many cities have less than 30% of car user?
(a) Five (b) Two (c) Three (d) Four
Solution 1: Number of Persons using buses in
Surat Þ 75% of 3000 Þ 2250; Raisen Þ 50% of 6000 Þ 3000; Kanpur Þ 50% of 4000 Þ 2000
Pune Þ 25% of 2000 Þ 500; Thus, Pune is the required city
Solution 2: Number of persons using trains in
Pune Þ 75% of 2000 Þ 1500; Raisen Þ 50% of 6000 Þ 3000
Kanpur Þ 50% of 4000 Þ 2000; Trivandrum Þ 12.5% of 8000 Þ 1000
1500 + 2000 + 3000 + 1000 7500
Thus, required average number = = Þ 1875
4 4
Solution 3: No. of persons using buses Þ 25% of 2000 + 50% of 4000 + 50% of 6000 + 75% of 3000 + 87.5% of 8000
Þ 14750, No. of persons using trains
Þ 75% of 2000 + 50% of 4000 + 50% of 6000 + 25% of 3000 + 12.5% of 8000 = 8250

337 @BEST300MCQ For More Study Material


Visit: studyiq.com
Number of persons using cars
Þ 50% of 2000 + 62.5% of 4000 + 37.5% of 6000 + 62.5% of 3000 + 43.75% of 8000 Þ 11125
Hence, Trains are used by the least no. of persons in the given cities.
Solution 4: Total no. of cars users = 11125 & 30% of 11125 = 3337.5
Thus, in four cities, the no. of car users is less than 3337.5 i.e less than 30% of the car users. These cities are
Pune, Kanpur, Raisen, Surat.
Case Studies or Puzzle D.I.

Table (Case Studies) Puzzle (Venn Diagram)


Study the following information carefully to answer the question.
The officer’s colony of Badown has 2800 members out of which 650 members read only English newspaper.
500 members read only Hindi newspaper and 450 member read only Marathi newspapers. The number
of member reading all the three newspapers is 200. 400 members read Hindi as well as Marathi newspapers
only and 300 members read English and as well as Marathi newspaper only and 300 members read
English and as well as Hindi newspaper only.
1. How many members read atleast two newspaper?
(a) 1200 (b) 1800 (c) 1500 (d) 1100
2. Find the number of members reading Hindi newspaper?
(a) 750 (b) 980 (c) 1400 (d) 1020
3. How many members read only one newpaper?
(a) 1560 (b) 1600 (c) 1650 (d) 1540
4. Find the number of members reading atleast three newspaper?
(a) 200 (b) 300 (c) 460 (d) 750
Given information can be easily described by the following venn diagram

English
Hindi
650
300 500
200
300
400

450

Marathi
Solution 1: From the above venn diagram Þ 300 + 300 + 400 + 200 = 1200
Solution 2: No. of members reading Hindi newspaper Þ 500+300+200+400 = 1400
Solution 3: The number of members reading only one newspaper Þ 650+ 500 +450 = 1600
Solution 4: The number of members reading atleast three newspapers = 200
Case Study (Table Formate): Study the information carefully to answer the question that follow.
A. Company produced five different products viz. mobile phone, pen drive, calculators, televisions and washing
machines. Total number of all the five products is 1650. 24% of the total number of products is mobile phones. One
sixth of the total number of products is Pendrives. 14% of the total number of product is calculators. Remaining
products are either television or washing machine; number of washing machine is 50 more than the number of
televisions produced.

338 @BEST300MCQ For More Study Material


Visit: studyiq.com
1. What is the difference between the total number of televisions and mobile phones together and the number of
calculators produced?
(a) 534 (b) 524 (c) 514 (d) 523
2. Number of televisions produced is approximately what % of the total number of calculators and washing machines
produced together?
(a) 63% (b) 55% (c) 59% (d) 51%
3. What is the total number of pen drives, calculator and washing machines produced by the company?
(a) 907 (b) 917 (c) 925 (d) 905
Solution: Total no. of products = 1650 Þ Number of mobile phone = 24% of 1650 = 396
1
Number of pen drive = th of 1650 = 275, Number of calculators = 14% of 1650 = 231
6
Remaining number of products = 1650 – (396 + 275 + 231) = 748
These remaining products i.e, 748 products are either televisions or washing machines.
Let the number of washing machine and televisions be x and y respectively then,
x + y = 748; x – y = 50 Þ x = 399, y = 349
Now,

Products Number of products


Mobile phones 396
Pen drives 275
Calculators 231
Washing Machines 399
Televisions 349

Solution 1: Total no. of television & mobile phones together = 396 + 349 = 745
And No. of calculators = 231, Required difference = 745 – 231 = 514
349
Solution 2: No. of televisions = 349 Þ Required% = ´ 100% Þ 55%
630
Solution 3: Total no. of pen drives, Calculators & Washing Machines = 275 + 231 + 399 Þ 905

Q ue st ion 1. W.B. produces approximately what percent of the total


rice produced in India?
(a) 12.2% (b) 13.5%
Directions (1 – 5): Study the following questions and choose
the correct answer. (c) 18% (d) 15%
State wise production of different crops for the year 1993- 2. The amount of wheat produced by U.P. is more than
94 (in million tonnes) the amount of wheat produced by the other four states
listed in the table, by
(a) 6.97 m tonnes (b) 6.16 m tonnes
State Rice Wheat Pulses
(c) 5.52 m tonnes (d) 5.89 m tonnes
U.P. 7.18 15.97 2.76 3. What percentage of the total wheat production was
W.B. 8.09 0.81 0.35 produced by states other than those who are not given
M.P. 3.67 3.72 2.32 in the table?
(a) 38% (b) 42%
Maharashtra 1.94 0.86 1.11
(c) 48% (d) 58%
Haryana 1.36 4.42 0.36
4. U.P. produced approximately what percent of the all
All India 58.64 44.23 12.2 India production of all the three crops take together?

339 @BEST300MCQ For More Study Material


Visit: studyiq.com
(a) 27.75% (b) 26.2% 10. The number of railroad passengers in 1992 when
(c) 24.18% (d) 22.52% compared to the number of railroad passengers in
5. Which of the following statements is true? 1987 were less by?
(a) 35 (b) 40
(a) U.P. was the top producer of all the three crops.
(c) 70 (d) 90
(b) M.P. ranked third in the production of all three
crops. Directions (11 – 15): These questions are based on
following pie graph.
(c) Maharashtra was the lowest producer of rice.
Spending Pattern of an Average Worker
(d) Haryana was the lowest producer of wheat.
Directions (6 – 10): Study the following graph carefully
and answer the questions that follow.
Passengers travelling by Various Modes from 1987 to 1992
(In Millions)

spending patterns of hard worker 100% = Rs. 3500


Spending Pattern of Mr. Hardworker
1987 1988 1989 1990 1991 1992

6. In 1989, bus passengers represented approximately


what percentage of all passengers by buses, railroads,
and airlines in that year?
(a) 35% (b) 45%
(c) 55% (d) 65%
7. From 1987 to 1992 (both inclusive), how many millions
of passengers approximately travelled by railroad?
(a) 1000 (b) 1300
(c) 1500 (d) 1700
8. If in 1992 the average bus fare per passenger was $
0.50 and if the average Airline fare per passenger was 11. For the categories which are common for both, for
$ 50. What is the ratio between total fares of Airline which category does Mr. Hard Worker spend more
passengers to total fare of bus passengers? than the average worker?
(a) Rent (b) Durables
6 30 (c) Semi-durables (d) Entertainment
(a) (b)
5 1 12. How much does Mr. Hard worker spends on
categories which are not included in the spending
60 50
(c) (d) pattern of an average worker?
1 1 (a) Rs. 100 (b) Rs. 225
9. In 1991, if 25 Airline companies was there then what (c) 525 (d) Rs. 450
was the average number of passenger was serviced 13. The additional saving of 5% of Mr. Hard worker come
by each Airline companies? from his savings under?
(a) 6 million (b) 4 million (a) Food (b) Repayment of loans
(c) 7 million (d) 8 million (c) Misc. (d) Data inadequate

340 @BEST300MCQ For More Study Material


Visit: studyiq.com
14. If the salary of Mr. Hard worker is increased by 20% (a) Petrol (b) House Rent
and his expenses under all the heads remain the (c) Food (d) Entertainment
same, what will be his percentage savings? Directions (21 – 25): Study the graph and answer the
(a) 10% (b) 15% following questions.
(c) 20% (d) 25% Power Supply Position in India (in Billion KWH)
15. Which of the following is definitely true?
I. Mr. Hard worker has a smaller family than the
average worker.
II. Mr. Hard worker has not borrowed any money.
III. Mr. Hard worker spends less on necessities and
more on comforts.
(a) I only (b) II only
(c) I, II and III only (d) Data inadaquate
Directions (16 – 20): The following questions are to be
answered on the basis of the pie-chart given below.
Person’s monthly salary distributed over different expense 21. What was the percentage increase in supply of power
heads between 1980-81 and 1984-85?
Entertainment (a) 56% (b) 145%
Petrol 15% (c) 43% (d) 85%
10% 22. The cumulative shortfall between requirement and
supply from 1980 – 81 to the end of 1985 was (in
billion Kwh)?
(a) 56 (b) 85
(c) 45 (d) 76
Savings 23. The requirement of power in 1984-85 was how many
23% times the availability of supply in 1978-79?
House Rent
30% (a) 2.6 (b) 1.75
(c) 2.75 (d) 2.0
Clothes 24. The percentage of growth in power requirement from
7% 1979-80 to 1984-85 was less than the percentage of
growth in power requirement from 1974-75 to 1979-
16. For a person, whose monthly salary is Rs. 6,000 p.m.,
80 by what figure?
how many items are there on which he has to spends
more than Rs. 1000 p.m.? (a) 3 (b) 4
(a) 1 (b) 2 (c) 15 (d) 7
(c) 3 (d) 4 25. Between 1979-80 and 1983-84, the power generation
17. For the same person, an expenditure of Rs. 1800 p.m. has generally lagged behind power demand by how
takes place on? many years?
(a) Petrol (b) House Rent (a) 1 (b) 2
(c) Food (d) Clothes (c) 3 (d) 4
18. The annual savings for such a person will be Directions (26 – 30): Following table shows the selling of
approximately? different Brand Mobile in different years (Data given in
thousands)?
(a) Rs. 5,000 (b) Rs. 10,000
(c) Rs. 15,000 (d) Rs. 16,560 Years ® 2006 2007 2008 2009 2010 2011
19. The monthly salary for a person who follows the same Brand ¯
expense pattern, but has a petrol expense for Rs. 500 M 21 21.5 23 27.6 28.2 30
p.m., is? N 18 21 20.4 22.1 24.5 24
(a) Rs. 2,500 (b) Rs. 3,000 O 30 32 27 28 30 28
(c) Rs. 5,000 (d) Rs. 6,500 P 16 12 18.6 18.4 19.8 19
20. The percentage of money spent on clothes and saving Q 26 21.5 25 24.5 28 30
is equal to which other single item of expense? R 15 12 18 26.5 28.5 28.2

341 @BEST300MCQ For More Study Material


Visit: studyiq.com
26. What is the difference between the average selling of 34. What was the total number of female working in Bank
mobile in year 2010 and that in year 2006? M in the year 1997, females working in Bank N in the
(a) 5300 (b) 5100 year 1995 and male working in Bank P in the year
(c) 5500 (d) 5400 1996 together?
27. Selling of Brand O in year 2008 is what percentage of (a) 1600 (b) 1550
selling of brand R mobile in year 2006? (c) 1450 (d) 1500
(a) 150% (b) 180% 35. Total number of female employees working in Bank
(c) 120% (d) 240% O over all the year together was approximately what
percentage of total number of employees working in
28. What is the approximate percentage increase in the year 1997 in all the Banks?
selling of Brand P mobile in year 2006 to 2009?
(a) 45 (b) 50
(a) 12% (b) 15%
(c) 55 (d) Cannot be determined
(c) 20% (d) 24%
Directions (36 – 40): Following Pie-chart show the
29. In the following years, in which year is total selling of distribution of different grade of employees of a Bank and
mobile of all Brand is maximum? percentage of male among them.
(a) 2007 (b) 2008 % of different grade employee (Total employee 8000)
(c) 2009 (d) 2011
30. Selling of Brand N mobile in year 2007 is how much
percentage more than the selling of Brand P mobile in
the same year? U P
20% 18%
(a) 75% (b) 60%
Q
(c) 50% (d) 25%
T 12%
Directions (31 – 35): Study the following table and answer 19%
the following questions: R
S 16%
BANK 15%
Year M N O P
M F M F % of male (Total male = 3200)
M F F M
1994 400 200 450 250 600 350 450 400
1995 150 250 100 400 350 550 600 550
1996 250 400 400 850 500 350 450 650
1997 400 650 150 500 500 650 600 700
1998 300 750 350 600 300 400 400 650
31. What was the ratio of the number of females, in Bank
P in the year 1995, the number of male in bank M is
the year 1996 and the number of female in Bank N in
the year 1995?
(a) 11 : 5 : 9 (b) 12 : 5 : 7 36. What is the ratio of female to male employee in Grade
(c) 11 : 6 : 8 (d) 11 : 5 : 8 U?
32. What was the average number of male employees (a) 14 : 15 (b) 15 : 16
working in all the Banks together in 1994? (c) 11 : 12 (d) 12 : 13
(a) 450 (b) 475 37. What is the half of difference between male and female
(c) 385 (d) 300 employee in Grade R?
33. If 20 percent of female and 30 percent of male (a) 288 (b) 128
employees in Bank N in year 1994 were handicapped, (c) 186 (d) 172
then what was the total number of handicaped 38. The number of female employees of Grade P is what
candidates in the Bank in that year? % of the number of male employees of Grade P?
(a) 185 (b) 170 (a) 80% (b) 92%
(c) 190 (d) 195 (c) 105% (d) 125%

342 @BEST300MCQ For More Study Material


Visit: studyiq.com
39. In which grade is the number of female employee the (a) 3 : 2 (b) 2 : 3
maximum? (c) 5 : 8 (d) Cannot determine
(a) Q (b) R 45. What is the total number of employees working in all
(c) S (d) T departments together?
40. The number of female in Grade S is how much % (a) 1300 (b) 1350
more than the number of male employee in the same (c) 1450 (d) 1400
grade? Directions (46 – 50): Study the following graph carefully
(a) 32% (b) 75% to answer the questions:
(c) 48% (d) 112.5% Quantity of Onion (in hundred tonnes) Purchased by three
Directions (41 – 45): Study the following table and answer Dealers over the years
the following questions:

Quantity of Onion (In hundred Tonnes)


No. of Employees in Different Departments Dealer Q Dealer P Dealer R

Department Males Female 2001 2002 2003 2004 2005 2006 2007
IT 2 3 Year
46. The percentage rise in purchase of onions from the
HR 7 8
previous year was the maximum during which year
Finance 11 4 for dealer Q?
Calling 12 13 (a) 2003 (b) 2004
(c) 2005 (d) 2007
Marketing 17 8
47. What is the average purchases of Onions by dealer Q
Production 5 9 for all the years? (in hundred tones rounded off to
41. What is the total number of males in all departments two digits after decimal)
together? (a) 571.42 (b) 667.14
(a) 829 (b) 709 (c) 766.67 (d) 756.57
(c) 614 (d) 689 48. Total Onions purchased by dealer P for all the years
42. What is the difference between total number of male is approximately what percent of the total onions
working in HR department and Finance department purchased by dealer Q for all the years?
together and female working in calling department (a) 128 (b) 139
and Production department together? (c) 68 (d) 82.5
(a) 225 (b) 215 49. What is the percent increase in the purchase of onions
(c) 420 (d) 205 of dealer R from 2003 to 2007?
43. In which department the lowest number of female (a) 150 (b) 133.33
working? (c) 33.33 (d) 16.66
(a) IT (b) HR 50. What is the ratio of total quantity of onions purchased
(c) Finance (d) Calling by all the three dealers in 2002 to that in 2004?
44. What is the respective ratio of male working in calling (a) 51 : 29 (b) 9 : 40
department to female working in HR department? (c) 29 : 51 (d) 29 : 42

343 @BEST300MCQ For More Study Material


Visit: studyiq.com
Directions (1 – 5): Study the following table carefully and 7. Public debt rose by _________ (in crores) from 1983
choose the correct options from the alternatives:- to 1984?
India’s foreign trade for the year 1980-81 to 1986-87 (All (a) 1000 (b) 1200
figures in Rs. Crores) (c) 1250 (d) 1350
8. Public debt rose by about 5.3% during the period?
Year (a) 1980 – 81 (b) 1982 – 83
1980-81 (c) 1983 – 84 (d) 1984 – 86
9. Increase in public debt was minimum during the
1981-82 period?
1982-83 (a) 1980 – 81 (b) 1981 – 83
(c) 1980 – 84 (d) 1984 – 86
1983-84
10. Percentage Increase in public debt was maximm
1984-85 during the period?
1985-86 (a) 1981 – 82 (b) 1982 – 83
(c) 1983 – 84 (d) 1985 – 86
1986-87 Directions (11 – 15): Study the graph and solve the
1. Which of the following showed an increase every following questions.
year? Sales/Net Profits of Company (in lakhs)
(a) Exprots (b) Imports
(c) Trade deficit (d) All of these
61.5 61.5 64
2. The ratio of imports to exports was maximum in the
year? 52
(a) 1980 – 81 (b) 1985 – 86
47.6
(c) 1985 – 86 (d) 1983 – 84 29.1
37.4
3. The percentage increase in exports was maximum in
the year? 14.3 15.6
(a) 1981 – 82 (b) 1986 – 87 15.4 5.2
(c) 1984 – 85 (d) 1983 – 84 -3.2 0
2.1 3.5
4. The total trade deficit (in Rs. crores) for the last five -4.2
years is? 1973 1974 1975 1976 1977 1978 1979 1980
(a) 28508 (b) 32332 11. In which year was the net profit as a percentage of the
(c) 44322 (d) 33232 sales revenue the greatest?
5. The difference between imports and exports was (a) 1973 (b) 1974
maximum in the year? (c) 1979 (d) 1980
(a) 1985 – 86 (b) 1986 – 87 12. Which was the year in which the sales revenue
(c) 1984 – 85 (d) 1983 – 84 registered the highest percentage growth compared
to that of the previous year?
Directions (6 – 10): Study the following graph carefully
(a) 1974 (b) 1976
and answer the questions that follow.
(c) 1978 (d) 1980
External Public Debt from 1980 to 1986 (in Rs. crore) 13. What was the cumulative net profit/loss of the first
five years for which particulars are shown?
(a) 2.2 lakhs (b) (–) 1.2 lakhs
(c) (–)1.8 lakhs (d) 3.2 lakhs
14. Which was the year in which the expenses of the
company were the highest?
(a) 1980 (b) 1979
(c) 1978 (d) 1976
6. During the years 1980-82, public debt rose by 15. If a tax of 55% was paid on gross profit before the net
approximately? profit was arrived at, what was the combined gross
profit for the years 1979 and 1980? (in Rs. lakhs)
(a) 6% (b) 8%
(a) 66.44 (b) 62.44
(c) 12% (d) 15%
(c) 46.44 (d) 38.44

344 @BEST300MCQ For More Study Material


Visit: studyiq.com
Directions (16 – 20): These questions are based on
following graph. The G - 15 Pop. 1990 (Millions) Real GNP per capita ($)
Share of world population Algeria 25 2,230
12.5% G-7 Argentina 2,767
32
nations
Brazil 143 2,540
Egypt 51 640
India 839 350
Indonesia 180 500
Jamaica 2.5 1,260
Malaysia 18 2,160
30% G-15
57.5% other 1,958
nations Mexico 85
nations
Nigeria 114 250
Peru 21.5 1,010
Senegal 7.5 850
Venezuela 19.5 2,450
Yugoslavia 24 2,920
Zimbabwe 10 650
The G - 7 Pop. 1990 (Millions) GNP per capita ($)
Canada 26 19,030 16. Which G-7 country has the highest GNP per capita?
France 56 17,820 (a) Japan (b) USA
Germany 62 20,440 (c) Canada (d) UK
Italy 58 15,120 17. The lowest GNP per capita of any G-15 nation is what
Japan 123 23,810 percent of the highest GNP per capita of any other G-
7 nation?
U.K. 58 14,610
(a) 1.7% (b) 5.3%
U.S.A. 250 20,910 (c) 4.3% (d) 1.05%
18. What is the total GNP of the other world countries if their average GNP per capita is 1140?
(a) $340 8600 (b) $348 9000 (c) $ 468 9600 (d) $3319425
19. Which of the following is false about the data given.
(a) Using information given in the above table we can conclude that total GNP of USA is greater than total GNP of
other nations put together
(b) Total GNP of Malaysia is more than total GNP of Egypt.
(c) Per capita GNP of only five G-15 nations is more than 15% of the per capita GNP of UK
(d) All of these
20. What is the average of GNP- per capita of G - 7 nations
(a) $18820 (b) $14820 (c) $17820 (d) $19820
Directions (21 – 25): Study the table carefully and answer the following questions:
Number of candidates appeared and passed in the examination from six state during the years

Cities Delhi Mumbai Chennai Kolkata Lucknow Kanpur


Year
A P A P A P A P A P A P
2006 1100 125 1500 115 1400 110 1350 120 1650 125 1700 130
2007 1600 155 1250 120 1450 115 1200 115 1600 120 1800 115
2008 1750 165 1100 110 1500 120 1200 160 1250 145 1350 125
2009 1650 130 1550 125 1550 125 1450 135 1450 115 1100 170
2010 1300 140 1200 140 1500 130 1300 140 1550 135 1250 155
2011 1450 145 1350 125 1400 135 1500 135 1500 140 1500 180

345 @BEST300MCQ For More Study Material


Visit: studyiq.com
21. In the given years what is the approximate of the
average numbers of candidates failed in the
examination from city Kolkata? Department Men Women
(a) 1100 (b) 1150 Production 1 4
(c) 1175 (d) 1200
HR 13 12
22. What is average candidate appeared in 2011?
(a) 1300 (b) 1450 IT 3 7
(c) 1500 (d) 2400 Marketing 2 3
23. What will be ratio of candidate fail in Delhi in 2006
Accounts 7 6
and candidate fail Mumbai in 2010?
(a) 195 : 212 (b) 212 : 195
26. What is the number of Women working in the
(c) 195 : 211 (d) 211 : 195
Marketing department?
24. What will be difference between candidate pass in
(a) 462 (b) 454
2009 from all cities and candidate pass in Lucknow
all over the years? (c) 418 (d) 429
(a) 40 (b) 50 27. What is the respective ratio of the number of Men
working in the HR department and the number of
(c) 30 (d) 20
Women working in the IT department?
25. Candidate fail in Chennai in 2010 is what percentage
(a) 11 : 12 (b) 17 : 29
of total number of candidate appeared in Delhi all
years together? (c) 13 : 28 (d) 12 : 35
(a) 25% (b) 30% 28. The number of Women working in the Production
department of the organization is what percent of the
(c) 40% (d) 15.48%
total number of employees working in that
Directions (26 – 30): Study the following pie charts carefully department?
to answer the questions
(a) 88 (b) 90
Degree-wise breakup of employees working in various
(c) 75 (d) 80
departments of an organization and the ratio of Men to
Women 29. The number of Men working in the IT department of
the organization is what percent of the total number
Total number of employees = 3250
of employees in the organization from all departments
together?
Production HR Dept. IT Dept. Marketing Accounts
Dept. Dept. Dept. (a) 3.2 (b) 4.8
136.8 36° 57.6° 79.2° 50.4° (c) 6.3 (d) 5.6
30. What is the total number of women working in the
organization?
Accounts (a) 2198 (b) 2147
Dept. (c) 2073 (d) 2236
(50.4°) Directions (31 – 35): Study the table carefully to answer
Production the questions that follow:
Marketing Dept. Percentage of Marks obtained by Different students in
Dept. (136.8°) Different subjects
(79.2°)
Hindi Eng. Maths S.St. Sci. Marathi
HR Students (100) (50) (150) (125) (150) (50)
IT Dept. Dept. Aditya 98 88 84 88 64 68
(57.6°) (36°)
Ram 77 92 92 88 84 80
Sanjay 81 80 84 80 78 90
Dep. = Department Girish 90 86 76 64 66 56
Respective Ratio of Men to Women in Each Mahesh 69 66 66 76 72 72
Department
Anil 89 90 70 92 60 66

346 @BEST300MCQ For More Study Material


Visit: studyiq.com
31. How much marks did Aditya get in all the subjects (a) None (b) one
together? (c) two (d) three
(a) 508 (b) 477 34. What is Sanjay’s overall percentage of marks in all
(c) 454 (d) 537 subjects together?
32. What is the average marks obtained by all students (a) 85.92 (b) 72.64
together in Marathi? (c) 81.44 (d) 76.32
(a) 72 (b) 48 35. Who has scored the highest marks in all the subjects
(c) 28 (d) 36 together?
33. How many students have scored the highest marks (a) Aditya (b) Ram
in more than one subject? (c) Mahesh (d) Anil
Directions (36 – 40): Study the following graph carefully to answer the question that follow:

Number of Students Studying Different


Subject in a College Over the Years
800

700
No. Of students

600

500

400

300

200

100

2006 2007 2008 2009 2010


History 500 250 700 250 650
Geography 600 150 350 350 450
Math 350 700 750 500 600

36. What will be the ratio of student studying History in 39. The number of student who study History in 2010 is
2007, 2009 and 2006 together to those who study Math approximately what percent of all the student in 2007?
in 2007 and 2010 together? (a) 60 (b) 65
(a) 5 : 6 (b) 10 : 13 (c) 68 (d) 59
(c) 13 : 10 (d) 6 : 5 40. In which of the year no. of student in all subject together
37. What will be ratio of No. of students studying History, is lowest?
Geography and math all the year together? (a) 2007 (b) 2009
(a) 46 : 38 : 59 (b) 47 : 37 : 57 (c) 2008 (d) both a and b
(c) 48 : 38 : 57 (d) 47 : 38 : 58 Directions (41 – 45): Study the following Pie-Chart
carefully and answer the questions given below:
38. What will be difference between total number of
student of Math & History all the year together except A survey conducted on 2900 Villagers Staying in Various
2007? Villages and having Various Favourite Sports
(a) 200 (b) 150
(c) 100 (d) 125

347 @BEST300MCQ For More Study Material


Visit: studyiq.com
Favourite Game People Staying in Various Villages

Ba 12

14 ball
dm %
S

y
P

%
lle
in
27%

to

Vo
Football 21%

n
15% Tennis
11%
R Q
Cricket Hockey 30% 22%
20% 28%

41. What is the total number of people having their favorite (a) 1440 (b) 1540
game Hockey and Badminton together? (c) 1640 (d) 348
(a) 1200 (b) 812 44. If the favorite game of 40% people of village R is Hockey
(c) 388 (d) 1160 then what will be the Ratio of people who like Hockey
42. How many people having favourite game Cricket? expect village R and total no. of people in village P?
(a) 580 (b) 420 (a) 16 : 21 (b) 21 : 16
(c) 680 (d) 480 (c) 21 : 31 (d) 31 : 21
43. If 40% of the people from village R have cricket as 45. 10% of the people from village R have favorite game
favourite game and 20% of the people have Tennis as Tennis then what is the no. of other villagers who like
favourite game. Then how many people like other Tennis:
game in village R? (a) 87 (b) 58
(c) 29 (d) 232
Directions (46 – 50): Study the following table carefully to answer the questions that follow:
Number (N) of Six Type of Electronic Products Sold by Six different stores in a month and the price per product (P) (price
in Rs. ‘1000’) charged by each store.

Store A B C D E F
Product N P N P N P N P N P N P
L 54 135 48 112 60 104 61 124 40 136 48 126
M 71 4.5 53 3.8 57 5.6 49 4.9 57 5.5 45 4.7
N 48 12 47 18 52 15 54 11.5 62 10.5 56 11
O 52 53 55 48 48 50 54 49 59 47 58 51
P 60 75 61 68 56 92 44 84 46 76 59 78
Q 43 16 44 15 45 14.5 48 15.6 55 18.2 55 14.9
46. Number of L type products sold by store F is what (a) Rs. 14,700 (b) Rs. 15,700
percent of the number of the same type of products (c) Rs. 15,200 (d) Rs. 14,800
sold by store E? 49. What is the difference in the amount earned by store
(a) 76.33 (b) 124 A through the sale of P type products and that earned
(c) 83.33 (d) 120 by store B through the sale of the Q type of products?
47. What is the respective ratio of total number of N and (a) 38.4 lakhs (b) 0.384 lakhs
L type of products sold by store D and the same (c) 3.84 lakhs (d) 384 lakhs
products sold by the store A? 50. What is the difference between number of all type of
(a) 119 : 104 (b) 102 : 115 product sold by store D and total number of O type
(c) 104 : 115 (d) 115 : 102 product sold by all the store together?
48. What is the average price per product charged by all (a) 116 (b) 26
the stores together for product Q? (c) 36 (d) 16

348 @BEST300MCQ For More Study Material


Visit: studyiq.com
Directions (1 – 5): Study the following graph carefully to Total number of employees = 4200
answer these questions:

Maintenance
Percent profit earned by selling Wheat and Sugar over the

Dept 10%
years HR Dept
18%
Profit
%Pr ofit = ´ 100
Expenditure IT Dept Accounts
25% Dept 12%
Sugar × Wheat
Marketing
50
Production Dept 14%
45 Dept 21%
40
35
Out of which the number of employees promoted
Percent profit

30

Ma ept 46
25 ×

inte
D
20 × ×

nan
HR Dept
15 × × 82

ce
10 × IT Dept
78 Accounts
5
M Dept 24
a
0 1999 2000 2001 2002 2003 2004 De rke
t
Year Production pt 3 ing
Dept 48 6
1. If the expenditure on production of sugar and wheat
in 2002 was equal, what was the respective ratio of
income through sales of these products? 6. What is the number of employees working in the IT
(a) 6 : 5 (b) 7 : 8 department?
(c) 7 : 6 (d) 2 : 1 (a) 1045 (b) 1150
2. The ratio of the amount of profit earned through sale (c) 1140 (d) None of these
of sugar in 2000 to that in 2001 was 7 : 6. What was 7. The number of employees who got promoted from
the respective ratio of the expenditures? the Marketing department is what percent of the total
(a) 2 : 3 (b) 3 : 4 number of employees in that department? (rounded
(c) 4 : 3 (d) Cannot be determined off to the nearest integer)
3. Income through sales of wheat in 2004 was Rs. 16.8 (a) 12 (b) 10
lakhs. What was the expenditure on production? (c) 3 (d) 6
(a) Rs. 13.8 lakhs (b) Rs. 14.2 lakhs 8. What is the ratio of the number of employees not
(c) Rs. 13.6 lakhs (d) None of these promoted from HR department to that from
4. If the total expenditure on production of wheat in Production department?
2001 was Rs. 13 lakhs, what was the amount of profit (a) 216 : 389 (b) 337 : 417
earned? (c) 389 : 216 (d) 417 : 337
(a) Rs. 32,500 (b) Rs. 3,25,000 9. What is the percentage of employees promoted from
(c) Rs. 2,65,000 (d) Rs. Rs. 26,500 the Maintenance and Accounts departments together
5. If the total income through sales of wheat and sugar with respect to the total number of employees from
in 1999 was Rs. 45 lakhs, what was the total these two departments? (rounded off to two digits
expenditure on these two products? after decimal)
(a) Rs. 32 lakhs (b) Rs. 35 lakhs (a) 7.58 (b) 6.27
(c) Rs. 37.5 lakhs (d) Cannot be determined (c) 9.15 (d) 8.34
Directions (6 – 10): Study the pie-charts carefully to answer 10. Which department has the highest percentage of
the questions that follow: Percentage of employees in promoted employees with respect to the total number
different departments of our organisation out of which of employees of that department?
the number of employees promoted are given in two pie (a) IT (b) Marketing
chart respectively (c) Maintenance (d) HR

349 @BEST300MCQ For More Study Material


Visit: studyiq.com
Directions (11-15): Study the table carefully to answer the (a) Ravi (b) Suresh
questions that follow: (c) Satish (d) Arun
Per annum income of six men and the percentage break- 13. The amount spent by Suresh on health is what
up of their spending pattern percent of the total amount spent by all the men
Per Annum Percentage break-up of spending pattern together on health? (rounded off to two digits after
Man Income
(Rs. in lakhs) Eating Shopping Watching Health Savings Miscella- decimal)
out Movies neous
Ravi 2.05 21.8 14.6 20.4 16.4 14.5 12.3
(a) 16.24 (b) 14.60
(c) 18.04 (d) 12.35
Satish 2.25 20.9 15.7 15.6 11.4 15.3 21.1
14. The per annum income of Arif is approximately what
Arun 1.95 14.3 16.6 18.5 21.5 16.4 12.7
percent of the total per annum income of all the men
Vilas 2.00 18.6 14.5 18.7 21.5 16.2 10.5 together?
Arif 1.75 17.8 12.8 19.5 21.0 18.2 10.7 (a) 22 (b) 27
Suresh 1.70 12.0 18.6 10.5 18.2 22.4 18.3 (c) 19 (d) 15
11. What is the total amount of savings of all the men 15. What is the ratio of the total amount spent by Ravi on
together? eating out and watching movies together to the total
(a) Rs. 1,98,460 (b) Rs. 1,42,524 amount spent by Arun on the same?
(c) Rs. 1,89,520 (d) Rs. 1,56,625 (a) 156 : 211 (b) 217 : 253
12. Who spends the maximum amount on shopping? (c) 253 : 217 (d) 211 : 156
Directions (16 – 20): Study the following pie-chart and table carefully to answer the questions that follow:
Total cars = 700
Distribution of cars

State - 1
14%
State - 4
26%

State - 2
28%
State - 3
32%

Table showing the ratio of diesel to petrol engine cars which are distributed among four different states

State Diesel Engine Cars Petrol Engine Cars

State - 1 3 4

State - 2 5 9

State - 3 5 3

State - 4 1 1

16. What is the difference between the number of diesel diesel engine cars in State - 3 which are non-AC?
engine cars in State - 2 and the number of petrol (a) 75 (b) 45
engine cars in State-4? (c) 95 (d) 105
(a) 159 (b) 21 19. What is the difference between the total number of
(c) 28 (d) 34 cars in State - 3 and the number of petrol engine cars
17. The number of petrol engine cars in State - 3 is what in State - 2?
percent more than the number of diesel engine cars (a) 106 (b) 112
in State - 1? (c) 102 (d) 98
(a) 100 (b) 200 20. What is the average number of petrol engine cars in
(c) 300 (d) 125 all the states together?
18. If 25% of diesel engine cars in State - 3 are AC and the (a) 86.75 (b) 89.25
remaining cars are non-AC, what is the number of (c) 89.75 (d) 86.25

350 @BEST300MCQ For More Study Material


Visit: studyiq.com
Directions (21 – 25): Study the following bar-diagram and table carefully, and answer the questions given below:
The bar-graph shows the number of recruitments for various posts (Income Tax Inspector, Custom Inspector and
Excise Inspector) in different years
900 Income tax Custom Excise
800
700
600
500
400
300
200
100
0
2008 2009 2010 2011 2012
Year
The table shows the percentage of various categories (General, OBC, SC and ST) for different posts in different years
Year Income Tax Inspector Custom Inspector Excise Inspector
Gen OBC SC ST Gen OBC SC ST Gen OBC SC ST
2008 54 28 12 6 52 25 16 7 50 36 10 4
2009 42 36 14 8 48 30 15 7 46 36 14 4
2010 51 30 15 4 46 38 10 6 52 26 16 6
2011 60 22 12 6 58 23 14 5 54 28 12 6
2012 55 24 16 5 55 27 12 6 60 18 14 8
Note: Gen Þ General
(a) 7 : 52 (b) 14 : 23
21. In which of the following years the number of SC
(c) 7 : 36 (d) 52 : 7
candidates selected for the post of Income Tax
24. The number of OBC candidates selected during 2008-
Inspector is the maximum?
12 for the post of Income Tax Inspector is what percent
(a) 2010 (b) 2008
of the General candidates selected during 2008-12
(c) 2009 (d) Both 2011 and 2012
for the post of Custom Inspector?
22. In which of the following years the number of OBC
(a) 46.49% (b) 43.41%
candidates selected for Excise Inspector is the
(c) 39.51% (d) 37.61%
minimum?
25. What is the ratio of Income tax Inspectors from SC
(a) 2012 (b) 2010
Category in 2009, Custom Inspectors from OBC
(c) 2008 (d) 2011
Category in 2010 and Excise Inspectors from ST
23. What is the ratio of General candidates to ST
Category in 2011?
candidates selected for the post of Custom Inspector
(a) 34 : 113 : 39 (b) 35 : 133 : 36
in 2008?
(c) 32 : 131 : 37 (d) 31 : 117 : 47
Directions (26 – 30): These questions are based on the following table. Study it carefully and answer the questions:
Number of Items (in thousands) manufactured (M), rejected (R) and sold (S) by five different companies over the years
Company A B C D E
Year M R S M R S M R S M R S M R S
2001 136 1.2 125 98 0.5 90 165 3.5 158 158 1.5 149 85 0.6 80
2002 164 1.7 138 115 1.1 102 172 2.9 166 169 1.9 162 96 0.8 90
2003 148 1.5 136 152 2.6 132 169 2.3 160 173 2.3 168 88 0.5 83
2004 156 2.2 145 147 1.8 140 178 3.2 172 166 2.1 159 102 0.9 98
2005 168 2.5 160 138 1.3 129 158 1.8 152 159 2.0 150 86 0.7 81
2006 175 2.8 168 168 2.2 148 180 2.4 171 171 2.4 165 105 0.8 101
Note: Number of items accepted = Number of items manufactured – Number of items rejected

351 @BEST300MCQ For More Study Material


Visit: studyiq.com
26. What is the percentage (rounded off to two digits (a) 707600 (b) 77600
after decimal) of items rejected out of the total items (c) 70760 (d) 776000
manufactured by Company B in the year 2003? 29. Approximately, what was the average number of
(a) 1.97 (b) 1.71 items rejected by Company D for all the given years?
(c) 1.82 (d) 1.96 (a) 2060 (b) 2090
27. How many items remained unsold out of the accepted (c) 1990 (d) 2030
items by Company A in 2004? 30. What was the total number of items manufactured
(a) 800 (b) 880 by all the companies together in 2006?
(c) 8000 (d) 8800 (a) 582000 (b) 827000
28. What is the total number of items accepted by all the (c) 799000 (d) 595000
five companies together in 2002?
Directions (31 – 35): Study the following graph carefully to answer the questions given below it: Number of articles
manufactured (in lakhs) by two Companies P and Q and percentage of articles sold by both over the years
Articles manufactured by P
× Articles manufactured by Q
Percentage sold by P
Percentage sold by Q
40 80
Number of Articles (in lakhs)

35 × 70
× ×
30 × × × 60

Percentage sold
25 50
20 40
15 30
10 20
5 10
0 0
2002 2003 2004 2005 2006 2007
Years
31. How many articles were sold by both companies (a) 136.85 (b) 152.78
together in 2005? (c) 35 (d) 40
(a) 14,62,000 (b) 42,18,000 34. What is the ratio of number of the articles not sold by
(c) 31,25,000 (d) 29,60,000 Company P to that of those by Company Q in the
32. What is the approximate average number of articles year 2002?
manufactured by Company P over the years? (a) 5 : 7 (b) 9 : 13
(a) 29 (b) 38,14,886 (c) 7 : 5 (d) 13 : 8
(c) 38 (d) 29,16,667 35. In which year was the maximum number of articles
33. What is the percent increase in the number of articles manufactured from both Companies together?
sold by Company Q in 2004 from the year 2002? (a) 2005 (b) 2005 and 2007
(rounded off to two digits after decimal) (c) 2007 (d) None of these
Directions (36 – 40): Study the following tables carefully and answer the questions given below: Number of candidates
appeared in a competitive examination from five centres over the years

Centre Mumbai Delhi Kolkata Hyderabad Chennai


Year
2001 35145 65139 45192 51124 37346
2002 17264 58248 52314 50248 48932
2003 24800 63309 56469 52368 51406
2004 28316 70316 71253 54196 52315
2005 36503 69294 69632 58360 55492
2006 29129 59216 64178 48230 57365
2007 32438 61345 56304 49178 58492

352 @BEST300MCQ For More Study Material


Visit: studyiq.com
Approximate percentages of candidates qualified to appeared in the competitive examination from five centres over
the years
Centre Mumbai Delhi Kolkata Hyderabad Chennai
Year

2001 12 24 18 17 9
2002 10 28 12 21 12
2003 15 21 23 25 10
2004 11 27 19 24 8
2005 13 23 16 23 13
2006 14 20 21 19 11
2007 16 19 24 20 14

36. In which of the following years was the difference in (a) 27250 (b) 25230
number of candidates appeared from Mumbai over (c) 30150 (d) 28150
the previous year is minimum?
39. Approximately how many candidates appearing
(a) 2004 (b) 2006 from Kolkata in 2004 qualified in the competitive
(c) 2007 (d) 2002 examination?
37. In which of the following years was the number of (a) 13230 (b) 13540
candidates qualified from Chennai is maximum (c) 15130 (d) 15400
among the given years?
40. Approximately what was the difference between the
(a) 2007 (b) 2006 number of candidates qualified from Hyderabad in
(c) 2005 (d) 2003 2001 and that in 2002?
38. Approximately what was the total number of (a) 2440 (b) 1450
candidates qualified from Delhi in 2002 and 2006 (c) 2060 (d) 1860
together?

Directions (1 – 5): The piechart, given here, shows the 2. If the miscellaneous charges are Rs. 1500, then the
expenditure incurred by a publisher in bringing out a advertisement charges are:
book. (a) Rs. 3750 (b) Rs. 4500
(c) Rs. 5250 (d) Rs. 5400
3. The ratio of the cost of Printing to Royalty is:
Cost of (a) 3 : 7 (b) 7 : 3
printing
35% (c) 7 : 1 (d) 1 : 7
Cost of
paper 4. If the cost of printing is Rs. 10,500, then the cost of
Binding 15% s paper is:
se
charges 5% pen (a) Rs. 4500 (b) Rs. 5400
t
18 ge en

12%
x
ar em

lty

.E
(c) Rs. 6000 (d) Rs. 6250
%

isc
ch rtis

% s

Roya
5

M
1
e

5. The central angle of the sector for the binding charges


dv
A

is:
(a) 64.8° (b) 54°
Study the pie-chart and answer the questions based on it.
(c) 43.2° (d) 36°
1. If 5000 copies are published, miscellaneous expenses
Direction (6– 10): The pie chart given below, shows the
amount to Rs. 1500 and the publisher wants a profit
expenditure on various items and savings of a family
of 25%, then the marked price of a copy of the book
during the year 2009. Study the pie chart and answer
should be:
these questions.
(a) Rs. 7.50 (b) Rs. 10
PERCENTAGE OF MONEY SPENT ON VARIOUS
(c) Rs. 12.50 (d) Rs. 15 ITEMS AND SAVINGS BY A FAMILY DURING 2009

353 @BEST300MCQ For More Study Material


Visit: studyiq.com
(a) 18 (b) 12

Ed 5% ort
(c) 15 (d) 20

sp
Others

12 ldr n
12. The combination of three crops which contribute to

Tran

o
20%

% en
ch ati
more than 50% of the total area under the food crops

of uc
i
is:
Savings
Housing (a) wheat, rice and maize
15%
15%
(b) wheat, rice and jowar
Clothing (c) wheat, rice and bajra
Food 23% 10%
(d) rice, barley and maize
13. The ratio of the land used for rice and barley is:
6. If the total expenditure of the family for the year 2009 (a) 3 : 1 (b) 1 : 2
was Rs. 1,50,000 then the difference between the (c) 2 : 1 (d) 3 : 2
expenditures on housing and transport was: 14. If 10% of the land reserved for rice be distributed to
(a) Rs. 15,000 (b) Rs. 10,000 wheat and barley in the ratio 2 : 1, then the angle
(c) Rs. 12,000 (d) Rs. 7,500 corresponding to wheat in the new pie-chart will be:
7. Maximum expenditure of the family other than on (a) 38.4° (b) 76.8°
food, was on: (c) 75.6° (d) 45.5°
(a) Housing (b) Clothing 15. If the production of rice is 5 times that of jowar and
(c) Others (d) Education of children the production of jowar is 2 times that of bajra, then
8. The savings of the family for the given year were the ratio between the yield per acre of rice and bajra
equal to the expenditure on: is:
(a) Food (b) Housing (a) 5 : 2 (b) 3 : 1
(c) Education of children (c) 4 : 1 (d) 6 : 1
(d) Clothing Directions (16 – 20): The bar chart given below shows the
9. The percentage of the expenditure which was spent percentage distribution of the production of various
on clothing, education of children and transport models of a mobile manufacturing company in 2007 and
together is: 2008. The total production in 2007 was 35 lakh mobile
phones and in 2008 the production was 44 lakh. Study
(a) 17 (b) 20
the chart and answer the following question.
(c) 22 (d) 27
10. If the total expenditure of the family was Rs. 1,50,000 100 F F
then the money spent on food was: E A
(a) Rs. 20,000 (b) Rs. 23,000 90 E
D B
(c) Rs. 30,000 (d) Rs. 34,500 80 D
Directions (11 – 15): The piechart provided below gives C
C
the distribution of land (in a village) under various food 70
crops. Study the pie-chart carefully and answer the C
60 B D
questions from 11 to 15.
50 E
B
Wheat 40 A F
Rice 30 A
Barley 72° 72°
36°
20
18°
Jowar 99°
18°
45° 10
Bajra
Others
Maize 0
2007 2008
16. Total number of mobiles of models A, B and E
11. If the total area under bajra was three hundred acres, manufactured in 2007 was:
then the total area (in hundred) acres under rice and (a) 2450000 (b) 2275000
barley together is: (c) 2100000 (d) 1925000

354 @BEST300MCQ For More Study Material


Visit: studyiq.com
17. For which models was the percentage variation in Directions (26 – 30): The following bar diagram analyses
production from 2007 to 2008 the maximum? the sale of a company from 2000 to 2005, Examine the
(a) B and C (b) C and D diagram and answer the questions:
(c) D and E (d) A and B 450
18. What was the difference in the number of B type 440
430
mobiles produced in 2007 and 2008? 420
(a) 355000 (b) 27000 410
(c) 225000 (d) 175000 400
19. If the percentage production of A type mobiles in 2008 390
380
was same as that in 2007, then the number of A type

Sale
370
mobiles produced in 2008 would have been: 360
(a) 1400000 (b) 1320000 350
(c) 1170000 (d) 1050000 340
20. If 85% of the D type mobiles produced in each year 330
320
were sold by the company, how many D type mobiles
310
remained unsold? 300
(a) 76500 (b) 93500
2000 2001 2002 2003 2004 2005
(c) 118500 (d) 144750
Year
Directions (21 – 25): The following bar chart shows the 26. The sales in 2004 are what percentage of those in
sales of a company XYZ (in Rs. crore). Study the chart 2005?
and answer the following questions: (a) 40 (b) 4
10000 (c) 110 (d) 1.1
8730

27. In which year did the sales show that least decrease
to that of the preceding year?
8000 (a) 2004 (b) 2001
(c) 2003 (d) 2005
7786

28. By what amount are the sales in 2003 more that those
Sale (in Rs. crore)

7107

6000 in 2001?
(a) Rs. 100 (b) Rs. 10000
5345
4106

(c) Rs. 100000 (d) Rs. 1000000


4000
2953

29. The sales in 2001 are how many times those of 2002?
2053

(a) 8 times (b) 0.8 times


1841

(c) 2.5 times (d) 3 times


1773

2000
1115

30. In which year did the sales show the least per cent
924

increase to that of the preceding year?


(a) 2000 (b) 2002
1 2 4 5 6 7 8 9 10 11
3 (c) 2003 (d) 2004
Years
21. Total sales in 2nd and 3rd yr together is: Directions (31 – 35): Read the bar graph given below and
(a) Rs. 2688 crore (b) Rs. 2888 crore answer the questions:
(c) Rs. 2788 crore (d) Rs. 2488 crore
20
22. The 2nd highest sale is in the year
18 Bar Graph
Wheat

(a) 10 (b) 9
Production (in million tonnes)

(c) 8 (d) 7 16
23. The 2nd least sale is in year 14
(a) 2 (b) 3 12
(c) 6 (d) 4
10
Rice

24. The mean of the highest and the lowest sale (in Rs.
Rice

Wheat

8
crore) is:
Wheat

(a) 4922.5 (b) 4827 6


Rice

Wheat
Rice

(c) 4365 (d) 4922


Wheat

4
Rice

25. The sale in the 4th yr is less than the sale in the 8th yr, 2
by:
(a) Rs. 3608 crore (b) Rs. 3504 crore 0 Mahar- Haryana
WB
UP

MP

(c) Rs. 3127 crore (d) Rs. 3427 crore ashtra


States

355 @BEST300MCQ For More Study Material


Visit: studyiq.com
31. Which of the above states is least producer of wheat? (a) 24 : 13 (b) 48 : 17
(a) Maharashtra (b) WB (c) 12 : 7 (d) 6 : 5
(c) MP (d) Haryana 39. Percentage increase in profit in 1986 over 1982 is:
32. Which of the above states is the largest producer of (a) 150% (b) 120%
rice? (c) 100% (d) 80%
(a) UP (b) WB 40. The total income exceeds the total expenditure over
(c) MP (d) Haryana the years 1982 to 1986 by:
33. What fraction of rice is produced by haryana of the (a) 85 lakh (b) 105 lakh
total production of rice by all the above states? (c) 115 lakh (d) 120 lakh
1 1 Directions (41 – 44): Given, here is a multiple bar diagram
(a) (b) depicting the changes in the strength of a college in four
8 12
departments from 1990 – 91 to 1992 – 93. Study the
1 1 diagram and answer the questions based on it.
(c) (d)
4 6 Arts Science Commerce Law
34. In which of the above states the total production of 600
rice and wheat is the least?
550
(a) WB (b) MP
500
(c) Maharashtra (d) Haryana
450
35. Which of the above states is the largest producer of
Student strength
400
wheat?
350
(a) MP (b) Haryana
300
(c) Maharashtra (d) UP 250
Directions (36 – 40): Read the graph and answer the 200
following questions: profit = Income – expenditure
150
100
50
0
1990–91 1991–92 1992–93
Years
41. In which department was there a regular decrease in
students strength?
(a) Arts (b) Science
(c) Commerce (d) Law
42. The percentage of students in Science department in
1990 – 91 was:
(a) 26.9 (b) 27.8
(c) 29.6 (d) 30.2
43. The total students strength in 1991 – 92 was how
many times that of commerce students in the same
year?
(a) 3 (b) 4
36. What is the difference in profit between 1983 and
1984 (in Rs. lakh rupees)? (c) 5 (d) 6
(a) No profit (b) 5 44. What was the percent increase in Science students
from the year, 1990 – 91 to 1992 – 93?
(c) 10 (d) 15
37. The number of years in which the income is more 2
than the average income of the given years is: (a) 50 (b) 66
3
(a) One (b) Two (c) 75 (d) 150
(c) Three (d) Four Directions (45 – 48): The following table shows production
38. The ratio of the average income of all the years to the fo cars (in thousands) in a company from 1999 to 2004.
average profit is: Study, the table and answer the questions.

356 @BEST300MCQ For More Study Material


Visit: studyiq.com
(a) 94.24 (b) 95.45
1999 2000 2001 2002 2003 2004 (c) 82.45 (d) 104.76
P 8 20 16 17 21 6 52. What is the ratio of number of students enrolled in
Q 16 10 14 12 12 14 Scout and Red cross activites together to those
R 21 17 16 15 13 8 enrolled in Debating club activities?
S 4 6 10 16 20 31 (a) 3 : 1 (b) 4 : 1
T 25 18 19 30 14 27 (c) 1 : 4 (d) 1 : 3
Total 74 71 75 90 80 86 53. Which two clubs have the enrolment in the ratio of
2:1?
45. In which year, the total production of cars of types P
and Q together was equal to the total production of (a) Eco club, HRD club (b) Eco club, NCC
cars of types R and S together? (c) HRD club, Eco club (d) Debating club, Eco club
(a) 2000 (b) 2001 Directions (54 – 56): The pie charts, given here show some
(c) 2003 (d) 2004 automobile parts manufactured by an automobile
46. In which year, the production of all types of cars taken company at its Pune and Nagpur plants in the year 2009.
together was approximately equal to the annual Study the pie charts and answer questions. The number
average of the total production during the period 1999 of total auto parts produced at both plants are same.
to 2004? Pune Plant
(a) 1999 (b) 2001
(c) 2003 (d) 2004 Tyres
47. The production of which type of cars in any year was 30%
25% of the total production of all types of cars the
year 2003? Lights
(a) P (b) Q Tubes 20%
(c) R (d) S 28%
48. There was a continuous increase in production of Gear box

7% n
which type of cars during the period 1999 to 2004? 15%

r
Ho
(a) P (b) Q
(c) R (d) S Nagpur Plant
Directions (49 – 53): The pie chart given below shows the
number of students enrolled in a school in different
activities. Total number of students in the school is 1200. Tubes
Study the chart and answer the questions. 25%

Tyres
Gear box 32%
Ecob club Scout 14%
22% 18%
Ho

Lights
rn

26%
3%

Debating
Red cross
club
21%
13%
HRD 54. If the Nagpur plant produced 800000 tyres, then the
Club NCC number of horns produced by it was:
11% 15%
(a) 12000 (b) 18500
(c) 75000 (d) 60000
49. How many students are enrolled in NCC activities? 55. How many percent more tubes were produced at the
(a) 180 (b) 120 Pune plant than those Nagpur plant?
(c) 72 (d) 240 (a) 14% (b) 12%
50. What is the total number of students enrolled in (c) 8% (d) 3%
Debating club and HRD Club? 56. The ratio of number of horns produced at Nagpur
(a) 144 (b) 216 plant to that produced at Pune plant is:
(c) 288 (d) 72 (a) 3 : 7 (b) 10 : 3
51. The number of students enrolled in Eco club is what (c) 7 : 3 (d) 7 : 10
percent of those enrolled in Red cross activities?

357 @BEST300MCQ For More Study Material


Visit: studyiq.com
S olut ions 15. (d); In the question, Information is not given.
æ 1000 ö
16. (b); 1000 Rs. is ç ´ 100 = 16.66% ÷ of 6000.
1. (b); Total rice production = 58.64 è 6000 ø
W. B production = 8.09
\ Required No of items
8.09 = House rent (1) + saving (1) = 2
\Required percentage = ´ 100
58.64 æ 1800 ö
= 13.8 » 13. 5% 17. (b); 1800 is ç ´ 100 = 30% ÷ of 6000
è 6000 ø
2. (b); Required amount
Which is equal to expense on house rent
= (15.97) – (0.81 + 3.72 + 0.86 + 4.42)
= 15.97 – 9.81 = 6.16 m tonnes 23
18. (d); Annual saving = 12 ´ ´ 6000
3. (b); Wheat production was produced by states other 100
then those given in the table = 12 × 23 × 60 = 720 × 23 = 16560
= (44.23) – (15.97 + 0.81 + 3.72 + 0.86 + 4.42) 19. (c); Petrol expense = 500
= (44.23) – (25.78) = 18.45 Let the monthly, salary be x Rs.
18.45 10
\Required % = ´ 100 = 41.71 » 42% ´ x = 500 Þ x = 5000
44.23 100
7.18 + 15.97 + 2.76 20. (b); Total percentage of money spent on clothes and
4. (d); Required percentage = ´ 100 saving = 7 + 23 = 30 %
58.64 + 44.23 + 12.2
Which is equal to house-rent
25.91
= ´ 100 = 22.52% 151 - 105 46
115.07 21. (c); % increase = ´ 100 = ´ 100 » 43%
105 105
5. (a); Clearly we can sea that in all of the states U.P
22. (d); Total commulative shortfall
was the top producer of all the three crops
= 15 + 14 + 13 + 15 + 19 = 76
375
6. (b); Required percentage = ´ 100 170
( 375 + 300 + 175 ) 23. (b); Required number = = 1.75
97
375 24. (d); % growth in power requirment
= ´ 100 » 45%
850 170 - 118
From 1979 – 80 to 1984 – 85 = ´ 100
7. (d); Passengers travelled by railroad 118
= 300 + 300 + 300 + 275 + 260 + 265 = 1700
52
= ´ 100 = 44%
200 ´ 50 = 10000 50 118
8. (d); Required ratio = =
400 ´ 0.5 200 1 % growth in power requirment from
175 118 - 78
9. (c); Average = million = 7million 1974 – 75 to 1979 – 80 = ´ 100
25 78
10. (a); Required units = (300 – 265) = 35
11. (c); From both pie-charts we can say that in semi- 40
= ´ 100 = 51.2%
durables hard-workers spend more then average 78
workers. \ % growth = (51.2 – 44) = 7% (Approx.)
12. (c); There are two such items - durables, Entertainmnet 25. (c); In 1979 – 80 demand was 118 which completed
= (8 + 7) % of 3500 = 15% of 3500 = 525 in 1982 – 83 means 3 years.
13. (d); We can’t conclude it. Because clearly it is not 159
mentioned any where in the question. 26. (c); Average selling in 2010 = ´ 1000 = 26500
6
14. (d); New salary = 120% of 3500 = 4200
New saving amount = 700 + 10% of 3500 = 1050 126
Average selling in 2006 = ´ 1000 = 21000
1050 6
\ Saving percentage = ´ 100 = 25%
4200 Difference = 26500 – 21000 = 5.5 thousands

358 @BEST300MCQ For More Study Material


Visit: studyiq.com
27 1
27. (b); Required % = ´ 100 = 180% \ Required half of difference = ( 928 - 352 )
15 2

2.4 1
28. (b); Required percentage = ´ 100 = 15% = ´ 576 = 288
16 2
29. (d); 2006 ® 126 20
38. (d); Male employee of grade , P = ´ 3200 = 640
2007 ® 120 100
2008 ® 132
18
2009 ® 147.1 Total employee of grade P = ´ 8000 = 1440
100
2010 ® 159.0
\ Female employee in grade P = 1440 – 640
2011 ® 159.2
= 800
Clearly in year 2011, the sale of mobiles is
maximum 800
\ Recquired % = ´ 100 = 125%
640
21 - 12 9
30. (a); Required percentage = ´ 100 = ´ 100
18
12 12 39. (d); Total employee P = ´ 8000 = 1440
100
3
= ´ 100 = 75%
4 12 16
Q= ´ 8000 = 960 , R = ´ 8000 = 1280
31. (d); Required Ratio = 550 : 250 : 400 100 100
= 55 : 25 : 40 = 11 : 5 : 8 15 19
S= ´ 8000 = 1200 , T = ´ 8000 = 1520
32. (b); Average no of male employees working in all the 100 100
æ 400 + 450 + 600 + 450 ö 20
Banks together in 1994 = ç ÷ø U= ´ 8000 = 1600
è 4 100
1900 20
== 475 Male employee in grade (P) = ´ 3200 = 640
4 100
33. (a); Total no. of handicaped candidates
17 11
= 20% of 250 + 30% of 450 = 50 + 135 = 185 Q= ´ 3200 = 544 , R = ´ 3200 = 352
100 100
34. (d); Total no. of required persons = 650 + 400 + 450
= 1500 12 14
S= ´ 3200 = 384 , T = ´ 3200 = 448
35. (c); Required percentage 100 100
350 + 550 + 350 + 650 + 400 26
= ´ 100 U= ´ 3200 = 832
( 400 + 650 + 150 + 500 + 500 + 650 + 600 + 700 ) 100
2300 \ Female employee in grade
= ´ 100 = 55% P = 1440 – 640 = 800
4150
Q = 960 – 544 = 416, R = 1280 – 352 = 928
26 S = 1200 – 384 = 816, T = 1520 – 448 = 1072
36. (d); Male in grade U = ´ 3200 = 832
100 U = 1600 – 932 = 768
Total employee in grade, U = 1600 Clearly in grade T there are maximum females
\ Female = 1600 – 832 = 768 816 - 384 432
40. (d); Required % = ´ 100 = ´ 100
768 96 24 12 384 384
\ Ratio = = = = = 112.5%
832 104 26 13
41. (d); Total No. of males in all department together
16
37. (a); Employee in grade R = ´ 8000 = 1280 2 7 11
100 = ´ 125 + ´ 225 + ´ 150
5 15 15
11 12 17 5
Male in grade R = ´ 3200 = 352 + ´ 375 + ´ 175 + ´ 350
100 25 25 14
\ Female = 1280 – 352 = 928 = 50 + 105 + 110 + 180 + 119 + 125 = 689

359 @BEST300MCQ For More Study Material


Visit: studyiq.com
42. (d); Male in Hr department and finance department 46. (d); %rise in
= 105 + 110 = 215
150
Female in calling and Production departement 2002 = ´ 100 = 37.5%
400
13 9
´ 375 + ´ 350 = 195 + 225 = 420
25 14 – 50
2003 = ´ 100 = – 9.09%
required difference = 420 – 215 = 205 550
43. (c); Female in diffenrent departments-
200
2004 = ´ 100 = 40%
3 8 500
IT ® ´ 125 = 75 , HR ® ´ 225 = 120
5 15 2005 = % down
4 2006 = % down
Finance ® ´ 150 = 40
15 250 ´ 100
2007 = % = 50%
13 500
Calling ® ´ 375 = 195
25 47. (a); Average purchages of onions by dealer
8 400 + 550 + 500 + 700 + 600 + 500 + 750
Marketing ® ´ 175 = 56 =
25 7
9
Production ® ´ 350 = 225 4000
14 = = 571.42
7
\ Less in Finance
48. (d); Total onion purchased by dealer P = 350 + 500 +
12 400 + 600 + 550 +400 + 500 = 3300
´ 375
180
44. (a); Requried Ratio = 25 = =3:2 3300
8 120 \ Required % = ´ 100 = 82.5%
´ 225 4000
15
45. (d); Total no. of employees 700 - 600 1
49. (d); Required % = ´ 100 = ´ 100 = 16.66%
= 125 + 225 + 150 + 375 + 175 + 350 = 1400 600 6
Let us simplify the chart
400 + 550 + 500
50. (d); Required Ratio =
R 500 400 600 800 900 700 700 800 + 700 + 600
Q 400 550 500 700 600 500 750
1450 145
P = = = 29 : 42
350 500 400 600 550 400 500 2100 210
2001 2002 2003 2004 2005 2006 2007

1. (b); From the tabe, if we see the imports in each year 3. (c); Percentage increase in exports-
then we will find that each year it is increasing. 7806 - 6711
2. (a); Ratio of imports to exports in: in year 1981-82 = ´ 100 = 16.31%
6711
12549 13608 8803 - 7806
1980-81 = = 1.87, 1981-82 = = 1.74 in year 1982-83 = ´ 100 = 12.77%
6711 7806 7806
14293 15831 9771 - 8803
1982-83 = = 1.62, 1983-84 = = 1.62 in year 1983-84 = ´ 100 = 11%
8803 9771 8803
17173 18371 11855 - 9771
1984-85 = = 1.44, 1985-86 = = 1.76 in year 1984-85 = ´ 100 = 21.32%
11855 10420 9771
20063 in year 1985-86 = Percentage decrease
1986-87 = = 1.6
12550 12550 - 10420
in year 1986-87 = ´ 100 = 20.44%
Maximum ratio = 1980-81 10420
Required year = 1984-85

360 @BEST300MCQ For More Study Material


Visit: studyiq.com
4. (b); Total trade deficit for the last five years 16. (a); From fig, it is clearly that Japan
= 5490 + 6060 + 5318 + 7951 + 7513 = 32332
250
5. (a); The difference between imports and exports in 17. (d); Required percentage = ´ 100 = 1.05%
23810
year
1980-81 = 12549 – 6711 = 5838 18. (d); 12.5% of the population
1981-82 = 13608 – 7806 = 5802 = (26 + 56 + 62 + 58 + 123 + 58 + 250)
1982-83 = 14293 – 8803 = 5490 12.5% of x = 633
1983-84 = 15831 – 9771 = 6060 633 ´ 100
1984-85 = 17173 – 11855 = 5318 \x = Þ x = 5064
12.5
1985-86 = 18371 – 10420 = 7951
57.5% of x = 2911.8 millions
1986-87 = 20063 – 12550 = 7513
\ Required total GNP = 2911.8 × 1140 = $ 3319452
Required year = 1985-86
19. (a); From the table we can see that statement I is false
23380 - 20825 20. (a); Average GNP of G-7 nations
6. (c); Public debt rose by = ´ 100
20825 19030 + 17820 + 20440 + 15120 + 23810 + 14610 + 20910
=
7
2555 2555
= ´ 100 = ´ 100 = 12.26% » 12% = 18820
20825 20825
21. (d); Average
7. (d); Public debt rose by from 1983 to 1984
= (26880 – 25530) = 1350 1230 + 1085 + 1040 + 1315 + 1160 + 1365
=
8. (c); In each year, public debt increased in percentage 6
21570 - 20825 7195
1981 = ´ 100 = 3.57% = » 1200
20825 6
22. (b); Average
23380 - 21570
1982 = ´ 100 = 8.39%
21570 1450 + 1350 + 1400 + 1500 + 1500 + 1500
=
6
25530 - 23380
1983 = ´ 100 = 9.19%
23380 8700
= = 1450
6
26880 - 25530
1984 = ´ 100 = 5.28%
25530 1100 - 125 = 975 195
23. (a); Radio = =
28200 - 26880 1200 - 140 1060 212
1985 = ´ 100 = 4.91% 24. (d); Required difference
26880
= (130 + 125 + 125 + 135 + 115 + 170)
30500 - 28200 – (125 + 120 + 145 + 115 + 135 + 140)
1986 = ´ 100 = 8.15%
28200 = 800 – 780 = 20
Required Year = 1983 – 84 1500 - 130
9. (a); Required period = 1980-81 25. (d); Required percentage = ´ 100
8850
10. (b); Required period = 1982-83
11. (d); Clearly, highest sale and net profit recorded in 1370
= ´ 100 = 15.48%
the year 80. 8850
12. (a); From year 1973 to 1974, sales is double it self. Solutions: (26 – 30)
\ Required year = 1974 First we have to convert degree into percentage
13. (c); Net Profit / Loss of first five years
5
= 2.1 + 3.5 – 3.2 – 4.2 + 0 = –1.8 laks for converting it we have to multiply by degree
18
14. (c); From the figure it is clearly shows that the required
year = 1978 because

100 100 100 = 5


15. (a); Combined gross profit = ´ 14.3 + (15.6) 360° = 100%, 1° =
45 45 360 18

100 5
= (29.9) = 66.44 lakhs Production = 136.8°× = 38%
45 18

361 @BEST300MCQ For More Study Material


Visit: studyiq.com
35. (b); Ram has greatest marks then Aditya in all subjects
5 5
H R = 36 ´ = 10%, IT = 57.6 ´ = 16% except hindi. Rest are not in picture.
18 18
250 + 250 + 500 1000
5 36. (b); Required ratio = = = 10:13
Marketing = 79.2 ´ = 22% 700 + 600 1300
18
37. (d); History – 500 + 250 + 700 + 250 + 650 = 2350
5 Geography - 600 + 150 + 350 + 350 + 450 = 1900
Account = 50.4 ´ = 14%
18 Math - 350 + 700 + 750 + 500 + 600 = 2900
26. (d); Women in marketing department \Ratio will be- 2350 : 1900 : 2900 = 47 : 38 : 58
22 3 38. (c); Math = 2900 – 700 = 2200
= ´ ´ 3250 = 429
100 5 History = 2350 – 250 = 2100
Difference = 2200 – 2100 = 100
10 13
3250 ´ ´ 650
27. (c); Ratio = 100 25 = 10 ´ 13 ´ 10 = 13 : 28 ´ 100 = 59%
39. (d); Required percentage =
16 7 16 ´ 7 ´ 25 1100
3250 ´ ´
100 10 40. (d); 2006 ® 1450
4 2007 ® 1100
28. (d); Required percentage = ´ 100 = 80% 2008 ® 1800
5
(we can easily calulate the required percentage 2009 ® 1100
by given ratio). 2010 ® 1700
\ Required year = 2007 and 2009
3 16
3250 ´´ ( 28 + 12 )
29. (b); Required % = 10 100 ´ 100 = 48 = 4.8 % 41. (d); Required people = ´ 2900
3250 10 100
30. (b); Women working in organization 40
= ´ 2900 = 1160
38 4 100
Production = ´ ´ 3250 = 988
100 5 42. (a); People having favourite game cricket
12 10 20
HR = ´ ´ 3250 = 156 = ´ 2900 = 580
25 100 100
7 16 43. (d); Cricket as favourite game from village R
IT = ´ ´ 3250 = 364
10 100
40 30
3 22 = ´ ´ 2900 = 12 × 29 = 348
Marketing = ´ ´ 3250 = 429 100 100
5 100
Tennis as favourite game from all village
6 14
Account = ´ ´ 3250 = 210 20 30
13 100 = ´ ´ 2900 = 29 × 2 × 3 = 174
Total = 988 + 156 + 364 + 429 + 210 = 2147 100 100
31. (a); Required marks \ Required people = 870 – (174 + 348)
88 3 5 3 68 = 870 – 522 = 348
= 98 + + 84 ´ + 88 ´ + 64 ´ + 44. (a); People who like hockey in village R
2 2 4 2 2
= 98 + 44 + 126 + 110 + 96 + 34 = 508 30 40
= 2900 ´ ´ = 29 × 12 = 348
1 é 68 + 80 + 90 + 56 + 72 + 66 ù 100 100
32. (d); Average marks =
2 êë 6 ú
û 28
Total people who like hockey = ´ 2900
1 æ 432 ö 216 100
= ç ÷ = = 36 = 28 × 29 = 812
2è 6 ø 6
33. (b); From the table, we can see that only Ram scored People except village R like hockey
the highest marks in more then one subject. = 812 – 348 = 464
34. (c); Sanjay’s overall percentage 21
81 + 40 + 126 + 100 + 117 + 45 Total No. of people in village P = ´ 2900 = 609
= ´ 100 100
625
509 464
= ´ 100 = 81.44% \ Required Ratio = = 16 : 21
625 609

362 @BEST300MCQ For More Study Material


Visit: studyiq.com
48. (b); Required average
10 30
45. (d); People like tennis in village R = ´ ´ 2900 43 ´ 16 + 44 ´ 15 + 45 ´ 14.5 + 48 ´ 15.6 + 55 ´ 18.2 + 55 ´ 14.9
100 100 =
43 + 44 + 45 + 48 + 55 + 55
= 29 × 3 = 87
688 + 660 + 652.5 + 748.8 + 1001 + 819.5
11 ´ 2900 =
No. of the villagers who like = – 87 = 232 290
100 4569.8
= = 15.757 thousand » 15700
48 290
46. (d); Required percentage = ´ 100 = 120% 49. (a); A ® 60 × 75 = 4500, B ® 44 × 15 = 660
40
Difference = 4500 – 660 = 3840 thousands
61 + 54 115 = 38.40 Lacs
47. (d); Required ratio = = 50. (d); All type of product sold by store D = 310
54 + 48 102
Total no. of O type product = 326
Difference = 326 – 310 = 16

1. (c); Suppose the expenditures on production of sugar 8. (b); Number of employees not promoted from HR
and wheat in 2002 are Rs. 100 each. department = 18 × 42 – 82 = 756 – 82 = 674
Number of employees not promoted from
140
Then the required ratio = =7:6 Production department = 21 × 42 – 48
120
= 882 – 48 = 834
2. (a); Suppose the amount of profit earned through the
\ required ratio = 674 : 834 = 337 : 417
sales of sugar in 2000 and 2001 was Rs. 700 and
Rs. 600 respectively. 9. (a); Total employees in Maintenance and Accounts
department
This implies that 35% of expenditure on sugar in
2000 is equal to Rs. 700. Similarly, 20% of = (10 + 12) = 22% of 4200 = 22 × 42 = 924
expenditure on sugar in 2001 is equal to Rs. 600. 46 + 24 70 ´ 100
Hence, the required ratio \ required % = ´ 100 = » 7.58
924 924
700 10. (c); To find the department in which the highest
´ 100 percentage of employees are promoted we need
700 20
= 35 = ´ = 2 : 3. to compare the fractions:
600 35 600
´ 100
20 82 24 36
HR: ; Account: ; Marketing:
3. (d); The sales of the wheat in 2004 was 120% of the 18 12 14
expenditure on wheat in that year. Hence, the
required expenditure 48 78
Production: ; IT department: ;
21 25
16.8
= ´ 100 = Rs. 14 lakhs
120 46
Maintenance:
4. (b); The required profit = 25% of Rs. 13 lakhs 10
= Rs. 3,25,000 Clearly, the competition is between HR and
5. (d); We need some more information to answer the Maintenance departments.
question. 82 46
So, finally compare and
6. (d); Number of employees working in IT department 18 10
4200 Apply cross-multiplication.
= 25% of 4200 = = 1050
4 Since 82 × 10 < 18 × 46
7. (d); Number of employees in Marketing department 46 82
= 14% of 4200 = 588 \ >
10 18
36 Þ The required % is the highest for Maintenance
Required % = ´ 100 » 6
588 department.

363 @BEST300MCQ For More Study Material


Visit: studyiq.com
11. (a); The required savings (in Rs. lakhs)
28
16. (b); Number of cars in State-2 = 700 ´ = 196
1 æ 14.5 ´ 2.05 + 15.3 ´ 2.25 + 16.4 ´ 1.95 ö 100
= ç ÷
100 è +16.2 ´ 2 + 18.2 ´ 1.75 + 22.4 ´ 1.70 ø
5
= Rs. 1.9846 lakhs = Rs. 1,98,460 Number of diesel cars in State-2 = 196 ´ = 70
14

12. (c); 26
Amount on shopping Number of cars in State-4 = 700 ´ = 182
Person (in Rs. lakhs) 100

Ravi 0.2993 1
Number of petrol cars in State-4 = 182 ´ = 91
2
Satish 0.3532
\ Required difference = 91 – 70 = 21
Arun 0.3237
14
Vilas 0.2900 17. (a); Number of cars in State-1 = 700 ´ = 98
100
Arif 0.2240
Number of diesel engine cars in State-1
Suresh 0.3162
3
= 98 ´ = 42
7
So, Satish spends maxmum on shopping
32
Number of cars in State-3 = 700 ´ = 224
13. (b); 100
Amount spent on health
Person Number of petrol engine cars in State-3
(in Rs. lakhs)
Ravi 0.3362 3
= 224 ´ = 84
Satish 0.2565 8

Arun 0.41925 84 - 42 42
\ Required % = = ´ 100 = 100%
Vilas 0.4300 42 42

Arif 0.3675
32
Suresh 0.3094 18. (d); Number of cars in State-3 = 700 ´ = 224
100
Total 2.11885
Number of diesel engine cars in State-3

5
Hence, the required percent = 224 ´ = 140
8
0.3094 ´ 100 Number of diesel engine cars which are AC
= » 14.60%
2.11885
14. (d); The required percent 25
= 140 ´ = 35
100
1.75
= ´ 100 \ Number of non-AC diesel cars
2.05 + 2.25 + 1.95 + 2.00 + 1.75 + 1.70
= 140 – 35 = 105
1.75
= ´ 100 » 14.95% 32
11.7 19. (d); Number of cars in State-3 = 700 ´ = 224
100
15. (d); The required ratio
Number of petrol engine cars in State-2
2.05 ´ ( 21.8 + 20.4 )
= 28 9
1.95 ´ ( 14.3 + 18.5 ) = 700 ´ ´ = 126
100 14
2.05 ´ 42.2 86510 \ Required difference = 224 – 126 = 98
= = = 211 : 156
1.95 ´ 32.8 63960

364 @BEST300MCQ For More Study Material


Visit: studyiq.com
20. (b); Required average In 2011, the number of OBC candidates selected
28
14 4 28 9 as Excise Inspector = 600 ´ = 168
700 ´ ´ + 700 ´ ´ + 100
100 7 100 14
32 3 26 1 In 2012, the number of OBC candidates selected
700 ´ ´ + 700 ´ ´
= 100 8 100 2 18
4 as Excise Inspector = 850 ´ = 153
100
Clearly, in the year 2008 the number of OBC
56 + 126 + 84 + 91 357
= = = 89.25 candidates selected as Excise Inspector is the
4 4 minimum.
21. (d); During 2008, total number of candidates selected 23. (d); Number of General candidates selected as
as Income Tax Inspector = 650
52
Number of candidates from SC category selected Custom Inspector in 2008 = 700 ´ = (52 × 7)
100
12 Number of ST candidates selected as Custom
as Income Tax Inspector = 650 ´ = 78
100
7
During 2009, total number of candidates selected Inspector in 2008 = 700 ´ = (7 ´ 7 )
100
as Income Tax Inspector = 250
Required ratio = (52 × 7) : (7 × 7) = 52 : 7
Number of candidates from SC category selected
24. (a); Number of OBC candidates selected during 2008
14 to 2012 for the post of Income Tax Inspector
as Income Tax Inspector = 250 ´ = 35
100
28 36 30
= 650 ´ + 250 ´ + 550 ´ + 800
During 2010, total number of candidates selected 100 100 100
as Income Tax Inspector = 550
Number of candidates from SC category selected 22 24
´ + 600 ´
100 100
15
as Income Tax Inspector = 550 ´ = 82.5 = 182 + 90 + 165 + 176 + 144 = 757
100
Number of General candidates selected during
During 2011, total number of candidates selected 2008 to 2012 for the post of Custom Inspector
as Income Tax Inspector = 800
52 48 46
Number of candidates from SC category selected = 700 ´ + 650 ´ + 350 ´ + 700
100 100 100
12
as Income Tax Inspector = 800 ´ = 96
58 55
100 ´ + 700 ´
100 100
During 2012, the number of candidates from SC
category selected as Income Tax Inspector = 364 + 312 + 161 + 406 + 385 = 1698
Percentage of OBC candidates selected during
16 2008 - 12 for the post of Income Tax Inspector is
= 600 ´ = 96
100 to the general candidates slected during 2008-12
Clearly, in the years 2011 and 2012 the number for custom Inspector
of SC candidates selected as Income Tax 757 ´ 100
Inspector is the maximum = 46.49
1628
22. (c); In 2008, the number of OBC candidates selected
per cent of the General Candidates selected
36 during 2008 - 12 for the post of Custom Inspector.
as Excise Inspector = 400 ´ = 144
25. (b); Number of Income Tax Inspectors from SC
100
category in 2009
In 2009, the number of OBC candidates selected
36 14
= 250 ´ = 35
as Excise Inspector = 750 ´ = 270 100
100
Number of Custom Inspectors from OBC
In 2010, the number of OBC candidates selected
category in 2010
26
as Excise Inspector = 750 ´ = 195 38
100 = 350 ´ = 133
100

365 @BEST300MCQ For More Study Material


Visit: studyiq.com
Number of Excise Inspectors from ST category in 33. (b); Number of articles sold by Q in 2004
6 = 70% of 32.5 lakhs = 22,75,000
2011 = 600 ´ = 36
Number of articles sold by Q in 2002
100
Required ratio = 35 : 133 : 36 = 30% of 30 lakhs = 9,00,000
2.6 Required % increase
26. (b); Required % = ´ 100 = 1.71%
152 13,75, 000
27. (d); Number of items that remain unsold out of the = ´ 100 = 152.78%
9,00, 000
accepted items for company A in year 2004
= (156 – 2.2) – 145 = 8.8 34. (a); Required ratio
Since the figure is in thousands, the accepted = 60% of 2500000 : 70% of 30,00000
items that remain unsold = 8.8 × 1000 = 8800. = 6 × 25 : 7 × 30 = 6 × 5 : 7 × 6 = 5 : 7
28. (a); Total number of items accepted in 35. (b); Maximum number of articles manufactured from
year 2002 = (164 – 1.7) + (115 – 1.1) + (172 – 2.9) both companies together is (30 + 35) = 65 lakhs
+ (169 – 1.9) + (96 – 0.8) in both the years 2005 and 2007.
= 162.3 + 113.9 + 169.1 + 167.1 + 95.2 = 707.6
36. (c); Two possible answers are years 2007 and 2004.
Since the figure is in thousands, number of items
Between these two, the difference in number of
accepted in year 2002
candidates appeared from Mumbai over the
= 707.6 × 1000 = 707600
previous year will be the lowest for the year 2007.
29. (d); Number of items rejected by Company D
= 1.5 + 1.9 + 2.3 + 2.1 + 2.0 + 2.4 = 12.2 58492 ´ 14
Since the figure is in thousands, number of items 37. (a); The value of will be the highest
100
rejected = 12.2 × 1000 = 12200
among other options.
12200 38. (d); Required number of candidates
\ Average number = = 2033.33 @ 2030
6
28 20
30. (c); The required answer = (175 + 168 + 180 + 171 + = 58248 ´ + 59216 ´
105) thousand = 799000 100 100
31. (c); Number of articles sold by P in 2005 » 16310 + 11845 = 28155 » 28150
= 55% of 35 lakhs = 19,25,000 39. (b); Required number of candidates
Number of articles sold by Q in 2005
= 40% of 30 lakhs = 12,00,000 19
= 71253 ´ = 13538.07 » 13540
\ required value = 19,25,000 + 12,00,000 100
= 31,25,000
40. (d); Required difference
32. (d); Required average
25 + 22.5 + 30 + 35 + 32.5 + 30 21 17
= lakhs = 50248 ´ - 51124 ´ » 10552 - 8691
6 100 100
= 29,16,667 = 1861 » 1860

1. (a); mis. expenses amount = 1500 4. (a); given cost of printing = Rs. 10500
1500 10500
\ Total cost = ´ 100 = 30000 so cost of paper = ´ 15 = Rs. 4500
5 35
30000 125 12
So marked price = ´ = Rs. 7.50 5. (c); central angle of binding = ´ 360 = 43.2°
5000 100 100
2. (d); given mis. charge = 1500 6. (a); Required difference between expenditure on
1500 housing and transport
so advertising charge = ´ 18 = Rs. 5400
5 150000
= ( 15 - 5 ) ´ = Rs. 15000
35 7 100
3. (b); Ratio between Printing and Royalty = =
15 3 7. (c); It is clear from the chart.

366 @BEST300MCQ For More Study Material


Visit: studyiq.com
8. (b); According to question, So from the options we can say the answer will
Expenditure on saving is equal to expenditure be (d) A and B
on Housing
15
9. (d); expenditure on clothing, education and transport 18. (a); Production of B in 2007 = ´ 35 = 5.25 Lakh.
= 10% + 12% + 5% = 27% 100

23 20
10. (d); money spent on the food = 150000 ´ and Production of B in 2008 = ´ 44
100 100
= Rs. 34500 = 8.8 Lakh
3 So, difference = 355000
11. (a); Area of rice and barley together = ´ ( 75 + 36 ) 19. (b); According to question
18
= 3 × 6 = 18 Hundred Acres. Number of 'A' type mobiles in 2008
12. (a); Wheat + Rice + Maze 30
¯ ¯ ¯ = 44 ´ Lakh = 1320000
100
72° + 72° + 45° = 189°
20. (d); required number of unsold mobile (D type)
(which is more than 50%)
13. (c); Ratio between land used for Rice and Barley 15 15 10 15
2007 and 2008 = 35 ´ ´ + 44 ´ ´
72 100 100 100 100
= = 2:1
36 = .7875 lakh + .66 lakh = 1.4475 lakh or 144750
14. (b); 10% of rice = 7.2° 21. (b); Total sale in 2nd and 3rd year = 1773 + 1115
then after distribution of it between wheat and = 2888 crore
barley, the new angle of wheat 22. (a); it is obviously option (a)
2 23. (b); 2nd least sale is in the year of '3' = 1115
= 72° + ´ 7.2° = 76.8°
3 8730 + 924
24. (b); mean of highest and lowest sale = =
2
15. (a); Rice : Jowar : Bajra
= 4827
5´2 : 1´1
25. (b); It will be = 5345 – 1841 = Rs. 3504 crore
2 : 1
26. (c); According to question
10 : 2 : 1
440
Percentage = ´ 100 = 110
10 400
Rice 72 27. (b); According to question
\ per Acre production Ratio = = =5:2
Bajra 1 least decrease is from 2000 to 2001
18
20 100
16. (d); Total no. of mobiles of model A + B + E in 2007 = ´ 100 = %
340 17
35 35 28. (a); According to question
= (30 + 15 + 10) × = 55 ´ = 19.25 lakh
100 100 = 420 – 320 = Rs. 100
17. (d); 2007 2008 320
10 1 29. (b); it will be = = .8 times
for 'A ' 30% 40% = ´ 100 = 33 % 400
30 3
30. (d); It is clear from the graph that least percent
5 1 increase is from 2003 to 2004.
for 'B' 15% 20% = ´ 100 = 33 %
15 3 31. (b); from the graph it is W.B.
5
for 'C ' 20% 15% = ´ 100 = 25% 32. (b); largest producer of rice is W.B.
20
33. (b); production of rice by Haryana = 2
5 1
for 'D ' 15% 10% = ´ 100 = 33 % production of rice all states = 7 + 8 + 4 + 3 + 2 = 24
15 3
for 'E' 10% 10% = Part of Production by Haryana to all
5 2 1
and for 'F ' 10% 5% = ´ 100 = 50% = = part
10 24 12

367 @BEST300MCQ For More Study Material


Visit: studyiq.com
34. (c); Maharashtra 44. (a); percent increase in science student from
35. (d); Largest producer of wheat is UP. 600 - 400
36. (c); Profit in 1983 = 50 – 30 = 20 lakh 1990 – 91 to 1992 – 93 = ´ 100 = 50
400
and profit in 1984 = 40 – 30 = 10 lakh 45. (c); in year 2003 P + Q = R + S = 33
so, difference = 20 – 10 = 10 lakhs 46. (c); Average total production of cars from 1999 to
30 + 50 + 40 + 60 + 60 74 + 71 + 75 + 90 + 80 + 86
37. (c); Average income = 2004 = = 79.3 App.
5 6
So, in 2003 the production of car is equal to
240 Average production of cars.
= = 48 lakhs
5 47. (a); Production of car 'P' type is 25% of total
So, here are three years (1983, 1985 and 1986) production of cars in 2003
48. (d); According to given table we can say that S type
which have more income than Average income.
car is showing continuous production
240 increament from 1999 to 2004.
Average income
38. (b); Ratio of = = 5 49. (a); student enrolled in NCC activities = 1200 ´
15
Average Profit 85 100
5 = 180
Profit = 48 : 17 50. (c); Number of students in debating + HRD
39. (a); Percentage increase in profit in 1986 over 1982 is 13 + 11 24
= 1200 ´ = 1200 ´ = 288
25 - 10 100 100
= ´ 100 = 250 - 100 = 150% 51. (d); According to question
10
40. (a); Total income from 1982 to 1986 = 240 Eco c lub 22
= ´ 100 = ´ 100 = 104.76
Total exp. from 1982 to 1986 = 155 Red cross 21
\ difference = 240 – 155= 85 lakh.
Scout + Red cross 18 + 21
41. (a); from the graph it is clear that option (a) is our 52. (a); Ratio of = = = 3:1
Debating 13
answer.
42. (c); Total students in 1990–91 = 1350 53. (a); From the pie-chart the required ratio is between
and number of students in science = 400 Eco club 2
= =
HRD club 1
400
So, ´ 100 = 29.6%
1350 800000
54. (c); Number of horns produces = ´ 3 = 75000
43. (d); Total student in 1991 – 92 32
= 550 + 500 + 250 + 200 = 1500 28 - 25
commerce student is 250 55. (b); Percentage of tubes = ´ 100 = 12
25
1500 56. (a); Ratio of number of horns produced at Nagpur
So, Total student is =
250 3
and Pune =
= 6 times of commerce student 7

368 @BEST300MCQ For More Study Material


Visit: studyiq.com
Quantitative Aptitude

15 Practice Sets
Based on
Latest Pattern

@BEST300MCQ For More Study Material


369
Visit: studyiq.com
Quantitative Aptitude

1. In a classroom there are certain number of benches. (a) Rs. 35 (b) Rs. 350
If 6 students are made to sit on a bench, then to (c) Rs. 245 (d) cannot be determined
accommodate all of them, one more bench is needed. 9. Reena borrowed Rs. 6,450 at 5 percent simple
However, if 7 students are made to sit on a bench, interest repayable in 4 equal installments. What will
then, after accommodating all of them, space for 5 be the annual installment payable by him ?
students is left. What is the total number of students (a) Rs. 1710 (b) Rs. 1800
in the class? (c) Rs. 1910 (d) Rs. 1860
(a) 30 (b) 42 10. Mr. X invested an amount for 2 years at 15% per
(c) 72 (d) None of these annum at simple interest. Had the interest been
2. The average monthly salary of the workers in a compounded, he would have earned Rs. 450/- more
workshop is Rs. 8,500. If the average monthly salary as interest. What was the amount invested ?
of 7 technicians is Rs. 10,000 and the average (a) Rs. 22,000 (b) Rs. 25,000
monthly salary of the rest is Rs. 7,800, then the total (c) Rs. 18,000 (d) None of these
number of workers in the workshop is 11. In a business, Abhiram and Chiru invested amounts
(a) 18 (b) 20 in the ratio 2 : 1, whereas the ratio between
(c) 22 (d) 24 amounts invested by Abhiram and Bikash was 3 : 2.
3. There were 35 students in a hostel. If the number of If Rs. 1,57,300 was their profit, how much amount
students increases by 7, the expenses of mess did Bikash receive?
increase by Rs. 42 per day while the average (a) Rs. 72600 (b) Rs. 48400
expenditure per head diminishes by 1. Find the (c) Rs. 36300 (d) Rs. 24200
original expenditure of the mess. 12. Instead of dividing of Rs. 117 among P, Q, R in the
1 1 1
(a) Rs. 320 (b) Rs. 420 ratio : ∶ , it was divided in the ratio 2 : 3 : 4 by
2 3 4
(c) Rs. 160 (d) Rs. 158
mistake. Who gained in this transaction?
4. A man is 50 years old. His brother is 7 years older
(a) Only P (b) Only Q
than him and his sister is 12 years younger than his
(c) Only R (d) Both Q and R
brother. When his sister was 15 years old, then the
13. An Insurance employer reduces the number of his
men’s age was :
employees in the ratio 9 : 8 and increases their
(a) 27 yrs (b) 19 yrs
wages in the ratio 14 : 15. If the original wage bill
(c) 15 yrs (d) 20 yrs
was Rs. 18,900, find the ratio of decreased wage and
5. A man walked diagonally across a square plot.
original wage.
Approximately what was the percent distance saved
(a) 20 : 21 (b) 21 : 29
by not walking along the edges ?
(c) 20 : 19 (d) 19 : 21
(a) 30 (b) 20
14. The number obtained by interchanging the digits of
(c) 33 (d) 24
a two-digit number is less than the original number
6. The ratio of the number of boys to that of girls in a
by 63. If the sum of the digits of the number is 11,
school is 4 : 1. If 75% of boys and 70% of the girls
what is the original number?
are scholarship holders, then the percentage of
(a) 29 (b) 92
students who do not get scholarship is
(c) 74 (d) Cannot be determined
(a) 50 (b) 28
15. The respective ratio between the present ages of
(c) 75 (d) 26
Roshan and Rakesh is 6: 11. Four years ago, the ratio
7. A tradesman gives 4% discount on the marked price
of the ages was 1: 2 respectively. What will be
and gives 1 article free for buying every 15 articles
Rakesh’s age after five years?
and thus gains 35%. The marked price is increased
(a) 45 years (b) 29 years
above the cost price by.
(c) 49 years (d) Cannot be determined
(a) 40% (b) 39%
16. The numbers 11284 and 7655, when divided by a
(c) 50% (d) 20%
8. A certain amount earns simple interest of Rs. 1750 certain number of three digits, leave the same
after 7 years. Had the interest per annum been 2% remainder. Find that number of three digits.
more, how much more interest would it have (a) 161 (b) 171
earned? (c) 181 (d) 191

@BEST300MCQ For More Study Material


370
Visit: studyiq.com
Quantitative Aptitude

17. Four Iron metal rods of lengths 78 cm, 104 cm, 117 24. A started a business investing Rs. 250000 in year
cm and 169 cm are to be cut into parts of equal 2008. In year 2009 he invested an additional
length. Each part must be as long as possible. What amount of Rs. 100000 and B joined him with an
amount of Rs. 350000. In year 2010, A invested
is the maximum number of pieces that can be cut?
another additional amount of Rs. 100000 and C
(a) 27 (b) 36 joined them with an amount of Rs. 350000. What
(c) 43 (d) 400 will be B’s share in the profit of Rs. 1500000 earned
18. A person lent Rs. 5,000 partly at the rate of 4 at the end of three years from the start of the
percent and partly at the rate of 5 per cent per business in 2008?
annum, at simple interest for purchased a wrist (a) Rs. 700000 (b) Rs. 500000
watch. The total interest after 2 years is Rs. 440. The
(c) Rs. 450000 (d) Rs. 750000
sum of money lent at each of the above rates is to be
25. The work done by a female worker in 8 h is equal to
divided in the ratio:
the work done by a male worker in 6h and by a child
(a) 4: 5 (b) 3: 2
worker in 12 h. If working 6h/ day 9 male worker
(c) 5: 4 (d) 2: 3
9 can finish the work in 6 days. If 12 male , 12 female
19. Simple interest on a certain amount is of the workers and 12 children together finish the same
16
principal. If the numbers representing the rate of work working 8 h/day in-
interest in percent and time in years be equal, then 1
(a) 1 days (b) 3 days
time, for which the principal is lent out, is: 2
2 2
1 1 (c) 3 days (𝑑) 1 days
(a) 5 years (b) 6 years 3 3
2 2
26. A shopkeeper purchased 150 identical pieces of
(c) 7 years (d) 7.5 years
table watch at the rate of Rs. 250 each. He spent an
20. Anubhav lends Rs. 2500 to Bose and a certain sum
amount of Rs. 2500 on packing. He fixed the label
to Chandra at the same time at 7% annual simple
price of each table watch at Rs. 320. However, he
interest. If after 4 years Anubhav received interest of
decided to give a discount of 5% on the labeled
Rs. 1120 from Bose and Chandra, the sum lent to
price. What is the percent profit earned by him?
Chandra is
(a) 16% (b) 15%
(a) Rs. 700 (b) Rs. 6500
(c) 14% (d) 20%
(c) Rs. 40000 (d) Rs. 1500
27. The jogging track in a sports complex is 726 m in
21. A man moves √2𝑥 km East from his residence and
circumference. Suresh and his wife start from the
then moves x km North. He then goes x km North-
same point and walk in opposite directions at 4.5
East and finally he takes a turn of 90° towards right
Km/h and 3.75 Km/h, respectively. They will meet
and moves a distance x km and reaches his office. for the first time in:
What is the shortest distance of the office from his (a) 5.5 minutes (b) 6.0 minutes
residence?
(c) 5.28 minutes (d) 4.9 minutes
(a) (2√2 + 1) 𝑥 𝑘𝑚 (b) 3 x km
28. A fort has food for 220 men for 16 weeks at the rate
(c) 2√2 𝑥 𝑘𝑚 (d) 3√2𝑥 𝑘𝑚 of 450 gm per day per man. How many men should
22. A leak was found in a ship when it was 77 km from go out so that the food may last for 24 weeks at the
the shore. It was found that the leak admits 2.25 rate of 330 gm per day per man?
tones of water in 5.5 min. 92 tones of water will be (a) 20 (b) 30
sufficient to sink the ship. But the pumps can throw (c) 25 (d) 40
2
out 12 tones an hour. Find the average rate of sailing 29. A can cultivate th of land in 6 days and B can
5
at which the ship may reach the shore as it begins to cultivate
1
rd of the same land in 10 days. Working
3
sink. 4
together A and B cultivate th of the land in
(a) 10.5 km/hr (b) 14.5 km/hr 5
(c) 9.75 km/hr (d) 13 km/hr (a) 4 days (b) 5 days
23. A tank has a leak which would empty the completely (c) 8 days (d) 10 days
filled cistern in 20 hours. If the tank is full of water 30. X and Y entered into a partnership investing Rs.
and a tap is opened which admits 2 liters of water 16000 and Rs. 12000 respectively. After 3 months, X
withdrew Rs. 5000 while Y invested Rs. 5000 more.
per minute in the tank, the leak now takes 30 hours
After 3 more months Z joins the business with a
to empty the tank. How many liters of water does capital of Rs. 21000. The share of Y exceeds that of Z,
the tank hold? out of a total profit of Rs. 26400 after one year, by
(a) 2400 (b) 4500 (a) Rs. 2100 (b) Rs. 1200
(c) 1200 (d) 7200 (c) Rs. 2400 (d) Rs. 3600

@BEST300MCQ For More Study Material


371
Visit: studyiq.com
Quantitative Aptitude

Set-1: Solutions
1. (c); Let the no. of benches = 𝑥 We can’t solve these because three unknowns
ATQ— 6(𝑥 + 1) = 7𝑥 − 5 are there, 𝑥 r and S.I.
𝑥 = 11 So, can’t be determined.
So, No. of students = 6(𝑥 + 1) = 72 9. (b); 𝐴𝑚𝑜𝑢𝑛𝑡 𝑜𝑓 100 𝑎𝑓𝑡𝑒𝑟 4 𝑦𝑒𝑎𝑟 = (100 +
2. (c); Let no. of workers = 𝑥 100×5×4
ATQ, )
100
𝑥 × 8500 = 7 × 10000 + (𝑥 − 7) × 7800 ∴ present value of 120 due after 4 yrs = 100
𝑥 = 22 ∴ 𝑃𝑟𝑒𝑠𝑒𝑛𝑡 𝑣𝑎𝑙𝑢𝑒 𝑜𝑓 𝑥 𝑑𝑢𝑒 𝑎𝑓𝑡𝑒𝑟 4 𝑦𝑟𝑠 = 𝑥
5
3. (b); Let the average expenditure per student = 𝑥 6
Original total expenses = 35𝑥
New total expenses = (35𝑥 + 42)
New Avg. expenditure per student = (𝑥 − 1) 𝑆𝑖𝑚𝑖𝑙𝑎𝑟𝑙𝑦, 𝑝𝑟𝑒𝑠𝑒𝑛𝑡 𝑣𝑎𝑙𝑢𝑒 𝑜𝑓 𝑥 𝑑𝑢𝑒 𝑎𝑓𝑡𝑒𝑟 3 𝑦𝑒𝑎𝑟𝑠
20
𝐴𝑇𝑄 — (
35𝑥+42
) = (𝑥 − 1) ⇒ 𝑥 = 12 = 𝑥
35+7 23
The original expenditure = 35 × 12 = 420
4. (d); Age of man = 50 years
∴ age of his elder brother = 50 + 7 = 57 years 𝑆𝑖𝑚𝑖𝑙𝑎𝑟𝑙𝑦, 𝑝𝑟𝑒𝑠𝑒𝑛𝑡 𝑣𝑎𝑙𝑢𝑒 𝑜𝑓 𝑥 𝑑𝑢𝑒 𝑎𝑓𝑡𝑒𝑟 2 𝑦𝑒𝑎𝑟𝑠
10
Age of his sister = 57 – 12 = 45 years = 𝑥
11
Diff. in age = (50 – 45) = 5 years.
Thus, when the age of his sister was 15 years,
Then the age of man was = 15 + 5 = 20 years. 𝑆𝑖𝑚𝑖𝑙𝑎𝑟𝑙𝑦, 𝑃𝑟𝑒𝑠𝑒𝑛𝑡 𝑣𝑎𝑙𝑢𝑒 𝑜𝑓 𝑥 𝑑𝑢𝑒 𝑎𝑓𝑡𝑒𝑟 1 𝑦𝑒𝑎𝑟𝑠
5. (a); Let the side of square plot = 𝑎 meter. 20
= 𝑥
∴ length of diagonal of square plot = 𝑎√2 m 21
5 20 10 20
∴ 𝑥 + 𝑥 + 𝑥 + 𝑥 = 6450
6 23 11 21
𝑥 = 1800 (approx..)
% 𝑑𝑖𝑠𝑡𝑎𝑛𝑐𝑒 𝑠𝑎𝑣𝑒𝑑 𝑏𝑦 𝑛𝑜𝑡 𝑤𝑎𝑙𝑘𝑖𝑛𝑔 𝑎𝑙𝑜𝑛𝑔 𝑡ℎ𝑒 𝑒𝑑𝑔𝑒𝑠 10. (d); We know that,
2𝑎−𝑎√2 2−√2 2
= × 100% = ( ) × 100% ≈ 30% 𝐷𝑖𝑓𝑓𝑒𝑟𝑒𝑛𝑐𝑒 = 𝑃𝑟𝑖𝑛𝑐𝑖𝑝𝑎𝑙 × (
𝑟
)
2𝑎 2
100
6. (d); Let the no. of boys = 4𝑥 𝑎𝑛𝑑 450×100×100
Girls = 𝑥 𝑃= = 20000
225
𝑁𝑜. 𝑜𝑓 𝐵𝑜𝑦𝑠 𝑤ℎ𝑜 ℎ𝑜𝑙𝑑 𝑠𝑐ℎ𝑜𝑙𝑎𝑟𝑠ℎ𝑖𝑝 11. (b); Ratio Abhiram :Bikash = 3 : 2
75 And Abhiram : Chiru = 2 : 1
= × 4𝑥 = 3𝑥
100
∴Abhiram :Bikash : Chiru = 6 : 4 : 3
𝐴𝑛𝑑 𝑛𝑜. 𝑜𝑓 𝐺𝑖𝑟𝑙𝑠 𝑤ℎ𝑜 ℎ𝑜𝑙𝑑 𝑠𝑐ℎ𝑜𝑙𝑎𝑟𝑠ℎ𝑖𝑝 4
70×𝑥 7𝑥 Profit share of Bikash= × 1,57,300
= = 13
100 10
Number of students who do not hold = Rs. 48400.
1 1 1
scholarship 12. (d); If Rs. 117 are divide in the ratio ∶ : that is, 6:
2 3 4
7𝑥 7𝑥 13𝑥
= 5𝑥 − (3𝑥 + ) = 2𝑥 − = 4: 3 among P, Q and R, then
10 10 10
13𝑥 Share of P = Rs. 54
𝑇ℎ𝑒 𝑟𝑒𝑞𝑢𝑖𝑟𝑒𝑑 𝑝𝑒𝑟𝑐𝑒𝑛𝑡𝑎𝑔𝑒 = × 100 = 26%
10
Share of Q = Rs. 36
5𝑥
7. (c); Let the CP of each article is 1 Rs. Share of R = Rs. 27
C.P. of 16 book = 16 Rs. If Rs. 117 are divided in the ratio 2: 3: 4 among,
𝑆. 𝑃. 𝑜𝑓 15 𝑏𝑜𝑜𝑘 = 16 ×
135
=
108 P, Q and R then
108
100
36
5 Share of P = Rs. 26
𝑆. 𝑃. 𝑜𝑓 1 𝑏𝑜𝑜𝑘 = = Share of Q = Rs. 39
5×15 25
36
𝑁𝑜𝑤, 96% 𝑜𝑓 𝑚𝑎𝑟𝑘𝑒𝑑 𝑝𝑟𝑖𝑐𝑒 = Share of R = Rs. 52
25
36×100 3 13. (a); Let the initial number of employees be 9x and
𝑀𝑎𝑟𝑘𝑒𝑑 𝑝𝑟𝑖𝑐𝑒 = = = 1.5
25×96 2
0.5
the employer gives Rs. 14y as wage to each.
𝑇ℎ𝑒 𝑟𝑒𝑞𝑢𝑖𝑟𝑒𝑑 % 𝑖𝑛𝑐𝑟𝑒𝑎𝑠𝑒 = × 100 = 50% Now, according to the question,
1
8. (d); S.I.= 1750 9𝑥 × 14𝑦 = 18900
𝑥×7×𝑟
1750 = ⇒𝑥𝑦 =
18900
= 150
100
9 × 14
𝑥𝑟 = 25000 … (𝑖)
Now, And the late bill = 8𝑥 × 15𝑦 = 120𝑥𝑦
𝑃×(𝑟+2)×7 = 120 × 150 = 18000
𝑆. 𝐼. = … (𝑖𝑖) ∴ Required ratio = 18000 : 18900 = 20 : 21
100

@BEST300MCQ For More Study Material


372
Visit: studyiq.com
Quantitative Aptitude

14. (b); Let the digits at unit’s & ten’s place are 𝑥&𝑦 21. (b); In ∆ BCD,
respectively.
number = 10𝑦 + 𝑥
𝑥 + 𝑦 = 11 … (i)
10𝑦 + 𝑥 − (10𝑥 + 𝑦) = 63
10𝑦 + 𝑥 − 10𝑥 − 𝑦 = 63
9𝑦 − 9𝑥 = 63
𝑦−𝑥 = 7 … (ii)
From (i) & (ii)
𝑦=9 ⇒ 𝑥=2
Original number = 92
15. (c);
𝑅𝑜𝑠ℎ𝑎𝑛 𝑎𝑔𝑒
=
6𝑥 𝐵𝐷2 = 𝐵𝐶 2 + 𝐶𝐷2 = 𝑥 2 + 𝑥 2
𝑅𝑎𝑘𝑒𝑠ℎ 𝐴𝑔𝑒 11𝑥
6𝑥−4 1
⇒ 𝐵𝐷 = √2x ⇒ 𝐵𝐷 = 𝐴𝐸 = √2𝑥
A.T.Q. = 2
11𝑥−4 2
∴ 𝑂𝐷 = √(2√2𝑥) + 𝑥 2 = √8𝑥 2 + 𝑥 2 = 3𝑥 𝑘𝑚
12𝑥 − 8 = 11𝑥 − 4 ⇒ 𝑥 = 4
Rakesh Age = 44 & After five years = 44 + 5 22. (a); ∴ Water that leaks in 5.5 min = 2.25 tones
2.25
= 49 years ∴ Water that leaks in 60 min = × 60
5.5
1350
16. (d); The required number must be a factor of (11284 = tones
55
– 7655) or 3629. After pumping water that is left in boat in 60
Now, 3629 = 19 × 191 min.
1350 690
So 191 is the required number. = − 12 = 𝑡𝑜𝑛𝑒𝑠
55 55
17. (b); Since each Iron rod must be cut into parts of ∴ 92 tones water that remains in boat in
55 22
equal length and each part must be as long as × 92 = hr
690 3
possible, so HCF should be taken. ∴ Required speed =
77
=
231
= 10.5 𝑘𝑚/ℎ
22
22
HCF of 78, 104, 117 and 169 = 13. 3

No. of parts from 78cm. rod = 78 /13=6 23. (d); Let the capacity of the tank = x litres
According to the question,
No. of parts from 104 cm. rod =104/13=8 Quantity of water emptied by the leak in
No. of parts from 117 cm. rod =117/ 13=9 𝑥
1 hour = litres
20
No. of parts from 169 cm. rod =169 /13=13 Quantity of water filled by the tap in 1 hour
Maximum no. of pieces = 6 + 8 + 9 + 13 = 36 = 120 litres
18. (b); Let the rate of interest (final) = R% According to the question,
5000×𝑅×2 𝑥 𝑥 𝑥 𝑥
440 = − = 120 ⇒ − = 240
100 20 30 10 15
3𝑥−2𝑥 𝑥
4.4% = 𝑅 ⇒ = 240 ⇒ = 240
30 30
⇒ 𝑥 = 240 × 30 = 7200 𝑙𝑖𝑡𝑟𝑒𝑠
24. (b); A’s investments for 3 yrs.
= (250000 × 3 + 100000 × 2 + 100000 × 1)
= 𝑅𝑠. 1050000
B’s investment for 3 yrs
= 350000 × 2 = 𝑅𝑠. 700000
Ratio = 3: 2 C’s investment for 3 yrs. = 350000 × 1
9 = Rs. 350000
19. (d); Let sum = x. Then, S.I. = x
16
∴ Ratio of investment of A : B : C = 1050000 :
Let rate = R% and time = R years.
x×R×R 9x 900 30 1
700000 : 350000
∴( )= ⇔ R2 = ⇔R= =7 =3:2:1
100 16 16 4 2
1 2
Hence, time = 7 years. B’s profit = × 1500000 = Rs. 500000
2 3+2+1
2500×4×7 25. (a); Work done/hour by a female, a male and a child
20. (d); Interest on 2500 = = 700
100 are x, y and z, unit respectively.
Interest on 2nd amount = 1120 − 700 = 420 So, 8x = 6y = 12z
𝑃×7×4 3 𝑌
420 = ⇒ 𝑃 = 1500 ⇒ 𝑥 = 𝑦 𝑎𝑛𝑑 𝑧 =
100 4 2

@BEST300MCQ For More Study Material


373
Visit: studyiq.com
Quantitative Aptitude
4500 3750
9 males can complete a work in 6 days working ⇒ 𝑥+ 𝑥 = 726 ⇒ 𝑥 = 5.28
60 60
6 h/ day.
28. (a); ∴ At 450g per man per day the provision is for
∴ Work done = 9 × 6 × 6y = 324y
16 weeks for 220 men
Work done by 12 males, 12 females and 12
∴ 1 g per man per day provision is for 1 week
children in 1 day = 8h/day
= 220 × 450 × 16
= (12𝑦 + 12𝑥 + 12𝑧) × 8
3 𝑦
∴ 330 g per man per day the provision is for
= [12𝑦 + 12 × 𝑦 + 12 × ] × 8 = 216𝑦 24 weeks =
220×450×16
= 200
4 2
324𝑦 1 330×24
Days required to finish work = = 1 𝑑𝑎𝑦𝑠 Number of men to go out = 220 – 200 = 20
216𝑦 2
2
26. (c); Cost price of each table watch = 250 +
2500 29. (c); The part of field cultivated by A in 1 day =
5×6
150 1
=
800 =
15
3
The part of field cultivated by B in 1 day
Labeled price = Rs. 320 1 1
B= =
SP = 320 – 5% of 320 = 304 3×10 30
800
304−[ ] ∴ The part of field cultivated by A and B
3
Profit percent = 800 × 100 1
together in 1 day = + = =
1 3 1
3 15 30 30 10
112 3 4
= × × 100 = 14% ∴ part of field is cultivated by A and B together
3 800 5
4 1 4×10
27. (c); Suppose they meet after x minutes in 1 day = / days = = 8 days
5 10 5
4500 meters are covered by Suresh in 60
30. (d); X : Y : Z = Rs. (16000 × 3 + 11000 × 9 : 12000 × 3
minutes.
4500 + 17000 × 9 : 21000 × 6)
∴ In x min, Suresh covers x meters. =7:9:6
60
3750 meters are covered by Suresh’s wife in 60 ∴ (Y’s share – Z’s share)
minutes. 9 6
= Rs. [(26400 × ) − (26400 × )]
3750 22 22
∴ In x min, Suresh’s wife covers = x meters. = Rs. (10800 – 7200) = Rs. 3600
60

@BEST300MCQ For More Study Material


374
Visit: studyiq.com
Quantitative Aptitude

1. By selling an article at 80% of its marked price, a 9. A and B started a business with Rs. 20000 and Rs.
trader makes a loss of 10%. What will be the profit 35000 respectively. They agreed to share the profit
percentage if he sells it at 95% of its marked price? in the ratio of their capital. C joins the partnership
(a) 6.9 (b) 5 with the condition that A, B and C will share profit
(c) 5.9 (d) 12.5 equally and C pays Rs. 220000 as premium for this,
2. If the work done by(𝑥 − 1) men in (𝑥 + 1) days and to be shared between and A and B. this is to be
the work done by (𝑥 + 2) men in (𝑥 − 1) days is in divided between A and B in the ratio of
the ratio of 9 : 10, then 𝑥 is equal to (a) 10 : 1 (b) 1 : 10
(a) 5 (b) 6 (c) 9 : 10 (d) 10 : 9
10. A can complete a work in 20 days and B in 30 days.
(c) 7 (d) 8
A worked alone for 4 days and then B completed the
3. On compound interest a certain amount becomes 𝑝
remaining work along with C in 18 days. In how
times in a year, then in how many years it will
become 𝑞 times? many days can C working alone complete the work?
𝑙𝑜𝑔 𝑞 3 𝑙𝑜𝑔 𝑞 (a) 12 (b) 68
(a) (b) (c) 72 (d) 90
𝑙𝑜𝑔 𝑝 𝑙𝑜𝑔 𝑝
2 𝑙𝑜𝑔 𝑝 𝑙𝑜𝑔 𝑝
(c) (d) 11. I was born 30 years after my father was born. My
𝑙𝑜𝑔 𝑞 𝑙𝑜𝑔 𝑞
sister was born 25 years after my mother was born.
4. A man on the top of a vertical tower observes a car
The average age of my family is 26.25 years right
moving at a uniform speed coming directly towards
now. My sister will get married 4 years from now
it. If it takes 12 minutes for the angle of depression
and will leave the family. Then the average age of
to change from 30° to 45°. How soon after this, will 107
the car reach the tower ? the family will be years. What is the age of my
3
(a) 16 min 23 sec (b) 17 min 34 sec father?
(c) 15 min 32 sec (d) 14 min 41 sec (a) 30 year (b) 35 year
5. In a race of 600 m, A can beat B by 60 m and in a (c) 40 year (d) 45 year
race of 500 m, B can beat C by 25m. By how many 12. A shopkeeper marks his goods 60% above the cost
metres will A beat C in a 400 m race ? 1th
price. He sold half quantity at mark price, of
(a) 56 m (b) 60 m 4
(c) 58 m (d) 54 m quantity at 10% discount and rest of the quantity is
6. Of the adult population in a certain city, 45% of men sold after giving two successive discounts of 10%
and 25% of women are married. Assuming that no and 8%. Find his profit or loss during whole
man marries more than one women and vice versa, transaction.
the percentage of total population of adults who are
(a) 49.12% profit (b) 33.12% loss
married, is
(a) 33.33 (b) 32.14 (c) 36% profit (d) 50% loss
(c) 31.1 (d) 30 13. The milk and water in two vessels A and B are in the
7. In an examination paper of five questions, 5% of the ratio 4 : 3 and 2 : 3 respectively. In what ratio, the
candidates answered all of them and 5% answered liquids in both the vessels be mixed to obtain a new
none. Of the rest, 25% candidates answered only mixture in vessel C containing half milk and half
one question and 20% answered 4 questions. If 396 water ?
candidates answered either 2 questions or 3
(a) 7 : 5 (b) 5 : 2
questions, the number of candidates that appeared
for the examination was (c) 3 : 11 (d) 1 : 2
(a) 800 (b) 1000 14. A, B and C can do a job in 6 days, 12 days and 15
1
(c) 850 (d) 900 days respectively. After of the work is completed, C
8
8. Arun and Bhaskar start from place P at 6 am and
7:30 am respectively and run in the same direction. leaves the job. Rest of the work is done by A and B
Arun and Bhaskar run at 8 km/hr and 12 km/hr, together. Time taken to finish the rest of the work is
5 1
respectively. Bhaskar overtakes Arun at (a) 5 days (b) 5 days
6 4
(a) 10 : 30 am (b) 9 am 1 3
(c) 11: 30 am (d) 11 cm (c) 3 days (d) 3 days
2 4

@BEST300MCQ For More Study Material


375
Visit: studyiq.com
Quantitative Aptitude

15. In a two-digit number ,if it is known that its unit’s 23. If we divide 3150 into four parts such that half of the
digit exceeds its ten’s digit by 2 and that the product first part, a third of the second part and a fourth of
of given number and the sum of its digit is equal to the third part are equal to one-twelfth of the fourth
144 ,the number is? part. Then find the largest part?
(a) 46 (b) 42 (a) 1800 (b) 1200
(c) 26 (d) 24 (c) 600 (d) None of these
16. The price of ten chairs is equal to that of four tables 24. The total monthly sales of two companies A and B
.The price of 15 chairs and 2 tables is Rs. 4000 .The are in the ratio 2 : 3 and their total monthly
total price of 12 chairs and 3 tables is? expenditures are in the ratio 3 : 4. Find the ratio of
(a) Rs 3750 (b) Rs 3840 the profits of two companies given that company A’s
(c) Rs 3500 (d) Rs 3900 profit is equal to one fifth of its sales.
17. Excluding stoppages, the speed of a bus is 54 km/h (a) 6 : 13 (b) 8 : 15
and including stoppages, it is 45 km/h. For how (c) 13 : 6 (d) 15 : 8
1 1
many minutes does the bus stop per hour? 25. Two equal glasses are and full of milk
3 4
(a) 12 (b) 10 respectively. If they are completely filled up with
(c) 9 (d) 20 water, the contents are then mixed up in another
18. Average salary of all women in a company is 13 vessel. What will be the ratio of milk and water
more than the average salary of all employees vessel?
together in that company. If number of female (a) 1 : 5 (b) 7 : 17
employees in the company is 91. Find the number of (c) 3 : 7 (d) 2 : 5
male employee in the company .Average salary of 26. Three glasses of capacity 2 L, 5 L and 9 L contain
male employee is 20 less than the average salary of mixture of milk and water with milk concentrations
women. 90%, 80% and 70% respectively. The contents of
(a) 250 (b) 260 three glasses are emptied into a large vessel. Find
(c) 169 (d) 117 the ratio of milk to water in the resultant mixture
19. Time taken to complete a work by A alone is 200% (a) 52 : 49 (b) 31 : 151
more than the time taken by both A and B to (c) 125 : 51 (d) 121 : 39
complete the work. B is thrice as efficient as C. B and 27. A daily wage worker was paid Rs. 1700 during a
C together take 15 days to complete the same work. period of 30 days. During this period he was absent
for 4 days and was fined Rs. 15 per day for absence.
How many days A will take to complete the work
He was paid the full salary only for 18 days as he
alone?
came late on the other days. Those who came late
(a) 30 days (b) 40 days
were given only half the salary for that day. What
(c) 50 days (d) 45 days
was the total salary paid per month to a worker who
20. Sashi spends 36% of her income in various expenses came on time everyday and was never absent?
of the remaining, he spends one eigth(1/8)th on her (a) Rs. 2250 (b) Rs. 2500
brother while 3/8 th on her sister and remaining (c) Rs. 3000 (d) Rs. 2400
keep as savings if the difference betweeen her 28. Two trains start at the same time from point A and
money spending on brother and sister is Rs 3280. point B and proceed towards each other at the rate
What is Sashi’s saving? of 70 km/hr and 85 km/hr respectively. When they
(a) 6480 (b) 6000 meet, it is found that first train has travelled 165 km
(c) 6600 (d) 6560 less than the second. Find the distance between
21. The captain of a famous club Hockey team of eleven point A and point B.
members is 26 yrs old and the goal keeper is 3 yrs (a) 1507 km (b) 1705 km
older than him. If the ages of these two are excluded, (c) 1750 km (d) 1570 km
7
the average age of the remaining players is one year 29. A car running at of its own speed reached a place
11
less than the average age of the whole team. What is in 22 hrs. How much time could be saved if the car
the average age of the team? would run at its own speed?
(a) 14 hrs (b) 7 hrs
(a) 22 yrs (b) 23 yrs
(c) 8 hrs (d) 16 hrs
(c) 24 yrs (d) 25 yrs 30. Two trains of lengths 80 m and 100 m respectively
22. The sum of the ages of a mother and son is 45 yrs, run on parallel lines of rails. When running in the
Five years ago the product of their ages was 4 times same direction, the faster train passes the slower
the mother’s age at that time. The present age of the one in 18 s but when they are running in opposite
mother and son, respectively are directions with the same speeds as earlier, they pass
each other in 9 s. Find the speed of faster train
(a) 36 yrs, 9 yrs (b) 39 yrs, 6 yrs (a) 5 m/s (b) 10 m/s
(c) 42 yrs, 12 yrs (d) 25 yrs, 10 yrs (c) 15 m/s (d) 20 m/s
@BEST300MCQ For More Study Material
376
Visit: studyiq.com
Quantitative Aptitude

Set-2: Solutions
1. (a); M.P. = Rs. 100 = 200 : 180 : 171
S.P. = 80 So, when A runs 200 m → C runs 171 m
80×100 800 171
𝐶. 𝑃. = = When A runs 1 m → runs m
90 9 200
800
𝑃𝑟𝑜𝑓𝑖𝑡 = 95 − When A runs 400 m → C runs
9 171×400
855−800 = = 342 𝑐𝑚
200
9
800 A can beat C by 400 – 342 = 58 m
95−
%𝑝𝑟𝑜𝑓𝑖𝑡 = 800
9
× 100% 6. (b); Let total number of men = 𝑥
55
9 and total number of women = 𝑦
55 45𝑥
= 9
800 × 100 = % = 6.9% ∴ Number of married men =
8 100
9 25𝑦
(𝑥−1)(𝑥+1) 9 and number of married women =
2. (d); (𝑥+2)(𝑥−1) = 100
10
A.T.Q,
10𝑥 + 10 = 9𝑥 + 18 45𝑥 25𝑦 9𝑥
𝑥=8 = ⇒ 𝑦= … (𝑖)
100 100 5
3. (a); 𝑥 =principle (say) Also,
45𝑥 25𝑦
Time = n = 1 yrs. 𝑇𝑜𝑡𝑎𝑙 𝑛𝑢𝑚𝑏𝑒𝑟 𝑜𝑓 𝑚𝑎𝑟𝑟𝑖𝑒𝑑 𝑎𝑑𝑢𝑙𝑡𝑠 = +
100 100
r = Rate of interest (annually) 9𝑥 9𝑥
= + [𝑓𝑟𝑜𝑚 𝑒𝑞. … (𝑖)]
ATQ, 20
9𝑥
20
𝑟 1 𝑟 =
𝑝𝑥 = 𝑥 [1 + ] ⇒𝑝=1+ 10
100 100
𝑟 𝑛 𝑟 𝑛 and total population in city = 𝑥 + 𝑦
𝑞𝑥 = 𝑥 [1 + ] ⇒ 𝑞 = (1 + ) 9𝑥
= 𝑥 + [𝑓𝑟𝑜𝑚 𝑒𝑞. … (𝑖𝑖)]
100 100 5
⇒ 𝑞 = 𝑝𝑛 ⇒ 𝑙𝑜𝑔 𝑞 = 𝑛 𝑙𝑜𝑔 𝑝 14𝑥
𝑙𝑜𝑔 𝑞 =
5
⇒𝑛= 9𝑥
𝑙𝑜𝑔 𝑝
4. (a); ∴ 𝑅𝑒𝑞𝑢𝑖𝑟𝑒𝑑 𝑝𝑒𝑟𝑐𝑒𝑛𝑡𝑎𝑔𝑒 = 10
14𝑥 × 100 = 32.14%
5
Alternate Solution:
45𝑥 25𝑦 𝑦 9
= ⇒ =
100 100 𝑥 5
As we know that the ratio of x: y = 5:9
Let us assume the value of x and y as 100 and
180.
Then no of married men: women= 45 and 45
and
𝐴𝐶
= 𝑐𝑜𝑡 30° No of unmarried men : women = 55 and 135
𝐴𝐵
AC =√3 h Total married adults = 90
𝐴𝐷 Total population = 280
= 𝑐𝑜𝑡 45° 90
𝐴𝐵 Percentage of married population= ∗ 100
AD = h 280
= 32.14%
(√3ℎ − ℎ) units travelled in 12 minutes
12×ℎ 7. (a); Let the total number of candidates = 𝑥
ℎ 𝑢𝑛𝑖𝑡𝑠 𝑡𝑟𝑎𝑣𝑒𝑙𝑙𝑒𝑑 𝑖𝑛 = Number of candidates who answered all the 5
(√3ℎ−ℎ)
5𝑥
=
12
×
√3+1
= 6(√3 + 1) questions =
100
√3−1 √3+1
= 16.39 𝑚𝑖𝑛𝑢𝑡𝑒𝑠 Also, Number of candidates who answered not a
5𝑥
or single question =
100
= 16 minutes 23 seconds ∴ 𝑟𝑒𝑚𝑎𝑖𝑛𝑖𝑛𝑔 𝑠𝑡𝑢𝑑𝑒𝑛𝑡𝑠 = 𝑥 − ( + ) =
5𝑥 5𝑥 9𝑥
5. (c); In 600 m race, ratio of distances 100 100 10
𝐴 : 𝐵 ⇒ 𝑁𝑢𝑚𝑏𝑒𝑟 𝑜𝑓 𝑐𝑎𝑛𝑑𝑖𝑑𝑎𝑡𝑒𝑠 𝑤ℎ𝑜 𝑎𝑛𝑠𝑤𝑒𝑟𝑒𝑑 𝑜𝑛𝑙𝑦
9𝑥 25 9𝑥
600 : 540 𝑜𝑛𝑒 𝑞𝑢𝑒𝑠𝑡𝑖𝑜𝑛 = × =
10 100 40
10 : 9 Number of candidates who answered four
In 500 m race, ratio of distances questions
𝐵 : 𝐶 9𝑥 20 9𝑥
500 : 475 = × =
10 100 50
20 : 19 Given, number of candidates who answered
∴ A : B : C = (10 × 20) : (9 × 20) : (19 × 9) either two questions, or three questions = 396

@BEST300MCQ For More Study Material


377
Visit: studyiq.com
Quantitative Aptitude

5𝑥 5𝑥 9𝑥 9𝑥 Use allegation ⇒
⇒𝑥−( + + + ) = 396
100 100 40 50
10+10+45+36
⇒𝑥−( ) 𝑥 = 396
200
200−101
⇒ 𝑥( ) = 396
200
396×200
⇒𝑥= = 800
99
8. (a); Distance between Arun and Bhaskar at 7 : 30 am
1
= 8 × 1 = 12 𝑘𝑚
2
Time taken by Bhaskar to cover a distance of 12
12
km = (12−8) = 3 hours
∴ Required time = 10 : 30 am 1 1
9. (b); Ratio of total capital of A and B Required ratio = : =7∶5
10 14
= 20000 × 12 : 35000× 12 14. (c);
= 240000 : 420000 = 4:7 Days Let total work efficiency
Now C gives 220000 to both to make the capital A 6 10
equal. B 12 5
∴ A’s capital : B’s capital C 15 4
4 1 7 1 1 60 15
= ( − ) : ( − ) = 1:10 𝑜𝑓 𝑡𝑜𝑡𝑎𝑙 𝑤𝑜𝑟𝑘 = =
11 3 11 3 8 8 2
15 15
∴ Required ratio of divided amount = 1 : 10 Time taken by C to complete = = days
2×4 8
10. (d); 7
Rest of work = × 60
8
7×60 28
Time taken by A and B = = days
8×15 8
1
= 3 days
2
Work done by A in 4 days = 3 × 4 = 12 15. (d); Go hit and trial method
Work done by B in 18 days = 2 × 18 = 36 The number will be 24
Remaining work = 60 – (12 + 36) = 12 16. (d); Price of 10 chairs = Price of 4 tables
Work done by C in 18 days = 12 𝑃𝑟𝑖𝑐𝑒𝑜𝑓 1 𝑐ℎ𝑎𝑖𝑟𝑠 2
= ⇒
Time required by C to complete work 𝑃𝑟𝑖𝑐𝑒𝑜𝑓 1 𝑡𝑎𝑏𝑙𝑒𝑠 5
18
= × 60 = 90 𝑑𝑎𝑦𝑠 𝑙𝑒𝑡𝑝𝑟𝑖𝑐𝑒𝑜𝑓 1 𝑐ℎ𝑎𝑖𝑟𝑎𝑛𝑑 1 𝑡𝑎𝑏𝑙𝑒𝑖𝑠 2𝑎𝑎𝑛𝑑 5𝑎.
12 Now, ATQ,
11. (d); Let my age = M 15(2𝑎) + 2(5𝑎) = 4000
Father’s age = F = M + 30 40𝑎 = 4000
Sister’s age = S 𝑎 = 100
Mother’s age = S + 25 Total price of 12 chairs and 3 tables
𝑆+𝑆+25+𝑀+𝑀+30
= 26.25, 𝑆 + 𝑀 = 25 … (𝑖) = 12(2a) + 3(5a) = 39a = 3900
4
𝑆+29+𝑀+4+𝑀+34 107 17. (b); Suppose the distance be 270 km (LCM of 54 &
=
3 3
45)
S + 2M = 40 ... (ii) 270
From (i) and (ii) Without stoppage, time taken = = 5 hrs
54
270
M = 15, F = 45 With stoppage, time taken = = 6 hrs
45
12. (a); Let number of goods is 100 and cost price of 6−5 1
∴ Stoppage per hour = = ℎ𝑟𝑠 = 10 min
each article is 100x 6 6
Now ATQ, 18. (b); Let average salary of all employees is A the
MRP of each goods = 160x average salary of All women is (A + 13) and
Profit at 50y goods = 60x average salary of male employees (A - 7)
Profit at 25y goods = (160𝑥 ×
90
− 100𝑥) Now use allegation
100
= 44𝑥
Profit at rest 25y goods
90 92
= (160𝑥 × × − 100𝑥) = 32.48 𝑥
100 100
50×60𝑥+25×44𝑥+25×32.48𝑥
= total profit = = 49.12%
100𝑥
13. (a);

91
Total employees = × 20 = 260
7

@BEST300MCQ For More Study Material


378
Visit: studyiq.com
Quantitative Aptitude
1
19. (b); Let time taken by C to complete the work alone ∴ 2𝑥 − 3𝑦 = (2𝑥)
5
= 𝑥 days 4 8
Then time taken by B to complete the work ⇒ (2𝑥) = 3𝑦 ⇒ 𝑦 = 𝑥
5 15
𝑥 Profit of company
alone = days
3 8 13𝑥
1 B = 3𝑥 − 4𝑦 = 3𝑥 − 4 × 𝑥 =
1 1 = 15 ⇒ 𝑥 = 60 15 15
+
𝑥 𝑥
3
Hence the ratio of the profits of the two
⇒ time taken by B alone to complete the work companies are
= 20 days. 2
𝑥:
13𝑥
= 6 ∶ 13
Again let A take 𝑦 days to complete the work 5 15
1 2
alone 25. (b); Milk : water in first glass = ∶ = 4 ∶ 8
3 3
1 1 3
𝑦 = 3 (1 1 ) ⇒ 𝑦 = 40 days And Milk : Water in second glass = ∶ = 3 ∶ 9
+ 4 4
𝑦 20
20. (d); Let Sashi’s Income = 100𝑥 Milk in the vessel = 4 + 3 = 7
36% of her income spends on various expenses Water in the vessel = 8 + 9 = 17
then remaining = 64𝑥 Ratio of milk and water in the vessel = 7 : 17
1th
of income spends on her brother = 8𝑥 26. (d); Total quantity of milk
8
3th = 2 × 0.9 + 5 × 0.8 + 9 × 0.7 = 12.1 litre
of income spends on her sister = 24𝑥
8 Milk concentration in the resultant mixture
Then saving = 32𝑥 12.1
24𝑥 − 8𝑥 = 3280 = × 100 = 75.625%
2+5+9
𝑥 = 205 Water concentration in the resultant mixture
Saving = 32𝑥 = 6560 = 100 – 75.625% = 24.375%
21. (b); Let, Average age of 11 member hockey team = x 75625
years ⇒ 𝑀𝑖𝑙𝑘 ∶ 𝑊𝑎𝑡𝑒𝑟 = = 121 ∶ 39
24375
⇒ Total age of hockey team = 11x years. 27. (d); Let each day’s salary = Rs. x
When captain aged 26 yrs and goalkeeper 𝑥
Given, 18𝑥 + 8 × − 60 = 1700
Age 26 + 3 = 29 yrs. Are excluded. 2
1760
Total age of remaining 9 players ⇒𝑥=
22
= 11𝑥 − (26 + 29) = (11𝑥 − 55) yrs. 1760
⇒ 𝑀𝑜𝑛𝑡ℎ𝑙𝑦 𝑆𝑎𝑙𝑎𝑟𝑦 = × 30 = 2400
Now, ATQ, 22
11𝑥−55 𝑆𝑢𝑚 𝑜𝑓 𝑠𝑝𝑒𝑒𝑑
=𝑥−1 28. (b); Distance = Difference ×
9 𝐷𝑖𝑓𝑓𝑒𝑟𝑒𝑛𝑐𝑒 𝑖𝑛 𝑠𝑝𝑒𝑒𝑑
𝑜𝑟, 11𝑥 − 55 = 9𝑥 − 9 = 2𝑥 = 46 = 165 ×
155
= 1705 km
⇒ 𝑥 = 23 𝑦𝑒𝑎𝑟𝑠 15
7
22. (a); Let the present ages of mother and son be x 29. (c); Since the car runs at th of its own speed, the
11
years and (45 – x) years respectively. 11
time it takes will be th of its usual speed.
Then, (𝑥 − 5)(45 − 𝑥 − 5) = 4(𝑥 − 5) 7
⇒ 𝑥 2 − 41𝑥 + 180 = 0 ⇒ 𝑥 = 36 Let the usual time taken by t h.
∴ The present ages of mother and son are 36 yrs Then we can write,
11
𝑡 = 22
7
& 9 yrs. respectively. 22×7
23. (a); Let the four parts into which 3150 is divided are ∴𝑡= = 14 h
11
a, b, c and d. ∴ Time saved = 22 – 14 = 8 h
𝑎 𝑏 𝑐 𝑑
⇒ = = = =𝑘 30. (c); The speeds of the trains be a m/s and b m/s.
2 3 4 12
Then a = 2k, b = 3k, c = 4k and d = 12k Now, when they are moving in the same
As a + b + c + d = 3150 direction.
⇒ (2𝑘 + 3𝑘 + 4𝑘 + 12𝑘) = 3150 ⇒ Relative speed = (a – b) m/s
⇒ 21𝑘 = 3150 ⇒ 𝑘 = 150 100+80
Hence the four parts are 300, 450, 600, 1800 ∴𝑎−𝑏 = = 10
18
So, the largest part is 1800 Similarly a + b =
100+80
= 20
24. (a); Let the total monthly sales of companies A and B 9

be Rs. 2x and Rs. 3 x and their total monthly By solving Eq. (i) and Eq. (ii), we get
expenditure be Rs. 3y &Rs. 4y. a = 15 m/s and b = 5m/s
Given that A’s profit = 1/5 of sales = 2x/5 ⇒ Speed of faster train = 15 m/sec.

@BEST300MCQ For More Study Material


379
Visit: studyiq.com
Quantitative Aptitude

1. Raman can do a work in 5 days, Jatin can do the 9. A shopkeeper earns a profit of 25% after selling a
same work in 7 days and Sachin can do the same book at 20% discount on the marked price. The ratio
work in 9 days. If they do the same work together of marked price to the cost price is
and they are paid Rs 5720, then what is the share (in (a) 23 : 16 (b) 25 : 16
Rs) of Raman? (c) 23 : 14 (d) 25 : 14
(a) 2520 (b) 1260 10. A man has 3 sons, 2 daughters and a wife. They
(c) 2600 (d) 2460 divided a sum of Rs. 23000 among themselves such
2. After deducting 60% from a certain number and that each daughter got 2.5 times the amount
then deducting 15% from the remainder, 1428 is received by each son and his wife received 400 less
left. What was the initial number? than each son. What is the total amount (in Rs.)
(a) 4200 (b) 3962 received by the 3 sons together ?
(c) 4150 (d) 4300 (a) 7800 (b) 6900
3. A local train travels 40% faster than a car. Both start (c) 6300 (d) 5700
from point A at the same time and reach point B, 140 11. The average of 3 consecutive even numbers is A. If
km away at the same time. On the way the train next 6 even numbers are added, then what is the
takes 25 minutes for stopping at the stations. What average of these 9 numbers?
is the speed (in km/hr) of the train? (a) A + 3 (b) A + 4
(a) 67 (b) 134.4 (c) A + 5 (d) A + 6
(c) 145.9 (d) 160 12. While selling an article for Rs. 18450, a person
4. Guru Randhawa lent Rs 10800 to Jasbir for 3 years suffered a loss of 25%. At what price he should have
and Rs 7500 to Kaisey for 2 years on simple interest sold the article (in Rs.) to earn a profit of 25% ?
at the same rate of interest for start a telephone
business and received Rs 1422 in all from both of (a) Rs. 30750 (b) Rs. 55350
them as interest. The rate of interest per annum is (c) Rs. 24600 (d) Rs. 27675
(a) 3.5 percent (b) 4 percent 13. 8% of the voters in an election did not cast their
votes. In this election, there were only two
(c) 3 percent (d) 4.5 percent
candidates. The winner by obtaining 48% of the
5. A thief is stopped by a policeman from a distance of
total votes defeated his contestant by 1200 votes.
150 metres. When the policeman starts the chase,
The total number of voters in the election was :
the thief also starts running. Assuming the speed of
(a) 20000 (b) 30000
the thief as 7.75 km/hr and that of policeman as
(c) 35000 (d) 25000
9.75 km/hr, how far the thief would have run, before
14. A train of length 200 m crosses another train of
he is over- taken by the policeman?(approx) length 150 m, running on a parallel track in the
(a) 420 metres (b) 630 metres opposite direction in 18 seconds. If the speed of
(c) 525 metres (d) 581 metres train having length 150 m is 40 km/hr, then what is
6. P and Q can finish a work in 24 and 12 days the speed (in km/hr) of the other train ?
respectively. In how many days can they complete (a) 30 (b) 48
50% of the work if they work together? (c) 50 (d) 60
(a) 8 days (b) 2 days 15. The compound interest on a sum of Rs. 10,000 at 8%
(c) 4 days (d) 16 days per annum for 9 months when interest is
7. A Mogra rice trader buys 16 quintals of rice for Rs. compounded quarterly is :-
1,280. 18% rice is in lost in transportation. An what (a) Rs. 600 (b) Rs. 612
rate should he sell to earn 23% profit? (c) Rs. 612.08 (d) Rs. 612.48
(a) Rs. 53.33 per quintal (b) Rs. 98.4 per quintal 16. What is the sum of the first 13 terms of an
(c) Rs. 122.5 per quintal (d) Rs. 120 per quintal arithmetic progression if the 5th term is 1 and the
8. A, B and C can complete a work in 40, 48 and 60 8th term is -17?
days respectively. They received Rs. 13200 to (a) -140 (b) 61
complete the work. They begin the work together (c) -143 (d) 166
but B left 2 days before the completion of the work, 17. A fraction becomes 6/5 when 5 is added to its
and C left 5 days before the completion of the work. numerator and becomes 1/2 when 4 is added to its
A has completed the remaining work alone. What is denominator. What will be the double value of the
the share of B (in Rs.) fraction?
(a) 4000 (b) 4400 (a) 8/9 (b) 7/5
(c) 4500 (d) 4860 (c) 7/10 (d) 6/11
@BEST300MCQ For More Study Material
380
Visit: studyiq.com
Quantitative Aptitude

18. Amar can complete a work in 25 days and Pulkit can 25. For an amount, simple interest at the rate of interest
complete the same work in 20 days. Pulkit alone of 12% per annum for 6 years is Rs 25920. What will
worked at it for 10 days and then left the work. In be the compound interest (in Rs) on same amount at
how many days will Amar alone complete the the rate of interest of 8% per annum compounding
remaining work? annually for 2 years?
1 1
(a) 11 (b) 12 (a) 4326.3 (b) 5563.4
2 2
1 1 (c) 5888.6 (d) 5990.4
(c) 13 (d) 14 26. If the square of sum of three positive consecutive
2 2
19. What is the discount percentage offered on a test natural numbers exceeds the sum of their squares
series having marked price Rs 2150 being sold at Rs by 292, then what is the largest of the three
1892? numbers?
(a) 12 % (b) 13 % (a) 5 (b) 6
(c) 14 % (d) 16 % (c) 7 (d) 8
20. Rs 60500 is divided among W, X and Y such that W 27. Ankit can do a piece of work in 6 days working 8
receives 2/9 as much as X and Y together and X hours a day while Banku can do the same work in 4
receives 3/7 as much as W and Y together. What is days working 10 hours a day. If the work has to be
the share of Y (in Rs)? completed in 5 days, so how many hours do they
(a) 29850 (b) 30120 need to work together in a day?
(c) 31350 (d) 37250 (a) 4 (b) 5
4

21. The average age of a dancing mob of 6 girls is x 11


4 4
years. Four new girls having ages x – 2, x + 2, x + 4 (c) 6 (d) 4
11 11
and x + 6 joins the mob. What is the new average age 28. A cloths merchant allows 25% discount on the
(in years) of the mob? marked price of an article and he suffered a loss of
(a) x + 1 (b) x + 2 15%. What will be the profit percent if the article is
(c) 2.5x (d) x + 2.5 sold at marked price?
22. How much water (in litres) must be added to 80 (a) 11.76 (b) 12.12
litres solution of acid and water containing 10% (c) 13.33 (d) 14.28
acid, so that it becomes a 5% acid solution? 29. Three boxes of granary of capacity 24 kg, 36 kg and
(a) 10 (b) 20 84 kg are completely filled with three varieties of
(c) 40 (d) 80 granary wheat flour, Corn flour and rice flour
23. A wholesaler sells a solar cooker to a retailer at a respectively. All the three boxes were emptied and
profit of 5% and the retailer sells it to a customer at the three types of granary were thoroughly mixed
a profit of 10%. If the customer pays Rs. 2,000, what and the mixture was put back in the three boxes.
had it cost (in Rs) the wholesaler? How many kg of wheat flour would be there in the
(a) 1731.6 (b) 3210.6 third box (in kg)?
(c) 1931.6 (d) 2310.6 (a) 10 (b) 12
24. A bus travels 2/5 of a total journey at its usual (c) 14 (d) 16
speed. The remaining distance was covered by bus 30. A group of boys in the picnic has an average weight
at 6/7 of its usual speed. Due to slow speed it of 36 kg. One boy weighing 42 kg leaves the group
reaches its destination 50 minutes late. If the total and another boy weighing 30 kg joins the group. If
distance is 200 kms, then what is the usual speed (in the average now becomes 35.7 kg, then how many
km/hr) of bus? boys are there in the group?
(a) 20.57 (b) 24 (a) 30 (b) 32
(c) 28 (d) 26.52 (c) 40 (d) 56

Set-3: Solutions
1. (a); We know, the wages are divided in the ratio of Now,
the work efficiency (63 + 45 + 35) units → 5720
5720
∴ 63 units → × 63 = 2520
143
∴ Raman’s share = Rs 2520
2. (a); Let the initial number be x
∴ x × 0.4 × 0.85 = 1428
1428
⇒𝑥= = 4200
0.4×0.85

@BEST300MCQ For More Study Material


381
Visit: studyiq.com
Quantitative Aptitude
𝑀𝑃
3. (b); Let speed of the car be x km/h ∴ 𝑅𝑒𝑞𝑢𝑖𝑟𝑒𝑑 𝑟𝑎𝑡𝑖𝑜 =
𝐶𝑃
Then the speed of the train will be 1.4x km/h 100 25
∴ ATQ, = × 125 =
80×100 16
140

140
=
25

140×1.4−140
=
25 10. (a); Let the amount received by each son be Rs. x
𝑥 1.4𝑥 60 1.4𝑥 60 ATQ,
672
⇒ 1.4𝑥 = ⇒ x = 96 km/h 3x + 2 × 2.5x + (x – 400) = 23000
5
∴ Speed of train = 1.4x ⇒ 3x + 5x + x – 400 = 23000
= 1.4 × 96 = 134.4 km/h ⇒ 9x = 23400 ⇒ x = 2600
10800×𝑅×3 7500×𝑅×2
4. (c); + = 1422 ∴ Total amount by 3 sons together = 3 × 2600 =
100 100
Rs. 7800
324R + 150R = 1422
11. (d); Let the 3 consecutive even numbers be x –2, x, x
R= 3%
+2
5. (d); Time taken to catch the thief 𝑥 –2+𝑥+𝑥+2
150 ∴ 𝐴𝑣𝑒𝑟𝑎𝑔𝑒 =𝐴 ⇒ x=A
= 5 = 270 sec 3
(9.75−7.75)×
18 ∴ Area of 9 numbers
5 (𝑥 –2)+𝑥+(𝑥+2)+(𝑥+4)+(𝑥+6)+(𝑥+8)+
required distance = 270 × 7.75 ×
18 (𝑥+10)+(𝑥+12)+(𝑥+14)
= 581.25 metres(approx) 9
3𝑥+6𝑥+54
6. (c); = =𝑥+6=𝐴+6
9
100
12. (a); 𝐶. 𝑃. 𝑜𝑓 𝑡ℎ𝑒 𝑎𝑟𝑡𝑖𝑐𝑙𝑒 = 18450 × = 𝑅𝑠. 24600
75
For 25% profit,
Time to complete 50% of work 125
24 𝑆𝑃 = 24600 × = 𝑅𝑠. 30750
12 100
= 2
= = 4 days 13. (b); Let the total number of votes be 100
(1+2) 3
7. (d); CP = Rs 1280 No. of uncast votes = 8
P% = 23% ∴ No. of votes polled = 92
SP = Rs 1574.4 No. of votes obtained by the winner = 48
𝑆𝑃 1574.4
= 82 = 𝑅𝑠 120 per quintal ∴ No. of votes obtained by loser = 92 – 48 = 44
𝑞𝑢𝑖𝑛𝑡𝑎𝑙 ×16
100
8. (b); ⇒ It the difference of win be 4 votes, total voters
= 100
∴ when the difference be 1200 votes,
100
Total voters = × 1200 = 30,000
4
14. (a); Let the speed of other train be x m/s.
⇒ B left 2 days before completion (200+150) 350 5
B’s 2 days work → 5 × 2 = 10 units ∴ 𝐴𝑇𝑄, 5 = 18 ⇒ = 𝑥 + 40 ×
𝑥+40× 18 18
⇒ C left 5 days before completion 350 200
18
150
C’s 5 days work → 5 × 4 = 20 units ⇒𝑥= – =
18 18 18
∴ Now total work = (240 + 10 + 20) units 150 18
∴ 𝑥 (𝑖𝑛 𝑘𝑚𝑝ℎ) = × = 30 𝑘𝑚𝑝ℎ
= 270 units 18 5
∴ Time taken for the work to be completed 15. (c); In one year, there are ‘4’ quarterly months
270
= = 18 𝑑𝑎𝑦𝑠 For 9 months → 3 quarters
6+5+4 8 1
Now 𝑁𝑒𝑤 𝑟𝑎𝑡𝑒 = = 2% =
4 50
A worked for 18 days
B worked for 16 days
C worked for 13 days
∴ Wages will be divided in the ratio
A : B : C = 18 ×6 : 16 × 5 : 13 × 4
A : B : C = 27 : 20 : 13
20
∴ 𝑆ℎ𝑎𝑟𝑒 𝑜𝑓 𝐵 = × 13200 = 𝑅𝑠. 4400
27+20+13
9. (b);

Total CI = (200 + 200 + 200) + (4 + 4 + 4) + 0.08


= Rs. 612.08

@BEST300MCQ For More Study Material


382
Visit: studyiq.com
Quantitative Aptitude

16. (c); T5 = 1 24. (b); Let slow speed = 6x


a + 4d = 1 … (i) And normal speed = 7x
and Time difference will occur only in remaining
T8 = –17 3/5 of distance
a + 7d = –17 … (ii) Let total distance = D
3 3
On solving (i) and (ii) we get, 𝐷 𝐷 50
A = 25 and d = – 6
5
–5 =
6𝑥 7𝑥 60
13 120 120 5
S13 = [2 × 25 + (13– 1)(– 6)] – =
2 6𝑥 7𝑥 6
13 120(7 –6) 5
= [50 – 72] = –143 =
2 42𝑥 6
𝑥 24
17. (b); Let fraction be 𝑥=
𝑦 7
𝑥+5 6 24
= 𝑈𝑠𝑢𝑎𝑙 𝑠𝑝𝑒𝑒𝑑 = × 7 𝑘𝑚 = 24 𝑘𝑚⁄ℎ𝑟
𝑦 5 7
5x + 25 = 6y 25. (d); 12% × 6 = 72%
6y – 5x = 25 … (i) 72% → 25920
25920
and 1% =
𝑥 1 72
= ⇒ 2𝑥 = 𝑦 + 4 … (𝑖𝑖) 100% =
25920
× 100 = 36000
𝑦+4 2
72
Solving (i) and (ii) 𝑅 2
x = 7 and y = 10 𝐶. 𝐼. = 𝑃 [(1 + ) – 1]
100
7 2
so, value of double fraction is 8 27 2
5 = 𝑃 [(1 + ) – 1] = 36000 [( ) – 1]
100 25
18. (b); Let Amar alone can complete the remaining 729–625 104
work in x days. = 36000 [ ] = 36000 × = 𝑅𝑠. 5990.4
625 625
So, 26. (d); Let three consecutive natural numbers be
𝑥 10
+ =1 (x – 1), x, (x +1)
25 20
𝑥 1 So,
=
25 2 [(x – 1) + x + (x + 1)]²
x = 12.5 days = 292 + (x – 1)² + x² + (x + 1)²
2150 –1892
19. (a); 𝑃𝑒𝑟𝑐𝑒𝑛𝑡𝑎𝑔𝑒 𝑑𝑖𝑠𝑐𝑜𝑢𝑛𝑡 = × 100 = 12% 9x² = 292 + x² + 1 – 2x + x² + x² + 1 + 2x
2150
20. (c); W= (𝑋 + 𝑌)
2 9x² = 292 + 3x² + 2
9 6x² = 294
9
X+𝑌 = 𝑊 x² = 49
2
9
𝑆𝑜, 𝑊 + 𝑊 = 60500 x=±7
2
11W = 60500 × 2 so, x =7 since x is +ve natural no.
W = Rs. 11000 largest number is x + 1 = 8
And 27. (d); Total hours taken by A = 8 × 6 = 48 h
3 Total hours taken by B = 4 × 10 = 40 h
𝑋 = (𝑊 + 𝑌)
7 They together will complete whole work in
7𝑋
𝑊+𝑌 = =
48×40
=
240
ℎ𝑜𝑢𝑟𝑠
3
7 88 11
X+ 𝑋 = 60500 If work has to be completed in 5 days then daily
3
240 4
X = Rs. 18150 working hours will be= = 4 ℎ𝑜𝑢𝑟𝑠
5×11 11
Y = 60500 – 11000 – 18150 = Rs. 31350 28. (c); 75% of MP = 85% of CP
21. (a); Total age of 6 girls = 6x 𝑀𝑃 17
6𝑥+4𝑥+10 =
𝑁𝑒𝑤 𝑎𝑣𝑒𝑟𝑎𝑔𝑒 𝑎𝑔𝑒 = =𝑥+1 𝐶𝑃 15
10 2 2×20 40
Profit percent will be= × 100 = = %
22. (d); Let water added = x litre 15 3 3
90
×80+𝑥 95
= 13.33%
100
= 29. (c); Ratio of capacity of boxes = 2 : 3 : 7
80+𝑥 100
72+𝑥 19 2
= Type A which in third box = × 84 = 14 𝑘𝑔
80+𝑥 20 12
1440 + 20x = 1520 + 19x 30. (c); Let number of boys = x
36×𝑥–42+30
x = 80 litre 35.7 =
𝑥
23. (a); Let it cost Rs. ‘x’ to the wholesaler
105 110 35.7x + 36x –12
∴x× × = 2000 0.3x = 12
100 100
x = Rs. 1731.6 x = 40
@BEST300MCQ For More Study Material
383
Visit: studyiq.com
Quantitative Aptitude

1. If the square of sum of three positive consecutive 9. The SI and CI that can be earned by a man in two
natural numbers exceeds the sum of their years at the same rate is Rs 8000 and Rs 8360
corresponding squares by 484 then what is the respectively. If he gives a loan of Rs 15000 to his
largest of the three numbers? friend at the same rate (percent per annum) of
(a) 7 (b) 8
interest at simple interest, how much will he earn
(c) 9 (d) 10
2. A, B and C can complete a work in 10, 12 and 15 from his friend in 3 years?
days respectively. All three of them start together (a) 4000 (b) 4025
but after 3 days A leaves the job and B left the job 2 (c) 4050 (d) 4030
days before the work was completed. C completed 10. A bus travels 2/5 of a total journey at its usual
the remaining work alone. In how many days was speed. The remaining distance was covered by bus
the total work completed? at 6/7 of its usual speed. Due to slow speed it
25 52
(a) days (b) days reaches its destination 50 minutes late. If the total
9 9
26 53 distance is 200 kms, then what is the usual speed (in
(c) days (d) days
9 9
3. An article is listed at Rs 1925. A man purchases it at km/hr) of bus?
two successive discounts of 40% and 60% and (a) 20.57 (b) 24
spends Rs 185 on repairing of article. If he sells the (c) 28 (d) 26.52
article at a profit of 58.5%, then what is the selling 11. On being divided by a three digit number, the
price (in Rs) of the article? numbers 98558 and 93675 leave the same
(a) Rs 647 (b) Rs 825.49 remainder. Find the number and the remainder ?
(c) Rs 847 (d) Rs 1025.49 (a) 223 & 115 (b) 257 & 127
4. If A : B = 3 : 5, C : B = 5 : 2 and C : D = 3 : 2 then what
(c) 233 & 117 (d) None of these
is the value of A+B :B+ C : C - D?
(a) 48 : 105 : 25 (b) 48 : 75 : 75 12. Two men undertook to do a job for Rs. 1960. One of
(c) 48 : 105 : 125 (d) 48 : 75 : 30 them can do it alone in 7 days, and the other in 8
5. A group of girls has an average weight of 34 kg. One days. With the assistance of a boy they together
girl weighing 58 kg leaves the group and another complete the work in 3 days. How much money will
girl weighing 40 kg joins the group. If the average the boy get ?
now becomes 33.4 kg, then how many girls are there (a) Rs. 300 (b) Rs. 325
in the group? (c) Rs. 385 (d) Rs. 425
(a) 40 (b) 30 13. The difference between a discount of 35% on Rs.
(c) 42 (d) 32
1700 and two successive discounts of 26% and 4%
6. A dealer sells two bikes at Rs 14000 each. On one he
gains 20% and on the other he loses 20%. Had he on the same amount is :-
marked up their prices by 25% and give 10% (a) 92.48 (b) 98.68
discount, then what is his profit/loss percentage in (c) 102.68 (d) 104.68
the transaction? 14. Suresh solved 60% of the question in an
(a) No loss & no loss (b) 12.5% loss examination correctly. If out of 71 Questions solved
(c) 10.24% loss (d) 12.5% profit by Suresh, 37 question are correct and out of
7. In an examination 62% student passed in geography remaining every 7 questions, 5 questions have been
and 59% students passed in English. If 6% students solved by Suresh correctly. Then find the total
failed in both the subjects, then what is the
number of questions asked in the examination?
percentage of students who have passed in both the
subjects? (a) 49 (b) 120
(a) 25% (b) 27% (c) 81 (d) 111
(c) 29% (d) 31% 15. Mr. Mehta had ‘n’ apples. He distributed them
1 1 1 1
8. A car starts running with the initial speed of 20 among 4 people in the ratio of ∶ : : . If he gave
2 3 5 9
km/h and its speed increases every hour by 4 km/h.
How many hours will it take to cover a distance of each one a complete apple, the minimum number of
504 km? apples that he had.
(a) 4 (b) 8 (a) 90 (b) 96
(c) 12 (d) 16 (c) 102 (d) 103
@BEST300MCQ For More Study Material
384
Visit: studyiq.com
Quantitative Aptitude

16. Amit sold a car at 5% profit and a bike at 10% profit, 24. The average weight of Pankaj, Qasif and Rose is 45
in whole transaction he gets Rs 15000 as a profit. If kg. If the average weight of Pankaj and Qasif be 36.5
he sold the car at 5% loss and the bike at 15% profit, kg and that of Qasif and Rose be 52 kg, then what is
he gets Rs 5000 as a profit. Find the cost price of 3/7th of the weight (in kgs) of Qasif?
car? (a) 18 (b) 42
(a) 80000 (b) 140000 (c) 36 (d) 28
(c) 160000 (d) 120000 25. A trader sold an article at profit of 20%. Had
17. When price of sugar is increased by 35% a person he bought that article at 60% less price and
gets 14 kg less sugar in Rs 160. Find the new price of sold it at Rs 90 less, then he would have
sugar per kg? gained 50%. What is the value (in Rs) of
(a) Rs 31kg (b) Rs 4 kg cost price?
(c) Rs 5 kg (d) Rs 6 kg (a) 150 (b) 200
18. A train travelling 60% faster than a car. Both start (c) 250 (d) 300
from point A at the same time and reach point B, 160 26. Amit donated 20% of his income to a school
km away at the same time. On the way the train and deposited 20% of the remainder in his
takes 20 minutes for stopping at the stations. What bank. If he is having Rs 12800 now, then
is the speed (in km/hr) of the train ? what is the income(in Rs) of Amit?
(a) 144 (b) 168 (a) 18000 (b) 20000
(c) 198 (d) 288 (c) 24000 (d) 32000
19. The difference between compound interest and 27. Two trains are moving in the opposite
simple interest at the same rate on Rs 10,000 for 2 directions at speed of 43 km/h and 51 km/h
years is Rs 1296. The rate of interest per annum is respectively. The time taken by the slower
(a) 12% (b) 24% train to cross a man sitting in the faster
(c) 36% (d) 48% train is 9 seconds. What is the length
20. What least number must be added to 4131, so that (in metres) of the slower train?
the sum is completely divisible by 19? (a) 235 (b) 338.4
(a) 10 (b) 11 (c) 470 (d) 940
(c) 9 (d) 12 28. A fraction is greater than its reciprocal by (72)/(77).
21. Sinu and Miku do (8/13)thpart of a work What is the fraction?
and the rest of the work was completed by (a) 7/11 (b) 11/7
Anku. If Sinu, Miku and Anku take the same (c) 4/7 (d) 7/4
work for Rs2600, then what is the half part 29. What is the sum of the first 12 terms of an
of share (in Rs) of Anku? arithmetic progression if the first term is -19 and
(a) 1600 (b) 1000 last term is 36?
(c) 800 (d) 500 (a) 192 (b) 230
22. What is the effective discount (in %) on three (c) 102 (d) 214
successive discounts of 20% , 10% and 12% ? 30. Aman is 1.5 times efficient than Banku
(a) 38.64 (b) 34.54 therefore takes 8 days less than Banku to
(c) 35.54 (d) 36.64 complete a work. If Aman and Banku work
23. Profit of Rs 1,03,500 has to be divided between on alternate days and Aman works on first
three partners A, B and C in the ratio 11:7:5. How day, then in how many days the work will
much triple amount (in Rs) does B get? be completed?
(a) 31,500 (b) 49,500 (a) 17 (b) 19
(c) 52,500 (d) 94,500 (c) 19.5 (d) 21

Set-4: Solutions
1. (d); Let three positive consecutive natural numbers 2. (b);
be (x – 1), x & (x + 1)
∴ ATQ,
(3x)2 – [(x – 1)² + x² + (x + 1)²] = 484
⇒ 9x² - [x² + 1- 2x + x² + x² + 1 + 2x] = 484
⇒ 9x² - (3x² + 2) = 484
⇒ 9x² - 3x² - 2 = 484 ⇒ 6x² = 486
Work completed by A, B & C in 3 days
⇒ x² = 81 ⇒ x = 9
= (6 + 5 + 4) × 3 = 45
∴ Largest number = 10
@BEST300MCQ For More Study Material
385
Visit: studyiq.com
Quantitative Aptitude

∴ Remaining work = (60 – 45) = 15 Students failed in both subjects =73


Let the remaining work be completed in x days ∴ No. of students who have passed in both the
ATQ, subjects = 100 - 73 = 27%
4x + 5(x – 2) = 15 8. (c); The distance travelled by the car every hour will
⇒ 4x + 5x – 10 = 15 ⇒ 9x = 25 form an AP having the common difference of 4
25
⇒x= ie 20, 24, 28……..
9
25 52 ∴ ATQ
∴ Total work was completed in 3 + = days n
9 9 ⇒ SN = [2a + (n − 1)d]
2
3. (d); Total CP which the man has to pay n
60 40 ⇒ 504 = [2 × 20 + (n − 1)4] ⇒ n= 12
= 1925 × × + 185 = Rs 647 2
100 100 ∴ Time taken will be 12 hours
Now, 9. (c);
158.5
SP = 647 × = Rs 1025.49
100
4. (a);

360
Rate = × 100 = 9%
4000
Amount he earns from his friend
15000 ×9 ×3
= = Rs 4050
∴ A + B : B + C : C - D = 48 : 105 : 25 100
5. (b); Let the number of girls in the group be x 10. (b); Let slow speed = 6x
ATQ, And normal speed = 7x
33.4x = 34x – 58 + 40 Time difference will occur only in remaining
⇒ 0.6x = 18 ⇒ x = 30 3/5 (120km) of distance
6. (d); In such type of transactions there is always a 3
𝐷
3
𝐷 50
loss of 5
–5 =
6𝑥 7𝑥 60
120 120 5
– =
6𝑥 7𝑥 6
120(7 –6) 5
=
∴ % loss = 42𝑥 6
24
𝑥=
Total SP = Rs 28000 7
24
Loss% = 4% 𝑈𝑠𝑢𝑎𝑙 𝑠𝑝𝑒𝑒𝑑 = × 7 = 24 𝑘𝑚⁄ℎ𝑟
7
So, total CP = = Rs 29166.67 11. (b); Difference between given numbers
Now, he marked up their prices by 25% = 98558 – 93675 = 4883 = 19 × 257
⇒ 98558 = 257 × 383 + 127
New MP = = Rs 36458.34
⇒ So, the number and remainder are 257 and
Now, discount = 10% 127 respectively.
New SP = = Rs 32812.5 12. (c);

Overall Profit = = 12.5%


7. (b); Let the total no. of students be 100

⇒ Boy’s efficiency = Total efficiency – efficiency


of (A + B)
= 56 – (24 + 21) = 11
⇒ For 56 units (A + B + boy) gets Rs. 1960
1960
∴ 11 𝑢𝑛𝑖𝑡𝑠 𝑜𝑓 𝑏𝑜𝑦 = × 11 = 𝑅𝑠. 385
56
13. (c); 𝑅𝑒𝑞𝑑. 𝑑𝑖𝑓𝑓𝑒𝑟𝑒𝑛𝑐𝑒
(100–26) (100–4) (100 –35)
= 1700 × × – 1700 ×
Students failed in geography =38 100 100 100
Students failed in English = 41 = 102.68

@BEST300MCQ For More Study Material


386
Visit: studyiq.com
Quantitative Aptitude

14. (b); Suppose, there are 7x questions, Apart from the 20. (b);
71 questions, then
37 + 5x = 60% (71 + 7x)
3
⇒ 37 + 5x = (71 + 7x)
5
⇒ 5 × (37 + 5x) = 3(71 + 7x)
⇒ 185 + 25x = 213 + 21x
⇒ 4x = 213 – 185
28
⇒x= =7 ∴ Least number to be added = 11
4
∴ Total no. of = 71 + 7x 21. (d); ATQ, efficiency Ratio of 3 persons
Questions = 71 + 49 = 120 (𝑠𝑖𝑛𝑢+𝑀𝑖𝑘𝑢)
=
8
15. (d); LCM of 2, 3, 5, 9= 90 𝐴𝑛𝑘𝑢 5
5
Therefore, required minimum number of apples ∴ Anku’s share = × 2600 = Rs. 1000
1 1 1 1 (8+5)
= × 90 + × 90 + × 90 + × 90
2 3 5 9 So, the half part of share (in Rs) of Anku
= 45 + 30 + 18 + 10 = 103 =1000/2 = 500
16. (b); Let the CP of a car be Rs x and the SP of a bike be 20×10
Rs y. 22. (d); Effective discount = 20 + 10 − = 28
100
ATQ, And again discount = [28 +12 – (28*12)/100]
(1.05x + 1.1y) – (x + y) = 15000
= 36.64
⇒ 0.05x + 0.1y = 15000 … (i) 1,03,500
And, 23. (d); Share of B = × 7 = Rs 31,500
23
(0.95x + 1.15y) – (x + y) = 5000 much triple amount (in Rs) does B get= 3*31500
⇒ 0.15y – 0.05x = 5000 … (ii)
=94500
⇒ Adding eqn. (i) and (ii) we get
0.25y = 20000 24. (a); Let the weight of Qasif be x kg.
y=
20000×100
= 80000 ATQ,
25
Now putting in equation (i), we get 45 × 3 – (36.5 × 2 – x) – (52 × 2 – x) = x
0.05x = 15000 – 0.1 × 80000 ⇒ x = 42 kg
= 15000 – 8000 So, 3/7th of the weight (in kgs) of Qasif
7000×100
x= = 140000 = 42*3/7 = 18
5
17. (b); Or
Sum of age of Pankaj,Qasif and Rose (P+Q+R)
= 135 ... (1)
P+Q =73 ... (2)
Age of R=62 [𝑏𝑦 (1)𝑎𝑛𝑑(2)]
35 7
35% = = Q+R=104
100 20
Now, ATQ, Q=42
7 units → 14 kg So, 3/7th of the weight (in kgs) of Qasif
1 units → 2 kg = 42*3/7 = 18
Initial quantity (27) → 54 kg 25. (a); Let the CP of article be 100
Final quantity (20) → 40 kg
160 ATQ,
∴ New price = = Rs 4 /kg
40
60 3
18. (d); 60% = =
100 5
∴ Let speed of car be 5x and train be 8x
Now, ATQ
160 160 20
– = ⇒ x = 36
5𝑥 8𝑥 60
∴ speed of train = 36 × 8 = 288
19. (c); As we know
R D
=√
100 P
R 1296 81 9 60 Units = Rs. 90
⇒ =√ =√ = 1 unit = Rs. 1.5
100 10,000 625 25
⇒ R = 36% ∴ The CP = 1.5 × 100 = Rs. 150
@BEST300MCQ For More Study Material
387
Visit: studyiq.com
Quantitative Aptitude

26. (b); Let the income of Amit be 100. or


by given options
only option(b) satisfy given condition.
11 7 72
i.e − =
7 11 77
𝑛
29. (c); 𝑆𝑛 = (𝑎 + ℓ)
2
12
= × (−19 + 36) = 6 × 17
2
S12 = 102
30. (b); Let Aman complete the work in x days.
12800 ∴ Banku complete the work in (x + 8) days
∴ Income of Amit = × 100 = Rs. 20,000
64 As Aman is 1.5 times more different than Banku
27. (a); Let the length of slower train be ‘L’ m. ∴ Ratio of the work efficiency
5
ATQ, L = (43 + 51) × × 9 𝐴𝑚𝑎𝑛
=
3
18
𝐵𝑎𝑛𝑘𝑢 2
L = 235 metres Now, finding out the total work
28. (b); Let the fraction be x 3(x) = 2 (x + 8) ⇒ x = 16
1 72 72𝑥
∴𝑥− = ⇒ 𝑥2 − 1 = Hence, total work = 16 × 3= 48 units
𝑥 77 77
⇒ 77x² - 77 – 72x = 0 Cycle is (3 + 2) = 5 units/2days
⇒ 77x² - 72x – 77 = 0 Time taken to complete the whole work
On solving above eqn. = 19 days
11
𝑥=
7

@BEST300MCQ For More Study Material


388
Visit: studyiq.com
Quantitative Aptitude

1. At 9% discount the selling price of a washing 10. Of the 5 numbers whose average is 76, the first is
machine is Rs. 14000, what is the selling price if the 3/7 times the sum of other 4. The first number is
discount is 22%? (a) 171 (b) 114
(a) Rs. 12000 (b) Rs. 9360 (c) 76 (d) 228
(c) Rs. 10202.4 (d) Rs. 13322.4 11. Marked price of an item is Rs 400. On purchase of 1
2. A rice trader buys 22 quintals of rice for Rs. item discount is 6% and on purchase of 4 items
3,344.24% rice is lost in transportation. At what rate discount is 24%. Rachita buys 5 items, what is the
should he sell to earn 30% profit? effective discount?
(a) Rs. 88.86 per quintal (a) 34 percent (b) 28.56 percent
(b) Rs. 197.6 per quintal (c) 12.8 percent (d) 23.25 percent
(c) Rs. 269.2 per quintal 12. A shopkeeper, sold almonds at the rate Rs 1250 per
(d) Rs. 260 per quintal kg and bears a loss of 7%. Now if he decides to sell it
3. A missile travels at 1350 km/h. How many metres at Rs 1375 per kg, what will be the result?
does it travel in one second? (a) 4.6 percent gain (b) 2.3 percent loss
(a) 369 metres (b) 375 metres
(c) 2.3 percent gain (d) 4.6 percent loss
(c) 356 metres (d) 337 metres
13. Madhur works 2 times faster than Sagar. If Sagar can
4. Peeyush has done 1/3rd of a job in 30 days, Sanjiv
complete a job alone in 18 days, then in how many
completes the rest of the job in 60 days. In how days
days can they together finish the job?
can they together do the job?
(a) 5 days (b) 2 days
(a) 15 days (b) 45 days
(c) 30 days (d) 10 days (c) 6 days (d) 4 days
5. The first and last terms of an arithmetic progression 14. The bus fare between two cities is increased in the
are 33 and -57. What is the sum of the series if it has ratio 11:18. What would be the increase in the fare,
16 terms? if the original fare is Rs 550?
(a) –135 (b) –192 (a) Rs 350 (b) Rs 900
(c) –207 (d) –165 (c) Rs 180 (d) Rs 360
6. The average weight of Sopan, Guruprasad and Rima 15. Two students appeared for an examination. One of
is 65 kg. if the average weight of Sopan and them secured 20 marks more than the other and his
Guruprasad be 67 kg and that of Guruprasad and marks were 55% of the sum of their marks. The
Rima be 57 kg, then the weight of Guruprasad is marks obtained by them are
(a) 60 (b) 53 (a) 92 and 72 (b) 83 and 63
(c) 58 (d) 63 (c) 110 and 90 (d) 64 and 44
7. A bank offers 5% compound interest per half year. A 16. A man travels 400 km in, partly by rail and partly by
customer deposits Rs. 9600 each on 1 st January and steamer. He spends 9 hours more time on steamer. If
1st July of a year. At the end of the year, the amount the velocity of the steamer is 30 km/hr and the
he would have gained by way of interest is : velocity of rail is 70 km/hr, how much distance does
(a) Rs. 2928 (b) Rs. 1464 he cover by steamer?
(c) Rs. 732 (d) Rs. 366 (a) 309 km (b) 371 km
8. By increasing the price of entry ticket to a fair in the (c) 464 km (d) 556 km
ratio 3 : 5, the number of visitors to the fair has 17. Raheem sells a machine for Rs 48 lakhs at a loss. Had
decreased in the ratio 7 : 2. In what ratio has the
he sold it for Rs 60 lakh, his gain would have been 5
total collection increased or decreased?
(a) increased in the ratio 10 : 21 times the former loss. Find the cost price of the
(b) increased in the ratio 6 : 35 machine.
(c) decreased in the ratio 21 : 10 (a) Rs 58 lakhs (b) Rs 69.6 lakhs
(d) decreased in the ratio 35 : 6 (c) Rs 42 lakhs (d) Rs 50 lakhs
9. The difference between simple and compound 18. If the number 583_437 is completely divisible by 9,
interests compounded annually on a certain sum of
then the smallest whole number in the place of the
money for 2 years at 16% per annum is Rs 320.
What is the value of given sum (in Rs)? blank digit will be
(a) 25000 (b) 50000 (a) 4 (b) 5
(c) 37500 (d) 12500 (c) 3 (d) 6

@BEST300MCQ For More Study Material


389
Visit: studyiq.com
Quantitative Aptitude

19. Two numbers are 35% and 50% lesser than a third 25. The sum of a non-zero number and thrice its
number. By how much percent is the second number reciprocal is 52/7. Find the number.
to be enhanced to make it equal to the first number? (a) 8 (b) 9
(a) 23.08 percent (b) 15 percent (c) 7 (d) 6
(c) 25 percent (d) 30 percent 26. P and Q can complete a task in 50 and 20 days
20. A bank offers 15% compound interest per half year. respectively. In how many days can they complete
A customer deposits Rs 2400 each on 1st January 70% of the task if they work together?
and 1st July of a year. At the end of the year, the (a) 20 days (b) 30 days
amount he would have gained by way of interest is (c) 40 days (d) 10 days
(a) Rs 2268 (b) Rs 1134 27. A dishonest milkman buys milk at Rs 24 per litre
(c) Rs 567 (d) Rs 283 and adds 1/3 of water to it and sells the mixture at
21. Find two numbers such that their mean proportion Rs 32 per litre. What is his gain?
is 16 and third proportion is 1024. (a) 33.33 percent (b) 50 Percent
(a) 4 and 32 (b) 4 and 64 (c) 77.78 percent (d) 66.67 percent
(c) 8 and 64 (d) 8 and 32 28. A bank offers 15% compound interest per half year.
22. The average weight of Shubha, Govinda and Reshma A customer deposits Rs 7200 each on 1 st January
is 65 kg. If the average weight of Shubha and and 1st July of a year. At the end of the year, the
Govinda be 63 kg and that of Govinda and Reshma amount he would have gained by means of interest
be 70 kg, then the weight of Govinda is is
(a) 41 (b) 71 (a) Rs 6803 (b) Rs 3402
(c) 88 (d) 46 (c) Rs 1701 (d) Rs 850
23. Ticket for an adult is Rs 1500 and a child is Rs 800. 1 29. If the selling price is Rs 728 after getting a discount
child goes free with two adults. If a group has 25 of 9%, what was the marked price?
adults and 12 children what is the discount the (a) Rs 793.52 (b) Rs 800
group gets? (c) Rs 662.48 (d) Rs 667
(a) 26.47 percent (b) 20.38 percent 30. Two cars travel from city A to city B at a speed of 24
(c) 31.60 percent (d) 33.33 percent and 32 km/hr respectively. If one car takes 2.5
24. Prabodh has done 1/2 of a job in 30 days, Sapan
hours lesser time than the other car for the journey,
completes the rest of the job in 45 days. In how
many days can they together do the job? then the distance between City A and City B is
(a) 18 days (b) 48 days (a) 288 km (b) 360 km
(c) 27 days (d) 36 days (c) 240 km (d) 192 km

Set-5: Solutions
14000
1. (a); 𝑁𝑒𝑤 𝑆𝑃 = × 78 = Rs 12000 6. (b); Let the weight of Sopan, guruprasad and Rima
91
130 be S, G and R respectively
3344×
2. (d); SP = 76
100
= Rs 260 per quintal S + G + R = 65 × 3 = 195 … (i)
×22
100 S + G = 2 × 67 = 134
5
3. (b); Speed = 1350 × m/s = 375 m/s G + R = 2 × 57 = 114
18
4. (b); Let the efficiency of peeyush and Sanjiv be P and S + 2G + R = 248 ... (ii)
Substract (i) from (ii)
S respectively.
G = 53
ATQ, 5 2
𝑃×30 𝑆×60 𝑃 1 7. (b); Amount gained = 9600 [(1 + ) − 1] +
1 = 2 ⇒ = 100
𝑆 1 1
3 3 5
1×30 1×60 9600 [(1 + ) − 1] = Rs 1464
Total work = 1 𝑜𝑟 2 = 90 unit 100
3 3 8. (c);
Time to complete the work together
90
= = 45 days
(1+1)
n
5. (b); Sum = (first term + last term)
2
16
Sum = 33  (57) = –192 Hence, total collection has decreased in the ratio
2 21 : 10

@BEST300MCQ For More Study Material


390
Visit: studyiq.com
Quantitative Aptitude

9. (d); Let the sum be 100x. 21. (b); Let the nos. be x & y
CI for 2 years at 16% = 34.56x ATQ, √𝑥𝑦 = 16
SI for 2 years at 16% = 32x xy = 256 … (i)
𝑥 𝑦
CI – SI = 320 & = ⇒ 𝑦 2 = 1024𝑥 … (ii)
𝑦 1024
2.56x = 320 ⇒ x = 125 On solving (i) & (ii) we get
Sum = Rs. 12500 x & y = 4 and 64
10. (b); Let the no. be a, b, c, d, e. 22. (b); Let the weight of Shubha, Govinda and Reshma
ATQ, be S, G and R respectively.
a + b + c + d + c = 76 × 5 = 380 𝑆 + 𝐺 + 𝑅 = 65 × 3 = 195 … (i)
3
& 𝑎 = (𝑏 + 𝑐 + 𝑑 + 𝑒) 𝑆 + 𝐺 = 2 × 63 = 126
7 & G + R = 2 × 70 = 140
7
⇒ 𝑎 + 𝑎 = 380 ⇒ a = 114 ∴ 𝑆 + 2𝐺 + 𝑅 = 266 … (ii)
3
11. (b); 𝐸𝑓𝑓𝑒𝑐𝑡𝑖𝑣𝑒 𝑑𝑖𝑠𝑐𝑜𝑢𝑛𝑡 Subtract (i) from, (ii)
𝐷1 ×𝐷2 6×24 G = 266 – 195 ⇒ G = 71
= 𝐷1 + 𝐷2 – = 6 + 24 – = 28.56%
100 100 23. (b); Fare to be paid if no child goes free
100
12. (c); CP of 1 kg of almond = 1250 × = 𝑅𝑠. 1344 = 25 × 1500 + 12 × 800 = Rs. 47,100
93
Now, on selling it for Rs. 1375 ATQ, 12 children will go free with 24 adults, so
1375 –1344 every child goes free.
𝑃= × 100 = 2.3% ∴ Amount to be paid = 25 × 1500 = Rs. 37,500
1344
13. (c); Discount =
47100−37500
× 100 = 20.38%
47100
24. (d); Let the efficiency of Prabodh and Sapan be P and
S respectively.
ATQ,
𝑃×30 𝑆×45 𝑃 3
No. of days to complete the work together 1 = 1 ⇒ =
18 𝑆 2
2 2
= = 6 𝑑𝑎𝑦𝑠 3×30 2×45
1+2 Total work = 𝑜𝑟 = 180 units
1 1
550
14. (a); 𝐼𝑛𝑐𝑟𝑒𝑎𝑠𝑒 𝑖𝑛 𝑓𝑎𝑟𝑒 = × 7 = 𝑅𝑠. 350 2 2
11
Together they can complete the work
15. (c); Let the marks score by two students be x, 180
= = 36 days
(x + 20) 3+2

ATQ, 25. (c); Let the sum be x.


3 52
55 ATQ, 𝑥 + =
(𝑥 + 20) = (𝑥 + 𝑥 + 20) ⇒ x = 90 𝑥 7
100
⇒ 7𝑥 2 − 52𝑥 + 21 = 0
So, marks obtained by two students are 90, 110.
⇒ (𝑥 − 7)(7𝑥 − 3) = 0 ⇒ x = 7
16. (a); Let the distance covered by streamer be ‘x’ km. 26. (d);
𝑥 (400−𝑥)
ATQ, − =9 ⇒ x = 309 km
30 70
17. (d); ATQ,
5(𝐶𝑃 − 48) = (60 − 𝐶𝑃) 70
Time to complete 70% of work = = 10 𝑑𝑎𝑦𝑠
2+5
5CP – 240 = 60 – CP 27. (c); Let him buy 3 litre of milk.
6 CP = 300 ⇒ CP = 50 lakh Total CP = 3 × 24 = Rs. 72
18. (d); Let the missing terms be (x) 1
Quantity of water added = × 3 = 1 litre
3
For a number to be divisible by 9, his sum of
So, Total SP = (3 + 1) × 32 = Rs. 128
digits should be divisible by 9. 128−72
𝑃% = × 100 = 77.78%
Sum of digits = (30 + x) 72

(30 + x) will be divisible by 9 at x = 6. 28. (b); Amount Gained


15 2 15 1
19. (d); 𝑁1 ∶ 𝑁2 ∶ 𝑁3 = 7200 [(1 + ) − 1] + 7200 [(1 + ) − 1]
100 100
65 : 50 : 100 = 2322 + 1080 = Rs. 3402
65−50 728
Required % = × 100 = 30% 29. (b); MP = × 100= Rs. 800
50 91
20. (b); Amount Gained 30. (c); According to the question,
15 2 15 1 𝐷 𝐷 5 4−3 5
= 2400 [(1 + ) − 1] + 2400 [(1 + ) − 1] − = ⇒𝐷[ ]=
100 100 24 32 2 96 2
5
= 774 + 360 = Rs. 1134 D = 96 × = 240 kms
2

@BEST300MCQ For More Study Material


391
Visit: studyiq.com
Quantitative Aptitude

1. Mansukh can do a piece of work in 10 hours. If he is 10. When a number is increased by 21, it becomes
joined by Jayanti who is 50% more efficient, in what 114% of itself. What is the number?
time will they together finish the work? (a) 150 (b) 252
(a) 2 hours (b) 6 hours (c) 315 (d) 189
(c) 4 hours (d) 3 hours 11. The total cost of a microwave oven with mixer was
2. If the amount received at the end of 2nd and 3rd Rs 36750. The mixer was sold at a profit of 32% and
year at Compound Interest on a certain Principal is the microwave oven at a loss of 22%. If the sale
Rs 25088, and Rs 28098.56 respectively, what is the price was the same in both the items, then the cost
rate of interest? price of the cheaper item was _____.
(a) 6 percent (b) 24 percent (a) Rs 13100 (b) Rs 18375
(c) 12 percent (d) 19 percent (c) Rs 13650 (d) Rs 16850
3. The value of x for which the expressions 19 - 5x and 12. Amar is 2 times as good a workman as Badal and
19x + 5 become equal is ______. therefore is able to finish a job in 15 days less than
(a) 7/12 (b) -7/12 Badal. In how many days can they together complete
(c) 12/7 (d) -12/7 the job?
4. The mean of marks secured by 45 students in (a) 30 days (b) 40 days
division A of class X is 69, 65 students of division B (c) 10 days (d) 20 days
is 65 and that of 50 students of division C is 63. Find 13. If the radius of a circle is increased by 14% its area
the mean of marks of the students of three divisions increases by
of Class X. (a) 28 percent (b) 29.96 percent
(a) 65.5 (b) 64.8 (c) 14 percent (d) 14.98 percent
(c) 64.1 (d) 66.9 14. Among three numbers, the first is twice the second
5. 9/4thof 7/2 of a number is 126, then 7/2th of that and thrice the third. If the average of three numbers
number is _____. is 429, then what is the difference between the first
(a) 56 (b) 284 and the third number?
(c) 72 (d) 26 (a) 412 (b) 468
6. The ratio of present ages of Ratna and Shantanu is (c) 517 (d) 427
5:4. After 19 years the ratio of their ages will be 15. If the amount received at the end of 2nd and 3rd
10:9. What is Ratna's present age? year at compound interest on a certain Principal is
(a) 19 (b) 15 Rs 30250, and Rs 33275 respectively, what is the
(c) 9 (d) 72 rate of interest?
7. The ten's digit of a 2-digit number is greater than (a) 10 percent (b) 5 percent
the units digit by 4. If we subtract 36 from the (c) 20 percent (d) 16 percent
number, the new number obtained is a number 16. When a discount of 20% is given on an African safari
formed by interchange of the digits. Product of both ticket, the profit is 43%. If the discount is 28%, then
the numbers has only four factors. Find the number. the profit is
(a) 37 (b) 18 (a) 71 percent (b) 113.3 percent
(c) 81 (d) 73 (c) 13.6 percent (d) 28.7 percent
8. A thief is stopped by a policeman from a distance of 17. A missile travels at 1440 km/h. How many metres
450 metres. When the policeman starts the chase, does it travel in one second?
the thief also starts running. Assuming the speed of (a) 374 metres (b) 385 metres
the thief as 7 km/h and that of policeman as 12 (c) 394 metres (d) 400 metres
km/h, how far the thief would have run, before he is 18. At 13% discount the selling price of a oven is Rs
over-taken? 18000, what is the selling price if the discount is
(a) 504 metres (b) 756 metres 27.5%?
(c) 378 metres (d) 630 metres (a) Rs 10875 (b) Rs 12288.75
9. Marked price of an item is Rs 900. On purchase of 2 (c) Rs 15000 (d) Rs 16638.75
items discount is 6%, on purchase of 3 items 19. P and Q can do a project in 12 and 12 days
discount is 12%. Rajani buys 5 items, what is the respectively. In how many days can they complete
effective discount? 50% of the project if they work together?
(a) 17.5 percent (b) 9.6 percent (a) 6 days (b) 3 days
(c) 35 percent (d) 37 percent (c) 18 days (d) 1.5 days

@BEST300MCQ For More Study Material


392
Visit: studyiq.com
Quantitative Aptitude

20. To cover a distance of 225 km in 2.5 hours what 25. A sum fetched a total simple interest of Rs. 7728 at
should be the average speed of the car in meters/ the rate of 7% per year in 8 years. What is the sum?
second? (a) Rs 13800 (b) Rs 16560
(c) Rs 11040 (4) Rs 8280
(a) 90 m/s (b) 45 m/s
26. Mother can bake 20 cakes in 4 hours, Mother and
(c) 50 m/s (d) 25 m/s Dadima together can bake 60 cakes in 10 hours.
21. A rice trader buys 8 quintals of rice for Rs 3,600. How many cakes Dadima can bake in 30 hours?
10% rice is lost in transportation. At what rate (a) 30 (b) 75
should he sell to earn 15% profit? (c) 60 (d) 25
(a) Rs 352.1 per quintal (b) Rs 517.5 per quintal 27. If the amount received at the end of 2nd and 3rd year
(c) Rs 575 per quintal (d) Rs 582.3 per quintal at Compound Interest on a certain Principal is Rs
33708, and Rs 35730.48 respectively, what is the
22. Marked price of an item is Rs 200. On purchase of 1
rate of interest?
item discount is 5%, on purchase of 2 items discount (a) 6 percent (b) 3 percent
is 14%. Rajeshri buys 3 items, what is the effective (c) 12 percent (d) 10 percent
discount? 28. A car travels a certain distance at 34 km/h and
(a) 37 percent (b) 26.25 percent comes back at 66 km/h. What is the average speed
(c) 11 percent (d) 30.2 percent for total journey?
23. Among three numbers, the first is twice the second (a) 50 km/hr (b) 39.76 km/hr
(c) 55.12 km/hr (d) 44.88 km/hr
and thrice the third. If the average of three numbers
29. 30% discount is offered on an item. By applying a
is 198, then what is the difference between the first
promo code the customer wins 10% cash back. What
and the third number? is the effective discount?
(a) 216 (b) 297 (a) 40.7 percent (b) 40 percent
(c) 661 (d) 431 (c) 37 percent (d) 13 percent
24. If Gangadhar's salary is 3/11 times of Hari's and
30. Two fractions are such that their product is – 4 and
Shambhu's is 3/4 times of Hari's, what is the ratio of
Gangadhar's salary to Shambhu's? sum is – 32/15. Find the two fractions.
(a) 44:9 (b) 4:11 (a) 6/15, –10/3 (b) 6/5, –10/3
(c) 9:44 (d) 11:4 (c) 7/2, – 8/7 (d) 10/7, 14/5

Set-6: Solutions
S1. (c); 6. (a); Let the present age of Ratna and Shantanu be 5x
Mansukh Jayanti and 4x respectively.
Efficiency  2 : 3 ATQ,
5𝑥+19 10
As Mansukh can do the work in 10 hours =
4𝑥+19 9
∴ total work = 10 × 2 = 20 units 45x + 171 = 40x + 190 ⇒ 5x = 19
They together can finish the work in Ratna’s present age = 5x = 19 years
20
= = 4 hours 7. (d); Let the no. be (10x + y)
(2+3)
ATQ,
2. (c); Let the rate be ‘R%’
25088×𝑅×1 x -y = 4 ... (i)
(28098.56 − 25088) = and,
100
R = 12% (10x + y) - 36 = (10y + x)
3. (a); ATQ, 9x – 9y = 36
19 – 5x = 19x + 5 x -y = 4 ... (ii)
24x = 14 As we know that tens digit is greater than unit
7
𝑥= digit, no. could be 51, 62, 73, 84, 93
12
(45×69)+(65×65)+(50×63) As per the question the product of the no. and
4. (a); Required mean = = 65.5
(45+65+50) no. obtained by reversing its digits has only four
5. (a); Let the no. be ‘x’ factors, which means both the nos. will have to
ATQ, be prime numbers. In the above mentioned
9 7
× × 𝑥 = 126 ⇒ x = 16 numbers only 73 is a prime no. and 37 is also a
4 2
7 7 prime number. .
𝑥 = × 16 = 56
2 2 ∴ The number is 73.
@BEST300MCQ For More Study Material
393
Visit: studyiq.com
Quantitative Aptitude
33275−30250 R×1
8. (d); This distance of 450 m will be covered by 15. (a); = ⇒ R = 10%
30250 100
policeman with relative speed.
16. (d);
Time taken to catch the thief
450
= 5 = 324 Seconds
(12−7)×
18
5
Required distance = 7 × × 324 = 630 meters
18
9. (b); Let the effective discount be x%
By mixture allegation, 102.96 – 80
New Profit % = × 100 = 28.7%
80
5
17. (d); S = 1440 × m/s = 400 metre per second.
18
18000
18. (c); Required SP = (100 × (100 – 27.5)
− 13)
= Rs. 15,000
19. (b);

ATQ.
12−𝑥 2
=
𝑥−6 3 50% 𝑜𝑓 12
36 – 3x = 2x – 12 Required time = (1+1)
= 3 𝑑𝑎𝑦𝑠
5x = 48 ⇒ x = 9.6% 225 5
20. (d); Speed = × = 25 m/s
2.5 18
10. (a); Let the no. be ‘x’ 115
3600×
ATQ, 21. (c); Required rate = 90
100
= Rs 575 per quintal
114 ×8
x + 21 = 𝑥 ⇒ x = 150 100
100 22. (c); Let the effective discount be x%
11. (c); Let the CP of microwave & mixer be x & y
respectively
ATQ,
x + y = 36750 ... (i)
And,
0.78x = 1.32y
𝑥 22
= … (ii) 14−𝑥 1
𝑦 13
∴ = ⇒ 𝑥 = 11%
From (i) & (ii) we get 𝑥−5 2
x = Rs 23100 & y = Rs 13650 23. (a); N1 : N2 : N3
12. (c); 6:3:2
Avg. = 198
198×3
Required difference = (6+3+2) × (6 − 2) = 216
24. (b);
G H S
3 : 11 :
: 4 : 3
12 : 44 : 33
Time to complete the whole work together
30 G : S = 12 : 33 = 4 : 11
= = 10 days 25. (a); ATQ
2+1
14 × 14 𝑃×7×8
13. (b); Increase in Area = 14 + 14 + = 29.96% 7728 = ⇒ P= Rs 13,800
100 100
14. (b); 26. (a); Let the efficiency as mother and Dadima be M
N3 N1 N2 and D respectively.
2 : 1 ATQ,
𝑀×4 (𝑀+𝐷)×10
1 : 3 =
20 60
𝑀 5
2 : 6 : 3 =
𝐷 1
Let the no. be 6x, 3x & 2x Let Dadima can bake ‘x’ cakes in 30 hours.
According to the question, Now,
6𝑥+3𝑥+2𝑥 5×4 1×30
= 429 ⇒ x = 117 =
3 20 𝑥
Difference between N₁& N₃ = 4x = 468. x = 30 cakes

@BEST300MCQ For More Study Material


394
Visit: studyiq.com
Quantitative Aptitude

27. (a); Let the rate be ‘R’. ATQ,


33708 × 𝑅 × 1
(35730.48 – 33708) = xy = –4 … (i)
100 32
R = 6% x+y=– … (ii)
15
2 ×34 ×66
28. (d); Avg. speed = = 44.88 𝑘𝑚/ℎ𝑟. From Eqn (i) and (ii), we get
34 + 66 6 –10
D  D2 if x = , then y =
5 3
29. (c); Effective discount = D1 + D2 - 1 and
100 –10 6
30 × 10 if x = , then y =
= 30 + 10 – 3 5
100 – 10 6
= 37% Hence, the two fractions are and .
3 5
30. (b); Let the two fractions be x and y.

@BEST300MCQ For More Study Material


395
Visit: studyiq.com
Quantitative Aptitude

1. The weights of Mr. Gupta and Mrs. Gupta are in the 9. A and B run a kilometer and A wins by 25 sec. A and
ratio 7 : 8 and their total weight, combined is 120 kg. C run a kilometer and A wins by 275 m. When B and
After taking a dieting course Mr. Gupta reduces by 6 C run the same distance, B wins by 30 sec. The time
kg and the ratio between their weights changes to 5 : taken by A to run a kilometer is
6. So, Mrs. Gupta has reduced by (a) 2 min 25 sec (b) 2 min 50 sec
(a) 2 kg (b) 4 kg (c) 2 min 20 sec (d) 3 min 30 sec
(c) 3 kg (d) 5 kg 10. From a point on a circular track 5 km long, A, B and
2. In a class, the number of girls is 20% more than that C started running in the same direction at the same
1
of the boys. The strength of the class is 66. If 4 more time with speeds of 2 km per hour, 3 km per hour
2
girls are admitted to the class, the ratio of the and 2 km per hour respectively. Then on the starting
number of boys to that of the girls is. point all three will meet again after
(a) 1 : 2 (b) 3 : 4 (a) 30 hours (b) 6 hours
(c) 1 : 4 (d) 3 : 5 (c) 10 hours (d) 15 hours
1
3. A car can cover a certain distance in 4 ℎ. If the 11. If a BSES electricity bill is paid from Paytm before
2
1 due date, one gets a reduction of 4% on the amount
speed is increased by 5 km/hr, it would take h less
2 of the bill, by paying a bill before due date a person
to cover the same distance. Find the slower speed of got a reduction of Rs 13. The amount of his
the car. electricity bill was:
(a) 50 km/hr (b) 40 km/hr (a) 125 (b) 225
(c) 45 km/hr (d) 60 km/hr (c) 325 (d) 425
4. A does half as much work as B in one sixth of the 12. Vinugopal decided to donate 5% of his July’s salary
time. If together they take 10 days to complete a on the day of donation he changed his mind and
work, how much time shall B alone take to do it ? donated 1687.50, which was 75% of what he had
(a) 70 days (b) 30 days decided earlier. How much is Vinugopal’s salary?
(c) 40 days (d) 50 days (a) 45000 (b) 50000
5. A and B undertook to do a piece of work for Rs. (c) 44000 (d) 40000
4500. A alone could to it in 8 days and B alone in 12 13. A Raymond’s Trader marked the selling price of an
days. With the assistance of C they finished the work article at 10% above the cost price. At the same time
in 4 days. Then C’s share of the money is of selling he allowed certain discount and suffers a
(a) Rs. 2250 (b) Rs. 1500 loss of 1%. He allowed a discount of:
(c) Rs. 750 (d) Rs. 375 (a) 11% (b) 10%
6. Two friends P and Q stared a business investing (c) 9% (d) 10.5%
amount in the ratio of 5 : 6. R joined them after 6 14. Six-elevenths of a number is equal to 22% of second
months investing an amount equal to that of Q’s number. Second number is equal to the one-fourth
amount. At the end of year 20% profit was earned of third number. The value of the third number is
which was equal to Rs. 98,000. What was the share 2400. What is the 45% of the first number?
of R ? (a) 108.9 (b) 111.7
(a) Rs. 21,000 (b) Rs. 1,05,000 (c) 117.6 (d) None of these
(c) Rs. 17,500 (d) Rs. 22,500 15. In an entrance examination Rockie scored 56%
7. Water is flowing at the rate of 5 km/hr through a marks, perry scored 92% marks and Srifi scored 634
cylindrical pipe of diameter 14 cm into a rectangular marks. The maximum marks of the examination are
tank which is 50 m long and 44 m wide. Determine 875, what are the average mark scored by all the
the time in which the level of water in the tank will three girls together:
22 (a) 929 (b) 815
rise by 7 cm (𝑡𝑎𝑘𝑒 𝜋 = ).
7 (c) 690 (d) 643
(a) 2.1 h (b) 2.0 h 16. A diligent man was engaged on a job for 40 days on
(c) 2.5 h (d) 2.2 h the condition that he will get a wage of Rs. 180 for
8. A water filling pipe P is 5 times faster than second the day he works, but he will have to pay a fine of Rs.
pipe Q. If Q fills a cistern in 30 minutes. How long 20 for each day of his absence. If he gets Rs. 5200
will it take to fill the tank, when both the pipes are at the end of the 40 days, then, he was absent for
kept in operation simultaneous ? how many days?
(a) 7 min. (b) 6 min. (a) 12 days (b) 10 days
(c) 8 min. (d) 5 min. (c) 6 days (d) 8 days
@BEST300MCQ For More Study Material
396
Visit: studyiq.com
Quantitative Aptitude

17. How many kg of pure salt must be added to 40 kg of 24. A pipe can fill a cistern in 12 min and another pipe
4% solution of salt and water to increase it to 12% can fill it in 15 min, but a third pipe can empty it in 6
solution of mixture ? min. The first two pipes are kept open for 5 min in
(a) 2.04 kg (b) 2.64 kg the beginning and then the third pipe is also opened.
(c) 3.14 kg (d) 3.64 kg Number of minutes taken to empty the cistern is
18. The average of the essay-I test scores of a class of 'x' (a) 38 (b) 22
students is 80 and that of 'y' student is 94. When the (c) 42 (d) 45
scores of both the classes are combined, the average 25. In a factory, there are equal number of women and
becomes 86. What is the ratio of x to y? children. Women work for 6 h a day and children for
(a) 6 : 5 (b) 5 : 4 4 h a day. During festival time, the work load goes up
(c) 4 : 3 (d) 7 : 6 by 50%. The government rule does not allow
19. The ages of Angaad, Bankie and Cherry together are children to work for more than 6 h a day. If they are
57 years. Bankie is thrice as old as Angaad and equally efficient and the extra work is done by
Cherry is 12 years older than Angaad. Then, the women, then extra hours of work put in by women
respective age of Angaad, Bankie and Cherry is: every day are
(a) 8, 26, 23 (years) (b) 9, 27, 21 (years) (a) 5 (b) 3
(c) 8, 25, 24 (years) (d) 9, 26, 22 (years) (c) 4 (d) 9
20. If 35% of a number is subtracted from the second 26. The average salary per head of all workers of an
number the second number reduces to its four– institution is Rs. 60. The average salary per head of
fifths. What is the ratio the first number to the 12 officers is Rs. 400. the average salary per head on
second number? the rest is Rs. 56. Then, the total number of workers
(a) 4 : 7 (b) 4 : 9 in the institution is
(c) 5 : 9 (d) 5 : 7 (a) 1030 (b) 1032
21. A student took five papers in an examination, where (c) 1062 (d) 1060
the full marks were the same for each paper. His 27. While climbing a slippery pole which is 15 metre
marks in these papers were in the proportion of 6: 7 high, a monkey climbs 5 metre and slips down 3
: 8 : 9 : 10. In all papers together, the candidate metre respectively in alternate minute. To reach the
obtained 60% of the total marks. Then, the number top of the pole the monkey would take-
of papers in which he got more than 50% marks is (a) 15 minutes (b) 14 minutes
(a) 1 (b) 3 (c) 12 minutes (d) 11 minutes
(c) 4 (d) 5 28. There are four prime numbers written in ascending
22. Three girls Sangeeta, Namrata and Mandip are order. The product of the first three is 385 and that
playing a game by standing on circle of radius 5 m of that of the last three is 1001. The first number is
drawn in a park. Sangeeta throws a ball to Namrata, (a) 5 (b) 7
Namrata to Mandip, Mandeep to Sangeeta. If the (c) 11 (d) 17
distance between Sangeeta and Namrata and 29. Cost of ticket per person for a show for the 1st day is
between Namrata and Mandip is 6 m each, what is Rs. 15, for the second day it is Rs. 7.50 and for the 3rd
the distance between Sangita and Mandip ? day it is Rs. 2.50. Ratio of persons buying the tickets
(a) 7.6 m (b) 8.6 m on the three days is 2 : 5 : 13. Find the average cost
(c) 9.6 m (d) 4.6 m of ticket per person.
23. In the class of 100 students, the number of students (a) 5 (b) 6
passed in English is 46, in Maths is 46 and in (c) 12 (d) Can’t be determined
commerce is 58. The number who passed in English 30. The average age of 3 children in a family is 20% of
and Maths in 16, Maths and commerce is 24 and the average age of the father and the eldest child.
English and commerce is 26, and the number who The total age of the mother and the youngest child is
passed in all subjects is 7. Find the number of the 39 yrs. If the father’s age is 26 yrs, what is the age of
students who failed in all the subjects. the second child?
(a) 13 (b) 8 (a) 20 yrs (b) 18 yrs
(c) 9 (d) 7 (c) 12 yrs (d) Cannot be determined

@BEST300MCQ For More Study Material


397
Visit: studyiq.com
Quantitative Aptitude

Set-7: Solutions
1. (b); Let the weight of Mrs. Gupta = 7𝑥 and C covers the distance of 1 km in (𝑥 + 25 + 30) sec.
Mrs. Gupta = 8𝑥 → If B covers the distance of 1 km in (𝑥 + 25) sec.
Then, ATQ — 7𝑥 + 8𝑥 = 120 then C covers the distance of 1 km in (𝑥 + 55) sec.
𝑥 = 8 kg. Thus, in 𝑥 seconds, C covers the distance of 725 m.
𝑥
Initial weight of Mrs. Gupta = 7𝑥 = 7 × 8 = 56 kg ∴ × 1000 = 𝑥 + 55
725
After taking diet, weight of Mr. Gupta = 56 – 6 ⇒ 𝑥 = 145 𝑠𝑒𝑐. = 2mins 25 sec
= 50 kg 10. (c); A makes one complete round of the circular
50 5
𝑎𝑛𝑑 𝑟𝑎𝑡𝑖𝑜 𝑜𝑓 𝑡ℎ𝑒𝑖𝑟 𝑤𝑒𝑖𝑔ℎ𝑡 = = track in
60 6
Mrs. Gupta reduced weight = 64 – 60 = 4 kg. 5
= = 2 hours
5/2
2. (b); Let number of boys = 𝑥 5 5
𝑁𝑜. 𝑜𝑓 𝑔𝑖𝑟𝑙𝑠 =
120
𝑥 B in hours and C in hours
100 3 2
5 5
ATQ, Hence, req. time = LCM of 2, & hour
120 3 2
𝑥+ 𝑥 = 66 ⇒ 𝑥 = 30 =
𝐿𝐶𝑀 𝑜𝑓 2,5,5
= 10 ℎ𝑜𝑢𝑟
100
𝐻𝐶𝐹 𝑜𝑓 1,3,2
No. of girls = 66 – 30 = 36
So, New ratio = 30 : (36 + 4) = 30 : 40 = 3 : 4 11. (c); By paying the bill before due date a person got a
3. (b); Let slower speed = 𝑥 reduction of 4% which is equal to the 13
𝑥 × 4.5 ℎ𝑟 = (𝑥 + 5) × (4.5 − 0.5) ℎ𝑟. Let the payment of bill = x
4
𝑥 = 40 km/hr. 𝑥 = 13
100
4. (c); Suppose B takes = 𝑥 days 𝑥 = 13 × 25 = 325
1 𝑥
𝐴 𝑡𝑎𝑘𝑒𝑠 = (2 × 𝑥) = 𝑑𝑎𝑦𝑠. 12. (a); Vinugopal’s Salary = 1687.5 ×
100
×
100
6 3
75 5
(A + B) do whole work in 10 days.
1 = 45000
(𝐴 + 𝐵)’𝑠 1 𝑑𝑎𝑦’𝑠 𝑤𝑜𝑟𝑘 =
10 13. (b); Let the cost price is = 100
1 3 1
+ = Marked price = 110
𝑥 𝑥 10
1+3
=
1
⇒ 𝑥 = 40 𝑑𝑎𝑦𝑠. SP = 99
𝑥 10 110 − 99
1 1 1 ∴ Discount % = × 100 = 10%
5. (c); 𝐶’𝑠 1 𝑑𝑎𝑦’𝑠 𝑤𝑜𝑟𝑘 = − ( + ) 110
4 8 12
1 3+2 1 14. (a); Third number = 2400
= −( )= 1
4 24
1
24
1 1
Second number = × 2400 = 600
4
𝐴∶ 𝐵∶ 𝐶= ∶ : =3∶2∶1 6
8
1
12 24 First number × = 22% of 600
11
𝐶’𝑠 𝑠ℎ𝑎𝑟𝑒 = × 4500 = 750 11 22
6 First number = × 600 × = 242
6. (a); Ratio of investment of P, Q & R– 6 100
45 × 242
= 5𝑥 × 12 ∶ 6𝑥 × 12 ∶ 6𝑥 × 6 = 5 ∶ 6 ∶ 3 45% of the first number = = 108.90
100
Total profit = 98000 = 20% of total investment 15. (d); Maximum marks is examination = 875
3
𝑆𝑜, 𝑅’𝑠 𝑠ℎ𝑎𝑟𝑒 = × 98,000 = 21,000 56
14 Rocky’s marks = 875 × = 490
100
7. (b); Let’s ‘𝑡’ time taken to arise the water level by 92
7cm. And perry’s marks = 875 × = 805
100
14 490 +805 + 634
𝑁𝑜𝑤 𝑟𝑎𝑑𝑖𝑢𝑠 𝑜𝑓 𝑃𝑖𝑝𝑒 = = 7 𝑐𝑚 Hence, required average marks =
2 3
⇒ water flow by pipe = volume of tank = 643
7 7 5 7
𝜋× × × 5 × × 𝑡 = 50 × 44 × 16. (b); Let the number of days he was absent be x days.
100 100 18 100
𝑡 = 7200 sec 180 (40 – x) – 20 x = 5200
7200
𝑡= = 2ℎ 7200 – 180x – 20x = 5200
60×60
8. (d); Work done by both the pipes in 1 min. 7200 – 200x = 5200
1 5 6 1 x = 2000/200 = 10 days
= ( + ) = = 𝑝𝑎𝑟𝑡 𝑜𝑓 𝑐𝑖𝑠𝑡𝑒𝑟𝑛.
30 30 30 5 17. (d); Amount of salt in 40 kg of solution = 4/100 * 40
Hence they will fill 1 cistern in 5 minutes.
= 1.6 kg
9. (a); If A covers the distance of 1 km, in 𝑥 sec.
Let x kg of pure salt be added.
B covers the distance of 1 km in (𝑥 + 25) sec.
→ If A covers the distance of 1 km, Then, (1.6 + x)/ (40 + x) = 12/100
then in the same time C cover only 725 meter. 160 + 100x = 480 + 12x
→ If B covers the distance of 1 km in (𝑥 + 88x = 320
25)sec. x = 3.64 kg
@BEST300MCQ For More Study Material
398
Visit: studyiq.com
Quantitative Aptitude

18. (c); 23. (c);

19. (b); Let the age of Angad be x years


Bankie’s age = 3x
Cherry’s age = x + 12
x + 3x + x + 12 = 57 The Venn diagram represents the number of
or, 5x + 12 = 57 ⇒ x = 9 students who passed in the respective subjects.
Angad, Bankie and Cherry’s age are 9, 27, 21 Number of students who passed in one or more
years subjects
20. (a); N2 - 35% of N1 = 4/5 N2
= 11 + 9 + 13 + 17 + 15 + 19 + 7 = 91
N2 - 4/5N2 = 35/100N1
N2/5 = 35/100N1 Number of students who failed in all the
N1/N2 = 1/5 * 100/35 = 4 : 7 subjects = 100 – 91 = 9
21. (c); Let the marks scored in five subjects be 6x, 7x, 24. (d);
8x, 9x and 10x
Total marks in all the five subjects = 40x
40𝑥
Max marks of the five subjects =
0.6
(∴ 40x is 60% of total marks)
∴ max marks in each subject
40𝑥
= = 13.33𝑥
0.6×5
6
Cistern fill in first 5 minutes = (+ 5 + 4) × 5
Hence, percentage in each subject = × 100 = 45 units
13.33𝑥
7𝑥 8𝑥 9𝑥 × 100 10𝑥 × 100
× 100, × 100, and Time required to empty the cistern =
45
13.33 13.33𝑥 13.33𝑥 13.33𝑥
1
Or 45.01%, 52.51%, 60.01%, 67.51% and 75.01%
∴ Number of papers in which he got more than = 45 minutes
50% marks is 4. 25. (b); Let extra hours per day are 𝑥.
22. (c); By
𝑀1 𝐷1 𝐻1
=
𝑀2 𝐷2 𝐻2
𝑊1 𝑊2
𝑥(6+4) 𝑥(6+6+𝑥)
= 1
1 1
2
3
⇒ × 10 = 12 + 𝑥
2
⇒ 15 = 12 + 𝑥 ⇒ 𝑥 = 15 − 12 = 3
26. (b); Let the total number of workers = 𝑥
S, N, M referred for Sangeeta, Namrata and According to the question,
Mandip 60𝑥 = 12 × 400 + 56(𝑥 − 12)
Let OK = 𝑥 m ⇒ KN = 5 – 𝑥 cm ⇒ 60𝑥 − 56𝑥 = 4800 − 672
In ∆OSK
𝑆𝐾 2 = 𝑂𝑆 2 − 𝑂𝐾 2 ⇒ 4𝑥 = 4128
= 52 − 𝑥 2 … (i) ∴ 𝑥 = 1032
In ∆SNK 27. (d); Height of pole = 15 metre.
𝑆𝐾 2 = 62 − (5 − 𝑥)2 … (ii) Speed of climbing = 5 metre/min
∴ From (i) and (ii) Speed of sliding = 3 metre/min
52 − 𝑥 2 = 62 − (5 − 𝑥)2
∴ Distance climbed by monkey in 11 min.
⇒ 25 − 𝑥 2 = 36 − 25 + 10𝑥 − 𝑥 2
⇒ 25 − 11 = 10𝑥 = 5 × 6 − 3 × 5 = 15 𝑚.
14 7
⇒𝑥= = ∴ required time = 11 min.
10 5
28. (a); The four prime numbers are 5, 7, 11, 13 as 5 ×
49 625−49 24
∴ 𝑆𝐾 = √25 − =√ = 𝑚 = 4.8 𝑚 7 × 11 = 385
25 25 5
7 × 11 × 13 = 1001
So, distance between Sangita and Mandip = 4.8 ×
Hence, the first prime number is 5.
2 = 9.6 m
@BEST300MCQ For More Study Material
399
Visit: studyiq.com
Quantitative Aptitude

29. (a); Let no. of persons buying the tickets on the 30. (d); Let the ages of three children in increasing
three days are 2x, 5x, 13x respectively. order be 𝑥1 , 𝑥2 and 𝑥3 yrs.
∴ No. of total tickets bought = 20x Then,
Then from question, 𝑥1 +𝑥2 +𝑥3 20 26+𝑥3
= ( )
3 100 2
Total cost of tickets 𝑥 +𝑥 +𝑥 26+𝑥
= 15 × 2𝑥 + 7.5 × 5𝑥 + 2.5 × 13𝑥 ⇒ 1 2 3= 3
… (i)
3 10
= (30 + 37.5 + 32.5)𝑥 = Rs. 100x Also, 𝑀 + 𝑥1 = 39 … (𝑖𝑖)
∴ average cost of ticket per person From Eqs. (i) and (ii), we cannot determine the
= 100x/20x = Rs. 5 value of x2 .

@BEST300MCQ For More Study Material


400
Visit: studyiq.com
Quantitative Aptitude

1. Rohit bought 20 soaps and 12 toothpastes. He 8. In an assembly election there were only two
marked-up the soaps by 15% on the cost price of contestent, 6% of the voters did not vote and 2% of
each and the toothpastes by Rs. 20 on the cost price the voters pressed NOTA (none of the two) in EVM
of each. He sold 75% of the soaps and 8 toothpastes .If 160 votes were declared invalid and winning
and made a profit of Rs. 385. If the cost of a candidate got 48% of total votes and won election
toothpaste is 60%, the cost of a soap and he got no by 480 votes .Find the number of valid votes .
return on unsold items, what was his overall profit (a)7200 (b) 7360
or loss? (c) 8000 (d) 7500
(a) Loss of Rs. 355 (b) Loss of Rs. 210 9. Suresh can finish a piece of work by himself in 42
(c) Loss of Rs. 250 (d) Loss of Rs. 200
days. Mahesh, who is
1
times more efficient as
2. Two cars are running along the same road. The first 5
one, which is running at the rate of 30 km/h, starts 6 Suresh, requires X days to finish the work by
hours ahead of the second one, which is running at working all by himself. Then what is the value of X?
the rate of 50 km/h. How long will it take the second (a) 25 days (b) 30 days
car to catch up with the first one? (c) 35days (d) 20 days
(a) 6 hrs (b) 9 hrs 10. A plane left 30 min later than its scheduled time to
(c) 12 hrs (d) 15 hrs reach its destination 1500 km away. In order to
3. A car covers total 140 km distance, it covers some reach in time it increases its speed by 250 km/h.
part of the journey at the speed of 6 km/hr and the What is its original speed?
rest part of journey at 10 km/hr. If speed of 6 km/hr (a) 1000 km/h (b) 750 km/h
is replaced by 10 km/hr and speed 10 km/hr (c) 600 km/h (d) 800 km/h
replaced by 6 km/hr, it covers 8 km more distance 11. A person starts going for evening walk every day.
in the same time. Find time taken by the car to cover The distance walked by him on the first day was 4
140 km distance and also find average speed of the km. Everyday he walks half of the distance walked
car. on the previous day. What can be the maximum total
4 𝑘𝑚 7 distance walked by him in his life time?
(a) 8 ℎ𝑟, 5 (b) 15 ℎ𝑟, 30 𝑘𝑚/ℎ𝑟
9 ℎ𝑟 9
7 𝑘𝑚 9 (a) 36 km (b) 240 km
(c) 18 ℎ𝑟, 7 (d) 10 ℎ𝑟, 10 𝑘𝑚/ℎ𝑟 (c) 8 km (d) Data inadequate
9 ℎ𝑟 17
4. The average marks of three batches of 55, 60 and 45 12. Mr. Sanjay Sharma, a businessman had some income
students respectively is 50, 55 and 60 then average in the year 2000, such that he earned a profit of 20%
marks of all student together. on his investment in the business. In the year 2001,
(a) 54.68 (b) 53.33 his investment was less by Rs. 5 lakh but still had
(c) 55 (d) 57.33 the same income (Income = Investment + Profit) as
5. A man borrows Rs 6000 at 10% compound rate of that in year 2000. Thus the profit percent earned in
interest. He pays back Rs 2000 at the end of each year 2001 increased by 6%. What was his
year to clear his debt. The amount that he should investment in year 2000?
pay to clear all his dues at the end of third year is (a) Rs. 10200000 (b) Rs. 10500000
(a) Rs 6000 (b) Rs 3366 (c) Rs. 15050000 (d) Data inadequate
(c) Rs 3060 (d) Rs 3066
13. The ratio of males and females in a village is 7 : 8
6. A water tank is 30 cm long, 20 cm wide and 12 m
respectively and the percentage of children among
deep. It is made of iron sheet which is 3 m wide. The
males and females is 25% and 20% respectively. If
tank is open at the top. If the cost of iron sheet is Rs.
the number of adult females in the village is 235200,
10 per meter. Find total cost of iron required to
what is the total population of village?
build the tank ?
(a) Rs. 6000 (b) Rs. 5000 (a) 367500 (b) 551250
(c) Rs. 5500 (d) Rs. 5800 (c) 594000 (d) 376500
7. Price of sugar is increased by 26%.A man reduced 14. A man gives 20% of his money to his eldest son,
his consumption by 1 kg . But the expenditure 30% of the remaining he gives to his youngest son,
increased 20% . Then find the quantity of sugar 10% of the remaining he gives to a school for poor
bought after increment in price. boys. Still he has Rs. 100.80. Find his total sum.
(a) 21 kg (b) 20 kg (a) Rs. 250 (b) Rs. 300
(c) 18.33 kg (d) 19 kg (c) Rs. 200 (d) Rs. 500

@BEST300MCQ For More Study Material


401
Visit: studyiq.com
Quantitative Aptitude

15. A, B and C enter into partnership by making 23. The fare of a AC Janrath bus is Rs. x for the first five
investments in the ratio 3 : 5 : 7. After a year C kilometers and Rs 13 per km thereafter. If a
invests another Rs. 337600 while A withdrew Rs. passenger pays Rs. 2402 for journey of 187 km.
45600. The ratio of investments, then changes to 24 What is the value of x?
: 59 : 167. How much did A invest initially? (a) 29 (b) 39
(a) Rs. 141600 (b) Rs. 135000 (c) 36 (d) None of these
(c) Rs. 96000 (d) Rs. 45600 24. A sum of Rs. 1000 is lent out partly at 6% and the
14
16. The product of two fractions is and their quotient remaining at 10% per annum. If the yearly income
15
35 on the average is 9.2%. the both parts respectively
is . The greater fraction is are :
24
7 7
(a)
4
(b)
6
(a) Rs. 400, Rs. 600 (b) Rs. 400, Rs. 600
(c)
7
(d)
4 (c) Rs. 200, Rs. 800 (d) Rs. 550, Rs. 450
3 5 25. Zaffer, Tahir and Jamila together can complete a
17. If the digits of the age of Mr. Suman are reversed work in 4 days. If Zaffer and Tahir together can
then the new age so obtained is the age of his wife. complete the work in 24/5 days, Tahir and Jamila
1/11 of the sum of their ages is equal to the together can do it in 8 days, then Tahir alone can
difference between their ages. If Mr. Suman is elder complete the work in:
than his wife then find the ratio of their ages. (a) 16 days (b) 12 days
(a) 5 : 4 (b) 4 : 5 (c) 20 days (d) None of these
(c) 7 : 6 (d) 6 : 5 26. 4 men or 6 women can finish a piece of work in 20
18. The sum of four numbers is 64. If you add 3 to first days. In how many days can 6 men and 11 women
number, 3 is subtracted from the second number, finish the double work?
the third is multiplied by 3 and the fourth is divided (a) 18 days (b) 12 days
by 3, then all the results become equal. What is the (c) 14 days (d) None of these
difference between the largest and the smallest of 27. A and B, working together, can complete a piece of
the original numbers? work in 12 days B and C working together can
(a) 32 (b) 27 complete the same piece of work in 16 days. A
(c) 21 (d) None of these worked at it for 5 days and B worked at it for 7 days.
19. A number is greater than the square of 44 and C finished the remaining work in 13 days. How many
smaller than the square of 45. If one part of the no. is days would C alone take to complete it?
the square of 6 and the number is a multiple of 5, (a) 10 days (b) 24 days
then find the number. (c) 32 days (d) 40 days
(a) 1940 (b) 2080 28. One hundred Men in 10 days do one third of a piece
(c) 1980 (d) None of these of work. The work is then required to be completed
20. 1/5 of a number is equal to 5/8 of the second in another 13 days. On the next day (the eleventh
number. If 35 is added to the first number then it day) 60 more men are employed. How many men
becomes 4 times of second number. What is the must be discharged at the end of the 18th day so that
value of the second number? the rest of the men, working for the remaining time,
(a) 125 (b) 70 will just complete the work in time?
(c) 40 (d) 25 (a) 10 (b) 15
21. A car driver leaves Belgium at 8:30 am and expects (c) 11 (d) 16
29. At the first stop on his rout, a driver unloaded two
to reach a place 300 km from Belgium at 12:30. At
fifths of the packages in his Maruti van. After he
10:30 he finds that he was covered only 40% of the
unloaded another three packages at the next stop,
distance. By how much he has to increase the speed
half of the original number of packages in the Maruti
of car in order to keep up his schedule.
van remained. How many packages were in the
(a) 45 km/h (b) 40 km/h
Maruti van before the first delivery?
(c) 35 km/h (d) 30 km/h
(a) 10 (b) 25
22. A boy started from his house by Atlas bicycle at
(c) 30 (d) 36
10:00 am at a speed of 12 km/h. His elder brother 30. A batsman of Ireland Cricket team in his 17th
started after 1 hour 15 minute by Bajaj scooter innings, makes a score of 85 runs, and thereby,
along the same path and caught him at 1 : 30 pm. increase his average by 3 runs. What is his average
The speed of Bajaj scooter was (in km/h): after the 17th innings? He had never been ‘not out’.
(a) 4.5 (b) 36 (a) 47 (b) 37
2
(c) 18 (d) 9 (c) 39 (d) 43
3

@BEST300MCQ For More Study Material


402
Visit: studyiq.com
Quantitative Aptitude

Set-8: Solutions
1. (a); Let CP of 1 soap = x, CP of 1 toothpaste = 0.6x 8. (a); Let total voters = 100x
Total CP = 20x + 7.2x = 27.2x Total number of voters who voted = 94x
SP of 1 soap = 1.15x, SP of 1 toothpaste Number of valid votes = 94x – 160
= 0.6x + 20 Number of voters who choose NOTA = 2x
15 (1.15x) + 8 (0.6x + 20) – 19.8x = 385 Numbers of voters of winner = 48x
x = 100, total CP = 2720 Rs. Number of loosers votes = 44x – 160
SP of sold outsides = 22.05x + 160 = 2365 Rs. Now, 48x – (44x - 160) = 480
Loss = 2720 – 2365 = 355 Rs 4x = 320 ⇒ x = 80
2. (b); Let the time taken by second car to catch up Number of valid votes = 80 × 94 – 160=7200
with the first one be x hr. 9. (c); Suresh working alone 42 days = 1 unit of work
1
ATQ, Mahesh is times more efficient that Suresh. So
5
50𝑥 − 30(𝑥 + 6) = 0 Mahesh is
6
times as efficient as Suresh. Hence
𝑥 = 9 hr 5
5
3. (c); Let total time of journey is ‘t’ hours and in 140 Mahesh should required th of the time, the time
6
km distance the time for which car travels at 6 taken by Suresh.
kmph is ‘x’ hour Therefore, time taken by Mahesh
6x + 10(t - x) = 140 … (i) = 5/6 × 42= 35 days
Now again after replacing speed 10. (b); Let the original time be T hours and original
10x + 6(t -x) = 148 … (ii) speed be x km/h
1500
Solve (i) and (ii) =𝑇 … (𝑖)
𝑥
t = 18 hours and x = 10 hour 1500 30
140 7 =𝑇− … (𝑖𝑖)
Now average speed of car = = 7 kmph 𝑥+250 60
18 9 Solving equations (i) and (ii), we get
4. (a); Average marks of all batches Speed of plane = x
55×50+60×55+45×60
= = 54.68 = 750 or – 1000 (Not possible)
55+60+45
5. (b); Amount = 6000 ∴ x = 750 km/h
1
Rate = 10% 11. (c); 1st day = 4km, 2nd day = 4 × = 2𝑘𝑚,
6000×10×1 2
𝐹𝑖𝑟𝑠𝑡 𝑦𝑒𝑎𝑟 𝑖𝑛𝑡𝑒𝑟𝑒𝑠𝑡 = = 𝑅𝑠 600 1
100 3𝑟𝑑 𝑑𝑎𝑦 = 2 × = 1𝑘𝑚
2
At the end of first year amount ∴ Total distance S = 4 + 2 + 1 + + … ….
1 1
= 6000 + 600 – 2000 = 4600 2 4
At the end of second year Which is infinite GP with
1
𝐼𝑛𝑡𝑒𝑟𝑒𝑠𝑡 =
4600×10×1
= 460 a = 4, r =
2
100
Now, ∵ 𝑟 < 1
At the second year amount = 4600 + 460 – 2000 𝑎 4 4
= 3060 So, Sum; S = = 1 = 1 = 8𝑘𝑚
1−𝑟 1−
2 2
At the end of third year 12. (b); Investment and Income in 2000 = Rs. x and
3060×100×1
𝐼𝑛𝑡𝑒𝑟𝑒𝑠𝑡 = = 306 Rs.1.20 x
100
Amount at the end of third year = 3060 + 306 Investment and Income in 2001
= Rs 3366 = 𝑅𝑠. (𝑥 − 500000) and Rs. 1.20x
Amount refund in third year = Rs 3366 ∴ Profit = (20 + 6)% = 26%
6. (a); Surface area of tank Income in 2001 = Rs. (x-500000) × (1.00 + 0.26)
= 30 × 20 + 2 × 12 × 20 + 2 × 12 × 30 Thus, 1.26 (x–500000) = 1.20x
= 600 + 480 + 720 = 1800 cm² x = Rs. 10500000
1800 13. (b); Total population of town = 15x
Length of sheet = = 600 𝑐𝑚 𝑁𝑢𝑚𝑏𝑒𝑟 𝑜𝑓 𝑚𝑎𝑙𝑒𝑠 7
3 =
Cost of sheet = 600 × 10 = Rs 6000 𝑁𝑢𝑚𝑏𝑒𝑟 𝑜𝑓 𝑓𝑒𝑚𝑎𝑙𝑒𝑠 8
7. (b); Let initial price of sugar = 100x ∴ Number of males and females = 7x and 8x
Increased price = 126x Number of male children = 25% of 7x
25
Let initial consumption = y kg = × 7𝑥 = 1.75𝑥
100
ATQ, Number of adult females = 8x – 1.6x = 6.4x
120
100𝑥𝑦 × = 126𝑥 (𝑦 − 1) ⇒ 6.4𝑥 = 235200
100 235200
120𝑦 = 126𝑦 − 126 ⇒𝑥= = 36750
6.4
6𝑦 = 126 ⇒ 𝑦 = 21 𝑘𝑔 ∴ Total population of town = 15 × 36750
Reduced consumption = 21 – 1 = 20 kg = 551250

@BEST300MCQ For More Study Material


403
Visit: studyiq.com
Quantitative Aptitude

14. (c); Given, x = 20%, y = 30%, z = 10%, 20. (c); Let the numbers be A and B.
1 5
A = 100.80 Then of A = of B
𝐴×100×100×100 5 8
Required total money = 𝐴 5 5 25
(100−𝑥)(100−𝑦)(100−𝑧) ∴ = × =
100.80×100×100×100 100800 𝐵 8 1 8
= = = 𝑅𝑠. 200 Now : -
80×70×90 8×7×9
15. (a); Let Initial investments Let A = 25x, B = 8x
= 3x, 5x and 7x After one year According to question :-
(3𝑥 − 45600): 5𝑥: (7𝑥 + 337600) = 24: 59: 167 (A + 35) = (B × 4)

3𝑥−45600 24
= ⇒ 𝑥 = 47200 Or, (25x + 35) = 8x × 4
5𝑥 59 ∴𝑥=5
⇒ 𝑖𝑛𝑖𝑡𝑖𝑎𝑙 𝑖𝑛𝑣𝑒𝑠𝑡𝑚𝑒𝑛𝑡 𝑜𝑓 𝐴 = 47200 × 3
∴ 2nd number = 8x = 8 × 5 = 40
= Rs. 141600
𝑎 𝑐 14 21. (d); Distance covered by car in 2 hours
16. (b); Given, × = 𝑎𝑛𝑑 300×40
𝑏 𝑑 15 = = 120 km
𝑎 𝑐 35 100
÷ = Remaining distance = 180 km
𝑏 𝑑 24
𝑎
(where is greater fraction) Remaining time = 2h
𝑏 180
Now multiplying both the equations Required speed = = 90 km/h
2
𝑎𝑐 𝑎𝑑 14 35 120
× = × Speed of car = = 60 km/hr
𝑏𝑑 𝑏𝑐 15 24 2
𝑎2 49 𝑎 7 Required increase in speed = 90 – 60 = 30 km/h
⇒ = ⇒ =
𝑏2 36 𝑏 6 1
7 22. (c); Distance covered by Atlas cycling in 1 hr
⇒ the greater fraction is 1
4
6
17. (d); Let the present age of Mr. Suman = 12 + 12 × = 15 km
4
1
= 10x + y yrs. Elder brother catch the boy = in 2 hr
4
Age of his wife = 10y + x yrs. Now, according to question
ATQ, 15
(𝐵𝑎𝑗𝑎𝑗 𝑆𝑐𝑜𝑜𝑡𝑒𝑟)𝑆𝑝𝑒𝑒𝑑 − (𝐴𝑡𝑙𝑎𝑠 𝐶𝑦𝑐𝑙𝑖𝑛𝑔)𝑆𝑝𝑒𝑒𝑑 = 1
1
(10𝑥 + 𝑦 + 10𝑦 + 𝑥) = (10𝑥 + 𝑦) − (10𝑦 + 𝑥) 2
4
11 15 × 4
1
⇒ (11𝑥 + 11𝑦) = 9𝑥 − 9𝑦 (𝐵𝑎𝑗𝑎𝑗 𝑆𝑐𝑜𝑜𝑡𝑒𝑟)𝑠𝑝𝑒𝑒𝑑 = + 12
9
11 20 56 2
⇒ 𝑥 + 𝑦 = 9𝑥 − 9𝑦 = + 12 = = 18
3 3 3
⇒ −8𝑥 = −10𝑦 23. (c); Required equation,
𝑥 10 5
⇒ = = x + 182 × 13 = 2402
𝑦 8 4
x = 2402 – 2366 ⇒ x = 36
∴x:y=5:4
24. (c);
Age of Mr. Suman = (10 × 5 + 4) = 54 years
Age of wife of Mr. Suman = (10 × 4 + 5)
= 45 years
Required ratio = 54 : 45 = 6 : 5
18. (a); Let the 4 numbers are A, B, C and D.
According to question:-
(A + 3) = (B – 3) = (C × 3) = (D ÷ 3)
Let (A + 3) = (B – 3) = (C × 3) = (D ÷ 3) = k (say)
𝑘
Then, A = (k–3), B = (k+3); C = ( ) , 𝐷 = 3𝑘 1
3 1000 × = 200 & 800.
Also: - A + B + C + D = 64 5
𝑘 25. (b); Zaffer, Tahir and Jamila together can finish
⇒ (𝑘 − 3) + (𝑘 + 3) + ( ) + (3𝑘) = 64
𝐾
3 the work in 4 days.
⇒ 5𝐾 + = 64 ⇒ 16 𝑘 = 64 × 3 Zaffer and Tahir together can do it in days
24
3
5
𝑘 = 12 Tahir and Jamila together can do it in 8 days.
1st number = (𝑘 − 3) = 9 = 𝐴 Therefore, zaffer alone can complete the work in
2nd number = (k + 3) = 15 = B 𝑋𝑌 8×4
𝑘 = days = ( ) days
3rd number = ( ) = 4 = 𝐶 𝑌−𝑋 8−4
3 [Here, X = 4 and Y = 8]
4th number = 3k = 36 = D = 8 days.
So, required answer is = 36-4 =32 So, Tahir alone can complete the work in
19. (d); One part of the no. is the square of 6. 24
×8
𝑋𝑌
⇒ 36 𝑚𝑢𝑠𝑡 𝑏𝑒 𝑝𝑟𝑒𝑠𝑒𝑛𝑡 in the number and =( )days = ( 5 24 ) days
𝑌−𝑋 8−
among the options given, none of the options 5
24
fulfills this criteria. [𝐻𝑒𝑟𝑒, 𝑌 = 8 𝑎𝑛𝑑 𝑋 = ]
5
So, None of these. = 12 days.
@BEST300MCQ For More Study Material
404
Visit: studyiq.com
Quantitative Aptitude

26. (b); Here, a = 4, b = 6, n = 20, c = 6 and d = 11 10 × 3 Man days. Also 100 Men worked for
If a men or b women complete a work in n days first 10 days, 160 Men worked from
then time taken by c men and d women to beginning of 11th day to end of 18th day i.e.
complete the same work for 8 days. Now 18 days are already over
𝑛𝑎𝑏 20 × 4 × 6
=( ) days = ( ) days and 5 more days are required to finish the
𝑏𝑐 + 𝑎𝑑 6 × 6 + 4 × 11
= 6 days. work in total 10 + 13 i.e. 23 days. Let us
When work is double than no. of days= 12 days. assume X men will be discharged at the end
27. (b); Suppose, C alone can do this work in x days of 18th day. Hence (160 – X) Men will work
1
∴ C will do work in 1 day
𝑥
for another 5 days.
Now, work done by (B + C) in 1 day =
1 100 × 10 + 160 × 8 + (160 – X) × 5
1 1
16 = 100 × 10 × 3 ⇒ x = 16 men
∴ Work done by B in 1 day = ( − ) 29. (c); Suppose, there were x packages in the
16 𝑥
1
And, work done by (A + B) in 1 day =
12
Maruti van before deliver.
∴ Work done by A in 1 day =
1
−(
1
− )
1 ∴ After first deliver, the number of packages in
12 16 𝑥
1 1 the Maruti van
= + 2 3
48 𝑥 =𝑥− 𝑥= 𝑥
As per the question, 5 5
Work done by A in 5 days + work done by B in 7 After second delivery, the number of packages
days + work done by C in 13 days = whole work in the Maruti van
1 1 1 1 13 3 3𝑥 − 15
∴ 5( + ) + 7( − ) + = 1 = 𝑥−3=
48 𝑥 16 𝑥 𝑥 5 5
5 5 7 7 13 3𝑥 − 15 𝑥
Or, + + − + =1 ∴ = (Given)
48 𝑥 16 𝑥 𝑥 5 2
26 11 11 26
Or, + = 1, or, =1− ⇒ x = 30.
48 𝑥 𝑥 48
11 22
Or, = , or, x = 24 30. (b); Average score before 17th innings
𝑥 48
∴ C alone would complete this work in 24 days. = 85 - 3 × 17 = 34
∴ Average score after 17th innings
28. (d); Since 100 Men can complete one third work
= 34 + 3 = 37.
in 10 days therefore one third work is 100 ×
10 Man days therefore total work is 100 ×

@BEST300MCQ For More Study Material


405
Visit: studyiq.com
Quantitative Aptitude

1. A can do a work in 8 days, B can do the same work in 10. A shopkeeper marks up his wares by 80% and offers
10 days and C can do the same work in 12 days. If all 20% discount. What will be the selling price (in Rs)
three of them do the same work together and they if the cost price is Rs 450?
are paid Rs 7400, then what is the share (in Rs) of B? (a) 548 (b) 748
(a) 2600 (b) 3000 (c) 848 (d) 648
(c) 2400 (d) 2000 11. What number should be added to each of the
2. If the radius of the cylinder is increased by 25%, numbers 35, 115, 53 and 165, so that the resulting
then by how much percent the height must be numbers are in continued proportion?
reduced, so that the volume of the cylinder remains (a) 10 (b) 12
same? (c) 8 (d) 6
(a) 36 (b) 56 12. A batsman makes a score of 95 runs in the 13th
(c) 64 (d) 46 match and thus increases his average runs per
3. The marked price of an article is 20% more than its match by 4. What is his average after the 13th
cost price. If 5% discount is given on the marked match?
price, then what is the profit percentage? (a) 47 (b) 43
(a) 5 (b) 14 (c) 45 (d) 49
(c) 15 (d) 25 13. A vendor buys bananas at 12 for Rs 50 and sells at 5
4. The average runs conceded by a bowler in 5 for Rs 40. What will be the gain (in %)?
matches is 45 and 15.75 in other 4 matches. What is (a) 82 (b) 72
the average runs conceded by the bowler in 9 (c) 92 (d) 62
matches? 14. Two labourers A and B are paid a total of Rs. 750 per
(a) 15 (b) 32 day. If A is paid 150 percent of what is paid to B,
(c) 35 (d) 53.5 how much (in Rs) is B paid?
5. A person bought pens at 25 for a rupee and sold at (a) 450 (b) 250
15 for a rupee. What is his profit percentage? (c) 300 (d) 500
2 1
(a) 16 % (b) 33 % 15. A man travelled a distance of 50 km in 8 hours. He
3 3
2 travelled partly on foot at 5 km/hr and partly on
(c) 66 % (d) 40% bicycle at 7 km/hr. What is the distance (in kms)
3
6. 80 litre mixture of milk and water contains 10% travelled on foot?
milk. How much milk (in litres) must be added to (a) 20 (b) 25
make water percentage in the mixture as 80%? (c) 15 (d) 30
(a) 8 (b) 9 16. The compound interest earned in two years at 8%
(c) 10 (d) 12 per annum is Rs 4160. What is the sum (in Rs)
7. A bus starts running with the initial speed of 21 invested?
km/hr and its speed increases every hour by 3 (a) 24000 (b) 25000
km/hr. How many hours will it take to cover a (c) 30000 (d) 20000
distance of 252 km? 17. Sum of twice a fraction and 5 times its reciprocal is
(a) 3 (b) 5 7. What is the fraction?
(c) 8 (d) 10 (a) 2/5 (b) 5/4
8. A sum of Rs 400 becomes Rs 448 at simple interest (c) 5/2 (d) 4/5
in 2 years. In how many years will the sum of Rs 550 18. A, B and C can finish a job working alone in 12, 8 and
amounts to Rs 682 at the same rate? 24 days respectively. In how many days they can
(a) 2 (b) 3 finish the job if they worked together?
(c) 3.5 (d) 4 (a) 5 (b) 6
9. A can make a cupboard in 10 days and B can do it in (c) 4 (d) 3
50 days. Along with C, they did the job in 6.25 days 19. What is the effective discount (in %) on two
only. Then in how many days C alone can do the job? successive discounts of 20% and 10%?
(a) 20 (b) 25 (a) 30 (b) 32
(c) 16 (d) 15 (c) 35 (d) 28

@BEST300MCQ For More Study Material


406
Visit: studyiq.com
Quantitative Aptitude

20. Profit of Rs 1,03,500 has to be divided between 26. A, B and C can complete a work in 20, 24 and 30
three partners A, B and C in the ratio 11:7:5. How days respectively. All three of them starts together
much amount (in Rs) does B get? but after 4 days A leaves the job and B left the job 6
(a) 22,500 (b) 49,500 days before the work was completed. C completed
(c) 50,500 (d) 31,500 the remaining work alone. In how many days was
21. The average weight of P, Q and R is 45 kg. If the the total work completed?
average weight of P and Q be 36.5 kg and that of Q (a) 10 (b) 12
and R be 52 kg, then what is the weight (in kgs) of (c) 14 (d) 16
Q? 27. The average of 17 results is 60. If the average of first
(a) 42 (b) 44 9 results is 57 and that of the last 9 results is 65,
(c) 46 (d) 48 then what will be the value of 9th result?
22. A shopkeeper by selling 21 items earns a profit (a) 39 (b) 78
equal to the selling price of 1 item. What is his profit (c) 117 (d) 156
percentage? 28. For an article the profit is 170% of the cost price. If
(a) 5.5 (b) 2.2 the cost price increases by 20% but the selling price
(c) 2 (d) 5 remains same, then what is the new profit
23. What is the value of 10% of 150% of 400? percentage?
(a) 600 (b) 50 (a) 41 (b) 50
(c) 500 (d) 60 (c) 75 (d) 125
1
24. To cover a distance of 144 km in 3.2 hours what 29. A boat goes 15 km upstream and 10 km
2
should be the average speed of the car in downstream in 3 hours 15 minutes. It goes 12 km
meters/second? upstream and 14 km downstream in 3 hours. What
(a) 12.5 (b) 10 is the speed of the boat in still water?
(c) 7.5 (d) 15 (a) 4 (b) 6
25. If in 2 years at simple interest the principal (c) 10 (d) 14
increases by 18%, what will be the compound 30. A person lent certain sum of money at 5% per
annum simple interest and in 15 years the interest
interest (in Rs) earned on Rs 7000 in 3 years at the
amounted to Rs 250 less than the sum lent. What
same rate? was the sum lent (in Rs)?
(a) 1865.2 (b) 2065.2 (a) 1000 (b) 1500
(c) 1965.2 (d) 1765.2 (c) 2400 (d) 3000

Set-9: Solutions
1. (c); Ratio of time taken by them to do work 5. (c); Cost of 25 pens = 1 Rs.
= 8 : 10 : 12 = 4 : 5 : 6 100
𝐶𝑜𝑠𝑡 𝑜𝑓 1 𝑝𝑒𝑛 = 𝑝𝑎𝑖𝑠𝑒 = 4 𝑝𝑎𝑖𝑠𝑒
1 1 1 25
𝑅𝑎𝑡𝑖𝑜 𝑜𝑓 𝑒𝑓𝑓𝑖𝑐𝑖𝑒𝑛𝑐𝑦 = ∶ ∶ = 15 : 12 : 10 Selling price of 15 pens = 1 Rs.
4 5 6
Since share is divided in the ratio of efficiency Selling price of 1 pen =
100
𝑝𝑎𝑖𝑠𝑒
7400 15
𝑆𝑜, 𝑠ℎ𝑎𝑟𝑒 𝑜𝑓 𝐵 = × 12 = 𝑅𝑠. 2400 100
–4
37 40 2
2. (a); Let initial radius = 4r 𝑃𝑟𝑜𝑓𝑖𝑡 % = × 100 =
15
× 100 = 66 %
4 15×4 3
Increased radius = 5r 6. (c); Let x litre of milk is added to get 20% milk
initial volume is πr² h₁ = π × 16 r² h₂ solution
10
Find volume = π × 25 r² × h₂ ×80+𝑥 20 8+𝑥 1
ℎ 25 ∴ 100 = ⇒ =
π 16 r² × h₁ = π25 r² h₂ ⇒ 1 = 80+𝑥 100 80+𝑥 5
ℎ2 16 40 + 5x = 80 + x ⇒4x = 40 ⇒x = 10 litre
So, height have to be decreased by 7. (c); In first hour bus will cover 21 km
9
= × 100% = 36% In second hour it will cover 24 km
25
3. (b); Let CP = 1000x ⇒So, MP = 1200x In third hour it will cover 27 km
95 So,
𝑆. 𝑃. = × 1200𝑥 = 1140 𝑛
100
140𝑥 252 = [2 × 21 + (𝑛 – 1)3]
% 𝑝𝑟𝑜𝑓𝑖𝑡 = × 100 = 14% 2
1000𝑥 504 = 42n + 3n² – 3n ⇒ 3n² + 39n – 504 = 0
4. (b); Total runs conceded = 5 × 45 + 4 × 15.75
n² + 13n – 168 = 0 ⇒ n² + 21n – 8n – 168 = 0
= 225 + 63 = 288
288 n (n + 21) – 8 (n + 21) = 0 ⇒ n = 8, – 21
𝑅𝑒𝑞𝑢𝑖𝑟𝑒𝑑 𝑎𝑣𝑒𝑟𝑎𝑔𝑒 = = 32 So, n = 8 hours
9

@BEST300MCQ For More Study Material


407
Visit: studyiq.com
Quantitative Aptitude

8. (d); 2 year interest = 448 – 400 = 48 21. (a); Let the weight of Q be x kg.
1 year interest = 24 ATQ, 36.5 × 2 + 52 × 2 - 45 × 3 = x
24 550×6×𝑇
𝑅𝑎𝑡𝑒 = × 100 = 6%⇒682– 550 = ⇒ x = 42 kg
400 100
22. (d); Let the SP of 1 item be Rs 1
⇒1320 = 55 × 6 × T ⇒T = 4 years
∴ SP of 21 items = Rs 21
9. (b);
Profit = Rs 1
∴ CP of 21 items = Rs 20
1
∴ Profit % = × 100 = 5%
20
10 150
23. (d); × × 400 = 60
100 100
144 5
A : B : C 24. (a); Avg. Speed = × = 12.5 m/s
3.2 18
Efficiency  5 : 1 : 2
50
25. (b); Let the principal be 100P
C, alone can do the work in = = 25 𝑑𝑎𝑦𝑠 ∴ 18P =
100𝑃×𝑅×2
⇒ R = 9%
2
180 80 100
10. (d); SP = 450 × × ⇒ SP = Rs. 648 ∴ Required
100 100
3
11. (a); Let the no. to be added to each no. be x CI = 7000 × [(1 +
9
) − 1]
35+𝑥 53+𝑥 100
∴ =
115+𝑥 165+𝑥 = 7000 × [(1.09)3 − 1] ⇒ CI = 2065.2
5775 + 165x + 35x + x2 = 6095 + 53x + 115x + x2 26. (c); From the LCM method,
5775 + 200x = 6095 + 168x
32x = 320 ⇒ x = 10
12. (a); Let the average after 13th match be x.
ATQ, (x – 4) × 12 + 95 = 13 × x
12x – 48 + 95 = 13x ⇒ x = 47 So, we have total work = 120
50
13. (c); CP of 1 banana = Rs = 𝑅𝑠. 4.17 4 day’s work by (A + B +C) = (6 + 5 + 4) × 4 = 60
12
40 6 day’s work by alone ‘c’ = 4 × 6 = 24
SP of 1 banana = Rs. = 𝑅𝑠. 8 So, remaining work alone by (B + C)
5
8 – 4.17 120−(60+24)
Gain % = × 100 =91.84 ≈ 92% = =4
4.17 (5+4)
14. (c); So, total days = [4 +6 + 4] = 14 days
A : B 27. (b); The required value
Payment Ratio  3 : 2 = [60 + [(65 − 60) × 9 − (60 − 57) × 9]]
B’s payment =
2
× 750 = 𝑅𝑠. 300 = 60 + 18 = 78
(3+2) Alternate method,
15. (c); Let the distance travelled on foot be ‘D’ km. 9𝑡ℎ 𝑟𝑒𝑠𝑢𝑙𝑡 = [(57 × 9 + 65 × 9) − (60 × 17)] = 78
𝐷 (50–𝐷) 7𝐷+250–5𝐷
ATQ, + =8 ⇒ =8 28. (d); Profit percentage = 170%
5 7 35
170 →𝑃𝑟𝑜𝑓𝑖𝑡
2D + 250 = 280 ⇒ D = 15 km ⇒
2 100 → 𝐶𝑃
8
16. (b); 4160 = 𝑃 × [(1 + ) – 1] So, SP = 100 + 170 = 270
100
104 New CP = 100 + 20 = 120
4160 = P × ⇒ P = Rs. 25000 150
625
So, new profit = × 100 = 125
17. (c); Let the fraction be x. 120
5 29. (c); Total time = Upstream time + Down Stream time
ATQ, 2x + = 7 1
𝑥 15 102 1
2x2 – 7x + 5 = 0 ⇒ + =3 … (𝑖)
(𝑥−𝑦) (𝑥+𝑦) 4
2x2 – 2x – 5x + 5 = 0 12 14
⇒ + =3 … (𝑖𝑖)
2x (x – 1) – 5 (x – 1) = 0 (𝑥−𝑦) (𝑥+𝑦)
(2x – 5) (x – 1) = 0 After solving both equations
5 (𝑥 − 𝑦) = 𝑈𝑝𝑠𝑡𝑟𝑒𝑎𝑚 𝑠𝑝𝑒𝑒𝑑
x=
2 (𝑥 + 𝑦) = 𝑑𝑜𝑤𝑛 𝑠𝑡𝑟𝑒𝑎𝑚 𝑠𝑝𝑒𝑒𝑑
18. (c);
𝑥 → 𝑆𝑝𝑒𝑒𝑑 𝑜𝑓 𝑏𝑜𝑎𝑡 𝑖𝑛 𝑠𝑡𝑖𝑙𝑙 𝑤𝑎𝑡𝑒𝑟

{ 𝑦 → 𝑆𝑝𝑒𝑒𝑑 𝑜𝑓 𝑐𝑢𝑟𝑟𝑒𝑛𝑡
The value of
x = 10 ⇒ y = 4
If they worked together =
24
=4 30. (a); ATQ
6
20×10
Let principal = x
19. (d); Effective discount = 20 + 10 − = 28 So, interest = (x – 250)
100
1,03,500 𝑥×5×15
20. (d); Share of B = × 7 = Rs 31,500 (𝑥 − 250) = ⇒ x= 1000
23 100

@BEST300MCQ For More Study Material


408
Visit: studyiq.com
Quantitative Aptitude

1. Two train 130 m and 110 m long, while going in the 9. A person lent out a certain sum on simple interest
same direction. The faster train take one minute to and the same sum on compound interest at a certain
pass the completely. If they are moving in opposite rate of interest per annum. He noticed that the ratio
direction, they pass each other completely in 3 between the difference of compound interest and
seconds. Find the speed of each train: simple interest of 3 years and that of 2 years is 25 :
(a) 42 m/s, 38 m/s (b) 38 m/s, 36 m/s 8. The rate of interest per annum is:
(c) 36 m/s, 44 m/s (d) None of these (a) 10% (b) 11%
1
2. Five men and 2 boys, working together, can (c) 12% (d) 12 %
2
complete four times as much work per hour as a 10. The average weight of A, B and C is 84 kg. If D joins
man and a boy completes working together. The the group, the average weight of the group becomes
work completed by a man and a boy should be in the 80 kg. If another man E who weights is 3 kg more
ratio: than D Replaces A, Then the average of B, C, D and E
(a) 1: 2 (b) 2: 1 becomes 79 kg. What is the weight of A?
(c) 1: 3 (d) 4: 1 (a) 64 (b) 72
3. A can do a piece of work in 30 days, B in 50 days and (c) 75 (d) 100
C in 40 days. If A assisted by B on one day and by C 11. The wheat sold by a grocer contained 10% low
on the next day alternately, the work will be quality wheat. What quantity of good quality wheat
completed in: should be added to 150 kg of wheat so that the
32 2
(a) 17 days (b) 19 days percentage of low quality wheat becomes 5%?
35 3
31
(c) 16 days
1
(d) 18 days (a) 85 kg (b) 50 kg
37 3 (c) 135 kg (d) 150 kg
4. Kaamran mixes 80 kg of sugar worth of Rs. 6.75 per 12. In a class of 50 students, 23 speak English, 15 speak
kg with 120 kg worth of Rs. 8 per kg. At what rate Hindi and 18 speak Punjabi. 3 speak only English
shall he sell the mixture to gain 20%? and Hindi, 6 speak only Hindi and Punjabi and 6
(a) Rs. 7.50 (b) Rs. 9 speak only English and Punjabi. If 9 can speak only
(c) Rs. 8.20 (d) Rs. 8.85 English, then how many students speak all the three
5. When an amount is distributed amongst 14 girls, languages?
each of them gets Rs.160 more than the amount (a) 1 (b) 2
received by each girl in that condition when the (c) 3 (d) 5
same amount is distributed equally amongst 18 13. When the price of sugar was increased by 32%, a
girls. Find the amount – family reduced its consumption in such a way that
(a) 10080 (b) 10070 the expenditure on sugar was only 10% more than
(c) 5040 (d) 5000 before. If 30 kg per month were consumed before,
6. 20 litres of a mixture of cocktail contains 20% find the new monthly consumption.
alcohol and the rest is water. If 4 litres of water be (a) 42 kg (b) 35 kg
mixed in it, the percentage of alcohol in the new (c) 25 kg (d) 16 kg
mixture will be: 14. One bacterial splits into eight bacteria of the next
1 2
(a) 33 % (b) 16 % generation. But due to environment, only 50% of
3 3
(c) 25%
1
(d) 12 % one generation can produce the next generation. If
2 the seventh-generation number is 4096 million,
7. Ravi Shastri spends on an average Rs. 269.47 for the what is the number in first generation?
first 7 months and Rs. 281.05 for the next 5 months. (a) 1 million (b) 2 million
Find out his monthly salary if he saved Rs. 308.46 (c) 4 million (d) 8 million
during the year. 15. At the start of a seminar, the ratio of the number of
(a) Rs. 400 (b) Rs. 500 male participants to the number of female
(c) Rs. 300 (d) Rs. 600 participants was 3 : 1. During the tea break, 16 male
8. A man borrows Rs. 4000 at 20% compound rate of participants left and 6 more female participants
interest. At the end of each year he pays back Rs. registered. The ratio of the male to the female
1500. How much amount should he pay at the end of participants became 2 : 1. The total number of
the third year to clear all his dues? participants at the start of the seminar was-
(a) 2952 (b) 2548 (a) 112 (b) 48
(c) 1500 (d) 1400 (c) 54 (d) 72

@BEST300MCQ For More Study Material


409
Visit: studyiq.com
Quantitative Aptitude

16. If the selling price of a product is increased by Rs. 24. In an election 20% of the voters did not vote and
162, then the business would make a profit of 17% 120 votes were invalid. Winner got 200 votes more
instead of a loss of 19%. What is the cost price of the than this opponent and secured 41% votes of the
product? total number of votes. What percentage of the votes
(a) Rs. 540 (b) Rs. 450 defeated candidate got out of the total votes cast?
(c) Rs. 360 (d) Rs. 600 (a) 45% (b) 41%
17. On the cash payment that will settle a bill for 250 (c) 36% (d) 47.5%
chairs at Rs. 50 per chair, discount of 20% and 15% 25. A typist types a page with 20 lines in 10 min but
with a further discount of 5% is given. The Cash
leaves 8% margin on left side of page new he has to
payment is-
type 23 pages with 40 lines on each page but leaves
(a) Rs. 8075 (b) Rs. 7025
10% margin. Find the time required to complete this
(c) Rs. 8500 (d) None of these
task.
18. A shopkeeper sold a TV set for Rs. 17,940 with a 1 2
discount of 8% and gained 19.6%. If no discount is (a) 7 hours (b) 7 hours
2 3
allowed, then what will be his gain per cent? 3
(c) 7 hours (d) None of these
4
(a) 25% (b) 26.4%
26. Cost price of 12 oranges is equal to the selling price
(c) 24.8% (d) 30%
of 9 oranges and discount on 10 oranges is equal to
19. At a certain rate of simple interest an amount of Rs.
2000 becomes Rs. 2620 in 5 years. If the rate of profit on 5 oranges. What is the percentage point
interest had been 3% more, the same amount would difference between the profit percentage and
have become- discount percentage?
(a) Rs. 3920 (b) Rs. 2920 (a) 20% (b) 22.22%
(c) Rs. 2530 (d) None of these (c) 16.66% (d) 15%
20. A man borrows Rs. 4000 from a bank at 7 %
1 27. What is the difference between the compound
2
interest and simple interest on a sum Rs. 2200 for a
compound interest. At the end of every year, he pays
Rs. 1500 as part repayment of loan and interest. period of 2 years, if the compound interest is at a
How much does he still owes to the bank after three rate of 30% p.a. and simple interest is at a rate of
such installments? 36% p.a.?
(a) Rs. 123.25 (b) Rs. 125 (a) Rs. 62 (b) Rs. 65
(c) Rs. 400 (d) Rs. 469.18 (c) Rs. 64 (d) Rs. 66
21. Time taken by A to take 5 steps is equal to time by B 28. A sum of Rs. 390200 is to be paid back in three equal
to take 6 steps and by C to take 7 steps. But distance annual instalments. What is the amount of each
covered in 6 steps of A are equal to 7 steps of B and installment at 4% p.a. and interest compounded
8 steps of C. Ratio of their speed is- annually.
(a) 77 : 144 : 156 (b) 9 : 14 : 11 (a) 140608 (b) 120560
(c) 140 : 144 : 147 (d) 15 : 21 : 28 (c) 100000 (d) 180000
22. The expenditure on food of a family having ‘n’ 29. A person divided Rs. 340000 between his 2 sons
number of people is equal to 5 times of the square of aged 10 year & 12 year in such a way that each could
number of people in family. If one person leaves, get an equal amount when they attain 18 year of age
there is a decrease of 95 in consumption, find if rate is 10% p.a. simple interest. Find share of elder
number of people in family.
son.
(a) 6 (b) 8
(a) Rs. 240000 (b) Rs. 260000
(c) 10 (d) 12
(c) Rs. 12000 (d) Rs. 180000
23. There are 500 students in a school out of which 60%
30. A, B and C can complete a piece of work in 15, 30
are boys. 40% of the boys play hockey and girls
don’t play hockey. 75% of the girls play badminton. and 40 days respectively. They started the work
If there are only two games to be played, number of together and A left the job 2 days and B left the job 4
students who don’t play any game. days before the completion of work. In how many
(a) 10.6 days was the work completed?
2
(b) 36.6 (a) Not possible (b) 10 days
15
(c) 46.6 3
(c) 7 days (d) None of these
(d) can’t be determined 10

@BEST300MCQ For More Study Material


410
Visit: studyiq.com
Quantitative Aptitude

Set-10: Solutions
1. (a); Let, speed of faster train =S1 m/s 8. (a);
Speed of slower train = S2 m/s
130+110
S1 − S2 = = 4 m/s
1×60
130+110
S1 + S2 = = 80
3
S1 = 42 m/s At the end of third year he should pay Rs. 2952
S2 = 38 m/s to clear all his dues.
2. (b); Obviously., 9. (d); Let the principal be Rs. P and rate of interest be
(5M + 2B) = 4(1M + 1B) R% per annum.
∴ M = 2B Difference of C.I. and S.I. for 2 years
𝑅 2 𝑃 ×𝑅 × 2 𝑃𝑅 2
∴ Work done by a man and a boy are in the ratio = [𝑃 × (1 + ) − 𝑃] − ( )=
100 100 104
2: 1.
1 1 8 Difference of C.I. and S.I. for 3 years
3. (a); (A + B)’s 1 day’s work = + = 𝑅 3 𝑃 ×𝑅 ×3 𝑃𝑅 2 300+𝑅
30 50 150 = [𝑃 × (1 + ) − 𝑃] − ( )= ( )
1 1 7 100 100 104 100
∴ (A + C)’s 1 day’s work = + =
30 40 120 𝑃𝑅2 300+𝑅
8 7 67 ( ) 25 300+𝑅 25
104 100
∴ Work done in first two days = + = ∴ 𝑃𝑅2
= ⇒ ( )=
150 120 600 8 100 8
67 × 8 67 104
Work done in 8 × 2 = 16 days = = 100 1
600 75 𝑅= = 12 %.
67 8 8 2
Work left = 1 − = 10. (c); A + B + C = 3 *84 = 252
75 75
On the 17th day, (A + B) will work and they will A + B + C + D = 4 * 80 = 320 … (i)
complete
8
work. So, D = 68& E = 68 + 3 = 71
150
8 8 8 4
B + C + D +E = 79 * 4 = 316 … (ii)
∴ Work left = − = = From Eq. (i) & (ii)
75 150 150 75
On the 18the day, (A + C) will work and they will A-E = 320 – 316 = 4
120 4 32 A = E +4 = 71 + 4 = 75
finish it in × = days.
7 75 35 11. (d); Let x kg of good quality wheat is added in 150
32
∴ The whole work will be done in 17 days kg of wheat.
35
4. (b); Total amount= 120/100[80*6.75 +120*8]=1800 ATQ, 95% of (150 + x) = 135 + x
Now, required value= 1800/200= 9 Rs/kg 150 × 95 + 95𝑥 = 13500 + 100𝑥
750
5. (a); Amount distributed to 14 girls = 14 × 160 Rs. = 𝑥 ⇒ 𝑥 = 150 𝑘𝑔
5
∴ Because these 160 Rs. distribute to all 14 girls 12. (d); Given, a= 9, b= 3, e=6
comes when these 4 girls are not received the
amount.
14 × 160
Amount received by a girl =
4
14 × 160
Total amount = × 18 = 10080
4
6. (b); In 20 litres of cocktail mixture,
20 × 20
Alcohol = = 4 litres
100
a + b + e + g=23
Water = 20 - 4 = 16 litres
g= 23-(9+3+6)= 5
On adding 4 litres of water, 13. (c); Let the price of sugar be Rs. x per kg.
Quantity of water = 16 + 4 = 20 litres ∴ Initial expenditure = Rs. 30x
Quantity of mixture = 24 litres New expenditure = Rs. 33x
4 50 2 33𝑥
Required per cent = × 100 = = 16 % ∴ New monthly consumption = = 25 𝑘𝑔
24 3 3 1.32𝑥
7. (c); Total spending in 12 months 14. (a); Let number of bacteria in the first generation be x
= Rs [269.47 × 7 + 281.05 × 5] ∴ Number of bacteria in the second, third,
= Rs. 3291.54 fourth……generation would be
Total income = spending’s + savings 𝑥 4𝑥 16𝑥
8 ( ) , 8 ( ) , 8 ( )…..and so on.
2 2 2
= Rs. 3291.54 + Rs. 308.46
⇒ x, 4x, 16x, 64x,….. it is a GP with common ratio
= Rs. 3600.00
3600 4. Hence seventh term of GP
∴ Monthly salary of Ravi Shastri = Rs. = 𝑥(4)6 = 4096 ⇒ 𝑥(2)12 = 4096
12
= Rs. 300. ⇒ x = 1 or 1 million

@BEST300MCQ For More Study Material


411
Visit: studyiq.com
Quantitative Aptitude
41
15. (a); Let the number of male and female Votes of winner candidate = × 4000 = 1640
100
Participants at the start of seminar be 3x and x Votes of defeated candidate = 1640 – 200
respectively. = 1440
3𝑥−16 2
Then, = % votes of defeated candidate = 80
1440
× 100
𝑥+6 1
×4000
⇒ 3𝑥 − 16 = 2𝑥 + 12 ⇒ x = 28 100

∴ Total number of participants at the start of = 45%


seminar = 3x + x = 4 × 28 = 112 25. (a); Actual work done by a typist in 10 min
92 92
16. (b); (17 + 19) = 36% of the cost price = Rs. 162 = 20 × = lines
100 5
162 90
∴ Cost price = × 100 = Rs. 450 Total work = 23 × 40 × = 828 lines
36 100
17. (a); Original price of 250 charis. 5 10 1
Time required = 828 × × = 7 hours
92 60 2
= 250 × 50 = Rs. 12500
80 85 95 26. (b); Cost price of 12 oranges = selling price of 9
Price after discount = 12500 × × × oranges
100 100 100
= Rs. 8075 C.P. : S.P. = 3 : 4
18. (d); SP = Rs. 17,940, Discount = 8% Discount on 10 oranges = Profit on 5 oranges
∴ MP =
17940
= 𝑅𝑠. 19500 Let cost price = Rs. 300
0.92 Selling price = Rs. 400
∵ Gain = 19.6% (given) Profit = Rs. 100
17940
∴ CP = = 𝑅𝑠. 15000 Discount =
100×5
= 𝑅𝑠. 50
1.196
10
New SP without discount = Rs. 19500 Mark price = Rs. 450
Gain = (19500 – 15000) = Rs. 4500 100 100
4500 Profit percent = × 100 = %
∴ Gain percent = × 100 = 30% 300
50
3
100
15000
𝑝×𝑟×𝑡 Discount percent = × 100 = %
19. (b); S.I. = 450
100 100
9
200
100
𝑆.𝐼.×100 620×100 Percentage point = − = = 22.22%
𝑟= = = 6.2% 3 9 9
𝑝×𝑡 2000×5 30% 30%
27. (d); C.I. → 100 → 130 → 169
If r = 6.2 + 3 = 9.2%
𝑝×𝑟×𝑡 92×5 C.I. → 69%
Then, S.I. = = 2000 × = 𝑅𝑠. 920 S.I. → 36 × 2 = 72%
100 10×100
∴ Amount = Rs. 2000 + Rs. 920 = Rs. 2920 Percentage difference = 72% – 69% = 3%
20. (a); Amount remaining after Difference between S.I. and C.I.
7.5 3
1 yr = 4000 (1 + ) − 1500 = 𝑅𝑠. 2800 = 2200 × = Rs. 66
100 100
7.5 1
2 𝑦𝑟 = 2800 (1 + ) − 1500 = 𝑅𝑠. 1510 28. (a); Rate of interest = 4% = If
100 25
7.5
3 𝑦𝑟 = 1510 (1 + ) − 1500 = Rs. 123.25
100
21. (c); Let all walk for 168 minutes.
25 × 676 + 625 × 26 + 15625 = Rs. 48775
If Rs. 48,775 is amount to be paid then
installment = Rs. 17576
17576
So, Annual installment = × 390200
48775
= 𝑅𝑠. 140608
22. (c); 𝐸1 = 5(𝑛)2 29. (d); Ratio of Amounts
𝐸2 = 5(𝑛 − 1)2 12 years son’s 10 years son’s
𝐴𝑇𝑄, 𝐸1 − 𝐸2 = 5(𝑛)2 − 5(𝑛 − 1)2 Amount : amount
95 = 5(2𝑛 − 1)(1) = 100 +
10×100×8
: 100 +
10×100×6
= 180 ∶ 160
𝑛 = 10 100 100
180
Member in families = 10 Amount of elder son = × 340000
340
23. (d); So, about boys we are uncertain and extra data Rs. 1,80,000
needed. 30. (b);
24. (a); Let total voters = x
80
Vote casted = 𝑥
100
80
Valid votes = 𝑥 − 120 15
100
ATQ,
41
𝑥=(
80
𝑥 − 120) −
41
𝑥 + 200 Total work units = 120
82
100
80
100 100 If A work 2 day and B work 4 more days
𝑥= 𝑥 + 80 Than total work done = 120 + 2 × 8 + 4 × 4 = 152
100 100
2 152 2
𝑥 = 80 ⇒ 𝑥 = 4000 So, work was finished = = 10 days
100 15 15

@BEST300MCQ For More Study Material


412
Visit: studyiq.com
Quantitative Aptitude

1. A confectionery shopkeeper sells a soap at a (a) Rs 900 (b) Rs 800


discount of 20% and earns a profit of 60%. If he sells (c) Rs 700 (d) Rs 600
the same soap at 40% discount, then what will be 9. In a school having roll strength 299, the ratio of boys
his new profit percent? and girls is 8 : 5. If 25 more girls get admitted into
(a) 20 (b) 30 the school, the ratio of boys and girls becomes
(c) 35 (d) 40 (a) 46 : 35 (b) 43 : 35
2. 5 years ago the average age of a family which (c) 56 :33 (d) 53 : 33
includes father, mother and a son was 35 years. 3 10. The average weight of A, B and C is 55 kg. If the
years ago the average age of father and mother was average weight of A and B be 50 kg and that of B and
46 years. What is the present age (in years) of the C be 53 kg, then the weight of B is
son? (a) 41 kg (b) 42 kg
(a) 20 (b) 24 (c) 39.5 kg (d) 45 kg
(c) 26 (d) 22 11. For an article the profit is 160% of the cost price. If
3. The cost price of 60 articles is same as the selling the cost price increases by 30% but the selling price
price of x articles. If there is a profit of 20%, then remains same, then what is the new profit
what is the value of x? percentage?
(a) 15 (b) 30 (a) 25 (b) 50
(c) 50 (d) 80 (c) 75 (d) 100
4. A person scores 45% of the total marks in the exam 12. A’s salary is 25% more than that of B. Then B’s
and still fails by 40 marks. The passing percentage of salary is less than that of A by
7
the exam is 55%. What is 35 % of the maximum (a) 25% (b) 22.5%
4
marks of the exam? (c) 20% (d) 17.5%
(a) 387 (b) 135 13. A boatman rows 1 km in 5 minutes, along the stream
(c) 147 (d) 400 and 6 km is 1 hour against the stream. The speed of
5. Walking from home at 2/3rd of his usual speed, a the boat in still water.
man reaches his office 30 minutes late. Had the (a) 3 km/h (b) 9 km/h
person walked at 5/4th of his usual speed, find the (c) 12 km/h (d) 6 km/h
time taken by the man to reach his office. 14. A person lent certain sum of money at 5% per
(a) 45 minutes (b) 50 minutes annum simple interest and in 15 years the interest
(c) 48 minutes (d) 60 minutes amounted to Rs 500 less than the sum lent. What
6. The difference of compound interest and simple was the sum lent?
interest for 3 years and for 2 years are in ratio 23 : 7 (a) 2000 (b) 1000
respectively. What is rate of interest per annum (in (c) 500 (d) 250
3
%)? 15. In a class of the students are girls and rest are
7
(a) 200/7 (b) 100/7 2 1
(c) 300/7 (d) 400/7 boys. If of the girls and of the boys are absent.
9 11
7. A, B and C can complete a piece of work in 9, 12 and What part of the total number of students are
18 days respectively. They started working together, present?
but A left the work before 4 days of its completion. B (a)
197
(b)
177
231 231
also left the work 2 days after A left. In how many 197 177
days was the work completed? (c) (d)
242 242
4 3
(a) 5 days (b) 6 days 16. If a given work is done by some men in 15 days in
9 9
4 3 such a way that on first day there only one man was
(c) 6 days (d) 4 days working on second day another man joined him on
9 9
8. A dealer offers a discount of 10% on the marked the third day another joined and so on. Then in how
price of an article and still makes a profit of 20%. If many days 4 men will finish the work?
its marked price is Rs 1200, then the cost price of (a) 32 (b) 30
the article is (c) 28 (d) 24

@BEST300MCQ For More Study Material


413
Visit: studyiq.com
Quantitative Aptitude

17. The cost of the article. 24. A builder decided to build a building in 50 days. He
(A) 10% (B) 20% employed 200 men in the beginning and 150 more
(C) 5% (D) 15% men after 30 days and completed the construction in
18. A vessel is full of chocolate syrup. ¼ of the syrup is the stipulated time. If he had not employed the
taken out of the vessel & replaced it with milk. This additional men, how many days behind schedule
process is repeated 2 more times and in this process would it have been finished?
74 ltr milk has been required. Find the total volume (a) 15 days (b) 10 days
of solution? (c) 9 days (d) 12 days
(a) 100 Ltr (b) 120Ltr 25. CP of 12 apples is equal to the SP of 9 apples and
(c) 128Ltr (d) 150Ltr discount on 20 apples is equal to the profit on 5
19. The average score of a cricketer in 8 innings is 44. apples. What is the percent point difference between
He had scored 60, 24, x, 50, 73, y, z, 13 respectively the profit % and discount % ?
in those innings. Find the average of x, y and z? (a) 22.22 (b) 25.33
(a) 40 (b) 44 (c) 27.45 (d) 29.33
(c) 48 (d) 52 26. The compound interest on a certain sum for 2 years
20. What annual payment will discharge a debt of Rs. is Rs. 882 and SI (simple interest) is Rs. 840. If the
6450 due in 4 years at 5% per annum simple sum is invested such that the SI is Rs. 2058 and the
interest? number of years is equal to the rate percent per
(a)Rs. 1400 (b)Rs. 1500 annum. Find the rate percent ?
(c)Rs. 1550 (d)Rs. 1600 (a) 6 (b) 7
21. Two men A and B are 60 km apart and are walking (c) 8 (d) 10
towards each other with the speed of 10 kmph & 5 27. The average salary per head of all the workers of an
kmph and a dog is running at 12 kmph from man A institution is Rs. 80. The average salary of 12 officers
towards Man B & then again towards man A and so is Rs. 500; the average salary per head of the rest is
on until A meets B. Find the distance travelled by the Rs. 64. The total number of worker is the institution
dog? is
(a) 60 km (b) 40 km (a) 580 (b) 327
(c) 50 km (d) 48 km (c) 109 (d) 480
22. Sohan earns Rs S a year. After one year he gets 28. A trader sells 20 kg of salt at Rs 400. A customer
promoted and there is a Y% increase in his earnings asks 20% discount and be agreed to it but instead of
and X% decrease in the expenditure. If he used to 1 kg, he gives 8% less salt. What is the effective
spend P Rs. per month then now his monthly savings discount that the customer gets?
7 7
would be equal to (a) 15 % (b) 11 %
23 23
𝑆 𝑌 𝑋 7 1
(a) (1 + ) + 𝑃 (1– ) (c) 19 % (d) 13 %
12 100 100 23 23

(b) 𝑆 (1 +
𝑌
) + 𝑃 (1–
𝑋
) 29. Three vessels of equal capacity containing mixture
100 100 of milk and water in the ratio 3 : 4, 3 : 11 and 4 : 17
𝑆 𝑌 𝑋
(c) (1 + ) − 𝑃 (1– ) respectively. These three vessels are emptied into a
12 100 100
𝑌 𝑋 large vessel, what will be the ratio of milk and water
(𝑑) 𝑆 (1 + ) – 𝑃 (1– ) in the large vessels?
100 100
23. Population of a city X is 1, 60,000. In the next three (a) 5 : 13 (b) 7 : 13
years, there is a total increase of 8% in male (c) 5 : 17 (d) 7 : 17
population and increase of 20% in female 30. A train leaves Patna at 9 am and reaches Kolkata at 3
population, which results in male to female ratio as pm on same day. Another train leaves Kolkata at
11:00 am and reaches Patna at 4:00 pm on same
3:2. Find the original male and female population of
day. What is the time of day (approximately), when
the city? the two trains will meet?
(a) 84000, 96000 (b) 100000, 60000 (a) 12:49 pm (b) 1:49 pm
(c) 120000, 45000 (d) 90000, 70000 (c) 12:27 pm (d) 1:27 pm

@BEST300MCQ For More Study Material


414
Visit: studyiq.com
Quantitative Aptitude

Set-11: Solutions
1. (a); Let MP of soap = Rs. 100 23 : 7
∴ A = 7 and B = 2
2
∴ Rate of interest = × 100%
7
200
=
7
7. (c);
80
CP = × 100 = 50
160
10
∴ Profit = × 100 = 20% ⇒ Here A left 4 days before
50
2. (d); Let the age of son 3 years ago be x → A would have done
∴ According to the question, (4 × 4) = 16 units
35 × 3 + 2 × 3 = 46 × 2 + x ⇒ B left 2 days after A, it means
⇒ x = 19 he left 2 days before the work finished
∴ Present age = 19 + 3 = 22 → B would have done
3. (c); According to the question, (3 × 2) = 6 units
60 × CP = x × SP ∴ Total work (new) = 36 + 16 + 6
𝐶𝑃 𝑥
⇒ = = 58 units
𝑆𝑃 60 58 𝑢𝑛𝑖𝑡𝑠
∴ After 20% profit ∴ Work finished in =
9 𝑢𝑛𝑖𝑡𝑠/𝑑𝑎𝑦𝑠
120 4
𝑥× = 60 = 6 𝑑𝑎𝑦𝑠
100 9
⇒ x = 50 8. (a); Let the marked price be Rs. 100
4. (c); Let the maximum marks be x
∴ 0.45x + 40 = 0.55x
⇒ x = 400 marks
7
But we need 35 % of the maximum marks
4
= 147/4 * 400/100 = 147
5. (c); If distance is constant then time taken in Now, ATQ
inversely proportional to the speed. When 100 → 1200
speed becomes two-third of the normal speed, ∴ 90 ×
100

1200
× 90 ×
100
3 120 100 120
time taken will be times of his normal time. If = Rs. 900
2
the normal time to reach the office is T, then the 9. (a); Initially number of boys
𝑇 8
man is taking time extra to reach the office. = × 299 = 184
2 8+5
𝑇
Given that = 30 ⇒ T = 60 ∴ Number of girls;
2 5
5
If the man walk at th of the normal speed, time = × 299 = 115
13
4
4 25 more girls get admitted
taken will be th of the normal time ∴ required ratio
5
4 184 46
= × 60 = 48 min = =
5 115+25 35
6. (a); 10. (a); B’s weight = (A + B)’s weight + (B + C)’s weight –
(A + B + C)’s weight
= (50 × 2 + 53 × 2) – 55 × 3
= (100 + 106) – 165 = 206 – 165
= 41 kg
11. (d); Let the CP be Rs. x
∴ Profit will be Rs. 1.6x
Now, new CP = 1.3x
And SP = x + 1.6x = 2.6x
∴ New Profit %
2.6𝑥−1.3𝑥
= × 100
1.3𝑥
= 100%
3A + B : A

@BEST300MCQ For More Study Material


415
Visit: studyiq.com
Quantitative Aptitude
1
12. (c); 25% = 37x = 74
4
It means if B → 4 units x=2
∴ A → 5 units hence 64 × 2 = 128 Ltr is vol. of total solution .
5−4 19. (b); Average score of the cricketer ⇒
∴ Required % = × 100 60+24+𝑥+50+73+𝑦+𝑧+13
5 44 =
= 20% 8
1 44 × 8 = 220 + x + y + z
13. (b); x → speed in downstream = × 60 352 – 220 = x + y + z
5
= 12 km/h 132 = x + y + z
y → speed in upstream → 6 km/h Average of x, y and z = 132/3 = 44
𝑥+𝑦 100𝐴
Speed of boat in still water = km/h 20. (b); =
2 𝑟𝑡(𝑡–1)
12+6 100𝑡+
2
= = 9 𝑘𝑚/ℎ 100∗6450
2 = 5∗4(4–1)
14. (a); Let principal = x 100∗4+
2
So interest = (x – 500) = 645000/430
x×5×15
∴ (x − 500) = = 1500
100
⇒ x = Rs. 2000 21. (d); Total time of travel required for A & B to meet
15. (a); Take LCM of 7, 9, 11 = 693 at a point
60
→ let the total no. of students be 693 =
10+5
3
→ No. of girls = × 693 = 297 = 4 hr
7
∴ No. of boys = 693-297 = 396 & dog will travel only for 4 hr (until A & B meet)
2 = 12 × 4 = 48 km
→ No. of girls who are absent = × 297 = 66
9 22. (c); Monthly salary = S/12
1 𝑆 𝑌
→ No. of boys who are absent = × 396 = 36 𝑁𝑒𝑤 𝑚𝑜𝑛𝑡ℎ𝑙𝑦 𝑠𝑎𝑙𝑎𝑟𝑦 𝑒𝑎𝑟𝑛𝑒𝑑 = (1 + )
11 12 100
→ Total no. of absent students = 66 + 36 = 102 𝑋
𝑁𝑒𝑤 𝑚𝑜𝑛𝑡ℎ𝑙𝑦 𝑒𝑥𝑝𝑒𝑛𝑑𝑖𝑡𝑢𝑟𝑒 = 𝑃 (1– )
→ No. of present students = 693 – 102 = 591 100
591 197 𝑆 𝑌 𝑋
→ Part of the present students = = 𝑀𝑜𝑛𝑡ℎ𝑙𝑦 𝑠𝑎𝑣𝑖𝑛𝑔𝑠 = (1 + ) – 𝑃 (1– )
693 231 12 100 100
16. (b); Man-days of work in 15 days will be 23. (b); Let the population of males be x & female
= 1 + 2+ 3 + 4 + ………….+15 population as y.
𝑛(𝑛+1)
∑⇒
2
15×16
=
2
= 120 Man-days 1.08x 3
=
For four person 1.2y 2
𝑥 5
W = M. D =
𝑦 3
120 = 4.D
Male : Female population = 100000 : 60000
30 = D
24. (a); Let one man can do 1 unit in 1 day
17. (c); Let us assume the cost of components P, Q, R, S
Work done by extra 150 men in (50 – 30)
is 40, 50, 60, 70 respectively.
i.e. 20 days = 150 × 20 = 3000 units
Now, 3000 units work done be 200 men
3000
= = 15 days
200
i.e. the work will be completed 15 days behind
231–220
the schedule
𝑇𝑜𝑡𝑎𝑙 % 𝑐ℎ𝑎𝑛𝑔𝑒 = 25. (c); CP of 12 apples = SP of 9 apples
220
= 5% CP : SP = 9 : 12 = 3 : 4
18. (c); After removing ¼ of the syrup remaining syrup Let CP of one apple = 300, SP of one apple = 400
will be ¾ times the original. & Profit = 400 – 300 = Rs 100
Similarly for second time amount of syrup ∴ Discount on 20 apples = 5 × 100 = Rs 500
2 500
remaining = (3⁄4) times original volume. ∴ Discount on 1 apple =
20
= 𝑅𝑠 25
3 3 MP = 400 + 25 = Rs 425
For third time = ( ) 𝑡𝑖𝑚𝑒𝑠 𝑜𝑟𝑖𝑔𝑖𝑛𝑎𝑙 𝑣𝑜𝑙. 100 100
27
4 ∴ 𝑝𝑟𝑜𝑓𝑖𝑡 % = × 100 =
300 3
= 25 100
64 𝑎𝑛𝑑, 𝑑𝑖𝑠𝑐𝑜𝑢𝑛𝑡 % = × 100 =
Assume if 64 is original vol. of syrup then 27 Ltr 425 17

will be remaining & 64 - 27 = 37 Ltr milk is ⇒ Required percent difference


100 100
required. = − = 27.45
3 17

@BEST300MCQ For More Study Material


416
Visit: studyiq.com
Quantitative Aptitude

26. (b); ie., SPof 920 gm salt = Rs 20


20 500
∴ SP of 1 kg salt = × 1000 =
920 23
500
25−
⇒ Effective discount = 23
× 100
25
1
= 13 %
23
29. (a); ATQ,
SI (2 years) = 420 + 420 = 840 Let the total capacity of vessel
CI (2 years) = 420 + 420 + 42 = 882 = LCM of (7, 14, 21) = 42
⇒ Now, Then,
𝑃×𝑥×𝑥 Milk : Water
= 𝑆𝐼
100 3 3 4  4 11 17 
4200×𝑥 2   21  14  21  21  21  :  7  21  14  21  21  21 
⇒ = 2058 7   
100 5 : 13
⇒ x² = 49 30. (a); Let the distance between Patna & Kolkata be D.
⇒x=7 Speed of first train moving from Patna to
27. (b); By allegation rule 𝐷
Kolkata =
6
Speed of second train moving from Kolkata to
𝐷
Patna =
5
Since the second train starts at 11:00 am
Distance travelled by first train in 2 hours
𝐷 𝐷
Officers : Rest = 16 : 420 = ×2=
6 3
𝐷 2𝐷
= 4 : 105 Remaining distance = D – =
3 3
= 12 : 315 Now time taken till they meet
Total no. of workers = 12 + 315 2𝐷

= 327 = 𝐷 𝐷
3
+
28. (d); SP of 20 kg salt= Rs 400 6 5
2
2 30
∴ SP of 1 kg salt =Rs 20 = 3
11 = ×
3 11
⇒ Discount = 20% 30
20 9
20 1→ 𝐷 = = 1 ℎ𝑟𝑠
𝐷= = 11 11
100 5 →𝑀𝑃
∴ Time at which they will meet is 11:00 am + 1
SP = 5 – 1 = 4 9
5 hour × 60 minutes
∴ MP of 1 kg salt = × 20 = 𝑅𝑠 25 11
4
=12:49 pm
Now, he gives 8% less salt instead of 1 kg
Ie., he gives 920 gm salt

@BEST300MCQ For More Study Material


417
Visit: studyiq.com
Quantitative Aptitude

1. S, T and U can complete a work in 40, 48 and 60 9. What is the compound interest (in Rs) for 1 year on
days respectively. They received Rs 10800 to a sum of Rs 20000 at the rate of 40% per annum
complete the work. They begin the work together compounding half yearly?
but T left 2 days before the completion of the work (a) 8000 (b) 8650
and U left 5 days before the completion of the work. (c) 8750 (d) 8800
S has completed the remaining work alone. What is 10. Asif is twice as good as workman as Bashir and
the share of S (in Rs) from total money? together they finish a piece of work in 30 days. In
(a) 4000 (b) 4320 how many days will Asif alone finish the work?
(c) 4500 (d) 4860 (a) 90 (b) 45
2. A farmer's land is in the shape of a trapezium which (c) 60 (d) 75
has its parallel sides measuring 2.56 yards and 3.44 11. What is the area (in sq cm) of a rectangle if its
yards and the distance between the parallel sides in diagonal is 25 cm and one of its sides is 24 cm?
1.44 yards. The cost of ploughing the land is Rs 1800 (a) 186 (b) 144
per square yard. What amount will (in Rs) have to (c) 132 (d) 168
be spent in order to plough the entire land? 12. A shopkeeper marks up his wares by 60% and offers
(a) 3672 (b) 6732 25% discount. What will be the selling price (in Rs)
(c) 7776 (d) 8214 if the cost price is Rs 1600?
3. If after giving a discount of 18%, a book is sold for (a) 1920 (b) 2000
Rs 1599, then what will be the marked price (in Rs) (c) 2120 (d) 2200
of the book? 13. A batsman makes a score of 111 runs in the 10th
(a) 1800 (b) 1880
match and thus increases his average runs per
(c) 1950 (d) 2000
match by 5. What will be his average after the 10 th
4. A man has 3 sons, 2 daughters and a wife. They
match?
divided a sum of Rs 19000 among themselves such
(a) 66 (b) 61
that each daughter got 1.5 times the amount
(c) 62 (d) 64
received by each son and his wife received 600 less
14. A vendor buys 6 bananas for Rs 25 and sells them at
than each son. What is the total amount (in Rs)
received by the 3 sons together? 3 for Rs 20. What is his profit percentage?
(a) 2800 (b) 3600 (a) 50 (b) 40
(c) 5600 (d) 8400 (c) 60 (d) 30
5. The average of 3 consecutive even numbers is A. If 15. Two labourers A and B are paid a total of Rs 650 per
next 5 even numbers are added, then what is the day. If A is paid 160 percent of what is paid to B,
average of these 8 numbers? how much (in Rs) is B paid?
(a) A + 3 (b) A + 4 (a) 250 (b) 400
(c) A + 5 (d) A + 7 (c) 350 (d) 450
6. While selling an article for Rs 18450, a person 16. A man travelled a distance of 60 km in 7 hours. He
suffered a loss of 50%. At what price he should have travelled partly on foot @ 6 km/hr and partly on
sold the article (in Rs) to earn a profit of 50%? bicycle @ 12 km/hr. What is the distance (in kms)
(a) 13837 (b) 52000
(c) 55350 (d) 56775 travelled on foot?
7. The present population of a town is 26010. It (a) 15 (b) 9
increases annually at the rate of 2%. What was the (c) 48 (d) 24
population of town 2 years ago? 17. The compound interest earned in two years at 12%
(a) 25000 (b) 25100 per annum is Rs 10176. What is the sum (in Rs)
(c) 25200 (d) 25500
invested?
8. A train of length 100 m crosses another train of
length 150 m, running on a parallel track in the (a) 50000 (b) 60000
opposite direction in 9 seconds. If the speed of train (c) 40000 (d) 80000
having length 150 m is 40 km/hr, then what is the 18. Sum of a fraction and thrice of its reciprocal is
speed (in km/hr) of the other train? 73/20. What is the fraction?
(a) 30 (b) 48 (a) 4/5 (b) 9/4
(c) 50 (d) 60 (c) 4/9 (d) 5/4
@BEST300MCQ For More Study Material
418
Visit: studyiq.com
Quantitative Aptitude

19. A and B together do a job in 12 days and A could do 26. A sum fetched a total simple interest of Rs.5,400 at
the job in 20 days if he worked alone. How many the rate of 12.5 %/yr in 4 years. What is the sum (in
days would B take to do the job if he worked alone? Rs)?
(a) 30 (b) 25 (a) 11800 (b) 12800
(c) 24 (d) 15 (c) 9800 (d) 10800
20. If 1 shirt is offered free on purchase of 4 shirts, what 27. If the square of sum of three positive consecutive
is the effective discount (in %) on each shirt? natural numbers exceeds the sum of their
(a) 25 (b) 20 corresponding squares by 292, then what is the
(c) 16 (d) 24 largest of the three numbers?
21. The ratio of present ages of P and Q is 7:9. Before 10 (a) 5 (b) 6
years the ratio of their ages was 5:7. What is Q's (c) 7 (d) 8
present age (in years)? 28. A can do a piece of work in 6 days working 8 hours a
(a) 35 (b) 45 day while B can do the same work in 4 days working
(c) 25 (d) 55 10 hours a day. If the work has to be completed in 5
22. The average marks of 40 students in an examination days, so how many hours do they need to work
was 34. It was later found that the marks of one together in a day?
4
student had been wrongly entered as 62 instead of (a) 4 (b) 5
11
26. What is the correct average? 4 4
(a) 33.1 (b) 33.3 (c) 6 (d) 4
11 11
(c) 33.5 (d) 33.7 29. A shopkeeper allows 25% discount on the marked
23. A wholesaler sells a good to a retailer at a profit of price of an article and he suffered a loss of 15%.
5% and the retailer sells it to a customer at a profit What will be the profit percent if the article is sold at
of 10%. If the customer pays Rs. 2,000, what had it marked price?
cost (in Rs) the wholesaler? (a) 11.76 (b) 12.12
(a) 1731.6 (b) 3210.6 (c) 13.33 (d) 14.28
(c) 1931.6 (d) 2310.6 30. Three boxes of capacity 24 kg, 36 kg and 84 kg are
24. When a number is increased by 69, it becomes completely filled with three varieties of wheat A, B
103% of itself. What is the number? and C respectively. All the three boxes were emptied
(a) 1300 (b) 3300 and the three types of wheat were thoroughly mixed
(c) 2300 (d) 4300
and the mixture was put back in the three boxes.
25. A boat goes a certain distance at 40 km/hr and
comes back the same distance at 24 km/hr. What is How many kg of type A wheat would be there in the
the average speed (in km/hr) for the total journey? third box?
(a) 32 (b) 28 (a) 10 (b) 12
(c) 34 (d) 30 (c) 14 (d) 16

Set-12: Solutions
1. (d); S : T : U = 18 × 6 : 16 × 5 : 13 × 4 = 108 : 80 : 52
So,
10800
S’s share ratio = × 108= 4860
240
1
2. (c); Required amount = (sum of parallel side) ×
2
height * cost
1
= (2.56 + 3.44) × 1.44 × 1800 = 7776
Total work = 240 2
1599
⇒ work done by S in last two days = 12 3. (c); Marked price = × 100 = 1950
82
⇒ work done by S & T in last 3 days after S two 4. (d); Given that,
days 𝐷
= 1.5 ... (i)
= (6 + 5) ×3 = 33 𝑆
So, remaining work to be completed by S, T & U and,
= 240 – (12 + 33) = 195 W = S – 600 ... (ii)
195 and,
So, No of days taken by S, T & U = (6+5+4) =13
3S + 2D + W = 19000 ... (iii)
So, 3
3S + 2 × 𝑆 + (S – 600) = 19000
Received amount is accordingly to efficiency 2
19600
and number of day 7S = 19600 ⇒ 3S = × 3= 8400
7

@BEST300MCQ For More Study Material


419
Visit: studyiq.com
Quantitative Aptitude

5. (c); Let 3 consecutive even number = 2, 4, 6 13. (a); Let the average after 9 matches be x
2+4+6 9𝑥+111
& it’s average = =4 ∴ =𝑥+5
3 10
and next 5 even number = 8, 10, 12, 14, 16 ⇒ 9x + 111 = 10x + 50 ⇒ x = 61
8+10+12+14+16
& it’s average = =12 ∴ Average after 10 match = 61 + 5 = 66
5 25
& total average of 8 number = = 9
72 14. (c); CP of 1 banana = Rs.
6
8 20
So, its increment = (𝐴 + 5) SP of 1 banana = Rs.
3
20 25
6. (c); We know that –
If he sells in 50% loss for article = 18450 ∴ 𝑃𝑟𝑜𝑓𝑖𝑡% = 3 6
25 × 100 = 60%
6
So, if he want to earn 50% profit then 15. (a); Let salary of B is x
= 18450 × 3 = 55350 ∴ salary of A = 1.6x
102 51
7. (a); 2% = = ∴ 1.6x + x = 650
100 50
50 51 ⇒ 2.6x = 650 ⇒ x = Rs. 250
50 51 16. (d); Let distance travelled on foot be x km
2500 2601
∴ ATQ,
𝑥 60 –𝑥
+ = 7 ⇒ 2x + 60 – x = 7 × 12
So, given that amount after 2 year ago = 26010 6 12
It is compared with amount of ratio it is to times ⇒ x + 60 = 84 ⇒ x = 24 km
12 3
its’s original value 17. (c); 𝐺𝑖𝑣𝑒𝑛 𝑅𝑎𝑡𝑒 = 12% = =
100 25
So, population before 2 year ago = 2500 × 10 Let Principal be = (25)² = 625
= 25000
𝐷𝑖𝑠𝑡𝑎𝑛𝑐𝑒
8. (d); relative speed =
𝑇𝑖𝑚𝑒
when in opposite direction speed (S1 + S2)
So,
Speed of another train S2
(100+150) 18
(40 + 𝑆2 ) = ×
9 5
S2 = 60 km/hr Now,
9. (d); Rate = 20% per half/year (75 + 75 + 9) = 159 units = Rs. 10176
10176
∴ 625 𝑢𝑛𝑖𝑡𝑠 = × 625 = 𝑅𝑠. 40,000
159
18. (d); Let the fraction be x
∴ ATQ,
3 73
So, compound interest = 800 × 11 = 8800 𝑥+ =
𝑥 20
10. (b); Let Asif alone complete work in x days 73𝑥
⇒ 𝑥2 + 3 = ⇒ 20x² – 73x + 60 = 0
20
∴ Bashir will complete in 2x days.
1 1 1 73±√(73)2 −4×20×60
∴ + = ∴𝑥=
𝑥 2𝑥 30 2×20
12 5

2+1
=
1 𝑥= &
5 4
2𝑥 30
19. (a); ATQ,
⇒ 30 × 3 = 2x ⇒ x = 45 days.
(A + B) × 12 = A × 20
11. (d); 𝐴 3
=
𝐵 2
Total work = (3 + 2) × 12 or 3 × 20 = 60 units
60
B alone can do the work in = 30 𝑑𝑎𝑦𝑠.
2
1
20. (b); Effective Discount = × 100 = 20%
1+4
21. (b); Let the present age of P and Q be 7x and 9x
AB = √(252 – 242 ) = 7 𝑐𝑚 respectively
∴ Area of rect ABCD = 24 × 7 = 168 cm² ATQ,
12. (a); Let CP be = Rs. 100 7𝑥 –10 5
= ⇒x=5
9𝑥 –10 7
∴ MP = Rs. 160 ∴ Q’s present age = 9x = 45 years.
After 25% discount, 22. (a); Correct average =
40 × 34 – 62+ 26
= 33.10
75 40
𝑆𝑃 = 160 × = 𝑅𝑠. 120 23. (a); Let it cost Rs. ‘x’ to the wholesaler
100
105 110
∵ 100 units → Rs. 1600 ∴x× × = 2000
1600 100 100
∴ 120 𝑢𝑛𝑖𝑡𝑠 → × 120 = 𝑅𝑠. 1920 x = Rs. 1731.6
100

@BEST300MCQ For More Study Material


420
Visit: studyiq.com
Quantitative Aptitude

24. (c); Let the no. be ‘x’. 28. (d); Total hours taken by A = 8 × 6 = 48 h
103
ATQ, (x + 69) = 𝑥 Total hours taken by B = 4 × 10 = 40 h
100
100x + 6900 = 103x They together will complete whole work in
48×40 240
3x = 6900 ⇒ x = 2300 = = ℎ𝑜𝑢𝑟𝑠
2 × 40 × 24 88 11
25. (d); Average speed = = 30 km/hr. If work has to be completed in 5 days then daily
40+24
26. (d); Let the sum be ‘P’ hours will be
𝑃×12.5×4
ATQ, 5400 = P = Rs. 10800 240 4
100 = =4 ℎ𝑜𝑢𝑟𝑠
5×11 11
27. (d); Let three consecutive numbers be (x–1), x, (x+1)
So, 29. (c); 75% of MP = 85% of CP
𝑀𝑃 17
[(x – 1) + x + (x + 1)]² = 292 + (x–1)²+ x²+ (x+1)² =
𝐶𝑃 15
9x² = 292 + x² + 1 – 2x + x² + x² + 1 + 2x Profit percent will be =
2
× 100
9x² = 292 + 3x² + 2 2×20 40
15

6x² = 294 = = % = 13.33%


3 3
x² = 49 30. (c); Ratio of capacity of boxes = 2 : 3 : 7
x=±7 Type A wheat in third box
so, x =7 since x is +ve natural. 2
= × 84 = 14 kg
12
Thus, no. largest number is x + 1 = 8

@BEST300MCQ For More Study Material


421
Visit: studyiq.com
Quantitative Aptitude

1. Product of three consecutive odd numbers is 1287. 10. If A and B together do a job in 7.5 days and if A could
What is the largest of the three numbers? do the job in 10 days if he worked alone. How many
(a) 9 (b) 11 days would B take to do the job if he worked alone?
(c) 13 (d) 17 (a) 30 (b) 40
2. 45 men or 60 boys can do a piece of work in 20 days. (c) 25 (d) 50
How many days will 15 men and 20 boys take to 11. What is the area (in sq cm) of a rectangle of
complete the work? perimeter 48 cm and breadth 10 cm?
(a) 23 (b) 45 (a) 140 (b) 480
(c) 30 (d) 25 (c) 240 (d) 440
3. The marked price of a shirt is Rs 1280. If the shirt is 12. If the selling price of an item is Rs 1000 after getting
being sold for Rs 900, then what is the discount a discount of 20%, then what was the marked price
percentage? (in Rs)?
(a) 31.31 (b) 25.57 (a) 1200 (b) 1250
(c) 29.68 (d) 34.36 (c) 800 (d) 1400
4. The ratio of number of cans of orange, pineapple and 13. If A = 2B = 4C; what is the value of A : B : C?
mixed fruit juices kept in a store is 8 : 9 : 15. If the (a) 4 : 2 : 1 (b) 1 : 2 : 4
store sells 25%, 33.33% and 20% of (c) 8 : 4 : 1 (d) 16 : 4 : 1
orange,pineapple and mixed fruit juices cans 14. The average cost of 4 items in a shopping list is Rs
respectively, then what is the ratio of number of 1,250. If one more item whose cost is Rs 2,000 is
cans of these juices in the remaining stock? added to the list what will be the new average
(a) 1 : 1 : 2 (b) 6 : 6 : 13 (inRs)?
(a) 1100 (b) 1500
(c) 12 : 15 : 19 (d) 4 : 9 : 13
(c) 1400 (d) 1250
5. The ratio of number of boys and girls in a class is 2 :
15. If a vendor sells a coconut at Rs 24 he makes 20%
3. The average weight of boys and girls in the class is
loss. If he sells at Rs 36 then what is his profit
18 kg and 21 kg respectively. What is the average
percentage?
weight (in kgs) of all the boys and girls together?
(a) 10 (b) 20
(a) 99/5 (b) 101/5
(c) 30 (d) 40
(c) 109/6 (d) 96/5
16. If a taxi going at 40 km/hr takes 25 minutes to travel
6. A milk merchant buys 50 litres of milk at the rate of a certain distance, by how much should it increase
Rs 40 per litre and mixes 5 litres of water in it. If he its speed (in km/hr) to travel the same distancein 20
sells this mixture at the rate of Rs 42 per litre,then minutes?
what is the profit percentage for the dealer? (a) 50 (b) 5
(a) 17.2 (b) 14.4 (c) 25 (d) 10
(c) 16.6 (d) 15.5 17. If amount received on a certain amount after 3rd
7. If A is 6 times more than B, then by what percentage year is Rs. 1,240, what will be the amount (in Rs)
is B is less than A? after the 4th year on the same amount at 9% rate of
(a) 64.82 (b) 83.33 interest?
(c) 28.56 (d) 85.71 (a) 1245.6 (b) 1521.6
8. A runner starts running from a point at 6:00 am (c) 1351.6 (d) 1220.6
with a speed of 8 km/hr. Another racer starts from 18. 10 women can do a piece of work in 6 days, 6 men
the same point at 8:30 am in the same direction with can do same work in 5 days and 8 children can do it
a speed of 10 km/hr. At what time of the day (in in 10 days. What is the ratio of the efficiency of a
p.m.) will the second racer overtake the other woman, a man and a child respectively?
runner? (a) 4 : 6 : 3 (b) 4 : 5 : 3
(a) 8:00 (b) 4:00 (c) 2 : 4 : 3 (d) 4 : 8 : 3
(c) 6:30 (d) 5:30 19. Kanchan bought a clock with 25% discount on
9. A sum amounts to Rs 7727.104 at the rate of 12% marked price. She sold it with 75% gain on the price
per annum compounded annually after three years. she bought. What was her profit percentage on the
What is the value of principal (in Rs)? marked price?
(a) 5000 (b) 5200 (a) 31.25 (b) 50
(c) 5350 (d) 5500 (c) 56.25 (d) 60

@BEST300MCQ For More Study Material


422
Visit: studyiq.com
Quantitative Aptitude

20. A, B and C received an amount of Rs 8400 and (a) 10 (b) 12.5


distributed among themselves in the ratio of 6 : 8 : 7 (c) 15 (d) 20
respectively. If they save in the ratio of 3 : 2 : 4 26. A group of boys has an average weight of 36 kg. One
respectively and B saves Rs 400, then what is the boy weighing 42 kg leaves the group and another
ratio of the expenditures of A, B and C respectively? boy weighing 30 kg joins the group. If the average
(a) 6 : 8 : 7 (b) 8 : 6 : 7 now becomes 35.7 kg, then how many boys are
(c) 9 : 14 : 10 (d) 12 : 7 : 9 there in the group?
21. The average age of 24 students is 12 years. It was (a) 30 (b) 32
observed that while calculating the average age, the (c) 40 (d) 56
age of a student was taken as 14 years instead of 8 27. A man gains 15% by selling a calculator for a certain
years. What will be the correct average age (in price. If he sells it at triple the previous selling price,
years)? then what will be the profit percentage?
(a) 11.25 (b) 11.5 (a) 125 % (b) 175 %
(c) 11.75 (d) 12.25 (c) 225 % (d) 245 %
22. 70% of the cost price of a article is equal to the 40% 28. In an election between two candidates, the winning
of its selling price. What is the profit or loss candidate has got 70% of the votes polled and has
percentage? won by 15400 votes. What is the number of votes
(a) 63% loss (b) 70% loss polled for loosing candidate?
(c) 75% profit (d) 80% profit (a) 38500 (b) 11550
23. a% of b + b% of a = ______ (c) 26950 (d) 13550
(a) 2a% of b (b) 2a% of 2b 29. A boat goes 4 km upstream and 4 km downstream in
(c) 2a% of 2a (d) 2b% of 2b 1 hour. The same boat goes 5 km downstream and 3
24. If I walk at 7/6 of my usual speed, then I reach my km upstream in 55 minutes. What is the speed (in
office 15 minutes early. What is the usual time taken km/hr) of boat in still water?
(in minutes) by me to reach the office? (a) 6.5 (b) 7.75
(a) 60 (b) 75 (c) 9 (d) 10.5
(c) 90 (d) 105 30. Simple Interest received by a person in 10 years on
25. A person lent Rs 10000 to B for 3 years and Rs 6000 a principal of Rs 9500 is 130% of the principal. What
to C for 4 years on simple interest at same rate of is the rate of interest (in %) per annum?
interest and received Rs 5400 in all from both of (a) 12 (b) 13
them as interest. What is the rate of interest (in %)? (c) 15 (d) 19

Set-13: Solutions
1. (c); So, 5. (a); Let number of boys and girls be 2x and 3x
On factorizing the given number i.e. 1287 respectively.
1287 = 3 × 3 × 11 × 13 = 9 × 11 × 13 Required average weight
2𝑥×18+3𝑥×21 36𝑥+63𝑥 99
So, highest number is 13. = = =
5𝑥 5𝑥 5
2. (c); 45m = 60 B ⇒ 3M = 4B
6. (d); Total C.P. = 50 × 40 = 2000 Rs.
(15m + 20B) x = 45m × 20
Total S.P. = (50 + 5) × 42
(15m + 15m)x = 45m × 20
= 55 × 42 = 2310
30m × x = 45m × 20 ⇒ x = 30 days 310
3. (c); MP = 1280 Profit percentage = × 100 = 15.5%
2000
𝐷𝑖𝑠𝑐𝑜𝑢𝑛𝑡 𝑝𝑒𝑟𝑐𝑒𝑛𝑡 =
1280 – 900
× 100 7. (d); Let B = x
1280 Then A will be = x + 6x = 7x
380
= × 100 = 29.6875% 7𝑥 –𝑥
1280 𝑅𝑒𝑞𝑢𝑖𝑟𝑒𝑑 𝑝𝑒𝑟𝑐𝑒𝑛𝑡𝑎𝑔𝑒 = × 100
4. (a); Let number of cans of Orange, pineapple and 7𝑥
6
mixed fruit be 800x, 900x and 1500x = × 100 = 85.71%
7
respectively Remaining cans of Orange 8. (c); Upto 8:30 AM first runner would have run for
75
= × 800𝑥 = 600x = 2.5 × 8 = 20 km
100
2
Remaining cans of Pineapple = × 900𝑥 = 600x Time taken by second runner to overtake first
3
20
Remaining cans of mixed Fruit Juices runner = = 10 hour
80 10 –8
= × 1500𝑥 = 1200x So, second runner will overtake first runner
100
Required ratio = 1 : 1 : 2 after 8 : 30 AM+ 10 hours = 6 : 30 PM
@BEST300MCQ For More Study Material
423
Visit: studyiq.com
Quantitative Aptitude

12 3 28 3 𝑆𝑃 7
9. (d); 7727.104 = 𝑃 (1 + ) = 𝑃 ( ) = 𝑅𝑠. 5500 22. (c); 70% of CP = 40% of SP =
100 25 𝐶𝑃 4
10. (a); (A + B) 7.5 = A × 10 ∴ Let CP be 4x and SP be 7x
𝐴 3 7𝑥–4𝑥
7.5 B = 2.5A ⇒ = ∴ 𝑃𝑟𝑜𝑓𝑖𝑡 𝑝𝑒𝑟𝑐𝑒𝑛𝑡 = × 100
𝐵 1 4𝑥
3𝑥
Total work = (3 + 1) × 7.5 or 3 × 10 = 30 units = × 100 = 75%
4𝑥
30 𝑎 𝑏 2𝑎𝑏
B alone can complete the work = = 30 days 23. (a); ×𝑏+ ×𝑎 =
1 100 100 100
11. (a); 2(l + b) = 48 = 2a% of b
2(l + 10) = 48 ⇒ l = 14 cm 24. (d); Let the usual speed be 6x km/hr
Area = l × b = 14 × 10 = 140 cm2 Therefore, speed at which he reaches 15 min.
12. (b); SP = Rs 1000 ⇒ D = 20% early is 7x km/hr
100
MP = 1000 × = Rs. 1250 ∴ ATQ,
80
𝑑 𝑑 15 𝑑 1 1
13. (a); A = 2B = 4C – = ⇒ [ ]=
1 1 6𝑥 7𝑥 60 𝑥 42 4
⇒A:B:C ⇒1: : ⇒4:2:1 𝑑
=
21
2 4
4× 1250 + 2000 𝑥 2
14. (c); New Average = = 1400 ∴ 𝑑𝑖𝑠𝑡𝑎𝑛𝑐𝑒 (𝑑) =
21
𝑥 𝑘𝑚
(4 +1)
2
15. (b); SP = Rs. 24 ⇒ L = 20% 21
𝑥
∴ CP = 24 ×
100
= 𝑅𝑠. 30 ∴ 𝑢𝑠𝑢𝑎𝑙 𝑡𝑖𝑚𝑒 = 2
6𝑥
80 7
New SP = Rs. 36 = ℎ𝑜𝑢𝑟𝑠 = 105 𝑚𝑖𝑛𝑢𝑡𝑒𝑠.
4
36 –30
∴ Profit = × 100 = 20% 25. (a); ATQ,
30 10000×3×𝑅 6000×4×𝑅
25 + = 5400
16. (d); D = 40 × = 16.67 𝑘𝑚 100 100
60
16.67 [where R is rate of interest]
Required speed = × 60 𝑘𝑚/ℎ𝑟 = 50 km/hr. 30R + 24R = 540
20
∴ Required increase in speed = 50 – 40 ∴𝑅=
540
= 10%
= 10 km/hr. 54
26. (c); Let number of boys = x
17. (c); Amount after 3 years = Rs. 1240 36×𝑥–42+30
Rate of interest = 9% 35.7 = ⇒ 35.7x = 36x –12
𝑥
∴ Amount after 4th year Or, 0.3x = 12 ⇒ Or, x = 40
1240×9×1
= 1240 + = Rs. 1351.6 27. (d); Let CP = 100
100
So, SP = 115 ⇒ New S.P. = 345
18. (d); Ratio of total work of women, men and children
= 10×6 : 6×5 : 8×10 ⇒ = 60 : 30 : 80 Now, Profit percent = 245%
1 1 1 28. (b); Let total votes polled = x
∴ Ratio of efficiency = ∶ ∶ 70% of x – 30% of x = 15400
60 30 80
=4:8:3 40% of x= 15400
19. (a); Let marked price be Rs. 100. 𝑥=
15400
× 100 ⇒ x = 38500
Therefore, cost price = Rs.75 40
175 Total votes losing candidate got
𝑆𝑒𝑙𝑙𝑖𝑛𝑔 𝑝𝑟𝑖𝑐𝑒 = 75 × = 𝑅𝑠. 131.25 = 38500 × 0.3 = 11550
100
131.25 –100
∴ Required profit percentage = × 100 29. (c); Let speed of boat in still water = x km/hr
100
= 31.25 % And speed of water current = y km/hr
4 4
20. (c); Amount received by A = 8400 ×
6
= 2400 + = 1 And,
𝑥 –𝑦 𝑥+𝑦
21 5 3 55
8 + =
Amount received by B = 8400 × = 3200 𝑥+𝑦 𝑥 –𝑦 60
21
7 1
Amount received by A = 8400 × = 2800 𝐿𝑒𝑡 =𝑉
21 𝑥−𝑦
400 1
𝑆𝑎𝑣𝑖𝑛𝑔 𝑜𝑓 𝐴 = × 3 = 600 𝐴𝑛𝑑 =𝑈
2 𝑥+𝑦
Saving of B = 400 4V + 4U = 1 …(i)
400 11
𝑆𝑎𝑣𝑖𝑛𝑔 𝑜𝑓 𝐶 = × 4 = 800 5𝑈 + 3𝑉 = … (𝑖𝑖)
2 12
Therefore ratio of expenditure Solving (i) and (ii)
= (2400 – 600) : (3200 – 400) : (2800 – 800) x = 9 km/hr and y = 3 km/hr
= 1800 : 2800 : 2000 ⇒ = 9 : 14 : 10 30. (b); SI = 130% of P
𝑃×10×𝑅
21. (c); Total age of 24 student = 24 × 12 = 288 130% 𝑜𝑓 𝑃 =
100
Correct total = 288 – 6 = 282 𝑅
282 1.3 = ⇒ R = 13%
∴ 𝐶𝑜𝑟𝑟𝑒𝑐𝑡 𝑎𝑣𝑒𝑟𝑎𝑔𝑒 = = 11.75 10
24

@BEST300MCQ For More Study Material


424
Visit: studyiq.com
Quantitative Aptitude

1. A can do 75% of a job in 18 days and B can do 25% 10. A is 1.5 times efficient than B therefore takes 8 days
of the job in 12 days. If they work on it together, in less than B to complete a work. If A and B work on
how many days can they do 75% of the job? alternate days and A works on first day, then in how
(a) 16 (b) 8 many days the work will be completed?
(c) 20 (d) 12 (a) 17 (b) 19
2. At 30% discount the selling price of an article is Rs (c) 19.5 (d) 21
1050. What is the selling price (in Rs) if the discount 11. A shopkeeper sells a table at a discount of 20% and
is 15%? earns a profit of 60%. If he sells the same table at
(a) 1200 (b) 1175 40% discount, then what will be his new profit
(c) 1100 (d) 1275 percent?
3. What is the average of all numbers between 9 and (a) 20 (b) 30
90 which are divisible by 8? (c) 35 (d) 40
(a) 53 (b) 52 12. 5 years ago the average age of a family which
(c) 51 (d) 50 includes father, mother and a son was 35 years. 3
4. A trader had 630 kgs of rice. He sold a part of it at years ago the average age of father and mother was
15% profit and the rest at 8% profit, so that he made 46 years. What is the present age (in years) of the
a total profit of 12%. How much rice (in kgs) did he son?
sell at 8% profit? (a) 20 (b) 24
(a) 270 (b) 300 (c) 26 (d) 22
(c) 280 (d) 290 13. The cost price of 60 articles is same as the selling
5. Excluding stoppages, the average speed of a train is price of x articles. If there is a profit of 20%, then
120 kmph and including stoppages, it is 50 kmph. what is the value of x?
For how many minutes does the train stop per hour? (a) 15 (b) 30
(a) 25 (b) 40 (c) 50 (d) 80
14. A person scores 45% of the total marks in the exam
(c) 35 (d) 20
and still fails by 40 marks. The passing percentage of
6. The simple and compound interest that can be
the exam is 55%. What is the maximum marks of the
earned in two years at the same rate is Rs 4000 and
exam?
Rs 4180 respectively. What is the rate (percent per
(a) 300 (b) 350
annum) of interest?
(c) 400 (d) 500
(a) 18 (b) 4.5
15. A man starts running from point P at 11:00 a.m. with
(c) 9 (d) 12
a speed of 10 km/hr. He runs for 2 hours and then
7. 6 times of a fraction is greater than 7 times of its
takes a 1 hour rest. He continues the pattern till he is
reciprocal by 11. What is the fraction?
caught by another man who starts at 2:00 p.m. from
(a) 5/3 (b) 7/3
point P and runs non-stop at a speed of 15 km/hr
(c) 5/4 (d) 4/5 towards the first man. At what time (in p.m.) will the
8. A watch having marked price as Rs 8400 is sold for first man be caught?
Rs 5040 after two successive discounts. If the first (a) 6:20 (b) 4:40
discount is 25% then what is the numerical value of (c) 6:00 (d) 5:30
second discount percentage? 16. The difference of compound interest and simple
(a) 15 (b) 20 interest for 3 years and for 2 years are in ratio 23 : 7
(c) 25 (d) 30 respectively. What is rate of interest per annum
9. Average age of 20 students is 21 years. 2 students (in %)?
leave the group and 1 new student joins the group. (a) 200/7 (b) 100/7
The average now becomes 20 years. If age of one of (c) 300/7 (d) 400/7
the student who left the group is 26 years and the 17. A fraction becomes 6/5 when 5 is added to its
one who joined is 20 years, then what is the age (in numerator and becomes 1/2 when 4 is added to its
years) of the other student who left the group? denominator. What will be the value of the fraction?
(a) 24 (b) 34 (a) 8/9 (b) 7/10
(c) 22 (d) 36 (c) 7/8 (d) 6/11
@BEST300MCQ For More Study Material
425
Visit: studyiq.com
Quantitative Aptitude

18. Amit can complete a work in 25 days and Punit can 25. A bus travels 2/5 of a total journey at its usual
complete the same work in 20 days. Punit alone speed. The remaining distance was covered by bus
worked at it for 10 days and then left the work. In at 6/7 of its usual speed. Due to slow speed it
how many days will Amit alone complete the reaches its destination 50 minutes late. If the total
remaining work? distance is 200 kms, then what is the usual speed (in
1 1
(a) 11 (b) 12 km/hr) of bus?
2 2
(c) 13
1
(d) 14
1 (a) 20.57 (b) 24
2 2
(c) 28 (d) 26.52
19. The measure of the four successive angles of a
quadrilateral are in the ratio 7 : 11 : 7 : 11. The 26. For an amount, simple interest at the rate of interest
quadrilateral is a ______. of 12% per annum for 6 years is Rs 25920. What will
(a) trapezium (b) rectangle be the compound interest (in Rs) on same amount at
(c) parallelogram (d) square the rate of interest of 8% per annum compounding
20. What is the discount percentage offered on a book annually for 2 years?
having marked price Rs 2150 being sold at Rs 1892? (a) 4326.3 (b) 5563.4
(a) 12 % (b) 13 % (c) 5888.6 (d) 5990.4
(c) 14 % (d) 16 % 27. Profit of Rs 12,400 has to be divided between three
21. Rs 60500 is divided among A, B and C such that A partners A, B and C in the ratio 5:7:8. How much
receives 2/9 as much as B and C together and B does B get (in Rs)?
receives 3/7 as much as A and C together. What is (a) 4340 (b) 3440
the share of C (in Rs)?
(c) 3340 (d) 4430
(a) 29850 (b) 30120
28. The average weight of P, Q and R is 47 kg. If the
(c) 31350 (d) 37250
22. The average age of a class of 6 girls is x years. Four average weight of P and Q be 32.5 kg and that of Q
new girls having ages x – 2, x + 2, x + 4 and x + 6 and R be 48.5 kg, then what is the weight of Q (in
joins the class. What is the new average age (in kgs)?
years) of the class? (a) 25 (b) 21
(a) x + 1 (b) x + 2 (c) 29 (d) 33
(c) 2.5x (d) x + 2.5 29. A shopkeeper by selling 5 items, earns a profit equal
23. A dealer sells two machines at Rs 12000 each. On to the selling price of 1 item. What is his profit
one it gains 32% and on the other it looses 32%. percentage?
What is its profit/loss percentage in the whole (a) 20% (b) 25%
transaction? (c) 16% (d) 22.5%
(a) No gain and no loss (b) 1% loss 30. If in 3 years at simple interest the principal
(c) 18% profit (d) 10.24% loss
increases by 18%, what will be the compound
24. How much water (in litres) must be added to 80
litres solution of milk and water containing 10% interest (in Rs) earned on Rs. 25,000 in 3 years at
milk, so that it becomes a 5% milk solution? the same rate?
(a) 10 (b) 20 (a) 4775.4 (b) 5774.4
(c) 40 (d) 80 (c) 4557.4 (d) 5575.4

Set-14: Solutions
1. (d); A can do 75% in 18 days 70 units → 1050
100% 1050
∴ A can do 100% in 18 × = 24 days 100 𝑢𝑛𝑖𝑡𝑠 → × 100 = 1500
75% 70
1500
⇒ B can do 25% in 12 days 85 𝑢𝑛𝑖𝑡𝑠 → × 85 = Rs 1275
100 100
∴ B can do 100% in 12 × = 48 days 3. (b); The numbers are 16, 24, ………..88
25
10
Sum of the numbers = × (16 + 88) = 520
2
520
∴ Average of the numbers = = 52
3 1 10
∴ 48 × × = 12 days 4. (a); Let cost of 1 kg rice be Rs 1
4 (2+1)
2. (d); And amount of rice sold at 8% profit be x
∴ ATQ,
108 115 112
𝑥× + (630 − 𝑥) × = 630 ×
100 100 100
⇒ x = 270

@BEST300MCQ For More Study Material


426
Visit: studyiq.com
Quantitative Aptitude
120
∴ After 20% profit 𝑥 × = 60
100
5. (c); Due to stoppages, the train covers (120 – 50)
⇒ x = 50
= 70 km less (in 1 hour)
70
14. (c); Let the maximum marks be x
∴ time taken = ( × 60) min= 35 min ∴ 0.45x + 40 = 0.55x
120
6. (c); ⇒ x = 400 marks
15. (b);

At 2:00 pm, man 2 starts at P with a speed 15


km/hr and distance between the two being
20 km
180 ∴ For 2 hours, this distance will be covered by
𝑅𝑎𝑡𝑒 = × 100 = 9% man 2 with a relative speed of 5 km/hr
2000
7. (b); Let the fraction be x ∴ At 4:00 pm, distance remaining will be 10 km
7
∴ 6𝑥 − = 11 Now, as man 1 rests.
𝑥
⇒ 6x² - 7 = 11x man 2 will cover this distance in
10
⇒ 6x² - 11x – 7 = 0 × 60 = 40 minutes
15
𝑥=
11±√121+4×7×6 ∴ Time will be 4:40 pm
12 16. (a);
7
⇒𝑥=
3
75 𝑥
8. (b); 8400 × × = 5040
100 100
5040×100
⇒𝑥= × 100
75×8400
⇒ 𝑥 = 80
∴ 20% discount
9. (b); Let age of other student who left the group be x
∴ 21 × 20 + 20 – 26 – x
= 20 × 19
⇒ x = 34 years
10. (b); Let A complete the work in x days.
∴ B complete the work in (x + 8) days 3A + B : A ⇒ 23 : 7
As A is 1.5 times more different than B ∴ A = 7 and B = 2
𝐴 3 2 200
∴ Ratio of the work efficiency = ∴ Rate of interest = × 100% =
𝐵 2 7 7
Now, finding out the total work 𝑥
17. (b); Let fraction be
𝑦
3(x) = 2 (x + 8) ⇒ x = 16 𝑥+5 6
Hence, total work = 16 × 3 = 48 =
𝑦 5
Cycle is (3 + 2) = 5 w/2days 5x + 25 = 6y
Work will be completed on 19th day. 6y – 5x = 25 … (i)
11. (a); Let MP of table = Rs. 100 and
𝑥 1
= ⇒ 2𝑥 = 𝑦 + 4 … (𝑖𝑖)
𝑦+4 2
Solving (i) and (ii)
x = 7 and y = 10
7
so, fraction is
10
CP =
80
× 100 = 50 18. (b); Let Amit alone can complete the remaining
160
10 work in x days.
∴ Profit = × 100 = 20%
50 So,
12. (d); Let the age of son 3 years ago be x 𝑥 10 𝑥 1
∴ According to the question, + =1 ⇒ = ⇒ x = 12.5 days
25 20 25 2
35 × 3 + 2 × 3 = 46 × 2 + x 19. (c); Pair of opposite angles are equal in a
⇒ x = 19 parallelogram. Rectangle and square are also
∴ Present age = 19 + 3 = 22 parallelogram but they have 90° angle at each
13. (c); According to the question,
corner.
60 × CP = x × SP 2150 –1892
⇒ =
𝐶𝑃 𝑥 20. (a); 𝑃𝑒𝑟𝑐𝑒𝑛𝑡𝑎𝑔𝑒 𝑑𝑖𝑠𝑐𝑜𝑢𝑛𝑡 = × 100 = 12%
2150
𝑆𝑃 60

@BEST300MCQ For More Study Material


427
Visit: studyiq.com
Quantitative Aptitude
2 24
21. (c); 𝐴 = (𝐵 + 𝐶) 𝑥=
9 7
9 24
𝐵+𝐶 = 𝐴 𝑈𝑠𝑢𝑎𝑙 𝑠𝑝𝑒𝑒𝑑 = × 7 𝑘𝑚 = 24 𝑘𝑚⁄ℎ𝑟
2 7
9
𝑆𝑜, 𝐴 + 𝐴 = 60500 26. (d); 12% × 6 = 72%
2
72% → 25920
11A = 60500 × 2 ⇒ A = Rs. 11000 25920
And 1% =
72
3 7𝐵 25920
𝐵 = (𝐴 + 𝐶) ⇒ 𝐴 + 𝐶 = 100% = × 100 = 36000
7 3 72
7 2
𝐵 + 𝐵 = 60500 ⇒ B = Rs. 18150 𝐶. 𝐼. = 𝑃 [(1 +
𝑅
) – 1]
3 100
C = 60500 – 11000 – 18150 = Rs. 31350 8 2
22. (a); Total age of 6 girls = 6x = 𝑃 [(1 + ) – 1]
100
6𝑥+4𝑥+10
𝑁𝑒𝑤 𝑎𝑣𝑒𝑟𝑎𝑔𝑒 𝑎𝑔𝑒 = =𝑥+1 27 2
10 = 36000 [( ) – 1]
23. (d); When selling price of two articles remain same 25
729–625
and first article is sold at x% loss and second at = 36000 [ ]
625
𝑥2 104
x% profit then there is always a loss of % = 36000 ×
100 625
(32)2
So, there would be % 𝑙𝑜𝑠𝑠 = 𝑅𝑠. 5990.4
100 7
= 10.24% loss 27. (a); B gets profit = 12,400 × = 4340
20
24. (d); Let water added = x litre 28. (b); Required value of Q
90
×80+𝑥 95 = (47 × 3) – [47 × 3 − 65] + [47 × 3– 97]
100
= = 141 − [76 + 44]
80+𝑥 100
72+𝑥 19
= = 141 − 120 = 21
80+𝑥 20
1440 + 20x = 1520 + 19x 29. (b); Let the SP of 1 item be Rs 1
x = 80 litre SP of 5 items = Rs 5
25. (b); Let slow speed = 6x Therefore, Profit earn = Rs 1 (SP of 1 item)
And normal speed = 7x CP of 5 items = Rs (5 - 1) = Rs 4
1
Time difference will occur only in remaining ∴Profit % = × 100 = 25%
4
3/5 of distance 30. (a); Let the sum invested at SI be ‘P’.
Let total distance = D ∴ 0.18P =
𝑃 ×𝑅 ×3
3 3 100
𝐷 𝐷 50
5
–5 = R=6%
6𝑥 7𝑥 60
120 120 5 Now, CI earned on Rs 25000 for 3 years at 6% is
– = 3
6𝑥 7𝑥 6 6
120(7 –6) 5 CI = 25000{( 1 + ) − 1} = Rs 4775.4
= 100
42𝑥 6

@BEST300MCQ For More Study Material


428
Visit: studyiq.com
Quantitative Aptitude

1 8 men, 5 women and 6 boys can do a piece of work 8. An alloy of copper and aluminium has 40% copper.
in 4 days .While 2 women and 12 boys or 4 men and An alloy of copper and zinc has copper and zinc in
3 women can do the same work each in 10 days the ratio 2 : 7. These two alloys are mixed in such
.Find the number of days required to 3 men ,4 way that in the overall alloy, there is more
women and 3 boys to the same work. aluminum than zinc, and copper constitute X% of
(a) 8 days (b) 10 days this alloy, what is the range of values 𝑋 can take.
(c) 15 days (d) 12 days (a) 20% ≤ X ≤ 40% (b) 30% ≤ X ≤ 40%
2. A man sold a TV at 25% profit on its cost price. If he
(c) 32.5 %< X ≤ 40% (d) 33.33% ≤ X ≤ 40%
sold it at 25% profit on this selling price, then he
9. A train after 3 hours from starting meet with an
gains 5% + Rs 100 more on its cost price. Find the
accident which detains hour. After this the train
cost price of TV.
proceeds at 75% of its former speed and arrives 4
(a) 6000 (b) 7000
(c) 8000 (d) 9000 hours late. Had the accident happened 150 km
1
3. Two equal sums of money were invested, one at 4% farther along the line, it would have arrived only 3
2
1
and the other at 4 % At the end of 7 years, the hours late. Find speed of the train and total distance
2
simple interest received from the latter exceeded of journey?
that received from the former by Rs. 31.50. Each (a) 100 km/hr, 1500 km (b) 200 km/hr, 1200 km
sum was : (c) 100 km/hr, 1200 km (d) 150 km/hr, 1200 km
(a) Rs. 1,000 (b) Rs. 500 10. Ratio of the fares of First, Second and Third class
(c) Rs. 750 (d) Rs. 900 category of a train between two stations is 10 : 7 : 2
4. If numerator of a fraction is increased by 1 and and the ratio of passenger travelling in these
denominator is decreased by 1, then it becomes category is 4 : 9 : 17 respectively. If the fare is
unity. Again, if numerator increased by 3 and increased by ¼ in First class, 1/8 in Second class
denominator is reduced by 2 then it becomes 2. and decreased by 10% in Third class so that the
What is original fraction?
5 7
ratio of number of passenger in the category
(a) (b) remains same. If the new collection is received of
7 9
4 3
(c) (d) Rs. 60590, then find the total amount received from
7 5
5. The batting average for 30 innings of a cricket third class category.
player is 40 runs. His highest score exceeds his (a) 13280 (b) 15400
lowest score by 100 runs. If these two innings are (c) 12240 (d) 12000
not included, the average of the remaining 28 11. 10 years ago the average age of a family of 4
innings is 38 runs. The lowest score of the player is : members was 24 yrs. Three children have been
(a) 15 (b) 18 born and the average age of the family is 22 yrs.
(c) 20 (d) 12 today. What are the present ages of children, if two
6. A boat goes 24 km upstream and 28 km downstream children are identical twins and differ by three
in 6 hours. It goes 30 km upstream and 21 km years from the younger one.
downstream in 6 hours and 30 minutes. The speed (a) 7, 7, 4 (b) 8, 8, 11
of the boat in still water is (c) 8, 8, 5 (d) 5, 5, 2
(a) 8 km/hr (b) 9 km/hr
12. A bank pays 16.5% per annum as interest on its
(c) 12 km/hr (d) 10 km/hr
deposits while it charges 20% per annum as
7. A student finds the average of 10, 2 – digit numbers.
interest from its borrowers. If at the end of the
If the digits of one of the numbers is interchanged,
the average increases by 3.6. The difference between years the bank earns Rs. 10.5 crore. Find how much
the digits of the 2-digit number is money was transacted during the year?
(a) 4 (b) 3 (a) 300 crore (b) 320 crore
(c) 2 (d) 5 (c) 412 crore (d) None of these

@BEST300MCQ For More Study Material


429
Visit: studyiq.com
Quantitative Aptitude

13. At his usual rowing rate, Ram can travel 12 miles 20. Two places A and B are 100 km apart on a highway.
downstream in a certain river taking 6 hour less One car starts from A and another from B at the
than that taken by him to travel the same distance same time. If the cars travel in the same direction at
upstream. But if he could double his usual rowing a constant speed they will meet in 5 hours. If they
rate for his 24 mile round trip, the downstream 12 travel towards each other, they will meet in 1 hr.
miles would then take only one hour less than the What is the speed of the car running faster?
upstream 12 miles. What is the speed of the current (a) 60 km/h (b) 50 km/h
in miles per hour? (c) 40 km/h (d) 32 km/h
1 21. A, B and C can do a work in 8, 16 and 24 days
(a) 1.5 mph (b) 2 mph
3 respectively. They all begin together, A continues to
1 2
(c) 1 mph (d) 2 mph work till it is finished, C left before 2 days and B,
3 3
14. A man borrows Rs. 1000 and agrees to repay it one day before its completion. In what time is the
with a total interest of Rs. 140 in 12 installments, work finished?
each installment being less than the one preceding (a) 7 days (b) 5 days
(c) 6 days (d) Can’t be determined
it by Rs. 10. What should be his first installment?
22. Anu can complete a work in 10 days. Manu is 25%
(a) 160 (b) 80
more efficient than Anu and Sonu is 60% more
(c) 150 (d) 40
efficient than Manu. Working together, how long
15. A train’s journey is disrupted due to an accident on would they take to finish the job?
its track after it has travelled 30 km. Its speed then 6 6
(a) 2 days (b) 5 days
comes down to 4/5 th of its original and 17 7
5 5
consequently it runs 45 min late. Had the accident (c) 3 days (d) 4 days
8 8
taken place 18 km farther away, it would have been 23. A person who has a certain amount with him goes
36 min late. Find the original speed of the train. to market. He can buy 40 oranges or 50 mangoes.
(a) 25 km/hr (b) 36 km/hr He retains 10% of the amount for taxi fares and
(c) 30 km/hr (d) 20 km/hr buys 20 mangoes and of the balance, he purchases
16. Population of a district is 2,96,000 of which oranges. Number of oranges he can purchase is
1,66,000 are males. 50% of the population is (a) 36 (b) 40
literate. If 70% males are literate, then the number (c) 15 (d) 20
of women who are literate is? 24. The average speed of a train is 20% less on the
(a) 32,900 (b) 31,800 return journey than on the onward journey. The
(c) 35,295 (d) 30,945 train halts for half an hour at the destination
17. Weights of two friend Ram and Shyam are in the station before starting on the return journey. If the
ratio of 4 : 5. Ram’s weight increases by 10% and total time taken for the to and fro journey is 23 hr,
covering a distance of 1000 km, the speed of the
the total weight of Ram and Shyam together
train on the return journey is
becomes 82.8 kg., an increase of 15%. By what
(a) 60 km/hr (b) 40 km/hr
percent did the weight of Shyam increase?
(c) 50 km/hr (d) 55 km/hr
(a) 19 (b) 20 25. X and Y entered into partnership with Rs. 700 and
(c) 15 (d) 17 Rs. 600 respectively. After 3 months, X withdrew
18. Mr. Maitray invested money in two schemes A and 2/7 of his stock but after 3 months, he puts back
B offering C.I. at the rate of 8 p.c.p.a. & 9 p.c.p.a. 3/5 of what he had withdrawn. The profit at the
respectively. If the total amount of interest end of the year is Rs. 726. How much of this should
incurred through the two schemes together in two X receive?
years was Rs. 4818.30 and the total amount (a) Rs. 336 (b) Rs. 366
invested was Rs. 27,000. What was the amount (c) Rs. 633 (d) Rs. 663
invested in scheme A? 26. A man who can swim 48 m/min in still water,
(a) 12000 (b) 15000 swims 200 m against the current and 200 m with
(c) 24000 (d) 20000 the current. If the difference between those two
times is 10 min, find the speed of the current.
19. Train ‘A’ leaves Mumbai Central for Lucknow at 11
(a) 30 m/min (b) 29 m/min
am running at the speed of 60km/h. Train ‘B’ leaves
(c) 31 m/min (d) 32 m/min
Mumbai Central for Lucknow by the same route at
27. A man arranges to pay off a debt of Rs. 3600 by 40
2 pm on the same day, running at the speed of 72
annual installments which are in AP When 30 of the
km/h. At what time will the two trains meet each
installments are paid he dies leaving one-third of
other? the debt unpaid. The value of the 8th instalment is
(a) 11 : 30 am Sunday (b) 11 pm Sunday (a) Rs. 35 (b) Rs. 50
(c) 2 am Tuesday (d) 5 am on Next day (c) Rs. 65 (d) None of these
@BEST300MCQ For More Study Material
430
Visit: studyiq.com
Quantitative Aptitude

28. A club consists of members whose ages are in AP, cards, I will have thrice as many cards as you have.”
the common difference being 3 months. If the How many cards did Pintu have?
youngest member of the club is just 7 years old and (a) 31 (b) 32
the sum of the ages of all the members is 250 yrs, (c) 29 (d) 30
then the number of members in the club are 30. Two pipes A and B can fill a tank in 20 hrs. and 30
hrs. respectively. Both the pipes are opened to fill
(a) 15 (b) 20
the tank but when the tank is one-third full, a leak
(c) 25 (d) 30
develops in the tank through which one-fourth
29. Pintu gave some cards to Mintu and himself from a
water supplied by both pipes goes out. What is the
full pack of playing cards and laid the rest aside. total time taken to fill the tank?
Pintu then said to Mintu, “If you give me a certain 2
(a) 14 ℎ𝑟𝑠. (b) 15 hrs.
number of your cards, I will have 4 times as many 3
1 1
cards as you have. If I give you the same number of (c) 12 ℎ𝑟𝑠. (d) 9 hrs.
2 2

Set-15: Solutions
1. (b); Let efficiencies of man, woman and boy is M, W 5. (b); Let average of two innings is x use allegation →
and B respectively.
(8M + 5W + 6B)4 = (2W + 12B)10
= (4M + 3W)10
⇒ 32M + 20W + 24B = 20W + 120B
𝑀 3
⇒ M = 3B ⇒ =
𝐵 1
Now consider efficiency of 1 man is 3 and 1 boy
is 1
Now again, 20W + 120B = 40M+ 30W
10W = 40 × 3 – 120 × 1
W = 0 (Women does not work) ⇒ x – 40 = 28 ⇒ x = 68
Total work = (8 × 3 + 5 × 0 + 6 × 1) × 4 = 120 Highest score + lowest score = 136
Efficiency of 3 men, 4 women and 3 boys = 3 × 3 Highest score – lowest score = 100
+ 4 × 0 + 3 × 1 = 12 ⇒ Highest score = 118
120 ⇒ Lowest score =18
𝑅𝑒𝑞𝑢𝑖𝑟𝑒𝑑 𝑡𝑖𝑚𝑒 = = 10 days
12 6. (d); Let speed of boat in still water is x kmph and
2. (c); Let CP = x speed of stream is y kmph.
5
SP = 𝑥 ATQ,
4
24 28
Now ATQ, + =6 … (𝑖)
1 5 𝑥 𝑥 𝑥−𝑦 𝑥+𝑦
× 𝑥 − = + 100 30 21
4 4 4 20
5 𝑥 𝑥 + = 6.5 . . . (𝑖𝑖)
𝑥− = + 100 𝑥−𝑦 𝑥+𝑦
16 4 20
𝑥 𝑥 4𝑥 Solve (i) and (ii) and we get
⇒ − = 100 = = 100 ⇒ 𝑥 = 8000
16 20 320 x= 10 kmph ⇒ y = 4 kmph
3. (d); If sum of money = P 7. (a); Let the two digit number is (10x +y)
𝑃×4.5×7 𝑃×4×7 𝑃×3.5
− = 31.50 ⇒ = 31.50 ATQ,
100 100 100
3150 (10x + y) – (10y + x) = 3.6 ×10
P= ⇒ P = Rs. 900
3.5
𝑎 9(x -y) = 36 ⇒ x –y = 4
4. (d); 𝐿𝑒𝑡 𝑂𝑟𝑖𝑔𝑖𝑛𝑎𝑙 𝑓𝑟𝑎𝑐𝑡𝑖𝑜𝑛 = 8. (c); In alloy ‘1’ ratio of copper and aluminium = 2 : 3
𝑏
𝑎+1
𝐴𝑇𝑄, =1 In alloy ‘2’ ratio of copper and zinc = 2 : 7
𝑏−1
Condition is aluminium is more than zinc ‘X’ will
𝑎+1=𝑏−1
be minimum when aluminium and zinc will be
𝑎 − 𝑏 = −2 … (𝑖)
𝑎+3 in proportion.
=2 To make aluminium and zinc equal multiply by
𝑏−2
𝑎 + 3 = 2𝑏 − 4 ‘7’ in first alloy’s ratio and by 3 in seconds
𝑎 − 2𝑏 = −7 … (𝑖𝑖) alloy’s ration
From (i) and (ii) we can evaluate So mixture will be in ratio of
𝑎 = 3, 𝑏 = 5 Copper : Aluminium : Zinc = (14 + 6) : 21 : 21
3
𝑅𝑒𝑞𝑢𝑖𝑟𝑒𝑑 𝑓𝑟𝑎𝑐𝑡𝑖𝑜𝑛 = = 20 : 21 : 21
5

@BEST300MCQ For More Study Material


431
Visit: studyiq.com
Quantitative Aptitude
20
% of copper = × 100 = 32.25% = 154 – 136 = 18 yrs.
62
Let the age of the one of the twins = x yrs.
% of aluminium will be maximum when zinc
∴ age of the youngest = (x – 3) yrs
will be zero.
Then:-
Maximum % of copper = 40%
2x + (x – 3) = 18
32.25% < × ≤ 40%
Or, 3x = 21
9. (c);
∴x=7
∴ Age of children = 7, 7, 4 yrs.
12. (a); Let the bank makes a transaction of Rs. x crores.
According to question:-
`
(20 – 16.5)% of x = 10.5 crore
In 1st case 3.5
Ratio of original speed and reduced speed = 100 × 𝑥 = 10.5
100
10.5×100
: 75 = 4 : 3 ∴𝑥= = 300 crore
1 3.5
∵ 𝑇𝑖𝑚𝑒 = 13. (d); Let Ram’s rowing rate is ‘x’
𝑆𝑝𝑒𝑒𝑑
3
𝑆𝑜 𝑡𝑖𝑚𝑒 𝑟𝑎𝑡𝑖𝑜 = = 1𝑢𝑛𝑖𝑡 — 4 ℎ𝑜𝑢𝑟 Speed of current is ‘y’
4 12
3 unit = 3 × 4 = 12 hour Downstream time taken =
𝑥+𝑦
12
Means if train does not meet with accident if Upstream time taken =
𝑥−𝑦
take 12 hour to cover remaining distance.
According to the question
In IInd case 12 12
3 − = 6 ⇒ 𝑥 2 − 𝑦 2 = 4𝑦 … (i)
𝑆𝑝𝑒𝑒𝑑 𝑟𝑎𝑡𝑖𝑜 = 𝑥−𝑦 𝑥+𝑦
4
4 7 Now, If speed of boat doubles ⇒ ′2𝑥′
𝑆𝑜 𝑡𝑖𝑚𝑒 𝑟𝑎𝑡𝑖𝑜 = = 1 𝑢𝑛𝑖𝑡 – ℎ𝑜𝑢𝑟
3 2 Time is 1hr. less as compared to upstream
21 12 12
3 𝑢𝑛𝑖𝑡 = ℎ𝑜𝑢𝑟 − = 1 ⇒ 4𝑥 2 − 𝑦 2 = 24𝑦 … (ii)
2 2𝑥−𝑦 2𝑥+𝑦
21 3
𝐷𝑖𝑓𝑓𝑒𝑟𝑒𝑛𝑐𝑒 𝑖𝑛 𝑡𝑖𝑚𝑒 = 12– ℎ𝑟𝑠 = ℎ𝑟𝑠 From (i) & (ii) we get
2 2
8 2
Time taken by train to cover 150 km as its y = = 2 mph.
3 3
normal speed = 3/2 hour 14. (c); The total amount = Rs. [1000 + 140] = Rs. 1140
150×2
𝑆𝑝𝑒𝑒𝑑 𝑜𝑓 𝑡𝑟𝑎𝑖𝑛 = = 100 𝑘𝑚⁄ℎ𝑟 Let the 1st installment = Rs. x According to
3
Total time = 3 + 9 = 12 hour question,
12
Total distance = 12 × 100 = 1200 km. 1140 = [2𝑥 + (12 − 1)(−10)]
2
10. (c); Let fares of different class, 1140
= [2𝑥 − 110]
First class = 10𝑥 6
190+110
Second class = 7𝑥 190 = 2x – 110 ⇒ x = = 150
2
Third class = 2𝑥 Ist installment = Rs. 150
Then increased fares; 15. (c); Case I: When accident occurs at 30 kms.
1
First class = 10𝑥 + × 10𝑥 = 12.5𝑥 Usual time taken by train to cover remaining
4
1 63 distance = 45 × 4 = 180 min = 3 hr.
Second class = 7𝑥 + × 7𝑥 = 𝑥
8
10×2𝑥
8 Case II: When accident occurs at 48 kms.
third class = 2𝑥 − = 1.8𝑥 Usual time taken by train to cover remaining
100
Ratio of passengers = 4 : 9 : 17 distance = 36 × 4 = 144 min
Assume passengers in first class, second class In (180 – 144) = 36 min., train covers 18 kms.
18
and third class are 4𝑦, 9𝑦 and 17𝑦 respectively Hence speed of train = × 60 = 30 km/hr
63 36
Then total fare = 4𝑦 × 12.5𝑥 + 9𝑦 × 𝑥 + 16. (b); Population of literates = 50% of 296000
8
17𝑦 × 1.8𝑥 = 0.50 × 296000 = 148000
60590 = 50𝑥𝑦 + 70.875𝑥𝑦 + 30.6𝑥𝑦 No. of males literates = 70% of 166000
60590 = 151.475𝑥𝑦 ⇒ 𝑥𝑦 = 400 = 0.7 × 166000 = 116200
Amount received from third class No. of female literates = 148000 – 116200
= 17𝑦 × 1.8𝑥 = 12240 = 31800
17. (a); Let Ram and Shyam weights are
11. (a); Total age of the 4 members of the family,
= 4x and 5x respectively
10 yrs ago = 24 × 4 = 96 yrs. Now:-
Present age of 4 members = 96 + 40 = 136 yrs Their previous weight (sum):-
Total age of the 7 members presently 82.8
× 100 = 72 𝑘𝑔
= 22 × 7 = 154 yrs. 115
According to question:-
Age of [𝑡𝑤𝑖𝑛𝑠 + 𝑦𝑜𝑢𝑛𝑔𝑒𝑠𝑡 𝑐ℎ𝑖𝑙𝑑]

@BEST300MCQ For More Study Material


432
Visit: studyiq.com
Quantitative Aptitude
1
⇒ 5𝑥 + 4𝑥 = 72 22. (a); Anu’s 1 day work = part
10
∴x=8 125 1 1
∴ Ram’s weight = 8 × 4 = 32 kg Manu’s 1 day work = × = part
100 10 8
160 1 1
Shyam’s weight = 5 × 4 = 40 kg Sonu’s 1 day work = × = part
100 8 5
increased weight of ram = 32×1/10 =3.2 1 1 1 17
Total work = + + =
Their increased weight=10.8 10 8 5 40
40 6
increased weight of shyam Total days = = 2 days
17 17
= [10.8 − 3.2] = 7.6 kg 23. (d); Suppose the person has Rs 100 with him
7.6
∴ % increase = × 100 = 19% ∴ Price per orange is Rs 2.5 and that of a mango
40
18. (a); Let amount invested in scheme A = Rs. x is Rs 2
Then in B = Rs. (27000 – x) After keeping Rs 10 for taxi, he is left with Rs 90
From allegation method, Price of 20 mangoes = Rs 40
Complete amount after 2 years at 8% = 4492.8 Remaining money = (90 - 40) = Rs. 50
50
Complete amount after 2 years at 9% = 5078.7 So, he can buy= = 20 oranges for this amount
2.5
But given that , amount after 2 years = 4818.3 24. (b); Let the speed of train on onward journey be x
So, interest = (5078.7 – 4818.3)= 260.4 km/h.
and another interest amount=4818.3- 4492.8 Then, the speed of train on return journey
= 325.5 = 0.8 x km/h.
so, the ratio = 4/5 Total time =
500
+ +
1 500
⇒ 23 =
1125
+
1
Amount of A = 4*27000/9 =12000 𝑥
2
2 0.8𝑥 𝑥 2
19. (d); ⇒ 𝑥 = 1125 × = 50 𝑘𝑚/ℎ
45
∴ Speed of train on return journey = 40 km/h
5
25. (b); X’s investment = (700 × 3) + (700 × × 3) +
7
3
Given that:- (500 + 200 × ) × 6
5
Speed of A = 60 km/hr = 𝑅𝑠. 7320
Distance travelled in 3 hr = 60 × 3 = 180 km Y’s investment = 600 × 12 = 𝑅𝑠. 7200
At 2 : 00 pm:- ∴ X’s share from profit
Speed of B = 72 km/hr 7320
= × 726 = 𝑅𝑠. 366
Time difference = 3 hrs (7320+7200)
Relative velocity = [72 − 60] = 12 km/hr 26. (d); Let the speed of current be x m/min.
Now:- Time – gap (meeting) Then, speed with current = (48 + x) m/min
=
180
= 15 hr after they meet & Speed against the current = (48 – x) km/h
12 ATQ,
⇒ They will meet at 2 pm + 15 hour = 5 am 200 200
20. (a); ⇒ − (48+𝑥) = 10
(48−𝑥)
⇒ 40𝑥 = (48)2 − 𝑥 2
⇒ 𝑥 2 + 40𝑥 − 2304 = 0
Let the speed of A = 𝑢 km/hr ⇒ (𝑥 + 72)(𝑥 − 32) = 0
Speed of B = 𝑣 km/hr ⇒ 𝑥 = 32 𝑚/𝑚𝑖𝑛
As per question:- 27. (c); Let the first installment be ‘a’ and the common
100 difference between any two consecutive
= 1 hr
(𝑢+𝑣) instalments be ‘d’.
(𝑣 + 𝑢) = 100 ... (i) Using the formula for the sum of an AP.
Again from question:- 𝑛
𝑆𝑛 = [2𝑎 + (𝑛 − 1)𝑑]
100 2
= 5, We have,
(𝑢−𝑣)
40
5𝑢 − 5𝑣 = 100 ... (ii) 3600 = [2𝑎 + (40 − 1)𝑑] = 20(2𝑎 + 39𝑑)
2
From equation (i) and (ii):- ⇒ 180 = 2𝑎 + 39 𝑑 ... (i)
10𝑢 = 600 ⇒ 𝑢 = 60 𝑘𝑚/ℎ𝑟 30
Again, 2400 = [2𝑎 + (30 − 1)𝑑]
21. (b); LCM of A,B and C = 48 2
Then efficiency of A,B and C is 6,3,2 respectively. 160 = 2𝑎 + 29𝑑 ... (ii)
Then work of last two days is = 6+6+3 =15 Solving Eqs. (i) and (ii),
Total work = 48 20 = 10d ⇒ d = 2
Remaining work = 48-15 =33 Therefore, 180 = 2a + 39 × 2
So, that work done by A,B and C ⇒ 2a = 102 ⇒ a = 51
= 33/(6+3+2) = 3 Value of 8th instalment
So, total work completed in= 2+3 = 5 days = 51 + (8 – 1) × 2 = 51 + 14 = Rs. 65

@BEST300MCQ For More Study Material


433
Visit: studyiq.com
Quantitative Aptitude

28. (c); Let n be the number of members in the club. Solving Eqs. (i) and (ii), we get
𝑛 3
Then, 250 = [2 × 7 + (𝑛 − 1) ] M = 9n and P = 31n
2 12
𝑛 𝑛 1 Put n = 1, we get P = 31
⇒ 250 = [14 + − ]
2 4 4 30. (a); Together both pipes can fill the tank in
𝑛2 𝑛 20×30
⇒ 250 = 7𝑛 + − ( ) ℎ = 12ℎ
8 8 20+30
⇒ 𝑛 = 25 One third tank can be filled in 4 h.
29. (a); Let P and M denote no. of cards Pintu and Mintu Now, there is a leak which can empty the tank in
have respectively. (12 × 4) h = 48h
Case 1:- P + n = 4(M – n) So, two-third tank can be filled in
⇒ P – 4M = –5n … (i) 2
×(
12×48 2
) ℎ = 10 ℎ
3 48–12 3
Case 2:- (P – n) = 3 (M + n) 2 2
⇒ P – 3M = 4n ... (ii) So, total time to fill the tank = 4 + 10 = 14 h.
3 3

@BEST300MCQ For More Study Material


434
Visit: studyiq.com

You might also like